Go Math Grade 4 Answer Key Homework FL Chapter 11 Angles Review/Test

go-math-grade-4-chapter-11-angles-review-test-answer-key

Test your knowledge and math skills by using the Go Math Grade 4 Answer Key Homework FL Chapter 11 Angles Review/Test and understand the topics clearly. Explore how to measure the angles using various methods with Go Math Grade 4 Ch 11 Angles Solution Key & score well. Moreover, you will get a detailed explanation with diagrams on Go Math Grade 4 Answer Key Homework FL Chapter 11 Angles Review/Test.

Go Math Grade 4 Answer Key Homework FL Chapter 11 Angles Review/Test

Access the links provided below and directly download the Go Math Grade 4 Answer Key Homework FL Chapter 11 Angles Review/Test in pdf. By using these Grade 4 Go Math Review/test Solutions of chapter 11 Angles, you can easily learn the topics & score high marks in the standard assessments, tests, etc. Make sure that the answers you practiced can be checked via Go Math Grade 4 Answer Key Homework FL Chapter 11 Angles & fix it properly.

Review/Test – Page No. 439

Choose the best term from the box.
Go Math Grade 4 Answer Key Homework FL Chapter 11 Angles Review Test img 1

Question 1.
The size of an angle can be measured using a tool called a
______________ .
________

Answer: Protractor
The size of an angle can be measured using a tool called a Protractor

Question 2.
___________ is the direction in which the hands of a clock move.
________

Answer: Clockwise
Clockwise is the direction in which the hands of a clock move.

Tell what fraction of the circle the shaded angle represents.

Question 3.
Go Math Grade 4 Answer Key Homework FL Chapter 11 Angles Review Test img 2
\(\frac{□}{□}\)

Answer: \(\frac{1}{4}\)

Explanation:
The figure shows that the 1/4th part of the circle is shaded. So, the fraction of the shaded angle is 1/4.

Question 4.
Go Math Grade 4 Answer Key Homework FL Chapter 11 Angles Review Test img 3
\(\frac{□}{□}\)

Answer: \(\frac{3}{4}\)

Explanation:
It has completed a 3/4 turn. So, the fraction of the shaded part is 3/4.

Question 5.
Go Math Grade 4 Answer Key Homework FL Chapter 11 Angles Review Test img 4
\(\frac{□}{□}\)

Answer: \(\frac{1}{2}\)

Explanation:
From the figure, we can see that the circle is rotating in the anti-clockwise direction. And it has completed the half turn.
Thus the fraction is 12 turn counterclockwise

Use a protractor to draw the angle.

Question 6.
68°
Type below:
________

Answer:

Go Math Grade 4 Chapter 11 Angles Review Test img_2

Question 7.
145°
Type below:
________

Answer:

Go Math Grade 4 Chapter 8 Review Test Answer Key img_1

Question 8.
Use a protractor to find the measure of each angle. Label each angle with its measure. Write the sum of the angle measures as an equation.
Go Math Grade 4 Answer Key Homework FL Chapter 11 Angles Review Test img 5

Answer: 110°, 120°, 130°

Explanation:
By using the protractor we can measure each angle of the above circle.
∠NMO = 110°,
∠OMP = 120°,
∠NMP = 130°

Review/Test – Page No. 440

Fill in the bubble completely to show your answer.

Question 9.
Which describes the turn the angle on the circle shows?
Go Math Grade 4 Answer Key Homework FL Chapter 11 Angles Review Test img 6
Options:
a. 90° clockwise
b. 90° counterclockwise
c. 180° clockwise
d. 180° counterclockwise

Answer: 180° counterclockwise

Explanation:
By seeing the above circle we can say that it turns counterclockwise at 180°.
Thus the correct answer is option d.

Question 10.
Which best describes the m/RST? Use a protractor to help you.
Go Math Grade 4 Answer Key Homework FL Chapter 11 Angles Review Test img 7
Options:
a. acute; 48°
b. obtuse; 48°
c. obtuse; 132°
d. obtuse; 148°

Answer: obtuse; 148°

Explanation:
By using the protractor we can measure the angle of the above figure.
The above figure is greater than 90 degrees, so it is an obtuse angle. The measure of the angle is 148 degrees.
Thus the correct answer is option d.

Question 11.
The pocket watch was invented in 1524. The time is 6 P.M. After 1 hour, how many degrees does the minute hand turn?
Go Math Grade 4 Answer Key Homework FL Chapter 11 Angles Review Test img 8
Options:
a. 45°
b. 90°
c. 180°
d. 360°

Answer: 360°

Explanation:
Pocket watches consist of a circular face and three hands that complete a full revolution at different rates: the second hand takes 60 seconds, the minute hand takes 60 minutes, and the hour hand takes 12 hours.
There are 60 seconds in one hour, so in one hour the minute hand has completed a single revolution.
Circles contain 360 degrees so the minute hand, by completing one circle, has traveled 360 degrees after one hour.
Thus the correct answer is option d.

Review/Test – Page No. 441

Fill in the bubble completely to show your answer.

Question 12.
What is the unknown angle measure?
Go Math Grade 4 Answer Key Homework FL Chapter 11 Angles Review Test img 9
Options:
a. 25°
b. 115°
c. 125°
d. 180°

Answer: 125°

Explanation:
Sum of the angles = 180°
65° + x° = 180°
x° = 180° – 65°
x° = 125°
Thus the correct answer is option c.

Question 13.
Which equation can you use to find the ∠WRT?
Go Math Grade 4 Answer Key Homework FL Chapter 11 Angles Review Test img 10
Options:
a. 84° + 69° = ■
b. 84°− 69° = ■
c. 84° × 69° = ■
d. 84° − 153° = ■

Answer: 84° + 69° = ■

Explanation:
To find the unknown angle, we have to do the sum of two angles.
84° + 69° = ■
Thus the correct answer is option a.

Question 14.
If an angle measures 100º, through what fraction of a circle does the angle turn?
Options:
a. \(\frac{1}{100}\)
b. \(\frac{1}{4}\)
c. \(\frac{100}{360}\)
d. \(\frac{1}{2}\)

Answer: \(\frac{100}{360}\)

Explanation:
The complete angle is 360°
The angle measures 100º
= \(\frac{100}{360}\)
Thus the correct answer is option c.

Review/Test – Page No. 442

Question 15.
How many right angles are there in an angle that turns through 360º? Explain how you know.
______ right angles

Answer: 4

Explanation:
A circle has 4 right angles. So, an angle that turns through 360º has four right angles.

Question 16.
Soccer practice began at 2:30 P.M. and stopped at 3:00 P.M. because of rain. During this time, through what fraction of a circle did the minute hand turn? How many degrees did the minute hand turn? Explain.

Answer: 30 minutes

Explanation:
A minute watch covers 360 degrees every 60 minutes. In 30 minutes the watch, therefore, covers 180 degrees.

Question 17.
Charlotte divided a whole pizza into 4 pieces. One piece formed a straight angle. One piece formed a right angle. Two pieces formed acute angles with the same degree measure.
A. Draw angles to represent the 4 pieces.
Go Math Grade 4 Answer Key Homework FL Chapter 11 Angles Review Test img 11

Answer:
Go-Math-Grade-4-Answer-Key-Homework-FL-Chapter-11-Angles-Review-Test-img-11

Question 17.
B. Label each angle with its degree measure.

Answer:
Go-Math-Grade-4-Answer-Key-Homework-FL-Chapter-11-Angles-Review-Test-img-11

Question 17.
C. Label each angle as a fraction of a circle.

Answer:
Go-Math-Grade-4-Answer-Key-Homework-FL-Chapter-11-Angles-Review-Test-img-11

Question 17.
D. Write an equation that represents the degree measure of the whole pizza as the sum of the measures of its parts.

Answer:
x = 60/360x + 60/360x + 110/360x + 130/360x
x = 1/6x + 1/6x + 11/36x + 13/36x

Conclusion:

Stay connected with our site and find Grade 4 Go Math Answer Key in pdf format to get the explanations for all the chapters. Have any queries about this Go Math Grade 4 Solution Key Chapter 11 Angles pdf article, check out our page or comment us below. Best Of Luck!!!

Go Math Grade 3 Answer Key Chapter 5 Use Multiplication Facts

go-math-grade-3-chapter-5-use-multiplication-facts-answer-key

Access Go Math Grade 3 Answer Key Chapter 5 Use Multiplication Facts here for quick reference. Solve various questions from Go Math Grade 3 Answer Key Chapter 5 and get basics of multiplication easily. Resolve your queries and practice on your own to understand where you stand in your preparation. Attain the logic behind each problem in the exercise questions from 3rd Grade Go Math Answer Key Ch 5 USe Multiplication Facts and clear the exam with better grades.

Go Math Grade 3 Answer Key Chapter 5 Use Multiplication Facts

Step by Step Solution is given in the HMH Go Math Grade 3 Answer Key by subject experts keeping in mind the student’s level of understanding. Have an overview of the concepts present in Grade 3 Chapter 5 through the quick links available. Utilize the Grade 3 HMH Go Math Answer Key Chapter 5 Use Multiplication Facts and clear your queries regarding the topics instantly. You just need to click on the below mentioned Go Math 3rd standard Grade 3 Chapter Key links and take your preparation to the next level.

Lesson 1: Algebra • Describe Patterns

Lesson 2: Algebra • Find Unknown Numbers

Mid-Chapter Checkpoint

Lesson 3: Problem Solving • Use the Distributive Property

Lesson 4: Multiplication Strategies with Multiples of 10

Lesson 5: Multiply 1-Digit Numbers by Multiplies of 10

Chapter 5 Review/Test

Describe Patterns Page No 265

Describe a pattern for the table. Then complete the table.

Question 1.
Go Math Grade 3 Answer Key Chapter 5 Use Multiplication Facts Describe Patterns img 1
Answer: Add 6 muffins for each pan; Multiply the number of pans by 6.

Explanation:

Now we have to multiple no. of pans by 6
4 × 6 = 24; 5 × 6 = 30

Question 2.

Wagons 2 3 4 5 6
Wheels 8 12 16 _______ _______

Answer:

Wagons 2 3 4 5 6
Wheels 8 12 16 20 24

Explanation:

Add 4 wheels for each Wagons; Multiply the number of Wagons by 4
5 × 4 = 20; 6 × 4 = 24

Question 3.

Vases Flowers
2 14
3 _______
4 28
5 _______
6 42

Answer:

Vases Flowers
2 14
3 21
4 28
5 35
6 42

Explanation:

Add 7 flowers for each vase; Multiply the number of flowers by 7
3 × 7 = 21; 5 × 7 = 35

Question 4.

Spiders Legs
1 8
2 _______
3 24
4 _______
5 40

Answer:

Spiders Legs
1 8
2 16
3 24
4 32
5 40

Explanation:

Add 8 legs for each spider and then multiply the number of spiders by 8
i.e., 8 × 2 = 16; 8 × 4 = 32

Problem Solving

Question 5.
Caleb buys 5 cartons of yogurt. Each carton has 8 yogurt cups. How many yogurt cups does Caleb buy?
_________ yogurt cups

Answer: 40 cups

Explanation:

Given,
Caleb buys 5 cartons of yogurt
Each carton has 8 yogurt cups
To find how many yogurt cups does Caleb buy ‘x’
x = 8 × 5 = 40
Thus the number of yogurt cups does Caleb buy is 40 cups

Question 6.
Libby bought 4 packages of pencils. Each package has 6 pencils. How many pencils did Libby buy?
_________ pencils

Answer: 24 pencils

Explanation:

Libby bought 4 packages of pencils
Each package has 6 pencils
Number of pencils did Libby buy is x
x × 1= 6 × 4
x = 24
Therefore no. of pencils Libby bought = 24 pencils

Describe Patterns Lesson Check Page No 266

Question 1.
Which of the following describes a pattern in the table?
Go Math Grade 3 Answer Key Chapter 5 Use Multiplication Facts Describe Patterns img 2
Options:
a. Multiply by 3.
b. Multiply by 5.
c. Add 1.
d. Add 4.

Answer: Multiply by 5

Explanation:

From the above table, we can see that each chair is added by 5 for each table.
So, multiply the number of tables by 5
The correct answer is option B

Question 2.
Which number completes this table?
Go Math Grade 3 Answer Key Chapter 5 Use Multiplication Facts Describe Patterns img 3
Options:
a. 30
b. 20
c. 24
d. 22

Answer: 24

Explanation:

Each butterfly is added by 4
Multiply the number of butterflies by 4
4 × 6 = 24 wings
So, the correct answer is option C

Spiral Review

Question 3.
Jennilee buys 7 packs of crayons. There are 6 crayons in each pack. How many crayons does Jennilee buy in all?
Options:
a. 13
b. 36
c. 42
d. 48

Answer: 42

Explanation:

Given that, Jennilee buys 7 packs of crayons
There are 6 crayons in each pack
1 pack = 6 crayons
7 packs = x
x × 1 = 6 × 7
x = 42

Question 4.
Maverick has 5 books of circus tickets. Each book has 5 tickets. How many tickets does Maverick have in all?
Options:
a. 10
b. 15
c. 20
d. 25

Answer: 25

Explanation:

Maverick has 5 books on circus tickets
Each book has 5 tickets
1 book =5 tickets
5 books = x tickets
x × 1 = 5 × 5
x = 25
Thus the correct answer is option D

Question 5.
Bailey walked his dog 2 times each day for 9 days. How many times did Bailey walk his dog in all?
Options:
a. 9
b. 11
c. 18
d. 27

Answer: 18

Explanation:

Given, Bailey walked his dog 2 times each day for 9 days
Number of times did Bailey walk his dog in all = x
x = 9 × 2
x = 18
Thus the correct answer is option C

Question 6.
Drew’s Tree Company delivers pear trees in groups of 4. Yesterday, the company delivered 8 groups of pear trees. How many pear trees were delivered in all?
Options:
a. 12
b. 16
c. 24
d. 32

Answer: 32

Explanation:

Given,

Drew’s Tree Company delivers pear trees in groups of 4
Yesterday, the company delivered 8 groups of pear trees
How many pear trees were delivered in all = x
x = 4 × 8
x = 32
Thus the correct answer is option D

Find Unknown Numbers Page No 271

Find the unknown factor.

Question 1.
n × 3 = 12
Think: How many groups of 3 equal 12?
n = 4

Answer: 4

Explanation:

n × 3 = 12
n = 12/4
n = 3

Question 2.
s × 8 = 64
s = ________

Answer: 8

Explanation:

How many groups of 8 equals 64?

s × 8 = 64
s = 64/8
s = 8

Question 3.
21 = 7 × n
n = ________

Answer: 3

Explanation:

Number of groups 7 equals 21

21 = 7 × n
n = 21/7
n = 3

Question 4.
y × 2 = 18
y = ________

Answer: 9

Explanation:

y × 2 = 18
y = 18/2
y = 9

Question 5.
5 × p = 10
p = ________

Answer: 2

Explanation:

p is the unknown factor

5p = 10
p = 10/5
p = 2

Question 6.
56 = 8 × t
t = ________

Answer: 7

Explanation:

8 × t = 56
8t = 56
t = 56/8
t = 7
Therefore, the answer is 7

Question 7.
m × 4 = 28
m = ________

Answer: 7

Explanation:

Here m is the unknown product
4 × m = 28
4m = 28
m = 28/4
m = 7

Question 8.
★ × 1 = 9
★ = ________

Answer: 9

Explanation:

Here the symbol ★ is the unknown product
★ = 9/1
★ = 9

Question 9.
18 = 6 × r
r = ________

Answer: 3

Explanation:

r is the unknown product
6 × r = 18
r = 18/6
r = 3

Question 10.
u × 5 = 30
u = ________

Answer: 6

Explanation:

u is the unknown product
u × 5 = 30
5u = 30
u = 30/5
u = 6

Question 11.
4 × ■ = 24
■ = ________

Answer: 6

Explanation:

■ is the unknown product
4 × ■ = 24
■ = 24/4
■ = 6
Therefore the answer is 6

Question 12.
w × 7 = 35
w = ________

Answer: 5

Explanation:

The letter w is the unknown product
w × 7 = 35
w = 35/7
w = 5

Question 13.
b × 6 = 54
b = ________

Answer: 9

Explanation:

b × 6 = 54
b = 54/6
b = 9
Thus the unknown product is 9

Question 14.
5 × ▲ = 40
▲ = ________

Answer: 8

Explanation:

Here the symbol ▲ is the unknown product
▲ × 5 = 40
▲ = 40/5
▲ = 8

Question 15.
30 = d × 3
d = ________

Answer: 10

Explanation:

d is the unknown product
30 = d × 3
30/3 = d
d = 30/3
d = 10
Therefore the answer is 10

Question 16.
7 × k = 42
k = ________

Answer: 6

Explanation:

k is the unknown product
7 × k = 42
k = 42/7
k = 6

Problem Solving

Question 17.
Carmen spent $42 for 6 hats. How much did each hat cost?
$ ________

Answer: 7

Explanation:

Given that
Carmen spent $42 for 6 hats
The cost spent on each hat = x
x × 6 = 42
x = 42/6
x = 7
Thus Carmen spent $7 for each hat

Question 18.
Mark has a baking tray with 24 cupcakes. The cupcakes are arranged in 4 equal rows. How many cupcakes are in each row?
________ cupcakes

Answer: 6 cupcakes

Explanation:

Mark has a baking tray with 24 cupcakes
The cupcakes are arranged in 4 equal rows
No. of cupcakes in each row = x
x × 4 = 24
x = 24/4
x = 6
Therefore the number of cupcakes in each row = 6

Find Unknown Numbers Lesson Check Page No 272

Question 1.
What is the unknown factor?
b × 7 = 56
Options:
a. 6
b. 7
c. 8
d. 9

Answer: 8

Explanation:

b × 7 = 56
b = 56/7
Now we have to check how many groups of 7 equals 56
7 × 8 = 56
b = 8
Thus the answer is option C

Question 2.
What is the unknown factor shown by this array?
Go Math Grade 3 Answer Key Chapter 5 Use Multiplication Facts Find Unknown Numbers img 4
3 × ■ = 24
Options:
a. 3
b. 6
c. 8
d. 9

Answer: 8

Explanation:

The unknown product is ■
Here we have to find the product of 3 × ■  which equals 24
3 × ■ = 24
■ = 24/3
■ = 8
Therefore the unknown factor shown by this array is 8

Spiral Review

Question 3.
Which is an example of the Commutative Property of Multiplication?
Options:
a. 6 + 4 = 4 + 6
b. 4 × 6 = 6 × 4
c. 4 × 3 = 4 + 8
d. 3 × 6 = 9 × 2

Answer: 4 × 6 = 6 × 4

Explanation:

According to the commutative property of multiplication, changing the order of the numbers we are multiplying, does not change the product.
a × b = b × a
So, the answer is 4 × 6 = 6 × 4

Question 4.
Find the product.
5 × (4 × 2)
Options:
a. 13
b. 22
c. 40
d. 80

Answer: 40

Explanation:

This is in the form of a × (b × c)
First, multiply 4 and 2
5 × (4 × 2) = 5 × 8
5 × 8 = 40
Therefore the correct answer is option D

Question 5.
Which number sentence is an example of the Distributive Property?
Options:
a. 4 × 7 = (4 × 3) + (4 × 4)
b. 4 × 7 = 7 × 4
c. 4 × 7 = 28
d. 7 × 4 = 15 + 13

Answer: 4 × 7 = (4 × 3) + (4 × 4)

Example:

To “distribute” means to divide something or give a share or part of something. According to the distributive property, multiplying the sum of two or more addends by a number will give the same result as multiplying each addend individually by the number and then adding the products together.
The example of Distributive Property is 4 × 7 = (4 × 3) + (4 × 4)

Question 6.
In a group of 10 boys, each boy had 2 hats. How many hats did they have in all?
Options:
a. 5
b. 12
c. 20
d. 40

Answer: 20

Explanation:

Given that,
Each boy has 2 hats
Group of 10 boys has x hats
x × 1 = 2 × 10
x = 20

Mid-Chapter Checkpoint Page No 273

Vocabulary
Choose the best term from the box.
Go Math Grade 3 Answer Key Chapter 5 Use Multiplication Facts Mid -Chapter Checkpoint img 5

Question 1.
An __________ is a number sentence that uses the equal sign to show that two amounts are equal.
__________

Answer: Equation

Explanation:

The definition of the Equation is the number sentence that uses an equal sign to show that two amounts are equal.

Concepts and Skills

Describe a pattern in the table. Then complete the table.

Question 2.
Go Math Grade 3 Answer Key Chapter 5 Use Multiplication Facts Mid -Chapter Checkpoint img 6
Type below:
__________

Answer:

Weeks 1 2 3 4 5
Days 7 14 21 28 35

Explanation:

First of all look for the pattern to complete the table.
As you look across the rows you can find the days are increased by 7 for each week.
Now use the pattern to find the number of days in 4 and 5 weeks.
7 × 4 = 28; 7 × 5 = 35

Question 3.
Go Math Grade 3 Answer Key Chapter 5 Use Multiplication Facts Mid -Chapter Checkpoint img 7
Type below:
__________

Answer:

Tickets 2 3 4 5 6
Cost $8 $12 $16 $20 $24

Explanation:

Look for the pattern to complete the table.
As you look across the rows you can find the cost increased by $4 for each ticket.
Now use the pattern to find the cost for 5 and 6 tickets
4 × 5 = 20; 4 × 6 = 24

Question 4.
Go Math Grade 3 Answer Key Chapter 5 Use Multiplication Facts Mid -Chapter Checkpoint img 8
Type below:
__________

Answer:

Project Teams Members
3 9
4 12
5 15
6 18
7 21

Explanation:

Look for the pattern by comparing the columns in the table. You can multiply number of project teams by 3 to find the members.
3 × 5 = 15; 3 × 7 = 21

Question 5.
Go Math Grade 3 Answer Key Chapter 5 Use Multiplication Facts Mid -Chapter Checkpoint img 9
Type below:
__________

Answer:

Tables Chairs
1 8
2 16
3 24
4 32
5 40

Explanation:

Look for the pattern by comparing the rows in the table.
Now multiply the number of tables by 8 so that you can find the number of chairs for 3 and 5 tables.
3 × 8 = 24; 5 × 8 = 40

Find the unknown number.

Question 6.
m × 5 = 30
m = _______

Answer: 6

Explanation:

m is the unknown product
m × 5 = 30
m = 30/5
m = 6
Therefore the value of m is 6

Question 7.
■ × 6 = 48
■ = _______

Answer: 8

Explanation:

■ is the symbol of the unknown product
■ × 6 = 48
■ = 48/6
■ = 8

Question 8.
n = 2 × 10
n = _______

Answer: 5

Explanation:

n is the unknown product
We have to product of where 2 and 10 meet
n = 2 × 10
n = 20

Question 9.
4 × 8 = p
p = _______

Answer: 32

Explanation:

p is the unknown product
p = 8 × 4
p = 32
So, the answer is 32

Question 10.
25 = y × 5
y = _______

Answer: 5

Explanation:

y is the unknown product
y × 5 = 25
y = 25/5
y = 5

Question 11.
★ × 10 = 10
★ = _______

Answer: 1

Explanation:

★ is the symbol of the unknown product
★ × 10 = 10
★ = 10/10
★ = 1

Mid-Chapter Checkpoint Lesson Check Page No 274

Question 12.
Describe a pattern in the table.
Go Math Grade 3 Answer Key Chapter 5 Use Multiplication Facts Mid -Chapter Checkpoint img 10
Type below:
__________

Answer: Multiply by 6

Explanation:

As you look across the rows, you can see that the number of stickers increases by 6 for each package.

Question 13.
What number makes the equation true?
a × 8 = 72
a = _______

Answer: 9

Explanation:

a is the unknown product
Here we have to find the number that makes the equation true
a × 8 = 72
a = 72/8
a = 9

Question 14.
Mia bought 2 copies of the same book. She spent $18. What was the cost of one book?
$ _______

Answer: $9

Explanation:

Given that, Mia bought 2 copies of the same book.
Mia spent $18 for 2 books
Cost of one book = x
x × 2 = $18
x = 18/2
x = 9
Therefore the cost of one book = $9

Question 15.
Kyle saves $10 every week for 6 weeks. How much money will Kyle have in Week 6?
Go Math Grade 3 Answer Key Chapter 5 Use Multiplication Facts Mid -Chapter Checkpoint img 11
a. 60

Answer:

Weeks 1 2 3 4 5 6
Amount $10 $20 $30 $40 $50 $60

Explanation:

Look for the pattern to complete the table.
As you look across the rows you can find the amount increased for each week.
You can multiply 10 by week 4, 5 and 6
i.e., 10 × 4 = 40; 10 × 5 = 50; 10 × 6 = 60

Question 16.
Tennis balls cost $7 for a can of 3. Steve gives the cashier $40 to buy balls and receives $12 in change. How many tennis balls did Steve buy?
_______ tennis balls

Answer: 12 tennis balls

Explanation:

Steve spent $40 – $12 = $28
Let y represent the number of cans.
Tennis balls cost $7 for a can of 3
7 × y = 28
y = 28/7
y = 4 cans
So, Steve buys 4 cans of 3 tennis balls
Then, we need to multiply the no. of cans, 4, by the number of tennis balls in each can, 3
4 × 3 = 12 tennis balls

Use the Distributive Property Page No 279

Read each problem and solve.

Question 1.
Each time a student turns in a perfect spelling test, Ms. Ricks puts an achievement square on the bulletin board. There are 6 rows of squares on the bulletin board. Each row has 30 squares. How many perfect spelling tests have been turned in?
Think: 6 × 30 = 6 × (10 + 10 + 10)
= 60 + 60 + 60 = 180
180 spelling tests

Answer: 180 spelling test

Explanation:

There are 6 rows of squares on the bulletin board
Each row has 30 squares
We can use the distributive property to find the number of perfect spelling tests have been turned in
6 × 30 = 6 × (10+10+10)
6 × 10 + 6 × 10 + 6 × 10
60 + 60 + 60 = 180 spelling test

Question 2.
Norma practices violin for 50 minutes every day. How many minutes does Norma practice violin in 7 days?
_______ minutes

Answer: 350 minutes

Explanation:

Given,
Norma practices violin for 50 minutes every day
To find:
How many minutes does Norma practice violin in 7 days?
We can solve this problem by using the distributive property
7 × 50 = 7 × (20 + 30) = (7 × 20) + (7 × 30)
= 350 minutes

Question 3.
A kitchen designer is creating a new backsplash for the wall behind a kitchen sink. The backsplash will have 5 rows of tiles. Each row will have 20 tiles. How many tiles are needed for the entire backsplash?
_______ tiles

Answer: 100 tiles

Explanation:

Given: The backsplash will have 5 rows of tiles
Each row will have 20 tiles
By using a distributive property we can know the no. of tiles are needed for the entire backsplash
5 × 20 = 5 × (10 + 10)
(5× 10) + (5× 10) = 50 +50
= 100 tiles
Therefore 100 tiles are needed for the entire backsplash

Question 4.
A bowling alley keeps shoes in rows of cubbyholes. There are 9 rows of cubbyholes, with 20 cubbyholes in each row. If there is a pair of shoes in every cubbyhole, how many pairs of shoes are there?
_______ pairs of shoes

Answer: 180 pairs of shoes

Explanation:

There are 9 rows of cubbyholes, with 20 cubbyholes in each row
To find the number of pairs of shoes in every cubbyhole
9 × 20 = 9 × (10 + 10)
9 × 10 + 9 × 10
90 + 90 = 180 pairs of shoes

Question 5.
The third-grade students are traveling to the science museum in 8 buses. There are 40 students on each bus. How many students are going to the museum?
_______ students

Answer: 320 students

Explanation:

Given that, The third-grade students are traveling to the science museum in 8 buses
There are 40 students on each bus
We can know the number of students going to the museum by using the distributive property
8 × 40 = 8 × (20 + 20) = 8 × 20 + 8 × 20
160 + 160 = 320 students
Therefore the number of students going to the museum is 320

Use the Distributive Property Lesson Check Page No 280

Question 1.
Each snack pack holds 20 crackers. How many crackers in all are there in 4 snack packs?
Options:
a. 60
b. 80
c. 100
d. 800

Answer: 80

Explanation:

Given:
Each snack pack holds 20 crackers
To find:
How many crackers in all are there in 4 snack packs
By using the Distributive property we can find the crackers in 4 snack packs
4 × 20 = 4 × (10 + 10)
4 × 10 + 4 × 10 = 40 + 40 = 80
Thus the correct answer is option B

Question 2.
A machine makes 70 springs each hour. How many springs will the machine make in 8 hours?
Options:
a. 500
b. 520
c. 540
d. 560

Answer: 560

Explanation:

Given,
A machine makes 70 springs each hour
To find:
How many springs will the machine make in 8 hours
8 × 70 = 8 × (35 + 35)
= (8 × 35) + (8 × 35)
= 280 + 280
= 560
Thus option D is the correct answer

Spiral Review

Question 3.
Lila read 142 pages on Friday and 168 pages on Saturday. Which is the best estimate of how many pages Lila read on Friday and Saturday combined?
Options:
a. 100
b. 200
c. 300
d. 400

Answer: 300

Explanation:

Lila read 142 pages on Friday and 168 pages on Saturday
We can estimate the number of pages Lila read on Friday and Saturday combined by using the Distributive property
142 + 168 = (2 × 71) + (2 × 84) = 300
So, the correct answer is option C

Question 4.
Jessica wrote 6 + 6 + 6 + 6 on the board. Which is another way to show 6 + 6 + 6 + 6?
Options:
a. 4 × 4
b. 4 × 6
c. 4 × 4 × 6
d. 6 × 6

Answer: 4 × 6

Explanation:
Jessica wrote 6 + 6 + 6 + 6 on the board
The another way to write 6 + 6 + 6 + 6 is 4 × 6
Because here 6 is added 4 times. So the multiplication form of 6 + 6 + 6 + 6 is 4 × 6
So, the correct answer is option B

Use the line plot for 5–6.
Go Math Grade 3 Answer Key Chapter 5 Use Multiplication Facts Use the Distributive Property img 12

Question 5.
Eliot made a line plot to record the number of birds he saw at his bird feeder. How many more sparrows than blue jays did he see?
Options:
a. 2
b. 3
c. 4
d. 5

Answer: 4

Explanation:

Number of sparrows = 5
Number of Blue Jays = 1
To know how many more sparrows than blue jays we have to subtract number of blue jay from number of sparrows
= 5 – 1 = 4
So, the correct answer is option C

Question 6.
How many robins and cardinals combined did Eliot see?
Options:
a. 2
b. 3
c. 4
d. 5

Answer: 5

Explanation:

Number of robins = 3
Number of Cardinals = 2
Total Number of robins and cardinals = 3 + 2 = 5
So, the answer is option D

Multiplication Strategies with Multiples of 10 Page No 285

Use a number line to find the product.

Question 1.
2 × 40 = 80
Go Math Grade 3 Answer Key Chapter 5 Use Multiplication Facts Multiplication Strategies with Multiples of 10 img 13

Answer: 80

Explanation:

The number line given above shows that there are 2 groups of 4 tens
So, 2 × 4 tens
2 × 40 = 80

Question 2.
Go Math Grade 3 Answer Key Chapter 5 Use Multiplication Facts Multiplication Strategies with Multiples of 10 img 14
4 × 30 = _______

Answer: 120

Explanation:

There are 4 groups of 3 tens
So, the number jumps from 0 to 30, 30 to 60, 60 to 90, and from 90 to 120.
4 × 3 tens = 4 × 30 = 120

Use place value to find the product.

Question 3.
5 × 70 = 5 × _______ tens
= _______ tens = _______

Answer:

i. 7 tens
ii. 35 tens
iii. 350

Explanation:

70 = 7 × 10 = 7 tens
5 × 70 = 35 × 10 = 35 tens = 350

Question 4.
60 × 4 = _______ tens × 4
= _______ tens = _______

Answer:

i. 6 tens
ii. 24 tens
iii. 240

Explanation:

Here 60 is multiplied with 4
60 = 6 × 10 = 6 tens
60 × 4 = 6 tens × 4
24 tens = 24 × 10 = 240

Question 5.
7 × 30 = 7 × _______ tens
= _______ tens = _______

Answer:

i. 3 tens
ii. 21 tens
iii. 210

Explanation:

30 = 3 × 10 = 3 × 1 ten = 3 tens
7 × 30 = 7 × 3 tens
= 21 tens
= 21 × 10 = 210

Question 6.
90 × 3 = _______ tens × 3
= tens = _______

Answer:

i. 9 tens
ii. 27 tens
iii. 270

Explanation:

90 = 9 × 1 ten = 9 × 10 = 9 tens
9 tens × 3 = 27 tens
27 tens = 27 × 1 ten
= 27 × 10 = 270

Problem Solving

Question 7.
One exhibit at the aquarium has 5 fish tanks. Each fish tank holds 50 gallons of water. How much water do the 5 tanks hold in all?
_______ gallons of water

Answer: 250 gallons of water

Explanation:

Given: One exhibit at the aquarium has 5 fish tanks
Each fish tank holds 50 gallons of water
5 × 50 = 5 × 5 tens
= 25 tens = 25 × 1 ten
25 × 10 = 250 gallons of water

Question 8.
In another aquarium display, there are 40 fish in each of 7 large tanks. How many fish are in the display in all?
_______ fish

Answer: 280 fish

Explanation:

There are 40 fish in each of 7 large tanks
To know the number of fishe are in the display in all
40 × 7 = 4 × 1 ten = 4 tens
4 tens × 7 = 28 tens
28 × 10 = 280 fishes

Multiplication Strategies with Multiples of 10 Lesson Check Page No 286

Question 1.
Each bag of pattern blocks contains 50 blocks. To make a class pattern, the teacher combines 4 bags of blocks. How many pattern blocks are there in all?
Options:
a. 20
b. 200
c. 240
d. 250

Answer: 200

Explanation:

Given,
Each bag of pattern blocks contains 50 blocks
To make a class pattern, the teacher combines 4 bags of blocks
Here we make use of multiplication strategies to know the number of pattern blocks
50 × 4 = 5 tens × 4
= 20 tens = 20 × 10 = 200
Therefore the correct answer is option B

Question 2.
A deli received 8 blocks of cheese. Each block of cheese weighs 60 ounces. What is the total weight of the cheeses?
Options:
a. 420 ounces
b. 460 ounces
c. 480 ounces
d. 560 ounces

Answer: 480 ounces

Explanation:

A deli received 8 blocks of cheese
Each block of cheese weighs 60 ounces
60 × 8 = 6 tens × 8
48 tens = 48 × 10 = 480 ounces
So, the correct answer is option C

Spiral Review

Question 3.
Alan and Betty collected cans for recycling. Alan collected 154 cans. Betty collected 215 cans. How many cans did they collect in all?
Options:
a. 369
b. 379
c. 469
d. 479

Answer: 369

Explanation:

Given, Alan and Betty collected cans for recycling
Alan collected 154 cans
Betty collected 215 cans
To know total cans they collected in all, we have to add both the cans of Alan and Betty
154 + 215 = 369 cans
Therefore the correct answer is option A

Question 4.
The third graders collected 754 cans. The fourth graders collected 592 cans. Which is the best estimate of how many more cans the third graders collected?
Options:
a. 50
b. 100
c. 200
d. 300

Answer: 200

Explanation:

Given,
The third graders collected 754 cans
The fourth graders collected 592 cans
To find the best estimate of how many more cans the third graders collected
We have to subtract fourth graders cans from third graders can
Here they are asking the estimated number if cans the third graders collected
754 rounded to the nearest hundred is 800 and
592 rounded to the nearest hundred is 600
800 – 600 = 200
Therefore the correct answer is 200

Use the bar graph for 5–6.
Go Math Grade 3 Answer Key Chapter 5 Use Multiplication Facts Multiplication Strategies with Multiples of 10 img 15

Question 5.
How many more books did Ed read than Bob?
Options:
a. 2
b. 3
c. 4
d. 5

Answer: 3

Explanation:

The bar graph shows that Ed read 8 books in June
Bob read 5 books in June
To know the number of books Ed read than Bob
We have to subtract the no. of books Bob read from Ed
= 8 – 5 = 3 books
So, the correct answer is option B

Question 6.
How many books in all did the four students read in June?
Options:
a. 22
b. 24
c. 26
d. 36

Answer: 26

Explanation:

Bob read 5 books in June
Ed read 8 books in June
Eve read 7 books in June
Ann read 6 books in June
Total = 5 + 8 + 7 + 6 = 26 books
So, the answer is option C

Multiply 1-Digit Numbers by Multiplies of 10 Page No 291

Find the product. Use base-ten blocks or draw a quick picture.

Question 1.
4 × 50 = 200
Go Math Grade 3 Answer Key Chapter 5 Use Multiplication Facts Multiply 1-Digit Numbers by Multiples of 10 img 16

Answer: 200

Explanation:

First, multiply the ones
4 × o ones = 0
Next, multiply the tens
4 × 5 tens = 200

Question 2.
60 × 3 = _______

Answer: 180

Explanation:

Mutliply the ones
3 × o ones = 0
Now multiply the tens
3 × 6 tens = 18 tens = 180

Question 3.
_______ = 60 × 5

Answer: 300

Explanation:

Multiply the ones
5 × 0 ones = 0
Multiply the tens
5 × 6 tens = 30 tens
= 300

Find the product.

Question 4.
3 0
× 8
—–
_______

Answer: 240

Explanation:

Multiply the ones
8 × 0 ones = 0
Multiply the tens
8 × 3 tens = 24 tens = 240

Question 5.
5 0
× 2
—–
_______

Answer: 100

Explanation:

Multiply the ones
2 × 0 ones = 0
Multiply the tens
2 × 5 tens = 10 tens = 100

Question 6.
6 0
× 7
—–
_______

Answer: 420

Explanation:

Multiply the ones
7 × 0 ones = 0
Multiply the tens
7 × 6 tens = 42 tens = 420

Question 7.
70
× 4
—–
_______

Answer: 280

Explanation:

Multiply the ones
4× 0 ones = 0
Multiply the tens
4 × 7 tens = 28 tens = 280

Question 8.
6 × 90 = _______

Answer: 540

Explanation:

Multiply the ones
6 × 0 ones = 0
Multiply the tens
6 × 9 tens = 54 tens = 540

Question 9.
9 × 70 = _______

Answer: 630

Explanation:

Multiply the ones
9 × 0 ones = 0
Multiply the tens
9 × 7 tens = 63 tens = 630

Question 10.
8 × 90 = _______

Answer: 720

Explanation:

Multiply the ones
8 × 0 ones = 0
Multiply the tens
8 × 9 tens = 72 tens = 720

Question 11.
_______ = 6 × 80

Answer: 480

Explanation:

Multiply the ones
6 × 0 ones = 0
Multiply the tens
6 × 8 tens = 48 tens = 480

Problem Solving

Question 12.
Each model car in a set costs $4. There are 30 different model cars in the set. How much would it cost to buy all the model cars in the set?
$ _______

Answer: $120

Explanation:

There are 30 different model cars in the set
Each model car in a set costs $4
To find the cost to buy all the model cars in the set
30 × $4 = $120

Question 13.
Amanda exercises for 50 minutes each day. How many minutes will she exercise in 7 days?
minutes _______

Answer: 350 minutes

Explanation:

Given,
Amanda exercises for 50 minutes each day
We have to find how many minutes will she exercise in 7 days
50 × 7 = 350 minutes

Multiply 1-Digit Numbers by Multiplies of 10 Lesson Check Page No 292

Question 1.
Each shelf in one section of the library holds 30 books. There are 9 shelves in that section. How many books will these shelves hold?
Options:
a. 220
b. 260
c. 270
d. 280

Answer: 270

Explanation:

Given,

Each shelf in one section of the library holds 30 books
There are 9 shelves in that section
30 × 9 = 3 tens × 9
= 27 tens = 270
Therefore the correct answer is option C

Question 2.
One can of juice mix makes 60 ounces of juice. How many ounces of juice can be made from 6 cans of juice mix?
Options:
a. 300 ounces
b. 360 ounces
c. 390 ounces
d. 600 ounces

Answer: 360 ounces

Explanation:

Given,
One can of juice mix makes 60 ounces of juice
Number of ounces of juice can be made from 6 cans of juice mix
60 ounces × 6 = 6 tens × 6
= 36 tens = 360 ounces
Thus the answer is option B

Spiral Review

Question 3.
Sue bought 7 cans of tennis balls. There are 3 balls in each can. How many balls did Sue buy?
Options:
a. 10
b. 21
c. 28
d. 37

Answer: 21

Explanation:

Sue bought 7 cans of tennis balls
There are 3 balls in each can
To know the number of balls Sue buy
We have multiply number of cans and number of balls
= 7 × 3 = 21 balls

Question 4.
Which is an example of the Commutative Property of Multiplication?
Options:
a. 3 + 4 = 4 + 3
b. 5 × 0 = 0
c. 1 × 7 = 7
d. 3 × 4 = 4 × 3

Answer: 3 × 4 = 4 × 3

Explanation:

According to the commutative property of multiplication, changing the order of the numbers we are multiplying, does not change the product.
a × b = b × a
3 × 4 = 4 × 3
Option D is the correct example of the Commutative property

Question 5.
Lyn drew this bar model to solve a problem. Which operation should she use to find the unknown number?
Go Math Grade 3 Answer Key Chapter 5 Use Multiplication Facts Multiply 1-Digit Numbers by Multiples of 10 img 17
Options:
a. addition
b. division
c. multiplication
d. subtraction

Answer: addition

Explanation:

In order to know the unknown number we have to add both the number of flowers
90 + 54 = 144

Question 6.
Joe drew this bar model to find the unknown number of balls. Which is the correct answer?
Go Math Grade 3 Answer Key Chapter 5 Use Multiplication Facts Multiply 1-Digit Numbers by Multiples of 10 img 18
Options:
a. 356
b. 256
c. 144
d. 124

Answer: 144

Explanation:

Given that
Joe drew this bar model to find the unknown number of balls
106 balls + ___ = 250 balls
let the unknown number be x
106 + x = 250
x = 250 – 106
x = 144 balls

Review/Test Page No 293

Question 1.
The camping club wants to rent rafts. Each raft can hold 8 people. Which equation could be used to find how many rafts are needed for 32 people?
Options:
a. 8 × 32 = ■
b. 32 × ■ = 8
c. ■ × 8 = 32
d. 32 × 8 = ■

Answer: ■ × 8 = 32

Explanation:

Each raft can hold 8 people
■ represents raft that can hold 32 people
Now we have to multiple number of people with rafts for 32 people
■ × 8 = 32
■ = 32/8
■ = 4
Therefore 4 rafts are needed for 32 people

Question 2.
Select the equations that show the Distributive Property. Mark all that apply.
Options:
a. 8 × 20 = 8 × (10 + 10)
b. 5 × 60 = 5 × (20 + 40)
c. 30 × 6 = 6 × 30
d. 9 × (4 + 3) = 9 × 7

Answer: a, b, d

Explanation:

The sum of two numbers times a third number is equal to the sum of each addend times the third number.
The above 3 options satisfy the law of Distributive Property

Question 3.
Choose the number from the box that makes the sentence true.
A library has 48 shelves of fiction books. There are 6 shelves in each cabinet.
There are Go Math Grade 3 Answer Key Chapter 5 Use Multiplication Facts Review/Test img 19 cabinets of fiction books in the library.
_________

Answer: 8

Explanation:

Let x represents cabinets of fiction in the library
x × 6 = 48
x = 48/6
x = 8
Therefore the answer is 8

Review/Test Page No 294

Question 4.
For numbers 4a–4d, choose True or False for each equation.
a. 5 × (4 + 4) = 8 × 5
i. True
ii. False

Answer: True

Explanation:

The above question satisfies the distributive property
5 × (4 + 4) = 5 × 8 = 40
8 × 5 = 40
Therefore LHS = RHS
So, the equation is true

Question 4.
b. 8 × (3 + 3) = 8 × 5
i. True
ii. False

Answer: False

Explanation:

8 × (3 + 3) = 8 × 6 = 48
8 × 5 = 40
So, the equation is false

Question 4.
c. (3 × 5) + (5 × 5) = 8 × 5
i. True
ii. False

Answer: True

Explanation:

(3 × 5) + (5 × 5) = 8 × 5
The above question satisfies the distributive property
8 × 5 = 40
(3 × 5) + (5 × 5) = 15 + 25 = 40
Both LHS and RHS are equal
So, the answer is true

Question 4.
d. (3 × 2) + (8 × 3) = 8 × 5
i. True
ii. False

Answer: False

Explanation:

(3 × 2) + (8 × 3)
3 × 2 = 6; 8 × 3 = 24
= (3 × 2) + (8 × 3) = 6 + 24
But it is given as 8 × 5
So, the answer is false

Question 5.
Alya planted 30 trays of flowers. Each tray held 8 flowers. Javon planted 230 flowers. Did Alya plant more flowers than Javon, the same number of flowers as Javon, or fewer flowers than Javon?
Options:
a. She planted more flowers than Javon.
b. She planted the exact same number of flowers as Javon.
c. She planted fewer flowers than Javon. 6.

Answer: She planted more flowers than Javon.

Explanation:

Given that, Alya planted 30 trays of flowers and each tray held 8 flowers
30 × 8 = 240 flowers
So, Alya planted 240 flowers
Javon planted 230 flowers
To know whether Alya plant more flowers than Javon, the same number of flowers as Javon, or fewer flowers than Javon
We have to subtract Number of flowers planted by Javon from Alya
240 – 230 = 10
So, Alya planted more flowers than Javon

Question 6.
For numbers, 6a–6d, choose Yes or No to show whether the unknown number is 6.

a. 4 × ■ = 32

i. Yes
ii. No

Answer: No

Explanation:

■ = 6
Now we have to substitute ■ = 6 in the above question
4 × 6 = 24
So, the answer is No

b. ■ × 6 = 36

i. Yes
ii. No

Answer: Yes

Explanation:

■ = 6
6 × 6 = 36
It satisfies the above equation
So, the answer is Yes

c. 8 × ■ = 49

i. Yes
ii. No

Answer: No

Explanation:

■ = 6
8 × 6 = 48 but not 49
So, the answer is No

d. ■ × 30 = 180

i. Yes
ii. No

Answer:

Explanation:

■ = 6
6 × 30 = 180
It satisfies the above equation
So, the answer is yes

Question 7.

Each train can carry 20 cars. Use the number line to find how many cars 6 trains can carry.

Go Math Grade 3 Chapter 5 Answer Key Review

Answer: 120 cars

Chapter 5 Go Math Grade 3 Solution Key Review solution image_1

Explanation:

Each train can carry 20 cars
6 trains can carry x cars
x × 1 = 20 × 6
x = 120 cars

Review/Test Page No 295

Question 8.
Samantha made this multiplication model. Complete the equation that represents the model.
Go Math Grade 3 Answer Key Chapter 5 Use Multiplication Facts Review/Test img 20
_____ × _____ = _____
Type below:
__________

Answer: 90

Explanation:

Each vertical bar represents a ten. Each group of 3 vertical bars represents 30
We have 3 groups of 30, therefore the equation that represents the model is
3 × 30 = 90 models

Question 9.
A printer prints newsletters for many groups every month. Which group uses the greatest number of pieces of paper?
Go Math Grade 3 Answer Key Chapter 5 Use Multiplication Facts Review/Test img 21
__________

Answer: Book Lovers Club

Explanation:

From the above table, we can say that Book Lovers Club group uses the greatest number of pieces of paper

Question 10.
A store has 30 boxes of melons. Each box holds 4 bags. Each bag holds 2 melons. What is the total number of melons in the store?
__________ melons

Answer: 240 melons

Explanation:

Given,
A store has 30 boxes of melons
Each box holds 4 bags
Each bag holds 2 melons
1 box holds 4 bags
30 × 4 = 120 bags
1 bag = 2 melons
120 bags = x
x = 120 × 2
x = 240 melons
Therefore total number of melons in the store = 240 melons

Question 11.
Heather’s puppy weighs 23 pounds. He has been gaining 3 pounds every month as he grows. If this pattern continues, how much will the puppy weigh 5 months
from now?
__________ pounds

Answer: 38 pounds

Explanation:

Heather’s puppy weighs 23 pounds
He has been gaining 3 pounds every month as he grows
To find the weigh of the puppy after 5 months
If puppy weights 3 pounds in 1 month
For 5 months it weighs = 3 × 5 = 15 pounds
Actual of Heather’s puppy + puppy weight after 5 months
= 23 + 15 = 38 pounds

Review/Test Page No 296

Question 12.
Tim describes a pattern. He says the pattern shown in the table is “Add 3.” Is Tim correct? Explain how you know.
Go Math Grade 3 Answer Key Chapter 5 Use Multiplication Facts Review/Test img 22
Type below:
__________

Answer:

No, Tim is not correct
The pattern works for the first pair of numbers 1 + 3 = 4
But it doesn’t work for any of the other pairs. The pattern should be Multiply the number of packages by 4

Question 13.
This shows a part of a multiplication table. Find the missing numbers. Explain how you found the numbers.
Go Math Grade 3 Answer Key Chapter 5 Use Multiplication Facts Review/Test img 23
Type below:
__________

Answer:

Go Math Grade 3 Key Chapter 5 Review solution image_2

Explanation:

The only numbers that have a product of 35 are 5 and 7. The only numbers that have a product of 40 are 5 and 8.
This tells us that 5 is the number for the row. That means 6 is the next row down, and 7 is the row after that. The factors of 7 and 8 are the columns, so we can multiply to find the missing numbers

Question 14.
Describe a pattern for this table.
Go Math Grade 3 Answer Key Chapter 5 Use Multiplication Facts Review/Test img 24
Pattern: _____
How would the table change if the pattern was “Multiply the number of tanks by 8”? Explain.
Type below:
__________

Answer: Multiply the number of tanks by 80
The table would change by taking a zero off each number of fish in the second row because you would be multiplying by ones, not tens.

Review/Test Page No 297

Question 15.
Devon has 80 books to pack in boxes. She packs 20 books in each box. How many boxes does she need?
Write an equation using the letter n to stand for the unknown factor. Explain how to find the unknown factor.
Type below:
__________

Answer: n × 20 = 80
We can draw an array of 80 squares with 20 squares in each row.
There are 4 rows, so n = 4. Devon needs 4 boxes.

Go Math 3rd Grade Answer Key for Chapter 5 Review solution image_4

Question 16.
The bookstore has 6 shelves of books about animals. There are 30 books on each shelf. How many books about animals does the bookstore have?
Shade squares to make a diagram to show how you can use the Distributive Property to find the number of books about animals in the bookstore.
Go Math Grade 3 Answer Key Chapter 5 Use Multiplication Facts Review/Test img 25
__________ animal books

Answer:

6 × (10 + 10 + 10) = (6 × 10) + (6 × 10) + (6 × 10)
= 60 + 60 + 60
= 180 animal books

Grade 3 Go Math Answer Key Chapter 5 Review solution image_3

Review/Test Page No 298

Question 17.
Cody saves all his nickels. Today he is getting them out of his piggy bank and wrapping them to take to the bank. He finds he has 360 nickels. It takes 40 nickels to fill each paper wrapper and make a roll. How many wrappers does he need?
Part A
Write an equation using n for the unknown number. Find the number of wrappers needed.
______ × ______ = ______

Answer: n × 40 = 360

Explanation:

n represents no. of wrappers need
Each paper wrapper needs 40 nickels
Cody has 360 nickels
n × 40 = 360

Question 17.
Part B
Explain how you solved this problem and how you know your answer is correct.
Type below:
__________

Answer:

Explanation:

Given that Cony has 360 nickels
Each paper wrapper needs 40 nickels
Let n be the number of wrappers needed
That means n × 40 = 360
n = 360/40
n = 9
So, n wrappers are needed to make the rolls

Question 18.
Ruben is collecting cans for the recycling contest at school. He makes two plans to try to collect the most cans.
Plan A: Collect 20 cans each week for 9 weeks.
Plan B: Collect 30 cans each week for 7 weeks.
Part A
Which plan should Ruben choose?
__________

Answer: Plan B

Collect 30 cans each week for 7 weeks.

Question 18.
Part B
Explain how you made your choice.
Type below:
__________

Answer:

Given that Ruben is collecting cans for the recycling contest at school
His plan is to collect more cans
That means he has to collect more number of cans in less number of weeks
So, Plan B is perfect to win the recycling contest at school

We hope the info shared regarding the Go Math Grade 3 Chapter 5 Answer Key has been beneficial to you. Refer to Go Math Grade 3 Answer Key Chapter 5 Use Multiplication Facts Extra Practice. Practice is the only key to success and make the most out of the Answer Key available and achieve success in your assessments or tests.

Go Math Grade 6 Answer Key Chapter 1 Divide Multi-Digit Numbers

go-math-grade-6-chapter-1-divide-multi-digit-numbers-answer-key

Students who are in search of the Answer Key of Go Math Grade 6 Chapter 1 Divide Multi Digit Numbers can get them on this page. Refer to our Go Math Grade 6 Answer Key  and solve the problems. We provide the step by step explanations for all the question by using the pictures here. By using Go Math 6th Grade Chapter, 1 Answer Key the scholars will not find any difficulty in solving the questions.

Go Math Grade 6 Answer Key Chapter 1 Divide Multi-Digit Numbers 

In order to excel in exam we suggest the students of 6th standard to refer the Go Math Answer Key Chapter 1 Divide Multi-Digit Numbers. The solutions in Go Math Grade 6 Answer Key Chapter 1 Divide Multi-Digit Numbers are prepared by the math experts. You can understand the concepts in depth with the help of Go Math Grade 6 Solution Key.

Lesson 1: Divide Multi-Digit Numbers

Lesson 2: Prime Factorization

Lesson 3: Least Common Multiple

Lesson 4: Greatest Common Factor

Lesson 5: Problem Solving • Apply the Greatest Common Factor

Mid-Chapter Checkpoint

Lesson 6: Add and Subtract Decimals

Lesson 7: Multiply Decimals

Lesson 8: Divide Decimals by Whole Numbers

Lesson 9: Divide with Decimals

Chapter 1 Review/Test 

Page No. 7

Estimate. Then find the quotient. Write the remainder, if any, as a fraction.

Question 3.
6,114 ÷ 63

Answer:  Quotient is 97 3/63= 97 1/21 and the remainder is 3

Explanation:

Go Math Grade 6 Answer Key Chapter 1 Divide Multi-Digit Numbers

Question 4.
11050 ÷ 26

Answer: Quotient is 425 and the remainder is 0.

Explanation:
Go Math Grade 6 Answer Key Chapter 1 Divide Multi-Digit Numbers

On Your Own

Estimate. Then find the quotient. Write the remainder, if any, as a fraction.

Question 5.
3150 ÷ 9

Answer: Quotient is 350 and the remainder is 0.

Explanation:
Go Math Grade 6 Answer Key Chapter 1 Divide Multi-Digit Numbers

Question 6.
2115 ÷ 72

Answer: Quotient is 29 27/72= 29 3/8 and the remainder is 27.

Explanation:
Go Math Grade 6 Answer Key Chapter 1 Divide Multi-Digit Numbers

Question 7.
20835 ÷ 180

Answer: Quotient is 115 135/180= 115 3/4 and the remainder is 135

Explanation:
Go Math Grade 6 Answer Key Chapter 1 Divide Multi-Digit Numbers

Question 8.
Find the least whole number that can replace ? to make the statement true.
110 < ? ÷ 47

Answer: Least whole number is 5,171.

Explanation: 110×47= 5,170

Question 9.
Use Reasoning Name two whole numbers that can replace ? to make both statements true.
2 × ? < 1800 ÷ 12                         ? > 3744 ÷ 52

Answer:

Explanation:

Question 10.
The 128 employees of a company volunteer 12,480 hours in 26 weeks. On average, how many hours do they all volunteer per week? On average, how many hours does each employee volunteer per week?

Answer: 3.75 hours.

Explanation: In 26  weeks 128 employees volunteer 12,480 hours, so in 1 week they volunteer 12,480÷26= 480 hours.
And each employee volunteer per week is 480÷128= 3.75 hours.

Question 11.
A factory produces 30,480 bolts in 12 hours. If the same number of bolts are produced each hour, how many bolts does the factory produce in 5 hours?

Answer: 12,700.

Explanation: As the factory produces 30,480 bolts in 12 hours, so in a 1-hour company produces 30,480÷12= 2,450 boults and in 5 hours it produces 2,450×5= 12,700.

Problem Solving + Applications – Page No. 8

Use the table for 12-15.
Go Math Grade 6 Answer Key Chapter 1 Divide Multi-Digit Numbers img 1

Question 12.
A Smooth Flight jet carried 6,045 passengers last week, and all of its flights were full. How many flights did the jet make last week?

Answer: 15 flights.

Explanation: As there are 403 seats in Smooth flight and 6,045 passengers are carried in last week, so no.of flights did the jet make in last week are 6045÷403= 15

Question 13.
Last month an airline made 6,322 reservations for flights from Newark, New Jersey, to Frankfurt, Germany. If there were 21 full flights and 64 reservations cancelled, which airplane made the flights?

Answer: Jet Set.

Explanation: Total reservations made by the airline are 6,322 and cancelled are 64, so completed reservations are 6,322-64= 6,258 and 21 flights are full so 6258÷21= 298 passengers in each flight and that airplane is Jet set

Question 14.
An airline carries about 750 passengers from Houston to Chicago each day. How many Blue Sky jets would be needed to carry this many passengers, and how many empty seats would there be?

Answer: 5 Sky jets would be needed and 50 empty seats would be there.

Explanation: Let’s round off 750 to 800, As there are 800 passengers each day so no. of Sky jets needed are 800÷160= 5. and 50 empty seats would be there.

Question 15.
Pose a Problem Refer back to Problem 12. Use the information in the table to write a similar problem involving airplane passenger seats.

Answer:

Explanation:

Question 16.
For numbers 16a – 16d, choose Yes or No to indicate whether the equation is correct.
16a. 1,350 ÷ 5 = 270 O Yes O No
16b. 3,732 ÷ 4 = 933 O Yes O No
16c. 4,200 ÷ 35 = 12 O Yes O No
16d. 1,586 ÷ 13 = 122 O Yes O No

16a. Answer: Yes

Explanation: 1,350÷5= 270.

16b. Answer: Yes

Explanation: 3732÷4= 933

16c. Answer: No

Explanation: 4200÷35= 120

16d. Answer: Yes

Explanation: 1586÷13= 122.

Divide Multi-Digit Numbers – Page No. 9

Estimate. Then find the quotient. Write the remainder, if any, with an r.

Question 1.
180)\(\overline { 20835 } \)

Answer: Quotient is 115 and remainder r135

Explanation:
Go Math Grade 6 Answer Key Chapter 1 Divide Multi-Digit Numbers

Question 2.
19)\(\overline { 800 } \)

Answer: Quotient is 42 and remainder r2

Explanation:
Go Math Grade 6 Answer Key Chapter 1 Divide Multi-Digit Numbers

Question 3.
68)\(\overline { 1025 } \)

Answer: Quotient is 15 and remainder r5

Explanation:
Go Math Grade 6 Answer Key Chapter 1 Divide Multi-Digit Numbers

Estimate. Then find the quotient. Write the remainder, if any, as a fraction.

Question 4.
20)\(\overline { 1683 } \)

Answer: Quotient is 84 and remainder r3.

Explanation:
Go Math Grade 6 Answer Key Chapter 1 Divide Multi-Digit Numbers

Question 5.
14124 ÷ 44

Answer: Quotient is 321 and remainder r0

Explanation:
Go Math Grade 6 Answer Key Chapter 1 Divide Multi-Digit Numbers

Question 6.
11629 ÷ 29

Answer: Quotient is 401 and remainder r0

Explanation:
Go Math Grade 6 Answer Key Chapter 1 Divide Multi-Digit Numbers

Find the least whole number that can replace ? to make the statement true.

Question 7.
? ÷ 7 > 800

Answer: The least whole number to makes the statement true is 5600

Explanation: 5600÷7> 800

Question 8.
? ÷ 21 > 13

Answer: The least whole number to makes the statement true is 273

Explanation:  273÷21>13

Question 9.
15 < ? ÷ 400

Answer: The least whole number to makes the statement true is 6000

Explanation: 15< 6000÷400

Problem Solving

Question 10.
A plane flew a total of 2,220 miles. Its average speed was 555 miles per hour. How many hours did the plane fly?

Answer: 4 hours

Explanation: Total miles does a plane flew is 2,220 miles and average speed is 555 miles per hour. So total hours did plane fly are 2,220÷555= 4 hours

Question 11.
A van is carrying 486 pounds. There are 27 boxes in the van. What is the average weight of each box in the van?

Answer: 18 lbs

Explanation: No.of pounds did the van carry are 486 pounds and no.of boxes in a van are 27. So the average weight of each box is 486÷27= 18

Question 12.
Find 56,794 ÷ 338. Write the quotient twice, once with the remainder as a fraction and once with an r.

Answer: 56,794÷338= 168 10/338= 168 5/169, r=10.

Explanation: 56,794÷338= 168 10/338= 168 5/169 and reminder is 10

Lesson Check – Page No. 10

Question 1.
A caterer’s fee is based on the number of meals she provides. How much is the price per meal if the total fee is $1,088 for 64 meals?

Answer: $17.

Explanation: No.of meals are 64 and the total fee is $1,088. Therefore the price per meal is $1,088÷64= $17.

Question 2.
Amelia needs 24 grains of beads to make a bracelet. She has 320 grams of beads. How many bracelets can she make?

Answer: 13 bracelets.

Explanation: No.of grains of beads Amelia need are 24 and she has 320 grams of beads. So no.of bracelets can Amelia make are 320÷24= 13.33 will round off to 13.

Spiral Review

Question 3.
Hank bought 2.4 pounds of apples. Each pound cost $1.95. How much did Hank spend on the apples?

Answer: $4.68

Explanation: No.of pounds of apples Hank bought is 2.4 pounds and each pound cost is $ 1.95, so total Hank spend on apples is 2.4×$1.95= $4.68

Question 4.
Gavin bought 4 packages of cheese. Each package weighed 1.08 kilograms. How many kilograms of cheese did Gavin buy?

Answer: 4.32kg

Explanation: No.of cheese packages Gavin bought are 4 and each package weight is 1.08 kg. So total weight of cheese is 4×1.08= 4.32 kg

Question 5.
Mr. Thompson received a water bill for $85.98. The bill covered three months of service. He used the same amount of water each month. How much does Mr. Thompson pay for water each month?

Answer: $28.66

Explanation: Water bill received to Mr. Thompson is $85.98 as he covered for 3 months the amount Mr.Thompson paid for each month is $85.98÷3= $28.66

Question 6.
Layla used 0.482 gram of salt in her experiment. Maurice use 0.51 gram of salt. Who used the greater amount of salt?

Answer: Maurice as 0.51 is greater than 0.482.

Explanation: Salt used by Layla is 0.482 grams and salt used by Maurice is 0.51 grams, so the greatest amount of salt used is Maurice as 0.51 is greater than 0.482.

Find the prime factorization – Page No. 13

Question 3.
75

Answer: 5×5×3.

Explanation:     75
15         5
5×3       5
5×5×3

Question 4.
12

Answer: 3×2×2.

Explanation:     12
6×2
3×2×2

Question 3.
65

Answer: 13×5

Explanation:   65
13×5

On Your Own

Write the number whose prime factorization is given.

Question 6.
2 × 2 × 2 × 7

Answer: 56

Question 7.
2 × 2 × 5 × 5

Answer: 100

Question 8.
2 × 2 × 2 × 2 × 3 × 3

Answer: 144

Practice: Copy and Solve Find the prime factorization.

Question 9.
45

Answer: 5×3×3

Explanation:
45
5×9
5×3×3

Question 10.
50

Answer: 5×5×2

Explanation:
50
5×10
5×5×2

Question 11.
32

Answer: 2×2×2×2×2

Explanation:
32
2×16
2×2×8
2×2×2×4
2×2×2×2×2

Question 12.
76

Answer:  2×2×19

Explanation:
76
2×38
2×2×19

Question 13.
108

Answer: 2×2×3×3×3

Explanation:
108
2×54
2×2×27
2×2×3×9
2×2×3×3×3

Question 14.
126

Answer:  2×7×3×3

Explanation:
126
2×63
2×7×9
2×7×3×3

Question 15.
The area of a rectangle is the product of its length and width. A rectangular poster has an area of 260 square inches. The width of the poster is greater than 10 inches and is a prime number. What is the width of the poster?

Answer: Width is 13.

Explanation: Area of a rectangular poster is 260 square inches i.e width×length= 260 sq inches. And the width of the poster is greater than 10 inches and it is a prime number, so the width will be 13 as 13 is a prime number and also 260 is divisible by 13, and length is 13×length= 260 in which length is 260÷13= 20.

Question 16.
Look for Structure Dani says she is thinking of a secret number. As a clue, she says the number is the least whole number that has three different prime factors. What is Dani’s secret number? What is its prime factorization?

Answer: Dani’s secret number is 30 and the prime factorization is 2,3,5.

Explanation: The least three prime numbers are 2,3,5, so the product of three prime numbers is 2×3×5= 30.

Problem Solving + Applications – Page No. 14

Use the table for 17–19. Agent Sanchez must enter a code on a keypad to unlock the door to her office.
Go Math Grade 6 Answer Key Chapter 1 Divide Multi-Digit Numbers img 2

Question 17.
In August, the digits of the code number are the prime factors of 150. What is the code number for the office door in August?

Answer: 2355.

Explanation: Prime Factors of 150 are 2×3×5×5, so code number for office door in August is 2355

Question 18.
In September, the fourth digit of the code number is 2 more than the fourth digit of the code number based on the prime factors of 225. The prime factors of what number were used for the code in September?

Answer: 315.

Explanation: Prime factors of 225 are 3×3×5×5 which is 3355 as the fourth digit of the code number is 2 more than the fourth digit, s0 5+2=7 and by replacing 7 in 3×3×5×5, then 3×3×5×7= 315.

Question 19.
One day in October, Agent Sanchez enters the code 3477. How do you know that this code is incorrect and will not open the door?

Answer: 4 is not a prime number.

Explanation: The code 3477 is incorrect as the code contains only a prime number and 4 is not a prime number.

Question 20.
Use the numbers to complete the factor tree. You may use a number more than once.
2 3 6 9 18

Answer: 36= 2×2×3×3

Explanation:
Go Math Grade 6 Answer Key Chapter 1 Divide Multi-Digit Numbers

Prime Factorization – Page No. 15

Find the prime factorization.

Question 1.
44

Answer: 2×2×11

Explanation:
44
2×22
2×2×11

Question 2.
90

Answer: 2×3×3×5

Explanation:
90
2×45
2×3×15
2×3×3×5

Question 3.
48

Answer:

Explanation:
48
2×24
2×2×12
2×2×2×6
2×2×2×2×3

Question 4.
204

Answer: 2×2×3×17

Explanation:
204
2×102
2×2×51
2×2×3×17

Question 5.
400

Answer: 2×2×2×2×5×5

Explanation:
400
2×200
2×2×100
2×2×2×50
2×2×2×2×25
2×2×2×2×5×5

Question 6.
112

Answer: 2×2×2×2×7

Explanation:
112
2×56
2×2×28
2×2×2×14
2×2×2×2×7

Problem Solving

Question 7.
A computer code is based on the prime factorization of 160. Find the prime factorization of 160.

Answer: 2×2×2×2×2×5

Explanation: Prime factors of 160 is 2×2×2×2×2×5

Question 8.
The combination for a lock is a 3-digit number. The digits are the prime factors of 42 listed from least to greatest. What is the combination for the lock?

Answer: 237.

Explanation: Prime factors of 42 is 2×3×7.

Question 9.
Describe two methods for finding the prime factorization of a number.

Answer:
1. Divison Method.
2. Factor Tree Method.

Explanation:
1. Division Method: In Division method first we will divide the number by smallest prime number, and repeat the process until the quotient became 1.
2. Factor Tree Method: In Factor Tree Method we will write a pair of factors as the branches of the tree and then we will factorize.

Lesson Check – Page No. 16

Question 1.
Maritza remembers her PIN because it is between 1,000 and 1,500 and it is the product of two consecutive prime numbers. What is her PIN?

Answer: Two consecutive prime numbers are 31 and 37 and PIN is 1147.

Explanation: As 31 and 37 are two consecutive prime numbers and their product is 1147 which is between 1,000 to 1,500.

Question 2.
Brent knows that the 6 -digit number he uses to open his computer is the prime factorization of 5005. If each digit of the code increases from left to right, what is his code?

Answer: 111357.

Explanation: Factors of 5005 are 5×7×11×13, as the increases from left to right so the code is 111357

Spiral Review

Question 3.
Piano lessons cost $15. What expressions could be used to find the cost in dollars of 5 lessons?

Answer: $15×5= $75

Explanation: We will use multiplication to find the cost in dollars of 5 lessons.

Question 4.
A jet plane costs an airline $69,500,000. What is the place value of the digit 5 in this number?

Answer: Hundred thousand.

Explanation: The place value of 5 in $69,500,000 is 500,000.

Question 5.
A museum has 13,486 butterflies, 1,856 ants, and 13,859 beetles. What is the order of the insects from least number to greatest number?

Answer: Ants, Butterflies, Beetles.

Explanation: The order of insects from least to greatest are Ants, Butterflies, Beetles.

Question 6.
Juan is reading a 312-page book for school. He reads 12 pages each day. How long will it take him to finish the book?

Answer: 26 days.

Explanation: As Juan reads 12 pages each day and the book has 312 pages, so he will finish in 312÷12= 26 days

Find the LCM – Page No. 19

Question 2.
3, 5

Answer: 15

Explanation:
Multiples of 3: 3,6,9,12,15
Multiples of 5: 5,10,15.
LCM is 15

Question 3.
3, 9

Answer: 9

Explanation:
Multiples of 3: 3,6,9
Multiples of 9: 9
LCM is 9

Question 4.
9, 15

Answer: 135

Explanation:
Multiples of 9: 9,18,27,36,45,54,63,72,81,90,99,108,117,126,135.
Multiples of 15: 15,30,45,60,75,90,105,120,135.
LCM is 135

On Your Own

Find the LCM.

Question 5.
5, 10

Answer: 10

Explanation:
Multiples of 5: 5,10
Multiples of 10: 10
LCM is 10

Question 6.
3, 8

Answer: 24

Explanation:
Multiples of 3: 3,6,92,15,18,21,24
Multiples of 8: 8,16,24
LCM is 24

Question 7.
9, 12

Answer: 108

Explanation:
Multiples of 9: 9,18,27,36,45,54,63,72,81,90,99,108
Multiples of 12: 12,24,36,48,60,72,84,96,108
LCM is 108

Use Reasoning Algebra Write the unknown number for ?.

Question 8.
5, 8      LCM : ?
? =

Answer: 40

Explanation:
Multiples of 5: 5,10,15,20,25,30,35,40
Multiples of 8: 8,16,24,32,40
LCM is 40

Question 9.
?, 6      LCM : 42
? =

Answer: 7

Explanation: 6×7= 42

Question 10.
How can you tell when the LCM of two numbers will equal one of the numbers or equal the product of the numbers?

Answer: If the other number is 1 then the LCM of two numbers will equal one.

Question 11.
Verify the Reasoning of Others Mr. Haigwood is shopping for a school picnic. Veggie burgers come in packages of 15, and buns come in packages of 6. He wants to serve veggie burgers on buns and wants to have no items left over. Mr. Haigwood says that he will have to buy at least 90 of each item, since 6 × 15 = 90. Do you agree with his reasoning? Explain.

Answer: No. We must find the least number of burgers and buns, so we must find LCM of 15 and 6.

Explanation:
Multiples of 15: 15,30
Multiples of 6: 6,12,18,24,30
LCM is 30.

Question 12.
A deli has a special one -day event to celebrate its anniversary. On the day of the event, every eighth customer receives a free drink. Every twelfth customer receives a free sandwich. If 200 customers show up for the event, how many of the customers will receive both a free drink and a free sandwich?

Answer: 24,48,72,96,120,144,168,192 are the customers who get both free drink and free sandwich.

Explanation: To find how many customers have received both a free drink and a sandwich, first we have to find who got a free sandwich and a free drink separately, so
Multiples of 8 are 8,16,24,32,40,48,56,64,72,80,88,96,104,112,120,128, 136,144,152,160,168,176,184,192 and 200 and
Multiples of 12 are 12,24,36,48,60,72,84,96,108,120,132,144,156,168,180 and 192. So common customers are 24,48,72,96,120,144,168,192 are the customers who get both free drink and free sandwich.

Unlock The Problem – Page No. 20

Question 13.
Katie is making hair clips to sell at the craft fair. To make each hair clip, she uses 1 barrette and 1 precut ribbon. The barrettes are sold in packs of 12, and the precut ribbons are sold in packs of 9. How many packs of each item does she need to buy to make the least number of hair clips with no supplies left over?
a. What information are you given?

Answer: 3 packs of barrettes and 4 packs of precut ribbons.

Explanation: As barrettes are sold in packs of 12 and precut ribbons are sold in packs of 9, so we need to find the number of packs of each item does she need to make the least number of hair clips with no supplies left over. So the LCM of 12 and 9.
Multiples of 12 are: 12,24,36
Multiples of 9 are: 9,18,27,36
LCM is 36
So Katie needs 36 barrettes and ribbons to make the least number of hair clips with no supplies left over, and she needs 3 packs of barrettes and 4 packs of precut ribbons.

Question 13.
b. What problem are you being asked to solve?

Answer: To find the number of packs of each item does she need to make the least number of hair clips with no supplies left over

Question 13.
c. Show the steps you use to solve the problem.

Answer:
Multiples of 12 are: 12,24,36
Multiples of 9 are: 9,18,27,36
LCM is 36

Question 13.
d. Complete the sentences.
The least common multiple of 12 and 9 is _____ .
Katie can make _____ hair clips with no supplies left over.
To get 36 barrettes and 36 ribbons, she needs to buy _____ packs of barrettes and _____ packs of precut ribbons.

Answer: 36, 3, 4.

Explanation:
The least common multiple of 12 and 9 is 36.
Katie can make 36 hair clips with no supplies left over.
To get 36 barrettes and 36 ribbons, she needs to buy 3 packs of barrettes and 4 packs of precut ribbons.

Question 14.
Reptile stickers come in sheets of 6 and fish stickers come in sheets of 9. Antonio buys the same number of both types of stickers and he buys at least 100 of each type. What is the least number of sheets of each type he might buy?

Answer: 108

Explanation: As Reptile stickers come in sheets of 6 and fish stickers come in sheets of 9, so we will find the LCM of 6 and 9 to get the least number of sheets,
Multiples of 6 are 6,12,18
Multiples of 9 are 9,18
LCM is 18
As Antonio buys at least 100 of each type, so multiples of 18 are 18,36,54,72,90,108 as 108 is the least number and more than 100 and nearest to 100, so the least number of sheets he might buy= 108

Question 15.
For numbers 15a -15d, choose Yes or No to indicate whether the LCM of the two numbers is 16.
15a. 2,8 O Yes O No
15b. 2,16 O Yes O No
15c. 4,8 O Yes O No
15d. 8,16 O Yes O No

15a. 2,8 O Yes O No

Answer: No

Explanation:
Multiples of 2 are 2,4,6,8
Multiples of 8 are 8
LCM is 8

15b. 2,16 O Yes O No

Answer: Yes

Explanation:
Multiples of 2 are 2,4,6,8,10,12,14,16
Multiples of 16 are 16
LCM is 16

15c. 4,8 O Yes O No

Answer: No

Explanation:
Multiples of 4 are 4,8
Multiples of 8 are 8
LCM is 8

15d. 8,16 O Yes O No

Answer: 16

Explanation:
Multiples of 8 are 8,16
Multiples of 16 are 16
LCM is 16

Least Common Multiple – Page No. 21

Find the LCM.

Question 1.
2, 7

Answer: 14

Explanation:
Multiples of 2 are 2,4,6,8,10,12,14.
Multiples of 7 are 7,14.
LCM is 14.

Question 2.
4, 12

Answer: 12

Explanation:
Multiples of 4 are 4,8,12
Multiples of 12 are 12
LCM is 12

Question 3.
6, 9

Answer: 54

Explanation:
Multiples of 6 are 6,12,18,24,30,36,42,48,54
Multiples of 9 are 9,18,27,36,45,54
LCM is 54

Question 4.
5, 4

Answer: 8

Explanation:
Multiples of 5 are 5,10,15
Multiples of 4 are 8
LCM is 8

Question 5.
5, 8, 4

Answer: 40

Explanation:
Multiples of 5 are 5,10,15,20,25,30,35,40
Multiples of 8 are 8,16,24,32,40
Multiples of 4 are 4,8,12,16,20,24,28,32,36,40
LCM is 40

Question 6.
12, 8, 24

Answer: 24

Explanation:
Multiples of 12 are 12,24
Multiples of 8 are 8,16,24
Multiples of 24 are 24
LCM is 24

Write the unknown number for the?

Question 7.
3, ?        LCM : 21
? =

Answer: 7

Explanation: 3×7= 21

Question 8.
?, 7        LCM : 63
? =

Answer: 9

Explanation: 9×7=63

Question 9.
10, 5     LCM : ?
? =

Answer: 10

Explanation:
Multiples of 10 are 10
Multiples of 5 are 5,10
LCM is 10

Problem Solving

Question 10.
Juanita is making necklaces to give as presents. She plans to put 15 beads on each necklace. Beads are sold in packages of 20. What is the least number of packages she can buy to make necklaces and have no beads left over?

Answer: 3 packages.

Explanation:
Multiples of 15: 15,30,45,60
Multiples of 20: 20,40,60
LCM is 60
As beads are sold in packages of 20 Juanita needs 3 least number of packages to make necklaces with no beads leftover.

Question 11.
Pencils are sold in packages of 10, and erasers are sold in packages of 6. What is the least number of pencils and erasers you can buy so that there is one pencil for each eraser with none left over?

Answer: 30 pencils and 30 erasers are the least numbers we can buy without any leftover.

Explanation:
Multiples of 10: 10,20,30.
Multiples of 6: 6,12,18,24,30.
LCM is 30.
So 30 pencils and 30 erasers are the least numbers we can buy without any leftover.

Question 12.
Explain when you would use each method (finding multiples or prime factorization) for finding the LCM and why.

Answer: When the numbers are smaller we can use finding multiples and when the numbers are larger then we can use prime factorization.

Lesson Check – Page No. 22

Question 1.
Martha is buying hot dogs and buns for the class barbecue. The hot dogs come in packages of 10. The buns come in packages of 12. What is the least number she can buy of each so that she has exactly the same number of hot dogs and buns? How many packages of each should she buy?
_________ packages of hot dogs
_________ packages of buns

Answer: 6 packages of hot dogs and 5 packages of buns she can buy.

Explanation:
Multiples of 10: 10,20,30,40,50,60.
Multiples of 12: 12,24,36,48,60.
LCM is 60.
So 60 is the least number she can buy and 6 packages of hot dogs and 5 packages of buns she can buy.

Question 2.
Kevin makes snack bags that each contain a box of raisins and a granola bar. Each package of raisins contains 9 boxes. The granola bars come 12 to a package. What is the least number he can buy of each so that he has exactly the same number of granola bars and boxes of raisins? How many packages of each should he buy?
_________ packages of raisins
_________ packages of granola bars

Answer: 4 packages of raisins and 3 packages of granola bars he should buy.

Explanation: Kevin’s every package contains 9 raisins boxes and 12 granola bars in each package, so LCM of 9 and 12 are
Multiples of 9: 9,18,21,36
Multiples of 12: 12,24,36
LCM is 36.
So 4 packages of raisins and 3 packages of granola bars he should buy.

Spiral Review

Question 3.
John has 2,456 pennies in his coin collection. He has the same number of pennies in each of 3 boxes. Estimate to the nearest hundred the number of pennies in each box.

Answer: 800 pennies.

Explanation: Let’s round off 2,456 to 2400, as he has the same no. of pennies in each of 3 boxes, so in each box no.of pennies are 2400÷3= 800 pennies.

Question 4.
What is the distance around a triangle that has sides measuring 2 \(\frac{1}{8}\) feet, 3 \(\frac{1}{2}\) feet, and 2 \(\frac{1}{2}\) feet?

Answer: 8 1/8 feet

Explanation: Distance around the triangle is 2 1/8+3 1/2+ 2 1/2= 8 1/8 feet

Question 5.
The 6th grade class collects $1,575. The class wants to give the same amount of money to each of 35 charities. How much will each charity receive?

Answer: $45

Explanation: The 6th-grade class collects $1575 and wants to give the same amount to 35 charities each, so each charity receives $1575÷35= $45.

Question 6.
Jean needs \(\frac{1}{3}\) cup of walnuts for each serving of salad she makes. She has 2 cups of walnuts. How many servings can she make?

Answer: 6.

Explanation: No.of servings made by 1/3 cup of walnuts is 1, so for 1 cup Jean serves 1/(1/3+1/3+1/3)= 3. So for 2 cups, no.of servings can she make are 3×2= 6.

Share and Show – Page No. 25

Question 1.
List the factors of 12 and 20. Circle the GCF.
Factors of 12 : __________
Factors of 20 : __________

Answer: 4

Explanation:
Factors of 12: 1,2,3,4,6,12
Factors of 20: 1,2,4,5,10,20
Common factors are 1,2,4
GCF is 4

Find the GCF.

Question 2.
16, 18

Answer: 2

Explanation:
Factors of 16: 1,2,4,8,16
Factors of 18: 1,2,3,6,9,18
Common factors are 1,2
GCF is 2

Question 3.
25, 40

Answer: 5

Explanation:
Factors of 25: 1,2,5,25
Factors of 40: 1,2,4,5,8,10,20,40
Common factors are 1,2,5
GCF is 5

Question 4.
24, 40

Answer: 8

Explanation:
Factors of 24: 1,2,3,4,6,8,12,24
Factors of 40: 1,2,4,5,8,10,20,40
Common factors are 1,2,4,8
GCF is 8

Question 5.
14, 35

Answer: 7

Explanation:
Factors of 14: 1,2,7,14
Factors of 35: 1,2,5,7,35
Common factors are 1,2,7
GCF is 7

Use the GCF and the Distributive Property to express the sum as a product.

Question 6.
21 + 28

Answer: 7×(3+4)

Explanation:
21+28= (7×3)+(7×4)
=7×(3+4)

Question 7.
15 + 27

Answer: 3×(5+9)

Explanation:
15+27= (3×5)+(3×9)
=3×(5+9)

Question 8.
40 + 15

Answer: 5×(8+3)

Explanation:
40+15= (5×8)+(5×3)
= 5×(8+3)

Question 9.
32 + 20

Answer: 4×(8+5)

Explanation:
32+20= (4×8)+(4×5)
= 4×(8+5)

On Your Own

Find the GCF.

Question 10.
8, 25

Answer: 1

Explanation:
Factors of 8: 1,2,4,8
Factors of 25: 1,5,25
Common factors are 1
GCF is 1

Question 11.
31, 32

Answer: 1

Explanation:
Factors of 31: 1,31
Factors of 32: 1,2,4,8,16,32
Common Factors are 1
GCF is 1

Question 12.
56, 64

Answer: 8

Explanation:
Factors of 56: 1,2,4,7,8,14,28,56
Factors of 64:  1,2,4,8,16,32,64
Common Factors are 1,2,4,8
GCF is 8

Question 13.
150, 275

Answer: 25

Explanation:
Factors of 150: 1,2,3,5,6,10,15,25,30,50,75,150
Factors of 275: 1,5,11,25,55,275
Common Factors are 1,5,25.
GCF is 25.

Use the GCF and the Distributive Property to express the sum as a product.

Question 14.
24 + 30

Answer: 6×(4+5)

Explanation:
24+30= (6×4)+(6×5)
=6×(4+5)

Question 15.
49 + 14

Answer: 7×(7+2)

Explanation:
49+14= (7×7)+(7×2)
=7×(7+2)

Question 16.
63 + 81

Answer: 9×(7+9)

Explanation:
63+81= (9×7)+(9×9)
=9×(7+9)

Question 17.
60 + 12

Answer: 12×(5+1)

Explanation:
60+12= (12×5)+(12×1)
=12×(5+1)

Question 18.
Describe the difference between the LCM and the GCF of two numbers.

Answer: In LCM we will get the Least Common Multiples of two numbers, and in GCF we will get Greatest Common Factor.

Problem Solving + Applications – Page No. 26

Use the table for 19-22. Teachers at the Scott School of Music teach only one instrument in each class. No students take classes for more than one instrument.
Go Math Grade 6 Answer Key Chapter 1 Divide Multi-Digit Numbers img 3

Question 19.
Francisco teaches group lessons to all of the violin and viola students at the Scott School of Music. All of his classes have the same number of students. What is the greatest number of students he can have in each class?

Answer: 6

Explanation: No. of students for Viola instrument is 30 and 36 for Violin,
Factors of 30: 1,2,3,5,6,10,15,30
Factors of 36: 1,2,3,4,6,9,12,18,36
GCF is 6
So the greatest number of students he can have in each class is 6

Question 20.
Amanda teaches all of the bass and viola students. All her classes have the same number of students. Each class has the greatest possible number of students. How many of these classes does she teach?
__________ bass classes
__________ viola classes

Answer: 2 bass classes and 3 viola classes.

Explanation:
Factors of 20: 1,2,4,5,10,20
Factors of 30: 1,2,3,5,6,10,15,30
GCF is 10
As the greatest number of possible students in each class is 10, So Amanda teaches 2 bass classes and 3 viola classes.

Question 21.
Mia teaches jazz classes. She has 9 students in each class, and she teaches all the classes for two of the instruments. Which two instruments does she teach, and how many students are in her classes?

Answer: 63 students.

Explanation:
Factors of 27: 1,3,9,27
Factors of 36: 1,2,3,4,6,9,12,18,36
GCF is 9
As 9 is the GCF of 27 and 36, So Mia teaches Cello and Violin classes for a total of 63 students.

Question 22.
Explain how you could use the GCF and the Distributive Property to express the sum of the number of bass students and the number of violin students as a product.

Answer: GCF is 4
Distributive property is 4×(5+9)

Explanation: The no.of bass students are 20 and no.of violin students are 36,
Factors of 20: 1,2,4,5,10,20
Factors of 36: 1,2,3,4,6,12,18,36
GCF is 4
And the Distributive property is 20+36
= (4×5)+(4×9)
= 4×(5+9)

Question 23.
Go Math Grade 6 Answer Key Chapter 1 Divide Multi-Digit Numbers img 4

Answer: 6

Explanation:
Factors of 6: 1,2,3,6
Factors of 12: 1,2,3,4,6
GCF is 6

Go Math Grade 6 Answer Key Chapter 1 Divide Multi-Digit Numbers

Greatest Common Factor – Page No. 27

List the common factors. Circle the greatest common factor. 

Question 1.
25 and 10

Answer: 5

Explanation:
Factors of 25: 1,5,25.
Factors of 10:  1,2,5,10
Common factors are 1,5
GCF is 5

Question 2.
36 and 90

Answer: 18

Explanation:
Factors of 36: 1,2,3,4,6,9,12,18,36
Factors of 90: 1,2,3,5,6,9,10,15,18,30,45,90
Common Factors are 1,2,3,6,9,18
GCF is 18

Question 3.
45 and 60

Answer: 15

Explanation:
Factors of 45: 1,3,5,9,15,45
Factors of 60: 1,2,3,4,5,6,10,12,15,20,30,60
Common Factors are 1,3,5,15
GCF is 15

Find the GCF.

Question 4.
14, 18

Answer: 2

Explanation:
Factors of 14: 1,2,7,14
Factors of 18: 1,2,3,6,9,18
Common Factors are 1,2
GCF is 2

Question 5.
6, 48

Answer: 6

Explanation:
Factors of 6: 1,2,3,6
Factors of 48: 1,2,3,4,6,8,12,24,48
Common Factors are 1,2,3,6
GCF is 6

Question 6.
16, 100

Answer: 4

Explanation:
Factors of 16: 1,2,4,8,16
Factors of 100: 1,2,4,5,10,20,25,50,100
Common Factors are 1,2,4
GCF is 4

Use the GCF and the Distributive Property to express the sum as a product.

Question 7.
20 + 35

Answer: 5×(4+7)

Explanation:
20+35= (5×4)+(5×7)
=5×(4+7)

Question 8.
18 + 27

Answer: 9×(2+3)

Explanation:
18+27= (9×2)+(9×3)
=9×(2+3)

Question 9.
64 + 40

Answer: 8×(8+5)

Explanation:
64+40= (8×8)+(8×5)
= 8×(8+5)

Problem Solving

Question 10.
Jerome is making prizes for a game at the school fair. He has two bags of different pins, one with 15 square pins and one with 20 round pins. Every prize will have one kind of pin. Each prize will have the same number of pins. What is the greatest number of pins Jerome can put in each prize?

Answer: 5

Explanation:
Factors of 15: 1,3,5,15
Factors of 20: 1,2,4,5,10,20
Common factors are 1,5
So the greatest number of pins Jerome can put in each prize is 5

Question 11.
There are 24 sixth graders and 40 seventh graders. Mr. Chan wants to divide both grades into groups of equal size, with the greatest possible number of students in each group. How many students should be in each group?

Answer: 8.

Explanation:
Factors of 24: 1,2,3,4,6,8,12,24
Factors of 40: 1,2,4,5,8,10,20,40
Common Factors are 1,2,4,8
So the greatest possible number of students are 8
Question 12.
Write a short paragraph to explain how to use prime factorization and the Distributive Property to express the sum of two whole numbers as a product.

Answer:
Prime Factorization is the product of prime numbers

Lesson Check – Page No. 28

Question 1.
There are 15 boys and 10 girls in Miss Li’s class. She wants to group all the students so that each group has the same number of boys and the same number of girls. What is the greatest number of groups she can have?

Answer: 5

Explanation:
Factors of 15: 1,3,5,15
Factors of 10: 1,2,5,10
Common Factors are 1,5
The greatest number of groups she can have is 5.

Question 2.
A pet shop manager wants the same number of birds in each cage. He wants to use as few cages as possible, but can only have one type of bird in each cage. If he has 42 parakeets and 18 canaries, how many birds will he put in each cage?

Answer: 6

Explanation:
Factors of 42: 1,2,3,6,7,14,21,42
Factors of 18: 1,2,3,6,9,18
Common Factors are 1,2,3,6
GCF is 6
So he will put 6 birds in each cage.

Spiral Review

Question 3.
There are 147 people attending a dinner party. If each table can seat 7 people, how many tables are needed for the dinner party?

Answer: 21 tables.

Explanation: Total no.of people attending a dinner party are 147 and 7 people can seat in each table, so 147÷7= 21 tables are needed for a dinner party.

Question 4.
Sammy has 3 pancakes. He cuts each one in half. How many pancake halves are there?

Answer: 6

Explanation: Sammy has 3 pancakes, as he cut each one into half so there are 3×2= 6 pancake halves.

Question 5.
The Cramer Company had a profit of $8,046,890 and the Coyle Company had a profit of $8,700,340 last year. Which company had the greater profit?

Answer: Coyle company

Explanation: Coyle company had a profit of $8,700,340 and Cramer Company had $8,046,890, So $8,700,340-$8,046,890= $653,450 Coyle company have greater profits.

Question 6.
There are 111 guests attending a party. There are 15 servers. Each server has the same number of guests to serve. Jess will serve any extra guests. How many guests will Jess be serving?

Answer: 6.

Explanation:
Total guests attending a party are 111 and no.of servers are 15, as each server has the same number of guests to serve so we will divide total guests by the number of servers 111÷15= 7.4 round off to 6. Therefore, no.of guests, will Jess be serving is 6.

Share and Show – Page No. 31

Question 1.
Toby is packaging 21 baseball cards and 12 football cards to sell at a swap meet. Each packet will have the same number of cards. Each packet will have cards for only one sport. What is the greatest number of cards he can place in each packet? How many packets will there be for each sport?

Answer: 7 packets of baseball cards and 4 packets of football cards and each packet contains 3 cards.

Explanation: The GCF of 21 and 12 are
Factors of 21: 1,3,7,21
Factors of 12: 1,2,3,4,6,12
GCF is 3
By Distributive property 21+12
= (3×7)+(3×4)
= 3×(7+4)
So there will be 7 packets of baseball cards and 4 packets of football cards and each packet contains 3 cards.

Question 2.
What if Toby had decided to keep one baseball card for himself and sell the rest? How would your answers to the previous problem have changed?

Answer: 5 packets of baseball cards and 3 football and each packet contains 4 cards.

Explanation: If Toby had decided to keep one baseball card for himself, so he will have 20 baseball cards and 12 football cards
Factors of 20: 1,2,4,5,10,20
Factors of 12: 1,2,3,4,6,12
GCF is 4
By Distributive property 20+12
= (4×5)+(4×3)
=4×(5+3)
So there will be 5 packets of baseball cards and 3 football and each packet contains 4 cards.

Question 3.
Melissa bought 42 pine seedlings and 30 juniper seedlings to plant in rows on her tree farm. She wants each row to have the same number of seedlings. She wants only one type of seedling in each row. What is the greatest number of seedlings she can plant in each row? How many rows of each type of tree will there be?

Answer: 7 rows of pine seedlings and 5 rows of juniper seedling with 6 seedlings in each row.

Explanation:
Factors of 42: 1,2,3,6,7,14,21,42
Factors of 30: 1,2,3,6,10,15,30
GCF is 6
By Distributive 42+30
=(6×7)+(6×5)
=6×(7+5)
So there will be 7 rows of pine seedlings and 5 rows of juniper seedling with 6 seedlings in each row.

On Your Own – Page No. 32

Question 4.
Make Sense of Problems A drum and bugle marching band has 45 members who play bugles and 27 members who play drums. When they march, each row has the same number of players. Each row has only bugle players or only drummers. What is the greatest number of players there can be in each row? How many rows of each type of player can there be?

Answer: 9 people in each row, And there will be 5 rows of bugle players and 3 rows of drummers.

Explanation:
Factors of 45: 1,3,5,9,15,45
Factors of 27: 1,3,9,27
GCF is 9
So there will be 9 people in each row and by the distributive law 45+27
= (9×5)+(9×3)
= 9×(5+3)
And there will be 5 rows of bugle players and 3 rows of drummers.

Question 5.
The “color guard” of a drum and bugle band consists of members who march with flags, hoops, and other props. How would your answers to Exercise 4 change if there were 21 color guard members marching along with the bugle players and drummers?

Answer: 15 rows of bugle players, 9 rows of drummers, and 7 rows color guard members with 3 marchers in each row.

Explanation:
Factors of 21: 1,3,7,21
Factors of 45: 1,3,5,9,15,45
Factors of 27: 1,3,9,27
GCF is 3
So there would be 15 rows of bugle players, 9 rows of drummers, and 7 rows color guard members with 3 marchers in each row.

Question 6.
If you continue the pattern below so that you write all of the numbers in the pattern less than 500, how many even numbers will you write?
4, 9, 14, 19, 24, 29…

Answer: 50

Explanation: You can write 50 numbers.

Question 7.
Mr. Yaw’s bookcase holds 20 nonfiction books and 15 fiction books. Each shelf holds the same number of books and contains only one type of book. How many books will be on each shelf if each shelf has the greatest possible number of books? Show your work.

Answer: 5

Explanation:
Factors of 15: 1,3,5,15
Factors of 20: 1,2,4,5,10,20.
GCF is 5
5 books will be on each self.

Problem Solving Apply the Greatest Common Factor – Page No. 33

Read the problem and solve.

Question 1.
Ashley is bagging 32 pumpkin muffins and 28 banana muffins for some friends. Each bag will hold only one type of muffin. Each bag will hold the same number of muffins. What is the greatest number of muffins she can put in each bag? How many bags of each type of muffin will there be?

Answer: 8 pumpkin muffins and 7 banana muffins with 4 greatest number of muffins in each bag.

Explanation:
Factors of 32: 1,2,4,8,16,32
Factors of 28: 1,2,4,7,14,28
GCF is 4
By distributive property 32+28
= (4×8)+(4×7)
=4×(8+7)
So there will be 8 pumpkin muffins and 7 banana muffins with 4 greatest number of muffins in each bag.

Question 2.
Patricia is separating 16 soccer cards and 22 baseball cards into groups. Each group will have the same number of cards, and each group will have only one kind of sports card. What is the greatest number of cards she can put in each group? How many groups of each type will there be?

Answer: Patricia has 8 soccer cards and 11 baseball cards and 2 groups each.

Explanation:
Factors of 16: 1,2,4,8,16
Factors of 22: 1,2,11,22
GCF is 2
By distributive property 16+22
= (2×8)+(2×11)
=2×(8+11)
Patricia has 8 soccer cards and 11 baseball cards and 2 groups each.

Question 3.
Bryan is setting chairs in rows for a graduation ceremony. He has 50 black chairs and 60 white chairs. Each row will have the same number of chairs, and each row will have the same color chair. What is the greatest number of chairs that he can fit in each row? How many rows of each color chair will there be?

Answer: 10 chairs per row and 5 black chairs and 6 white chairs.

Explanation:
By distributive law 50+60
= (10×5)+(10×60)
= 10×(5+6)
So there will 10 chairs per row and 5 black chairs and 6 white chairs.

Question 4.
A store clerk is bagging spices. He has 18 teaspoons of cinnamon and 30 teaspoons of nutmeg. Each bag needs to contain the same number of teaspoons, and each bag can contain only one spice. What is the maximum number of teaspoons of spice the clerk can put in each bag? How many bags of each spice will there be?

Answer: 6 no. of teaspoons of spices and 3 teaspoons of cinnamon 5 teaspoons of nutmeg.

Explanation:
By distributive property (18+30)
= (6×3)+(6×5)
= 6×(3+5)
So there will be 6 no. of teaspoons of spices and 3 teaspoons of cinnamon 5 teaspoons of nutmeg.

Question 5.
Write a problem in which you need to put as many of two different types of objects as possible into equal groups. Then use the GCF, Distributive Property, and a diagram to solve your problem

Answer: Jack has a bag full of 20 red apples and 32 green apples. Each bag needs to contain same number of apples and each bag can contain only one type of apple. What is the maximum number of apples can Jack put in each bag? How many bags of each apple will be there?

Explanation: By distributive property (20+32)
= (4×5)+(4×8)
= 4×(5+8)
So there will be 4 bags and in that 5 red apples and 8 green apples.

Lesson Check – Page No. 34

Question 1.
Fred has 36 strawberries and 42 blueberries. He wants to use them to garnish desserts so that each dessert has the same number of berries, but only one type of berry. He wants as much fruit as possible on each dessert. How many berries will he put on each dessert? How many desserts with each type of fruit will he have?

Answer: 6 berries on each dessert and 6 strawberries and 7 blueberries in each type of fruit.

Explanation:
By distributive property 36+42
= (6×6)+(6×7)
= 6×(6+7)
So he put 6 berries on each dessert and 6 strawberries and 7 blueberries in each type of fruit.

Question 2.
Dolores is arranging coffee mugs on shelves in her shop. She wants each shelf to have the same number of mugs. She only wants one color of mug on each shelf. If she has 49 blue mugs and 56 red mugs, what is the greatest number she can put on each shelf? How many shelves does she need for each color?
__________ shelves for blue mugs
__________ shelves for red mugs

Answer: 7 blue mugs and 8 red mugs.

Explanation:
By distributive property 49+56
= (7×7)+(7×8)
= 7×(7+8)
So the greatest number she can put on each shelf is 7, 7 blue mugs and 8 red mugs.

Spiral Review

Question 3.
A rectangle is 3 \(\frac{1}{3}\) feet long and 2 \(\frac{1}{3}\) feet wide. What is the distance around the rectangle?
_____ \(\frac{□}{□}\)

Answer: 11 1/3 feet

Explanation: Distance of a rectangle= 2(L+W)
= 2(3 1/3+ 2 1/3)
= 2(10/3+7/3)
= 2(17/3)
= 34/3
= 11 1/3 feet.

Question 4.
Lowell bought 4 \(\frac{1}{4}\) pounds of apples and 3 \(\frac{3}{5}\) pounds of oranges. How many pounds of fruit did Lowell buy?
_____ \(\frac{□}{□}\)

Answer: 7 17/20 pounds

Explanation: Lowell bought 4 1/4 pounds of apples and 3 3/5 pounds of oranges, so total pounds of fruits Lowell bought is 4 1/4+ 3 3/5=
= 17/4+ 18/5
= 157/20
= 7 17/20 pounds

Question 5.
How much heavier is a 9 \(\frac{1}{8}\) pound box than a 2 \(\frac{5}{6}\) pound box?
_____ \(\frac{□}{□}\)

Answer: 6 7/4 much heavier.

Explanation: 9 1/8 – 2 5/6
= 73/8 – 17/6
= 151/24
= 6 7/4

Question 6.
The combination of Clay’s locker is the prime factors of 102 in order from least to greatest. What is the combination of Clay’s locker?

Answer: 2317.

Explanation:
Prime Factors of 102 are 2,3,17, so the combination of Clay’s locker is 2317

Vocabulary – Page No. 35

Choose the best term from the box to complete the sentence.
Go Math Grade 6 Answer Key Chapter 1 Divide Multi-Digit Numbers img 5

Question 1.
The _____ of two numbers is greater than or equal to the numbers.

Answer: LCM

Question 2.
The _____ of two numbers is less than or equal to the numbers.

Answer: Greatest Common

Concepts and Skills

Estimate. Then find the quotient. Write the remainder, if any, with an r.

Question 3.
2,800 ÷ 25

Answer: Quotient is 112 and remainder is 0

Explanation:
Go Math Grade 6 Answer Key Chapter 1 Divide Multi-Digit Numbers

Question 4.
19,129 ÷ 37

Answer: Quotient is 517 and remainder is 0

Explanation:
Go Math Grade 6 Answer Key Chapter 1 Divide Multi-Digit Numbers

Question 5.
32,111 ÷ 181

Answer: Quotient is 177 and the remainder is 74

Explanation:
Go Math Grade 6 Answer Key Chapter 1 Divide Multi-Digit Numbers

Find the prime factorization.

Question 6.
44

Answer: 2×2×11

Explanation:
44= 4×11
2×2×11

Question 7.
36

Answer: 2×2×3×3

Explanation:
36= 2×18
=2×2×9
=2×2×3×3

Question 8.
90

Answer: 3×3×5×2

Explanation:
90= 9×10
=3×3×10
=3×3×5×2

Find the LCM.

Question 9.
8, 10

Answer: 40

Explanation:
Multiples of 8: 8,16,24,32,40
Multiples of 10: 10,20,30,40
LCM is 40

Question 10.
4, 14

Answer: 28

Explanation:
Multiples of 4:  4,8,12,16,20,24,28
Multiples of 14: 14,28
LCM is 28

Question 11.
6, 9

Answer: 18

Explanation:
Multiples of 6: 6,12,18
Multiples of 9: 9,18
LCM is 18

Find the GCF.

Question 12.
16, 20

Answer: 4

Explanation:
Factors of 16: 1,2,4,8,16
Factors of 20: 1,2,4,5,10,20
Common Factors are 1,2,4
GCF is 4

Question 13.
8, 52

Answer: 4

Explanation:
Factors of 8: 1,2,4,8
Factors of 52: 1,2,4,13,26,52
Common Factors are 1,2,4
GCF is 4

Question 14.
36, 54

Answer: 18

Explanation:
Factors of 36: 1,2,3,4,6,9,12,18,36
Factors of 54:  1,2,3,6,9,18,27,54
Common Factors are 1,2,3,6,9,18
GCF is 18

Page No. 36

Question 15.
A zookeeper divided 2,440 pounds of food equally among 8 elephants. How many pounds of food did each elephant receive?

Answer: 305 Pounds.

Explanation: Zookeeper divides 2,440 pounds of food equally among 8 elephants, so no. of pounds is
2,440÷8= 305 pounds.

Question 16.
DVD cases are sold in packages of 20. Padded mailing envelopes are sold in packets of 12. What is the least number of cases and envelopes you could buy so that there is one case for each envelope with none left over?

Answer: 60

Explanation:
Multiples of 20: 20,40,60
Multiples of 12: 12,24,36,48,60
LCM is 60
So the Least number of cases and envelopes without any leftover is 60.

Question 17.
Max bought two deli sandwich rolls measuring 18 inches and 30 inches. He wants them to be cut into equal sections that are as long as possible. Into what lengths should the rolls be cut? How many sections will there be in all?

Answer: 6 inches and 8 sections.

Explanation:
By distributive property 18+30
= (6×3)+(6×5)
= 6(3+5)
So Length of the rolls should cut at 6 inches and sections are (3+5)= 8 sections.

Question 18.
Susan is buying supplies for a party. If spoons only come in bags of 8 and forks only come in bags of 6, what is the least number of spoons and the least number of forks she can buy so that she has the same number of each?

Answer: So least no. of forks and spoons are 24.

Explanation:
Multiples of 8: 8,16,24
Multiples of 6: 6,2,18,24
LCM is 24
So least no. of forks and spoons are 24.

Question 19.
Tina is placing 30 roses and 42 tulips in vases for table decorations in her restaurant. Each vase will hold the same number of flowers. Each vase will have only one type of flower. What is the greatest number of flowers she can place in each vase? If Tina has 24 tables in her restaurant, how many flowers can she place in each vase?

Answer: Maximum flowers in a vase is 3.

Explanation: Tina is placing 30 roses and 42 tulips, so total flowers are 30+42= 72 flowers. The total number of tables are 24, as each vase hold same no. of flowers, Let the no. of flowers in each vase be X, so total no.of flowers to be decorate 24X,
24X = 72
X= 3.
So maximum flowers in a vase is 3.

Share and Show – Page No. 39

Question 1.
Find 3.42 − 1.9.

Answer: 1.52

Explanation: 3.42 − 1.9= 1.52.

Estimate. Then find the sum or difference.

Question 2.
2.3 + 5.68 + 21.047

Answer: 29.027

Explanation: 2.3 + 5.68 + 21.047= 29.027

Question 3.
33.25 − 21.463

Answer: 11.787

Explanation: 33.25 − 21.463= 11.787

Question 4.
Evaluate (8.54 + 3.46) − 6.749.

Answer: 5.251

Explanation:
(8.54 + 3.46) − 6.749= (12)-6.749
= 5.251

On Your Own

Estimate. Then find the sum or difference.

Question 5.
57.08 + 34.71

Answer: 91.79

Explanation:
57.08 + 34.71= 91.79

Question 6.
20.11 − 13.27

Answer: 33.38

Explanation:
20.11−13.27= 33.38

Question 7.
62 − 9.817

Answer: 52.183

Explanation:
62 − 9.817= 52.183

Question 8.
35.1 + 4.89

Answer: 39.99

Explanation:
35.1 + 4.89= 39.99

Practice: Copy and Solve Evaluate using the order of operations.

Question 9.
8.01 − (2.2 + 4.67)

Answer: 1.14

Explanation:
8.01 − (2.2 + 4.67)
= 8.01-(6.87)
= 1.14

Question 10.
54 + (9.2 − 1.413)

Answer: 61.787

Explanation: 54 + (9.2 − 1.413)
= 54+(7.787)
=61.787

Question 11.
21.3 − (19.1 − 3.22)

Answer: 5.42

Explanation: 21.3 − (19.1 − 3.22)
= 21.3-(15.88)
=5.42

Question 12.
Make Arguments A student evaluated 19.1 + (4.32 + 6.9) and got 69.2. How can you use estimation to convince the student that this answer is not reasonable?

Answer: The answer is not reasonable, because 19.1+4.32+6.9= 30.32

Explanation: 19.1 + (4.32 + 6.9)
= 19.1+(11.22)
= 30.32

Question 13.
Lynn paid $4.75 for cereal, $8.96 for chicken, and $3.25 for soup. Show how she can use properties and compatible numbers to evaluate (4.75 + 8.96) + 3.25 to find the total cost.

Answer: 16.96

Explanation: Total cost is (4.75 + 8.96) + 3.25
= (13.71)+3.25
= 16.96

Page No. 40

Question 14.
For numbers 14a–14d, select True or False for each equation.
14a. 3.76 + 2.7 = 6.46 True False
14b. 4.14 + 1.8 = 4.32 True False
14c. 2.01 – 1.33 = 0.68 True False
14d. 51 – 49.2 = 1.8 True False

14a. 3.76 + 2.7 = 6.46

Answer: True

Explanation: 3.76 + 2.7 = 6.46

14b. 4.14 + 1.8 = 4.32

Answer: False

Explanation: 4.14 + 1.8 = 5.94

14c. 2.01 – 1.33 = 0.68

Answer: True

Explanation: 2.01 – 1.33 = 0.68

14d. 51 – 49.2 = 1.8

Answer: True

Explanation: 51 – 49.2 = 1.8

Comparing Eggs

Different types of birds lay eggs of different sizes. Small birds lay eggs that are smaller than those that are laid by larger birds. The table shows the average lengths and widths of five different birds’ eggs.
Go Math Grade 6 Answer Key Chapter 1 Divide Multi-Digit Numbers img 6

Use the table for 15–17.

Question 15.
What is the difference in average length between the longest egg and the shortest egg?

Answer: 0.073

Explanation: The length of the longest egg is 0.086 and the shortest egg is 0.013, so the difference is
0.086-0.013= 0.073

Question 16.
Which egg has a width that is eight thousandths of a meter shorter than its length?

Answer: Turtledove

Explanation: The length of the turtledove egg is 0.031 and the width is 0.023, so 0.031-0.023= 0.08m shorter than length.

Question 17.
How many robin eggs, laid end to end, would be about equal in length to two raven eggs? Justify your answer

Answer: 5 robin eggs should be laid.

Explanation: The length of Two raven eggs is 0.049+0.049=0.098, so 5 robin eggs should be laid.

Add and Subtract Decimals – Page No. 41

Estimate. Then find the sum or difference.

Question 1.
43.53 + 27.67

Answer: 71.2

Explanation: 43.53 + 27.67=71.2

Question 2.
17 + 3.6 + 4.049

Answer: 24.649

Explanation: 17 + 3.6 + 4.049
=17+7.649
=24.649

Question 3.
3.49 − 2.75

Answer: 0.74

Explanation:
3.49-2.75= 0.74

Question 4.
5.07 − 2.148

Answer: 2.922

Explanation:
5.07-2.148= 2.922

Question 5.
3.92 + 16 + 0.085

Answer: 20.005

Explanation: 3.92 + 16 + 0.085
= 3.92+16.085
= 20.005

Question 6.
41.98 + 13.5 + 27.338

Answer: 82.818

Explanation: 41.98 + 13.5 + 27.338
= 41.98+ 40.838

Evaluate using the order of operations.

Question 7.
8.4 + (13.1 − 0.6)

Answer: 20.9

Explanation: 8.4 + (13.1 − 0.6)
= 8.4+(12.5)
= 20.9

Question 8.
34.7 − (12.07 + 4.9)

Answer: 17.73

Explanation: 34.7 − (12.07 + 4.9)
= 34.7-(16.97)
= 17.73

Question 9.
(32.45 − 4.8) − 2.06

Answer: 25.59

Explanation: (32.45 − 4.8) − 2.06
= 27.65- 2.06
= 25.59

Problem Solving

Question 10.
The average annual rainfall in Clearview is 38 inches. This year, 29.777 inches fell. How much less rain fell this year than falls in an average year?

Answer: 8.23

Explanation: Average annual rainfall in last year is 38 inches and this year is 29.777 inches, so 38-29.77= 8.23 inches less rainfall

Question 11.
At the theater, the Worth family spent $18.00 on adult tickets, $16.50 on children’s tickets, and $11.75 on refreshments. How much did they spend in all?

Answer: $46.25

Explanation: As the family spent $18.00 on adult tickets, $16.50 on children’s tickets, and $11.75 on refreshments,
So the total spent by the family is $18.00+$16.50+$11.75= $46.25

Question 12.
Write a word problem that involves adding or subtracting decimals. Include the solution.

Answer: Mark and Jack went to the park and cost of ticket is $6.50. Mark has $20. How much remaining did mark has left?

Explanation: As MArk and jack went to the park where ticket price is $6.50, so for both it will be
$6.50+$6.50= $13.00. As Mark has $20, remaining amount left with Mark is $20-$13= $7

Lesson Check – Page No. 42

Question 1.
Alden fills his backpack with 0.45 kg of apples, 0.18 kg of cheese, and a water bottle that weighs 1.4 kg. How heavy are the contents of his backpack?

Answer: 2.03kg

Explanation: The total weight of a backpack is 0.45+0.18+1.4= 2.03kg

Question 2.
Gabby plans to hike 6.3 kilometers to see a waterfall. She stops to rest after hiking 4.75 kilometers. How far does she have left to hike?

Answer: 1.55kms

Explanation: Gobby hikes 6.3kms and stops at 4.75kms, so she left at 6.3-4.75= 1.55kms
Spiral Review

Question 3.
A 6-car monorail train can carry 78 people. If one train makes 99 trips during the day, what is the greatest number of people the train can carry in one day?

Answer: 7,722.

Explanation: The greatest number of people the train can carry in one day is 78×99= 7,722.

Question 4.
An airport parking lot has 2,800 spaces. If each row has 25 spaces, how many rows are there?

Answer: 112 rows

Explanation: As parking lot has 2,800 spaces and each row has 25 spaces, no. of rows is 2800÷25= 112 rows

Question 5.
Evan brought 6 batteries that cost $10 each and 6 batteries that cost $4 each. The total cost was the same as he would have spent buying 6 batteries that cost $14 each. So, 6 × $14 = (6 × 10) + (6 × 4). What property does the equation illustrate?

Answer: Distributive property

Explanation: By distributive property (a×b)+(a×c)= a×(b+c), here a= 6, b=10, c=4.

Question 6.
Cups come in packages of 12 and lids come in packages of 15. What is the least number of cups and lids that Corrine can buy if she wants to have the same number of cups and lids?

Answer: 60 cups and 60 lids.

Explanation:
Multiples of 12: 12,24,36,48,60
Multiples of 15: 15,30,45,60
LCM is 60
So the least number of cups and lids she can buy is 60 cups and 60 lids.

Share and Show – Page No. 45

Estimate. Then find the product.

Question 1.
12.42 × 28.6

Answer: 355.212

Explanation: 12.42 × 28.6
= 355.212

Question 2.
32.5 × 7.4

Answer: 240.5

Explanation: 32.5 × 7.4
=240.5

Attend to Precision Algebra Evaluate using the order of operations.

Question 3.
0.24 × (7.3 + 2.1)

Answer: 2.256

Explanation: 0.24 × (7.3 + 2.1)
= 0.24×9.4
= 2.256

Question 4.
0.075 × (9.2 − 0.8)

Answer: 0.63

Explanation: 0.075 × (9.2 − 0.8)
= 0.075×(8.4)
= 0.63

Question 5.
2.83 + (0.3 × 2.16)

Answer: 3.478

Explanation: 2.83 + (0.3 × 2.16)
= 2.83+0.648
= 3.478

On Your Own

Estimate. Then find the product.

Question 6.
29.14 × 5.2

Answer: 151.528

Explanation: 29.14 × 5.2

= 151.528

Question 7.
6.95 × 12

Answer: 83.4

Explanation: 6.95 × 12
= 83.4

Question 8.
0.055 × 1.82

Answer: 0.1001

Explanation: 0.055 × 1.82
= 0.1001

Attend to Precision Algebra Evaluate using the order of operations.

Question 9.
(3.62 × 2.1) − 0.749

Answer: 6.853

Explanation: (3.62 × 2.1) − 0.749
= 7.602- 0.749
= 6.853

Question 10.
5.8 − (0.25 × 1.5)

Answer: 5.425

Explanation: 5.8 − (0.25 × 1.5)
= 5.8- (0.375)
= 5.425

Question 11.
(0.83 + 1.27) × 6.4

Answer: 13.44

Explanation: (0.83 + 1.27) × 6.4
= (2.1)×6.4
= 13.44

Question 12.
Jamal is buying ingredients to make a large batch of granola to sell at a school fair. He buys 3.2 pounds of walnuts for $4.40 per pound and 2.4 pounds of cashews for $6.25 per pound. How much change will he receive if he pays with two $20 bills?

Answer: $40-$29.08= $10.92.

Explanation: Jamal bought 3.2 pounds of walnuts for $4.40 per pound, so for 3.2 pounds it will be 3.2×4.40= 14.08,
and 2.4 pounds of cashew for $6.25 per pound, so for 2.4 pounds it will be 2.4×6.25= 15. Total Jamal spend is 14.08+15= 29.08. As he have two $20 so he will receive $40-$29.08= $10.92.

Unlock the Problem – Page No. 46

The table shows some currency exchange rates for 2009.
Go Math Grade 6 Answer Key Chapter 1 Divide Multi-Digit Numbers img 7

Question 13.
When Cameron went to Canada in 2007, he exchanged 40 U.S. dollars for 46.52 Canadian dollars. If Cameron exchanged 40 U.S. dollars in 2009, did he receive more or less than he received in 2007? How much more or less?
a. What do you need to find?

Answer: We need how much or less 40 US dollars are worth in Canadian dollars in 2009 compared to 2007.

Question 13.
b. How will you use the table to solve the problem?

Answer: The table provides exchange rates for 2009, will multiply to find the value of 40 US dollars in Canadian dollars in 2009.

Question 13.
c. Complete the sentences.
40 U.S. dollars were worth _____ Canadian dollars in 2009.
So, Cameron would receive _____ Canadian dollars in 2009.

Answer: 42.08 Canadian dollars in 2009
4.44 Canadian dollars in 2009

Explanation: In 2009 1 US dollar is 1.052, so 40 US dollars is 40×1.052= 42.08 and in 2007 Cameron received 46.52, so in 2009 Cameron would receive 46.52-42.08= 4.44 Canadian dollars in 2009.

Question 14.
At a convenience store, the Jensen family puts 12.4 gallons of gasoline in their van at a cost of $3.80 per gallon. They also buy 4 water bottles for $1.99 each, and 2 snacks for $1.55 each. Complete the table to find the cost for each item.
Go Math Grade 6 Answer Key Chapter 1 Divide Multi-Digit Numbers img 8
Mrs. Jensen says the total cost for everything before tax is $56.66. Do you agree with her? Explain why or why not.

Answer: No, the answer is not reasonable.

Explanation: As the total cost is 58.18
12.4×3.80= 47.12
4×1.99= 7.96
2×1.55= 3.1
So 47.12+7.96+3.1= $58.18

Multiply Decimals – Page No. 47

Estimate. Then find the product.

Question 1.
5.69 × 7.8

Answer: 44.382

Explanation: 5.69 × 7.8
= 44.382

Question 2.
3.92 × 0.051

Answer: 0.19992

Explanation: 3.92 × 0.051
= 0.19992

Question 3.
2.365 × 12.4

Answer: 29.326

Explanation: 2.365 × 12.4
= 29.326

Question 4.
305.08 × 1.5

Answer: 457.62

Explanation: 305.08 × 1.5
= 457.62

Evaluate the expression using the order of operations.

Question 5.
(61.8 × 1.7) + 9.5

Answer: 114.56

Explanation: (61.8 × 1.7) + 9.5
= 105.06+ 9.5
= 114.56

Question 6.
205 − (35.80 × 5.6)

Answer: 4.52

Explanation: 205 − (35.80 × 5.6)
= 205- 200.48
= 4.52

Question 7.
1.9 × (10.6 − 2.17)

Answer: 16.017

Explanation: 1.9 × (10.6 − 2.17)
= 1.9×( 8.43)
= 16.017

Problem Solving

Question 8.
Blaine exchanges $100 for yen before going to Japan. If each U.S. dollar is worth 88.353 yen, how many yen should Blaine receive?

Answer: 8835.3 yen

Explanation: As 1 US dollar is 88.353 yen, so when Blaine exchanges $100 to yen it will be $100×88.353=8835.3 yen

Question 9.
A camera costs 115 Canadian dollars. If each Canadian dollar is worth 0.952 U.S. dollars, how much will the camera cost in U.S. dollars?

Answer: 109.48.

Explanation: As 1 Canadian dollar is 0.952 US dollars, so camers cost is 115×0.952= 109.48.

Question 10.
Explain how to mentally multiply a decimal number by 100.

Answer: Move the decimal point two places right.

Lesson Check – Page No. 48

Question 1.
A gallon of water at room temperature weighs about 8.35 pounds. Lena puts 4.5 gallons in a bucket. How much does the water weigh?

Answer: 37.575

Explanation: As 1 gallon= 8.35 pounds, Lena put 4.5 gallons in a bucket. So water weight is 4.5×8.35= 37.575

Question 2.
Shawn’s rectangular mobile home is 7.2 meters wide and 19.5 meters long. What is its area?

Answer: 140.4

Explanation: Area= Length×wide, so 7.2×19.5= 140.4

Spiral Review

Question 3.
Last week, a store sold laptops worth a total of $3,885. Each laptop cost $555. How many laptops did the store sell last week?

Answer: 7 Laptops.

Explanation: Total Laptops sold is $3885 and each laptop cost is $555, so 3885÷555= 7 laptops were sold by the store.

Question 4.
Kyle drives his truck 429 miles on 33 gallons of gas. How many miles can Kyle drive on 1 gallon of gas?

Answer: 13 miles.

Explanation: As Kyle drives 429 miles on 33 gallons gas, so 429÷33= 13 miles he can drive on 1 gallon of gas.

Question 5.
Seven busloads each carrying 35 students arrived at the game, joining 23 students who were already there. Evaluate the expression 23 + (7 × 35) to find the total number of students at the game.

Answer: 268 students.

Explanation: 23+(7×35)
=23+(245)
=268.
Total students are 268.

Question 6.
A store is giving away a $10 coupon to every 7th person to enter the store and a $25 coupon to every 18th person to enter the store. Which person will be the first to get both coupons?

Answer: 126th person will get both coupons.

Explanation: LCM of 7 and 18 is 18×7= 126. So 126th person will get both coupons.

Estimate. Then find the quotient – Page No. 51

Question 2.
7)\(\overline { $17.15 } \)

Answer: 2.45

Explanation: $17.15÷7= 2.45

Question 3.
4)\(\overline { 1.068 } \)

Answer: 0.267

Explanation: 1.068÷4= 0.267

Question 4.
12)\(\overline { 60.84 } \)

Answer: 5.07

Explanation: 60.84÷12= 5.07

Question 5.
18.042 ÷ 6

Answer: 3.007

Explanation: 18.042÷6= 3.007

On Your Own

Estimate. Then find the quotient.

Question 6.
$21.24 ÷ 6

Answer: 3.54

Explanation: $21.24 ÷ 6= 3.54

Question 7.
28.63 ÷ 7

Answer: 4.09

Explanation: 28.63 ÷ 7= 4.09

Question 8.
1.505 ÷ 35

Answer: 0.043

Explanation: 1.505 ÷ 35= 0.043

Question 9.
0.108 ÷ 18

Answer: 0.006

Explanation: 0.108 ÷ 18= 0.006

Attend to Precision Algebra Evaluate using the order of operations.

Question 10.
(3.11 + 4.0) ÷ 9

Answer: 0.79

Explanation: (3.11 + 4.0) ÷ 9
= (7.11)+9
= 0.79

Question 11.
(6.18 − 1.32) ÷ 3

Answer: 1.62

Explanation: (6.18 − 1.32) ÷ 3
= (4.86)÷3
= 1.62

Question 12.
(18 − 5.76) ÷ 6

Answer: 2.04

Explanation: (18 − 5.76) ÷ 6
= (12.24)÷6
= 2.04

Question 13.
Use Appropriate Tools Find the length of a dollar bill to the nearest tenth of a centimeter. Then show how to use division to find the length of the bill when it is folded in half along the portrait of George Washington

Answer: 3.07 inches or 7.8 centimeter.

Explanation: As the length of a dollar bill to the nearest tenth of a centimeter is 15.6 cm, and length of the bill when it is folded in half along the portrait of George Washington is 3.07 inches or 7.8 centimeter.

Question 14.
Emilio bought 5.65 pounds of green grapes and 3.07 pounds of red grapes. He divided the grapes equally into 16 bags. If each bag of grapes has the same weight, how much does each bag weigh?

Answer: 0.545 pounds.

Explanation: Total weight of grapes is 5.65+3.07= 8.72 pounds, so each bag weight is 8.72÷16= 0.545 pounds.

Problem Solving + Applications – Page No. 52

Pose a Problem

Question 15.
This table shows the average height in inches for girls and boys at ages 8, 10, 12, and 14 years. To find the average growth per year for girls from age 8 to age 12, Emma knew she had to find the amount of growth between age 8 and age 12, then divide that number by the number of years between age 8 and age 12.
Go Math Grade 6 Answer Key Chapter 1 Divide Multi-Digit Numbers img 9
Emma used this expression: (60.50−50.75)÷4
She evaluated the expression using the order of operations.
Write the expression. (60.50−50.75)÷4
Perform operations in parentheses. 9.75÷4
Divide. 2.4375
So, the average annual growth for girls ages 8 to 12 is 2.4375 inches. Write a new problem using the information in the table for the average height for boys. Use division in your problem.

Answer: Find the average growth per year for girls 8 to 14.

Explanation: As (62.50-50.75)÷6
= (11.75)÷6
= 1.96
So the average annual growth for girls age 8 to age 14 is 1.96 inches.

Question 16.
The table shows the number of books each of three friends bought and the cost. On average, which friend spent the most per book? Use numbers and words to explain your answer
Go Math Grade 6 Answer Key Chapter 1 Divide Multi-Digit Numbers img 10

Answer: Nabil spent the most per book.

Explanation:
Joyce purchased 1 book which costs $10.95
Nabil purchased 2 books which costs $40.50, so 1 book cost is 40.50÷2= $20.26
Kenneth purchased 3 books for $51.15 , so 1 book cost is 51.15÷3= $17.05
So, Nabil spent the most per book.

Divide Decimals by Whole Numbers – Page No. 53

Estimate. Then find the quotient.

Question 1.
1.284 ÷ 12

Answer: 0.107

Explanation: 1.284÷12= 0.107

Question 2.
9)\(\overline { 2.43 } \)

Answer: 0.27

Explanation: 2.43÷9 = 0.27

Question 3.
25.65 ÷ 15

Answer: 1.71

Explanation: 25.65÷15= 1.71

Question 4.
12)\(\overline { 2.436 } \)

Answer: 0.203

Explanation: 2.436÷12 = 0.203

Evaluate using the order of operations.

Question 5.
(8 − 2.96) ÷ 3

Answer: 1.68

Explanation: (8 − 2.96) ÷ 3
= (5.04)÷3
= 1.68

Question 6.
(7.772 − 2.38) ÷ 8

Answer: 0.674

Explanation: (7.772 − 2.38) ÷ 8
= (5.392)÷8
= 0.674

Question 7.
(53.2 + 35.7) ÷ 7

Answer: 12.7

Explanation: (53.2 + 35.7) ÷ 7
= (88.9)÷7
= 12.7

Problem Solving

Question 8.
Jake earned $10.44 interest on his savings account for an 18-month period. What was the average amount of interest Jake earned on his savings account per month?

Answer: $0.58.

Explanation: Jake earned $10.44 interest on his savings account for an 18 month period, so average amount interest is 10.44÷18= $0.58.

Question 9.
Gloria worked for 6 hours a day for 2 days at the bank and earned $114.24. How much did she earn per hour?

Answer: $9.52.

Explanation: As gloria worked for 6 hours for 2 days, so total hours is 6×2= 12 hours earned $114.24. So per hour she earns $114.24÷12= $9.52.

Question 10.
Explain the importance of correctly placing the decimal point in the quotient of a division problem.

Answer: If you don’t have the decimals in right spot your answer could be differ.

Lesson Check – Page No. 54

Estimate each quotient. Then find the exact quotient for each question.

Question 1.
Ron divided 67.6 fluid ounces of orange juice evenly among 16 glasses. How much did he pour into each glass?

Answer: 4.225 ounces.

Explanation: As there are 16 glasses, he pours into each glass 67.6÷16= 4.225 ounces.

Question 2.
The cost of a $12.95 pizza was shared evenly by 5 friends. How much did each person pay?

Answer: $2.59.

Explanation: The cost of pizza is $12.95 which was shared by 5 friends, so each person pays $12.95÷5= $2.59

Spiral Review

Question 3.
What is the value of the digit 6 in 968,743,220?

Answer: 60 Lakhs.

Explanation: The place value of 6 is 60,00,000.

Question 4.
The Tama, Japan, monorail carries 92,700 riders each day. If the monorail runs 18 hours each day, what is the average number of passengers riding each hour?

Answer: 5150 passengers.

Explanation: No. of riders each day is 92,700 and he runs for 18 hours in each day, so average no. of passengers riding each hour is 92,700÷18= 5150 passengers.

Question 5.
Ray paid $812 to rent music equipment that costs $28 per hour. How many hours did he have the equipment?

Answer: 29 hours.

Explanation: As Ray paid $812 which costs $28 per hour, so no.of hours did he have the equipment is
$812÷$28= 29 hours.

Question 6.
Jan has 35 teaspoons of chocolate cocoa mix and 45 teaspoons of french vanilla cocoa mix. She wants to put the same amount of mix into each jar, and she only wants one flavor of mix in each jar. She wants to fill as many jars as possible. How many jars of french vanilla cocoa mix will Jan fill?

Answer: 9 jars.

Explanation: By distributive property (35+45)
= (5×7)+(5×9)
= 5(7+9)
So she will fill 9 jars.

Share and Show – Page No. 57

Question 1.
Find the quotient.
14.8)\(\overline { 99.456 } \)

Answer: 6.72

Explanation: 99.456÷14.8= 6.72

Estimate. Then find the quotient.

Question 2.
$10.80 ÷ $1.35

Answer: 8

Explanation:
$10.80 ÷ $1.35
= 8

Question 3.
26.4 ÷ 1.76

Answer: 15.113

Explanation:
26.4 ÷ 1.76
= 15.113

Question 4.
8.7)\(\overline { 53.07 } \)

Answer: 6.1

Explanation: 53.07÷8.7= 6.1

On Your Own

Estimate. Then find the quotient.

Question 5.
75 ÷ 12.5

Answer: 6

Explanation:

Question 6.
544.6 ÷ 1.75

Answer: 311.2

Explanation:

Question 7.
0.78)\(\overline { 0.234 } \)

Answer: 0.3.

Explanation: 0.234÷0.78= 0.3.

Attend to Precision Algebra Evaluate using the order of operations.

Question 8.
36.4 + (9.2 − 4.9 ÷ 7)

Answer: 44.9

Explanation: By BODMAS rule
36.4+(9.2−4.9÷ 7)
= 36.4+(9.2−(4.9÷7))
= 36.4+(9.2-(0.7))
= 36.4+(8.5)
= 44.9

Question 9.
16 ÷ 2.5 − 3.2 × 0.043

Answer: 6.2624

Explanation: 16 ÷ 2.5 − 3.2 × 0.043
= (16÷2.5) − (3.2 × 0.043)
= (6.4)-(3.2 × 0.043)
= 6.4 – 0.1376
= 6.2624

Question 10.
142 ÷ (42 − 6.5) × 3.9

Answer: 15.6

Explanation: 142 ÷ (42 − 6.5) × 3.9
= (142÷ 35.5) × 3.9
= 4×3.9
= 15.6

Question 11.
Marcus can buy 0.3 pound of sliced meat from a deli for $3.15. How much will 0.7 pound of sliced meat cost?

Answer: $7.35

Explanation: As 0.3 pound of sliced meat is $3.15, so cost of 1 pound of sliced meat is 3.15÷0.3= $10.5. And for 0.7 pound of sliced meat cost is 10.5×0.7= $7.35

Page No. 58

Question 12.
The table shows the earnings and the number of hours worked for three employees. Complete the table by finding the missing values. Which employee earned the least per hour? Explain.
Go Math Grade 6 Answer Key Chapter 1 Divide Multi-Digit Numbers img 11

Answer: Employee 2 has earned least per hour.

Explanation:
1. No. of hours worked is 34.02÷ 9.72= 3.5 hours.
2. Earnings per hour is 42.75÷4.5= $9.5
3. No.of hours worked is 52.65÷9.75= 5.4 hours
Employee 2 has earned least per hour.

Amoebas

Amoebas are tiny one-celled organisms. Amoebas can range in size from 0.01 mm to 5 mm in length. You can study amoebas by using a microscope or by studying photographic enlargements of them.

Jacob has a photograph of an amoeba that has been enlarged 1,000 times. The length of the amoeba in the photo is 60 mm. What is the actual length of the amoeba?
Divide 60 ÷ 1,000 by looking for a pattern.
60 ÷ 1 = 60
60 ÷ 10 = 6.0 The decimal point moves _____ place to the left.
60 ÷ 100 = ____ The decimal point moves _____ place to the left.
60 ÷ 1000 =____ The decimal point moves _____ place to the left.
So, the actual length of the amoeba is _____ mm.

Answer: 0.06mm

Explanation:
60 ÷ 10 = 6.0 The decimal point moves one place to the left.
60 ÷ 100 =0.6  The decimal point moves two place to the left.
60 ÷ 1000 =0.06 The decimal point moves three place to the left.
Actual length of amoeba is 0.06mm

Question 13.
Explain the pattern.

Answer: 0.06mm

Explanation: 60÷1000= 0.06mm.

Question 14.
Pelomyxa palustris is an amoeba with a length of 4.9 mm. Amoeba proteus has a length of 0.7 mm. How many Amoeba proteus would you have to line up to equal the length of three Pelomyxa palustris? Explain.

Answer: 21

Explanation:
Let N be the number, then
N×(proteus length)= 3× (palustris length)
N× 0.7= 3× 4.9
N×0.7= 14.7
N= 14.7÷0.7
N= 21

Divide with Decimals – Page No. 59

Estimate. Then find the quotient.

Question 1.
43.18 ÷ 3.4

Answer: 12.7

Explanation: 43.18 ÷ 3.4= 12.7

Question 2.
4.185 ÷ 0.93

Answer: 4.5

Explanation: 4.185 ÷ 0.93= 4.5

Question 3.
6.3)\(\overline { 25.83 } \)

Answer: 0.244

Explanation: 6.3÷25.83= 0.244

Question 4.
0.143 ÷ 0.55

Answer: 0.26

Explanation: 0.143 ÷ 0.55= 0.26

Evaluate using the order of operations.

Question 5.
4.92 ÷ (0.8 – 0.12 ÷ 0.3)

Answer: 12.3

Explanation: 4.92 ÷ (0.8 – 0.12 ÷ 0.3)
= 4.92÷(0.8-(0.12÷0.3))
= 4.92÷(0.8-(0.4))
= 4.92÷(0.4)
= 12.3

Question 6.
0.86 ÷ 5 – 0.3 × 0.5

Answer: 0.022

Explanation: 0.86 ÷ 5 – 0.3 × 0.5
= (0.86÷5) – (0.3 × 0.5)
= (0.172)-(0.3 × 0.5)
= 0.172 – (0.15)
= 0.022

Question 7.
17.28 ÷ (1.32 – 0.24) × 0.6

Answer: 9.6

Explanation: 17.28 ÷ (1.32 – 0.24) × 0.6
= (17.28 ÷ (1.32 – 0.24))× 0.6
= (17.28 ÷( 1.08))×0.6
= (16)×0.6
= 9.6

Problem Solving

Question 8.
If Amanda walks at an average speed of 2.72 miles per hour, how long will it take her to walk 6.8 miles?

Answer: 2.5 hours.

Explanation: Amanda walks at an average speed of 2.72 miles per hour, so for 6.8 miles it will be
6.8÷2.72= 2.5 hours.

Question 9.
Chad cycled 62.3 miles in 3.5 hours. If he cycled at a constant speed, how far did he cycle in 1 hour?

Answer: 17.8 miles.

Explanation: Chad cycled 62.3 miles in 3.5 hours, so in 1 hour 62.3÷3.5= 17.8 miles.

Question 10.
Explain how dividing by a decimal is different from dividing by a whole number and how it is similar.

Answer: By moving the decimals first the dividing will be different, and after that it will be same.

Lesson Check – Page No. 60

Question 1.
Elliot drove 202.8 miles and used 6.5 gallons of gasoline. How many miles did he travel per gallon of gasoline?

Answer: 31.2 miles.

Explanation: Elliot drove 202.8 miles and used 6.5 gallons of gasoline, so per gallon of gasoline he will travel 202.8÷6.5= 31.2 miles.

Question 2.
A package of crackers weighing 8.2 ounces costs $2.87. What is the cost per ounce of crackers?

Answer: 0.35 per ounce.

Explanation: As 8.2 ounces costs $2.87, so per ounce of crackers it will be 2.87÷8.2= 0.35 per ounce.

Spiral Review

Question 3.
Four bags of pretzels were divided equally among 5 people. How much of a bag did each person get?
\(\frac{□}{□}\)

Answer: 0.8

Explanation: As 4 bags were divided equally among 5 people, so each person gets 4÷5= 0.8 of a bag

Question 4.
A zebra ran at a speed of 20 feet per second. What operation should you use to find the distance the zebra ran in 10 seconds?

Answer: Multiplication.

Explanation: Per second zebra ran at a speed of 20 feet, so for 10 seconds 20×10= 200 feet.

Question 5.
Nira has $13.50. She receives a paycheck for $55. She spends $29.40. How much money does she have now?

Answer: $39.10.

Explanation: As Nira has $13.50 and she receives paycheck of $55, so total she had is $13.50+$55= $68.5. As she spent $29.40, so she has now $68.5-$29.40= $39.10.

Question 6.
A piece of cardboard is 24 centimeters long and 15 centimeters wide. What is its area?
_____ cm2

Answer: 360 cm2

Explanation:
Area= Length×wide
= 24×15
= 360 cm2

Chapter 1 Review/Test – Page No. 61

Question 1.
Use the numbers to complete the factor tree. You may use a number more than once.
Go Math Grade 6 Answer Key Chapter 1 Divide Multi-Digit Numbers img 12
Write the prime factorization of 54

Answer: 54=2×3×3×3

Explanation:
Go Math Grade 6 Answer Key Chapter 1 Divide Multi-Digit Numbers

Question 2.
For numbers 2a–2d, choose Yes or No to indicate whether the LCM of the two numbers is 15.
2a. 5, 3 Yes No
2b. 5, 10 Yes No
2c. 5, 15 Yes No
2d. 5, 20 Yes No

2a. 5, 3

Answer: Yes

Explanation:
Multiples of 5: 5,10,15
Multiples of 3: 53,6,9,12,15
LCM of 5,3 is 15

2b. 5, 10

Answer: No

Explanation:
Multiples of 5: 5,10
Multiples of 10: 10
LCM of 5,10 is 10

2c. 5, 15

Answer: Yes

Explanation:
Multiples of 5: 5,10,15
Multiples of 15: 15
LCM is 15

2d. 5, 20

Answer:  No

Explanation:
Multiples of 5: 5,10,15,20
Multiples of 20: 20
LCM is 20

Question 3.
Select two numbers that have 9 as their greatest common factor. Mark all that apply.
Options:
a. 3, 9
b. 3, 18
c. 9, 18
d. 9, 36
e. 18, 27

Answer: c,d,e

Explanation:
a. 3,9
Factors of 3: 1,3.
Factors of 9: 1,3,9.
GCF is 3

b. 3,18
Factors of 3: 1,3
Factors of 18: 1,2,3,6,9,18
GCF is 3

c. 9,18
Factors of 9: 1,3,9
Factors of 18: 1,2,3,6,9,18.
GCF is 9

d. 9,36
Factors of 9: 1,3,9
Factors of 36: 1,2,3,4,6,9,18,36
GCF is 9

e. 18,27
Factors of 18: 1,2,3,6,9,18
Factors of 27: 1,3,9,27
GCF is 9

Page No. 62

Question 4.
The prime factorization of each number is shown.
15 = 3 × 5
18 = 2 × 3 × 3
Part A
Using the prime factorization, complete the Venn diagram
Go Math Grade 6 Answer Key Chapter 1 Divide Multi-Digit Numbers img 13

Answer:
Prime factors of 15: 3×5
Prime factors of 18: 2×3×3
Common factors are: 3

Explanation:
Go Math Grade 6 Answer Key Chapter 1 Divide Multi-Digit Numbers

Question 4.
Part B
Find the GCF of 15 and 18.

Answer: 3

Explanation:
Factors of 15: 1,3,5
Factors of 18: 1,2,3,6,9,18
GCF is 3

Question 5.
For numbers 5a–5d, choose Yes or No to indicate whether each equation is correct.
5a. 222.2 ÷ 11 = 22.2 Yes No
5b. 400 ÷ 50 = 8 Yes No
5c. 1,440 ÷ 36 = 40 Yes No
5d. 7,236 ÷ 9 = 804 Yes No

5a. 222.2 ÷ 11 = 22.2

Answer: No

Explanation:
222.2 ÷ 11 = 20.2

5b. 400 ÷ 50 = 8

Answer: Yes

Explanation:
400 ÷ 50 = 8

5c. 1,440 ÷ 36 = 40

Answer: Yes

Explanation:
1,440 ÷ 36 = 40

5d. 7,236 ÷ 9 = 804

Answer: Yes

Explanation:
7,236 ÷ 9 = 804

Page No. 63

Question 6.
For numbers 6a–6d, select True or False for each equation.
6a. 1.7 + 4.03 = 6 True False
6b. 2.58 + 3.5 = 6.08 True False
6c. 3.21 − 0.98 = 2.23 True False
6d. 14 − 1.3 = 0.01 True False

6a. 1.7 + 4.03 = 6

Answer: False

Explanation:
1.7 + 4.03 = 5.73

6b. 2.58 + 3.5 = 6.08

Answer: True

Explanation:
2.58 + 3.5 = 6.08

6c. 3.21 − 0.98 = 2.23

Answer: True

Explanation:
3.21 − 0.98 = 2.23

6d. 14 − 1.3 = 0.01

Answer: False

Explanation:
6d. 14 − 1.3 = 12.7

Question 7.
Four friends went shopping at a music store. The table shows the number of CDs each friend bought and the total cost. Complete the table to show the average cost of the CDs each friend bought.
Go Math Grade 6 Answer Key Chapter 1 Divide Multi-Digit Numbers img 14
What is the average cost of all the CDs that the four friends bought? Show your work.

Answer: $8.94.

Explanation:
Lana purchased 4 CDs and total cost is $36.68, so cost of 1 CD is $36.68÷4= $9.17
Troy purchased 5 CDs and total cost is $40.55, so cost of 1 CD is $40.55÷5= $8.11
Juanita purchased 5 CDs and total cost is $47.15, so cost of 1 CD is $47.15÷5= $9.43
Alex purchased 6 CDs and total cost is $54.42, so cost of 1 CD is $54.42÷6= $9.07
Average cost of all CD’s i= (cost of all CD’s)÷(No.of CD’S)
=($36.68+$40.55+$47.15+$54.42)÷20
= (178.8) ÷20
= $8.94

Question 8.
The table shows the earnings and the number of hours worked for five employees. Complete the table by finding the missing values.
Go Math Grade 6 Answer Key Chapter 1 Divide Multi-Digit Numbers img 15

Answer:
1. No. of hours worked is 2.5 hours.
2. Earnings per hour is $93.654.
3. No. of hours worked is 4.4 hours.
4. Earnings per hour is $302.5.
5. Earnings per hour is $150.

Explanation:
1. No. of hours worked is $23.75÷$9.50= 2.5 hours.
2. Earnings per hour is $28.38×3.3= $93.654.
3. No. of hours worked is $38.50÷8.75= 4.4 hours.
4. Earnings per hour is $55×5.5= $302.5.
5. Earnings per hour is $60×2.5= $150.

Page No. 64

Question 9.
The distance around the outside of Cedar Park is 0.8 mile. Joanie ran 0.25 of the distance during her lunch break. How far did she run? Show your work.

Answer: 0.2 miles.

Explanation: Joanie ran 0.25 miles and the distance around the outside of cedar park is 0.8 mile, so she ran
0.25×0.8= 0.2 miles.

Question 10.
A one-celled organism measures 32 millimeters in length in a photograph. If the photo has been enlarged by a factor of 100, what is the actual length of the organism? Show your work.

Answer: 3200 millimeters.

Explanation: Length of one celled organism is 32 millimeters, as the photo was enlarged by a factor of 100, it’s actual length is 32×100= 3200 millimeters.

Question 11.
You can buy 5 T-shirts at Baxter’s for the same price that you can buy 4 T-shirts at Bixby’s. If one T-shirt costs $11.80 at Bixby’s, how much does one T-shirt cost at Baxter’s? Use numbers and words to explain your answer.

Answer: $9.44.

Explanation: As one T-shirt costs $11.80, so 4 T-shirts cost is 4×$11.80= 47.2. So 5 T-shirts at Baxter’s is 47.2 and one T-shirt cost is 47.2÷5= $9.44.

Page No. 65

Question 12.
Crackers come in packages of 24. Cheese slices come in packages of 18. Andy wants one cheese slice for each cracker. Patrick made the statement shown.
If Andy doesn’t want any crackers or cheese slices left over, he needs to buy at least 432 of each.
Is Patrick’s statement correct? Use numbers and words to explain why or why not. If Patrick’s statement is incorrect, what should he do to correct it?

Answer: Patrick’s statement is wrong.

Explanation:
Multiples of 18: 18,36,54,72
Multiples of 24: 24,48,72
LCM is 72
So the least packages he need to buy is 72.

Question 13.
There are 16 sixth graders and 20 seventh graders in the Robotics Club. For the first project, the club sponsor wants to organize the club members into equal-size groups. Each group will have only sixth graders or only seventh graders.
Part A
How many students will be in each group if each group has the greatest possible number of club members? Show your work.

Answer: Each group will have 4 members, and 4 groups of sixth grade and 5 groups of seventh grade.

Explanation: By distributive property 16+20
=(4×4)+(4×5)
=4×(4+5)
So each group will have 4 members, and 4 groups of sixth grade and 5 groups of seventh grade.

Question 13.
Part B
If each group has the greatest possible number of club members, how many groups of sixth graders and how many groups of seventh graders will there be? Use numbers and words to explain your answer
__________ groups of sixth graders
__________ groups of seventh graders

Answer: Each group will have 4 members, and 4 groups of sixth grade and 5 groups of seventh grade.

Explanation: By distributive property 16+20
=(4×4)+(4×5)
=4×(4+5)
So each group will have 4 members, and 4 groups of sixth grade and 5 groups of seventh grade.

Page No. 66

Question 14.
The Hernandez family is going to the beach. They buy sun block for $9.99, 5 snacks for $1.89 each, and 3 beach toys for $1.49 each. Before they leave, they fill up the car with 13.1 gallons of gasoline at a cost of $3.70 per gallon.
Part A
Complete the table by calculating the total cost for each item.
Go Math Grade 6 Answer Key Chapter 1 Divide Multi-Digit Numbers img 16

Answer: Total cost is $48.47+$9.45++$4.47+$9.99= $72.38

Explanation:
Gasoline 13.1×$3.70= $48.47
Snacks 5×$1.89= $9.45
Beach toys 3×$1.49= $4.47
Sun block 1×$9.99= $9.99
Total cost is $48.47+$9.45++$4.47+$9.99= $72.38

Question 14.
Part B
What is the total cost for everything before tax? Show your work.

Answer: $72.38.

Explanation: Total cost is $48.47+$9.45++$4.47+$9.99= $72.38.

Question 14.
Part C
Mr. Hernandez calculates the total cost for everything before tax using this equation.
Total cost = 13.1 + 3.70 × 5 + 1.89 × 3 + 1.49 × 9.99
Do you agree with his equation? Use numbers and words to explain why or why not. If the equation is not correct, write a correct equation.

Answer: No

Explanation: Mr. Hernandez calculated in a wrong way.
Total cost is (13.1×$3.70)+(5×$1.89)+(3×$1.49)+(1×$9.99)= $72.38.

Conclusion:

I hope the information provided in the Go Math Grade 6 Answer Key Chapter 1 Chapter 1 Divide Multi-Digit Numbers. For any queries you can post your comments in the below comment section.

Go Math Grade 3 Answer Key Chapter 6 Understand Division Extra Practice

go-math-grade-3-chapter-6-understand-division-extra-practice-answer-key

If you looking to practice Go Math 3rd Grade Textbook Questions then take the help of the Go Math Grade 3 Answer Key Chapter 6 Understand Division Extra Practice. You need to have strong fundamentals in Maths in order to become a pro in the Subject. You can easily understand the basics of the division with the Go Math Grade 3 Answer Key Chapter 6 Understand Division Extra Practice. Solve as Many Questions as possible from the Extra Practice and Clear the Final Exams with better grades.

3rd Grade Go Math Answer Key Ch 6 Understand Division Extra Practice

Repeated subtraction, Equal groups, Number line, related multiplication, and division facts are all the topics covered in the 3rd Grade Go Math Answer Key Ch 6. Before you begin your preparation firstly know the syllabus i.e. concepts in Chapter 6 Understand Division and prepare accordingly. Check out the Step by Step Solutions provided for 3rd Grade Go Math Answer Key Chapter 6 Understand Division Extra Practice and learn the concepts efficiently.

Common Core – Page No. 123000

Lessons 6.1–6.3 Make equal groups.

Complete the table.

Counters Number of Equal Groups Number in Each Group
1. 18 9 ________
2. 24 ________ 8
3. 12 6 ________
4. 35 7 ________
5. 32 ________ 4
6. 25 ________ 5

Answer:

Counters Number of Equal Groups Number in Each Group
1. 18 9 2
2. 24 3 8
3. 12 6 2
4. 35 7 5
5. 32 8 4
6. 25 5 5

Explanation:

1. Number of counters = 18
Number of equal groups = 9
Number in each group = x
x × 9 = 18
x= 18/9 = 2
Therefore number in each group = 2

2. Number of counters = 24
Number in each group = 8
Number of equal groups = x
x × 8 = 24
x = 24/8 = 3
Thus the number of equal groups = 3

3. Number of counters = 12
Number of equal groups = 6
Number in each group = x
x × 6 = 12
x = 12/6 = 2
So, the number in each group = 2

4. Number of counters = 35
Number of equal groups = 7
Number in each group = x
x × 7 = 35
x = 35/7 = 5
x = 5
Therefore number in each group = 5

5. Number of counters = 32
Number of equal groups = x
Number in each group = 8
x × 8 = 32
x = 32/8 = 4
Thus the number of equal groups = 4

6. Number of counters = 25
Number of equal groups = x
Number in each group = 5
x × 5 = 25
x = 25/5 = 5
So, the number of equal groups = 5

Lesson 6.4

Write a division equation for the picture.

Question 1.
Go Math Grade 3 Answer Key Chapter 6 Understand Division Extra Practice Common Core img 1
Type below:
__________

Answer: 27 ÷ 3 = 9 or 27 ÷ 9 = 3

Explanation:

Total number of counters = 27
Number of equal groups = 3
Number in each group = 9
The division equation is
Number of counters by number of groups = 27 ÷ 3 = 9
or
Number of counters by number in each group = 27 ÷ 9 = 3

Question 2.
Go Math Grade 3 Answer Key Chapter 6 Understand Division Extra Practice Common Core img 2
Type below:
__________

Answer: 24 ÷ 4 = 6 or 24 ÷ 6 = 4

Explanation:

Total number of counters = 24
Number of equal groups = 4
Number in each group = 6
The division equation is
Number of counters by number of groups = 24 ÷ 4 = 6
or
Number of counters by number in each group = 24 ÷ 6 = 4

Lesson 6.5

Write a division equation.

Question 3.
Go Math Grade 3 Answer Key Chapter 6 Understand Division Extra Practice Common Core img 3
Type below:
__________

Answer: 3 groups, 15 ÷ 5 = 3

Explanation:

Step 1:

Starts at 15

Step 2:

Count back by 5s as many times as you can.

Step 3:

Count the number of times you jumped back 5.
You jumped back by 15 three times
There are 3 jumps of 5 in 15.

Question 4.
Go Math Grade 3 Answer Key Chapter 6 Understand Division Extra Practice Common Core img 4
Type below:
__________

Answer: 24 ÷ 6 = 4

Explanation:

Step 1:

Begins at 24

Step 2:

Subtract with 6 until you get 0.

Step 3:

Count the number of times you subtract with 6.

You subtract 4 times
There are 4 groups of 6 with 24
So, 24 ÷ 6 = 4

Common Core – Page No. 124000

Lesson 6.6

Make an array. Then write a division equation.

Question 1.
12 tiles in 4 rows
______ ÷ ______ = ______

Answer: 12 ÷ 4 = 3

Explanation:

■ ■ ■
■ ■ ■
■ ■ ■
■ ■ ■
Total number of tiles = 12
Number of rows = 4
Number of tiles in each row = x
Divide the number of tiles by number of rows = 12 ÷ 4 = 3

Question 2.
18 tiles in 3 rows
______ ÷ ______ = ______

Answer: 18 ÷ 3 = 6

Explanation:

■ ■ ■ ■ ■ ■
■ ■ ■ ■ ■ ■
■ ■ ■ ■ ■ ■
Total number of tiles = 18
Number of rows = 3
Number of tiles in each row = y
Divide the number of tiles by no. of rows = 18 ÷ 3 = 6

Question 3.
35 tiles in 5 rows
______ ÷ ______ = ______

Answer: 35 ÷ 5 = 7

Explanation:

■ ■ ■ ■ ■ ■ ■
■ ■ ■ ■ ■ ■ ■
■ ■ ■ ■ ■ ■ ■
■ ■ ■ ■ ■ ■ ■
■ ■ ■ ■ ■ ■ ■

Total number of tiles = 35
Number of rows = 5
Number of tiles in each row = p
Divide the number of tiles by number of rows = 35 ÷ 5 = 7

Question 4.
28 tiles in 7 rows
______ ÷ ______ = ______

Answer: 28 ÷ 7 = 4

Explanation:

■ ■ ■ ■
■ ■ ■ ■
■ ■ ■ ■
■ ■ ■ ■
■ ■ ■ ■
■ ■ ■ ■
■ ■ ■ ■

Total number of tiles = 28
Number of rows = 7
Number of tiles in each row = x
Divide the number of tiles by number of rows = 28 ÷ 7 = 4

Lesson 6.7

Complete the equations.

Question 5.
8 × ______ = 40 40 ÷ 8 = ______

Answer: 5, 5

Explanation:

Let x be the unknown factor
8 × x = 40
x = 40/8
x = 5
Check whether the related multiplication and division facts are the same or not.
40 ÷ 8 = 5
The related facts of 40 and 8 are 5.

Question 6.
6 × ______ = 36 36 ÷ 6 = ______

Answer: 6, 6

Explanation:

Let y be the unknown factor
6 × y = 36
y = 36/6 = 6
Check if the related multiplication and division facts are the same or not.
36 ÷ 6 = 6
The related facts of 36 and 6 are 6.

Question 7.
3 × ______ = 21 21 ÷ 3 = ______

Answer: 7, 7

Explanation:

Let x be the unknown factor
3 × x = 21
x = 21
Check whether the related facts are the same or not.
21 ÷ 3 = 7
The quotient is 7.

Question 8.
2 × ______ = 18 18 ÷ 2 = ______

Answer: 9, 9

Explanation:

Let b be the unknown factor
2 × b = 18
b = 18/2 = 9
Check the related multiplication and division facts
18 ÷ 2 = 9
The related facts of 18 and 2 are 9.

Lesson 6.8 (pp. 239–243)

Write the related facts for the array.

Question 9.
■ ■ ■ ■ ■
■ ■ ■ ■ ■
■ ■ ■ ■ ■
______ × ______ = ______
______ × ______ = ______
______ ÷ ______ = ______
______ ÷ ______ = ______

Answer:

3 × 5 = 15
5 × 3 = 15
15 ÷ 3 = 5
15 ÷ 5 = 3

Explanation:

Total number of tiles = 15
Number of equal rows = 3
Number of rows in each group = 5
So, the related 5, 3 and 15 is 5× 3 = 15, 3×5 = 15, 15 ÷ 3 = 5 and 15÷ 5 = 3

Question 10.
■ ■ ■ ■ ■ ■
■ ■ ■ ■ ■ ■
■ ■ ■ ■ ■ ■
______ × ______ = ______
______ × ______ = ______
______ ÷ ______ = ______
______ ÷ ______ = ______

Answer:

3 × 6 = 18
6 × 3 = 18
18 ÷ 3 = 6
18 ÷ 6 = 3

Explanation:

Total number of tiles = 18
Number of equal rows = 3
Number of rows in each group = 6
So, the related 18, 3 and 6 is 3 × 6 = 18, 6 × 3 = 18, 18 ÷ 3 = 6 and 18 ÷ 6 = 3

Question 11.
■ ■ ■ ■ ■
■ ■ ■ ■ ■
______ × ______ = ______
______ × ______ = ______
______ ÷ ______ = ______
______ ÷ ______ = ______

Answer:

2 × 5 = 10
5 × 2 = 10
10 ÷ 2 = 5
10 ÷ 5 = 2

Explanation:

Total number of tiles = 10
Number of equal rows = 2
Number of rows in each group = 5
So, the related 2, 5 and 10 is 2 × 5 = 10, 5 × 2 = 10, 10 ÷ 2 = 5 and 10 ÷ 5 = 2

Lesson 6.9

Find the quotient.

Question 12.
7 ÷ 1 = ______

Answer: 7

Explanation:

Any number divided by 1 will be the same number. Thus the quotient is 7.

Question 13.
4 ÷ 4 = ______

Answer: 1

Explanation:

The number divided by the same number will be always 1. Thus the quotient is 1.

Question 14.
9 ÷ 1 = ______

Answer: 9

Explanation:

Any number divided by 1 will be always the same number. So, the quotient is 9.

Question 15.
0 ÷ 1 = ______

Answer: 0

Explanation:

0 divided by any number is always 0. So, the quotient is 0.

Question 16.
Anton has 8 flower pots. He plants 1 seed in each pot. How many seeds does Anton use?
______ seeds

Answer: 8 seeds

Explanation:

Anton has 8 flower pots.
He plants 1 seed in each pot.
Number of seeds Anton used = x
x × 1 = 8
x = 8/1
x = 8
Therefore there are 8 seeds in 8 flower pots.

All the Questions in Go Math Grade 3 Answer Key Chapter 6 Understand Division Extra Practice helps the students to be prepared for their exams. For any assistance needed you can always look upto  Go Math Grade 3 Answer Key Chapter 6 Understand Division.  You can get All Lessons Solutions in Chapter 6 Understand Division here.

Go Math Grade 4 Answer Key Chapter 13 Algebra: Perimeter and Area

go-math-grade-4-chapter-13-algebra-perimeter-and-area-answer-key

Students who are looking for a great study resource or prep resource can refer to this page. Here, we have curated a Grade 4 Answer Key of Go Math Chapter 13 Algebra: Perimeter and Area. Download HMH Go Math Grade 4 Answer Key Chapter 13 Algebra: Perimeter and Area pdf by accessing the links available over here. Save them and use the Go Math Grade 4 Answer Key Chapter 13 Algebra: Perimeter and Area as a reference purpose during your practice sessions & score good marks in the exam.

Go Math Grade 4 Answer Key Chapter 13 Algebra: Perimeter and Area

Students can find various concepts questions and solutions covered in the chapter 13 Algebra: Perimeter and Area from this Go math Gerade 4 Answer Keys. All these solutions are prepared by the subject experts in a well-organized and understanding manner. So, practice all exercise and homework problems through Go Math 4th Grade Key of Chapter 13 Perimeter and Area. Also, test your knowledge by answering the given sums and learn your mistakes using HMH Go Math Grade 4 Solution Key Chapter Perimeter and Area.

Common Core – New – Page No. 721

Perimeter

Find the perimeter of the rectangle or square.

Question 1.
Go Math Grade 4 Answer Key Chapter 13 Algebra Perimeter and Area Common Core - New img 1
9+3+9+3=24
24 inches

Explanation:

Length = 9 inches
Width = 3 inches
Perimeter of the rectangle = l + w + l + w
9+3+9+3=24
Therefore the Perimeter of the rectangle = 24 inches.

Question 2.
Go Math Grade 4 Answer Key Chapter 13 Algebra Perimeter and Area Common Core - New img 2
_____ meters

Answer: 32 meters

Explanation:

Side of a square = 8 meters
The perimeter of a square = 4a
= 4 × 8 meters = 32 meters
Thus the perimeter of a square = 32 meters.

Question 3.
Go Math Grade 4 Answer Key Chapter 13 Algebra Perimeter and Area Common Core - New img 3
_____ feet

Answer: 44 feet

Explanation:

Length = 10 ft
Width = 12 ft
Perimeter of the rectangle = l + w + l + w
P = 10 + 12 + 10 + 12 = 20 + 24 = 44 feets
Thus the perimeter of the rectangle = 44 feet.

Remember: perimeter is the total distance around the outside, which can be found by adding together the length of each side. In the case of a rectangle, opposite sides are equal in length, so the perimeter is twice its width plus twice its height.

Question 4.
Go Math Grade 4 Answer Key Chapter 13 Algebra Perimeter and Area Common Core - New img 4
_____ centimeters

Answer: 108 centimeters

Explanation:

Length = 30 cm
Width = 24 cm
Perimeter of the rectangle = l + w + l + w
= 30 + 24 + 30 + 24 = 60 + 48
= 108 centimeters
Therefore the perimeter of the rectangle = 108 centimeters

Question 5.
Go Math Grade 4 Answer Key Chapter 13 Algebra Perimeter and Area Common Core - New img 5
_____ inches

Answer: 216 inches

Explanation:

Length = 25 in.
Width = 83 in.
Perimeter of the rectangle = l + w + l + w
= 25 + 83 + 25 + 83
= 216 inches
Thus the perimeter of the rectangle = 216 inches

Question 6.
Go Math Grade 4 Answer Key Chapter 13 Algebra Perimeter and Area Common Core - New img 6
_____ meters

Answer: 240 meters

Explanation:

The side of a square = 60 meters
The perimeter of the square = 4a
= 4 × 60 meters = 240 meters
Thus the perimeter of the square = 240 meters.

Problem Solving

Question 7.
Troy is making a flag shaped like a square. Each side measures 12 inches. He wants to add ribbon along the edges. He has 36 inches of ribbon. Does he have enough ribbon? Explain.
_____

Answer: No. He needs 48 inches of ribbon.

Explanation:

Troy is making a flag shaped like a square. Each side measures 12 inches.
He wants to add a ribbon along the edges.
He has 36 inches of ribbon.
36 inches + 12 inches = 48 inches

Question 8.
The width of the Ochoa Community Pool is 20 feet. The length is twice as long as its width. What is the perimeter of the pool?
_____ feet

Answer: 120 feet

Explanation:

The width of the Ochoa Community Pool is 20 feet.
The length is twice as long as its width.
Length = 2 × 20 feet = 40 feet
Perimeter of the rectangle = l + w + l + w
= 40 + 20 + 40 + 20 = 120 feet
Thus the perimeter of the pool is 120 feet.

Common Core – New – Page No. 722

Lesson Check

Question 1.
What is the perimeter of a square window with sides 36 inches long?
Options:
a. 40 inches
b. 72 inches
c. 144 inches
d. 1,296 inches

Answer: 144 inches

Explanation:

Given, Side of a square = 36 inches
The perimeter of the square = 4 × side = 4a
= 4 × 36 inches = 144 inches
Thus the perimeter of the square = 144 inches
The correct answer is option C.

Question 2.
What is the perimeter of the rectangle below?
Go Math Grade 4 Answer Key Chapter 13 Algebra Perimeter and Area Common Core - New img 7
Options:
a. 11 meters
b. 14 meters
c. 18 meters
d. 400 meters

Answer: 18 meters

Explanation:

Length of the rectangle = 5 meter
Width of the rectangle = 4 meters
The perimeter of the rectangle = l + w + l + w
= 5 + 4 + 5 + 4 = 18 meters
Thus the correct answer is option C.

Spiral Review

Question 3.
Which is the most reasonable estimate for the measure of the angle Natalie drew?
Go Math Grade 4 Answer Key Chapter 13 Algebra Perimeter and Area Common Core - New img 8
Options:
a. 30°
b. 90°
c. 180°
d. 210°

Answer: 90°

Explanation:

By seeing the above figure we can say that it is the right angle.
The correct answer is option B.

Question 4.
Ethan has 3 pounds of mixed nuts. How many ounces of mixed nuts does Ethan have?
Options:
a. 30 ounces
b. 36 ounces
c. 48 ounces
d. 54 ounces

Answer: 48 ounces

Explanation:

Given that, Ethan has 3 pounds of mixed nuts.
1 pound = 16 ounces
3 pounds = 3 × 16 ounces = 48 ounces
Therefore the correct answer is option C.

Question 5.
How many lines of symmetry does the shape below appear to have?
Go Math Grade 4 Answer Key Chapter 13 Algebra Perimeter and Area Common Core - New img 9
Options:
a. 0
b. 1
c. 2
d. more than 2

Answer: 1

Explanation:

The above shape has 1 line of symmetry.
The correct answer is option B.

Question 6.
Which of the following comparisons is correct?
Options:
a. 0.70 > 7.0
b. 0.7 = 0.70
c. 0.7 < 0.70
d. 0.70 = 0.07

Answer: 0.7 = 0.70

Explanation:

a. 0.70 > 7.0
7.0 = 7
0.7 is less than 7

b. 0.7 = 0.70
0.7 is nothing but 0.70
So, the comparision is correct.
The answer is option B.

Page No. 725

Question 1.
Find the area of the rectangle.
Go Math Grade 4 Answer Key Chapter 13 Algebra Perimeter and Area img 10
A = _____ square cm

Answer: 143 square cm

Explanation:

Length = 11 cm
Width = 13 cm
Area of the rectangle = l × w
= 11 cm × 13 cm = 143 square cm
Therefore the area of the rectangle = 143 square cm

Find the area of the rectangle or square.

Question 2.
Go Math Grade 4 Answer Key Chapter 13 Algebra Perimeter and Area img 11
A = _____ square inches

Answer: 14 square inches

Explanation:

Length = 7 inches
Width = 2 inches
Area of the rectangle = l × w
= 7 inches × 2 inches = 14 inches
Therefore the area of the rectangle = 14 square inches

Question 3.
Go Math Grade 4 Answer Key Chapter 13 Algebra Perimeter and Area img 12
A = _____ square meters

Answer: 81 square meters

Explanation:

Side of the square = 9 m
Area of a square = s × s
= 9 m × 9 m = 81 square meters
Thus the area of a square = 81 square meters

Question 4.
Go Math Grade 4 Answer Key Chapter 13 Algebra Perimeter and Area img 13
A = _____ square feet

Answer: 112 square feet

Explanation:

Length = 8 feet
Width = 14 feet
Area of the rectangle = l × w
= 8 feet × 14 feet = 112 square feet
Therefore, area of the rectangle = 112 square feet

Find the area of the rectangle or square.

Question 5.
Go Math Grade 4 Answer Key Chapter 13 Algebra Perimeter and Area img 14
A = _____ square feet

Answer: 65 square feet

Explanation:

Length of the rectangle = 13 ft
Width of the rectangle = 5 feet
Area of a rectangle = l × w
= 13 feet × 5 feet = 65 square feet
Thus, the area of the rectangle = 65 square feet

Question 6.
Go Math Grade 4 Answer Key Chapter 13 Algebra Perimeter and Area img 15
A = _____ square yards

Answer: 169 square yards

Explanation:

Side of the square = 13 yards
Area of a square = s × s
= 13 yards × 13 yards = 169 square yards
Therefore, the area of a square = 169 square yards

Question 7.
Go Math Grade 4 Answer Key Chapter 13 Algebra Perimeter and Area img 16
A = _____ square centimeters

Answer: 40 square centimeters

Explanation:

Length of the rectangle = 20 cm
Width of the rectangle = 2 cm
Area of a rectangle = l × w
= 20 cm × 2 cm = 40 square centimeters
Therefore the area of the rectangle = 40 square centimeters.

Practice: Copy and Solve Find the area of the rectangle.

Question 8.
base: 16 feet
height: 6 feet
A = _____ square feet

Answer: 96 square feet

Explanation:

base: 16 feet
height: 6 feet
Area of a rectangle = b ×h
= 16 feet × 6 feet = 96 square feet
Thus the area of the rectangle = 96 square feet

Question 9.
base: 9 yards
height: 17 yards
A = _____ square yards

Answer: 153 square yards

Explanation:

base: 9 yards
height: 17 yards
Area of a rectangle = b × h
9 yards × 17 yards = 153 square yards
The area of the rectangle = 153 square yards

Question 10.
base: 14 centimeters
height: 11 centimeters
A = _____ square centimeters

Answer: 154 square centimeters

Explanation:

base: 14 centimeters
height: 11 centimeters
Area of a rectangle = b × h
14 centimeters × 11 centimeters = 154 square centimeters
The area of the rectangle = 154 square centimeters

Question 11.
Terry’s rectangular yard is 15 meters by 18 meters. Todd’s rectangular yard is 20 meters by 9 meters. How much greater is the area of Terry’s yard than Todd’s yard?
_____ square meters

Answer: 90 square meters

Explanation:

Given,
Terry’s rectangular yard is 15 meters by 18 meters.
Todd’s rectangular yard is 20 meters by 9 meters.
Terry’s rectangular yard:
Area of a rectangle = b × h
= 15 meters × 18 meters = 270 square meters
Todd’s rectangular yard:
Area of a rectangle = b × h
20 meters × 9 meters = 180 square meters
270 square meters – 180 square meters = 90 square meters
Terry’s yard is 90 square meters greater than Todd’s yard.

Question 12.
Reason Quantitatively Carmen sewed a square baby quilt that measures 36 inches on each side. What is the area of the quilt?
A = _____ square inches

Answer: 1296 square inches

Explanation:

Carmen sewed a square baby quilt that measures 36 inches on each side.
Area of a square = s × s
= 36 inches × 36 inches = 1296 square inches
Therefore the area of the quilt is 1296 square inches.

Page No. 726

Go Math Grade 4 Answer Key Chapter 13 Algebra Perimeter and Area img 17

Question 13.
Nancy and Luke are drawing plans for rectangular flower gardens. In Nancy’s plan, the garden is 18 feet by 12 feet. In Luke’s plan, the garden is 15 feet by 15 feet. Who drew the garden plan with the greater area? What is the area?
a. What do you need to find?
Type below:
__________

Answer: I need to find who drew the garden plan with the greater area.

Question 13.
b. What formula will you use?
Type below:
__________

Answer: I will Area of rectangle and Area of a square formula

Question 13.
c. What units will you use to write the answer?
Type below:
__________

Answer: Square feet units

Question 13.
d. Show the steps to solve the problem.
Type below:
__________

Answer:
First, we need to find the area of Nancy’s plan
Length = 18 feet
Width = 12 feet
Area of a rectangle = l × w
A = 18 feet × 12 feet = 216 square feet
And now we need to find the area of Luke’s plan
A = s × s
A = 15 feet × 15 feet = 225 square feet

Question 13.
e. Complete the sentences.
The area of Nancy’s garden is _______.
The area of Luke’s garden is _______.
_______ garden has the greater area.
Type below:
__________

Answer:
The area of Nancy’s garden is 216 square feet.
The area of Luke’s garden is 225 square feet.
Luke’s garden has a greater area.

Question 14.
Victor wants to buy fertilizer for his yard. The yard is 35 feet by 55 feet. The directions on the bag of fertilizer say that one bag will cover 1,250 square feet. How many bags of fertilizer should Victor buy to be sure that he covers the entire yard?
______ bags

Answer: 2 bags

Explanation:
Given that,
Victor wants to buy fertilizer for his yard. The yard is 35 feet by 55 feet.
The directions on the bag of fertilizer say that one bag will cover 1,250 square feet.
A = b × h
A = 35 feet × 55 feet
A = 1925 square feet
1925 square feet is greater than 1,250 square feet.
So, Victor has to buy 2 bags to be sure that he covers the entire yard.

Question 15.
Tuan is an artist. He is painting on a large canvas that is 45 inches wide. The height of the canvas is 9 inches less than the width. What is the area of Tuan’s canvas?
A = ______ square inches

Answer: 1620 square inches

Explanation:
Tuan is an artist. He is painting on a large canvas that is 45 inches wide.
The height of the canvas is 9 inches less than the width.
So, h = 45 – 9 = 36 inches
A = b × h
A = 45 inches × 36 inches
A = 1,620 square inches
Therefore the area of Tuan’s canvas is 1620 square inches.

Common Core – New – Page No. 727

Area

Find the area of the rectangle or square.

Question 1.
Go Math Grade 4 Answer Key Chapter 13 Algebra Perimeter and Area Common Core - New img 18

Question 2.
Go Math Grade 4 Answer Key Chapter 13 Algebra Perimeter and Area Common Core - New img 19
______ square yards

Answer: 64 square yards

Explanation:

Side of the square = 8 yards
Area of the square = s × s
8 yards × 8 yards = 64 square yards
Therefore, The area of the square is 64 square yards.

Question 3.
Go Math Grade 4 Answer Key Chapter 13 Algebra Perimeter and Area Common Core - New img 20
_____ square meters

Answer: 45 square meters

Explanation:

Length of the rectangle = 15 m
Width of the rectangle = 3 m
Area of the rectangle = b × h
= 15 m × 3 m = 45 square meters
Thus the area of the rectangle is 45 square meters.

Question 4.
Go Math Grade 4 Answer Key Chapter 13 Algebra Perimeter and Area Common Core - New img 21
______ square inches

Answer: 78 square inches

Explanation:

The base of the rectangle = 13 in.
Height of the rectangle = 6 in.
Area of the rectangle = b × h
13 in. × 6 in. = 78 square inches
Thus the area of the rectangle is 78 square inches.

Question 5.
Go Math Grade 4 Answer Key Chapter 13 Algebra Perimeter and Area Common Core - New img 22
______ square centimeters

Answer: 150 square centimeters

Explanation:

The base of the rectangle = 30 cm
Height of the rectangle = 5 cm
Area of the rectangle = b × h
30 cm × 5 cm = 150 square centimeters
Therefore, the area of the rectangle = 150 square centimeters

Question 6.
Go Math Grade 4 Answer Key Chapter 13 Algebra Perimeter and Area Common Core - New img 23
______ square feet

Answer: 56 square feet

Explanation:

The base of the rectangle = 14 feet
Height of the rectangle = 4 feet
Area of the rectangle = b × h
14 feet × 4 feet = 56 square feet
Therefore, the area of the rectangle = 56 square feet.

Problem Solving

Question 7.
Meghan is putting wallpaper on a wall that measures 8 feet by 12 feet. How much wallpaper does Meghan need to cover the wall?
______ square feet wallpaper

Answer: 96 square feet

Explanation:

Meghan is putting wallpaper on a wall that measures 8 feet by 12 feet.
The base of the rectangle = 8 feet
Height of the rectangle = 12 feet
Area of the rectangle = b × h
8 feet × 12 feet = 96 square feet
Thus the Area of the rectangle = 96 square feet

Question 8.
Bryson is laying down sod in his yard to grow a new lawn. Each piece of sod is a 1-foot by 1-foot square. How many pieces of sod will Bryson need to cover his yard if his yard measures 30 feet by 14 feet?
______ pieces

Answer: 420 pieces

Explanation:

Bryson is laying down sod in his yard to grow a new lawn.
Each piece of sod is a 1-foot by 1-foot square.
The base of the rectangle = 30 feet
Height of the rectangle = 14 feet
Area of the rectangle = b × h
= 30 feet × 14 feet = 420 sq. ft.
Therefore Bryson needs 420 pieces of sod to cover his yard.

Common Core – New – Page No. 728

Lesson Check

Question 1.
Ellie and Heather drew floor models of their living rooms. Ellie’s model represented 20 feet by 15 feet. Heather’s model represented 18 feet by 18 feet. Whose floor model represents the greater area? How much greater?
Options:
a. Ellie; 138 square feet
b. Heather; 24 square feet
c. Ellie; 300 square feet
d. Heather; 324 square feet

Answer: Heather; 24 square feet

Explanation:

Given,
Ellie and Heather drew floor models of their living rooms.
Ellie’s model represented 20 feet by 15 feet.
Heather’s model represented 18 feet by 18 feet.
Area of Ellie’s model = 20 feet × 15 feet = 300 square feet
Area of Heather’s model = 18 feet × 18 feet = 324 square feet
Now subtract the area of Ellie’s model from Heather’s model = 324 square feet – 300 square feet = 24 square feet
Thus the area of Heather’s model is greater than Ellie’s model
The correct answer is option B.

Question 2.
Tyra is laying down square carpet pieces in her photography studio. Each square carpet piece is 1 yard by 1 yard. If Tyra’s photography studio is 7 yards long and 4 yards wide, how many pieces of square carpet will Tyra need?
Options:
a. 10
b. 11
c. 22
d. 28

Answer: 28

Explanation:

Tyra is laying down square carpet pieces in her photography studio.
Each square carpet piece is 1 yard by 1 yard. Tyra’s photography studio is 7 yards long and 4 yards wide
Area of the rectangle = b × h
= 7 yards × 4 yards
= 28 square yards
Thus the correct answer is option D.

Spiral Review

Question 3.
Typically, blood fully circulates through the human body 8 times each minute. How many times does blood circulate through the body in 1 hour?
Options:
a. 48
b. 240
c. 480
d. 4,800

Answer: 480

Explanation:

Blood fully circulates through the human body 8 times each minute.
1 minute = 60 seconds
8 × 60 seconds = 480 seconds
The correct answer is option C.

Question 4.
Each of the 28 students in Romi’s class raised at least $25 during the jump-a-thon. What is the least amount of money the class raised?
Options:
a. $5,200
b. $700
c. $660
d. $196

Answer: $700

Explanation:

Each of the 28 students in Romi’s class raised at least $25 during the jump-a-thon.
Multiply number od students with $25
28 × $25 = $700
The correct answer is option B.

Question 5.
What is the perimeter of the shape below if 1 square is equal to 1 square foot?
Go Math Grade 4 Answer Key Chapter 13 Algebra Perimeter and Area Common Core - New img 24
Options:
a. 12 feet
b. 14 feet
c. 24 feet
d. 28 feet

Answer: 28 feet

Explanation:

Given that 1 square is equal to 1 square foot
There are 14 squares
Length = 14 squares
Width = 2 squares
Area of the rectangle = l × w = 14 × 2 = 28 sq. feets
The correct answer is option D.

Question 6.
Ryan is making small meat loaves. Each small meat loaf uses \(\frac{3}{4}\) pound of meat. How much meat does Ryan need to make 8 small meat loaves?
Options:
a. 4 pounds
b. 6 pounds
c. 8 pounds
d. 10 \(\frac{2}{3}\) pounds

Answer: 6 pounds

Explanation:

Ryan is making small meatloaves.
Each small meatloaf uses \(\frac{3}{4}\) pound of meat.
Ryan need to make 8 small meatloaves.
\(\frac{3}{4}\) × 8 = 6 pounds
The correct answer is option B.

Page No. 731

Question 1.
Explain how to find the total area of the figure.
Go Math Grade 4 Answer Key Chapter 13 Algebra Perimeter and Area img 25
A = ______ square units

Answer: 23 square units

Explanation:
Rectangle:
Each square box = 1 unit
There are 7 units
Base = 7 units
Height = 2 units
The area of the figure = b × h
A = 7 units × 2 units = 14 square units
Square:
The side is 3 units
Area of the square = 3 units × 3 units = 9 square units
Add both the areas = 14 square units + 9 square units = 23 square units
Therefore the area of the above figure is 23 square units.

Find the area of the combined rectangles.

Question 2.
Go Math Grade 4 Answer Key Chapter 13 Algebra Perimeter and Area img 26
A = ______ square mm

Answer: 72 square mm

Explanation:
Area of top rectangle = b × h
Base = 12 mm
Height = 3 mm
A = 12 mm × 3 mm = 36 square mm
Area of square = s × s
s = 6 mm
A = 6 mm × 6 mm = 36 square mm
Area of the figure = 36 square mm + 36 square mm = 72 square mm
Thus the area of the above figure is 72 square mm.

Question 3.
Go Math Grade 4 Answer Key Chapter 13 Algebra Perimeter and Area img 27
A = ______ square miles

Answer: 146 square miles

Explanation:
Area of rectangle = b × h
Area of the first rectangle = 10 mi × 9 mi
A = 90 square miles
Area of the second rectangle = 8 mi × 7 mi
A = 56 square miles
Area of the figure = Area of first rectangle + Area of the second rectangle
Area of the figure = 90 square mi + 56 square miles
Thus the Area of the figure = 146 square miles

Question 4.
Go Math Grade 4 Answer Key Chapter 13 Algebra Perimeter and Area img 28
A = ______ square feet

Answer: 96 square feet

Explanation:
There are 2 squares and one rectangle in this figure
Area of the square = s × s
A = 4 ft × 4 ft = 16 square ft
Area of the square = s × s
A = 4 ft × 4 ft = 16 square ft
Area of the rectangle = b × h
A = 16 ft × 4 ft = 64 square ft
Area of the figure = 16 square ft + 16 square ft + 64 square ft
Thus the Area of the figure = 96 square feet.

Find the area of the combined rectangles.

Question 5.
Attend to Precision Jamie’s mom wants to enlarge her rectangular garden by adding a new rectangular section. The garden is now 96 square yards. What will the total area of the garden be after she adds the new section?
Go Math Grade 4 Answer Key Chapter 13 Algebra Perimeter and Area img 29
A = ______ square yards

Answer: 180 square yards

Explanation:

There are 2 rectangles in the above figure
Area of rectangle = b × h
A = 12 yard × 8 yards  = 96 square yards
Area of rectangle = b × h
A = 6 yards × 14 yards = 84 square yards
Area of the figure = 96 square yards + 84 square yards
Therefore the area of the figure = 180 square yards.

Question 6.
Explain how to find the perimeter and area of the combined rectangles at the right.
Go Math Grade 4 Answer Key Chapter 13 Algebra Perimeter and Area img 30
P = ______ feet; A = ______ square feet

Answer: A = 92 square feet; P = 52 feet

Explanation:
There are 2 rectangle in the figure
Area of rectangle = b × h
A = 5 ft × 4 ft = 20 square ft
Area of rectangle = b × h
A = 8 ft × 9 ft = 72 square ft
Area of the figure = 20 square ft + 72 square ft = 92 square ft
Perimeter of the rectangle = 2l + 2w
P = 2 × 5 + 2 × 4 = 10 + 8 = 18 feet
Perimeter of the rectangle = 2l + 2w
P = 2 × 8 + 2 × 9 = 16 + 18 = 34 feet
Perimeter of the figure = 52 feet

Page No. 732

Go Math Grade 4 Answer Key Chapter 13 Algebra Perimeter and Area img 31

Question 7.
The diagram shows the layout of Mandy’s garden. The garden is the shape of combined rectangles. What is the area of the garden?
a. What do you need to find?
Type below:
__________

Answer: I need to find the area of the garden.

Question 7.
b. How can you divide the figure to help you find the total area?
Type below:
__________

Answer: I will divide the figure into 3 parts to find the total area

Question 7.
c. What operations will you use to find the answer?
Type below:
__________

Answer: I will use the addition operation to find the area.

Question 7.
d. Draw a diagram to show how you divided the figure. Then show the steps to solve the problem.
Type below:
__________

Answer:
Go-Math-Grade-4-Answer-Key-Chapter-13-Algebra-Perimeter-and-Area-img-31
There are 2 rectangles and 1 square in this figure.
Area of rectangle = b × h
Base = 1 ft
H = 7 ft
A = 1 ft × 7 ft = 7 square ft
Area of rectangle = b × h
Base = 5 ft
H = 2 ft
A = 5 ft × 2 ft = 10 square ft
Area of the square = s × s
A = 3 ft × 3 ft = 9 square ft
Total area = 7 square ft + 10 square ft + 9 square ft
= 26 square ft

Question 8.
Workers are painting a large letter L for an outdoor sign. The diagram shows the dimensions of the L. For numbers 8a–8c, select Yes or No to tell whether you can add the products to find the area that the workers will paint.
Go Math Grade 4 Answer Key Chapter 13 Algebra Perimeter and Area img 32
8a. 2 × 8 and 2 × 4
i. yes
ii. no

Answer: Yes
Explanation:
There are 2 rectangles in the above figure
B = 2 ft
H = 8 ft
A = 2 × 8
B = 4 ft
H = 2 ft
A = 4 × 2
Thus the above statement is correct.

Question 8.
8b. 2 × 6 and 2 × 8
i. yes
ii. no

Answer: No
There are 2 rectangles in the above figure
B = 6 ft
H = 2 ft
A = 2 × 6
Then 2 will be subtracted from 8 = 6
So, the above statement 2 × 6 and 2 × 8 is false.

Question 8.
8c. 2 × 6 and 6 × 2
i. yes
ii. no

Answer: Yes
Explanation:
There are 2 rectangles in the above figure
B = 6 ft
H = 2 ft
A = 6 × 2
B = 2 ft
H = 6 ft
A = 2 × 6
Thus the above statement is true.

Common Core – New – Page No. 733

Area of Combined Rectangles

Find the area of the combined rectangles.

Question 1.
Go Math Grade 4 Answer Key Chapter 13 Algebra Perimeter and Area Common Core - New img 33

Question 2.
Go Math Grade 4 Answer Key Chapter 13 Algebra Perimeter and Area Common Core - New img 34
______ square feet

Answer: 143 square feet

Explanation:

Go-Math-Grade-4-Answer-Key-Chapter-13-Algebra-Perimeter-and-Area-img-34

Area of A = 9 ft × 5 ft = 45 sq. ft.
Area of B = 14 ft. × 7 ft. = 98 sq. ft.
Total Area = Area of A + Area of B
= 45 sq. ft. + 98 sq. ft. = 143 square feet
Therefore the total Area = 143 square feet

Question 3.
Go Math Grade 4 Answer Key Chapter 13 Algebra Perimeter and Area Common Core - New img 35
______ square inches

Answer: 63 square inches

Explanation:

Go-Math-Grade-4-Answer-Key-Chapter-13-Algebra-Perimeter-and-Area-img-35

Area of A = 9 in. × 5 in. = 45 square inches
Area of B = 6 inches × 3 inches = 18 square inches
Total Area = Area of A + Area of B
Total Area = 45 square inches + 18 square inches
Total Area = 63 square inches

Question 4.
Go Math Grade 4 Answer Key Chapter 13 Algebra Perimeter and Area Common Core - New img 36
______ square feet

Answer: 50 square feet

Explanation:

Go-Math-Grade-4-Answer-Key-Chapter-13-Algebra-Perimeter-and-Area-img-36

Area of A = 4 feet × 2 feet = 8 square feet
Area of B = 7 feet × 6 feet = 42 square feet
Total Area = Area of A + Area of B
Total Area = 8 square feet + 42 square feet
Total Area = 50 square feet

Question 5.
Go Math Grade 4 Answer Key Chapter 13 Algebra Perimeter and Area Common Core - New img 37
______ square centimeters

Answer: 180 square centimeters

Explanation:

Go-Math-Grade-4-Answer-Key-Chapter-13-Algebra-Perimeter-and-Area-img-37

Area of A = 12 cm × 7 cm = 84 square cm
Area of B = 16 cm × 6 cm = 96 square cm
Total Area = Area of A + Area of B
Total Area = 84 square cm + 96 square cm
Total Area = 180 square centimeters

Question 6.
Go Math Grade 4 Answer Key Chapter 13 Algebra Perimeter and Area Common Core - New img 38
______ square yards

Answer: 68 square yards

Explanation:

Go-Math-Grade-4-Answer-Key-Chapter-13-Algebra-Perimeter-and-Area-img-38

Area of A = 14 yd × 1 yd = 14 square yards
Area of B = 9 yd × 6 yd = 54 square yards
Total Area = Area of A + Area of B
Total Area = 14 square yards + 54 square yards
Total Area = 68 square yards

Problem Solving

Use the diagram for 7–8.

Nadia makes the diagram below to represent the counter space she wants to build in her craft room.
Go Math Grade 4 Answer Key Chapter 13 Algebra Perimeter and Area Common Core - New img 39

Question 7.
What is the area of the space that Nadia has shown for scrapbooking?
______ square feet

Answer: 52 square feet

Explanation:

Length = 13 feet
Width = 9 feet – 5 feet = 4 feet
Area of scrapbooking = l × w
= 13 feet × 4 feet
= 52 square feet
Therefore the area of the space that Nadia has shown for scrapbooking is 52 square feet.

Question 8.
What is the area of the space she has shown for painting?
______ square feet

Answer: 25 square feet

Explanation:
The space for painting is a square.
Side of the square is 5 feet
Area of the square = 5 feet × 5 feet
= 25 square feet
Thus the area of the space she has shown for painting is 25 square feet.

Common Core – New – Page No. 734

Lesson Check

Question 1.
What is the area of the combined rectangles below?
Go Math Grade 4 Answer Key Chapter 13 Algebra Perimeter and Area Common Core - New img 40
Options:
a. 136 square yards
b. 100 square yards
c. 76 square yards
d. 64 square yards

Answer: 76 square yards

Explanation:
Area of 1st rectangle = 5 yards × 8 yards = 40 square yards
Area of 2nd rectangle = 12 yards × 3 yards = 36 square yards
Area of the figure = Area of 1st rectangle + Area of 2nd rectangle
Area of the figure = 40 square yards + 36 square yards
Therefore, the Area of the figure is 76 square yards.
So, the correct answer is option C.

Question 2.
Marquis is redecorating his bedroom. What could Marquis use the area formula to find?
Options:
a. how much space should be in a storage box
b. what length of wood is needed for a shelf
c. the amount of paint needed to cover a wall
d. how much water will fill up his new aquarium

Answer: the amount of paint needed to cover a wall

Spiral Review

Question 3.
Giraffes are the tallest land animals. A male giraffe can grow as tall as 6 yards. How tall would the giraffe be in feet?
Options:
a. 2 feet
b. 6 feet
c. 12 feet
d. 18 feet

Answer: 18 feet

Explanation:
Giraffes are the tallest land animals. A male giraffe can grow as tall as 6 yards.
6 yards + 6 yards + 6 yards = 18 yards
The correct answer is option D.

Question 4.
Drew purchased 3 books for $24. The cost of each book was a multiple of 4. Which of the following could be the prices of the 3 books?
Options:
a. $4, $10, $10
b. $4, $8, $12
c. $5, $8, $11
d. $3, $7, $14

Answer: $4, $8, $12

Explanation:
Given that,
Drew purchased 3 books for $24.
The cost of each book was a multiple of 4.
So, the prices of books will be multiple of 4.
That means $4 × 1, $4 × 2, $4 × 3
=  $4, $8, $12
The correct answer is option B.

Question 5.
Esmeralda has a magnet in the shape of a square. Each side of the magnet is 3 inches long. What is the perimeter of her magnet?
Options:
a. 3 inches
b. 7 inches
c. 9 inches
d. 12 inches

Answer: 12 inches

Explanation:
Esmeralda has a magnet in the shape of a square. Each side of the magnet is 3 inches long.
Side = 3 inches
The perimeter of the square = 4s
P = 4 × 3 = 12 inches
The correct answer is option D.

Question 6.
What is the area of the rectangle below?
Go Math Grade 4 Answer Key Chapter 13 Algebra Perimeter and Area Common Core - New img 41
Options:
a. 63 square feet
b. 32 square feet
c. 18 square feet
d. 16 square feet

Answer: 63 square feet

Explanation:
Area of the rectangle = base × height
Base = 9 feet
Height = 7 feet
A = 9 feet × 7 feet
A = 63 square feet
Thus the correct answer is option A.

Page No. 735

Choose the best term from the box.
Go Math Grade 4 Answer Key Chapter 13 Algebra Perimeter and Area img 42

Question 1.
A square that is 1 unit wide and 1 unit long is a ________.
__________

Answer: Square unit

Question 2.
The _______ of a two-dimensional figure can be any side.
__________

Answer: Base

Question 3.
A set of symbols that expresses a mathematical rule is called a ______.
__________

Answer: Formula

Question 4.
The ______ is the distance around a shape.
__________

Answer: Perimeter

Find the perimeter and area of the rectangle or square.

Question 5.
Go Math Grade 4 Answer Key Chapter 13 Algebra Perimeter and Area img 43
Perimeter = ______ cm
Area = ______ square cm

Answer:
Perimeter = 52 cm
Area = 169 square cm

Explanation:
P = 4s
P = 4 × 13 = 52 cm
A = s × s
A = 13 × 13 = 169 square cm

Question 6.
Go Math Grade 4 Answer Key Chapter 13 Algebra Perimeter and Area img 44
Perimeter = ______ ft
Area = ______ square ft

Answer:
Perimeter: 48 ft
Area = 63 square ft

Explanation:
Base = 21 ft
Height = 3 ft
P = 2l +2w
P = 2 (21 ft + 3 ft)
P = 2 × 24 = 48 feet
A = b × h
A = 21 × 3
A = 63 square ft

Question 7.
Go Math Grade 4 Answer Key Chapter 13 Algebra Perimeter and Area img 45
Perimeter = ______ in.
Area = ______ square in.

Answer:
Perimeter = 46 in.
Area = 120 square in.

Explanation:
P = 2l +2w
P = 2 × 15 + 2 × 8
P = 30 + 16 = 46 inches
A = l × w
A = 15 × 8 = 120 square inches

Question 8.
Go Math Grade 4 Answer Key Chapter 13 Algebra Perimeter and Area img 46
Area = ____ square yd

Answer:
Area of the rectangle = 20 yards × 5 yards = 100 square yards
Area of the rectangle = 18 yards × 5 yards = 90 square yards
Area of the figure = 100 square yards + 90 square yards = 190 square yards

Question 9.
Go Math Grade 4 Answer Key Chapter 13 Algebra Perimeter and Area img 47
Area = ____ square meters

Answer:
A = b × h
A = 5 m × 2 m = 10 square meters
A = b × h
A = 5 m × 2 m = 10 square meters
A = b × h
A = 4 m × 2 m = 8 square meters
Now add all the areas
10 square meters + 10 square meters + 8 square meters
= 28 square meters
Therefore the area of the figures is 28 square meters

Question 10.
Go Math Grade 4 Answer Key Chapter 13 Algebra Perimeter and Area img 48
Area = ____ square feet
Answer:
Area of the rectangle = b × h
A = 14 ft × 2 ft = 28 square feet
A = s × s
A = 8 ft × 8 ft = 64 square feet
Area of the figures = 64 square feet + 28 square feet
Therefore Area of the figure = 92 square feet

Page No. 736

Question 11.
Which figure has the greatest perimeter?
Go Math Grade 4 Answer Key Chapter 13 Algebra Perimeter and Area img 49
Go Math Grade 4 Answer Key Chapter 13 Algebra Perimeter and Area img 50
Go Math Grade 4 Answer Key Chapter 13 Algebra Perimeter and Area img 51
Go Math Grade 4 Answer Key Chapter 13 Algebra Perimeter and Area img 52
________

Answer: Figure B has the highest perimeter.

Explanation:
Go Math Grade 4 Answer Key Chapter 13 Algebra Perimeter and Area img 49
P = 2l +2w
P = 2 × 3 + 2 ×5 = 6 + 10 = 16
Go Math Grade 4 Answer Key Chapter 13 Algebra Perimeter and Area img 51

P = 2 × 6 + 2 × 3 = 12 + 6 = 18
Go Math Grade 4 Answer Key Chapter 13 Algebra Perimeter and Area img 50
P = 4a = 4 × 4 = 16
Go Math Grade 4 Answer Key Chapter 13 Algebra Perimeter and Area img 52
P = 2 × 4 + 2 × 3 = 8+ 6 = 14
Thus the greatest perimeter is figure B.

Question 12.
Which figure has an area of 108 square centimeters?
Go Math Grade 4 Answer Key Chapter 13 Algebra Perimeter and Area img 53
Go Math Grade 4 Answer Key Chapter 13 Algebra Perimeter and Area img 54
Go Math Grade 4 Answer Key Chapter 13 Algebra Perimeter and Area img 55
Go Math Grade 4 Answer Key Chapter 13 Algebra Perimeter and Area img 56
________

Answer: Figure C

Explanation:
Go Math Grade 4 Answer Key Chapter 13 Algebra Perimeter and Area img 53
A = 13 cm × 6 cm = 78 square cm.
Go Math Grade 4 Answer Key Chapter 13 Algebra Perimeter and Area img 55
A = 11 cm × 11 cm = 121 square cm.
Go Math Grade 4 Answer Key Chapter 13 Algebra Perimeter and Area img 54
A = 12 cm × 9 cm = 108 square cm.
Go Math Grade 4 Answer Key Chapter 13 Algebra Perimeter and Area img 56
A = 16 cm × 38 cm = 608 square cm.
Thus the area of 108 square centimeters is Figure C.

Question 13.
Which of the combined rectangles has an area of 40 square feet?
Go Math Grade 4 Answer Key Chapter 13 Algebra Perimeter and Area img 57
Go Math Grade 4 Answer Key Chapter 13 Algebra Perimeter and Area img 58
Go Math Grade 4 Answer Key Chapter 13 Algebra Perimeter and Area img 59
Go Math Grade 4 Answer Key Chapter 13 Algebra Perimeter and Area img 60
________

Answer: Figure A

Explanation:
Go Math Grade 4 Answer Key Chapter 13 Algebra Perimeter and Area img 57
Area of top rectangle = 6 ft × 2 ft = 12 square feet
Area of bottom rectangle = 6 ft × 2 ft = 12 square feet
Area of square = 4 ft × 4 ft = 16 square feet
Add Area of top rectangle, Area of bottom rectangle and Area of square
= 12 square feet +  12 square feet + 16 square feet = 40 square feet.
Thus the correct answer is option A.

Page No. 739

Question 1.
Find the unknown measure. The area of the rectangle is 36 square feet.
Go Math Grade 4 Answer Key Chapter 13 Algebra Perimeter and Area img 61
A = b × h
The base of the rectangle is ________ .
base = _____ ft

Answer: 12 feet

Explanation:
Given,
The area of the rectangle = 36 square feet
Height = 3 feet
Base =?
A = b × h
36 square feet = b × 3 feet
b × 3 feet = 36 square feet
b = 36/3 = 12 feet
The base of the rectangle is 12 feets.

Find the unknown measure of the rectangle.

Question 2.
Go Math Grade 4 Answer Key Chapter 13 Algebra Perimeter and Area img 62
Perimeter = 44 centimeters
width = _____ cm

Answer: 10 cm

Explanation:
Given,
Perimeter = 44 centimeters
Length = 12 cm
width =?
The perimeter of the rectangle = 2 (l + w)
P = 2l + 2w
44 cm = 24 cm + 2w
2w = 44 cm – 24 cm
2w = 20 cm
w = 20/2 = 10
Therefore width = 10 cm

Question 3.
Go Math Grade 4 Answer Key Chapter 13 Algebra Perimeter and Area img 63
Area = 108 square inches
height = _____ in.

Answer: 12 inches

Explanation:
Given,
Area = 108 square inches
Base = 9 inches
height = _____ in.
A = b × h
108 square inches = 9 inches × h
h = 108/9
Height = 12 inches
Therefore the height of the rectangle = 12 inches

Question 4.
Go Math Grade 4 Answer Key Chapter 13 Algebra Perimeter and Area img 64
Area = 90 square meters
base = _____ cm

Answer: 18 meters

Explanation:
Given,
Area = 90 square meters
Height = 5 meters
base = _____ cm
A = b × h
90 square meters = b × 5 meters
b × 5 meters = 90 square meters
b = 90/5 = 18 meters
Therefore the base of the rectangle = 18 meters

Question 5.
Go Math Grade 4 Answer Key Chapter 13 Algebra Perimeter and Area img 65
Perimeter = 34 yards
length = _____ yd

Answer: 12 yards

Explanation:
Given,
Perimeter = 34 yards
Width = 5 yards
Length =?
The perimeter of the rectangle = 2 (l + w)
P = 2l + 2w
34 yards = 2 × l + 2 × 5 yards
34 yards = 2 × l + 10 yards
2 × l + 10 yards = 34 yards
2l = 34 yards – 10 yards
2l = 24 yards
l = 24/2 = 12 yards
Therefore the length of the rectangle = 12 yards.

Question 6.
Go Math Grade 4 Answer Key Chapter 13 Algebra Perimeter and Area img 66
Area = 96 square feet
base = ______ ft

Answer: 12 feet

Explanation:
Given,
Area = 96 square feet
Height = 8 feet
Base =?
A = b × h
96 square feet = b × 8 feet
b × 8 feet = 96 square feet
b = 96/8 = 12 feet
Thus base of the rectangle = 12 feet.

Question 7.
Go Math Grade 4 Answer Key Chapter 13 Algebra Perimeter and Area img 67
Area = 126 square centimeters
height = _____ centimeters

Answer: 14 centimeters

Explanation:
Given,
Area = 126 square centimeters
Base = 9 cm
height = _____ centimeters
A = b × h
126 square centimeters = 9 cm × h
9 cm × h = 126 square centimeters
h = 126/9 = 14 centimeters
Therefore the Height of the rectangle = 14 centimeters

Question 8.
A square has an area of 49 square inches. Explain how to find the perimeter of the square.
Type below:
________

Answer:

Explanation:
Given that,
A square has an area of 49 square inches.
A = 49 square inches
s^2 = 49 square inches
The square root of 49 is 7
So, each side of the square is 7 inches
The perimeter of the square = 4 × s
4 × 7 inches = 28 inches.
Therefore the perimeter of the square is 28 inches.

Page No. 740

Question 9.
Identify Relationships The area of a swimming pool is 120 square meters. The width of the pool is 8 meters. What is the length of the pool in centimeters?
length = _____ centimeters

Answer:
Given that the area of a swimming pool is 120 square meters.
The width of the pool is 8 meters.
We have to find the length of the pool in centimeters.
We know that Area of the rectangle = l × w
A = l × w
120 square meters = l × 8 meters
l × 8 meters = 120 square meters
l = 120/8 = 15 meters
Therefore, the length of the pool = 15 meters
Convert meters to centimeters
1 meter = 100 centimeters
15 meters = 1500 centimeters.
The length of the pool in centimeters = 1500 centimeters

Question 10.
An outdoor deck is 7 feet wide. The perimeter of the deck is 64 feet. What is the length of the deck? Use the numbers to write an equation and solve. A number may be used more than once.
Go Math Grade 4 Answer Key Chapter 13 Algebra Perimeter and Area img 68
P=(2 × l) + (2 × w)
So, the length of the deck is _______ feet.
length = _____ ft

Answer:
An outdoor deck is 7 feet wide.
The perimeter of the deck is 64 feet.
We know that,
P=(2 × l) + (2 × w)
64 feet = (2 × l) + (2 × 7)
64 feet = 2l + 14 feet
2 × l = 64 feet – 14 feet
2 × l = 50 feet
l = 50/2 = 25 feet
Therefore the length of the deck = 25 feet.

Question 11.
A male mountain lion has a rectangular territory with an area of 96 square miles. If his territory is 8 miles wide, what is the length of his territory?
Go Math Grade 4 Answer Key Chapter 13 Algebra Perimeter and Area img 69
length = _____ miles

Answer:
A male mountain lion has a rectangular territory with an area of 96 square miles.
Width = 8 miles
Length =?
A = l × w
96 square miles = l × 8 miles
l × 8 miles = 96 square miles
l = 96/8
l = 12 miles
Therefore, length of his territory = 12 miles

Common Core – New – Page No. 741

Find Unknown Measures

Find the unknown measure of the rectangle.

Question 1.
Go Math Grade 4 Answer Key Chapter 13 Algebra Perimeter and Area Common Core - New img 70
Perimeter = 54 feet
width = 7 feet
Think: P = (2 × l) + (2 × w)
54 = (2 × 20) + (2 × w)
54 = 40 + (2 × w)
Since 54 = 40 + 14, 2 × w = 14, and w = 7.

Question 2.
Go Math Grade 4 Answer Key Chapter 13 Algebra Perimeter and Area Common Core - New img 71
Perimeter = 42 meters
length = _____ meters

Answer: length = 12 meters

Explanation:

Given, Perimeter = 42 meters
Width = 9 meters
P = (2 × l) + (2 × w)
P = (2 × l) + (2 × 9 m)
42 m = 2l + 18 m
42 m – 18 m = 2l
2l = 24 meters
l = 24 meters/2 = 12 meters
Therefore length = 12 meters

Question 3.
Go Math Grade 4 Answer Key Chapter 13 Algebra Perimeter and Area Common Core - New img 72
Area = 28 square centimeters
height = _____ centimeters

Answer: height = 7 centimeters

Explanation:

Given,
Area = 28 square centimeters
Base = 4 cm
A = b × h
28 square centimeters = 4 cm × h
4 × h = 28
h = 28/4 = 7 cm
The height of the rectangle = 7 centimeters

Question 4.
Go Math Grade 4 Answer Key Chapter 13 Algebra Perimeter and Area Common Core - New img 73
Area = 200 square inches
base = _____ inches

Answer: base = 8 inches

Explanation:

Given,
Area = 200 square inches
Height = 25 inches
Base = ?
Area of the rectangle = b × h
200 square inches = b × 25 inches
b × 25 inches = 200 square inches
b = 200/25 = 8 inches
The base of the rectangle = 8 inches.

Problem Solving

Question 5.
Susie is an organic vegetable grower. The perimeter of her rectangular vegetable garden is 72 yards. The width of the vegetable garden is 9 yards. How long is the vegetable garden?
length = _____ yards

Answer: 27 yards

Explanation:

Susie is an organic vegetable grower.
The perimeter of her rectangular vegetable garden is 72 yards.
The width of the vegetable garden is 9 yards.
P = 72 yards
W = 9 yards
L =?
We know that,
P = (2 × l) + (2 × w)
72 yards = (2 × l) + (2 × 9)
72 yards – 18 yards = (2 × l)
(2 × l) = 72 yards – 18 yards
2l = 54 yards
l = 54/2 = 27 yards
Thus the vegetable garden is 27 yards long.

Question 6.
An artist is creating a rectangular mural for the Northfield Community Center. The mural is 7 feet tall and has an area of 84 square feet. What is the length of
the mural?
length = _____ feet

Answer: 12 feet

Explanation:

An artist is creating a rectangular mural for the Northfield Community Center.
The mural is 7 feet tall and has an area of 84 square feet.
A = 84 square feet
W = 7 feet
L =?
A = l × w
84 square feet = l × 7 feet
l × 7 feet = 84 square feet
l = 84/7 = 12 feet
Thus the length of Murali is 12 feet.

Common Core – New – Page No. 742

Lesson Check

Question 1.
The area of a rectangular photograph is 35 square inches. If the width of the photo is 5 inches, how tall is the photo?
Options:
a. 5 inches
b. 7 inches
c. 25 inches
d. 30 inches

Answer: 7 inches

Explanation:

The area of a rectangular photograph is 35 square inches.
Width = 5 inches
A = l × w
35 square inches = l × 5 inches
Length = 35/5 = inches
Thus the photo is 7 inches tall.
The correct answer is option B.

Question 2.
Natalie used 112 inches of blue yarn as a border around her rectangular bulletin board. If the bulletin board is 36 inches wide, how long is it?
Options:
a. 20 inches
b. 38 inches
c. 40 inches
d. 76 inches

Answer: 20 inches

Explanation:

Natalie used 112 inches of blue yarn as a border around her rectangular bulletin board.
Width = 36 inches
A = 112 inches
A = l × w
112 inches = l × 36 inches
l × 36 inches = 112 inches
l = 112/36 = 20 inches
Length = 20 inches
The correct answer is option A.

Spiral Review

Question 3.
A professional basketball court is in the shape of a rectangle. It is 50 feet wide and 94 feet long. A player ran one time around the edge of the court. How far did the player run?
Options:
a. 144 feet
b. 194 feet
c. 238 feet
d. 288 feet

Answer: 288 feet

Explanation:

A professional basketball court is in the shape of a rectangle.
It is 50 feet wide and 94 feet long.
A player ran one time around the edge of the court.
P = (2 × l) + (2 × w)
P = (2 × 94 feet) + (2 × 50 feet)
P = 188 feet + 100 feet = 288 feet
Therefore the perimeter of the rectangle is 288 feet.

Question 4.
On a compass, due east is a \(\frac{1}{4}\) turn clockwise from due north. How many degrees are in a \(\frac{1}{4}\) turn?
Options:
a. 45°
b. 60°
c. 90°
d. 180°

Answer: 90°

Explanation:

On a compass, due east is a \(\frac{1}{4}\) turn clockwise from due north.
\(\frac{1}{4}\) × 360° = 360°/4 = 90°
The correct answer is option C.

Question 5.
Hakeem’s frog made three quick jumps. The first was 1 meter. The second jump was 85 centimeters. The third jump was 400 millimeters. What was the total length of the frog’s three jumps?
Options:
a. 189 centimeters
b. 225 centimeters
c. 486 centimeters
d. 585 millimeters

Answer: 225 centimeters

Explanation:

Hakeem’s frog made three quick jumps.
The first was 1 meter. The second jump was 85 centimeters. The third jump was 400 millimeters.
Convert other units to centimeters
1 meter = 100 centimeters
400 millimeters = 40 centimeters
100 + 85 + 40 = 225 centimeters
Thus the correct answer is option B.

Question 6.
Karen colors in squares on a grid. She colored \(\frac{1}{8}\) of the squares blue and \(\frac{5}{8}\) of the squares red. What fraction of the squares are not colored in?
Options:
a. \(\frac{1}{8}\)
b. \(\frac{1}{4}\)
c. \(\frac{1}{2}\)
d. \(\frac{3}{4}\)

Answer: \(\frac{1}{4}\)

Explanation:

Karen colors in squares on a grid.
She colored \(\frac{1}{8}\) of the squares blue and \(\frac{5}{8}\) of the squares red.
\(\frac{1}{8}\) + \(\frac{5}{8}\) = \(\frac{6}{8}\)
Total number of fractions = \(\frac{8}{8}\)
\(\frac{8}{8}\) – \(\frac{6}{8}\) = \(\frac{2}{8}\)
\(\frac{1}{4}\) fraction of the squares are not colored.

Page No. 745

Question 1.
Lila is wallpapering one wall of her bedroom, as shown in the diagram. She will cover the whole wall except for the doorway. How many square feet of wall does Lila need to cover?
Go Math Grade 4 Answer Key Chapter 13 Algebra Perimeter and Area img 74
First, find the area of the wall.
A = b × h
Awall = _____ square feet

Answer:
Base = 12 feet
Height = 8 feet
A = b × h
Awall = 12 feet × 8 feet
Awall = 96 square feet

Question 1.
Next, find the area of the door.
A = b × h
Adoor = _____ square feet

Answer:
Base = 3 feet
Height = 7 feet
A = b × h
Adoor = 3 feet × 7 feet
Adoor = 21 square feet

Question 1.
Last, subtract the area of the door from the area of the wall.
_____ – _____ = _____ square feet
So, Lila needs to cover _____ of wall.
Type below:
________

Answer:
Adoor = 21 square feet
Awall = 96 square feet
Last, subtract the area of the door from the area of the wall.
A = Awall – Adoor
A = 96 square feet – 21 square feet
A = 75 square feet
So, Lila needs to cover 75 square feet

Question 2.
What if there was a square window on the wall with a side length of 2 feet? How much wall would Lila need to cover then? Explain.
______ square feet

Answer:
If there is a square window of length 2 feet
Area of square = s × s
Awindow = 2 × 2 = 4 square feet
Now Subtract the area of the door, area of the window from the area of the wall.
A = 96 square feet – 21 square feet – 4 square feet
A = 71 square feet
Therefore Lila need to cover 71 square feet.

Question 3.
Ed is building a model of a house with a flat roof, as shown in the diagram. There is a chimney through the roof. Ed will cover the roof with square tiles. If the area of each tile is 1 square inch, how many tiles will he need? Explain.
Go Math Grade 4 Answer Key Chapter 13 Algebra Perimeter and Area img 75
_____ tiles

Answer:
Roof:
Base = 20 inches
Height = 30 inches
Area of the roof = b × h
Aroof = 20 inches × 30 inches
Aroof = 600 inches
Chimney:
Base = 3 inches
Height = 4 inches
Area of the chimney = b × h
Achimney = 3 × 4 = 12 inches
Now subtract Area of Chimney from Area of the roof
A = 600 inches – 12 inches
A = 588 inches
Therefore Ed needs 588 tiles.

Page No. 746

Question 4.
Make Sense of Problems Lia has a dog and a cat. Together, the pets weigh 28 pounds. The dog weighs 3 times as much as the cat. How much does each pet weigh?
cat weight = _____  pounds dog weight = _____ pounds

Answer:
Given that, the pets weigh 28 pounds.
28 = 7 + 7 + 7 + 7
The dog weighs 3 times as much as the cat.
= 3 × 7 = 21 pounds
The dog weighs 21 pounds
28 – 21 = 7
The cat weighs = 7 pounds.

Question 5.
Mr. Foster is covering two rectangular pictures with glass. One is 6 inches by 4 inches and the other one is 5 inches by 5 inches. Does he need the same number of square inches of glass for each picture? Explain.
_____

Answer: No

Explanation:
Mr. Foster is covering two rectangular pictures with glass.
One is 6 inches by 4 inches and the other one is 5 inches by 5 inches.
Area of first rectangular picture = 6 × 4 = 24 square inches
Area of second rectangular picture = 5 × 5 = 25 square inches
Area of two rectangular pictures = 25 square inches – 24 square inches
1 square inch.
Therefore, he doesn’t need the same number of square inches of glass for each picture.

Question 6.
Claire says the area of a square with a side length of 100 centimeters is greater than the area of a square with a side length of 1 meter. Is she correct? Explain.
_____

Answer: No

Explanation:
Claire says the area of a square with a side length of 100 centimeters is greater than the area of a square with a side length of 1 meter.
Her statement is not correct because 1 meter = 100 centimeters.
So, the area of a square with a side length of 100 centimeters is equal to the area of a square with a side length of 1 meter.

Question 7.
A rectangular floor is 12 feet long and 11 feet wide. Janine places a rug that is 9 feet long and 7 feet wide and covers part of the floor in the room. Select the word(s) to complete the sentence.
To find the number of square feet of the floor that is NOT covered by the rug,
Go Math Grade 4 Answer Key Chapter 13 Algebra Perimeter and Area img 76 the Go Math Grade 4 Answer Key Chapter 13 Algebra Perimeter and Area img 77 Go Math Grade 4 Answer Key Chapter 13 Algebra Perimeter and Area img 78 the area of the floor.
_____ square feet

Answer:
Length = 12 feet
Width = 11 feet
Area of the rectangular floor = l × w
= 12 feet × 11 feet = 132 square feet
Room:
Length = 9 feet
Width = 7 feet
Area of the floor in the room = l × w
= 9 feet × 7 feet
= 63 square feet
Subtract the area of the rug from the area of the floor
= 132 square feet – 63 square feet = 69 square feet
The number of square feet of the floor that is NOT covered by the rug is 69 square feet.

Common Core – New – Page No. 747

Problem Solving Find the Area

Solve each problem.

Question 1.
A room has a wooden floor. There is a rug in the center of the floor. The diagram shows the room and the rug. How many square feet of the wood floor still shows?
Go Math Grade 4 Answer Key Chapter 13 Algebra Perimeter and Area Common Core - New img 79
82 square feet
Area of the floor: 13 × 10 = 130 square feet
Area of the rug: 8 × 6 = 48 square feet
Subtract to find the area of the floor still showing: 130 – 48 = 82 square feet

Question 2.
A rectangular wall has a square window, as shown in the diagram.
Go Math Grade 4 Answer Key Chapter 13 Algebra Perimeter and Area Common Core - New img 80
What is the area of the wall NOT including the window?
The area of the wall NOT including the window = _____ square feet

Answer: 96 square feet

Explanation:
Wall:
Base = 14 feet
Height = 8 feet
Area of the wall = b × h
A = 14 feet × 8 feet
A = 112 square feet
Window:
Length = 4 feet
Area of the square = s × s
Area of the window = 4 feet × 4 feet = 16 square feet
Now subtract Area of the window from the area of the rectangular wall
= 112 square feet – 16 square feet
= 96 square feet
Therefore the area of the wall NOT including the window = 96 square feet.

Question 3.
Bob wants to put down new sod in his backyard, except for the part set aside for his flower garden. The diagram shows Bob’s backyard and the flower garden.
Go Math Grade 4 Answer Key Chapter 13 Algebra Perimeter and Area Common Core - New img 81
How much sod will Bob need?
The area covered with new sod = _____ square yards

Answer: 235 square yards

Flower Garden:
Base = 20 yards
Height = 14 yards
Area of the rectangular flower garden = b × h
A = 20 yards × 14 yards
A = 280 square yards
Sod:
Base = 5 yards
Height = 9 yards
Area of sod = b × h
= 5 yards × 9 yards = 45 square yards
Now subtract area of sod from area of flower garden
= 280 square yards – 45 square yards
= 235 square yards
Thus the area covered with new sod = 235 square yards

Question 4.
A rectangular painting is 24 inches wide and 20 inches tall without the frame. With the frame, it is 28 inches wide and 24 inches tall. What is the area of the frame not covered by the painting?
The area of the frame = _____ square inches

Answer: 192 square inches

Explanation:
A rectangular painting is 24 inches wide and 20 inches tall without the frame.
A = b × h
A = 24 inches × 20 inches
A = 480 square inches
With the frame, it is 28 inches wide and 24 inches tall.
A = b × h
A = 28 inches × 24 inches
A = 672 square inches
The area of the frame not covered by the painting
= 672 square inches – 480 square inches
= 192 square inches
Therefore, The area of the frame = 192 square inches

Question 5.
One wall in Jeanne’s bedroom is 13 feet long and 8 feet tall. There is a door 3 feet wide and 6 feet tall. She has a poster on the wall that is 2 feet wide and 3 feet tall. How much of the wall is visible?
The area of the wall visible = _____ square feet

Answer: 80 square feet

Explanation:
One wall in Jeanne’s bedroom is 13 feet long and 8 feet tall.
Area of Jeanne’s bedroom = 13 feet × 8 feet = 104 square feet
Area of door = 3 feet × 6 feet = 18 square feet
Area of the wall = 2 feet × 3 feet = 6 square feet
To find the area of the wall visible we have to subtract Area of the wall, Area of the door from Area of Jeanne’s bedroom.
104 square feet – 18 square feet – 6 square feet
= 80 square feet
The area of the wall visible = 80 square feet

Common Core – New – Page No. 748

Lesson Check

Question 1.
One wall in Zoe’s bedroom is 5 feet wide and 8 feet tall. Zoe puts up a poster of her favorite athlete. The poster is 2 feet wide and 3 feet tall. How much of the wall is not covered by the poster?
Options:
a. 16 square feet
b. 34 square feet
c. 35 square feet
d. 46 square feet

Answer: 34 square feet

Explanation:
One wall in Zoe’s bedroom is 5 feet wide and 8 feet tall.
Area of the wall in Zoe’s bedroom = b × h
A = 5 feet × 8 feet
A = 40 square feet
Zoe puts up a poster of her favorite athlete. The poster is 2 feet wide and 3 feet tall.
Area of the poster = b × h
A = 2 feet × 3 feet = 6 square feet
Now subtract Area of the poster from the Area of the wall in Zoe’s bedroom
= 40 square feet – 6 square feet
= 34 square feet
Thus the area of the wall is not covered by the poster = 34 square feet.
The correct answer is option B.

Question 2.
A garage door is 15 feet wide and 6 feet high. It is painted white, except for a rectangular panel 1 foot high and 9 feet wide that is brown. How much of the garage door is white?
Options:
a. 22 square feet
b. 70 square feet
c. 80 square feet
d. 81 square feet

Answer: 81 square feet

Explanation:
A garage door is 15 feet wide and 6 feet high.
Area of the garage door = b × h
A = 15 feet × 6 feet
A = 90 square feet
It is painted white, except for a rectangular panel 1 foot high and 9 feet wide that is brown.
b = 9 feet
h = 1 foot
A = b × h
A = 9 feet × 1 feet
A = 9 square feet
Area of the garage door is white = 90 square feet – 9 square feet
Area of the garage door is white = 81 square feet
The correct answer is option D.

Spiral Review

Question 3.
Kate baked a rectangular cake for a party. She used 42 inches of frosting around the edges of the cake. If the cake was 9 inches wide, how long was the cake?
Options:
a. 5 inches
b. 12 inches
c. 24 inches
d. 33 inches

Answer: 12 inches

Explanation:
Kate baked a rectangular cake for a party. She used 42 inches of frosting around the edges of the cake.
Width = 9 inches
P = (2 × l) + (2 × w)
42 inches = (2 × l) + (2 × 9)
(2 × l) + (2 × 9) = 42 inches
(2 × l) = 42 inches – 18 inches
2l = 24 inches
l = 24/2 = 12 inches
Therefore the cake is 12 inches long.
Thus the correct answer is option B.

Question 4.
Larry, Mary, and Terry each had a full glass of juice. Larry drank \(\frac{3}{4}\) of his. Mary drank \(\frac{3}{8}\) of hers. Terry drank \(\frac{7}{10}\) of his. Who drank less than \(\frac{1}{2}\) of their juice?
Options:
a. Larry
b. Mary
c. Mary and Terry
d. Larry and Terry

Answer: Mary

Explanation:
Larry, Mary, and Terry each had a full glass of juice.
Larry drank \(\frac{3}{4}\), Mary drank \(\frac{3}{8}\) and Terry drank \(\frac{7}{10}\) of \(\frac{1}{2}\)
\(\frac{3}{8}\) is less than \(\frac{1}{2}\) of their juice.
The correct answer is Option B.

Question 5.
Which of the following statements is NOT true about the numbers 7 and 9?
Options:
a. 7 is a prime number.
b. 9 is a composite number.
c. 7 and 9 have no common factors other than 1.
d. 27 is a common multiple of 7 and 9.

Answer: 27 is a common multiple of 7 and 9

Explanation:
a. 7 is a prime number is true.
b. 9 is a composite number is true
c. 7 and 9 have no common factors other than 1 is true.
d. 27 is a common multiple of 7 and 9 is not true because 7 is not the multiple of 27.
Thus the correct answer is option D.

Question 6.
Tom and some friends went to a movie. The show started at 2:30 P.M. and ended at 4:15 P.M. How long did the movie last?
Options:
a. 1 hour 35 minutes
b. 1 hour 45 minutes
c. 1 hour 55 minutes
d. 2 hours 15 minutes

Answer: 1 hour 45 minutes

Explanation:
Tom and some friends went to a movie. The show started at 2:30 P.M. and ended at 4:15 P.M.
Subtract 2:30 P.M. from 4:15 P.M.
4 hour 15 minutes
-2 hour 30 minutes
1 hour 45 minutes
The movie last for 1 hour 45 minutes
Thus the correct answer is option B.

Page No. 749

Question 1.
For numbers 1a–1e, select Yes or No to indicate if a rectangle with the given dimensions would have a perimeter of 50 inches.
a. length: 25 inches; width: 2 inches
i. yes
ii. no

Answer: No

Explanation:
P = (2 × l) + (2 × w)
50 inches = (2 × 25 in.) + (2 × w)
(2 × w) = 50 inches – 50 inches
w = 0
Thus the above statement is false

Question 1.
b. length: 20 inches; width: 5 inches
i. yes
ii. no

Answer: Yes

Explanation:
P = (2 × l) + (2 × w)
50 inches = (2 × 20 in.) + (2 × 5)
50 inches = 40 in. + 10 in.
Thus the above statement is true.

Question 1.
c. length: 17 inches; width: 8 inches
i. yes
ii. no

Answer: Yes

Explanation:
P = (2 × l) + (2 × w)
50 inches = (2 × 17 in.) + (2 × 8 in.)
50 inches = 34 in. + 16 in.
Thus the above statement is true.

Question 1.
d. length: 15 inches; width: 5 inches
i. yes
ii. no

Answer: No

Explanation:
P = (2 × l) + (2 × w)
50 inches = (2 × 15 in.) + (2 × 5 in.)
50 inches = 30 in. + 10 in.
50 inches = 40 inches
Thus the above statement is false.

Question 1.
e. length: 15 inches; width: 10 inches
i. yes
ii. no

Answer: Yes

Explanation:
P = (2 × l) + (2 × w)
50 inches = (2 × 15 in.) + (2 × 10 in.)
50 inches = 30 in. + 20 in.
50 inches = 50 inches
Thus the above statement is true.

Question 2.
The swimming club’s indoor pool is in a rectangular building.
Marco is laying tile around the rectangular pool.
Go Math Grade 4 Answer Key Chapter 13 Algebra Perimeter and Area img 82
Part A
What is the area of the pool and the area of the pool and the walkway? Show your work.
A(pool) = ____ m2    A(building) = ____ m2

Answer:
Pool:
Base = 20 m
Height = 16 m
A = b × h
Area of the pool = 20 m × 16 m = 320 square meters
Pool and the walkway:
Area of the pool and the walkway = 26 m × 22 m = 572 square meters

Question 2.
Part B
How many square meters of tile will Marco need for the walkway?
Explain how you found your answer.
A(walkway) = ____ m2

Answer: 252 square meters

Explanation:
Area of walkway = Area of the pool and the walkway – Area of pool
Area of the walkway = 572 square meters – 320 square meters
= 252 square meters
Therefore the Area of walkway = 252 square meters

Page No. 750

Question 3.
Match the dimensions of the rectangles in the top row with the correct area or perimeter in the bottom row
Go Math Grade 4 Answer Key Chapter 13 Algebra Perimeter and Area img 83

Answer:

Go-Math-Grade-4-Answer-Key-Chapter-13-Algebra-Perimeter-and-Area-img-83

Question 4.
Kyleigh put a large rectangular sticker on her notebook. The height of the sticker measures 18 centimeters. The base is half as long as the height. What area of the notebook does the sticker cover?
________ square centimeters

Answer: 162 square centimeters

Explanation:
Kyleigh put a large rectangular sticker on her notebook.
The height of the sticker measures 18 centimeters.
The base is half as long as the height.
Base = h/2 = 18/2 = 9 centimeters
Area of the rectangle = b × h
A = 9 cm × 18 cm
A = 162 square centimeters
Thus the area of the notebook the sticker cover is 162 square centimeters.

Question 5.
A rectangular flower garden in Samantha’s backyard has 100 feet around its edge. The width of the garden is 20 feet. What is the length of the garden? Use the numbers to write an equation and solve. A number may be used more than once.
Go Math Grade 4 Answer Key Chapter 13 Algebra Perimeter and Area img 84
□ = (2 × l) + (2 × □)
□ = 2 × l + □
□ = 2 × l
□ = l
So, the length of the garden _____ feet.

Answer:
P = (2 × l) + (2 × w)
100 = (2 × l) + (2 × 20)
100 – 40 = 2 × l
2 × l = 60
l = 60/2 = 30 feet
Length = 30 feet
So, the length of the garden 30 feet.

Question 6.
Gary drew a rectangle with a perimeter of 20 inches. Then he tried to draw a square with a perimeter of 20 inches.
Draw 3 different rectangles that Gary could have drawn. Then draw the square, if possible.
Type below:
__________

Answer:
The possible rectangles with a perimeter of 20 inches are:
Go Math Grade 4 Chapter 13 Answer Key review solution image-1HMH Grade 4 Go Math Answer Key review solution image-2Go Math 4th Grade Solution Key Review solution image-3
The possible square with a perimeter of 20 inches is:
Go Math Grade 4 Chapter 13 solution key review solution image-4

Page No. 751

Question 7.
Ami and Bert are drawing plans for rectangular vegetable gardens. In Ami’s plan, the garden is 13 feet by 10 feet. In Bert’s plan, the garden is 12 feet by 12 feet. For numbers 7a−7d, select True or False for each statement.
a. The area of Ami’s garden is 130 square feet.
i. True
ii. False

Answer: True

Explanation:
A = b × h
Area of Ami’s garden = 13 feet × 10 feet =
Area of Ami’s garden = 130 square feet
The above statement is true.

Question 7.
b. The area of Bert’s garden is 48 square feet.
i. True
ii. False

Answer: False

Explanation:
Area of Bert’s garden = 12 feet × 12 feet = 144 square feet
The above statement is false.

Question 7.
c. Ami’s garden has a greater area than Bert’s garden.
i. True
ii. False

Answer: False

Explanation:
Area of Ami’s garden = 13 feet × 10 feet = 130 square feet
Area of Bert’s garden = 12 feet × 12 feet = 144 square feet
130 square feet is less than 144 square feet
The area of Ami’s garden is less than Area of Bert’s garden.
The above statement is false.

Question 7.
d. The area of Bert’s garden is 14 square feet greater than Ami’s.
i. True
ii. False

Answer: True

Explanation:
Area of Ami’s garden = 13 feet × 10 feet = 130 square feet
Area of Bert’s garden = 12 feet × 12 feet = 144 square feet
144 square feet – 130 square feet = 14 square feet
The above statement is true.

Question 8.
A farmer planted corn in a square field. One side of the field measures 32 yards. What is the area of the cornfield? Show your work.
_______ square yards

Answer: 1024 square yards

Explanation:
A farmer planted corn in a square field. One side of the field measures 32 yards.
Area of the square = 32 yards × 32 yards
A = 1,024 square yards
Therefore the area of the cornfield is 1,024 square yards.

Question 9.
Harvey bought a frame in which he put his family’s picture.
Go Math Grade 4 Answer Key Chapter 13 Algebra Perimeter and Area img 85
What is the area of the frame not covered by the picture?
_______ square inches

Answer: 136 square inches

Explanation:
Area of the picture = 12 in. × 18 in.
A = 216 square inches
Area of the frame = 16 in. × 22 in.
A = 352 square inches
The area of the frame not covered by the picture = 352 square inches – 216 square inches
= 136 square inches
Therefore the area of the frame not covered by the picture is 136 square inches.

Question 10.
Kelly has 236 feet of fence to use to enclose a rectangular space for her dog. She wants the width to be 23 feet. Draw a rectangle that could be the space for Kelly’s dog. Label the length and width.
Type below:
________

Answer:

Kelly has 236 feet of fence to use to enclose a rectangular space for her dog.
She wants the width to be 23 feet.
Perimeter = (2 × l) + (2 × w)
236 = (2 × l) + (2 × w)
236 = (2 × l) + (2 × 23)
236 – 46 = (2 × l)
(2 × l) = 190HMH Go Math Answer Key Grade 4 Chapter 13 review solution img- 5
l = 190/2
l = 95 feet
Therefore length = 95 feet.

Page No. 752

Question 11.
The diagram shows the dimensions of a new parking lot at Helen’s Health Food store.
Go Math Grade 4 Answer Key Chapter 13 Algebra Perimeter and Area img 86
Use either addition or subtraction to find the area of the parking lot. Show your work.
_______ square yards

Answer: 1100 square yards

Explanation:
Addition:
Top:
Base = 40 yards
Height = 20 yards
Area of the top rectangle = b × h
A = 40 yards × 20 yards = 800 square yards
Bottom:
Base = 30 yards
Height = 10 yards
Area of the rectangle = b × h
A = 30 yards × 10 yards = 300 square yards
Area of the parking = Area of top + Area of bottom
A = 800 square yards + 300 square yards
Area of parking = 1100 square yards.

Question 12.
Chad’s bedroom floor is 12 feet long and 10 feet wide. He has an area rug on his floor that is 7 feet long and 5 feet wide. Which statement tells how to find the amount of the floor that is not covered by the rug? Mark all that apply.
Options:
a. Add 12 × 10 and 7 × 5.
b. Subtract 35 from 12 × 10
c. Subtract 10 × 5 from 12 × 7.
d. Add 12 + 10 + 7 + 5.
e. Subtract 7 × 5 from 12 × 10.
f. Subtract 12 × 10 from 7 × 5.

Answer: B, F

Chad’s bedroom floor is 12 feet long and 10 feet wide.
A = 12 feet × 10 feet = 120 square feet
Area rug on his floor = 7 feet × 5 feet = 35 square feet
To find the amount of the floor that is not covered by the rug we have to subtract 120 square feet from 35 square feet or 35 square feet from 12 × 10.
So, the correct answers are B and F.

Question 13.
A row of plaques covers 120 square feet of space along a wall. If the plaques are 3 feet tall, what length of the wall do they cover?
____ feet

Answer: 40 feet

Explanation:
Given that,
A row of plaques covers 120 square feet of space along a wall.
Height = 3 feet
A = b × h
120 square feet = b × 3 feet
b = 120/3 = 40
Therefore the base is 40 feet.

Page No. 753

Question 14.
Ms. Bennett wants to buy carpeting for her living room and dining room.
Go Math Grade 4 Answer Key Chapter 13 Algebra Perimeter and Area img 87
Explain how she can find the amount of carpet she needs to cover the floor in both rooms. Then find the amount of carpet she will need.
____ square feet

Answer:
She can find the area of each rectangle and then find the sum. The area of the living room is 20 × 20 = 400 square feet.
The area of the dining room is 15 × 10 = 150 square feet.
The sum of the two rooms = 400 + 150 = 550 square feet.
She needs 550 square feet of carpeting.

Question 15.
Lorenzo built a rectangular brick patio. He is putting a stone border around the edge of the patio. The width of the patio is 12 feet. The length of the patio is two feet longer than the width.
How many feet of stone will Lorenzo need? Explain how you found your answer.
____ feet

Answer: 52 feet

Explanation:
Width = 12 feet
Length = 2 × width
Length = 2 + 12 feet = 14 feet
Perimeter = (2 × l) + (2 × w)
P = (2 × 14) + (2 × 12)
P = 28 + 24
P = 52 feet

Page No. 754

Question 16.
Which rectangle has a perimeter of 10 feet? Mark all that apply.
Go Math Grade 4 Answer Key Chapter 13 Algebra Perimeter and Area img 88
Rectangle: ____
Rectangle: ____

Answer: A, C

Explanation:
i. Perimeter of A = (2 × l) + (2 × w)
P = (2 × 1) + (2 × 4) = 2 + 8 = 10 feet
ii. Perimeter of B = (2 × l) + (2 × w)
P = (2 × 2) + (2 × 5) = 4 + 10 = 14 feet
iii. Perimeter of C = (2 × l) + (2 × w)
P = (2 × 2) + (2 × 3) = 4 + 6 = 14 feet
iv. Perimeter of D = (2 × l) + (2 × w)
P = (2 × 4) + (2 × 6) = 8 + 12 = 20 feet
The correct answer is option A and C.

Question 17.
A folder is 11 inches long and 8 inches wide. Alyssa places a sticker that is 2 inches long and 1 inch wide on the notebook. Choose the words that correctly complete the sentence.
To find the number of square inches of the folder that is NOT covered by the sticker,
Go Math Grade 4 Answer Key Chapter 13 Algebra Perimeter and Area img 89
Type below:
________

Answer: Subtract the area of the sticker from the area of the notebook.

Question 18.
Tricia is cutting her initial from a piece of felt. For numbers 18a–18c, select Yes or No to tell whether you can add the products to find the number of square centimeters Tricia needs.
Go Math Grade 4 Answer Key Chapter 13 Algebra Perimeter and Area img 90
a. 1 × 8 and 5 × 2 _______
b. 3 × 5 and 1 × 8 _______
c. 2 × 5 and 1 × 3 and 1 × 3 _______

Answer:
a. 1 × 8 and 5 × 2 _______
Yes
b. 3 × 5 and 1 × 8 _______
No
c. 2 × 5 and 1 × 3 and 1 × 3 _______
No

Question 19.
Mr. Butler posts his students’ artwork on a bulletin board.
Go Math Grade 4 Answer Key Chapter 13 Algebra Perimeter and Area img 91
The width and length of the bulletin board are whole numbers. What could be the dimensions of the bulletin board Mr. Butler uses?
Type below:
________

Answer: 5 feet long by 3 feet wide
Area of the rectangle = l × w
A = 15 square feet
The factor of 15 is 5 and 3.
So, the length = 5 feet long
Width = 3 feet long.

Quick learning is not only important but also understanding is important to learn the concepts. You can’t love maths if you don’t understand the subject. So, to help you guys we have provided the images for your better understanding. Learn the simple techniques to solve the problem in less time in our Go Math Answer Key.

Conclusion:

We wish you all have satisfied with the solutions exists in the Go Math Grade 4 Answer Key Chapter 13 Algebra: Perimeter and Area. For better practice sessions refer to the questions given at the end of the chapter and solve them properly with the help of topic-wise chapter 13 Go Math 4th Grade Answer Key. Practice all problems easily and score well in any standard tests or exams.

Go Math Grade 3 Answer Key Chapter 1 Addition and Subtraction within 1,000

go-math-grade-3-chapter-1-addition-and-subtraction-within-1-000-answer-key

Become a master in maths taking the help of Go Math Grade 3 Answer Key Chapter 1 Addition and Subtraction within 1000. Begin your preparation and learn all the fundamental topics in it. Solve all the Problems in Chapter 1 easily from here and understand the concept behind them. Download Go Math Grade 3 Answer Key Chapter 1 Addition and Subtraction within 1,000 free of cost and learn the fundamentals easily.

Go Math Grade 3 Answer Key Chapter 1 Addition and Subtraction within 1,000

You can make the most out of the Grade 3 Go Math Solutions Key Chapter 1 through the quick links available. Simply tap on the topic you want and learn various questions involved in it easily. There are different models in addition and subtraction. We have listed all of them by considering enough examples and explained every problem step by step.

Lesson 1: Number Patterns 

Lesson 2: Round to the Nearest Ten

Lesson 3: Estimate Sums

Lesson 4: Mental Math Strategies for Addition

Lesson 5: Use Properties to Add

Lesson 6: Use the Break Apart Strategy to Add

Lesson 7: Use Place Value to Add

Mid Chapter Check Point

Lesson 8: Estimate Differences

Lesson 9: Mental Math Strategies for Subtraction

Lesson 10: Use Place Value to Subtract

Lesson 11: Combine Place Values to Subtract

Lesson 12: Problem Solving • Model Addition and Subtraction

Review/Test

Number Patterns – Page No. 9

Find the sum. Then use the Commutative Property of Addition to write the related addition sentence.
Question 1:
9 + 2 =  11 

Answer:

What is the commutative property of addition?
To “commute” means to move around or travel.
According to the commutative property of addition, changing the order of the numbers we are adding, does not change the sum.
if you are adding nine and two together, the commutative property of addition says that you will get the same answer whether you are adding
9 + 2 or 2 + 9.
2 + 9 =  11 

Question 2:
4 + 7 = 
    +      =  11

Answer:

If you are adding four and seven together, the commutative property of addition says that you will get the same answer whether you are adding 4 + 7 or 7 + 4.
4 + 7 =  11
7  +  4  =  11

Question 3:
3 + 6 =
     +      =  11

Answer:

According to the commutative property of addition, changing the order of the numbers we are adding, does not change the sum.
There will be no change in the sum whether you add 3 + 6 or 6 + 3.
3 + 6 =  9
 6  +  3  =  9

Question 4:
3 + 10 =
     +      =  11

Answer:

According to the commutative property of addition, changing the order of the numbers we are adding, does not change the sum.
There will be no change in the sum whether you add 3 + 10 or 10 + 3 = 13.
3 + 10 =  13
 10  +  3  =  13

Question 5:
6 + 7 =
     +      =  13

Answer:

According to the commutative property of addition, changing the order of the numbers we are adding, does not change the sum.
There will be no change in the sum whether you add 6 + 7 or 7 + 6 = 13.
6 + 7 =  13
 7  +  6  =  13

Question 6:
7 + 5 =
     +      =  12

Answer:

If you are adding five and seven together, the commutative property of addition says that you will get the same answer whether you are adding 7 + 5 or 5 + 7 = 12
7 + 5 =  12
 5  +  7  =  12

Question 7:
8 + 9 =
     +      =  17

Answer:

If you are adding eight and nine together, the commutative property of addition says that you will get the same answer whether you are adding 8 + 9 or 9 + 8.
8 + 9 =  17
 9  +  8  =  17

Question 8:
0 + 4 =
     +      =  4

Answer:

According to the commutative property of addition, changing the order of the numbers we are adding, does not change the sum.
0 + 4 =  4
 4  +  0  =  4

Question 9:
9 + 6 =
     +      =  15

Answer:

According to the commutative property of addition, changing the order of the numbers we are adding, does not change the sum.
There will be no change in the sum whether you add 9 + 6 or 6 + 9 = 15
9 + 6 =  15
 6  +  9  =  15

Is the sum even or odd? Write even or odd.

Even Numbers:

Any integer that can be divided exactly by 2 is an even number. The last digit is 0, 2, 4, 6 or 8
Example: −24, 0, 6 and 38 are all even numbers

Odd Numbers:

Any integer that cannot be divided exactly by 2 is an odd number. The last digit is 1, 3, 5, 7 or 9
Example: −3, 1, 7 and 35 are all odd numbers
Odd numbers are in between the even numbers.

Question 10:
5 + 2

Answer:

The sum of two odd numbers is an odd number.
5 + 2 = 7.
∴ 7 is an odd number.

Question 11:
6 + 4

Answer:

The sum of two even numbers is always an even number.
6 + 4 = 10.
∴ 10 is an even number.

Question 12:
1 + 0

Answer:
The Sum of any number with zero is always the same number.
1 + 0 = 1.
∴ 1 is an odd number.

Question 13:
5 + 5

Answer:
Any integer that can be divided exactly by 2 is an even number.
5 + 5 = 10.
∴ 10 is an even number.

Question 14:
3 + 8

Answer:

The sum of an even and odd number is an odd number.
3 + 8 = 11.
∴ 11 is an odd number.

Question 15:
7 + 7

Answer:

7 + 7 = 14.
∴ 14 is an even number.

Question 16:
Ada writes 10 + 8 = 18 on the board. Maria wants to use the Commutative Property of Addition to rewrite Ada’s addition sentence. What number sentence should Maria write?

Answer:

According to the commutative property of addition, changing the order of the numbers we are adding, does not change the sum.
If you are adding ten and eight together, the commutative property of addition says that you will get the same answer whether you are adding 10 + 8 or 8 + 10.
Maria should write 8 + 10 =18.

Question 17:
Jackson says he has an odd number of model cars. He has 6 cars on one shelf and 8 cars on another shelf. Is Jackson correct? Explain.

Answer:

Jackson has 6 cars on one shelf and 8 cars on another shelf, hence the total number of cars Jackson has = sum of the cars on both shelves = 6 + 8 = 14.
14 is an even number ends with ‘4’ in the last digit.
Given Statement is False, Jackson has an even number of model cars i.e. 14.

Number Patterns Lesson Check Page No 10

Question 1
Marvella says that the sum of her addends is odd. Which of the following could be Marvella’s addition problem?

Options:
(a) 5 + 3
(b) 9 + 7
(c) 2 + 8
(d) 5 + 6

Answer:

(a) 5 + 3 = 8 (Even Number)
(b) 9 + 7 = 16 (Even Number)
(c) 2 + 8 = 10 (Even Number)
(d) 5 + 6 = 11 (Odd Number)

Option (d) is Correct.

Question 2
Which number sentence shows the Commutative Property of Addition?
3 + 9 = 12

(a) 12 – 9 = 3
(b) 12 = 8 + 4
(c) 9 + 3 = 12
(d) 12 – 3 = 9

Answer:

3 + 9 = 12 and 9 + 3 = 12 are commutative.
Option (c) is correct.

Spiral Review
Question 3
Amber has 2 quarters, a dime, and 3 pennies. How much money does Amber have?

(a) 53 ¢
(b) 58 ¢
(c) 63 ¢
(d) 68 ¢

Answer:

Amber has
2 quarters = 2 x 25 = 50 ¢
1 dime = 10 ¢
3 pennies = 3 ¢
∴ Money does Amber have = 2 quarters + 1 dime + 3 pennies
= 50 ¢ + 10 ¢ + 3 ¢ = 63 ¢
Option (c) is correct.

Question 4
Josh estimates the height of his desk. Which is the best estimate?

(a) 1 foot
(b) 2 feet
(c) 5 feet
(d) 9 feet

Answer:

So, from the given choices, 2 feet or approximately 24 inches would be the most viable answer because most standard desks have a height around 28 inches to 30 inches. 1 foot is too low, while 5 feet and 9 feet are too high. Therefore, 3 to 4 feet would be the most perfect answer but since we have a limited number of options, the nearest reasonable estimate value would be 2 feet. The original numbers from a problem does not exceed in a reasonable estimate.

Option (b) is correct.

Use the bar graph for 5–6.

Go Math Grade 3 Answer Key Chapter 1 Number Patterns Page 10

Question 5
Who read the most books?

(a) Alicia
(b) Bob
(c) Juan
(d) Maria

Answer:

The number of Books Read:
Juan – 6
Bob – 2
Maria – 4
Alicia – 5
Juan Read the most books.
Option (c) is correct.

Question 6
Who read 3 more books than Bob?

(a) Alicia
(b) Juan
(c) Maria
(d) no one

Answer:

Alicia read 3 books.
Option (a) is correct.

Lesson 2: Round to the Nearest Ten Page 15

Round to the Nearest Ten or Hundred

Locate and label 739 on the number line.
Round to the nearest hundred.

Go Math Grade 3 Chapter 1 Round to the Nearest Ten or Hundred Page 15 Answer Key

Question 1

(i) 739 is between __ and __

Answer:

739 is between 700 and 800.

Question 2

739 is closer to       than it is to      .

Answer:

739 is closer to  700 than it is to  800.

Round to the nearest ten and hundred.

Round to the nearest ten

Rounding Numbers to the nearest 10 means finding which 10 they are nearest to. For example, 68 rounded to the nearest 10 is 70.

Rule for rounding to the nearest 10

Look at the number in the one’s place and…

Rule for rounding to the nearest 10

Work through the examples below that show rounding to the nearest 10.

Rounding numbers to the nearest 10

Round to the nearest hundred

Rounding numbers to the nearest 100 means finding which 100 they are nearest to. For example, 680 rounded to the nearest 100 is 700.

Rule for rounding to the nearest 100

Look at the number in the tens’ place and…

Rule for rounding to the nearest 10

Work through the examples below that show rounding to the nearest 100.

Rounding numbers to the nearest 100

Question 3
739 rounded to the nearest hundred is

Answer:

Let’s round 739 to the nearest 100.
The nearest 100’s on both sides of 739 are 700 and 800.
700 is the nearest 100 to 739.
∴ 739 rounded to the nearest hundred is ‘700’

Question 4
363
Round to nearest ten: 
Round to nearest hundred:      

Answer:

(i) Let’s round 363 to the nearest 10.

The nearest 10’s on both sides of 363 are 360 and 370.
360 is the nearest 10 to 363.
∴ 363 rounded to the nearest ten is ‘360’
Round to nearest ten: 360

(ii) Let’s round 363 to the nearest 100.

The nearest 100’s on both sides of 363 are 300 and 400.
400 is the nearest 100 to 363.
∴ 363 rounded to the nearest hundred is ‘400’
Round to nearest hundred: 400

Question 5
829
Round to nearest ten:     
Round to nearest hundred:      

Answer:

(i) Lets round 829 to the nearest 10.
The nearest 10’s on both sides of 829 is 820 and 830.
Round to nearest ten: 830

(ii) Let’s round 829 to the nearest 100
The nearest 100’s on both sides of 829 is 800 and 900.
The number rounded to 829 nearest to 100 is 800.
Round to nearest hundred: 800

Question 6
572
Round to nearest ten:     
Round to nearest hundred:      

Answer:

(i) Lets round 572 to the nearest 10.
The nearest 10’s on both sides of 572 is 560 and 570.
The number rounded to 572 is 570.
Round to nearest ten: 570

(ii) Let’s round 572 to the nearest 100
The nearest 100’s on both sides of 572 is 500 and 600.
The number rounded to 572 nearest to 100
Round to nearest hundred: 600

Question 7
209
Round to nearest ten: 
Round to nearest hundred:      

Answer:

(i) Lets round 209 to the nearest 10.
The nearest 10’s on both sides of 209 is 200 and 210.
The number rounded to Round to nearest ten: 210

(ii) Let’s round 209 to the nearest 100
The nearest 100’s on both sides of 209 are 200 and 300.
The number rounded to 209 nearest to 100 is 200.
Round to nearest hundred: 200

Question 8
663
Round to nearest ten: 
Round to nearest hundred:      

Answer:

(i) Lets round 663 to the nearest 10.
The nearest 10’s on both sides of 663 are 660 and 670.
Round to nearest ten: 660

(ii) Let’s round 663 to the nearest 100
The nearest 100’s on both sides of 663 are 600 and 700.
The number rounded to 663 nearest to 100 is 700.
Round to nearest hundred: 700

Question 9
949
Round to nearest ten: 
Round to nearest hundred:      

Answer:

(i) Lets round 949 to the nearest 10.
The nearest 10’s on both sides of 949 is 940 and 950.
Round to nearest ten: 950

(ii) Let’s round 949 to the nearest 100
The nearest 100’s on both sides of 949 are 900 and 1000.
The number rounded to 949 nearest to 100 is 900.
Round to nearest hundred: 900

Question 10
762
Round to nearest ten: 
Round to nearest hundred:      

Answer:

(i) Lets round 762 to the nearest 10.
The nearest 10’s on both sides of 762 is 760 and 770.
Round to nearest ten:  760

(ii) Let’s round 762 to the nearest 100
The nearest 100’s on both sides of 762 are 700 and 800.
The number rounded to 762 nearest to 100 is 800.
Round to nearest hundred:  800 

Question 11
399
Round to nearest ten: 
Round to nearest hundred:      

Answer:

(i) Lets round 399 to the nearest 10.
The nearest 10’s on both sides of 399 is 390 and 400.
Round to nearest ten: 400

(ii) Let’s round 399 to the nearest 100
The nearest 100’s on both sides of 399 are 300 and 400.
The number rounded to 399 nearest to 100 is 400
Round to nearest hundred: 400

Question 12
402
Round to nearest ten: 
Round to nearest hundred:      

Answer:

(i) Lets round 402 to the nearest 10.
The nearest 10’s on both sides of 402 is 400 and 410.
Round to nearest ten: 400

(ii) Let’s round 402 to the nearest 100
The nearest 100’s on both sides of 402 are 400 and 500.
The number rounded to 402 nearest to 100 is 400.
Round to nearest hundred: 400

Problem Solving

Question 13
The baby elephant weighs 435 pounds. What is its weight rounded to the nearest hundred pounds?
          pounds

Answer:

Let’s round 435 to the nearest hundred pounds.
The nearest 100’s on both sides of 435 are 400 and 500.
400 is the nearest 100 to 435.
∴ 435 rounded to the nearest hundred pounds is ‘400’
Round to nearest hundred: 400 pounds

Question 14
Jayce sold 218 cups of lemonade at his lemonade stand. What is 218 rounded to the nearest ten?
          cups

Answer:

Let’s round 218 to the nearest 10.
The nearest 10’s on both sides of 218 are 210 and 220.
218 is the nearest 10 to 220.
∴ 218 rounded to the nearest ten is ‘220’
Round to nearest ten: 220

Lesson 2: Round to the Nearest Ten Lesson Check Page No 16

Lesson Check

Question 1
One day, 758 people visited the Monkey House at the zoo. What is 758 rounded to the nearest hundred?

(a) 700
(b) 760
(c) 800
(d) 860

Answer:
Let’s round 758 to the nearest hundred.
The nearest hundred on both sides is 700 and 800.
∴ 758 rounded to the nearest hundred is 800.
So, the answer is option C.

Question 2
Sami ordered 132 dresses for her store. What is 132 rounded to the nearest ten?

(a) 100
(b) 130
(c) 140
(d) 200

Answer:
Let’s round 132 to the nearest 10.
The nearest 10’s on both sides of 132 are 130 and 140.
∴ 132 rounded to the nearest ten is ‘130’
132 rounded to the nearest ten: 130
Option B is the correct answer.

Spiral Review
Question 3
Which describes the number sentence?
6 + 0 = 6

(a) Commutative Property of Addition
(b) Identity Property of Addition
(c) even + odd = odd
(d) odd + odd = odd

Answer:
The Identity Property of Zero, also called the Additive Identity Property, states that if you add 0 to any number, the result will be that number. Likewise, if you subtract 0 from any number, the result will be that number.
6 + 0 = 6
∴ Option B is the correct answer.

Question 4
Which has an even sum?

(a) 7 + 4
(b) 2 + 6
(c) 5 + 4
(d) 3 + 2

Answer:
The sum of even numbers is always even.
(a) 7 + 4 = 11 is odd number
(b) 2 + 6 = 8 is even number
(c) 5 + 4 = 9 is odd number
(d) 3 + 2 = 5 is odd number
So, the answer is option (b)

Question 5
What name describes this shape?

HMH Go Math Answer Key Grade 3 Chapter 1 image_1

(a) cone
(b) cube
(c) rectangle
(d) triangle

Answer: Triangle
∴ Option D is the correct answer.

Question 6
What word describes the equal shares of the shape?

Go Math Answer Key Grade 3 Chapter 1 image_2

(a) wholes
(b) thirds
(c) halves
(d) fourths

Answer:
The rectangle is divided into 4 equal rectangles.
So, the name for the equal shapes is fourths.
∴ The answer is option D.

Estimate Sums Page No – 21

Compatible Numbers:

Compatible numbers are the numbers that are easy to compute mentally and are close to the real numbers.

Use rounding or compatible numbers to estimate the sum.

Question 1
198 + 727 =         

Answer:

200 +725 = 925

Explanation:

Step 1:
First round 198 to the nearest hundred.
The number rounded to 198 nearest hundred is 200.
Write zeros for the tens and ones digit.

Step 2:
Write the number closer to 727.
The number closer to 727 is 725.

Step 3:
Now find the sum of the rounded numbers.
200 + 725 = 925

Question 2
87 + 34

Estimate:

         +         =        

Answer: 90 + 30 = 120

Explanation:

Step 1:
First round 87 to the nearest ten.
The number rounded to 87 nearest ten is 90.
Write zeros for the ones digit.

Step 2:
Write the number closer to 34.
The number closer to 34 is 30.

Step 3:
Now find the sum of the rounded numbers.
90 + 30 = 120

Question 3
222 + 203

Estimate:

         +         =        

Answer: 200 +200 = 400

Explanation:

Step 1:
First round 222 to the nearest hundred.
The number rounded to 222 nearest hundred is 200.
Write zeros for the tens and ones digit.

Step 2:
Write the number closer to 203.
The number closer to 203 is 200.

Step 3:
Now find the sum of the rounded numbers.
200 + 200 = 400
The estimated sum of 222 + 203 is 400.

Question 4
52 + 39

Estimate:

         +         =        

Answer: 50 + 40 = 90

Explanation:

Step 1:
First round 52 to the nearest ten.
The number rounded to 52 nearest ten is 50.
Write zeros for the ones digit.

Step 2:
Write the number closer to 39.
The number closer to 39 is 40.

Step 3:
Now find the sum of the rounded numbers.
50 + 40 = 90

Question 5
256 + 321

Estimate:

         +         =        

Answer: 250 + 325 = 575

Explanation:

Step 1:
First round 256 to the nearest ten.
The number rounded to 256 nearest ten is 250.
Write zeros for the ones digit.

Step 2:
Write the number closer to 321.
The number closer to 321 is 325.

Step 3:
Now find the sum of the rounded numbers.
250 + 325 = 575
The estimated sum of 256 + 321 is 575.

Question 6
302 + 412

Estimate:

         +         =        

Answer: 300 + 400 = 700

Step 1:
First round 302 to the nearest ten.
The number rounded to 302 nearest ten is 300.
Write zeros for the ones digit.

Step 2:
Write the number closer to 412.
The number closer to 412 is 400.

Step 3:
Now find the sum of the rounded numbers.
300 + 400 = 700

Question 7
519 + 124

Estimate:

         +         =        

Answer: 500 + 100 = 600

Explanation:

Step 1:
First round 519 to the nearest hundred.
The number rounded to 519 nearest hundred is 500.
Write zeros for the tens and ones digit.

Step 2:
Write the number closer to 124.
The number closer to 124 is 100.

Step 3:
Now find the sum of the rounded numbers.
500 + 100 = 600
The estimated sum is 600.

Question 8
790 + 112

Estimate:

         +         =        

Answer: 800 + 100 = 900

Explanation:

Step 1:
First round 790 to the nearest hundred.
The number rounded to 790 nearest hundred is 800.
Write zeros for the tens and ones digit.

Step 2:
Write the number closer to 112.
The number closer to 112 is 100.

Step 3:
Now find the sum of the rounded numbers.
800 + 100 = 900
The estimated sum of 790 + 112 is 900.

Question 9
547 + 326

Estimate:

         +         =        

Answer: 550 + 325 = 875

Explanation:

Step 1:
First round 547 to the nearest ten.
The number rounded to 547 nearest ten is 550.
Write zeros for the ones digit.

Step 2:
Write the number closer to 326.
The number closer to 326 is 325.

Step 3:
Now find the sum of the rounded numbers.
550 + 325 = 875

Question 10
325 + 458

Estimate:

         +         =        

Answer: 325 + 500 = 825

Explanation:

First round 458 to the nearest hundred.
The number rounded to 458 nearest hundred is 500.
Write zeros for the tens and ones digit.
Now add 325 and 500,
You get, 325 + 500 = 825

Question 11
620 + 107

Estimate:

         +         =        

Answer: 600 + 100 = 700

Explanation:

The number closer to 620 is 600.
And the number closer to 107 is 100.
600 + 100 = 700
Now the estimated sum of 620 + 107 = 700

Problem Solving
Question 12
Stephanie read 72 pages on Sunday and 83 pages on Monday. About how many pages did Stephanie read during the two days?

About         pages

Answer: 150 pages

Explanation:

Stephanie read 72 pages on Sunday and 83 pages on Monday.
The number rounded to 72 is 70 and the number rounded to 83 is 80.
To know how many pages he read in two days we need to add the number of pages he read.
70 + 80 = 150 pages.
Therefore Stephanie read about 150 pages in 2 days.

Question 13
Matt biked 345 miles last month. This month he has biked 107 miles. Altogether, about how many miles has Matt biked last month and this month?

About         miles

Answer: About 450 miles

Explanation:

Matt biked 345 miles last month.
This month he has biked 107 miles.
The number rounded to 345 is 350.
And the number closer to 107 is 100.
Now add number of miles he rides in last month and this month.
350 + 100 = 450 miles.

Estimate Sums Lesson Check – Page No – 22

Lesson Check
Question 1
The McBrides drove 317 miles on one day and 289 on the next day. What is the best estimate of the number of miles the McBrides drove in all during the two days?

(a) 100
(b) 400
(c) 500
(d) 600

Answer: 600

Explanation:

The McBrides drove 317 miles on one day and 289 on the next day.
First, round 317 to the nearest hundred.
The number rounded to 317 nearest hundred is 300.
Write zeros for the tens and ones digit.
Next round 289 to the nearest hundred.
The number rounded to 289 nearest hundred is 300.
Write zeros for the tens and ones digit.
300 +300 = 600.
Option D is the correct answer.

Question 2
Ryan counted 63 birds in his backyard last week. This week, he counted 71 birds in his backyard. About how many birds did Ryan count in all?

(a) about 70
(b) about 100
(c) about 130
(d) about 200

Answer: about 130

Explanation:

Ryan counted 63 birds in his backyard last week. This week, he counted 71 birds in his backyard.
The number closer to 63 is 60.
The number closer to 71 is 70.
Now add 60 and 70 we get 130.
Therefore Ryan count about 130 birds.
So, the correct answer is option C.

Spiral Review
Question 3
What name describes this shape?

Go Math Grade 3 Chapter 1 Round to the Nearest Ten or Hundred Page 22 What name describes this shape

(a) cone
(b) cube
(c) quadrilateral
(d) square

Answer: cube

A cube is a three-dimensional solid object bounded by six square faces, facets or sides, with three meeting at each vertex. The cube is the only regular hexahedron and is one of the five Platonic solids. It has 6 faces, 12 edges, and 8 vertices.

Question 4
Which has an odd sum?

(a) 9 + 9
(b) 5 + 3
(c) 6 + 7
(d) 2 + 8

Answer: 6 + 7

Explanation:

(a) 9 + 9 = 18 even number
(b) 5 + 3 = 8 even number
(c) 6 + 7 = 13 odd number
(d) 2 + 8 = 10 even number
So, the answer is option C.

Question 5
What is 503 rounded to the nearest hundred?

(a) 500
(b) 510
(c) 600
(d) 610

Answer: 500

The number 503 rounded to the nearest hundred is 500.
So, the correct answer is option A.

Question 6
What is 645 rounded to the nearest ten?

(a) 600
(b) 640
(c) 650
(d) 700

Answer: 650

645 rounded to the nearest ten is 650.
So, the correct answer is option C.

Mental Math Strategies for Addition Page No – 27

Count by tens and ones to find the sum.

Use the number line to show your thinking.

Question 1

Go Math Grade 3 Chapter 1 Count by tens and ones to find the sum Question 1

Answer: 29 + 14 = 43

Question 2

Go Math Grade 3 Chapter 1 Count by tens and ones to find the sum Question 2

36 + 28 =

Answer: 36 + 28 = 64

Question 3

Go Math Grade 3 Chapter 1 Count by tens and ones to find the sum Question 2

45 + 26 =

Answer: 45 + 26 = 71

Question 4

Go Math Grade 3 Chapter 1 Count by tens and ones to find the sum Question 2

52 + 34 =

Answer: 52 + 34 = 86

Use mental math to find the sum.

Draw or describe the strategy you use.

Question 5
52 + 19 =

Answer: 52 + 19 = 71

I Used friendly numbers.
Subtract 2 from 52.
52 – 2 = 50
Then add 2 to 19.
19 + 2 = 21
Add 50 and 21
50 + 21 = 71

Question 6
122 + 306 =

Answer: 122 + 306 = 428

I Used compatible numbers
122 = 120 + 2
306 = 300 + 6
120 + 300 = 420
2 + 6 = 8
420 + 8 = 428

Problem Solving
Question 7
Shelley spent 17 minutes washing the dishes. She spent 38 minutes cleaning her room. Explain how you can use mental math to find how long Shelley spent on the two tasks

        minutes

Answer: 20 + 35 = 55 minutes

Explanation:

Shelley spent 17 minutes washing the dishes.
She spent 38 minutes cleaning her room.
First, make friendly numbers.
Add 3 to 17 to make it easy for addition
17 + 3 = 20
Next, subtract 3 from 38.
38 – 3 = 35
Now add both, 35 + 20 = 55
Shelly spent 55 minutes on the two tasks.

Question 8
It took Marty 42 minutes to write a book report. Then he spent 18 minutes correcting his report. Explain how you can use mental math to find how long Marty spent on his book report.

        minutes

Answer: 50 + 10 = 60 minutes

Explanation:

It took Marty 42 minutes to write a book report.
Then he spent 18 minutes correcting his report.
Make a friendly number
Subtract 2 from 42
42 – 2 = 40 minutes
Now add 2 to 18 minutes
18 + 2 = 20 minutes
Now add both, 20 + 40 = 60 minutes
Therefore Marty spent 60 minutes on his book report

Mental Math Strategies for Addition Page No – 28

Lesson Check
Question 1
Sylvia spent 36¢ for a pencil and 55¢ for a notepad. Use mental math to find how much she spent in all.

(a) 80¢
(b) 81¢
(c) 90¢
(d) 91¢

Answer: 91¢

Explanation:

Sylvia spent 36¢ for a pencil and 55¢ for a notepad.
Step 1:
Make a friendly number
Add 36¢ and 55¢
36¢ + 55¢ = 91¢
So, the correct answer is option D.

Question 2
Will spent 24 minutes putting together a model plane. Then he spent 48 minutes painting the model. How long did Will spend working on the model plane?

(a) 62 minutes
(b) 68 minutes
(c) 72 minutes
(d) 81 minutes

Answer: 72 minutes

Explanation:

Will spent 24 minutes putting together a model plane. Then he spent 48 minutes painting the model.
Add 24 and 48
24 + 48 = 72 minutes
Option C is the correct answer.

Spiral Review

Question 3
What name describes this shape?

Go Math Grade 3 Chapter 1 What name describes this shape

(a) hexagon
(b) pentagon
(c) quadrilateral
(d) triangle

Answer: pentagon

Explanation:

From the figure, we can observe that there are 5 sides. A pentagon is a 5-sided Polygon
So, the correct answer is option B.

Question 4
What word describes the equal shares of the shape?

Go Math Grade 3 Chapter 1 What word describes the equal shares of the shape

(a) fourths
(b) halves
(c) sixths
(d) thirds

Answer: fourths

Explanation:

The circle is divided into 4 equal parts. The name for the equal shares of circle is fourths.

Question 5
Tammy wrote an addition problem that has an odd sum. Which could be Tammy’s addition problem?

(a) 2 + 6
(b) 3 + 5
(c) 5 + 6
(d) 7 + 7

Answer: 5 + 6

Explanation:

(a) 2 + 6 = 8 even number
(b) 3 + 5 = 8 even number
(c) 5 + 6 = 11 odd number
(d) 7 + 7 = 14 even number
11 is an odd number. So, the correct answer is option C.

Question 6
Greg counted 83 cars and 38 trucks in the mall parking lot. Which is the best estimate of the total number of cars and trucks Greg counted?

(a) 100
(b) 110
(c) 120
(d) 130

Answer: 120

Explanation:

Greg counted 83 cars and 38 trucks in the mall parking lot.
The number closer to 83 is 80.
And the number closer to 38 is 40.
80 + 40 = 120.
So, the correct answer is option C.

Use Properties to Add Page No 33

Use addition properties and strategies to find the sum.

Question 1
Go Math Grade 3 Chapter 1 Use Properties of Add Question 1

Question 2
27 + 68 + 43 =

Answer: 138

Explanation:

Step 1:
Line up the numbers by place value.
27
68
+43

Step 2:
Group the ones to make them easy to add.
Make a 10
27
68
+43

7 + 3 = 10
1 will be carried to tens place.
8 will be in the ones place.

Step 3:
Group the tens to make them easy to add.
27
68
+43
6 + 4 = 10
10 + 3 = 13

27 + 68 + 43 = 138

Question 3
42 + 36 + 18 =

Answer: 96

Explanation:

Step 1:
Line up the numbers by place value.

42
36
+18

8 + 2 = 10
1 will be carried to the tens place
6 will be in the ones place.

Step 2:
Group the ones to make them easy to add.
Make a 10
42
36
+18

40 + 30 + 10 + 10 = 90

Step 3:
Group the tens to make them easy to add.
90 + 6 = 96

Question 4
74 + 35 + 16 + 45 =

Answer: 170

Explanation:

Step 1:
Line up the numbers by place value.
74
35
16
+45

Step 2:
Group the ones to make them easy to add.
Make a 10
74
35
16
+45

6 + 4 = 10
5 + 5 = 10
10 + 10 = 20
2 will be carried to tens place
0 will be in the ones place.

Step 3:
Group the tens to make them easy to add.
70 + 30 + 10 + 40 +20 = 170

Question 5
41 + 26 + 149 =

Answer: 216

Explanation:

Step 1:
Line up the numbers by place value.

149
41
+26

Step 2:
Group the ones to make them easy to add.
Make a 10

149
41
+26

9 + 1 = 10
1 will be carried to tens place.
6 will be in the ones place.

Step 3:
Group the tens to make them easy to add.
140 + 40 + 20 + 10 = 210
210 + 6 = 216

Question 6
52 + 64 + 28 + 44 =

Answer: 188

Explanation:

Step 1:
Line up the numbers by place value.
52
64
28
+44

Step 2:
Group the ones to make them easy to add.
Make a 10

52
64
28
+44

8 + 2 = 10
4 + 4 = 8
1 will be carried to the tens place.
8 will be in the ones place.

Step 3:
Group the tens to make them easy to add.
50 + 60 + 20 + 40 + 10 = 180
180 + 8 = 188

Problem Solving
Question 7
A pet shelter has 26 dogs, 37 cats, and 14 gerbils. How many of these animals are in the pet shelter in all?

        animals

Answer: 77 animals

Explanation:

Given that, A pet shelter has 26 dogs, 37 cats, and 14 gerbils.
Add the total number of pets
26 + 37 + 14 = 77 animals.

Question 8
The pet shelter bought 85 pounds of dog food, 50 pounds of cat food, and 15 pounds of gerbil food. How many pounds of animal food did the pet shelter buy?

        pounds

Answer: 150 pounds

Explanation:

Step 1:
Line up the numbers by place value.

85
50
+15

Step 2:
Group the ones to make them easy to add.
Make a 10

85
50
+15

5 + 5 = 10

Step 3:
Group the tens to make them easy to add.
80 + 50 + 10 + 10 = 150

Use Properties to Add Page No 34

Lesson Check
Question 1
At summer camp there are 52 boys, 47 girls, and 18 adults. How many people are at summer camp?

(a) 97
(b) 107
(c) 117
(d) 127

Answer: 117

Explanation:

At summer camp there are 52 boys, 47 girls, and 18 adults.
57
47
+18

52 + 47 + 18 = 117
Therefore 117 people are at summer camp.
The correct answer is option C.

Question 2
At camp, 32 children are swimming, 25 are fishing, and 28 are canoeing. How many children are swimming, fishing, or canoeing?

(a) 75
(b) 85
(c) 95
(d) 105

Answer: 85

Explanation:

At camp, 32 children are swimming, 25 are fishing, and 28 are canoeing.
32
25
+28

Make a group of 10.

32
25
+28

8 + 2 = 10
1 will be carried to the tens place.
5 will be in the ones place.
30 + 20 + 20 + 10 = 80
80 + 5 = 85
The correct answer is option B.

Spiral Review
Question 3
Four students estimated the width of the door to their classroom. Who made the best estimate?

(a) Ted: 1 foot
(b) Hank: 3 feet
(c) Ann: 10 feet
(d) Maria: 15 feet

Answer: Hank: 3 feet

Question 4
Four students estimated the height of the door to their classroom. Who made the best estimate?

(a) Larry: 1 meter
(b) Garth: 2 meters
(c) Ida: 14 meters
(d) Jill: 20 meters

Answer: Garth: 2 meters

Question 5
Jeff’s dog weighs 76 pounds. What is the dog’s weight rounded to the nearest ten pounds?

(a) 70 pounds
(b) 80 pounds
(c) 90 pounds
(d) 100 pounds

Answer: 80 pounds

Explanation:

Jeff’s dog weighs 76 pounds.
76 rounded to the nearest ten is 80.
The correct answer is option B.

Question 6
Ms. Kirk drove 164 miles in the morning and 219 miles in the afternoon. Which is the best estimate of the total number of miles she drove that day?

(a) 100 miles
(b) 200 miles
(c) 400 miles
(d) 500 miles

Answer: 400 miles

Explanation:

Ms. Kirk drove 164 miles in the morning and 219 miles in the afternoon.
The number closer to 164 is 200.
The number closer to 219 is 200.
Now add the total number of mile
200 + 200 = 400 miles.
The correct answer is option C.

Use the Break Apart Strategy to Add Page No 39

Estimate. Then use the break apart strategy to find the sum.

Question 1

Question 2
518 + 372

Estimate: 900

Sum:
518 = 500 + 10 + 8
+372 = 300 + 70 + 2
890     800 + 80 + 10

Question 3
473 + 123

Estimate: 600

Sum:
473 = 400 + 70 + 3
123 = 100 + 20 + 3
596 = 500 + 90 + 6

Question 4
208 + 569

Estimate: 800

Sum:
208 = 200 + 00 + 8
569 = 500 + 60 + 9
777 = 700 + 70 + 7

Question 5
731 + 207

Estimate: 900

Sum:
731 = 700 + 30 + 1
207 = 200 + 00 + 7
938 = 900 + 30 + 8

Question 6
495 + 254

Estimate: 800

Sum:
495 = 400 + 90 + 5
254 = 200 + 50 + 4
749 = 700 + 40 + 9

Problem Solving
Use the table for 7–8.

Go Math Grade 3 Chapter 1 Problem Solving

Question 7
Laura is making a building using Set A and Set C. How many blocks can she use in her building?

        blocks

Answer: 410 blocks

Add set A and Set C
165 + 245 = 410 blocks

165 = 100 + 60 +5
245 = 200 + 40 + 5
410 = 300 + 100 + 10
She can use 410 blocks in her building.

Question 8
Clark is making a building using Set B and Set C. How many blocks can he use in his building?

        blocks

Answer: 433 blocks

Add Set B and Set C
188 + 245 =

188 = 100 + 80 + 8
245 = 200 + 40 + 5
433 = 300 + 120 + 13
He can use 433 blocks in his building.

Use the Break Apart Strategy to Add Page No 40

Lesson Check
Question 1
Arthur read two books last week. One book has 216 pages. The other book has 327 pages. Altogether, how many pages are in the two books?

(a) 533
(b) 543
(c) 633
(d) 643

Answer: 543

Explanation:

Add 216 and 327
216 = 200 + 10 + 6
327 = 300 + 20 + 7
543 = 500 + 30 + 13

So, the correct answer is option B.

Question 2
One skeleton in a museum has 189 bones. Another skeleton has 232 bones. How many bones in all are in the two skeletons?

(a) 311
(b) 312
(c) 411
(d) 421

Answer: 421

Explanation:

Add 189 and 232
189 = 100 + 80 + 9
232 = 200 + 30 + 2
421 = 300 + 110 + 11
Thus the answer is option D.

Spiral Review
Question 3
Culver has 1 quarter, 3 dimes, and a penny. How much money does he have?

(a) 41¢
(b) 55¢
(c) 56¢
(d) 86¢

Answer: 56¢

Explanation:

1 quarter = $0.25
1 dime = $0.10
3 dimes = $0.10 × 3 = $0.30
1 penny = $0.01
Add $0.25 + $0.30 + $0.01 = $0.56 = 56 cents
Thus the correct answer is option C.

Question 4
Felicia has 34 quarters, 25 dimes, and 36 pennies. How many coins does Felicia have?

(a) 75
(b) 85
(c) 95
(d) 105

Answer: 95

Explanation:

1 quarter = $0.25
34 quarters = $0.25 × 34 = $8.5
25 dimes = $0.10 × 25 = $2.5
36 pennies = $0.01 × 36 = 0.36
Option C is the correct answer.

Question 5
Jonas wrote 9 + 8 = 17. Which number sentence shows the Commutative Property of Addition?

(a) 9 + 0 = 9
(b) 8 + 9 = 17
(c) 17 – 9 = 8
(d) 17 – 8 = 9

Answer: 8 + 9 = 17

According to the commutative property of addition, changing the order of the numbers we are adding, does not change the sum.
So, the correct answer is option B.

Question 6
At Kennedy School there are 37 girls and 36 boys in the third grade. How many students are in the third grade at Kennedy School?

(a) 63
(b) 73
(c) 81
(d) 91

Answer: 73

Explanation:

Given that,
At Kennedy School there are 37 girls and 36 boys in the third grade.
Add number of girls and boys = 37 + 36 = 73
Therefore the correct answer is option B.

Use Place Value to Add Page No 45

Estimate. Then find the sum.

Question 1
Estimate: 600

324 + 285 = 609

324
285
609

Question 2
519  + 347

Estimate: 500 + 300 = 800

Sum: 519 + 347
519
347
866

Question 3
323 + 151

Estimate: 323 + 151 = 325 + 150= 475

Sum:

323
151
474

Question 4
169 + 354

Estimate: 150 + 350 = 500

Sum:

169
354
523

Question 5
148 + 285

Estimate: 150 + 300 = 450

Sum: 148 + 285 = 433

148
285
433

Question 6
270 + 453

Estimate: 300 + 450 = 750

Sum: 270 + 453 = 723

270
453
723

Question 7
275 + 116

Estimate: 275 + 100 = 375

Sum:

275
116
391

Question 8
157 + 141

Estimate: 150 + 150 = 300

Sum:

157
141
298

Question 9
127 + 290

Estimate: 100 + 300 = 400

Sum:

127
290
417

Question 10
258 + 565

Estimate: 250 + 550 = 800

Sum:

258
565
823

Question 11
311 + 298

Estimate: 300 + 300 = 600

Sum:

311
298
609

Question 12
534 + 256

Estimate: 550 + 250 = 800

Sum:

534
256
790

Problem Solving
Question 13
Mark has 215 baseball cards. Emily has 454 baseball cards. How many baseball cards do Mark and Emily have altogether?

        cards

Answer: 669 baseball cards.

Explanation:

Given,
Mark has 215 baseball cards.
Emily has 454 baseball cards.
Total number of baseball cards = 215 + 454 = 669
Therefore there are 669 baseball cards.

Question 14
Jason has 330 pennies. Richie has 268 pennies. Rachel has 381 pennies. Which two students have more than 700 pennies combined?

Answer: Jason and Rachel

Explanation:

Jason has 330 pennies. Richie has 268 pennies. Rachel has 381 pennies.
The rounded number of 330 is 300.
The number closer to 268 is 300
The number rounded to 381 is 400.
You will get 700 when to combine the pennies of Jason and Rachel

Lesson Check Page No 46

Question 1
There are 167 students in the third grade. The same number of students is in the fourth grade. How many third graders and fourth graders are there in all?

(a) 224
(b) 234
(c) 324
(d) 334

Answer: 334

Explanation:

Given that there are 167 students in the third grade.
The same number of students is in the fourth grade.
That means there are 167 students in the fourth grade.
To find the total number of students in third grade and fourth grade
You need to add 167 and 167
167 + 167 = 334.
Thus the correct answer is option D.

Question 2
Jamal read a book with 128 pages. Then he read a book with 179 pages. How many pages did Jamal read in all?

(a) 397
(b) 307
(c) 297
(d) 207

Answer: 307

Explanation:

Jamal read a book with 128 pages. Then he read a book with 179 pages.
128 + 179 = 307
So, the answer is option B.

Spiral Review
Question 3
Adam travels 248 miles on Monday. He travels 167 miles on Tuesday. Which is the best estimate for the total number of miles Adam travels?

(a) 200
(b) 300
(c) 400
(d) 500

Answer: 400

Explanation:

Adam travels 248 miles on Monday. He travels 167 miles on Tuesday.
The number closer to 248 is 200
And the number closer to 167 is 200.
200 + 200 = 400
Thus the estimated number of miles Adam travels is 400.

Question 4
Wes made $14, $62, $40, and $36 mowing lawns. How much did he make in all mowing lawns?

(a) $116
(b) $152
(c) $166
(d) $188

Answer: $152

Explanation:

Add
14
62
40
+36
152
Thus the correct answer is option B.

Question 5
There are 24 students in Mrs. Cole’s class and 19 students in Mr. Garmen’s class. How many students in all are in the two classes?

(a) 43
(b) 40
(c) 33
(d) 5

Answer: 43

Add 24 and 19
24 + 19 = 43
Thus the correct answer is option A.

Question 6
There were 475 children at the baseball game on Sunday. What is 475 rounded to the nearest ten?

(a) 400
(b) 470
(c) 480
(d) 500

Answer: 480

Explanation:

There were 475 children at the baseball game on Sunday.
475 rounded to the nearest ten is 480.
So, the answer is option C.

Mid Chapter Check Point – Vocabulary Page No 47

Choose the best term from the box.

Go Math Grade 3 Chapter 1 Choose the best term from the box.

Question 1
A ________ is an ordered set of numbers or objects in which the order helps you predict what comes next.

Answer: Pattern is an ordered set of numbers or objects in which the order helps you predict what comes next.

Question 2
The _________ states that when you add zero to any number, the sum is that number.

Answer: Identity property of Addition states that when you add zero to any number, the sum is that number

Concepts and Skills
Is the sum even or odd? Write even or odd.

Question 3
8 + 3

Answer: 8 + 3 = 11 is an odd number.

Question 4
9 + 7

Answer: 9 + 7 = 16 is an even number

Question 5
4 + 6

Answer: 4 + 6 = 10 is an even number

Use rounding or compatible numbers to estimate the sum.

Question 6
56+32

Estimate:

         +         =        

Answer:
The number which is compatible to 56 is 50.
The number compatible to 32 is 25
50
25
75

50 + 25 = 75

Question 7
271+425

Estimate:

         +         =        

Answer:

The number close to 271 is 275
425 will be the same.

275
425
700
425 + 275 = 700

Question 8
328+127

Estimate:

         +         =        

Answer:

The number closer to 328 is 325
The number closer to 127 is 125
325 + 125 = 450

Use mental math to find the sum.

Question 9
46 + 14 =

Answer: 60

Explanation:

Break apart the addends to make them compatible
46 = 40 + 6
14 = 10 + 4
Now add both
40 + 6
10 + 4
50 + 10 = 60
46 + 14 = 60

Question 10
39 + 243 =

Answer: 282

Explanation:

Break apart the addends to make them compatible
39 = 35 + 4
243 = 240 + 3
Now add
240 + 3
  35 + 4
275 + 7

275 + 7 = 282
39 + 243 = 282

Question 11
326 + 402 =

Answer: 728

Explanation:

Break apart the addends to make them compatible.
326 = 325 + 1
402 = 400 + 2
Now add

325 + 1
400 + 2
725 + 3 = 728
326 + 402 = 728

Estimate. Then find the sum.

Question 12
356+442
Estimate: 800
Sum: 798

Answer:

356
442
798
The sum of 356 and 442 is 798
The number close to 798 is 800.
Therefore the estimated sum is 800.

Question 13
164+230
Estimate: 400
Sum: 394

Answer:

230
164
394

The sum of 230 and 164 is 394
The number 394 rounded to the nearest hundred is 400.
Thus the estimated sum is 400.

Question 14
545+139
Estimate: 700
Sum: 684

Answer:

545
139
684

The sum of 545 and 139 is 684.
684 rounded to the nearest hundred is 700.
So, the estimated difference is 700.

Question 15
437+184
Estimate: 600
Sum: 621

Answer:

437
184
621

The sum of 437 and 184 is 621
621 rounded to the nearest hundred is 600.
The estimated sum is 600.

Mid Chapter Check Point – Vocabulary Page No 48

Question 16
Nancy planted 77 daisies, 48 roses, and 39 tulips. About how many roses and tulips did she plant?

about         roses and tulips

Answer: 90 roses and tulips

Explanation:

Given that, Nancy planted 77 daisies, 48 roses, and 39 tulips.
To know how many roses and tulips did she plant
We have to add a number of roses and a number of tulips.
48 and 39.
The number closer to 48 is 50.
And the number closer to 39 is 40.
So, the estimated sum is 90.

Question 17
Tomas collected 139 cans for recycling on Monday, and twice that number on Tuesday. How many cans did he collect on Tuesday?

        cans

Answer: 278 cans

Explanation:

Tomas collected 139 cans for recycling on Monday, and twice that number on Tuesday.
Twice is nothing but double.
139 + 139 = 278
Therefore Tomas collected 278 cans on Tuesday.

Question 18
There are 294 boys and 332 girls in the Hill School. How many students are in the school?

        students

Answer: 626 students

Explanation:

Given,
There are 294 boys and 332 girls in the Hill School.
To find the total number of students in the students
We need to add the total number of boys and number of girls = 294 + 332
294
332
626
So, there are 626 students in the class.

Question 19
Monday’s art group made 25 paper models. Tuesday’s group made 32 paper models. Wednesday’s group made 15 paper models. How many paper models did the groups make?

        paper models

Answer: 72 paper models

Explanation:

Monday’s art group made 25 paper models.
Tuesday’s group made 32 paper models.
Wednesday’s group made 15 paper models.
Add 25, 32 and 15
25
32
15
72

Estimate Differences Page No 53

Use rounding or compatible numbers to estimate the difference.

Question 1
40 – 13 = 
40 – 10
Estimate: 30

Question 2
762 – 332

Estimate:

         –          =        

Estimate: 500

The number closer to 762 is 800
The number closer to 332 is 300
The difference between 800 and 300 is 500

Question 3
823 – 242

Estimate:

         –          =        

Estimate: 550

The number 823 rounded to the nearest hundred is 800.
The number closer to 242 is 250
800
-250
550

Question 4
98 – 49

Estimate:

         –          =        

Estimate: 50

The number closer to 98 is 100
The round number of 49 is 50.
100
-50
50

Question 5
287 – 162

Estimate:

         –          =        

Estimate: 100

282 rounded to the nearest hundred is 300
162 rounded to the nearest hundred is 200
300
-200
100

Question 6
359 – 224

Estimate:

         –          =        

Estimate: 125

The number closer to 359 is 350
The number closer to 224 is 225
350
-225
125

Question 7
68 – 31

Estimate:

         –          =        

Estimate: 40
The round number of 68 is 70
The number closer to 31 is 30
70
-30
40

Question 8
476 – 155

Estimate:

         –          =        

Estimate: 325

The number closer to 476 is 475
The number closer to 155 is 150
475
-150
325

Question 9
622 – 307

Estimate:

         –          =        

Estimate: 300

622 nearest to the hundred is 600
307 nearest to the hundred is 300
600
-300
300

Question 10
771 – 531

Estimate:

         +         =        

Estimate: 225

The number closer to 771 is 775
531 nearest to ten is 550
775
550
225

Question 11
299 – 61

Estimate:

         +         =        

Estimate: 240

The number closer to 299 is 300
The number closer to 61 is 60
300
-60
240

Problem Solving

Question 12
Ben has a collection of 812 stamps. He gives his brother 345 stamps. About how many stamps does Ben have left?

About         stamps

Answer: About 450 stamps

Explanation:

Ben has a collection of 812 stamps. He gives his brother 345 stamps.
812 to the nearest hundred is 800
345 to the nearest ten is 350
800
-350
450
Thus about 450 stamps are left.

Question 13
Danika is making necklaces. She has 512 silver beads and 278 blue beads. About how many more silver than blue beads does Danika have?

About         more silver than blue beads

Answer: About 200 more silver than blue beads

Explanation:

Given,
Danika is making necklaces.
She has 512 silver beads and 278 blue beads.
The number closer to 512 is 500
278 to the nearest hundred is 300
The difference between 500 and 300 is 200.
Therefore Danika has about 200 more silver than blue beads.

Lesson Check Page No 54

Question 1
Jorge has 708 baseball cards and 394 basketball cards. About how many more baseball cards than basketball cards does Jorge have?

(a) about 200
(b) about 300
(c) about 400
(d) about 500

Answer: about 300

Explanation:

Jorge has 708 baseball cards and 394 basketball cards.
The number closer to 708 is 700.
The number closer to 394 is 400
700
-400
300
So, the correct answer is option A.

Question 2
Danika is making necklaces. She has 512 silver beads and 278 blue beads. About how many more silver than blue beads does Danika have?

(a) about 200
(b) about 300
(c) about 400
(d) about 800

Answer: about 200

Explanation:

Danika is making necklaces. She has 512 silver beads and 278 blue beads.
The number closer to 512 is 500
278 to the nearest hundred is 300
The difference between 500 and 300 is 200.
Therefore Danika has about 200 more silver than blue beads.
So, the correct answer is option A.

Spiral Review
Question 3
A store manager ordered 402 baseball caps and 122 ski caps. Which is the best estimate of the total number of caps the manager ordered?

(a) 300
(b) 500
(c) 600
(d) 700

Answer: 500

Explanation:

A store manager ordered 402 baseball caps and 122 ski caps.
To find the best estimate of the total number of caps the manager ordered
We have to add baseball caps and ski caps.
The number closer to 402 is 400
The number closer to 122 is 100.
400 + 100 = 500
So, the correct answer is option B.

Question 4
Autumn collected 129 seashells at the beach. What is 129 rounded to the nearest ten?

(a) 100
(b) 120
(c) 130
(d) 200

Answer: 130

Explanation:

Autumn collected 129 seashells at the beach.
129 rounded to the nearest ten is 130
So, the correct answer is option C.

Question 5
Find the sum.

585 + 346

(a) 239
(b) 821
(c) 900
(d) 931

Answer: 931

585
+346
931
The correct answer is option D.

Question 6
Julie made $22, $55, $38, and $25 babysitting. How much did she make in all babysitting?

(a) $102
(b) $115
(c) $140
(d) $165

Answer: $140

Explanation:

Julie made $22, $55, $38, and $25 babysitting.
Put all the numbers in the order
22
55
38
+25
140
So, the correct answer is option C.

Mental Math Strategies for Subtraction Page No – 59

Use mental math to find the difference.
Draw or describe the strategy you use.
Question 1:
74 – 39 = 35

Go Math Grade 3 Chapter 1 Mental Math Strategies for Subtraction

Question 2
93 – 28 =

Answer: 65

I use friendly numbers.
Add 2 to the 93.
93 + 2 = 95
Add 2 to 28
28 + 2 = 30
95 – 30 = 65

Question 3
51 – 9 =

Answer: 42
I used friendly numbers to subtract 9 from 51.
Now add 1 to 9
9 + 1 = 10
Now subtract 10 from 51
51 – 10 = 41
Now add 1 to 41
41 + 1 = 42

Question 4
76 – 23 =

Answer:

I used friendly numbers
Subtract 1 from 76
76 – 1 = 75
75 – 23 = 52
Now add 1 to 52
52 + 1 = 53.
76 – 23 = 53

Question 5
357 – 214 =

Answer:

I use break apart strategy.
300 – 200 = 100
50 – 10 = 40
7 – 4 = 3
100 + 40 + 3 = 143
357 – 214 = 143

Question 6
285 – 99 =

Answer:

I used friendly numbers.
The number close to 99 is 100
285 – 100 = 185
Now add 1 to 185
185 + 1 = 186
285 – 99 = 186

Problem Solving
Question 7
Ruby has 78 books. Thirty-one of the books are on shelves. The rest are still packed in boxes. How many of Ruby’s books are still in boxes?

        books

Answer: 47 books

Explanation:

Ruby has 78 books. Thirty-one of the books are on shelves. The rest are still packed in boxes.
To know the remaining books in the boxes.
Subtract 31 from 78.
78 – 31
70 – 30 = 40
8 – 1 = 7
40 + 7 = 47 books
Therefore 47 of Ruby’s books are still in boxes.

Question 8
Kyle has 130 pins in his collection. He has 76 of the pins displayed on his wall. The rest are in a drawer. How many of Kyle’s pins are in a drawer?

        pins

Answer: 54 pins

Explanation:

Kyle has 130 pins in his collection.
He has 76 of the pins displayed on his wall. The rest are in a drawer.
130 – 76 = 54
Thus there are 54 pins in a drawer.

Lesson Check Page No – 60

Question 1
One day, a baker made 54 fruit pies. At the end of the day, only 9 of the pies were NOT sold. How many pies were sold that day?

(a) 43
(b) 45
(c) 63
(d) 65

Answer: 45

Explanation:

Given,
One day, a baker made 54 fruit pies.
At the end of the day, only 9 of the pies were NOT sold.
Number of pies sold that day = x
x + 9 = 54
x = 54 – 9 = 45
x = 45
Therefore the number of pies sold that day = 45
So, the correct answer is option B.

Question 2
George’s father bought a 50-pound bag of wild bird seed. At the end of two weeks, 36 pounds of seed were left in the bag. How many pounds of seed had been used?

(a) 14 pounds
(b) 24 pounds
(c) 26 pounds
(d) 86 pounds

Answer: 14 pounds

Explanation:

George’s father bought a 50-pound bag of wild bird seed.
At the end of two weeks, 36 pounds of seed were left in the bag.
Number of pounds used = x
x + 36 = 50
x = 50 – 36
x = 14
Therefore George’s father used 14 pounds.
The correct answer is option A.

Spiral Review
Question 3
For a party, Shaun blew up 36 red balloons, 28 white balloons, and 24 blue balloons. How many balloons did he blow up in all?

(a) 78
(b) 81
(c) 87
(d) 88

Answer: 88

Explanation:

For a party, Shaun blew up 36 red balloons, 28 white balloons, and 24 blue balloons.
Total number of balloons = 36 + 28 + 24
36
28
+24
88
So, the answer is option D.

Question 4
Tiffany has read 115 pages of her book. She has 152 pages left to read. How many pages are in the book?

(a) 37
(b) 267
(c) 277
(d) 367

Answer: 267

Explanation:;

Tiffany has read 115 pages of her book.
She has 152 pages left to read.
Total number of pages = 152 + 115
152
+115
267

Question 5
The flower shop had 568 flowers on Monday. By Tuesday, the shop had 159 flowers left. About how many flowers had been sold?

(a) about 200
(b) about 300
(c) about 400
(d) about 500

Answer: about 400

Explanation:

The flower shop had 568 flowers on Monday.
By Tuesday, the shop had 159 flowers left.
The number closer to 568 is 600.
The number closer to 159 is 200
Subtract 200 from 600.
600 – 200 = 400
The correct answer is option C.

Question 6
There are 383 books in one section of the school library. Of the books, 165 are fiction books. Which is the best estimate of the number of books in that section that are NOT fiction?

(a) about 200
(b) about 300
(c) about 400
(d) about 500

Answer: about 200

Explanation:

There are 383 books in one section of the school library.
Of the books, 165 are fiction books.
383 to the nearest hundred is 400.
165 to the nearest hundred is 200
400 – 200 = 200
So, the correct answer is option A.

Use Place Value to Subtract Page No 65

Estimate. Then find the difference.

Question 1
Estimate: 500

585 – 119

Subtract 119 from 585

585
119
466
585 – 119 = 466

Question 2
738 – 227

Estimate: 500

Difference: 511

Subtract 227 from 738
738
227
511
The estimated difference of 511 is 500.
738 – 227 = 511

Question 3
651 – 376
Estimate: 300
Difference: 275

Subtract 376 from 651
651
376
275
651 – 376 = 275
The estimated difference is 300

Question 4
815 – 281
Estimate: 500 
Difference: 534

Subtract 281 from 815

815
281
534
815 – 281 = 534
The estimated difference is 500

Question 5
487 – 290

Estimate: 200
Difference: 197

487
290
197
487 – 290 = 197
The estimated difference is 200.

Question 6
936 – 329

Estimate: 600
Difference: 607

936
329
607
936 – 329 = 607
The estimated difference is 600.

Question 7
270 – 128

Estimate: 140
Difference: 142

Subtract 128 from 270
270
128
142
270 – 128 = 142
The estimated difference is 140.

Question 8
364 – 177

Estimate: 200
Difference: 187

Subtract 177 from 364
364
177
187
364 – 177 = 187
The estimated difference is 200.

Question 9
627 – 253

Estimate: 400
Difference: 374

Subtract 253 from 627
627
253
374
627 – 253 = 374
The estimated difference is 374

Question 10
862 – 419

Estimate: 450
Difference: 443

Subtract 419 from 862
862
419
443
862 – 419 = 443
The estimated difference is 450.

Question 11
726 – 148

Estimate: 550
Difference: 578

Subtract 148 from 726
726
148
578
726 – 148 = 578
The estimated difference is 550.

Question 12
543 – 358

Estimate: 200
Difference: 185

Subtract 358 from 543
543
358
185
543 – 358 = 185
The estimated difference is 200.

Problem Solving
Question 13
Mrs. Cohen has 427 buttons. She uses 195 buttons to make puppets. How many buttons does Mrs. Cohen have left?

        buttons

Answer: 232 buttons

Explanation:

Mrs. Cohen has 427 buttons.
She uses 195 buttons to make puppets.
To find how many buttons left, we have to subtract number of buttons she used to make puppets from the total number of buttons.
427 – 195 = 232
Therefore 232 buttons are left.

Question 14
There were 625 ears of corn and 247 tomatoes sold at a farm stand. How many more ears of corn were sold than tomatoes?

        more ears

Answer: 378 more ears of corn

Explanation:

There were 625 ears of corn and 247 tomatoes sold at a farm stand.
To know number of ears of corn were sold than tomatoes we have to subtract 247 from 625
625
-247
378
Thus 378 more ears of corn were sold than tomatoes.

Use Place Value to Subtract Page No 66

Question 1
On Saturday, 453 people go to a school play. On Sunday, 294 people go to the play. How many more people go to the play on Saturday?

(a) 159
(b) 169
(c) 259
(d) 747

Answer: 159

Explanation:

On Saturday, 453 people go to a school play. On Sunday, 294 people go to the play.
To find how many more people go to the play on Saturday
We need to subtract number of people go to the play on Sunday from the number of people go to the play on Saturday
= 453 – 294 = 159
159 more people go to the play on Saturday.
Thus the correct answer is option A.

Question 2
Corey has 510 marbles. He fills one jar with 165 marbles. How many of Corey’s marbles are NOT in the jar?

(a) 675
(b) 455
(c) 350
(d) 345

Answer: 345

Explanation:

Corey has 510 marbles.
He fills one jar with 165 marbles.
Let the number of Corey’s marbles are NOT in the jar be x
x + 165 = 510
x = 510 – 165
x = 345
Therefore 345 marbles are NOT in the jar.
The correct answer is option D.

Spiral Review
Question 3
Pattie brought 64 peppers to sell at the farmers’ market. There were 12 peppers left at the end of the day. How many peppers did Pattie sell?

(a) 50
(b) 52
(c) 62
(d) 78

Answer: 52

Explanation:

Pattie brought 64 peppers to sell at the farmers’ market.
There were 12 peppers left at the end of the day.
To find number of peppers did Pattie sell
Subtract 12 from 64
64 – 12 = 52
The correct answer is option B.

Question 4
An airplane flies 617 miles in the morning. Then it flies 385 miles in the afternoon. About how many more miles does the airplane fly in the morning?

(a) about 100 miles
(b) about 200 miles
(c) about 300 miles
(d) about 900 miles

Answer: about 200 miles

Explanation:

An airplane flies 617 miles in the morning.
Then it flies 385 miles in the afternoon.
Here we have to use the concept of estimated difference.
The number closer to 617 is 600
The number closer to 385 is 400
600 – 400 = 200
About 200 miles airplane fly in the morning.
So, the correct answer is option B.

Question 5
What is the unknown number?

(■ + 4) + 59 = 70

(a) 4
(b) 6
(c) 7
(d) 8

Answer: 7

Explanation:

Let ■ be the unknown number
(■ + 4) + 59 = 70
(■ + 4) = 70 – 59
(■ + 4) = 11
(■ = 11 – 4
■ = 7
Thus the correct answer is option C.

Question 6
Dexter has 128 shells. He needs 283 more shells for his art project. How many shells will Dexter use for his art project?

(a) 155
(b) 165
(c) 401
(d) 411

Answer: 411

Explanation:

Dexter has 128 shells. He needs 283 more shells for his art project.
To know the total number of shells that Dexter used for his art project
you need to add 128 and 283
283 + 128 = 411
So, the correct answer is option D.

Combine Place Values to Subtract Page No – 71

Estimate. Then find the difference.

Question 1
Estimate: 200

476 – 269

476
-269
207
The estimated difference is 200.

Question 2
615 – 342

Estimate: 300
Difference: 273

615
-342
273
The difference between 615 and 342 is 273
The estimated difference is 300.

Question 3
508 – 113

Estimate: 400
Difference: 395

508
-113
395
The difference between 508 and 113 is 395
The estimated difference is 400

Question 4
716 – 229

Estimate: 500
Difference: 487

716
229
487
The number closer to 487 is 500.
The difference is 487.

Question 5
700 – 326

Estimate: 400
Difference: 374

700
326
374
The number closer to 374 is 400.
The difference is 374.

Question 6
325 – 179

Estimate: 100
Difference: 146

325
179
146
The number closer to 146 is 100
The difference is 146.

Question 7
935 – 813

Estimate: 100
Difference: 122

935
813
122
The number closer to 122 is 100.
The difference is 122.

Question 8
358 – 292

Estimate: 50
Difference: 66

358
292
66
The number closer to 66 is 50.
The difference is 66.

Question 9
826 – 617

Estimate: 200
Difference: 209

826
617
209
The number closer to 209 is 200.
The difference is 209.

Question 10
900 – 158

Estimate: 750
Difference: 742

900
158
742
The number closer to 742 is 750.
The difference is 742

Question 11
607 – 568

Estimate: 40
Difference: 39

607
568
39
The number closer to 39 is 40.
The difference is 40.

Question 12
973 – 869

Estimate: 100

Difference: 104

973
869
104
The number closer to 104 is 100.
The difference is 104.

Problem Solving
Question 13
Bev scored 540 points. This was 158 points more than Ike scored. How many points did Ike score?

      points

Answer: 382 points

Explanation:

Bev scored 540 points. This was 158 points more than Ike scored.
Let the number of points Ike scored = x
x + 158 = 540
x = 540 – 158
x = 382
Therefore the points that Ike scored is 382.

Question 14
A youth group earned $285 washing cars. The group’s expenses were $79. How much profit did the group make washing cars?

$       profit

Answer: $206

Explanation:

A youth group earned $285 washing cars.
The group’s expenses were $79.
To find how much profit did the group make washing cars.
Subtract 79 from 285
285 – 79
285
-79
206
The group makes $206 profit by washing cars.

Lesson 11: Combine Place Values to Subtract Page No 72

Question 1
A television program lasts for 120 minutes. Of that time, 36 minutes are taken up by commercials. What is the length of the actual program without the commercials?

(a) 84 minutes
(b) 94 minutes
(c) 104 minutes
(d) 156 minutes

Answer: 84 minutes

Explanation:

A television program lasts for 120 minutes.
Of that time, 36 minutes are taken up by commercials.
To find the length of the actual program without the commercials
Subtract 36 minutes from 120 minutes
120
-36
84
Thus the length of the actual program without the commercials is 84 minutes.
The correct answer is option A.

Question 2
Syd spent 215 minutes at the library. Of that time, he spent 120 minutes on the computer. How much of his time at the library did Sid NOT spend on the computer?

(a) 85 minutes
(b) 95 minutes
(c) 105 minutes
(d) 335 minutes

Answer: 95 minutes

Explanation:

Syd spent 215 minutes at the library.
Of that time, he spent 120 minutes on the computer.
To find How much of his time at the library did Sid NOT spend on the computer
We have to subtract the time he spent on the computer from the total time he spent at the library.
i.e., 215 – 120 = 95 minutes
So, the correct answer is option B.

Spiral Review
Question 3
Xavier’s older brother has 568 songs on his music player. To the nearest hundred, about how many songs are on the music player?

(a) 500
(b) 600
(c) 700
(d) 800

Answer: 600

Explanation:

Xavier’s older brother has 568 songs on his music player.
568 to the nearest hundred is 600.
Thus the correct answer is option B.

Question 4
The students traveled to the zoo in 3 buses. One bus had 47 students. The second bus had 38 students. The third bus had 43 students. How many students in all were on the three buses?

(a) 108
(b) 118
(c) 128
(d) 138

Answer: 128

Explanation:

The students traveled to the zoo in 3 buses.
One bus had 47 students.
The second bus had 38 students.
The third bus had 43 students.
Total number of students in all three buses = x
x = 47 + 38 + 43
x = 128 students.
So, the correct answer is option C.

Question 5
Callie has 83 postcards in her collection. Of the postcards, 24 are from Canada. The rest of the postcards are from the United States. How many of the postcards are from the United States?

(a) 58
(b) 59
(c) 61
(d) 69

Answer: 59

Explanation:

Callie has 83 postcards in her collection.
Of the postcards, 24 are from Canada.
The rest of the postcards are from the United States.
Subtract 24 from 83 we get the number of postcards is from the United States.
83 – 24 = 59
So, the correct answer is option B.

Question 6
There were 475 seats set up for the school play. At one performance, 189 of the seats were empty. How many seats were filled at that performance?

(a) 286
(b) 296
(c) 314
(d) 396

Answer: 286

Explanation:

There were 475 seats set up for the school play.
At one performance, 189 of the seats were empty.
Let the Number of seats were filled at that performance = x
x + 189 = 475
x = 475 – 189
x = 286
Thus the correct answer is option A.

Problem Solving • Model Addition and Subtraction Page No – 77

Use the bar model to solve the problem.

Question 1
Elena went bowling. Elena’s score in the first game was 127. She scored 16 more points in the second game than in the first game. What was her total score?

Go Math Grade 3 Chapter 1 Model Addition and Substraction Problem Solving Question 1

Question 2
Mike’s Music sold 287 CDs on the first day of a 2-day sale. The store sold 96 more CDs on the second day than on the first day. How many CDs in all were sold during the 2-day sale?

Go Math Grade 3 Chapter 1 Model Addition and Substraction Problem Solving Question 2

      CDs

Answer: 670 CDs

Explanation:

Mike’s Music sold 287 CDs on the first day of a 2-day sale.
The store sold 96 more CDs on the second day than on the first day.
The means Mike’s music sold CDs on the second day = 287 + 96 = 383.
★ = 283 CDs
Total CDs were sold during the 2-day sale = 383 + 287
♦ = 383 + 287 = 670 CDs

Lesson Check Page No – 78

Question 1
Ms. Hinely picked 46 tomatoes from her garden on Friday. On Saturday, she picked 17 tomatoes. How many tomatoes did she pick in all?

(a) 109
(b) 63
(c) 53
(d) 29

Answer: 63

Explanation:

Ms. Hinely picked 46 tomatoes from her garden on Friday.
On Saturday, she picked 17 tomatoes.
First, find how many tomatoes did she pick in all.
46 + 17 = ★
★ = 63
So, the correct answer is option B.

Question 2
Rosa read 57 pages of a book in the morning. She read 13 fewer pages in the afternoon. How many pages did Rosa read in the afternoon?

(a) 44
(b) 60
(c) 70
(d) 83

Answer: 44

Explanation:

Rosa read 57 pages of a book in the morning.
She read 13 fewer pages in the afternoon.
57 – 13 = ♦
♦ = 57 – 13
♦ = 44
Thus the correct answer is option A.

Spiral Review
Question 3
Mike has 57 action figures. Alex has 186 action figures. Which is the best estimate of the number of action figures Mike and Alex have altogether?

(a) 150
(b) 250
(c) 350
(d) 400

Answer: 250

Explanation:

Mike has 57 action figures.
Alex has 186 action figures.
186 – 57 = ★
★ = 186 – 57
★ = 129
Now Add Mike and Alex action figures
♦ = 186 + 57 = 243
The estimated figure of 243 is 250.
Thus the correct answer is option B.

Question 4
There are 500 sheets of paper in the pack Hannah bought. She has used 137 sheets already. How many sheets of paper does Hannah have left?

(a) 363
(b) 463
(c) 400
(d) 637

Answer: 363

Explanation:

There are 500 sheets of paper in the pack Hannah bought. She has used 137 sheets already.
To find how many sheets of paper does Hannah have left
We have to subtract the number of sheets used from the total number of sheets.
500 – 137 = ★
★ = 500 – 137
★ = 363
Therefore 343 sheets are left.
The correct answer is option A.

Question 5
There were 378 visitors to the science museum on Friday. There were 409 visitors on Saturday. How many more people visited the museum on Saturday?

(a) 25
(b) 31
(c) 171
(d) 787

Answer: 31

Explanation:

There were 378 visitors to the science museum on Friday.
There were 409 visitors on Saturday.
To find how many more people visited the museum on Saturday.
Subtract the number of visitors on Friday from the number of visitors on Saturday.
409 – 378 = 31
31 people visited more the museum on Saturday.
So the correct answer is option B.

Question 6
Ravi scores 247 points in a video game. How many more points does he need to score a total of 650?

(a) 897
(b) 430
(c) 417
(d) 403

Answer: 403

Explanation:

Ravi scores 247 points in a video game.
Let x be the points he needs to score a total of 650
x + 247 = 650
x = 650 – 247
x = 403
Thus he needs 403 points to make a score of 650.
The correct answer is option D.

Review/Test – Page No 79

Question 1

For numbers 1a–1d, choose Yes or No to tell whether the sum is even.

a. 5 + 8

(a) yes
(b) no

Answer: No

Explanation:

5 + 8 = 13 is an odd number.
So, the answer is no.

Question 1
b. 9 + 3

(a) yes
(b) no

Answer: Yes

Explanation:

9 + 3 = 12 is an even number.
So, the answer is yes.

Question 1
c. 6 + 7

(a) yes
(b) no

Answer: No

Explanation:

6 + 7 = 13 is an odd number.
So, the answer is no.

Question 1
d. 9 + 5

(a) yes
(b) no

Answer: Yes

Explanation:

9 + 5 = 14 is an even number.
So, the answer is yes.

Question 2
Select the number sentences that show the Commutative Property of Addition. Mark all that apply.

(a) 14 + 8 = 22
(b) 8 + 14 = 14 + 8
(c) 8 + (13 + 1) = (8 + 13) + 1
(d) (5 + 9) + 8 = (9 + 5) + 8

Answer: 8 + 14 = 14 + 8

Explanation:

According to the commutative property of addition, changing the order of the numbers we are adding, does not change the sum.
So, the answer is option B.

Question 3
Select the numbers that round to 300 when rounded to the nearest hundred. Mark all that apply.

(a) 238
(b) 250
(c) 283
(d) 342
(e) 359

Answer: 283

Explanation:
283 rounded to the nearest hundred is 300.
So, the correct answer is option C.

Question 4
There are 486 books in the classroom library. Complete the chart to show 486 rounded to the nearest 10.

Go Math Grade 3 Chapter 1 Model Addition and Substraction Problem Solving Question 4

Answer:

Hundreds Tens Ones
400 90 0

486 rounded to the nearest ten is 490.

Review/Test – Page No – 80

Question 5
Write each number sentence in the box below the better estimate of the sum.

393+225=■ 481+215=■

352+328=■ 309+335=■

Write each number sentence in the box below the better estimate of the sum

Answer:

600 700
393+225 = 618
The estimated sum is 600.309+335= 644
The estimated sum is 600.
481+215= 696
The estimated sum is 700.352+328= 680
The estimated sum is 700.

Explanation:

393+225=■
■ = 618
The number closer to 618 is 600

481+215=■
■ = 696
The number closer to 696 is 700

352+328=■
■ = 680
The number closer to 680 is 700.

309+335=■
■ = 644
The number closer to 644 is 600.

Question 6
Diana sold 336 muffins at the bake sale. Bob sold 287 muffins. Bob estimates that he sold 50 fewer muffins than Diana. How did he estimate? Explain.

Answer:

Diana sold 336 muffins at the bake sale.
Bob sold 287 muffins.
Bob estimates that he sold 50 fewer muffins than Diana.
To know whether his estimation is right or wrong we have to subtract muffins that Bob sold from muffins that Diana sold
336 – 287 = 49
The number closer to 49 is 50.
So, Bob’s estimation is correct.

Question 7
The table shows how many books each class read.

Go Math Grade 3 The table shows how many books each class read

For numbers 7a–7d, select True or False for each statement.

a. Ms. Martin’s class read about 100 more books than Mr. Lopez’s class.

(i) True
(ii) False

Answer: True

Explanation:

Number of books that Mr. Lopez’s class read = 273
Number of books that Ms. Martin’s class read = 402
402
– 273
129
So, the statement Ms. Martin’s class read about 100 more books than Mr. Lopez’s class is true.

Question 7
b. The 3 classes read over 900 books altogether.

(i) True
(ii) False

Answer: True

Explanation:

Number of books that Mr. Lopez’s class read = 273
Number of books that Ms. Martin’s class read = 402
Number of books that Mrs. Wang read = 247
273
402
274
949
Therefore the statement the 3 classes read over 900 books altogether is true.

Question 7
c. Mrs. Wang’s class read about 50 fewer books than Mr. Lopez’s class.

(i) True
(ii) False

Answer: False

Explanation:

Number of books that Mrs. Wang read = 247
Number of books that Mr. Lopez’s class read = 273
273
– 247
26
Thus the statement Mrs. Wang’s class read about 50 fewer books than Mr. Lopez’s class is false.

Question 7
d. Ms. Martin’s and Mrs. Wang’s class read about 700 books.

(i) True
(ii) False

Answer: False

Explanation:

Number of books that Ms. Martin’s class read = 402
Number of books that Mrs. Wang read = 247
402
247
649
Therefore the statement Ms. Martin’s and Mrs. Wang’s class read about 700 books is false.

Review/Test – Page No – 81

Question 8
Janna buys 2 bags of dog food for her dogs. One bag weighs 37 pounds. The other bag weighs 15 pounds. How many pounds do both bags weigh? Explain how you solved the problem.

      pounds

Answer: 52 pounds

Explanation:

Janna buys 2 bags of dog food for her dogs. One bag weighs 37 pounds. The other bag weighs 15 pounds.
I used friendly numbers

37 = 35 + 2
15 = 15 + 0
52 =  50 + 2
The weight of 2 bags is 52 pounds.

Question 9
Choose the property that makes the statement true.

The Go Math grade 3 Chapter 1 answer key review image_1 Property of addition states that you can group addends in different ways and get the same sum.

Answer: The Associative Property of addition states that you can group addends in different ways and get the same sum.

Use the table for 10–12.

Go Math Grade 3 Chapter 1 Problem Solving Use the table for 10–12

Question 10
The table shows the number of sweaters sold online in three months. How many sweaters were sold in January and February?

        sweaters

Answer: 700 sweaters

Explanation:

The number of sweets sold in the month of January = 402
The number of sweets sold in the month of February = 298
First, make the friendly numbers to make the addition easy.|
Subtract 2 from 402 = 402 – 2 = 400
Next add 2 to 298 = 298 + 2 = 300
Now add both
400 + 300 = 700
Therefore 700 sweaters were sold in January and February.

Question 11
How many more sweaters were sold in January than March?

        sweaters

Answer: 231 sweaters

Explanation:

The number of sweets sold in the month of January = 402
The number of sweets sold in the month of March = 171
To find how many more sweaters were sold in January than March, we have subtracted the number of sweaters sold in the march from January
402 – 171 = 231
231 more sweaters were sold in January than March.

Question 12

How many more sweaters were sold in February and March than in January?

        sweaters

Answer: 67 sweaters

Explanation:

The number of sweets sold in the month of January = 402
The number of sweets sold in the month of February = 298
The number of sweets sold in the month of March = 171
Total number of sweaters sold in February and March = 298 + 171 = 469
Now subtract 402 from 469
469 – 402 = 67 sweaters
67 more sweaters were sold in February and March than in January.

Review/Test – Page No – 82

Question 13
Help Dana find the sum.

346 + 421 + 152
For numbers 13a–13d, select Yes or No to tell Dana when to regroup.

a. Regroup the ones.

(a) yes
(b) no

Answer: Yes

Question 13
b. Add the regrouped ten.

(a) yes
(b) no

Answer: No

Question 13
c. Regroup the tens.

(a) yes
(b) no

Answer: Yes

Question 13
d. Add the regrouped hundred.

(a) yes
(b) no

Answer: Yes

Question 14
Alexandra has 78 emails in her inbox. She deletes 47 emails. How many emails are left in her inbox? Draw jumps and label the number line to show your thinking.

Go Math Grade 3 Chapter 1 Alexandra has 78 emails in her inbox. She deletes 47 emails

        emails

Answer: 31 emails

Explanation:

Alexandra has 78 emails in her inbox.
She deletes 47 emails.
Let x be the number of emails left in her inbox
x + 47 = 78
x = 78 – 47
x = 31
Therefore, 31 emails are left in her inbox.

Question 15
Daniel has 402 pieces in a building set. He uses 186 pieces to build a house. How many pieces does he have left? Show your work.

        pieces

Answer: 216 pieces

Explanation:

Daniel has 402 pieces in a building set.
He uses 186 pieces to build a house.
x be the number of pieces he had left
x + 186 = 402
x = 402 – 186
x = 216 pieces
Thus he left 216 pieces to build a house

Review/Test – Page No – 83

Question 16
Luke solves this problem. He says the difference is 214. Explain the mistake Luke made. What is the correct difference?

352−148 =        

Answer: 204

Explanation:

Make friendly numbers to make the subtraction easy.
First subtract 2 from 352 = 350
350
148
202
1 will be borrowed from tens place. So 0 becomes 10.
10 – 8 = 2
4 – 4 = 0
300 – 100 = 200
200 + 2 = 202
Now add 2 to 202 you get 204.

Question 17
Sunnyday Elementary School is having its annual Read-a-thon. The third graders have read 573 books so far. Their goal is to read more than 900 books. What is the least number of books they need to read to reach their goal? Explain.

        books

Answer: 327 books

Explanation:

Sunnyday Elementary School is having its annual Read-a-thon.
The third graders have read 573 books so far. Their goal is to read more than 900 books.
Let the 3rd graders have to read the total number of books = x
x + 573 = 900
x = 900 – 573
x = 327
Thus the least number of books they need to read to reach their goal is 327 books.

Question 18
There are 318 fiction books in the class library. The number of nonfiction books is 47 less than the number of fiction books.

Part A

About how many nonfiction books are there in the class library? Explain.

About         nonfiction books

Answer: About 270 nonfiction books

Explanation:

Given that,
There are 318 fiction books in the class library.
The number of nonfiction books is 47 less than the number of fiction books.
Number of non fictions books = x
x + 47 = 318
x = 318 – 47
x = 271
The number closer to 271 is 270.
So, there are about 270 nonfiction books.

Question 18
Part B

How many fiction and nonfiction books are there in the class library altogether? Show your work.

        total books

Answer: 589

Explanation:

Number of fiction books = 318
Number of nonfiction books = 271
To find the total number of books we need to add both fiction and nonfiction books
= 318 + 271 = 589
There are 589 books in the class library.

Review/Test – Page No – 84

Question 19
Alia used 67 + 38 = 105 to check her subtraction. Which math problem could she be checking? Mark all that apply.

67−38=■
105−67=■
105+38=■
105−38=■

Answer: 105−67= 38; 105−38=67
She can use option B and Option D to check her subtraction.

Question 20
Alex and Erika collect shells. The tables show the kinds of shells they collected.

Go Math Grade 3 Chapter 1 Problem Solving Alex and Erika collect shells. The tables show the kinds of shells they collected.

Part A

Who collected more shells? How many did she collect? About how many more is that? Explain how you solved the problem.

       

Answer: Alex

Alxe’s Shells:
Number of Scallop = 36
Number of Jingle shells = 95
Number of Clam = 115
Now add all the three shells = 36 + 95 + 115 = 246 shells

Erika’s shells:

Number of Scallop = 82
Number of Whelk shells = 28
Number of Clam = 108
Now add all the three shells = 82 + 28 + 108 = 218 shells
Alex collected about 250 shells.

Question 20
Part B

Alex and Erika have the greatest number of what kind of shell? How many shells of that kind do they have? Show your work.

Answer: Clam

The greatest number of shells that Alex and Erika collected are Clam.

Conclusion

In addition to the exercise and homework problems we also provide the solutions for the Extra Practice. So, the students are advised to go through the Go Math Answer Key Grade 3 Chapter 1 Addition and Subtraction within 1,000 Extra Practice to test your math skills in this chapter. You can also your friends to improve their math skills by sharing this link.

Go Math Grade 6 Answer Key Chapter 13 Variability and Data Distributions

go-math-grade-6-chapter-13-variability-and-data-distributions-answer-key

Go Math Grade 6 Answer Key Chapter 13 deals with Variability and Data Distributions. The HMH Go Math 6th Grade Answer Key is a very helpful resource for students to prepare for the exams. The solutions are mentioned topic-wise to all the questions for chapter 13 Variability and Data Distributions. You can understand the problem-solving methods in a better way by using Go Math Grade 6 Answer Key Chapter 13 Variability and Data Distributions for free.

Go Math Grade 6 Answer Key Chapter 13 Variability and Data Distributions

The main concepts of chapter 13 Variability and Data Distributions are discussed in the below sections. So, tap the links and practice the problems. This Go Math Grade 6 Answer Key Ch 13 Variability and Data Distributions helps you to secure the highest marks in the exams. For better performance, you try to compare the problems with real time.

Chapter 13 – Lesson: 1

Chapter 13 – Lesson: 2

Chapter 13 – Lesson: 3

Chapter 13 – Lesson: 4

Chapter – 13 – Mid-Chapter Checkpoint

Chapter 13 – Lesson: 5

Chapter 13 – Lesson: 5

Chapter 13 – Lesson: 6

Chapter 13 – Lesson: 7

Chapter 13 – Review/Test

Share and Show – Page No. 709

For 1–3, use the dot plot.

Question 1.
The dot plot shows the number of paintings students in the art club displayed at the art show. Does the dot plot contain any gaps?
If so, where?
Go Math Grade 6 Answer Key Chapter 13 Variability and Data Distributions img 1
Type below:
_________________

Answer: Between the intervals of 4 – 7 excluding 4 and 7

Explanation:
Go Math Grade 6 Answer Key Chapter 13 Variability and Data Distributions img 1
The dots are filled from 1 – 4 and 7 but the region between these two intervals is left unfilled so the region containing gaps is 5-6 including 5 and 6

Question 2.
Identify any clusters in the data.
Type below:
_________________

Answer: 1-4

Explanation:
A group of dots is called a cluster
The dots form a cluster at 1 – 4

Question 3.
Summarize the information in the dot plot.
Type below:
_________________

Answer: It says about the number of paintings done by each student in the art club.

Explanation:
The number of paintings is represented by the number line. The dots represent the students.
Therefore we can say that It says about the number of paintings done by each student in the art club.

On Your Own

Question 4.
What patterns do you see in the histogram data?
Go Math Grade 6 Answer Key Chapter 13 Variability and Data Distributions img 2
Type below:
_________________

Answer:

Explanation:
STEP 1 Identify any peaks in the data.
The histogram has 6 peaks.
The interval representing the greatest number of visitors is for ages between 60 and 69 age group.
STEP 2 The data changes across the intervals.
The number of visitors increases from 0 to 29 age group and from 40 to 69 age group.
So, the data values increase to one peak in the interval from 0 to 9 age group and then decrease.
The visitors of the age group 30 – 39 did not visit the zoo.

Question 5.
The dot plot shows the number of errors made by a baseball team in the first 16 games of the season. For numbers 5a-5e, choose Yes or No to indicate whether the statement is correct.
Go Math Grade 6 Answer Key Chapter 13 Variability and Data Distributions img 3
5a. There is a gap from 4 to 5.
5b. There is a peak at 0.
5c. The dot plot has symmetry.
5d. There are two modes.
5e. There is one cluster.
5a. __________
5b. __________
5c. __________
5d. __________
5e. __________

Answer:
5a. Yes
5b. Yes
5c. No
5d. No
5e. No

Explanation:
5a. There are dots between 4-5 so we can say that there is a gap from 4 to 5.
5b. The number of dots is more at the interval 0 So we can say that there is a peak at 0.
5c. The symmetrical view is nothing but having the same number of dots on both sides of the figure but we cannot observe it in the above figure. Therefore we can say that the dot plot has no symmetry.
5d. The most frequently occurring observation is known as a mode. One dot repeats in all the intervals so we can say that the mode is 1.
5e. A group of observations form a cluster, there are more than 1 group of dots in the figure given above.

Big Cats – Page No. 710

There are 41 species of cats living in the world today. Wild cats live in places as different as deserts and the cold forests of Siberia, and they come in many sizes. Siberian tigers may be as long as 9 feet and weigh over 2,000 pounds, while bobcats are often just 2 to 3 feet long and weigh between 15 and 30 pounds.

You can find bobcats in many zoos in the United States. The histogram below shows the weights of several bobcats. The weights are rounded to the nearest pound.
Go Math Grade 6 Answer Key Chapter 13 Variability and Data Distributions img 4
Use the histogram for 6 and 7.

Question 6.
Look for a Pattern Describe the overall shape of the histogram.
Type below:
_________________

Answer: The graph starts from a small interval and increases to the highest and then decreases to the smallest interval.
The histogram has rectangles which are closely packed.

Explanation:
STEP 1 Identify any peaks in the data.
The histogram has 1 peak(s).
The interval representing the greatest number of bobcats is for weights between 18 and 20 pounds.
STEP 2 Describe how the data changes across the intervals. The number of bobcats increases from 12 to 17 pounds and from 21 to 29 pounds.
STEP 3 Describe any symmetry the graph has. If I draw a vertical line through the interval for 18 to 20 pounds, the left and right parts of the histogram are very close to being mirror images. The histogram has line symmetry.

So, the data values increase to one peak in the interval for 18 to 20 pounds and then decrease. The data set has a vertical line
symmetry about the peak.

Question 7.
Sense or Nonsense? Sunny says that the graph might have a different shape if it was redrawn as a bar graph with one bar for each number of pounds. Is Sunny’s statement sense or nonsense? Explain.
Type below:
_________________

Answer: Sense

Explanation:
Bar graph also contains rectangles but they are not closely packed hence the statement is correct which is said by Sunny as a bar graph with one bar for each number of pounds.

Patterns in Data – Page No. 711

For 1–2, use the dot plot.
Go Math Grade 6 Answer Key Chapter 13 Variability and Data Distributions img 5

Question 1.
The dot plot shows the number of omelets ordered at Paul’s Restaurant each day. Does the dot plot contain any gaps?
Type below:
_________________

Answer: Yes, the dot plot contain gaps

Explanation:
The dots are filled from 10 – 11, from 14 – 16 and from 18 – 19 but the region between these two intervals is left unfilled so the region containing gaps is 12-13 including 12 and 13, 17 is also left unfilled.

Question 2.
Identify any clusters in the data.
Type below:
_________________

Answer: 14 – 16 and 18 – 19

Explanation:
A group of dots is called a cluster. The dots which form a cluster are 14 – 16 and from 18 – 19.

For 3–4, use the histogram.
Go Math Grade 6 Answer Key Chapter 13 Variability and Data Distributions img 6

Question 3.
The histogram shows the number of people that visited a local shop each day in January. How many peaks does the histogram have?
Type below:
_________________

Answer: The histogram has only one peak.

Explanation:
The rectangle with tall length represents the highest peak in the graph given above.
The number of people who visited a local shop each day in January were among 0 – 9  visitors and this was the highest frequency having 14 days.
The highest peak is in the interval of 0 – 9.

Question 4.
Describe how the data values change across the intervals.
Type below:
_________________

Answer: They decrease from highest to low values in the given picture above.

Explanation:
The graph represents the number of visitors in the month of January the visitors of number 0 – 9 have the highest frequency,
10 – 19 are the second-highest among the visitors who went to the local shop in the month of January, followed by 20 – 29,
30 – 39

Problem Solving

Question 5.
Look at the dot plot at the right. Does the graph have symmetry? Explain.
Go Math Grade 6 Answer Key Chapter 13 Variability and Data Distributions img 7
Type below:
_________________

Answer: Yes, the graph has a symmetry

Explanation:
If I draw a vertical line through the interval for _ to_ pounds, the left and right parts of the histogram are very close to being mirror images. The histogram __ line symmetry.
A geometric figure has line symmetry if you can draw a line through it so that the two parts are mirror images of each other.
So, the data values increase to one peak in the interval for _ to _ pounds and then decrease. The data set __ line symmetry about the peak.

Question 6.
A histogram that shows the ages of students at a library has intervals 1–5, 6–10, 11–15, 16–20, and 21–25. There is a peak at 11–15 years and the graph is symmetric. Sketch what the histogram could look like and describe the patterns you see in the data.
Type below:
_________________

Answer:

The histogram shows the ages of students at a library has intervals 1–5, 6–10, 11–15, 16–20, and 21–25. There is a peak at 11–15 years and the graph is symmetric.

Explanation:

The histogram is a graph with continuous rectangles which are closely packed.
The asymmetric graph is a graph which has a mirror-like view with equal rectangles on each side.
The graph with the highest peak represents the highest number of students who visit the library in that age group 11 – 15

Lesson Check – Page No. 712

Question 1.
What interval in the histogram has the greatest frequency?
Go Math Grade 6 Answer Key Chapter 13 Variability and Data Distributions img 8
Type below:
_________________

Answer: 10 – 14 interval has highest frequency of 6

Explanation:
The rectangle with a peak can be said as it has the greatest frequency. The interval with a peak is 11 – 15 and the frequency of the peak is 6

Question 2.
Meg makes a dot plot for the data 9, 9, 4, 5, 5, 3, 4, 5, 3, 8, 8, 5. Where does a gap occur?
Type below:
_________________

Answer: 6 – 7 including 6 and 7

Explanation:
Let us consider an axis with 3 to 9 numbers on it plot the dots as given in the question at the points 3,4,5,8 and 9 the gap occurs between 6 and 7 including 6 and 7.

Spiral Review

Question 3.
A rectangular fish tank is 20 inches long, 12 inches wide, and 20 inches tall. If the tank is filled halfway with water, how much water is in the tank?
________ in.

Answer: 37500 cubic centimeter

Explanation:
The length of the rectangle of the rectangular fish tank = 20 inches x 2.5 cm = 50 cm (since 1 inch = 2.5 cm)
The breadth of the rectangle of the rectangular fish tank = 12 inches x 2.5 cm = 30 cm (since 1 inch = 2.5 cm)
The height of the rectangle of the rectangular fish tank = 20 inches x 2.5 cm = 50 cm (since 1 inch = 2.5 cm)
Water filled in the tank = Volume of the tank = 50 x 50 x 30 = 75000 cubic centimeter
If the tank is filled halfway = volume of the tank / 2 = 37500 cubic centimeter

Question 4.
Look at the histogram below. How many students scored an 81 or higher on the math test?
________ students

Answer: 14

Explanation:
The interval 81 – 90 has 10 frequency and the interval 91 – 100 has 4 frequency. So the total number of students = 14

Question 5.
The Little League coach uses a radar gun to measure the speed of several of Kyle’s baseball pitches. The speeds, in miles per hour, are 52, 48, 63, 47, 47. What is the median of Kyle’s pitch speeds?
The median is ________ miles.

Answer: Median is 48

Explanation:
First, write the observations in ascending order or descending order.
The formula to calculate the median is (n+1/2)th observation, for odd number of observations
n = 5 (odd)
Median = (5 + 1 / 2) = (6/2) = 3rd observation = 48
Therefore the median is 48.

Share and Show – Page No. 715

Find the median, lower quartile, and upper quartile of the data.

Question 1.
the scores of 11 students on a geography quiz:
87, 72, 80, 95, 86, 80, 78, 92, 88, 76, 90
Type below:
_________________

Answer: Median: 86, lower quartile: 72, upper quartile: 95

Explanation:
First, write the observations in ascending order or descending order.
The formula to calculate the median is (n+1/2)th observation, for odd number of observations
n = 11 (odd)
Median = (11 + 1 / 2) = (12/2) = 6th observation = 86
Therefore the median is 86.

Lower quartile: 72  Upper quartile: 95

Question 2.
the lengths, in seconds, of 9 videos posted online:
50, 46, 51, 60, 62, 50, 65, 48, 53
Type below:
_________________

Answer: Median: 51 Lower quartile: 46 Upper quartile: 65

Explanation:
First, write the observations in ascending order or descending order.
The formula to calculate the median is (n+1/2)th observation, for odd number of observations
n = 9 (odd)
Median = (9 + 1 / 2) = (10/2) = 5th observation = 51
Therefore the median is 51.

Lower quartile: 46 Upper quartile: 65

Question 3.
Make a box plot to display the data set in Exercise 2.
Type below:
_________________

Answer: The box plot is drawn on the topic: Lengths of the videos (in seconds) posted in the online.

Explanation:
The box is drawn to understand the clear view of the raw data, in a precise manner.
This box gives us information about lengths of the videos posted in the online. We can directly say the median, lower quartile, upper quartile seeing the box plot.

On Your Own

Find the median, lower quartile, and upper quartile of the data.

Question 4.
13, 24, 37, 25, 56, 49, 43, 20, 24
Type below:
_________________

Answer: 25

Explanation:
First, write the observations in ascending order or descending order.
The formula to calculate the median is (n+1/2)th observation, for odd number of observations
n = 9 (odd)
Median = (9 + 1 / 2) = (10/2) = 5th observation =25
Therefore the median is 25.

Question 5.
61, 23, 49, 60, 83, 56, 51, 64, 84, 27
Type below:
_________________

Answer: 58

Explanation:
First, write the observations in ascending order or descending order.
The formula to calculate the median is mean of (n/2) and (n/2+1)th observations, for even number of observations
n = 10 (even)
Median = Mean of (5)th and (6)th observations = 56 + 60 divided by 2 = 116/2 = 58
Therefore the median is 58.

Question 6.
The chart shows the height of trees in a park. Display the data in a box plot.
Go Math Grade 6 Answer Key Chapter 13 Variability and Data Distributions img 9
Type below:
_________________

Answer:

Explanation:
Lower limit: 8
Upper limit: 30
Median:
First, write the observations in ascending order or descending order.
The formula to calculate the median is mean of (n/2) and (n/2+1)th observations, for even number of observations
n = 12 (even)
Median = Mean of (6)th and (7)th observations = 18 + 20 divided by 2 = 38/2 = 19
Therefore the median is 19.

Question 7.
Analyze Eric made this box plot for the data set below. Explain his error.
Go Math Grade 6 Answer Key Chapter 13 Variability and Data Distributions img 10
Go Math Grade 6 Answer Key Chapter 13 Variability and Data Distributions img 11
Type below:
_________________

Answer: The lower and upper limits are marked wrong.

Explanation:
The box drew above the number line is wrong.
It does not show the correct upper and lower limits.
The lower limit is 5 and the upper limit is 35.

Problem Solving + Applcations – Page No. 716

Pose a Problem

Question 8.
The box plots show the number of flights delayed per day for two different airlines. Which data set is more spread out?
Go Math Grade 6 Answer Key Chapter 13 Variability and Data Distributions img 12
Airline A: greatest value − least value = _____
Airline B: greatest value − least value = _____
So, the data for _____ is more spread out.
Write a new problem that can be solved using the data in the box plots.
Type below:
_________________

Answer:
Airline A: greatest value − least value = 8
Airline B: greatest value − least value = 10
The data for airline B is more spread out.

A problem which can be solved using the box plot can be:

Find the median, lower and upper limits.

Explanation:
The greatest value and lowest value can be identified by seeing the box drew above the number line. The ends represent the lower and upper limits in both the box plots.

The solution to the question framed:
The start and end of the rectangle represent the lower and upper limits. And the middle line represents the median.
The lower limit is 5
Upper limit is 35
Median:
First, write the observations in ascending order or descending order.
The formula to calculate the median is mean of (n/2) and (n/2+1)th observations, for even number of observations
n = 6 (even)
Median = Mean of (3)th and (4)th observations = 15 + 25 divided by 2 = 40/2 = 20
Therefore the median is 20.

Question 9.
The data set shows the cost of the dinner specials at a restaurant on Friday night.
Go Math Grade 6 Answer Key Chapter 13 Variability and Data Distributions img 13
The median is _____.
The lower quartile is _____.
The upper quartile is _____.

Answer:
Median: 24
The lower quartile is 16.
The upper quartile is 30.

Explanation:
Seeing the data in the box we can identify the lower and upper quartiles.
Median:
First, write the observations in ascending order or descending order.
The formula to calculate the median is (n+1/2)th observation, for odd number of observations
n = 11 (odd)
Median = (11 + 1 / 2) = (12/2) = 6th observation =24
Therefore the median is 24.

Box Plots – Page No. 717

Find the median, lower quartile, and upper quartile of the data.

Question 1.
the amounts of juice in 12 glasses, in fluid ounces:
11, 8, 4, 9, 12, 14, 9, 16, 15, 11, 10, 7
Type below:
_________________

Answer:
Median: 10.5
Lower quartile: 4
Upper quartile: 16

Explanation:
Median:
First, write the observations in ascending order or descending order.
The formula to calculate the median is mean of (n/2) and (n/2+1)th observations, for even number of observations
n = 12 (even)
Median = Mean of (6)th and (7)th observations = 10 + 11 divided by 2 = 21/2 = 10.5
Therefore the median is 10.5.
After writing in ascending or descending order the first and last terms justify the lower and upper limits respectively.
They are:
Lower quartile: 4
Upper quartile: 16

Question 2.
the lengths of 10 pencils, in centimeters:
18, 15, 4, 9, 14, 17, 16, 6, 8, 10
Type below:
_________________

Answer:
Median: 12
Lower quartile: 4
Upper quartile: 18

Explanation:
Median:
First, write the observations in ascending order or descending order.
The formula to calculate the median is mean of (n/2) and (n/2+1)th observations, for even number of observations
n = 10 (even)
Median = Mean of (5)th and (6)th observations = 10 + 14 divided by 2 = 24/2 = 12
Therefore the median is 12.
After writing in ascending or descending order the first and last terms justify the lower and upper limits respectively.
They are:
Lower quartile: 4
Upper quartile: 18

Question 3.
Make a box plot to display the data set in Exercise 2.
Type below:
_________________

Answer:

The above box plot represents the lower and upper quartiles, the median.

Explanation:
Box plot is drawn using the number line and the rectangle which is drawn above it.
The ends of the rectangles say about the lower and upper limits and the middle line indicates the median.

Question 4.
The numbers of students on several teams are 9, 4, 5, 10, 11, 9, 8, and 6. Make a box plot for the data.
Type below:
_________________

Answer:

Explanation:
Box plot is drawn using the number line and the rectangle which is drawn above it.
The ends of the rectangles say about the lower and upper limits and the middle line indicates the median.
Therefore the lower and upper quartiles are 4 and 11 respectively.
Median:
First, write the observations in ascending order or descending order.
The formula to calculate the median is mean of (n/2) and (n/2+1)th observations, for even number of observations
n = 8 (even)
Median = Mean of (4)th and (5)th observations = 8 + 9 divided by 2 = 17/2 = 8.5
Therefore the median is 8.5.

Problem Solving

Question 5.
The amounts spent at a gift shop today are $19, $30, $28, $22, $20, $26, and $26. What is the median? What is the lower quartile?
Type below:
_________________

Answer:
Median: $26
Lower quartile: $19
Upper quartile: $30

Explanation:
Median:
First, write the observations in ascending order or descending order.
The formula to calculate the median is (n+1/2)th observation, for odd number of observations
n = 7 (odd)
Median = (7 + 1 / 2) = (8/2) = 4th observation =26
Therefore the median is 26.
After writing in ascending or descending order the first and last terms justify the lower and upper limits respectively.
They are:
Lower quartile: $19
Upper quartile: $30

Question 6.
The weights of six puppies in ounces are 8, 5, 7, 5, 6, and 9. What is the upper quartile of the data?
Type below:
_________________

Answer: Upper quartile: 9

Explanation:
The highest value in the data is defined as the upper quartile.
The highest value in the raw data given is 9

Question 7.
Draw a box plot to display this data: 81, 22, 34, 55, 76, 20, 56.
Type below:
_________________

Answer:

Explanation:
A box plot gives information about the lower and upper quartiles and about the median.
The box plot is drawn using a rectangle and the number line.

Lesson Check – Page No. 718

Question 1.
The values in a data set are 15, 7, 11, 12, 6, 3, 10, and 6. Where would you draw the box in a box plot for the data?
Type below:
_________________

Answer: The box is drawn above the number line.

Explanation:
Example:

The rectangle which can be seen above the number line is the box plot which is drawn.
The box plot gives information about the lower and upper quartiles and about the median.

Question 2.
What is the lower quartile of the following data set?
22, 27, 14, 21, 22, 26, 18
Type below:
_________________

Answer: 14

Explanation:
The value which is lowest in the given data is called the lowest quartile.
Therefore the lowest quartile in the given data is 14.

Spiral Review

Question 3.
Jenn says that “What is the average number of school lunches bought per day?” is a statistical question. Lisa says that “How many lunches did Mark buy this week?” is a statistical question. Who is NOT correct?
Type below:
_________________

Answer: Lisa’s statement is wrong.

Explanation:

Question 4.
The prices of several chairs are $89, $76, $81, $91, $88, and $70. What is the mean of the chair prices?
The mean is $ _________

Answer: $82.5

Explanation:
Number of observations= 6
Mean of the observations= $89 + $76 + $81+ $91+$88+ $70/ 6= 495/6 = $82.5

Question 5.
By how much does the mean of the following data set change if the outlier is removed?
13, 19, 16, 40, 12
Type below:
_________________

Answer: The mean shows a difference if the lower limit is removed the mean increases and if the upper limit is removed the mean decreases.

Explanation:
Outliers are nothing but both upper and lower limits.
The actual mean is 20
But when the lower limit is removed the mean increases to 22 while when the upper limit is removed the mean decreases to 15.
Therefore, we can say that the mean shows a difference when the outliers are removed.

Question 6.
Where in the dot plot does a cluster occur?
Go Math Grade 6 Answer Key Chapter 13 Variability and Data Distributions img 14
Type below:
_________________

Answer: 52 – 54

Explanation:
A cluster is nothing but a group of dots.
In the intervals 52 – 54 a cluster has occurred.

Share and Show – Page No. 721

Use counters, a dot plot, or iTools to find the mean absolute deviation of the data.

Question 1.
Find the mean absolute deviation for both data sets. Explain which data set is more spread out.
the number of laps Shawna swam on 5 different days:
5, 6, 6, 8, 10
mean = 7

the number of laps Lara swam on 5 different days:
1, 3, 7, 11, 13
mean = 7
Type below:
_________________

Answer: Case 2 is more spread out.

Explanation:
CASE1
The number of laps Shawna swam on 5 different days:
5,6,6,8,10
Mean = 7
Deviations:
7 – 5 = 2
7 – 6 = 1
7 – 6 = 1
7 -8 = -1
7 -10=-3
Mean of deviations = 2+1+1+1+3/5 = 8/5 = 1.6

CASE2
The number of laps Lara swam on 5 different days:
1, 3, 7, 11, 13
Mean = 7
Deviations:
7 – 1 = 6
7 – 3 = 4
7 – 7 = 0
7 -11= -4
7 -13= -6
Mean of deviations = 6+ 4 + 0 + 4 + 6 / 5 = 20/5 = 4

Use the dot plot to find the mean absolute deviation of the data.

Question 2.
mean = 7 books
Go Math Grade 6 Answer Key Chapter 13 Variability and Data Distributions img 15
______ books

Answer: Mean absolute deviation is 2.4

Explanation:
STEP 1 Label each dot with its distance from the mean.
Starting from left to right:
4: 7-4=3
5: 7-5=2
6: 7-6=1
9: 7-9=-2
10: 7-10=-3
11: 7-11=-4

STEP 2 Find the mean of the distances.
(3) + (2) +(2) +(2) +(2) +(1) + (2) +(3) +(3) +(4) / 10 = 24/10 = 2.4

So, the mean absolute deviation of the data is 2.4

Question 3.
mean = 29 pounds
Go Math Grade 6 Answer Key Chapter 13 Variability and Data Distributions img 17
_______ pounds

Answer: Mean Absolute deviation is 3.2

Explanation:
STEP 1 Label each dot with its distance from the mean.
Starting from left to right:
26: 29-26=03
27: 29-27=02
32: 29-32=-3
33: 29-33=-4
35: 29-35=-6

STEP 2 Find the mean of the distances.
(3) + (2) +(3) +(4) +(6) +(3) + (3) +(2) / 8 = 26/8 = 3.2

So, the mean absolute deviation of the data is 3.2

Question 4.
The mean absolute deviation of the number of daily visits to Scott’s website for February is 167.7. In March, the absolute mean deviation is 235.9. In which month did the number of visits to Scott’s website vary more? Explain how you know.
Type below:
_________________

Answer: As the mean absolute deviation is more in the month of February we can say that there are more visitors in this month.

Explanation:
As the mean of the month of February is less it means that the number of observations are more.
Similarly, as the mean of the month of March is more it means that the number of observations are less.
Therefore we can say that the number of visitors were more in the month of February compared to March.

Question 5.
Write an Inequality Algebra In April, the data for Scott’s website visits are less spread out than they were in February. Use a to represent the mean absolute deviation for April. Write an inequality to describe the possible values of a.
Type below:
_________________

Answer: a < February

Explanation:
Since the data is more spread out in the month of April than they were in February. Therefore the inequality represents “less than” sign.

Problem Solving + Applcations – Page No. 722

Question 6.
Use the table.
Go Math Grade 6 Answer Key Chapter 13 Variability and Data Distributions img 18
The mean of the data is 11. What is the mean absolute deviation of the data?
_______ days

Answer: 3

Explanation:
STEP 1 Label each observation with its distance from the mean.
Starting from left to right:
10: 11-10= 1
12: 11-12=-1
13: 11-13=-2
18: 11-18=-7
10: 11-10= 1
08: 11-08= 3
07: 11-07= 4
06: 11-06= 5
16: 11-16=-5
14: 11-14=-3
08: 11-08= 3
10: 11-10= 1

STEP 2 Find the mean of the distances.
1+1 +2 +7 +1 +3 +4 +5+5+3+3+1/ 12
= 36/12 = 3

So, the mean absolute deviation of the data is 3.

Question 7.
Suppose all of the players on a basketball team had the same height. Explain how you could use reasoning to find the mean absolute deviation of the players’ heights.
Type below:
_________________

Answer: 0

Explanation:
If the players on a basketball team had the same height.
The mean deviation will be equal to the 0 because the difference between the mean and the observations is 0.
Let the observations be 2,2,2,2,2
Mean = 10/5 = 2
Mean deviation =  (2-2)+(2-2)+(2-2)+(2-2)+(2-2)/5 = 0/5 = 0

Question 8.
Explain Tell how an outlier that is much greater than the mean would affect the mean absolute deviation of the data set. Explain your reasoning.
Type below:
_________________

Answer: An outlier increases the mean absolute deviation of the data set.

Explanation:
The difference between the outlier and the mean is a greater number when added in the sum of observations the mean absolute deviation increases.

Question 9.
The data set shows the number of soccer goals scored by players in 3 games.
Go Math Grade 6 Answer Key Chapter 13 Variability and Data Distributions img 19
For numbers 9a–9c, choose Yes or No to indicate whether the statement is correct.
9a. The mean absolute deviation of Player A is 1.
9b. The mean absolute deviation of Player B is 0.
9c. The mean absolute deviation of Player C is greater than the mean absolute deviation of Player A.
9a. __________
9b. __________
9c. __________

Answer:
9a. No
9b. Yes
9c. No

Explanation:
Player A
Mean = 1+2+3/3 = 6/3 = 2
Mean absolute deviation = 1+0+1/3 = 2/3 = 0.6

Player B
Mean =2+2+2/3 = 6?3 = 2
Mean absolute deviation = 0/3 = 0

Player C
Mean = 1+2+1/3 = 4/3 = 1.3
Mean absolute deviation = 0.3+0+0.3/3 = 0.2

Mean Absolute Deviation – Page No. 723

Use counters and a dot plot to find the mean absolute deviation of the data.

Question 1.
the number of hours Maggie spent practicing soccer for 4 different weeks:
9, 6, 6, 7
mean = 7 hours
_______ hour

Answer: The mean absolute deviation of the data is 1.

Explanation:
STEP 1 Label each observation with its distance from the mean.
Starting from left to right:
9: 7-9=-2
6: 7-6=-1
6: 7-6=-1
7: 7-7= 0

STEP 2 Find the mean of the distances.
2 +1 +1+0/ 4
= 4 /4 = 1

So, the mean absolute deviation of the data is 1.

Question 2.
the heights of 7 people in inches:
60, 64, 58, 60, 70, 71, 65
mean = 64 inches
_______ inches

Answer: The mean absolute deviation of the data is 4.

Explanation:
STEP 1 Label each observation with its distance from the mean.
Starting from left to right:
60: 64-60= 4
64: 64-64= 0
58: 64-58= 6
60: 64-60= 4
70: 64-70=-6
71: 64-71=-7
65: 64-65=-1

STEP 2 Find the mean of the distances.
4+0+6+ 4+6+7+1/7
= 28/7 = 4

So, the mean absolute deviation of the data is 4.

Use the dot plot to find the mean absolute deviation of the data.

Question 3.
mean = 10
Go Math Grade 6 Answer Key Chapter 13 Variability and Data Distributions img 20
_______ year

Answer: The mean absolute deviation of the data is 1

Explanation:
STEP 1 Label each dot with its distance from the mean.
Starting from left to right:
08: 10-08=02
09: 10-09=01
10: 10-10= 0
11: 10-11=-1
12: 10-12=-2

STEP 2 Find the mean of the distances.
(2) + (1) +(0) +(1) +(2) +(2)+(1)+(0)+(0)+(0)+(1)+(2) /12 = 12/12= 1

So, the mean absolute deviation of the data is 1

Question 4.
mean = 8
Go Math Grade 6 Answer Key Chapter 13 Variability and Data Distributions img 21
_______ hours

Answer: The mean absolute deviation of the data is 2.4

Explanation:
STEP 1 Label each dot with its distance from the mean.
Starting from left to right:
03: 8-03=05
04: 8-04=04
05: 8-05=03
07: 8-07=01
08: 8-08= 0
09: 8-09=-1
10: 8-10=-2
11: 8-11=-3
12: 8-12=-4

STEP 2 Find the mean of the distances.
(5) + (4) +(3) +(1) +(0) +(1) + (2) +(3) +(4)+(5)+(0)+(1)+(1)+(2)+(4)/ 15 = 36/15 = 2.4

So, the mean absolute deviation of the data is 2.4

Problem Solving

Question 5.
In science class, Troy found the mass, in grams, of 6 samples to be 10, 12, 7, 8, 5, and 6. What is the mean absolute deviation?
_______ grams

Answer: The mean absolute deviation of the data is 2.

Explanation:
Mean = 10+12+7+8+5+6/6 = 48/6 = 8

Mean absolute deviation:

STEP 1 Label each observation with its distance from the mean.
Starting from left to right:
10: 8-10= -2
12: 8-12= -4
07: 8-07= 01
08: 8-08= 0
05: 8-05=03
06: 8-06=02

STEP 2 Find the mean of the distances.
2+4+1+0+3+2/6
= 12/6 = 2

So, the mean absolute deviation of the data is 2.

Question 6.
Five recorded temperatures are 71°F, 64°F, 72°F, 81°F, and 67°F. What is the mean absolute deviation?
_______ °F

Answer: The mean absolute deviation of the data is 4.4.

Explanation:
Mean = 71+64+72+81+67/5 = 355/5 = 71

Mean absolute deviation:

STEP 1 Label each observation with its distance from the mean.
Starting from left to right:
71: 71-71= 0
64: 71-64= 07
72: 71-72= -1
81: 71-81=-10
67: 71-67= 04

STEP 2 Find the mean of the distances.
0+7+1+10+4/5
= 22/5 = 4.4

So, the mean absolute deviation of the data is 4.4.

Question 7.
Make a dot plot of the following data: 10, 10, 11, 12, 12, 13, 13, 15. Use the dot plot to find the mean absolute deviation.
Type below:
_________________

Answer: The mean absolute deviation of the data is 1.25

Explanation:
Mean = 10+10+11+12+12+13+13+15/8 = 96/8 = 12

Mean absolute deviation:

Box plot:

STEP 1 Label each dot with its distance from the mean.
Starting from left to right:
10: 12-10=02
11: 12-11=01
12: 12-12=0
13: 12-13=-1
15: 12-15=-3

STEP 2 Find the mean of the distances.
(2) + (2) +(1) +(0) +(0) +(1) + (1) +(3) / 8 = 10/8 = 1.25

So, the mean absolute deviation of the data is 1.25

Lesson Check – Page No. 724

Question 1.
Six test grades are 86, 88, 92, 90, 82, and 84. The mean of the data is 87. What is the mean absolute deviation?
_______

Answer: The mean absolute deviation of the data is 3.5

Explanation:

Mean absolute deviation:

STEP 1 Label each observation with its distance from the mean.
Starting from left to right:
86: 87-86= 01
88: 87-88= -1
92: 87-92= -5
90: 87-81= 06
82: 87-82= 05
84: 87-84= 03

STEP 2 Find the mean of the distances.
1+5+1+6+5+3/6
= 21/6 = 3.5

So, the mean absolute deviation of the data is 3.5

Question 2.
Eight heights in inches are 42, 36, 44, 46, 48, 42, 48, and 46. The mean of the data is 44. What is the mean absolute deviation?
_______ inches

Answer:

Explanation: The mean absolute deviation of the data is 3

Mean absolute deviation:

STEP 1 Label each observation with its distance from the mean.
Starting from left to right:
42: 44-42= 02
36: 44-36= 08
44: 44-44= 0
46: 44-46= -2
48: 44-48= -4
42: 44-42= 02
48: 44-48= -4
46: 44-46= -2

STEP 2 Find the mean of the distances.
2+8+2+4+0+2+4+2/8
= 24/8 = 3

So, the mean absolute deviation of the data is 3

Spiral Review

Question 3.
What is the volume of a rectangular prism with dimensions 4 meters, 1 \(\frac{1}{2}\) meters, and 5 meters?
_______ m3

Answer: 30m3

Explanation:
Dimentions: 4 meters, 1 1/2 meters, 5 meters
Change the mixed fraction into improper fraction = 3/2
Volume of the rectangle = 4 x 3/2 x 5 = 30m3

Question 4.
Carrie is making a frequency table showing the number of miles she walked each day during the 30 days of September. What value should she write in the Frequency column for 9 to 11 miles?
Go Math Grade 6 Answer Key Chapter 13 Variability and Data Distributions img 22
_______

Answer: 1

Explanation:
Total number of days in the month of September = 30
Number of days given in the frequency table = 17+8+4 = 29
Frequency in the interval 9 – 11 = 30 – 29 = 1 day

Question 5.
The following data shows the number of laps each student completed. What number of laps is the mode?
9, 6, 7, 8, 5, 1, 8, 10
The mode is _______ laps.

Answer: The mode is 8 laps.

Explanation:
The most frequently occurring observation is known as mode.
8 is the mode in the above raw data given.

Question 6.
What is the upper quartile of the following data?
43, 48, 55, 50, 58, 49, 38, 42, 50
The upper quartile is _______

Answer: The upper quartile is 58

Explanation:
The highest observation in the data given is known as upper quartile. The upper quartile is 58

Share and Show – Page No. 727

Question 1.
Find the range and interquartile range of the data in the box plot.
Go Math Grade 6 Answer Key Chapter 13 Variability and Data Distributions img 23
The range is $ __________ .
The interquartile range is $ __________ .

Answer: $12, $3

Explanation:
The difference between the highest observation and the lowest observation is called a range.
Range = 19 – 7 = $12
The difference and the highest and the lowest dots of the dot plot is called as interquartile range.
Interquartile range = 15 – 12 = $3

Practice: Copy and Solve Find the mean absolute deviation for the data set.

Question 2.
heights in inches of several tomato plants:
16, 18, 18, 20, 17, 20, 18, 17
_______ inch

Answer: The mean absolute deviation of the data is 1

Explanation:
Mean:
Mean = 16+18+18 +20+17+20+18+17/8 = 144/8 = 18

Mean absolute deviation:

STEP 1 Label each observation with its distance from the mean.
Starting from left to right:
16: 18-16= 02
18: 18-18= 0
18: 18-18= 0
20: 18-20= -2
17: 18-17= 01
20: 18-20= -2
18: 18-18= 0
17: 18-17= 01

STEP 2 Find the mean of the distances.
2+0+0+2+1+2+0+1/8
= 8/8 = 1

So, the mean absolute deviation of the data is 1

Question 3.
times in seconds for students to run one lap:
68, 60, 52, 40, 64, 40
_______ seconds

Answer: The mean absolute deviation of the data is 10

Explanation:
Mean:
Mean = 68+60+52+40+64+40/6 = 54

Mean absolute deviation:

STEP 1 Label each observation with its distance from the mean.
Starting from left to right:
68: 54-68= -14
60: 54-60= -6
52: 54-52= 02
40: 54-40= 14
64: 54-64= -10
40: 54-40= 14

STEP 2 Find the mean of the distances.
14+6+2+14+10+14/6
= 60/6 = 10

So, the mean absolute deviation of the data is 10

On Your Own

Use the box plot for 4 and 5.
Go Math Grade 6 Answer Key Chapter 13 Variability and Data Distributions img 24

Question 4.
What is the range of the data?
$ _______

Answer: $24

Explanation:
The difference between the highest observation and the lowest observation is called a range.
Range = $56 – $32 = $24

Question 5.
What is the interquartile range of the data?
$ _______

Answer: $16

Explanation:
The difference and the highest and the lowest dots of the dot plot is called as interquartile range.
Interquartile range = $52 – $36 = $16

Practice: Copy and Solve Find the mean absolute deviation for the data set.

Question 6.
times in minutes spent on a history quiz:
35, 35, 32, 34, 34, 32, 34, 36
_______ minute

Answer: The mean absolute deviation of the data is 1

Explanation:
Mean:
Mean = 35+ 35+ 32+ 34+34+ 32+ 34+36/8 = 272/8 = 34

Mean absolute deviation:

STEP 1 Label each observation with its distance from the mean.
Starting from left to right:
35: 34-35= -1
35: 34-35= -1
32: 34-32= 02
34: 34-34= 0
34: 34-34= 0
32: 34-32=02
34: 34-34=0
36: 34-36=-2

STEP 2 Find the mean of the distances.
1+1+2+0+0+2+0+2/8
= 8/8 = 1

So, the mean absolute deviation of the data is 1

Question 7.
number of excused absences for one semester:
1, 2, 1, 10, 9, 9, 10, 6, 1, 1
_______

Answer: The mean absolute deviation of the data is 3.8

Explanation:

Mean:
Mean =1+2+1+10+9+9+10+6+1+1 /10 = 50/10 = 5

Mean absolute deviation:

STEP 1 Label each observation with its distance from the mean.
Starting from left to right:
1: 5-1= 4
2: 5-2= 3
1: 5-1= 4
10: 5-10= -5
9: 5-9= -4
9: 5-9=-4
10: 5-10=-5
6: 5-6=-1
1: 5-1=4
1: 5-1=4

STEP 2 Find the mean of the distances.
4+3+4+5+4+4+5+1+4+4/10
=38/10 = 3.8

So, the mean absolute deviation of the data is 3.8

Question 8.
The chart shows the price of different varieties of dog food at a pet store. Find the range, interquartile range, and the mean absolute deviation of the data set.
Go Math Grade 6 Answer Key Chapter 13 Variability and Data Distributions img 25
Type below:
_________________

Answer:

The mean absolute deviation of the data is 3.6
Range = 32-16 = 16
Interquartile range = 24 – 20 = 4

Explanation:

Mean:
Mean =18+24+20+26+24+20+32+20+16+20 /10 = 220/10 = 22

Mean absolute deviation:

STEP 1 Label each observation with its distance from the mean.
Starting from left to right:
18: 22-18= 4
24: 22-24= -2
20: 22-20= 2
26: 22-26= -4
24: 22-24= -2
20: 22-20= 2
32: 22-32=-10
20: 22-20= 2
16: 22-16= 6
20: 22-20= 2

STEP 2 Find the mean of the distances.
4+2+2+4+2+2+10+2+6+2/10
=36/10 = 3.6

So, the mean absolute deviation of the data is 3.6
The difference between the highest observation and the lowest observation is called a range.
Range = 32-16 = 16
The difference and the highest and the lowest dots of the dot plot is called as interquartile range.
Interquartile range = 24 – 20 = 4

Problem Solving + Applications – Page No. 728

Question 9.
Hyato’s family began a walking program. They walked 30, 45, 25, 35, 40, 30, and 40 minutes each day during one week. At the right, make a box plot of the data. Then find the interquartile range.
_______ minutes

Answer: 35 minutes

Explanation:
Ascending order: 25,30,30,35,40,40,45
n=7 (odd)
Median= Number of (n+1/2) = 8/2 = 4th observation = 35
Median:(four terms of the data)
Median = 30+30/2 = 60/2 = 30
Median:(last 3 terms of the data)
Median = (n+1/2) = 2nd observation = 40
Interquartile range = 30+40/2 = 70/2 = 35

Question 10.
Compare Jack recorded the number of minutes his family walked each day for a month. The range of the data is 15. How does this compare to the data for Hyato’s family?
Type below:
_________________

Answer: Jack’s family walked less number of minutes each day compared to Hyato’s family.

Explanation:
The range of Hyato’s family is 20 while the range of Jack’s family is 15. Therefore we can say that Hyato’s family walked more minutes compared to Jack’s family in a day.
Range can define the data with large observations and the data with least observations.

Question 11.
Sense or Nonsense? Nathan claims that the interquartile range of a data set can never be greater than its range. Is Nathan’s claim sense or nonsense? Explain.
Type below:
_________________

Answer: Nonsense, Interquartile range of a data set can be less than or greater than range.

Explanation:
The interquartile range is the difference between the medians of the observations.
Nathan’s claim is nonsense as he said that, ” The interquartile range can never be greater than its range.”
The range is the difference between the highest observation and the lowest observation.
The interquartile range can be less than greater than the range.

Example:
Ascending order: 25,30,30,35,40,40,45
n=7 (odd)
Median= Number of (n+1/2) = 8/2 = 4th observation = 35
Median:(four terms of the data)
Median = 30+30/2 = 60/2 = 30
Median:(last 3 terms of the data)
Median = (n+1/2) = 2nd observation = 40
Interquartile range = 30+40/2 = 70/2 = 35

Range= 45-25 = 20

In the above case the interquartile range is more than the range proving that the given statement is nonsense.

Question 12.
The box plot shows the heights of corn stalks from two different farms.
Go Math Grade 6 Answer Key Chapter 13 Variability and Data Distributions img 26
The range of Farm A’s heights is _____ the range of Farm B’s heights.

Answer: greater than

Explanation:
The range is the difference between the highest and the lowest observations.
Range of Farm A: 72-58 = 14
Range of Farm B: 70-55 = 15

Therefore, The range of Farm A’s heights is greater than the range of Farm B’s heights.

Measures of Variability – Page No. 729

Question 1.
Find the range and interquartile range of the data in the box plot.
Go Math Grade 6 Answer Key Chapter 13 Variability and Data Distributions img 27
The range is __________ miles.
The interquartile range is __________ miles.

Answer: 16, 8

Explanation:
The difference between the highest and the lowest observations is range.
Range = 17 – 1 = 16
The difference between the highest and lowest observations of the box is the interquartile range.
Interquartile range = 12 – 4 = 8

Use the box plot for 2 and 3.
Go Math Grade 6 Answer Key Chapter 13 Variability and Data Distributions img 28

Question 2.
What is the range of the data?
_____

Answer: 35

Explanation:
The difference between the highest and the lowest observations is range.
Range = 95 – 60 = 35

Question 3.
What is the interquartile range of the data?
_____

Answer: 20

Explanation:
The difference between the highest and lowest observations of the box is the interquartile range.
Interquartile range = 90 – 70 = 20

Find the mean absolute deviation for the set.

Question 4.
heights in centimetres of several flowers:
14, 7, 6, 5, 13
_____ cm

Answer: The mean absolute deviation of the data is 3.6

Explanation:

Mean:
Mean =14+7+ 6+5+13/5= 45/5 = 9

Mean absolute deviation:

STEP 1 Label each observation with its distance from the mean.
Starting from left to right:
14: 9-14= -5
07: 9-07= 02
06: 9-06= 03
05: 9-05= 04
13: 9-13= -4

STEP 2 Find the mean of the distances.
5+2+3+4+4/5
= 18/5 = 3.6

So, the mean absolute deviation of the data is 3.6

Question 5.
ages of several children:
5, 7, 4, 6, 3, 5, 3, 7
_____ years

Answer: The mean absolute deviation of the data is 1.25

Explanation:
Mean:
Mean = 5+7+4+6+ 3+5+3+7/8 = 40/8 = 5

Mean absolute deviation:

STEP 1 Label each observation with its distance from the mean.
Starting from left to right:
5: 5-5= 0
7: 5-7= -2
4: 5-4= 01
6: 5-6= -1
3: 5-3= 02
5: 5-5= 0
3: 5-3= 02
7: 5-7=-2

STEP 2 Find the mean of the distances.
0+2+1+1+2+0+2+2/8
= 10/8 = 1.25

So, the mean absolute deviation of the data is 1.25

Problem Solving

Question 6.
The following data set gives the amount of time, in minutes, it took five people to cook a recipe. What is the mean absolute deviation for the data?
33, 38, 31, 36, 37
_____ minutes

Answer: The mean absolute deviation of the data is 2.4

Explanation:
Mean:
Mean = 33+38+31+36+37/5 = 175/5 = 35

Mean absolute deviation:

STEP 1 Label each observation with its distance from the mean.
Starting from left to right:
33: 35-33= 02
38: 35-38= -3
31: 35-31= 04
36: 35-36= -1
37: 35-37= -2

STEP 2 Find the mean of the distances.
2+3+4+1+2/5
= 12/5 = 2.4

So, the mean absolute deviation of the data is 2.4

Question 7.
The prices of six food processors are $63, $59, $72, $68, $61, and $67. What are the range, interquartile range, and mean absolute deviation for the data?
Type below:
_________________

Answer: Range = $9 The mean absolute deviation of the data is 4

Explanation:
The difference between the highest and the lowest observations is range.
Range = $68 – $59 = $9
The difference between the highest and lowest observations of the box is the interquartile range.
Interquartile range = 12 – 4 = 8

Mean:
Mean = $63+$59+$72+$68+$61+$67/6 = 390/6 = 65

Mean absolute deviation:

STEP 1 Label each observation with its distance from the mean.
Starting from left to right:
63: 65-63= 02
59: 65-59= 06
72: 65-72= -7
68: 65-68= -3
61: 65-61= -4
67: 65-67= -2

STEP 2 Find the mean of the distances.
2+6+7+3+4+2/6
= 24/6 = 4

So, the mean absolute deviation of the data is 4

Question 8.
Find the range, interquartile range, and mean absolute deviation for this data set: 41, 45, 60, 61, 61, 72, 80.
Type below:
_________________

Answer: The mean absolute deviation of the data is 9.7

Explanation:

Mean:
Mean = 41+45+60+61+61+72+80 /7 = 420/7 = 60

Mean absolute deviation:

STEP 1 Label each observation with its distance from the mean.
Starting from left to right:
41: 60-41= 19
45: 60-45= 15
60: 60-60= 0
61: 60-61= -1
61: 60-61= -1
72: 60-72= -12
80: 60-80= -20

STEP 2 Find the mean of the distances.
19+15+0+1+1+12+20/7
= 68/7 = 9.7

So, the mean absolute deviation of the data is 9.7

Lesson Check – Page No. 730

Question 1.
Daily high temperatures recorded in a certain city are 65°F, 66°F, 70°F, 58°F, and 61°F. What is the mean absolute deviation for the data?
_____ °F

Answer: The mean absolute deviation of the data is 3.6

Explanation:

Mean:
Mean = 65+66+70+58+61 /5 = 320/5 = 64

Mean absolute deviation:

STEP 1 Label each observation with its distance from the mean.
Starting from left to right:
65: 64-65=-1
66: 64-66=-2
70: 64-70=-6
58: 64-58=06
61: 64-61=03

STEP 2 Find the mean of the distances.
1+2+6+6+3/5
= 18/5 = 3.6

So, the mean absolute deviation of the data is 3.6

Question 2.
Eight different cereals have 120, 160, 135, 144, 153, 122, 118, and 134 calories per serving. What is the interquartile range for the data?
_____ calories

Answer: 42cereals

Explanation:
Ascending order of the data: 118,120,122,134,135,144,153,160
Median:(for first 4 terms)
Median= 120+122/2 = 242/2 = 121
Median:(for first 4 terms)
Median= 144+153/2 = 297/2 = 148.5
The difference and the highest and the lowest dots of the dot plot is called as interquartile range.
Interquartile range = 148.5 – 121 = 27.5

Spiral Review

Question 3.
Look at the histogram. How many days did the restaurant sell more than 59 pizzas?
Go Math Grade 6 Answer Key Chapter 13 Variability and Data Distributions img 29
________

Answer: 20

Explanation:
After 59 there is 1 interval 60-79
Number of days the restaurant sell more than 59 pizzas = 20

Question 4.
Look at the histogram. Where does a peak in the data occur?
Go Math Grade 6 Answer Key Chapter 13 Variability and Data Distributions img 30
Type below:
_________________

Answer: 20 – 39

Explanation:
Number of days the restaurant sold the maximum pizzas = 30
Number pizzas sold in each day = 20 – 39

Question 5.
What is the mode of the data set?
14, 14, 18, 20
The mode is ________

Answer: 14

Explanation:
The most frequently occurring observation is known as a mode.
In the above data mode is 14.

Question 6.
The data set below lists the ages of people on a soccer team. The mean of the data is 23. What is the mean absolute deviation?
24, 22, 19, 19, 23, 23, 26, 27, 24
________

Answer: The mean absolute deviation of the data is 2

Explanation:

Mean absolute deviation:

STEP 1 Label each observation with its distance from the mean.
Starting from left to right:
24: 23-24=-1
22: 23-22= 1
19: 23-19= 4
19: 23-19= 4
23: 23-23=0
23: 23-23=0
26: 23-26=-3
27: 23-27=-4
24: 23-24=-1

STEP 2 Find the mean of the distances.
1+1+4+4+3+4+1+0+0/9
= 18/9 = 2

So, the mean absolute deviation of the data is 2

Mid-Chapter Checkpoint – Vocabulary – Page No. 731

Choose the best term from the box to complete the sentence.
Go Math Grade 6 Answer Key Chapter 13 Variability and Data Distributions img 31

Question 1.
The _____ is the difference between the upper quartile and the lower quartile of a data set.
Type below:
_________________

Answer: Range

Explanation:
The difference between the upper and lower quartiles of the data is known as range.

Question 2.
A graph that shows the median, quartiles and least and greatest values of a data set is called a(n) _____.
Type below:
_________________

Answer: Box plot

Explanation:
The figure which shows the median, quartiles and least and greatest values of a data set is called a box plot, a box plot is a figure which represents median with a horizontal line and the starting and ending line represents the upper and lower quartiles and the end dots represent the upper limit and the lower limit.

Question 3.
The difference between the greatest value and the least value in a data set is the _____.
Type below:
_________________

Answer: Range

Explanation:
Each data set consists of upper and lower limits the difference between these limits is called as range.

Question 4.
The _____ is the mean of the distances between the values of a data set and the mean of the data set.
Type below:
_________________

Answer: Mean absolute

Explanation:
Mean absolute deviation is calculated by subtracting each observation from the mean and then the mean is calculated for these observations.
Therefore we can say that the mean absolute is the mean of the distances between the values of a data set and the mean of the data set.

Concepts and Skills

Question 5.
Make a box plot for this data set: 73, 65, 68, 72, 70, 74.
Type below:
_________________

Answer: Median = 71

Explanation:
Median:
Ascending order: 65, 68, 70, 72, 73,74
n = even = 6
Median = Mean of 3rd and 4th terms
= 70+72/2 = 142/2 = 71

Find the mean absolute deviation of the data.

Question 6.
43, 46, 48, 40, 38
________

Answer: The mean absolute deviation of the data is 3.2

Explanation:
Mean:

Mean = 43+46+48+40+38/5 = 215/5 = 43

Mean absolute deviation:

STEP 1 Label each observation with its distance from the mean.
Starting from left to right:
43: 43-43=0
43: 43-46=-3
43: 43-48=-5
43: 43-40= 3
43: 43-38= 5

STEP 2 Find the mean of the distances.
0+3+5+3+5/5 = 16/5 = 3.2

So, the mean absolute deviation of the data is 3.2

Question 7.
26, 20, 25, 21, 24, 27, 26, 23
________

Answer: The mean absolute deviation of the data is 2.125

Explanation:
Mean:

Mean = 26+20+25+21+24+27+26+23/8 = 192/8 = 24

Mean absolute deviation:

STEP 1 Label each observation with its distance from the mean.
Starting from left to right:
26: 24-26=-3
20: 24-20= 4
25: 24-25= -1
21: 24-21= 3
24: 24-24=0
27: 24-27=-3
26: 24-26=-2
23: 24-23=1

STEP 2 Find the mean of the distances.
3+4+1+3+0+3+2+1/8
= 17/8 = 2.125

So, the mean absolute deviation of the data is 2.125

Question 8.
99, 70, 78, 85, 76, 81
________

Answer: The mean absolute deviation of the data is 2

Explanation:
Mean:

Mean = 99+70+78+85+76+81/6 = 489/6 = 81.5

Mean absolute deviation:

STEP 1 Label each observation with its distance from the mean.
Starting from left to right:
24: 23-24=-1
22: 23-22= 1
19: 23-19= 4
19: 23-19= 4
23: 23-23=0
23: 23-23=0
26: 23-26=-3
27: 23-27=-4
24: 23-24=-1

STEP 2 Find the mean of the distances.
1+1+4+4+3+4+1+0+0/9
= 18/9 = 2

So, the mean absolute deviation of the data is 2

Find the range and interquartile range of the data.

Question 9.
2, 4, 8, 3, 2
The range is _________ .
The interquartile range is _________ .

Answer: 6

Explanation:
The difference between the upper quartile and lower quartile.
Range = 8 – 2 =6

Question 10.
84, 82, 86, 87, 88, 83, 84
The range is _________ .
The interquartile range is _________ .

Answer: 6

Explanation:
The difference between the upper quartile and lower quartile.
Range = 88 – 82 = 6

Question 11.
39, 22, 33, 45, 42, 40, 28
The range is _________ .
The interquartile range is _________ .

Answer: 23

Explanation:
The difference between the upper quartile and lower quartile.
Range = 45 – 22 = 23

Page No. 732

Question 12.
Yasmine keeps track of the number of hockey goals scored by her school’s team at each game. The dot plot shows her data.
Go Math Grade 6 Answer Key Chapter 13 Variability and Data Distributions img 32
Where is there a gap in the data?
Type below:
_________________

Answer: There is a gap in the data in the intervals: between 1 and 2

Explanation:
As shown in the dot plot there is a gap between 1 and 2. This means that Yasmine’s team did not score only one goal when they played the game.

Question 13.
What is the interquartile range of the data shown in the dot plot with Question 12?
The interquartile range is _________ .

Answer: 2

Explanation:
The interquartile range is the difference between the lower and upper quartiles.
Interquartile range = 3-1 = 2

Question 14.
Randall’s teacher added up the class scores for the quarter and used a histogram to display the data. How many peaks does the histogram have? Explain how you know.
Go Math Grade 6 Answer Key Chapter 13 Variability and Data Distributions img 33
Type below:
_________________

Answer: The graph has only one peak

Explanation:
In the given graph there is are rectangles out of which one has a tall rectangle which can be addressed as peak.

Question 15.
In a box plot of the data below, where would the box be drawn?
55, 37, 41, 62, 50, 49, 64
Type below:
_________________

Answer: The box is drawn above the number line and this represents the median and the lower and upper limits.

Explanation:
A box is drawn to represent the median and  the upper and lower limits in a box plot.

Share and Show – Page No. 735

Question 1.
The distances in miles students travel to get to school are 7, 1, 5, 9, 9, and 8. Decide which measure(s) of center best describes the data set. Explain your reasoning.
Type below:
_________________

Answer: Mean: 6.5
Median: 7.5
Mode: 9

Explanation:
Mean:
7+1+5+9+9+8/6 = 39/6 = 6.5
Median:
Ascending order: 1,5,7,8,9,9
Median = Mean of 7 and 8 = 7+8/2 = 15/2 = 7.5
Mode:
The most frequently occurring observation is known as the mode.
The mode is 9.

Question 2.
Use Graphs The numbers of different brands of orange juice carried in several stores are 2, 1, 3, 1, 12, 1, 2, 2, and 5. Make a box plot of the data and find the range and interquartile range. Decide which measure better describes the data set and explain your reasoning.
Type below:
_________________

Answer: Range: 11      Interquartile: 3 Interquartile range is the best way to represent the data.

Explanation:
Range = 12 – 1 = 11
Interquartile range :
Median(of first 4 terms):
Median = 1+1/2 = 2/2 = 1
Median (of last 4 terms):
Median = 3+5/2 = 8/2 = 4
Interquartile range = 4 – 1 = 3

On Your Own

Question 3.
Use Reasoning The ages of students in a computer class are 14, 13, 14, 15, 14, 35, 14. Decide which measure of center(s) best describes the data set. Explain your reasoning.
Type below:
_________________

Answer: Mean:17  Median:14   Mode:14  Median and mode is the best ways to represent the data.

Explanation:
Mean:
14+13+14+15+14+35+14/7 = 119/7 = 17
Median:
Ascending order: 13,14,14,14,14,15,35
Median = 14
Mode:
The most frequently occurring observation is known as the mode.
The mode is 14.

Question 4.
Mateo scored 98, 85, 84, 80, 81, and 82 on six math tests. When a seventh math test score is added, the measure of center that best describes his scores is the median. What could the seventh test score be? Explain your reasoning.
Type below:
_________________

Answer: Median is the best way to represent the data.

Explanation:
Median:
Ascending order: 80,81,82,84,85,98
Median = Mean of 3 and 4 = 82+84/2 = 166/2 = 83
The seventh score can be 83

Unlock the Problem – Page No. 736

Question 5.
Jaime is on the community swim team. The table shows the team’s results in the last 8 swim meets. Jaime believes they can place in the top 3 at the next swim meet. Which measure of center should Jaime use to persuade her team that she is correct? Explain.
Go Math Grade 6 Answer Key Chapter 13 Variability and Data Distributions img 34
a. What do you need to find?
Type below:
_________________

Answer: Mean, median,mode

Explanation:
Mean= 1+2+2+3+3+1+18+2/8 = 32/8 = 4
Median:
Ascending order: 1,1,2,2,2,3,3,18
Median = 2+2/2 = 2
Mode:
The most frequently occurring observation is called a mode.
Mode=2

Question 5.
b. What information do you need to solve the problem?
Type below:
_________________

Answer: We need to have the data to find the centre of tendencies.

Explanation:
The given data can be used to find the mean, median and mode.

Question 5.
c. What are the measures of center?
Type below:
_________________

Answer: Mean = 4 Median = 2 Mode = 2

Explanation:
There are three measures to calculate their approximate values.

Question 5.
d. Which measure of center should Jaime use? Explain.
Type below:
_________________

Answer: Median or mode

Explanation:
Median or mode are nearer to the solution, therefore, they can be used.

Question 6.
The numbers of sit-ups students completed in one minute are 10, 42, 46, 50, 43, and 49. The mean of the data values is 40 and the median is 44.5. Which measure of center better describes the data, the mean or median? Use words and numbers to support your answer.
Type below:
_________________

Answer: Median is the better way to represent the data.

Explanation:
44.5 is closer and represents the more number of observations compared to the mean.

Choose Appropriate Measures of Center and Variability – Page No. 737

Question 1.
The distances, in miles, that 6 people travel to get to work are 14, 12, 2, 16, 16, and 18. Decide which measure(s) of center best describes the data set. Explain your reasoning.
Type below:
_________________

Answer: Mean= 13 miles Median= 15 miles Mode= 16 miles

Explanation:
Mean is less than the data points.
Median describes the data in the best way compared to mean and mode.

Question 2.
The numbers of pets that several children have are 2, 1, 2, 3, 4, 3, 10, 0, 1, and 0. Make a box plot of the data and find the range and interquartile range. Decide which measure better describes the data set and explain your reasoning.
Type below:
_________________

Answer: Range = 10-0 = 10
Interquartile range = 3.5 – 0.5 = 3
The interquartile range is the best way to represent the data.

Explanation:
Ascending order: 0,0,1,1,2,2,3,3,4,10
Median = 2+2/2 = 2
Lower quartile = 0.5
Upper quartile= 7/2 = 3.5
Highest observation= 10
Lowest observation = 0
Range = 10-0 = 10
Interquartile range = 3.5 – 0.5 = 3
The interquartile range is the best way to represent the data.

Problem Solving

Question 3.
Brett’s history quiz scores are 84, 78, 92, 90, 85, 91, and 0. Decide which measure(s) of center best describes the data set. Explain your reasoning.
Type below:
_________________

Answer: Mean is the best measure of centre to describe the data set.

Explanation:
Mean:
Mean= 84+78+92+90+85+91+0/7 = 74.2
Median:
Ascending order: 0,78,84,85,90,91,92
Median = 4th observation = 85

Question 4.
Eight students were absent the following number of days in a year: 4, 8, 0, 1, 7, 2, 6, and 3. Decide if the range or interquartile range better describes the data set, and explain your reasoning.
Type below:
_________________

Answer: 8 represents all the terms range is more preferable compared to the interquartile range.

Explanation:
Ascending order: 0,1,2,3,4,6,7,8
Range = 8-0 = 8
Median = Mean of 3 and 4 = 3+4/2 = 7/2 = 3.5
Median of first 3 terms = 1
Median of last 3 terms = 7
Interquartile range = 7-1 = 6
Since 8 represents all the terms range is more preferable compared to the interquartile range.

Question 5.
Create two sets of data that would be best described by two different measures of centre.
Type below:
_________________

Answer: The given below are the examples of two sets of data that would be best described by two different measures of centre.

Explanation:
Example 1:

The numbers of pets that several children have are 2, 1, 2, 3, 4, 3, 10, 0, 1, and 0. Make a box plot of the data and find the range and interquartile range. Decide which measure better describes the data set and explain your reasoning.
Type below:
_________________

Answer: Range = 10-0 = 10
Interquartile range = 3.5 – 0.5 = 3
The interquartile range is the best way to represent the data.

Explanation:
Ascending order: 0,0,1,1,2,2,3,3,4,10
Median = 2+2/2 = 2
Lower quartile = 0.5
Upper quartile= 7/2 = 3.5
Highest observation= 10
Lowest observation = 0
Range = 10-0 = 10
Interquartile range = 3.5 – 0.5 = 3
The interquartile range is the best way to represent the data.

Example 2:

Brett’s history quiz scores are 5,6,7,8,9,10. Decide which measure(s) of centre best describes the data set. Explain your reasoning.
Type below:
_________________

Answer: Mean and median are the best measure of centre to describe the data set.

Explanation:
Mean:
Mean= 5+6+7+8+9+10/6 = 7.5
Median:
Ascending order: 5,6,7,8,9,10
Median = Mean of 7 and 8 = 15/2 = 7.5

Lesson Check – Page No. 738

Question 1.
Chloe used two box plots to display some data. The box in the plot for the first data set is wider than the box for the second data set. What does this say about the data?
Type below:
_________________

Answer: The graphs say that the interquartile range is more for the second graph compared to the first.

Explanation:
The interquartile range is the difference between the lower and upper quartiles.
It is more for wider data compared to the compact data.

Question 2.
Hector recorded the temperature at noon for 7 days in a row. The temperatures are 20°F, 20°F, 20°F, 23°F, 23°F, 23°F, and 55°F. Which measure of center would best describe the data?
Type below:
_________________

Answer: Mode

Explanation:
The most frequently occurring observation is known as mode.
The mode of the above data describes the data well, the mode of the data is 20°F

Spiral Review

Question 3.
By how much does the median of the following data set change if the outlier is removed?
13, 20, 15, 19, 22, 26, 42
Type below:
_________________

Answer: 0.5

Explanation:
Median:
Ascending order: 13,15,19,20,22,26,42
Median = 20
If the outlier is removed then the median=
19+20/2 = 39/2 = 19.5
The difference in the medians = 0.5

Question 4.
What percent of the people surveyed spent at least an hour watching television?
Go Math Grade 6 Answer Key Chapter 13 Variability and Data Distributions img 35
_______ %

Answer: 8 people

Explanation:
Total number of people= 40
Percentage = 8/40 x 100 = 20%

Question 5.
What is the lower quartile of the following data?
12, 9, 10, 8, 7, 12
The lower quartile is _______ .

Answer:

Explanation:
Ascending order: 7,8,9,10,12,12
Median = 9+10/2 = 9.5
Lower quartile = 8

Question 6.
What is the interquartile range of the data shown in the box plot?
Go Math Grade 6 Answer Key Chapter 13 Variability and Data Distributions img 36
The interquartile range is _______ .

Answer: 5

Explanation:
The difference between the upper and lower quartiles is called as interquartile range.
Interquartile range = 14 – 9 = 5

Share and Show – Page No. 741

Question 1.
Zoe collected data on the number of points her favourite basketball players scored in several games. Use the information in the table to compare the data.
Go Math Grade 6 Answer Key Chapter 13 Variability and Data Distributions img 37
The mean of Player 1’s points is __ the mean of Player 2’s
points.
The interquartile range of Player 1’s points is __ the
interquartile range of Player 2’s points.
So, Player 2 typically scores __ points than Player 1, but
Player 2’s scores typically vary __ Player 1’s scores
Type below:
_________________

Answer: less than ; less than ; more ; more

Explanation:
The mean of Player 1’s points is less than the mean of Player 2’s points.
The interquartile range of Player 1’s points is less than the interquartile range of Player 2’s points.
So, Player 2 typically scores more points than Player 1, but Player 2’s scores typically vary in more Player 1’s scores

Question 2.
Mark collected data on the weights of puppies at two animal shelters. Find the median and range of each data set, and use these measures to compare the data.
Go Math Grade 6 Answer Key Chapter 13 Variability and Data Distributions img 38
Type below:
_________________

Answer: They differ slightly but on an average we can say that shalter B is more as compared to shelter A

Explanation:
Shelter A
Median:
Ascending order: 5,7,7,7,10,12,15
Median = 4th observation = 7
Range = 15-5 = 10
Shelter B
Median:
Ascending order: 4,5,5,11,11,13,15
Median = 4th observation = 11
Range = 15-4 = 11

On Your Own

Kwan analyzed data about the number of hours musicians in her band practice each week. The table shows her results. Use the table for Exercises 3–5.
Go Math Grade 6 Answer Key Chapter 13 Variability and Data Distributions img 39

Question 3.
Which two students typically practised the same amount each week, with about the same variation in practice times?
Type below:
_________________

Answer: Sally and Jennifer

Explanation:
They are slightly different but on a whole, the average shows no difference and we can say that Sally and Jennifer practised for the same amount each week

Question 4.
Which two students typically practised the same number of hours, but had very different variations in their practice times?
Type below:
_________________

Answer: Tim and Sally

Explanation:
They are different in range but on a whole, the average shows no difference and we can say that Sally and Tim practised for the same number of hours, but had very different variations in their practice times.

Question 5.
Which two students had the same variation in practice times, but typically practised a different number of hours per week?
Type below:
_________________

Answer: Matthew and Tim

Explanation:
Matthew and Tim practised for the same number of hours but they had a high variation in the range.

Problem Solving + Applications – Page No. 742

Question 6.
Compare The table shows the number of miles Johnny ran each day for two weeks. Find the median and the interquartile range of each data set, and use these measures to compare the data sets.
Go Math Grade 6 Answer Key Chapter 13 Variability and Data Distributions img 40
Type below:
_________________

Answer: Interquartile range is the best way to compare the data in the week 1
While the median is the best way to compare the data in the week 2

Explanation:
Week 1
Median:
Ascending order: 1,2,2,3,3,4,5
Median = 4th observation = 3
Lower quartile range= 2
Upper quartile range= 4
Interquartile range = 4-2 = 2

Week 2
Median:
Ascending order: 1,1,1,3,3,8,8
Median = 4th observation = 3
Lower quartile range= 1
Upper quartile range= 8
Interquartile range = 8-1 = 7

Question 7.
Sense or Nonsense? Yashi made the box plots at right to show the data he collected on plant growth. He thinks that the variation in bean plant growth was about the same as the variation in tomato plant growth. Does Yashi’s conclusion make sense? Why or why not?
Go Math Grade 6 Answer Key Chapter 13 Variability and Data Distributions img 41
Type below:
_________________

Answer: Sense

Explanation:
Yashi said that thinks that the variation in bean plant growth was about the same as the variation in tomato plant growth.
It is a true statement because the range of both bean and tomato plants growth is the same and they have the same medians.

Question 8.
Kylie’s teacher collected data on the heights of boys and girls in a sixth-grade class. Use the information in the table to compare the data.
Go Math Grade 6 Answer Key Chapter 13 Variability and Data Distributions img 42
The mean of the boys’ heights is _____ the mean of the girls’ heights.
The range of the boys’ heights is _____ the range of the girls’ heights.

Answer: more than ; more than

Explanation:
Mean of boys height:
Mean = 72+68+70+56+58+62+64/7 = 64.2
Range= 72-56 = 16

Mean of girls height:
Mean = 55+60+56+51+60+63+65/7 = 58.5
Range= 65-51 = 14
The mean of the boys’ heights is _more than____ the mean of the girls’ heights.
The range of the boys’ heights is _more than____ the range of the girls’ heights.

Apply Measures of Center and Variability – Page No. 743

Solve.

Question 1.
The table shows temperature data for two cities. Use the information in the table to compare the data.
Go Math Grade 6 Answer Key Chapter 13 Variability and Data Distributions img 43
The mean of City 1’s temperatures is the ———————– mean of City 2’s temperatures.
The ———————- of City 1’s temperatures is————— the —————–of City 2’s temperatures.
So, City 2 is typically —————-City 1, but City 2’s temperatures
vary ——————-City 1’s temperatures.
Type below:
_________________

Answer: less than; interquartile range; less than; interquartile range; warmer than; more than

Explanation:
The mean of City 1’s temperatures is the —-less than———- mean of City 2’s temperatures.
The –interquartile range——— of City 1’s temperatures is—less than—– the —-interquartile range—–of City 2’s temperatures.
So, City 2 is typically —warmer than—–City 1, but City 2’s temperatures
vary —–more than——-City 1’s temperatures.

Question 2.
The table shows weights of fish that were caught in two different lakes. Find the median and range of each data set, and use these measures to compare the data.
Go Math Grade 6 Answer Key Chapter 13 Variability and Data Distributions img 44
Type below:
_________________

Answer: Lake A’s average is greater but varies more.

Explanation:
Lake A
Median:
Ascending order: 4,6,7,9,10,12
Median = Mean 3rd and 4th observation = 7+9/2 = 8
Range = 12 – 4 = 8

Lake B
Median:
Ascending order: 4,4,5,6,6,7
Median = Mean 3rd and 4th observation = 5+6/2 = 5.5
Range = 7 – 3 = 4

Problem Solving

Question 3.
Mrs. Mack measured the heights of her students in two classes. Class 1 has a median height of 130 cm and an interquartile range of 5 cm. Class 2 has a median height of 134 cm and an interquartile range of 8 cm. Write a statement that compares the data.
Type below:
_________________

Answer: Class 2 is greater but varies more.

Explanation:
The interquartile range is the difference between the lower and upper quartiles. Since the interquartile range is more for class 2 we can say that the extremes are greater while the interquartile range is less for class 1 which means that the data is compact.

Question 4.
Richard’s science test scores are 76, 80, 78, 84, and 80. His math test scores are 100, 80, 73, 94, and 71. Compare the medians and interquartile ranges.
Type below:
_________________

Answer: Medians are equal but the interquartile range varies a large, math test scores are more spread out compared to science test scores.

Explanation:
Science test scores:
Median:
Ascending order: 76,78,80,80,84
Median = 3rd observation = 80
Interquartile range = 84 – 76 = 10

Math test scores:
Median:
Ascending order: 71,73,80,94,100
Median = 3rd observation = 80
Interquartile range = 100 – 71 = 29

Medians are equal but the interquartile range varies a large, math test scores are more spread out compared to science test scores.

Question 5.
Write a short paragraph to a new student that explains how you can compare data sets by examining the mean and the interquartile range.
Type below:
_________________

Answer: average and consistency

Explanation:
If the mean is more it means that the data has more observations or observations with more value.
Interquartile range and median range say about the consistency.

Lesson Check – Page No. 744

Question 1.
Team A has a mean of 35 points and a range of 8 points. Team B has a mean of 30 points and a range of 7 points. Write a statement that compares the data.
Type below:
_________________

Answer: Similar variation but team A average is more than team B

Explanation:
The range has only a difference of 1 point which can be said as a slight variation but while the average/ mean has a large variation.

Question 2.
Jean’s test scores have a mean of 83 and an interquartile range of 4. Ben’s test scores have a mean of 87 and an interquartile range of 9. Compare the students’ scores.
Type below:
_________________

Answer: Ben’s average is more than Jean’s but Ben is less consistent compared to Jean.

Explanation:
Ben’s average score is more than the average scores of Jean while the interquartile range

Spiral Review

Question 3.
Look at the box plots below. What is the difference between the medians for the two groups of data?
Go Math Grade 6 Answer Key Chapter 13 Variability and Data Distributions img 45
_______ students

Answer: 2 students

Explanation:
Median of students in a class of school A = 24
Median of students in a class of school B = 26
Difference between the medians of the schools = 26-24 = 2 students

Question 4.
The distances in miles that 6 people drive to get to work are 10, 11, 9, 12, 9, and 27. What measure of center best describes the data set?
Type below:
_________________

Answer: Median is the centre best describes the data set

Explanation:
Median:
AScending order: 9,9,10,11,12,27
Median= 3rd and 4th observations = 10+11/2 = 21/2 = 10.5

Question 5.
Which two teams typically practice the same number of hours, but have very different variations in their practice times?
Go Math Grade 6 Answer Key Chapter 13 Variability and Data Distributions img 46
Type below:
_________________

Answer: Team A and C

Explanation:
Team A and C have medians which nare only slightly different but the consistency varies a lot that is range.

Share and Show – Page No. 747

Connie asked people their ages as they entered the food court at the mall. Use the histogram of the data she collected for 1–5.
Go Math Grade 6 Answer Key Chapter 13 Variability and Data Distributions img 47

Question 1.
What statistical question could Connie ask about her data?
Type below:
_________________

Answer: Mean, Median, Mode

Explanation:
The graph shows the age and number of people. The questions which can be asked can be of mean, median, mode.

Question 2.
Describe any peak or gap in the data.
Type below:
_________________

Answer: Peak : 21-30   Gap : 61-70

Explanation:
There is a peak in the graph at the interval 21-30
There is a gap in between the bars of the histogram the bar which had a gap before it was 61-70

Question 3.
Does the graph have symmetry? Explain your reasoning.
Type below:
_________________

Answer: No the graph doesn’t have symmetry

Explanation:
The symmetry of the graph means there must be equal parts of the graph on both the sides of the line of the graph.
This is not possible in the above situation.

On Your Own

Question 4.
The lower quartile of the data set is 16.5 years, and the upper quartile is 51.5 years. Find the interquartile range. Is it a better description of the data than the range? Explain your reasoning.
Type below:
_________________

Answer: Interquartile range = 35; The interquartile range is better than the range.

Explanation:
The interquartile range is the difference between the upper quartile and the lower quartile.
Interquartile range = 51.5-16.5 = 35
The interquartile range is better than the range because if we take the example of the above graph we can see thatthe most of the data fall in the range of the interquartile range ie. 35.
Therefore we can say that the interquartile range is better than the range.

Question 5.
Make Arguments The mode of the data is 16 years old. Is the mode a good description of the center of the data? Explain
Type below:
_________________

Answer: No mode is not a good description of the data.

Explanation:
The mode is just a frequently occurring observation.
It cannot be the best way to describe the data.

Problem Solving + Applications – Page No. 748

Use the dot plot for 6–8.

Question 6.
Make Arguments Jason collected data about the number of songs his classmates bought online over the past 3 weeks. Does the data set have symmetry? Why or why not?
Go Math Grade 6 Answer Key Chapter 13 Variability and Data Distributions img 48
Type below:
_________________

Answer: No, the data has no symmetry.

Explanation:
No, the data has no symmetry. Because there are gaps between the dots drawn.

Question 7.
Jason claims that the median is a good description of his data set, but the mode is not. Does his statement make sense? Explain.
Type below:
_________________

Answer: Median can be a better centre of description. Therefore his statement makes a sense.

Explanation:
Median = 7+8/2 = 15/2 = 7.5
The number 7.5 represents more number of observations.

Question 8.
Trinni surveyed her classmates about how many siblings they have. A dot plot of her data increases from 0 siblings to a peak at 1 sibling and then decreases steadily as the graph goes to 6 siblings. How is Trinni’s dot plot similar to Jason’s? How is it different?
Type below:
_________________

Answer: Trinni graph represents a part of Jason’s graph

Explanation:
In Jason’s graph, there is a peak in the middle and then it decreases on both sides.
But according to Trinni graph, there is no peak in the middle.

Question 9.
Diego collected data on the number of movies seen last month by a random group of students.
Go Math Grade 6 Answer Key Chapter 13 Variability and Data Distributions img 49
Draw a box plot of the data and use it to find the interquartile range and range.
Type below:
_________________

Answer: Range = 12  Interquartile range = 2

Explanation:
The range is the difference between the highest and lowest observations.
Range = 12-0 = 12
The interquartile range is the difference between the upper and lower quartiles.
Ascending order: 0,0,1,1,2,2,2,2,3,3,3,5,12
Lower quartile = 1+1/2 = 2/2 = 1
Upper quartile = 3+3/2 = 6/2 = 3
Interquartile range = 3-1 = 2

Describe Distributions – Page No. 749

Chase asked people how many songs they have bought online in the past month. Use the histogram of the data he collected for 1–4.
Go Math Grade 6 Answer Key Chapter 13 Variability and Data Distributions img 50

Question 1.
What statistical question could Chase ask about the data?
Type below:
_________________

Answer: What is the median number of songs purchased?

Explanation:
Many questions can be formed from the data given to us
We can ask about the mean, median, mode.

Question 2.
Describe any peaks in the data.
Type below:
_________________

Answer: Peak : 0-4

Explanation:
The peak is a bar in the histogram which has the highest value. The peak of the given graph is 0-4

Question 3.
Describe any gaps in the data.
Type below:
_________________

Answer: There are no gaps in the graph.

Explanation:
The gap is something between a bar of the histogram and all the other adjacent bars.
There no such case in the graph. Therefore there are no gaps in the graph.

Question 4.
Does the graph have symmetry? Explain your reasoning.
Type below:
_________________

Answer: No the graph doesn’t have symmetry

Explanation:
The symmetry of the graph means there must be equal parts of the graph on both the sides of the line of the graph.
This is not possible in the above situation.

Problem Solving

Question 5.
Mr. Carpenter teaches five classes each day. For several days in a row, he kept track of the number of students who were late to class and displayed the results in a dot plot. Describe the data.
Go Math Grade 6 Answer Key Chapter 13 Variability and Data Distributions img 51
Type below:
_________________

Answer: Peaks: At 6 and 8
Gaps: Between 3 and 5 , 6 and 8
Clusters: Between 0-3 ; 9-11

Explanation:
The highest points in the graph are known as peaks.
They are at 6 and 8 in this graph.
The space between the dots in the dot plot graph is known as gaps.
Gaps are between 3 and 5; 6 and 8
The group of the dots in the dot plot are known as clusters.
The clusters are at the 0-3; 9-11

Question 6.
Describe how a graph of a data set can be used to understand the distribution of the data.
Type below:
_________________

Answer: Mean, median, mode

Explanation:
There are three measures of centre which can be used to describe the data given in the form of a graph.
The three measures of centre are mean, median, mode.

Lesson Check – Page No. 750

Question 1.
The ages of people in a restaurant are 28, 10, 44, 25, 18, 8, 47, and 30. What is the median age of the people in the restaurant?
_______ years old

Answer: Median age of the people in the restaurant is 26.5 approximately 27

Explanation:
Median:
Ascending order: 8,10,18,25,28,30,44,47
Median = Mean of 4th and 5th observations = 25+28/2 = 53/2 = 26.5

Question 2.
What is the median in the dot plot?
Go Math Grade 6 Answer Key Chapter 13 Variability and Data Distributions img 52
$ ________

Answer: 11

Explanation:
Median is the middlemost value and it is 11 in the above graph.
We need to consider the middle value by neglecting the same number on both the sides.

Spiral Review

Question 3.
Look at the dot plot. Where does a gap occur in the data?
Go Math Grade 6 Answer Key Chapter 13 Variability and Data Distributions img 53
Type below:
_________________

Answer: 30-33

Explanation:
The gap is a space between the intervals.
The intervals are 30-33.

Question 4.
Look at the dot plot. Where does a peak occur in the data?
Go Math Grade 6 Answer Key Chapter 13 Variability and Data Distributions img 54
Type below:
_________________

Answer: 37

Explanation:
The highest point in the graph is known as the peak.
The peak in the dot plot is 37.

Question 5.
Which two teams had similar variations in points earned, but typically earned a different number of points per game?
Go Math Grade 6 Answer Key Chapter 13 Variability and Data Distributions img 55
Type below:
_________________

Answer: Red and blue

Explanation:
The difference between the upper and lowest observations is called a range.
The range (consistency) in the data given is the same but they vary in the mean.
But we can say that Red and Blue teams typically earned a different number of points per game.

Question 6.
Manny’s monthly electric bills for the past 6 months are $140, $165, $145, $32, $125, and $135. What measure of center best represents the data?
Type below:
_________________

Answer: Median is the best way to represent the data. Median= 137.5

Explanation:
Median:
Ascending order: 32,125,135,140,145,165
Median = 135+140/2 = 275/2 = 137.5

Share and Show – Page No. 753

Question 1.
Josh is playing a game at the carnival. If his arrow lands on a section marked 25 or higher, he gets a prize. Josh will only play if most of the players win a prize. The carnival worker says that the average (mean) score is 28. The box plot shows other statistics about the game. Should Josh play the game? Explain your reasoning.
Go Math Grade 6 Answer Key Chapter 13 Variability and Data Distributions img 56
First, look at the median. The median is _ points.
Next, work backwards from the statistics.
The median is the __ value of the data.
So, at least __ of the values are scores
less than or equal to _.
Finally, use the statistics to draw a conclusion.
Type below:
_________________

Answer: middlemost value; 20; 15

Explanation:
First, look at the median. The median is 20 points.
Next, work backwards from the statistics.
The median is the middlemost value of the data.
So, at least 20 of the values are scores
less than or equal to 15.
Finally, use the statistics to draw a conclusion.

Question 2.
What if a score of 15 or greater resulted in a prize? How would that affect Josh’s decision? Explain.
Type below:
_________________

Answer: It doesn’t affect his decision.

Explanation:
Josh wanted to play only when most of the prizes were awarded.
Therefore if the minimum score was 15 or greater than it then they would get the prize. So there will be no difference in his decision.

Question 3.
A store collects data on the sales of DVD players each week for 3 months. The manager determines that the data has a range of 62 players and decides that the weekly sales were very inconsistent. Use the statistics in the table to decide if the manager is correct. Explain your answer.
Go Math Grade 6 Answer Key Chapter 13 Variability and Data Distributions img 57
Type below:
_________________

Answer: No the manager is not correct.

Explanation:
The range is given to be 62.
The range is correct when we check it with the help of the given data, therefore, the data is not inconsistent.

On Your Own – Page No. 754

Question 4.
Gerard is fencing in a yard that is 21 feet by 18 feet. How many yards of fencing material does Gerard need? Explain how you found your answer.
_______ yards

Answer: 78 yards of the fencing material is required

Explanation:
Length = 21 feet  Breadth = 18 feet
The perimeter of the rectangle = The number of yards of fencing material required = 2(l+b) = 2(21+18) = 2(39) = 78 yards

Question 5.
Susanna wants to buy a fish that grows to be about 4 in. long. Mark suggests she buys the same type of fish he has. He has five of these fish with lengths of 1 in., 1 in., 6 in., 6 in., and 6 in., with a mean length of 4 in. Should Susanna buy the type of fish that Mark suggests? Explain.
Type below:
_________________

Answer: Yes Susanna buy the type of fish that Mark suggests

Explanation:
The length of fish suggested by Mark is 4 in. long.
Mark has 5 fishes with a mean length of 4 in.
To buy a fish of that grows to be about 4 in. long. Susanna should buy the fishes suggested by Mark.

Question 6.
Look for a Pattern The graph shows the number of stamps that Luciano collected over several weeks. If the pattern continues, how many stamps will Luciano collect in Week 8? Explain.
Go Math Grade 6 Answer Key Chapter 13 Variability and Data Distributions img 58
_______ stamps

Answer: 7 stamps

Explanation:
In week 4 and 6, the number of stamps are 4, 5. Therefore in week 6 and 8, the number of stamps are 5, 7

Question 7.
The data set shows the number of hours Luke plays the piano each week. Luke says he usually plays the piano 3 hours per week. Why is Luke’s statement misleading?
Go Math Grade 6 Answer Key Chapter 13 Variability and Data Distributions img 59
Type below:
_________________

Answer: According to the question he should spend 3 hours per week. His statement is correct.

Explanation:
Sum of the data = 1+2+1+3+2+10+2 = 21
Number of days in a week = 7
Mean = 21/7 = 3 hours

Problem Solving Misleading Statistics – Page No. 755

Mr Jackson wants to make dinner reservations at a restaurant that has most meals costing less than $16. The Waterside Inn advertises that they have meals that average $15. The table shows the menu items.
Go Math Grade 6 Answer Key Chapter 13 Variability and Data Distributions img 60

Question 1.
What is the minimum price and the maximum price?
minimum: $ _________
maximum: $ _________

Answer: minimum: $6
maximum: $19

Explanation:
The minimum value is the most minimum price in the given data.
The maximum value is the most maximum price in the given data.

Question 2.
What is the mean of the prices?
$ ________

Answer: $15

Explanation:
Mean = sum of all the observations/ total number of observations = 6+16+18+16+18+19/6 = 93/6 = 15.2
Approximately therefore the mean of the observations is $15

Question 3.
Construct a box plot for the data.
Type below:
_________________

Answer: The box plot is a diagram which signifies the information about the data.

Explanation:

The box plot represents the range, lower and upper quartiles.

Question 4.
What is the range of the prices?
$ ________

Answer: the range is $13

Explanation:
The difference between the upper and lower observations is known as the range.
Range = 19-6 = $13

Question 5.
What is the interquartile range of the prices?
$ ________

Answer: $7.5

Explanation:
Ascending order: $6, $16, $16, $18, $18, $19
Median = 16+18/2 = 34/2 = 17
Lower quartile = 6+16/2 = 11
Upper quartile = 18+19/2 = 18.5
Interquartile range = 18.5-11 = 7.5

Question 6.
Does the menu match Mr. Jackson’s requirements? Explain your reasoning.
Type below:
_________________

Answer: Yes the menu matches Mr Jackson’s requirements.

Explanation:
Mr Jackson wants to make dinner arrangements with cost less than $16.
The mean of the items in the menu:
Mean = $6+$16+$16+$18+$18+$19/6 = 93/6 = $15.5
Therefore the requirements of Mr Jackson is satisfied.

Question 7.
Give an example of a misleading statistic. Explain why it is misleading.
Type below:
_________________

Answer: The Waterside Inn advertises the misleading statement.

Explanation:
According to the information given in the question, The Waterside Inn advertises that they have meals that average $15. But it is more than that, so this is the misleading statement.
The mean of the items in the menu:
Mean = $6+$16+$16+$18+$18+$19/6 = 93/6 = $15.5
Therefore the requirements of Mr Jackson is satisfied.

Lesson Check – Page No. 756

Question 1.
Mary’s science test scores are 66, 94, 73, 81, 70, 84, and 88. What is the range of Mary’s science test scores?
________

Answer: 28

Explanation:
The difference between the highest and the lowest observations is called a range.
Range = 94 – 66 = 28

Question 2.
The heights in inches of students on a team are 64, 66, 60, 68, 69, 59, 60, and 70. What is the interquartile range?
________

Answer: Interquartile range = 9

Explanation:
Ascending order: 59,60,60,64,66,68,69,70
Median = Mean of 64 and 66 = 64+66/2 = 130/2 = 65
Lower quartile = 60
Upper quartile = 69
Interquartile range = 69 – 60 = 9

Spiral Review

Question 3.
By how much does the median of the following data set change if the outlier is removed?
26, 21, 25, 18, 0, 28
Type below:
_________________

Answer: The median changes by 3.5

Explanation:
Ascending order: 0,18,21,25,26,28
Median = 21+28/2 = 49/2 = 24.5
If the outlier is removed then the
Median = 21
Difference between the 1st and 2nd median = 24.5 – 21 = 3.5

Question 4.
Look at the box plot. What is the interquartile range of the data?
Go Math Grade 6 Answer Key Chapter 13 Variability and Data Distributions img 61
________

Answer: Interquartile range = 6

Explanation:
The difference between the lower and upper quartiles is known as the interquartile range.
Interquartile range = 50 – 44 = 6

Question 5.
Erin is on the school trivia team. The table shows the team’s scores in the last 8 games. Erin wants to build confidence in her team so that they will do well in the last game. If a score of 20 is considered a good score, what measure of center would be best for Erin to use to motivate her teammates?
Go Math Grade 6 Answer Key Chapter 13 Variability and Data Distributions img 62
Type below:
_________________

Answer: Mean and median are the best centre of tendencies to compare the data.

Explanation:
Mean = 20+20+18+19+23+40+22+19/8 = 181/8 = 22.6
Median:
Ascending order: 18,19,19,20,20,22,23,40
Median = 40/2 = 20

Chapter 13 Review/Test – Page No. 757

Question 1.
The dot plot shows the number of chin-ups done by a gym class.
Go Math Grade 6 Answer Key Chapter 13 Variability and Data Distributions img 63
For numbers 1a–1e, choose Yes or No to indicate whether the statement is correct.
1a. There are two peaks.
1b. There are no clusters.
1c. There is a gap from 6 to 8.
1d. The most chin-ups anyone did was 15.
1e. The modes are 3, 4, and 9.
1a. ____________
1b. ____________
1c. ____________
1d. ____________
1e. ____________

Answer: 1a. Yes
1b. No
1c. Yes
1d. No
1e. Yes

Explanation:
1a. The highest point in the dot plot is called the peak. The peak in the given dot plot is at 5 and 11 the value of the peak is 3
1b. The group of dots form a cluster with 3 or more intervals.
1c. There is a gap between the intervals 6-8
1d. The maximum number of people did 11 chin-ups while only a single person did 15 chin-ups.
1e. The most frequently occurring observation is known as mode.
The mode of the given data is at the intervals 3,4 and 9.

Question 2.
The histogram shows the high temperatures in degrees Fahrenheit of various cities for one day in March.
Go Math Grade 6 Answer Key Chapter 13 Variability and Data Distributions img 64
Select the best word to complete each sentence.
The histogram has _____ peak(s).
The histogram _____ symmetry.

Answer: The histogram has 1 peak(s).
The histogram is symmetry.

Explanation:
The is one and only one peak at the interval 41 – 50
We can say that the graph is symmetrical because if we draw a line between the graph we can observe that the graph has two parts symmetric to each other.

Chapter 13 Review/Test – Page No. 758

Question 3.
The data set shows the scores of the players on the winning team of a basketball game.
Go Math Grade 6 Answer Key Chapter 13 Variability and Data Distributions img 65
The median is _____.
The lower quartile is _____.
The upper quartile is _____.

Answer:Median = 6
Lower quartile = 1
Upper quartile =19.5

Explanation:
Ascending order: 0,0,1,1,4,5,6,9,13,17,22,30,47
Median = 6
Lower quartile = Mean of 1 and 1 = 1+1/2 = 2/2 = 1
Upper quartile = Mean of 17 and 22 = 17+22/2 = 39/2 = 19.5

Question 4.
The data set shows the number of desks in 12 different classrooms.
Go Math Grade 6 Answer Key Chapter 13 Variability and Data Distributions img 66
Find the values of the points on the box plot.
Go Math Grade 6 Answer Key Chapter 13 Variability and Data Distributions img 67
Type below:
_________________

Answer: A= 16 B=17 C= 20 D= 21 E=24

Explanation:
Ascending order: 16,17,17,18,19,20,20,21,21,21,22,24
Median = 20+20/2 = 20
Lower quartile = 17
Upper quartile = 21

Question 5.
The box plot shows the number of boxes sold at an office supply store each day for a week.
Go Math Grade 6 Answer Key Chapter 13 Variability and Data Distributions img 68
For numbers 5a–5d, select True or False for each statement.
5a. The median is 18.
5b. The range is 15.
5c. The interquartile range is 9.
5d. The upper quartile is 18.
5a. ____________
5b. ____________
5c. ____________
5d. ____________

Answer: 5a.  false
5b. true
5c. true
5d. true

Explanation:
Median is the middlemost value of the given data.
Median of the data is 14
The range is the difference between the upper and lower observations.
Range = 21-6 = 15
The interquartile range is the difference between the upper and lower observations.
Upper quartile range: 18
Interquartile range = 18-9 = 9

Chapter 13 Review/Test – Page No. 759

Question 6.
The data set shows the number of glasses of water Dalia drinks each day for a week.
Go Math Grade 6 Answer Key Chapter 13 Variability and Data Distributions img 69
Part A
What is the mean number of glasses of water Dalia drinks each day?
_______ glasses

Answer: The mean number of glasses of water Dalia drinks each day is 8 glasses.

Explanation:
Mean = sum of all the observations/ total number of observations= 6+7+9+9+8+7+10/7 = 8

Question 6.
Part B
What is the mean absolute deviation of the number of glasses of water Dalia drinks each day? Round your answer to the nearest tenth. Use words and numbers to support your answer.
_______

Answer: Mean absolute deviation is 1.14

Explanation:

Mean:
Mean = sum of all the observations/ total number of observations= 6+7+9+9+8+7+10/7 = 8

Mean absolute deviation:

STEP 1 Label each observation with its distance from the mean.
Starting from left to right:
06: 8-6  = 02
07: 8-7  = 01
09: 8-9  = -1
09: 8-9  = -1
08: 8-8  =  0
07: 8-7  = 01
10: 8-10= -2

STEP 2 Find the mean of the distances.
2+1+1+1+0+1+2/7
= 8/7 = 1.14

So, the mean absolute deviation of the data is 1.14

Question 7.
The numbers of emails Megan received each hour are 9, 10, 9, 8, 7, and 2. The mean of the data values is 7.5 and the median is 8.5. Which measure of center better describes the data, the mean or median? Use words and numbers to support your answer.
Type below:
_________________

Answer: Mean is the best center of the tendency to represent the data given in the question

Explanation:
Ascending order of the data: 2,7,8,9,9,10
Mean = 7.5
Mean represents the observations 8,9,9,10 which come after 7.5
Therefore mean is the best way to represent the data.

Question 8.
The number of miles Madelyn drove between stops was 182, 180, 181, 184, 228, and 185. Which measure of center best describes the data?
Options:
a. mean
b. median
c. mode

Answer: b. Median

Explanation:
Ascending order: 180,181,182,184,185,228
Median = 182+184/2 = 183
183 represents all the observations after 182
So the median is the best way to represent the data.

Chapter 13 Review/Test – Page No. 760

Question 9.
The histogram shows the weekly earnings of part-time workers. What interval(s) represents the most common weekly earnings?
Go Math Grade 6 Answer Key Chapter 13 Variability and Data Distributions img 70
Type below:
_________________

Answer: 321-330 ; 341-350

Explanation:
The histogram has 2 intervals which show equal heights which means that the monthly earnings of these intervals is the same.

Question 10.
Jordan surveyed a group of randomly selected smartphone users and asked them how many applications they have downloaded onto their phones. The dot plot shows the results of Jordan’s survey. Select the statements that describe patterns in the data. Mark all that apply.
Go Math Grade 6 Answer Key Chapter 13 Variability and Data Distributions img 71
Options:
a. The modes are 37 and 42.
b. There is a gap from 38 to 40.
c. There is a cluster from 41 to 44.
d. There is a cluster from 35 to 36.

Answer: b. There is a gap from 38 to 40.

Explanation:
The dot plot represents a gap between 38-40. So we can say that there is a gap between the intervals 38 to 40.

Chapter 13 Review/Test – Page No. 761

Question 11.
Mrs. Gutierrez made a histogram of the birth month of the students in her class. Describe the patterns in the histogram by completing the chart.
Go Math Grade 6 Answer Key Chapter 13 Variability and Data Distributions img 72
Go Math Grade 6 Answer Key Chapter 13 Variability and Data Distributions img 73
Type below:
_________________

Answer: There are 2 peaks, Yes there is an increase across the intervals, Yes there is a decrease across the intervals

Explanation:
The highest point in the histogram is called is as a peak.
There is a peak near the month’s May and August.

There is an increase between the bars in the bar graph.
At the months February, March, November there is an increase in the graph.

There is a decrease between the bars in the bar graph.
At the months September, October, December.

Question 12.
Ian collected data on the number of children in 13 different families.
Go Math Grade 6 Answer Key Chapter 13 Variability and Data Distributions img 74
Draw a box plot of the data and use it to find the interquartile range and range.
Type below:
_________________

Answer: Range = 8-0 = 8 Interquartile range = 3-1 = 2

Explanation:

Ascending order:
0,0,1,1,1,1,2,2,2,3,3,4,8
Median = 2
Lower quartile = 1+1/2 = 1
Upper quartile = 3+3/2 = 3
Range = 8-0 = 8
Interquartile range = 3-1 = 2

Chapter 13 Review/Test – Page No. 762

Question 13.
Gavin wants to move to a county where it rains about 5 inches every month. The data set shows the monthly rainfall in inches for a county. The mean of the data is 5 and the median is 4.35. After analyzing the data, Gavin says that this county would be a good place to move. Do you agree or disagree with Gavin? Use words and numbers to support your answer.
Go Math Grade 6 Answer Key Chapter 13 Variability and Data Distributions img 75
Type below:
_________________

Answer: Yes I agree that it is a good place to move.

Explanation:
After analyzing the data we can say that this country would be a good place to move for Gavin.
Gavin wants to move to a place which has an average of 5 cm rainfall. So this country is the best choice because it has a mean equal to 5 and median equal to 4.35

Question 14.
The data set shows the number of books Peyton reads each month. Peyton says she usually reads 4 books per month. Why is Peyton’s statement misleading?
Go Math Grade 6 Answer Key Chapter 13 Variability and Data Distributions img 76
Type below:
_________________

Answer: No Peyton’s statement is not misleading because the mean of the data is 4.
Therefore Peyton says she usually reads 4 books per month.

Explanation:
Mean = 2+3+2+4+3+11+3/7 = 28/7 = 4

Question 15.
The data set shows the scores of three players for a board game.
Go Math Grade 6 Answer Key Chapter 13 Variability and Data Distributions img 77
For numbers 15a–15d, choose Yes or No to indicate whether the statement is correct.
15a. The mean absolute deviation of Player B’s scores is 0.
15b. The mean absolute deviation of Player A’s scores is 0.
15c. The mean absolute deviation of Player B’s scores is greater than the mean absolute deviation of Player C’s scores.
15a. ___________
15b. ___________
15c. ___________

Answer:15a. No
15b. Yes
15c.  Yes

Explanation:
Mean of player A = 90+90+90/3 = 90

Mean absolute deviation:

STEP 1 Label each observation with its distance from the mean.
Starting from left to right:
90: 90-90  = 0
90: 90-90  = 0
90: 90-90  = 0

STEP 2 Find the mean of the distances.
0+0+0/3 = 0
So, the mean absolute deviation of player A is 0

Mean of player B = 110+100+90/3 = 100

Mean absolute deviation:

STEP 1 Label each observation with its distance from the mean.
Starting from left to right:
100: 100-110  = -10
100: 100-100  = 0
100: 100-90    = 10

STEP 2 Find the mean of the distances.
10+0+10/3 = 20/3 = 6.67
So, the mean absolute deviation of player B is 6.67

Mean of player C = 95+100+95/3 = 96.67

Mean absolute deviation:

STEP 1 Label each observation with its distance from the mean.
Starting from left to right:
96.67: 96.67-95    = 1.67
96.67: 96.67-100  = -3.33
96.67: 96.67-95    = 1.67

STEP 2 Find the mean of the distances.
1.67+3.33+1.67/3 = 6.67/3 = 2.22
So, the mean absolute deviation of player C is 2.22

Conclusion:

The solutions with explanations to all the questions are prepared by the math experts as per the latest syllabus. So, go through the solutions and overcome your difficulties in this chapter. Keep in touch with this page to get the solutions to all the 6th-grade chapters.

Go Math Grade 7 Answer Key Chapter 11 Analyzing and Comparing Data

go-math-grade-7-chapter-11-analyzing-and-comparing-data-answer-key

Download Go Math Grade 7 Answer Key Chapter 11 Analyzing and Comparing Data pdf for free of cost. Go through the solutions for all the topics in HMH Go Math Grade 7 Answer Key Chapter 11 Analyzing and Comparing Data. This Go Math Answer Key Chapter 11 Analyzing and Comparing Data helps the students to score top in the exams. Check out the solutions in  Go Math Grade 7 Answer Key Chapter 11 Analyzing and Comparing Data and start preparing now.

Go Math Grade 7 Answer Key Chapter 11 Analyzing and Comparing Data

Students can find the step by step explanation for each and every problem in Go Math 7th Grade Answer Key Chapter 11 Analyzing and Comparing Data. The 7th grade Go math answer key not only helps the students it also helps the teachers to find an easy method to teach the students. Know the different methods to solve the problems in Grade 7 Go Math Answer Key Chapter 11 Analyzing and Comparing Data.

Chapter 11 – Lesson: 1

Chapter 11 – Lesson: 2

Chapter 11 – Lesson: 3

Chapter 11 – Comparing Data Displayed in Dot Plots

Guided Practice – Page No. 338

The dot plots show the number of miles run per week for two different classes. For 1–5, use the dot plots shown.
Go Math Grade 7 Answer Key Chapter 11 Analyzing and Comparing Data img 1

Question 1.
Compare the shapes of the dot plots.

Answer: In Class A the dot plot is clustered around two areas and in Class B the dot plot is clustered in the middle.

Question 2.
Compare the centers of the dot plots.

Answer: In Class A the data is centered around 4 miles and 13 miles and in Class B the data is centered around 7 miles.

Question 3.
Compare the spreads of the dot plots.

Answer: In class A the spread of the dot plot is 4 miles to 14 miles and in Class B the spread is 3 miles to 9 miles.

Question 4.
Calculate the medians of the dot plots.

Answer: The median or the dot plots for Class A and Class B is 6.

Explanation: For Class A median is 4,4,4,4,4,5,5,5,6,6,12,13,13,13,13,14,14
= 6.
For Class B median is 3,4,4,4,5,5,5,5,6,6,7,7,7,7,7,8,8,9
= (6+6)/2
= 12/2
= 6.

Question 5.
Calculate the ranges of the dot plots.

Answer: The range of the dot plot For Class A is 10 and Class B is 6.

Explanation: For Class A the range is 14-4= 10.
For Class B the range is 9-3= 6.

Essential Question Check-In

Question 6.
What do the medians and ranges of two dot plots tell you about the data?

Answer: The median of dot plots tells that the values of each dot plot are centered and we can get to know which dot plot has greater values. The range of the dot plot tells about the spread of each value in each plot. The smaller the range, the closer will be the values.

Independent Practice – Page No. 339

The dot plot shows the number of letters in the spellings of the 12 months. Use the dot plot for 7–10.
Go Math Grade 7 Answer Key Chapter 11 Analyzing and Comparing Data img 2

Question 7.
Describe the shape of the dot plot.

Answer: There is a slight increase in the number 8.

Question 8.
Describe the center of the dot plot.

Answer: The center of the dot plot is 6.

Question 9.
Describe the spread of the dot plot.

Answer: The spread of the dot plot is from 3 to 9

Question 10.
Calculate the mean, median, and range of the data in the dot plot.

Answer:
The mean of the dot plot is 6.17.
The median of the dot plot is 6.5.
The range of the dot plot is 6.

Explanation: 3,4,4,5,5,6,7,7,8,8,8,9
The mean of the dot plot is \(\frac{3+4+4+5+5+6+7+7+8+8+8+9}{12}
= \frac{74}{12}\)
= 6.17.
The medain of the dot plot is \(\frac{6+7}{2}
= \frac{13}{2}\)
= 6.5.
The range of the dot plot is 9-3= 6.

The dot plots show the mean number of days with rain per month for two cities.
Go Math Grade 7 Answer Key Chapter 11 Analyzing and Comparing Data img 3

Question 11.
Compare the shapes of the dot plots.

Answer: The most number of days with rain for Montgomery is greater than 8 days and in Lynchburg, the number of days of rain is 12 days or less.

Question 12.
Compare the centers of the dot plots.

Answer: In Montgomery, the center of the dot plot is around 9 days. And in Lynchburg, the center of the dot plot is around 10 days.

Question 13.
Compare the spreads of the dot plots.

Answer: In Montgomery, the spread of the dot plot is from 1 to 12 days and the outlier is 1. And in Lynchburg, the spread of the data plot is from 8 to 12 days.

Question 14.
What do the dot plots tell you about the two cities with respect to their average monthly rainfall?

Answer: As the center of Lynchburg is greater than the center of Montgomery, so average monthly rainfall for Lynchburg is greater than the average monthly rainfall of Montgomery.

Page No. 340

The dot plots show the shoe sizes of two different groups of people.
Go Math Grade 7 Answer Key Chapter 11 Analyzing and Comparing Data img 4

Question 15.
Compare the shapes of the dot plots.

Answer: In Group A the shoe sizes are mostly less than 9. And in group B all the shoe sizes are 11.5 or less.

Question 16.
Compare the medians of the dot plots.

Answer:
The median of Group A is 8.
The median of Group A is 9.5.

Explanation: 6.5,7,7,7.5,7.5,7.5,8,8,8,8,8,8.5,8.5,9,13
The median of Group A is 8.
8.5,9,9,9,9,9.5,9.5,9.5,9.5,10,10,10.5,10.5,10.5,11.5
The median of Group B is 9.5.

Question 17.
Compare the ranges of the dot plots (with and without the outliers).

Answer:
The range with the outlier is 13-6.5= 6.5.
The range without the outlier is 9-6.5= 2.5.
The range is 11.5-8.5= 3.

Explanation: The outlier in Group A is 13
The range with the outlier is 13-6.5= 6.5.
The range without the outlier is 9-6.5= 2.5.
There is no outlier in Group B, so the range is 11.5-8.5= 3.

Question 18.
Make A Conjecture
Provide a possible explanation for the results of the dot plots.

Answer: Group A is Girls and Group B is boys. Because boys have large feet than girls.

H.O.T.

Focus on Higher Order Thinking

Question 19.
Analyze Relationships
Can two dot plots have the same median and range but have completely different shapes? Justify your answer using examples.

Answer: Yes, it is possible to have the same median and range with different shapes.

Explanation: Yes, it is possible to have the same median and range with different shapes. The median and the range of the below image is
image 1 data – 1,2,2,3,3,3,4,4,5.
The median of image 1 is 3.
image 2 data is – 2,2,2,2,3,3,4,4,5,5,6.
The median of image 2 is 3.
The range of image 1 is 5-1= 4.
The range of image 2 is 6-2= 4.

Go Math Grade 7 Answer Key Chapter 11 Analyzing and Comparing Data

Question 20.
Draw Conclusions
What value is most affected by an outlier, the median or the range? Explain. Can you see these effects in a dot plot?

Answer: The most affected by an outlier is range. The outlier increases the range as median values are in the middle, so the outlier will not mostly affect the median. Yes, in a dot plot we can see both range and median.

Guided Practice – Page No. 344

For 1–3, use the box plot Terrence created for his math test scores. Find each value.
Go Math Grade 7 Answer Key Chapter 11 Analyzing and Comparing Data img 5

Question 1.
Minimum = _____ ; Maximum = _____

Answer:
Minimum = 72.
Maximum = 88.

Explanation: The minimum value is the smallest value in the box plot, so the minimum value is 72, and the maximum value is the largest value in the box plot which is 88

Question 2.
Median = _____

Answer:
The Median is 79.

Explanation:
The data is 72,75,79,85,88
The Median is 79.

Question 3.
Range = _____ ; IQR = _____

Answer:
The range is 16.
The IQR is 10.

Explanation:
The range is 88-72= 16
IQR is the difference between upper quartiles and lower quartiles, so 85-75= 10.

For 4–7, use the box plots showing the distribution of the heights of hockey and volleyball players.
Go Math Grade 7 Answer Key Chapter 11 Analyzing and Comparing Data img 6

Question 4.
Which group has a greater median height?
_____

Answer:
The greater median height is Volleyball players with 74 in.

Explanation:
Hockey players data is 64,66,70,76,78.
The median height of hockey players is 70 in.
Volleyball players data is 67,68,74,78,85
The median height of the Volleyball player is 74 in.

Question 5.
Which group has the shortest player?
_____

Answer:
Hockey players have the shortest player with 64 in.

Explanation:
The minimum height of the hockey players is 64 in.
The minimum height of the Volleyball players is 67 in.

Question 6.
Which group has an interquartile range of about 10?
_____

Answer: The IQR for Hockey players and Volleyball players is 10.

Explanation:
The IQR for Hockey players is 76-66= 10.
The IQR for Volleyball players is 78-68= 10.

Essential Question Check-In

Question 7.
What information can you use to compare two box plots?

Answer: To compare two box plots we can use minimum, maximum values, ////////the median, the range, and the IQR.

Independent Practice – Page No. 345

For 8–11, use the box plots of the distances traveled by two toy cars that were jumped from a ramp.
Go Math Grade 7 Answer Key Chapter 11 Analyzing and Comparing Data img 7

Question 8.
Compare the minimum, maximum, and median of the box plots.

Answer:
The data of Car A is 165,170,180,195,210.
The data of Car B is 160,175,185,200,205.
The minimum value of Car A is 165.
The minimum value of Car B is 165.
The maximum value of Car A is 210.
The maximum value of Car B is 205.
The median of Car A is 180.
The median of Car B is 185.

Explanation:
The data of Car A is 165,170,180,195,210.
The data of Car B is 160,175,185,200,205.
The minimum value of Car A is 165.
The minimum value of Car B is 165.
The maximum value of Car A is 210.
The maximum value of Car B is 205.
The median of Car A is 180.
The median of Car B is 185.

Question 9.
Compare the ranges and interquartile ranges of the data in box plots.

Answer:
The range of Car A is 45.
The range of Car B is 45.
The IQR of Car A is  25.
The IQR of Car B is  25.

Explanation:
The range of Car A is 210-165= 45.
The range of Car B is 205-160= 45.
The IQR of Car A is 195-170= 25.
The IQR of Car B is 200-175= 25.

Question 10.
What do the box plots tell you about the jump distances of two cars?

Answer: The box plot tells about the minimum and the maximum jump distance, the median jump distance, and the spread of the jump distance.

Question 11.
Critical Thinking
What do the whiskers tell you about the two data sets?

Answer: The whiskers tells about the spread of maximum and minimum values of the bottom and top 25% of data.

For 12–14, use the box plots to compare the costs of leasing cars in two different cities.
Go Math Grade 7 Answer Key Chapter 11 Analyzing and Comparing Data img 8

Question 12.
In which city could you spend the least amount of money to lease a car? The greatest?
______

Answer: The least and the greatest amount is spent by City B.

Explanation:
The data set of City A is $425,$450,$475,$550,$600.
The data set of City B is $400,$425,$450,$475,$625.
The minimum cost of City A is $425 and the maximum is $600.
The minimum cost of City B is $400 and the maximum is $625.
The least and the greatest amount is spent by City B.

Question 13.
Which city has a higher median price? How much higher is it?
______

Answer: The higher median price is City A with $475 and $50 higher.

Explanation:
The median of City A is $475 and the median of City B is $450.
So the difference is $475-$425= $50.

Question 14.
Make a Conjecture
In which city is it more likely to choose a car at random that leases for less than $450? Why?
______

Answer: 450 corresponds to the first quartile of City A, which means 25% of the cars cost less than $450. 450 corresponds to the median for City B  which means 50% of the cars cost less than $450. So City B is more likely to have a car chosen randomly that costs less than $450.

Page No. 346

Question 15.
Summarize
Look back at the box plots for 12–14 on the previous page. What do the box plots tell you about the costs of leasing cars in those two cities?

Answer: City A has a smaller range than City B, but it has greater IQR. And City B has 4 key values of City A which means leasing a car is cheaper in City B.

H.O.T.

Focus on Higher Order Thinking

Question 16.
Draw Conclusions
Two box plots have the same median and equally long whiskers. If one box plot has a longer box than the other box plot, what does this tell you about the difference between the data sets?

Answer: If two box plots have the same median and equally long whiskers and one box is longer than the other, that means the box plot with the larger box has a greater range and IQR.

Question 17.
Communicate Mathematical Ideas
What you can learn about a data set from a box plot? How is this information different from a dot plot?

Answer: We can learn about the minimum and the maximum values, the median, the range, the IQR, and the range of 25% of the data.
and a data plot contains all data values. which a box plot doesn’t have.

Question 18.
Analyze Relationships
In mathematics, central tendency is the tendency of data values to cluster around some central value. What does a measure of variability tell you about the central tendency of a set of data? Explain.

Answer: If the range and IQR are small, the values are clustering around some central values.

Guided Practice – Page No. 350

The tables show the numbers of miles run by the students in two classes. Use the tables in 1–2.
Go Math Grade 7 Answer Key Chapter 11 Analyzing and Comparing Data img 9

Question 1.
For each class, what is the mean? What is the mean absolute deviation?
Class 1 mean: __________
Class 2 mean: __________
Class 1 MAD: __________
Class 2 MAD: __________

Answer:
Class 1 mean: 6
Class 2 mean: 11
Class 1 MAD: 3.067
Class 2 MAD: 3.067

Explanation:
The mean of Class 1 is \(\frac{12+6+1+10+1+2+3+10+3+8+3+9+8+6+8}{6}
= \frac{90}{15}\)
= 6
The mean of Class 2 is \(\frac{11+14+11+13+6+7+8+6+8+13+8+15+13+17+15}{15}
= \frac{165}{15}\)
= 11
The mean absolute deviation of Class 1 is
|12-6| = 6
|6-6| = 0
|1-6| = 5
|10-6| = 4
|1-6| = 5
|2-6| = 4
|3-6| = 3
|10-6| = 4
|3-6| = 3
|8-6| = 2
|3-6| = 3
|9-6| = 3
|8-6| = 2
|6-6| = 0
|8-6| = 2
The mean absolute deviation of Class 1 is \(\frac{6+0+5+4+5+4+3+4+3+2+3+3+2+0+2}{15}
= \frac{46}{15}\)
= 3.067

The mean absolute deviation of Class 2 is
|11-11| = 0
|14-11| = 3
|11-11| = 0
|13-11| = 2
|6-11| = 5
|7-11| = 4
|8-11| = 3
|6-11| = 5
|8-11| = 3
|13-11| = 2
|8-11| = 3
|15-11| = 4
|13-11| = 2
|17-11| = 6
|15-11| = 4
The mean absolute deviation of Class 2 is \(\frac{0+3+0+2+5+4+3+5+3+2+3+4+2+6+4}{15}
= \frac{46}{15}\)
= 3.067

Question 2.
The difference of the means is about _____ times the mean absolute deviations.
_____

Answer: 3, 1.67.

Explanation: The difference of the mean is 11-6=5, and the difference of the means is about 3 times the mean absolute deviations, so
5/3= 1.67.

Question 3.
Mark took 10 random samples of 10 students from two schools. He asked how many minutes they spend per day going to and from school. The tables show the medians and the means of the samples. Compare the travel times using distributions of the medians and means.
Go Math Grade 7 Answer Key Chapter 11 Analyzing and Comparing Data img 10

Answer:

Essential Question Check-In

Question 4.
Why is it a good idea to use multiple random samples when making comparative inferences about two populations?

Answer: It’s important to use multiple random samples, so you can draw more interferences about the populations. The more samples we use the more convincing arguments you can make about the distributions.

Independent Practice – Page No. 351

Josie recorded the average monthly temperatures for two cities in the state where she lives. Use the data for 5–7.
Go Math Grade 7 Answer Key Chapter 11 Analyzing and Comparing Data img 11

Question 5.
For City 1, what is the mean of the average monthly temperatures? What is the mean absolute deviation of the average monthly temperatures?
Mean: __________
MAD: __________

Answer:
Mean: 50 °F.
MAD: 13 °F.

Explanation:
The mean of city 1 is \(\frac{23+38+39+48+55+56+71+86+57+53+43+31}{12}
= \frac{600}{12}\)
= 50 °F.
|23-50|= 27
|38-50|= 12
|39-50|= 11
|48-50|= 2
|55-50|= 5
|56-50|= 6
|71-50|= 21
|86-50|= 36
|57-50|= 7
|53-50|= 3
|43-50|= 7
|31-50|= 19
The mean absolute deviation is \(\frac{27+12+11+2+5+6+21+36+7+3+7+19}{12}
= \frac{156}{12}\)
= 13 °F.

Question 6.
What is the difference between each average monthly temperature for City 1 and the corresponding temperature for City 2?
_______ °F

Answer: The difference between each average monthly temperature for City 1 and the corresponding temperature for City 2 is 15 °F

Explanation:
|23-8|= 15
|38-23|= 15
|39-24|= 15
|48-33|= 15
|55-40|= 15
|56-41|= 15
|71-56|= 15
|86-71|= 15
|57-42|= 15
|53-38|= 15
|43-28|= 15
|31-16|=  15
The difference between each average monthly temperature for City 1 and the corresponding temperature for City 2 is 15 °F

Question 7.
Draw Conclusions
Based on your answers to Exercises 5 and 6, what do you think the mean of the average monthly temperatures for City 2 is? What do you think the mean absolute deviation of the average monthly temperatures for City 2 is? Give your answers without actually calculating the mean and the mean absolute deviation. Explain your reasoning.
Mean = __________ °F
MAD __________ °F

Answer:
Mean =35 °F
MAD = 13°F

Explanation: As all the values of City 2 are 15 below the values of City 1, so the mean of the City 2 will be 50 less than the mean of City 1. Which means 50-15= 35. All of City 2’s values deviate from the mean the same way City 1’s values do which means that the mean absolute deviation is 13

Question 8.
What is the difference in the means as a multiple of the mean absolute deviations?
_______ (MAD)

Answer: 1.15.

Explanation:
(50-35)/13
= 15/13
= 1.15.
The difference in the means as a multiple of the mean absolute deviations 1.15.

Question 9.
Make a Conjecture
The box plots show the distributions of mean weights of 10 samples of 10 football players from each of two leagues, A and B. What can you say about any comparison of the weights of the two populations? Explain.
Go Math Grade 7 Answer Key Chapter 11 Analyzing and Comparing Data img 12

Answer: As both leagues have a lot of variability since the ranges and IQR’s are both very large. The middle halves overlap entirely. The variation and overlap in the distributions make it hard to make any convincing comparison.

Page No. 352

Question 10.
Justify Reasoning
Statistical measures are shown for the ages of middle school and high school teachers in two states.
State A: Mean age of middle school teachers = 38, mean age of high school teachers = 48, mean absolute deviation for both = 6
State B: Mean age of middle school teachers = 42, mean age of high school teachers = 50, mean absolute deviation for both = 4
In which state is the difference in ages between members of the two groups more significant? Support your answer.
_____________

Answer: State B has a difference in ages between members of the two groups more significant.

Explanation:
For State A the difference in the mean as a multiple of the mean absolute deviation is (48-38)/6
= 10/6
= 1.67.
So for State B, (50-42)/4
= 8/4
= 2.
As State B has a larger multiple, the differences in ages between members of the two groups are more significant.

Question 11.
Analyze Relationships
The tables show the heights in inches of all the adult grandchildren of two sets of grandparents, the Smiths and the Thompsons. What is the difference in the medians as a multiple of the ranges?
Go Math Grade 7 Answer Key Chapter 11 Analyzing and Comparing Data img 13
______ x range

Answer: The difference in the median is 1.75.

Explanation:
Smith: 64,65,65,66,66,67,68,68,69,70.
The Median is (66+67)/2
= 133/2
= 66.5.
The range is 70-64= 6.
Thompsons: 74,75,75,76,77,77,78,79,79,80.
The Median is (77+77)/2
= (154)/2
= 77.
The range is 80-74= 6.
The difference in the median is (77-66.5)/6
= 10.5/6
= 1.75.

H.O.T.

Focus on Higher Order Thinking

Question 12.
Critical Thinking
Jill took many samples of 10 tosses of a standard number cube. What might she reasonably expect the median of the medians of the samples to be? Why?
Median of the medians: ______

Answer:
Median of the medians: 3.5.

Explanation: The possible outcome of a number cube is 1,2,3,4,5,6. So median is
= (3+4)/2
= 7/2
= 3.5
The median of the medians should be close to the median of the populations, so it will also be about 3.5.

Question 13.
Analyze Relationships
Elly and Ramon are both conducting surveys to compare the average numbers of hours per month that men and women spend shopping. Elly plans to take many samples of size 10 from both populations and compare the distributions of both the medians and the means. Ramon will do the same, but will use a sample size of 100. Whose results will probably produce more reliable inferences? Explain.
_____________

Answer: The larger the sample size, the less variability there should be in the distributions of the medians and means. And Ramon will most likely produce more reliable inferences since he will be using a much larger sample size.

Question 14.
Counterexamples
Seth believes that it is always possible to compare two populations of numerical values by finding the difference in the means of the populations as a multiple of the mean absolute deviations. Describe a situation that explains why Seth is incorrect.

Answer: In order to compare two populations by finding the difference in the means of the populations as a multiple of the mean absolute deviations, so the mean absolute deviations of both populations need to be about the same. So if the mean absolute deviations are significantly different, like 5 and 10 and we cannot compare the populations this way.

11.1 Comparing Data Displayed in Dot Plots – Page No. 353

The two dot plots show the number of miles run by 14 students at the start and at the end of the school year. Compare each measure for the two dot plots. Use the data for 1–3.
Go Math Grade 7 Answer Key Chapter 11 Analyzing and Comparing Data img 14

Question 1.
Means
Start: _________
End: _________

Answer:
Mean
Start: 7.5 miles.
End: 8.2 miles.

Explanation:
The data for the start of the school year is 5,6,6,7,7,7,7,8,8,8,8,9,9,10.
The mean is \(\frac{5+6+6+7+7+7+7+8+8+8+8+9+9+10}{14}
= \frac{105}{14}\)
= 7.5 miles.
The data for the end of the school year is 6,6,7,7,8,8,8,8,9,9,9,10,10,10.
The mean is \(\frac{6+6+7+7+8+8+8+8+9+9+9+10+10+10}{14}
= \frac{115}{14}\)
= 8.2 miles.

Question 2.
Medians
Start: _________
End: _________

Answer:
Median
Start: 7.5 miles.
End: 8 miles.

Explanation:
The median for the start of the school year is
= (7+8)/2
= 15/2
= 7.5 miles.
The median for the end of the school year is
= (8+8)/2
= 16/2
= 8 miles.

Question 3.
Ranges
Start: _________
End: _________

Answer:
Ranges
Start: 5 miles.
End: 4 miles.

Explanation:
The range for the Start of the school year is 10-5= 5 miles.
The range for the end of the school year is 10-6= 4 miles.

11.2 Comparing Data Displayed in Box Plots

The box plots show lengths of flights in inches flown by two model airplanes. Use the data for 4–5.
Go Math Grade 7 Answer Key Chapter 11 Analyzing and Comparing Data img 15

Question 4.
Which has a greater median flight length?
_____________

Answer:
The greater median flight length is Airplane A which is 210 in.

Explanation:
The median of Airplane A is 210 in and the median of Airplane B is 204 in. So greater median flight length is Airplane A which is 210 in.

Question 5.
Which has a greater interquartile range?
_____________

Answer: The greater IQR is Airplane B with 35 in.

Explanation:
The IQR for Airplane A is 225-208= 17 in and The IQR for Airplane B is 230-195= 35 in. So the greater IQR is Airplane B.

11.3 Using Statistical Measures to Compare Populations

Question 6.
Roberta grows pea plants, some in shade and some in sun. She picks 8 plants of each type at random and records the heights.
Go Math Grade 7 Answer Key Chapter 11 Analyzing and Comparing Data img 16
Express the difference in the means as a multiple of their ranges.
______

Answer: The difference in the means as a multiple of their ranges is 2.4 in.

Explanation:
The mean of Shade plant heights is \(\frac{7+11+11+12+9+12+8+10}{8}
= \frac{80}{8}\)
= 10 in.
The range of Shade plant heights is 12-7= 5 in.
The mean of Sun plant heights is \(\frac{21+24+19+19+22+23+24+24}{8}
= \frac{176}{8}\)
= 22 in.
The range of Sun plant heights is 24-19= 5 in.
The difference in the means as a multiple of their ranges is  (22-10)/5
= 12/5
= 2.4 in.

Essential Question

Question 7.
How can you use and compare data to solve real-world problems?

Answer: We can use and compare data to solve real-world problems by determining if one set is larger than the other set in terms of values, means, and medians.

Selected Response – Page No. 354

Question 1.
Which statement about the data is true?
Go Math Grade 7 Answer Key Chapter 11 Analyzing and Comparing Data img 17
Options:
a. The difference between the medians is about 4 times the range.
b. The difference between the medians is about 4 times the IQR.
c. The difference between the medians is about 2 times the range.
d. The difference between the medians is about 2 times the IQR.

Answer: b is true.

Explanation:
Set 1 median is 60 and Set 2 median is 76
The range of Set 1 is 68-55= 13
The range of Set 2 is 80-65= 15
The IQR of Set 1 is 63-59= 4
The IQR of Set 2 is 77-73= 4
The difference in medians is 76-60= 16, So the difference between the medians is about 4 times the IQR.

Question 2.
Which is a true statement based on the box plots below?
Go Math Grade 7 Answer Key Chapter 11 Analyzing and Comparing Data img 18
Options:
a. The data for City A has a greater range.
b. The data for City B is more symmetric.
c. The data for City A has a greater interquartile range.
d. The data for City B has a greater median.

Answer: C is true.

Explanation: The length of the box for City A is much larger than for City B, so IQR for City A is greater.

Question 3.
What is −3 \(\frac{1}{2}\) written as a decimal?
Options:
a. -3.5
b. -3.05
c. -0.35
d. -0.035

Answer: a.

Explanation: −3 \(\frac{1}{2}\)
=  \(\frac{-7}{2}\)
= -3.5.

Question 4.
Which is a true statement based on the dot plots below?
Go Math Grade 7 Answer Key Chapter 11 Analyzing and Comparing Data img 19
Options:
a. Set A has the lesser range
b. Set B has a greater median.
c. Set A has the greater mean.
d. Set B is less symmetric than Set A.

Answer: c is a true statement.

Explanation:
The median of Set A is 30 and the median of Set B is 40, so Set A has the greater mean.

Mini-Task

Question 5.
The dot plots show the lengths of a random sample of words in a fourth-grade book and a seventh-grade book.
Go Math Grade 7 Answer Key Chapter 11 Analyzing and Comparing Data img 20
a. Compare the shapes of the plots.

Answer:
For Fourth grade, most of the words have a length of 6 or less and with two outliers 9 and 10.
For Seventh grade, most of the words have a length of 8 or less with 5 exceptions.

Question 5.
b. Compare the ranges of the plots. Explain what your answer means in terms of the situation.

Answer:
The Seventh grade has a larger range, so it has more variability.

Explanation:
The range for the fourth grade is 10-1=9.
The range for the seventh grade 14-2= 12.
As the Seventh grade has a larger range it has more variability.

EXERCISES – Page No. 356

Question 1.
Molly uses the school directory to select, at random, 25 students from her school for a survey on which sports people like to watch on television. She calls the students and asks them, “Do you think basketball is the best sport to watch on television?”
a. Did Molly survey a random sample or a biased sample of the students at her school?
_____________

Answer: Yes, Molly surveyed a random sample. As she selected 25 students from a school directory of the entire student’s population in her school.

Question 1.
b. Was the question she asked an unbiased question? Explain your answer.
_____________

Answer: No, the question is not unbiased. The question is biased because it assumes the person watches basketball on television.

Question 2.
There are 2,300 licensed dogs in Clarkson. A random sample of 50 of the dogs in Clarkson shows that 8 have ID microchips implanted. How many dogs in Clarkson are likely to have ID microchips implanted?
______ dogs

Answer: 368 dogs.

Explanation: Let the dogs in Clarkson to have ID microchips be X, so
X/2300 = 8/50
X= (8×2300)/50
X= 18,400/50
X= 368.

Question 3.
A store gets a shipment of 500 MP3 players. Twenty-five of the players are defective, and the rest are working. A graphing calculator is used to generate 20 random numbers to simulate a random sample of the players.
A list of 20 randomly generated numbers representing MP3 players is :
Go Math Grade 7 Answer Key Chapter 11 Analyzing and Comparing Data img 21
a. Let numbers 1 to 25 represent players that are _____
_____________

Answer: As there are twenty-five defective players, let the numbers 1 to 25 represent players that are defective.

Question 3.
b. Let numbers 26 to 500 represent players that are _____
_____________

Answer: Let the numbers 26 to 500 represent players that are working.

Question 3.
c. How many players in this sample are expected to be defective?
______ players

Answer: As there are 2 numbers in from 1 and 25 which are 5 and 9 are the players in the sample are expected to be defective.

Question 3.
d. If 300 players are chosen at random from the shipment, how many are expected to be defective based on the sample? Does the sample provide a reasonable inference? Explain.
______ players

Answer: 30 players.

Explanation:
X/300 = 2/20
X = (2×300)/20
X = 600/20
X = 30.
We may expect 25 out of 500 or 5% of the 300 players to be defective, which is only 15 players because the sample doesn’t provide a reasonable inference.

EXERCISES – Page No. 357

The dot plots show the number of hours a group of students spends online each week, and how many hours they spend reading. Compare the dot plots visually.
Go Math Grade 7 Answer Key Chapter 11 Analyzing and Comparing Data

Question 1.
Compare the shapes, centers, and spreads of the dot plots.

Answer:
Shape:
Time spent online- Most of the students spend 4 hours are more.
Time spent reading- The students spent a maximum of 6 hours.
Centers:,6
The no.of hours spent online is centered around 6 hours.
The no.of hours spent reading is centered around 5 hours.
Spread:
The range for time spent online is 7-0=7.
The range for time spent reading is 6-0=6.

Question 2.
Calculate the medians of the dot plots.
Time online: __________
Time reading: __________

Answer:
Time online: 6 hours.
Time reading: 5 hours.

Explanation:
The data of time online is 0,4,4,5,5,6,6,6,6,6,6,7,7,7,7
The Median is 6 hours.
The data of time reading is 0,0,0,0,1,1,2,5,5,5,6,6,6,6,6
The Median is 5 hours.

Question 3.
Calculate the ranges of the dot plots.
Time online: __________
Time reading: __________

Answer:
Time online: 7 hours.
Time reading: 6 hours.

Explanation:
The range of time online is 7-0= 7.
The range of time reading is 6-0= 6.

Page No. 358

Question 4.
The average times (in minutes) a group of students spend studying and watching TV per school day are given.
Studying: 25, 30, 35, 45, 60, 60, 70, 75
Watching TV: 0, 35, 35, 45, 50, 50, 70, 75
a. Find the mean times for studying and for watching TV.
Studying: __________
Watching TV: __________

Answer:
Studying: 50.
Watching TV: 40.

Explanation:
The mean for studying is \(\frac{25+30+35+45+60+60+0+75}{8}
= \frac{400}{8}\)
= 50.
The mean for watching TV is \(\frac{0+35+35+45+50+50+70+75}{8}
= \frac{360}{8}\)
= 45.

Question 4.
b. Find the mean absolute deviations (MADs) for each data set.
Studying: __________
Watching TV: __________

Answer:
Studying: 16.25
Watching TV: 16.25

Explanation:
|25-50|= 25
|30-50|= 20
|35-50|= 15
|45-50|= 5
|60-50|= 10
|60-50|= 10
|70-50|= 20
|75-50|= 25
The mean absolute deviation is \(\frac{25+20+15+5+10+10+20+25}{8}
= \frac{130}{8}\)
= 16.25.
|0-45|= 45
|35-45|= 10
|35-45|= 10
|45-45|= 0
|50-45|= 5
|50-45|= 5
|70-45|= 25
|75-45|= 30
The mean absolute deviation is \(\frac{45+10+10+0+5+5+25+30}{8}
= \frac{130}{8}\)
= 16.25.

Question 4.
c. Find the difference of the means as a multiple of the MAD, to two decimal places.
_____

Answer: 0.31.

Explanation: (50-45)/16.25 = 5/16.25
= 0.31.

Unit 5 Performance Tasks

Question 5.
Entomologist
An entomologist is studying how two different types of flowers appeal to butterflies. The box-and-whisker plots show the number of butterflies that visited one of two different types of flowers in a field. The data were collected over a two-week period, for one hour each day.
Go Math Grade 7 Answer Key Chapter 11 Analyzing and Comparing Data img 23
a. Find the median, range, and interquartile range for each data set.

Answer:
Type A:
The median is 11.5
The range is 4
The IQR is 3
Type B:
The median is 11
The range is 10
The IQR is 2

Explanation:
Type A:
The median is (11+12)/2
= 23/2
= 11.5
The range is 13-9= 4
The IQR is 12-9= 3
Type B:
The median is 11
The range is 17-7= 10
The IQR is 12-10= 2

Question 5.
b. Which measure makes it appear that flower type A had a more consistent number of butterfly visits? Which measure makes it appear that flower type B did? If you had to choose one flower as having the more consistent visits, which would you choose? Explain your reasoning.

Answer: As type A has a smaller range, the range makes it appear as if type A has a more consistent number of butterflies visits. And type B had a smaller IQR, the IQR makes it appear as if type A has a more consistent number of butterflies visits. We would choose type A has to have a more consistent number of butterflies visits and it has a much smaller range. The range of the fourth quartile for type Bis larger than the range for the entire data set of type A.

Selected Response – Page No. 359

Question 1.
Which is a true statement based on the dot plots below?
Go Math Grade 7 Answer Key Chapter 11 Analyzing and Comparing Data img 24
Options:
a. Set B has a greater range.
b. Set B has a greater median.
c. Set B has the greater mean.
d. Set A is less symmetric than Set B.

Answer: a

Explanation:
Set A has a range of 60-20= 40
Set B has a range of 60-10= 50.
So Set B has a greater range.

Question 2.
Which is a solution to the equation 7g − 2 = 47?
Options:
a. g = 5
b. g = 6
c. g = 7
d. g = 8

Answer: c

Explanation:
7g-2= 47
7g= 47+2
7g= 49
g= 49/7
g= 7.

Question 3.
Which is a true statement based on the box plots below?
Go Math Grade 7 Answer Key Chapter 11 Analyzing and Comparing Data img 25
Options:
a. The data for Team B has a greater range.
b. The data for Team A is more symmetric.
c. The data for Team B has a greater interquartile range.
d. The data for Team A has a greater median.

Answer: c

Explanation: The box of Team B is much larger than the box of Team A, so the data for Team B have the greater interquartile range.

Question 4.
Which is the best way to choose a random sample of people from a sold-out movie audience for a survey?
Options:
a. Survey all audience members who visit the restroom during the movie.
b. Assign each seat a number, write each number on a slip of paper, and then draw several slips from a hat. Survey the people in those seats.
c. Survey all of the audience members who sit in the first or last row of seats in the movie theater.
d. Before the movie begins, ask for volunteers to participate in a survey. Survey the first twenty people who volunteer.

Answer: b

Explanation:
A is not random because the people are being chosen are being surveyed in one place.
B is random as all members of the population can be chosen and each member has an equal chance of being selected.
C is may not assign every member of the population an equal chance of being chosen since the number of seats in the first or last rows may have more or fewer seats than the other rows.
D is not random because participants are self selecting to do the survey.

Question 5.
Find the percent change from 84 to 63.
Options:
a. 30% decrease
b. 30% increase
c. 25% decrease
d. 25% increase

Answer: c

Explanation:
(84-63)/84 = 21/84
= 0.25
= 25% decrease

Question 6.
A survey asked 100 students in a school to name the temperature at which they feel most comfortable. The box plot below shows the results for temperatures in degrees Fahrenheit. Which could you infer based on the box plot below?
Go Math Grade 7 Answer Key Chapter 11 Analyzing and Comparing Data img 26
Options:
a. Most students prefer a temperature less than 65 degrees.
b. Most students prefer a temperature of at least 70 degrees.
c. Almost no students prefer a temperature of fewer than 75 degrees.
d. Almost no students prefer a temperature of more than 65 degrees.

Answer: b.

Explanation: The last half of the data is about 73-85 which means 50% prefer a temperature above 73. This means that the most prefer a temperature of at least 70 degrees since more than 50% of the box plot is 70 degrees are more.

Page No. 360

Question 7.
The box plots below show data from a survey of students under 14 years old. They were asked on how many days in a month they read and draw. Based on the box plots, which is a true statement about students?
Go Math Grade 7 Answer Key Chapter 11 Analyzing and Comparing Data img 27
Options:
a. Most students draw at least 12 days a month.
b. Most students read less than 12 days a month.
c. Most students read more often than they draw.
d. Most students draw more often than they read.

Answer: c

Explanation: As 4 out of 5 key values for reading are greater than the corresponding values for drawing which means most of the students read more often than they draw.

Question 8.
Which describes the relationship between ∠NOM and ∠JOK in the diagram?
Go Math Grade 7 Answer Key Chapter 11 Analyzing and Comparing Data img 28
Options:
a. adjacent angles
b. complementary angles
c. supplementary angles
d. vertical angles

Answer: d

Explanation: ∠NOM and ∠JOK are vertical angles.

Question 9.
The tables show the typical number of minutes spent exercising each week for a group of fourth-grade students and a group of seventh-grade students.
Go Math Grade 7 Answer Key Chapter 11 Analyzing and Comparing Data img 29
a. What is the mean number of minutes spent exercising for fourth graders? For seventh graders?
4th grade: __________
7th grade: __________

Answer:
4th grade: 129
7th grade: 221

Explanation:
The mean for fourth grade is \(\frac{120+75+30+30+240+90+100+180+125+300}{10}
= \frac{1290}{10}\)
= 129
The mean for fourth grade is \(\frac{410+145+240+250+125+95+210+190+245+300}{10}
= \frac{2210}{10}\)
= 221

Question 9.
b. What is the mean absolute deviation of each data set?
4th grade: __________
7th grade: __________

Answer:
4th grade: 66.6
7th grade: 68

Explanation:
|120-129|= 9
|75-129|= 54
|30-129|= 99
|30-129|= 99
|240-129|=111
|90-129|= 39
|100-129|= 29
|180-129|= 51
|125-129|= 4
|300-129|= 171
The mean absolute deviation for fourth grade is \(\frac{9+54+99+99+111+39+29+51+4+171}{10}
= \frac{666}{10}\)
= 66.6
|410-221|= 189
|145-221|= 76
|240-221|= 19
|250-221|= 29
|125-221|= 96
|95-221|= 126
|210-221|= 11
|190-221|= 31
|245-221|= 24
|300-221|= 79
The mean absolute deviation for fourth grade is \(\frac{189+76+19+29+96+126+11+31+24+79}{10}
= \frac{680}{10}\)
= 68

Question 9.
c. Compare the two data sets with respect to their measures of center and their measures of variability.

Answer: The center of the fourth grade is much smaller than the center for 7th grade. The range is much smaller for a fourth grade than 7th grade which means that fourth graders spend less time exercising and have less variability in the number of minutes that they exercise.

Explanation:
The data of fourth grade is 30,30,75,90,100,120,125,180,240,300
Median is (100+120)/2
= 220/2
= 110
The range is 300-30= 270
The data of seventh grade is 95,125,145,190,210,240,245,250,300,410
Median is (210+240)/2
= 450/2
= 225
The range is 410-95= 315.
The center of the fourth grade is much smaller than the center for 7th grade. The range is much smaller for a fourth grade than 7th grade which means that fourth graders spend less time exercising and have less variability in the number of minutes that they exercise.

Question 9.
d. How many times the MADs is the difference between the means, to the nearest tenth?
_______

Answer: As the MADs are not the same we will find the average of them and then find the difference of the mean and divide by the average of the MADs.

Explanation:
(66.6+68)/2
= 134.6/2
= 67.3
(221-129)/67.3
= 92/67.3
= 1.37

Guided Practice – Page No. 371

Question 1.
In a hat, you have index cards with the numbers 1 through 10 written on them. Order the events from least likely to happen (1) to most likely to happen (8) when you pick one card at random. In the boxes, write a number from 1 to 8 to order the eight different events.
You pick a number greater than 0. __________
You pick an even number. __________
You pick a number that is at least 2. __________
You pick a number that is at most 0. __________
You pick a number divisible by 3. __________
You pick a number divisible by 5. __________
You pick a prime number. __________
You pick a number less than the greatest prime number. __________

Answer: 8,5,7,1,3,24,6.

Explanation:
As there are 10 numbers from 1 to 10 and thus there will be 10 possible outcomes. So,
The number greater than 0 is 1,2,3,4,5,6,7,8,9,10.
Even numbers are 2,4,6,8,10.
The number at least 2 is 2,3,4,5,6,7,8,9,10.
The number that is at most 0: as none of the integers are from 1 to 10 are at most 0.
The number divisible by 3 is 3,6,9.
The number divisible by 5 is 5,10.
The prime numbers are 2,3,5,7.
The number less than the greatest prime numbers are 1,2,3,4,5,6 as 7 is the greatest prime number from the numbers 1 to 10.
The more favorable outcomes correspond with an event, the more likely the events happen. Thus the number is at most 0 is the least likely and the greater than 0 is the most likely.
The number of events from the least likely to the most likely is
The number greater than 0 is 8
Even numbers are 5
The number at least 2 is 7
The number that is at most 0: 1
The number divisible by 3 is 3
The number divisible by 5 is 2
The prime numbers are 4
The number less than the greatest prime number is 6.

Final Words:

In addition to the exercise problems, students can also find solutions for homework exercises. Thus the Go Math Grade 7 Answer Key Chapter 11 helps to complete the homework in time. We wish the pdf helped you a lot in scoring marks in the exams. Keep in touch with us to get the latest information regarding all chapters in grade 7.

Go Math Grade 4 Answer Key Homework Practice FL Chapter 1 Place Value, Addition, and Subtraction to One Million

go-math-grade-4-chapter-1-place-value-addition-and-subtraction-to-one-million-pages-1-20-answer-key

Go Math Grade 4 Answer Key Homework Practice FL Chapter 1 Place Value, Addition, and Subtraction to One Million Pdf Download is available on this page for an efficient practice session. Provided solutions are very easy to understand and simple to learn the concept of Chapter 1 Place Value, Addition, and Subtraction to One Million. Enhance your kid’s logical and analytical thinking by answering every practice question covered in the HMH Go Math Grade 4 Answer Key Homework Practice FL Chapter 1 Place Value, Addition, and Subtraction to One Million.

Go Math Grade 4 Answer Key Homework Practice FL Chapter 1 Place Value, Addition, and Subtraction to One Million

Here, we have designed a new path for students to solve each and every chapter question & improve their problem-solving skills. It is nothing but the HMH Go Math Grade 4 Answer Key of Chapter 1. The topics of ch 1 Place Value, Addition, and Subtraction to One Million are Model Place Value Relationships, Compare and Order Numbers, Round Numbers, Add and subtract Whole Numbers, etc. Now, you’ll find all topics Go Math Grade 4 Answer Key Homework Practice FL Chapter 1 for better preparation.

Lesson: 1 – Model Place Value Relationships

Lesson: 2 – Read and Write Numbers

Lesson: 3 – Compare and Order Numbers

Lesson: 4 – Round Numbers 

Lesson: 5 – Rename Numbers

Lesson: 6 – Add Whole Numbers

Lesson: 7 – Subtract Whole Numbers

Lesson: 8 – Problem Solving Comparison Problems with Addition and Subtraction

Lesson: 9 

Common Core – Place Value, Addition, and Subtraction to One Million – Page No. 3

Model Place Value Relationships

Find the value of the underlined digit.

Question 1.
6,035
30

Question 2.
43,782
________

Answer: 700

Explanation:
Every digit in a number has a place value and the place value can be defined as the value represented by a digit in a number on the basis of its position in the number. So the place value of the digit 7 in 43,782 is 700.

Question 3.
506,087
________

Answer: 7

Explanation:
Every digit in a number has a place value and the place value can be defined as the value represented by a digit in a number on the basis of its position in the number. So the place value of the digit 7 in 506,087 is 7.

Question 4.
49,254
________

Answer: 9000

Explanation:
Every digit in a number has a place value and the place value can be defined as the value represented by a digit in a number on the basis of its position in the number. So the place value of the digit 9 in 49,254 is 9000.

Question 5.
136,422
________

Answer: 30,000

Explanation:
Every digit in a number has a place value and the place value can be defined as the value represented by a digit in a number on the basis of its position in the number. So the place value of the digit 3 in 136,422 is 30,000.

Question 6.
673,512
________

Answer: 500

Explanation:
Every digit in a number has a place value and the place value can be defined as the value represented by a digit in a number on the basis of its position in the number. So the place value of the digit 5 in 673,512 is 500.

Question 7.
814,295
________

Answer: 800,000

Explanation:
Every digit in a number has a place value and the place value can be defined as the value represented by a digit in a number on the basis of its position in the number. So the place value of the digit 8 in 814,295 is 800,000.

Question 8.
736,144
________

Answer: 6,000

Explanation:
Every digit in a number has a place value and the place value can be defined as the value represented by a digit in a number on the basis of its position in the number. So the place value of the digit 6 in 736,144 is 6,000.

Compare the values of the underlined digits.

Question 9.
6,300 and 530
The value of 3 in _____ is _____ times the value of 3 in _____.

Answer: The value of 3 in 6,300 is 10 times the value of 3 in 530.

Explanation:
Every digit in a number has a place value and the place value can be defined as the value represented by a digit in a number on the basis of its position in the number. So the place value of the digit 3 in 6,300 is 300. And the place value of the digit 3 in 530 is 30. As each hundred is 10 times as many as 10, so 3 hundreds are ten times as many as 3 tens. So, the value of 3 in 6,300 is 10 times the value of 3 in 530.

Question 10.
2,783 and 7,283
The value of 2 in _____ is _____ times the value of 2 in _____.

Answer: The value of 2 in 2783 is 10 times the value of 2 in 7283.

Explanation:
Every digit in a number has a place value and the place value can be defined as the value represented by a digit in a number on the basis of its position in the number. So the place value of the digit 2 in 2,783 is 2000. And the place value of the digit 2 in 7,283 is 200. As each hundred is 10 times as many as 10, so 2 thousands are ten times as many as 2 hundred. So, the value of 2 in 2783 is 10 times the value of 2 in 7283.

Question 11.
34,258 and 47,163
The value of 4 in _____ is _____ times the value of 4 in _____.

Answer: The value of 4 in 47,163 is 10 times the value of 4 in 34,258.

Explanation:
Every digit in a number has a place value and the place value can be defined as the value represented by a digit in a number on the basis of its position in the number. So the place value of the digit 4 in 34,258 is 4,000. And the place value of the digit 4 in 47,163 is 40,000. As each hundred is 10 times as many as 10, so 4 thousands are ten times as many as 4 thousand tens. So, the value of 4 in 47,163 is 10 times the value of 4 in 34,258.

Question 12.
503,497 and 26,475
The value of 7 in _____ is _____ times the value of 7 in _____.

Answer: The value of 7 in 26,475 is 10 times the value of 7 in 5,03,497.

Explanation:
Every digit in a number has a place value and the place value can be defined as the value represented by a digit in a number on the basis of its position in the number. So the place value of the digit 7 in 503,497 is 7. And the place value of the digit 7 in 26,475 is 70. As each hundred is 10 times as many as 10, so 7 are ten times as many as 7 tens. So, the value of 7 in 26,475 is 10 times the value of 7 in 5,03,497.

Problem Solving

Use the table for 13–14.
Go Math Grade 4 Answer Key Homework Practice FL Chapter 1 Place Value, Addition, and Subtraction to One Million Common Core - Place Value, Addition, and Subtraction to One Million img 1

Question 13.
What is the value of the digit 9 in the attendance at the Redskins vs. Titans game?
The value of 9 is _____

Answer: 9,000

Explanation:
Every digit in a number has a place value and the place value can be defined as the value represented by a digit in a number on the basis of its position in the number. So the place value of the digit 9 in 69,143 is 9,000.

Question 14.
The attendance at which game has a 7 in the ten thousands place?
__________

Answer: Ravens vs. Panthers

Explanation:
Each digit of the number holds its own value. The adjacent digits of the number differ from each other by 10 times. Starting from the leftmost digit going to the right, the order of place values starts from ones, tens, hundreds, thousands, and ten thousand. Therefore, the number should contain a digit 7 on the 5th digit from left to right.
Thus the attendance at Ravens vs. Panthers game has a 7 in the ten thousands place.

Question 15.
How does a digit in the ten thousands place compare to a digit in the thousands place?
Type below:
__________

Answer:
A digit in the ten thousand place has a value of 10,000 times the value of the mere digit. While a digit in the thousands place has a value 1,000 times the value of the digit. So to compare you can do 10,000 / 1,000 = 10, which means that a digit in the ten thousand place values ten times what the same digit values are it is the thousand place.

Common Core – Place Value, Addition, and Subtraction to One Million – Page No. 4

Lesson Check

Question 1.
During one season, a total of 453,193 people attended a baseball team’s games. What is the value of the digit 5 in the number of people?
Options:
a. 500
b. 5,000
c. 50,000
d. 500,000

Answer: 50,000

Explanation:
Given,
During one season, a total of 453,193 people attended a baseball team’s games.
Every digit in a number has a place value and the place value can be defined as the value represented by a digit in a number on the basis of its position in the number. So the place value of the digit 5 in 453,193 is 50,000.
Thus the correct answer is option C.

Question 2.
Hal forgot the number of people at the basketball game. He does remember that the number had a 3 in the tens place. Which number could Hal be thinking of?
Options:
a. 7,321
b. 3,172
c. 2,713
d. 1,237

Answer: 1,237

Explanation:
Given,
Hal forgot the number of people at the basketball game.
He does remember that the number had a 3 in the tens place.
a. 7,321 – the value of 3 in 7321 is 300.
b. 3,172 – the value of 3 in 3172 is 3000.
c. 2,713 – the value of 3 in 2713 is 3.
d. 1,237 – the value of 3 in 1237 is 30.
Thus the number 3 in tens place is 1,237.
Therefore, the correct answer is option D.

Spiral Review

Question 3.
Hot dog buns come in packages of 8. For the school picnic, Mr. Spencer bought 30 packages of hot dog buns. How many hot dog buns did he buy?
Options:
a. 24
b. 38
c. 110
d. 240

Answer: 240

Explanation:
Given,
Hot dog buns come in packages of 8.
For the school picnic, Mr. Spencer bought 30 packages of hot dog buns.
8 × 30 = 240 buns
He bought 240 hot dig buns.
Thus the correct answer is option D.

Question 4.
There are 8 students on the minibus. Five of the students are boys. What fraction of the students are boys?
Options:
a. \(\frac{3}{8}\)
b. \(\frac{5}{8}\)
c. \(\frac{5}{5}\)
d. \(\frac{8}{8}\)

Answer: \(\frac{5}{8}\)

Explanation:
There are 8 students on the minibus. Five of the students are boys.
Divide the number of boys by the total number of students on the minibus.
\(\frac{5}{8}\)
Thus the correct answer is option B.

Question 5.
The clock below shows the time when Amber leaves home for school. At what time does Amber leave home?
Go Math Grade 4 Answer Key Homework Practice FL Chapter 1 Place Value, Addition, and Subtraction to One Million Common Core - Place Value, Addition, and Subtraction to One Million img 2
Options:
a. 2:41
b. 8:02
c. 8:10
d. 8:20

Answer: 8:10

Explanation:
By seeing the above figure we can say that Amber leave home is 8:10.
Thus the correct answer is option C.

Question 6.
Jeremy drew a polygon with four right angles and four sides with the same length.
Go Math Grade 4 Answer Key Homework Practice FL Chapter 1 Place Value, Addition, and Subtraction to One Million Common Core - Place Value, Addition, and Subtraction to One Million img 3
What kind of polygon did Jeremy draw?
Options:
a. hexagon
b. square
c. trapezoid
d. triangle

Answer: square

Explanation:
A square has two pairs of parallel sides, four right angles, and all four sides are equal.
Thus the correct answer is option B.

Common Core – Place Value, Addition, and Subtraction to One Million – Page No. 5

Read and Write Numbers

Read and write the number in two other forms.

Question 1.
six hundred ninety-two thousand, four
standard form: 692,004;
expanded form: 600,000 + 90,000 + 2,000 + 4

Question 2.
314,207
Type below:
________

Answer:
Standard form: Three hundred fourteen thousand, two hundred seven.
Explanded form: 300,000 + 10,000 + 4,000 + 200 + 7

Question 3.
600,000 + 80,000 + 10
Type below:
________

Answer:
Standard form: 680,010
Expanded form: Six hundred eighty thousand ten.

Use the number 913,256.

Question 4.
Write the name of the period that has the digits 913.
________

Answer: thousands
Each group of three digits forms a period. The name of the period that has the digits 913 is thousands group.

Question 5.
Write the digit in the ten thousands place.
________

Answer: 1

Question 6.
Write the value of the digit 9.
________

Answer: 9 hundred thousands or 900,000.

Problem Solving

Use the table for 7 and 8.
Go Math Grade 4 Answer Key Homework Practice FL Chapter 1 Place Value, Addition, and Subtraction to One Million Common Core - Place Value, Addition, and Subtraction to One Million img 4

Question 7.
Which state had a population of eight hundred four thousand, one hundred ninety-four?
________

Answer: South Dakota

Explanation:
The standard form of eight hundred four thousand, one hundred ninety-four is 804,194.
We can see the population 804,194 in the above table in South Dakota.

Question 8.
What is the value of the digit 8 in Alaska’s population?
________

Answer: 8 ten thousands, or 80,000.

Explanation:
The population in Alaska is 686,293.
The value of the digit 8 in Alaska’s population is 80,000.

Common Core – Place Value, Addition, and Subtraction to One Million – Page No. 6

Lesson Check

Question 1.
Based on a 2008 study, children 6–11 years old spend sixty-nine thousand, one hundred eight minutes a year watching television. What is this number written in
standard form?
Options:
a. 6,918
b. 69,108
c. 69,180
d. 690,108

Answer: 69,108

Explanation:
Given,
Based on a 2008 study, children 6–11 years old spend sixty-nine thousand, one hundred eight minutes a year watching television.
The standard form of sixty-nine thousand, one hundred eight is 69,108.

Question 2.
What is the value of the digit 4 in the number 84,230?
Options:
a. 4
b. 400
c. 4,000
d. 40,000

Answer: 4,000

Explanation:
Every digit in a number has a place value and the place value can be defined as the value represented by a digit in a number on the basis of its position in the number. So the place value of the digit 4 in 84,230 is 4,000.
Thus the correct answer is option C.

Spiral Review

Question 3.
An ant has 6 legs. How many legs do 8 ants have in all?
Options:
a. 14
b. 40
c. 45
d. 48

Answer: 48

Explanation:
Given,
An ant has 6 legs.
To find:
How many legs do 8 ants have in all
6 legs × 8 = 48 legs
Thus the correct answer is option D.

Question 4.
Latricia’s vacation is in 4 weeks. There are 7 days in a week. How many days is it until Latricia’s vacation?
Options:
a. 9 days
b. 11 days
c. 20 days
d. 28 days

Answer: 28 days

Explanation:
Given,
Latricia’s vacation is in 4 weeks.
There are 7 days in a week.
4 × 7 days = 28 days
Thus the correct answer is option D.

Question 5.
Marta collected 363 cans. Diego collected 295 cans. How many cans did Marta and Diego collect in all?
Options:
a. 668
b. 658
c. 568
d. 178

Answer: 658

Explanation:
Marta collected 363 cans. Diego collected 295 cans.
363 cans + 295 cans = 658 cans
Marta and Diego collect 658 cans in all.
Thus the correct answer is option B.

Question 6.
The city Tim lives in has 106,534 people. What is the value of the 6 in 106,534?
Options:
a. 6,000
b. 600
c. 60
d. 6

Answer: 6,000

Explanation:
The city Tim lives in has 106,534 people.
The value of the 6 in 106,534 is 6,000.
Thus the correct answer is option A.

Common Core – Place Value, Addition, and Subtraction to One Million – Page No. 7

Compare and Order Numbers

Compare. Write < .> or =.

Question 1.
3,273 < 3,279

Question 2.
$1,323 ______ $1,400

Answer: <
The number $1,323 < $1,400 as 323 is less than 400.

Question 3.
52,692 ______ 52,692

Answer: =
The number 52,692 is equal to 52,692.

Question 4.
$413,005 ______ $62,910

Answer: >
The number $413,005 is greater than $62,910.

Question 5.
382,144 ______ 382,144

Answer: =
The number 382,144 is equal to 382,144.

Question 6.
157,932 ______ 200,013

Answer: <
The number 157,932 is less than 200,013.

Question 7.
401,322 ______ 410,322

Answer: <
The number 401,322 is less than 410,322.

Question 8.
989,063 ______ 980,639

Answer: >
The number 989,063 is greater than 980,639.

Question 9.
258,766 ______ 258,596

Answer: >
The number 258,766 is greater than 258,596.

Order from least to greatest.

Question 10.
23,710; 23,751; 23,715
______ < ______ < ______

Answer: 23,710; 23,715; 23,751

Question 11.
52,701; 54,025; 5,206
______ < ______ < ______

Answer: 5,206; 52,701; 54,025
The numbers from least to greatest is 5,206; 52,701; 54,025

Question 12.
465,321; 456,321; 456,231
______ < ______ < ______

Answer: 456,321; 456,231; 456,231
456,321 is less than 456,231 is less than 456,231. The numbers from least to greatest is 456,321; 456,231; 456,231.

Question 13.
$330,820; $329,854; $303,962
______ < ______ < ______

Answer: $303,962; $329,854; $330,820
$303,962 is less than $329,854 is less than $330,820. The numbers from least to greatest is $303,962; $329,854; $330,820.

Problem Solving

Question 14.
An online newspaper had 350,080 visitors in October, 350,489 visitors in November, and 305,939 visitors in December. What is the order of the months from greatest to least number of visitors?
1. ________
2. ________
3. ________

Answer:
1. November
2. October
3. December

Explanation:
Given,
An online newspaper had 350,080 visitors in October, 350,489 visitors in November, and 305,939 visitors in December.
350,489 is greater than 350,080 is greater than 305,939.
Thus the order of the months from greatest to least number of visitors is November, October and December.

Question 15.
The total land area in square miles of each of three states is shown below.
Colorado: 103,718
New Mexico: 121,356
Arizona: 113,635
What is the order of the states from least to greatest total land area?
1. ________
2. ________
3. ________

Answer:
1. Colorado
2. Arizona
3. New Mexico

Explanation:
The total land area in square miles of each of three states is shown below.
Colorado: 103,718
New Mexico: 121,356
Arizona: 113,635
The greatest number is 121,356, 113,635, 103,718
The order of the states from least to greatest total land area is Colorado, Arizona and New Mexico.

Common Core – Place Value, Addition, and Subtraction to One Million – Page No. 8

Lesson Check

Question 1.
At the yearly fund-raising drive, the nonprofit company’s goal was to raise $55,500 each day. After three days, it had raised $55,053; $56,482; and $55,593. Which amount was less than the daily goal?
Options:
a. $55,500
b. $55,053
c. $55,593
d. $56,482

Answer: $55,053

Explanation:
At the yearly fund-raising drive, the nonprofit company’s goal was to raise $55,500 each day. After three days, it had raised $55,053; $56,482; and $55,593.
$55,053 < $55,593 < $56,482
The amount was less than the daily goal is $55,053.
Thus the correct answer is option B.

Question 2.
Which of the following lists of numbers is in order from greatest to least?
Options:
a. 60,343; 60,433; 63,043
b. 83,673; 86,733; 86,373
c. 90,543; 90,048; 93,405
d. 20,433; 20,343; 20,043

Answer: 20,433; 20,343; 20,043

Explanation:
The lists of numbers is in order from greatest to least is 20,433; 20,343; 20,043
The correct answer is option D.

Spiral Review

Question 3.
Jess is comparing fractions. Which fraction is greater than \(\frac{5}{6}\)?
Options:
a. \(\frac{7}{8}\)
b. \(\frac{4}{5}\)
c. \(\frac{3}{4}\)
d. \(\frac{2}{3}\)

Answer: \(\frac{7}{8}\)

Explanation:
Given,
Jess is comparing fractions.
The fraction is greater than \(\frac{5}{6}\) is \(\frac{7}{8}\)
The correct answer is option A.

Question 4.
What is the perimeter of the rectangle below?
Go Math Grade 4 Answer Key Homework Practice FL Chapter 1 Place Value, Addition, and Subtraction to One Million Common Core - Place Value, Addition, and Subtraction to One Million img 5
Options:
a. 14 inches
b. 26 inches
c. 28 inches
d. 48 inches

Answer: 28 inches

Explanation:
Given,
l = 6 in
w = 8 in.
Perimeter of the rectangle = l + l + w + w
P = 6 in + 6 in + 8 in + 8 in
P = 28 inches
Thus the perimeter of the rectangle is 28 inches.
The correct answer is option C.

Question 5.
A website had 826,140 hits last month. What is the value of the 8 in 826,140?
Options:
a. 800
b. 8,000
c. 80,000
d. 800,000

Answer: 800,000

Explanation:
A website had 826,140 hits last month.
The value of the 8 in 826,140 is 800,000.
Thus the correct answer is option D.

Question 6.
Which is 680,705 written in expanded form?
Options:
a. 680 + 705
b. 68,000 + 700 + 5
c. 600,000 + 8,000 + 700 + 5
d. 600,000 + 80,000 + 700 + 5

Answer: 600,000 + 80,000 + 700 + 5

Explanation:
The expanded form of 680,705 is 600,000 + 80,000 + 700 + 5
The correct answer is option D.

Common Core – Place Value, Addition, and Subtraction to One Million – Page No. 9

Round Numbers 

Round to the place value of the underlined digit.

Question 1.
Go Math Grade 4 Answer Key Homework Practice FL Chapter 1 Place Value, Addition, and Subtraction to One Million Common Core - Place Value, Addition, and Subtraction to One Million img 6
Look at the digit to the right. If the digit to the right is less than 5, the digit in the rounding place stays the same.
Change all the digits to the right of the rounding place to zero.

Question 2.
123,499
_____

Answer: 123,000

Explanation:
Look at the digit to the right. If the digit to the right is less than 5, the digit in the rounding place stays the same.
Change all the digits to the right of the rounding place to zero.
The place value of 3 in 123,499 is 123,000.

Question 3.
552,945
_____

Answer: 600,000

Explanation:
Look at the digit to the right. If the digit to the right is less than 5, the digit in the rounding place stays the same.
Change all the digits to the right of the rounding place to zero.
The place value of 5 in 552,945 is 600,000.

Question 4.
389,422
_____

Answer: 390,000

Explanation:
Look at the digit to the right. If the digit to the right is less than 5, the digit in the rounding place stays the same.
Change all the digits to the right of the rounding place to zero.
The place value of 8 in 389,422 is 390,000.

Question 5.
209,767
_____

Answer: 200,000

Explanation:
Look at the digit to the right. If the digit to the right is less than 5, the digit in the rounding place stays the same.
Change all the digits to the right of the rounding place to zero.
The place value of 2 in 209,767 is 200,000.

Question 6.
191,306
_____

Answer: 191,000

Explanation:
Look at the digit to the right. If the digit to the right is less than 5, the digit in the rounding place stays the same.
Change all the digits to the right of the rounding place to zero.
The place value of 1 in 191,306 is 191,000.

Question 7.
66,098
_____

Answer: 70,000

Explanation:
Look at the digit to the right. If the digit to the right is less than 5, the digit in the rounding place stays the same.
Change all the digits to the right of the rounding place to zero.
The place value of 6 in 66,098 is 70,000.

Question 8.
73,590
_____

Answer: 74,000

Explanation:
Look at the digit to the right. If the digit to the right is less than 5, the digit in the rounding place stays the same.
Change all the digits to the right of the rounding place to zero.
The place value of 3 in 73,590 is 74,000.

Question 9.
149,903
_____

Answer: 100,000

Explanation:
Look at the digit to the right. If the digit to the right is less than 5, the digit in the rounding place stays the same.
Change all the digits to the right of the rounding place to zero.
The place value of 1 in 149,903 is 100,000.

Question 10.
684,303
_____

Answer: 684,000

Explanation:
Look at the digit to the right. If the digit to the right is less than 5, the digit in the rounding place stays the same.
Change all the digits to the right of the rounding place to zero.
The place value of 4 in 684,303 is 684,000.

Question 11.
499,553
_____

Answer: 500,000

Explanation:
Look at the digit to the right. If the digit to the right is less than 5, the digit in the rounding place stays the same.
Change all the digits to the right of the rounding place to zero.
The place value of 9 in 499,553 is 500,000.

Problem Solving

Use the table for 12–13.
Go Math Grade 4 Answer Key Homework Practice FL Chapter 1 Place Value, Addition, and Subtraction to One Million Common Core - Place Value, Addition, and Subtraction to One Million img 7

Question 12.
Find the height of Mt. Whitney in the table. Round the height to the nearest thousand feet.
_____ feet

Answer: 14,000 feet

Explanation:
The height to the nearest thousand feet for 14,494 is 14,000 feet.

Question 13.
What is the height of Mt. Bona rounded to the nearest ten thousand feet?
_____ feet

Answer: 20,000 feet

Explanation:
The height of Mt. Bona rounded to the nearest ten thousand feet for 16,500 is 20,000 feet.

Common Core – Place Value, Addition, and Subtraction to One Million – Page No. 10

Lesson Check

Question 1.
Which number is 247,039 rounded to the nearest thousand?
Options:
a. 200,000
b. 250,000
c. 247,000
d. 7,000

Answer: 247,000

Explanation:
Round off the value means making a number simpler but keeping its value close to what it was. The result is less accurate but easy to use. So the number is 247,039 rounded to the nearest thousand is 247,000.
Thus the correct answer is option C.

Question 2.
To the nearest ten thousand, the population of Vermont was estimated to be about 620,000 in 2008. Which might have been the exact population of Vermont in 2008?
Options:
a. 626,013
b. 621,270
c. 614,995
d. 609,964

Answer: 621,270

Explanation:
To the nearest ten thousand, the population of Vermont was estimated to be about 620,000 in 2008.
The exact population of Vermont in 2008 might be 621,270.
Thus the correct answer is option B.

Spiral Review

Question 3.
Which symbol makes the following number sentence true?
$546,322 Ο $540,997
Options:
a. <
b. >
c. =
d. +

Answer: >

Explanation:
$546,322 is greater than $540,997.
Thus the correct answer is option B.

Question 4.
Pittsburgh International Airport had approximately 714,587 passengers in August 2009. Which number is greater than 714,587?
Options:
a. 714,578
b. 704,988
c. 714,601
d. 714,099

Answer: 714,601

Explanation:
Given,
Pittsburgh International Airport had approximately 714,587 passengers in August 2009.
The number greater than 714,587 is 714,601.
Thus the correct answer is option C.

Question 5.
June made a design with 6 equal tiles. One tile is yellow, 2 tiles are blue, and 3 tiles are purple. What fraction of the tiles are yellow or purple?
Options:
a. \(\frac{1}{6}\)
b. \(\frac{2}{6}\)
c. \(\frac{3}{6}\)
d. \(\frac{4}{6}\)

Answer: \(\frac{4}{6}\)

Explanation:
Given,
June made a design with 6 equal tiles. One tile is yellow, 2 tiles are blue, and 3 tiles are purple.
We have to put the total number of tiles in the denominator.
The number of yellow or purple tiles is 3 + 1 = 4 put it in the numerator.
The fraction of the tiles are yellow or purple is \(\frac{4}{6}\).
Thus the correct answer is option D.

Question 6.
The fourth grade collected 40,583 cans and plastic bottles. Which of the following shows that number in word form?
Options:
a. forty thousand, five hundred eighty
b. forty thousand, five hundred eighty-three
c. four thousand, five hundred eighty-three
d. four hundred thousand, five hundred eighty

Answer: forty thousand, five hundred eighty-three

Explanation:
Given,
The fourth grade collected 40,583 cans and plastic bottles.
The expanded form of 40,583 is forty thousand, five hundred eighty-three.
Thus the correct answer is option B.

Common Core – Place Value, Addition, and Subtraction to One Million – Page No. 11

Rename Numbers

Rename the number. Use the place-value chart to help.

Question 1.
760 hundreds = 76,000
Go Math Grade 4 Answer Key Homework Practice FL Chapter 1 Place Value, Addition, and Subtraction to One Million Common Core - Place Value, Addition, and Subtraction to One Million img 8

Question 2.
805 tens = ________

THOUSANDS ONES
Hundreds Tens Ones Hundreds Tens Ones
_________ _________ _________ _________

Answer:

THOUSANDS ONES
Hundreds Tens Ones Hundreds Tens Ones
8 0 5 0

Question 3.
24 ten thousands = ________

THOUSANDS ONES
Hundreds Tens Ones Hundreds Tens Ones
_________ _________ _________ _________ _________ _________

Answer:

THOUSANDS ONES
Hundreds Tens Ones Hundreds Tens Ones
2 4 0 0 0 0

Rename the number.

Question 4.
720 = ____ tens

Answer: 72

Explanation:
720 can be calculated as 72 × 10 = 72 tens.

Question 5.
4 thousands 7 hundreds = 47 ________

Answer: hundreds

Explanation:
4 thousands 7 hundreds
4700 = 47 × 100 = 47 hundreds

Question 6.
25,600 = ____ hundreds

Answer: 256

Explanation:
25,600 = 256 × 100 = 256 hundreds

Question 7.
204 thousands = ____

Answer: 204,000

Explanation:
204 thousands = 204 × 1000 = 204,000.

Problem Solving

Question 8.
For the fair, the organizers ordered 32 rolls of tickets. Each roll of tickets has 100 tickets. How many tickets were ordered in all?
____ tickets

Answer: 3,200 tickets

Explanation:
Given,
For the fair, the organizers ordered 32 rolls of tickets. Each roll of tickets has 100 tickets.
32 × 100 tickets = 3200 tickets
Therefore 3200 tickets were ordered in all.

Question 9.
An apple orchard sells apples in bags of 10. The orchard sold a total of 2,430 apples one day. How many bags of apples was this?
____ bags

Answer: 243 bags

Explanation:
Given,
An apple orchard sells apples in bags of 10. The orchard sold a total of 2,430 apples one day.
2430/10 = 243 bags
There were 243 bags of apples.

Question 10.
Explain how you can rename 5,400 as hundreds. Include a quick picture or a place-value chart in your explanation.
____ hundreds

Answer: 54

Explanation:
It would be 54 hundreds because:
In 5400 there are 2 zeros
Also in 100, there are 2 zeros
2 zeros equals a hundred
100=1 hundred, because it has a 1 in front of the 2 zeros
5400=54 hundreds, because it has a 54 in front of the 2 zeros

Common Core – Place Value, Addition, and Subtraction to One Million – Page No. 12

Lesson Check

Question 1.
A dime has the same value as 10 pennies. Marley brought 290 pennies to the bank. How many dimes did Marley get?
Options:
a. 29
b. 290
c. 2,900
d. 29,000

Answer: 29

Explanation:
Given,
A dime has the same value as 10 pennies. Marley brought 290 pennies to the bank.
To find How many dimes did Marley get we have to divide 290 pennies by 10 pennies.
290/10 = 29
Therefore Marley gets 29 pennies.
Thus the correct answer is option a.

Question 2.
A citrus grower ships grapefruit in boxes of 10. One season, the grower shipped 20,400 boxes of grapefruit. How many grapefruit were shipped?
Options:
a. 204
b. 2,040
c. 20,400
d. 204,000

Answer: 204,000

Explanation:
Given,
A citrus grower ships grapefruit in boxes of 10.
One season, the grower shipped 20,400 boxes of grapefruit.
We need to find How many grapefruit were shipped.
Multiply 20,400 boxes with 10.
20,400 × 10 = 204,000
Therefore 204,000 grapefruit were shipped.
Thus the correct answer is option d.

Spiral Review

Question 3.
There were 2,605 people at the basketball game. A reporter rounded this number to the nearest hundred for a newspaper article. What number did the reporter use?
Options:
a. 2,600
b. 2,610
c. 2,700
d. 3,000

Answer: 2,600

Explanation:
Given,
There were 2,605 people at the basketball game. A reporter rounded this number to the nearest hundred for a newspaper article.
To find:
What number did the reporter use?
The number 2605 nearest to the hundred is 2600.
Thus the correct answer is option a.

Question 4.
To get to Level 3 in a game, a player must score 14,175 points. Ann scores 14,205 points, Ben scores 14,089 points, and Chuck scores 10,463 points. Which score is greater than the Level 3 score?
Options:
a. 14,205
b. 14,175
c. 14,089
d. 10,463

Answer: 14,205

Explanation:
Given,
To get to Level 3 in a game, a player must score 14,175 points. Ann scores 14,205 points, Ben scores 14,089 points, and Chuck scores 10,463 points.
By seeing the above points we can say that 14,205 is greater than level 3.
Thus the correct answer is option a.

Question 5.
Henry counted 350 lockers in his school. Hayley counted 403 lockers in her school. Which statement is true?
Options:
a. The 3 in 350 is 10 times the value of the 3 in 403.
b. The 3 in 350 is 100 times the value of the 3 in 403.
c. The 3 in 403 is 10 times the value of the 3 in 350.
d. The 3 in 403 is 100 times the value of the 3 in 350.

Answer: The 3 in 350 is 100 times the value of the 3 in 403.

Explanation:
Given,
Henry counted 350 lockers in his school. Hayley counted 403 lockers in her school.
The statement “The 3 in 350 is 100 times the value of the 3 in 403” is true.
Thus the correct answer is option b.

Question 6.
There are 4 muffins on each plate. There are 0 plates of lemon muffins. How many lemon muffins are there?
Options:
a. 4
b. 2
c. 1
d. 0

Answer: 0

Explanation:
Given,
There are 4 muffins on each plate. There are 0 plates of lemon muffins.
Multiply the number of muffins with the number of plates.
4 × 0 = 0
There are 0 lemon muffins.
Thus the correct answer is option d.

Common Core – Place Value, Addition, and Subtraction to One Million – Page No. 13

Add Whole Numbers

Estimate. Then find the sum.

Question 1.
Estimate: 90,000
Go Math Grade 4 Answer Key Homework Practice FL Chapter 1 Place Value, Addition, and Subtraction to One Million Common Core - Place Value, Addition, and Subtraction to One Million img 9

Question 2.
73,404
+ 27,865
————
Estimate: _______
Sum: _______

Answer:
Estimate: 100,000
Sum: 101,269
Estimate:
The number rounded to 73,404 is 70,000.
The number rounded to 27,865 is 30,000.
70,000
+30,000
100,000
Sum:
73,404
+ 27,865
101,269

Question 3.
404,446
+ 396,755
————
Estimate: _______
Sum: _______

Answer:
Estimate: 800,000
Sum: 800,201
Estimate:
The number rounded to 400,000
The number rounded to 400,000
400,000
+400,000
800,000
Sum:
404,446
+ 396,755
800,201

Question 4.
137,638
+ 52,091
————
Estimate: _______
Sum: _______

Answer:
Estimate: 200,000
Sum: 189,729
Estimate:
The number rounded to 150,000
The number rounded to 50,000
150,000
+50,000
200,000
Sum:
137,638
+ 52,091
189,729

Question 5.
200,629
+ 28,542
————
Estimate: _______
Sum: _______

Answer:
Estimate: 250,000
Sum: 229,171
Estimate:
The number rounded to 200,000
The number rounded to 50,000
200,000
+50,000
250,000
Sum:
200,629
+ 28,542
229,171

Question 6.
212,514
+ 396,705
————
Estimate: _______
Sum: _______

Answer:
Estimate: 600,000
Sum: 609,219
Estimate:
The number rounded to 200,000
The number rounded to 400,000
200,000
+400,000
600,000
Sum:
212,514
+ 396,705
609,219

Question 7.
324,867
+ 6,233
————
Estimate: _______
Sum: _______

Answer:
Estimate: 330,000
Sum: 331,100
Estimate:
The number rounded to 324,000
The number rounded to 6,000
324,000
+ 6,000
330,000
Sum:
324,867
+ 6,233
331,100

Question 8.
462,809
+ 256,738
————
Estimate: _______
Sum: _______

Answer:
Estimate: 800,000
Sum: 719,547
Estimate:
The number rounded to 500,000
The number rounded to 300,000
500,000
+300,000
800,000
Sum:
462,809
+ 256,738
719,547

Question 9.
624,836
+ 282,189
————
Estimate: _______
Sum: _______

Answer:
Estimate: 900,000
Sum: 907,025
Estimate:
The number rounded to 600,000
The number rounded to 300,000
600,000
+300,000
900,000
Sum:
624,836
+ 282,189
907,025

Problem Solving

Use the table for 10–12.
Go Math Grade 4 Answer Key Homework Practice FL Chapter 1 Place Value, Addition, and Subtraction to One Million Common Core - Place Value, Addition, and Subtraction to One Million img 10

Question 10.
Beth and Cade were on one team. What was their total score?
______

Answer: 407,502

Explanation:
The score of Beth is 251,567
The score of Cade is 155,935
251,567
+155,935
407,502
Thus the total score of Beth and Cade is 407,502.

Question 11.
Dillan and Elaine were on the other team. What was their total score?
______

Answer: 409,928

Explanation:
The score of Dillan is 188,983
The score of Elaine is 220,945
188,983
+220,945
409,928
The total score of Dillan and Elaine is 409,928.

Question 12.
Which team scored the most points?
_________

Answer: Dillan and Elaine
The total score of Dillan and Elaine is 409,928.
The total score of Beth and Cade is 407,502.
409,928
-407,502
002,226
Thus Dillan and Elaine team scored the most points.

Common Core – Place Value, Addition, and Subtraction to One Million – Page No. 14

Lesson Check

Question 1.
The coastline of the United States is 12,383 miles long. Canada’s coastline is 113,211 miles longer than the coastline of the United States. How long is the coastline of Canada?
Options:
a. 100,828 miles
b. 115,594 miles
c. 125,594 miles
d. 237,041 miles

Answer: 125,594 miles

Explanation:
Given,
The coastline of the United States is 12,383 miles long.
Canada’s coastline is 113,211 miles longer than the coastline of the United States.
113,211
+12,383 
125,594
Therefore, the coastline of Canada is 125,594 miles.
Thus the correct answer is option c.

Question 2.
Germany is the seventh largest European country and is slightly smaller by area than Montana. Germany has a land area of 134,835 square miles and a water area of 3,011 square miles. What is the total area of Germany?
Options:
a. 7,846 square miles
b. 131,824 square miles
c. 137,846 square miles
d. 435,935 square miles

Answer: 137,846 square miles

Explanation:
Given,
Germany is the seventh largest European country and is slightly smaller by area than Montana. Germany has a land area of 134,835 square miles and a water area of 3,011 square miles.
134,835
+ 3,011
137,846
Therefore the total area of Germany is 137,846 square miles.
Thus the correct answer is option c.

Spiral Review

Question 3.
In an election, about 500,000 people voted in all. Which number could be the exact number of people who voted in the election?
Options:
a. 429,455
b. 441,689
c. 533,736
d. 550,198

Answer: 533,736

Explanation:
Given,
In an election, about 500,000 people voted in all.
The number near to 500,000 is 533,736.
Thus the correct answer is option c.

Question 4.
In 2007, Pennsylvania had approximately 121,580 miles of public roads. What is 121,580 rounded to the nearest thousand?
Options:
a. 100,000
b. 120,000
c. 121,000
d. 122,000

Answer: 122,000

Explanation:
Given,
In 2007, Pennsylvania had approximately 121,580 miles of public roads.
121,580 rounded to the nearest thousand is 122,000.
Thus the correct answer is option d.

Question 5.
Which of the following lists of numbers is in order from greatest to least?
Options:
a. 33,093; 33,903; 33,309
b. 42,539; 24,995; 43,539
c. 682,131; 628,000; 682,129
d. 749,340; 740,999; 740,256

Answer: 749,340; 740,999; 740,256

Explanation:
a. 33,093; 33,903; 33,309
33,093 = 33,903 = 33,309
b. 42,539; 24,995; 43,539
42,539 > 24,995 < 43,539
c. 682,131; 628,000; 682,129
682,131 > 628,000 < 682,129
d. 749,340; 740,999; 740,256
749,340 > 740,999 > 740,256
Thus the correct answer is option d.

Question 6.
Which symbol makes the following statement true?
$413,115 Go Math Grade 4 Answer Key Homework Practice FL Chapter 1 Place Value, Addition, and Subtraction to One Million Common Core - Place Value, Addition, and Subtraction to One Million img 11 $431,511
Options:
a. <
b. >
c. =
d. +

Answer: <

Explanation:
The number $413,115 is less than $431,511
$413,115 < $431,511
Thus the correct answer is option a.

Common Core – Place Value, Addition, and Subtraction to One Million – Page No. 15

Subtract Whole Numbers

Estimate. Then find the difference.

Question 1.
Go Math Grade 4 Answer Key Homework Practice FL Chapter 1 Place Value, Addition, and Subtraction to One Million Common Core - Place Value, Addition, and Subtraction to One Million img 12

Question 2.
428,731
– 175,842
————-
Estimate: _______
Difference: _______

Answer:
Estimate: 200,000
Difference: 252,889
Estimate:
The number rounded to 428,731 is 400,000
The number rounded to 175,842 is 200,000
400,000
-200,000
200,000
Difference:
428,731
– 175,842
252,889

Question 3.
920,026
– 535,722
————-
Estimate: _______
Difference: _______

Answer:
Estimate: 400,000
Difference: 384,304
Estimate:
The number rounded to 920,026 is 900,000
The number rounded to 535,722 is 500,000
900,000
-500,000
400,000
Difference:
920,026
– 535,722
384,304

Question 4.
253,495
– 48,617
————-
Estimate: _______
Difference: _______

Answer:
Estimate: 200,000
Difference: 204,878
Estimate:
The number rounded to 253,495 is 250,000
The number rounded to 48,617 is 50,000

Subtract. Add to check.

Question 5.
735,249 – 575,388 = ______
______ + ______ = ______

Answer: 159,861
735,249
-575,388 
159,861
Now check whether the answer is correct or wrong.
159,861
+575,388 
735,249

Question 6.
512,724 – 96,473 = ______
______ + ______ = ______

Answer: 416,251
512,724
-96,473 
416,251
Now check whether the answer is correct or wrong.
416,251
96,473
512,724

Question 7.
600,000 – 145,782 = ______
______ + ______ = ______

Answer: 454,218
600,000
-145,782
454,218
Now check whether the answer is correct or wrong.
454,218
+145,782 
600,000

Problem Solving

Use the table for 8 and 9.
Go Math Grade 4 Answer Key Homework Practice FL Chapter 1 Place Value, Addition, and Subtraction to One Million Common Core - Place Value, Addition, and Subtraction to One Million img 13

Question 8.
How many more people attended the Magic’s games than attended the Pacers’ games?
______ people

Answer: 133,606

Explanation:
Number of people attended Magic’s games = 715,901
Number of people attended Pacers’ games = 582,295
To find:
How many more people attended the Magic’s games than attended the Pacers’ games
We need to subtract the Number of people attended Pacers’ games from the Number of people attended Magic’s games
715,901
-582,295
133,606

Question 9.
How many fewer people attended the Pacers’ games than attended the Clippers’ games?
______ people

Answer: 87,768

Explanation:
Number of people attended Pacers’ games = 582,295
Number of people attended Clippers’ games = 670,063
To find:
How many fewer people attended the Pacers’ games than attended the Clippers’ games
We need to subtract the number of people attended Pacers’ games from the Number of people attended Clippers’ games
670,063
-582,295
87,768

Common Core – Place Value, Addition, and Subtraction to One Million – Page No. 16

Lesson Check

Question 1.
This year, a farm planted 400,000 corn stalks. Last year, the farm planted 275,650 corn stalks. How many more corn stalks did the farm plant this year than last year?
Options:
a. 124,350
b. 125,450
c. 235,450
d. 275,650

Answer: 124,350

Explanation:
Given,
This year, a farm planted 400,000 corn stalks.
Last year, the farm planted 275,650 corn stalks.
400,000
-275,650 
124,350
Thus the correct answer is option a.

Question 2.
One machine can make 138,800 small paper clips in one day. Another machine can make 84,250 large paper clips in one day. How many more small paper clips than large paper clips are made by the two machines in one day?
Options:
a. 44,550
b. 54,550
c. 54,650
d. 154,650

Answer: 54,550

Explanation:
Given,
One machine can make 138,800 small paper clips in one day.
Another machine can make 84,250 large paper clips in one day
138,800
-84,250 
54,550
Thus the correct answer is option b.

Spiral Review

Question 3.
In three baseball games over a weekend, 125,429 people came to watch. The next weekend, 86,353 came to watch the games. How many people in all watched
the six baseball games?
Options:
a. 201,782
b. 211,772
c. 211,782
d. 211,882

Answer: 211,782

Explanation:
Given,
In three baseball games over a weekend, 125,429 people came to watch.
The next weekend, 86,353 came to watch the games.
125,429
+86,353 
211,782
Thus the correct answer is option c.

Question 4.
Kevin read the number “two hundred seven thousand, forty-eight” in a book. What is this number in standard form?
Options:
a. 27,048
b. 27,480
c. 207,048
d. 207,480

Answer: 207,048

Explanation:
Given,
Kevin read the number “two hundred seven thousand, forty-eight” in a book.
The standard form of two hundred seven thousand, forty-eight is 207,048.
Thus the correct answer is option c.

Question 5.
A museum had 275,608 visitors last year. What is this number rounded to the nearest thousand?
Options:
a. 275,600
b. 276,000
c. 280,000
d. 300,000

Answer: 276,000

Explanation:
A museum had 275,608 visitors last year.
The number 275,608 rounded to the nearest thousand is 276,000
Thus the correct answer is option b.

Question 6.
At the Millville Theater, a play ran for several weeks. In all, 28,175 people saw the play. What is the value of the digit 8 in 28,175?
Options:
a. 8
b. 800
c. 8,000
d. 80,000

Answer: 8,000

Explanation:
At the Millville Theater, a play ran for several weeks. In all, 28,175 people saw the play.
Every digit in a number has a place value and the place value can be defined as the value represented by a digit in a number on the basis of its position in the number. So the place value of the digit 8 in 28,175 is 8000.
Thus the correct answer is option c.

Common Core – Place Value, Addition, and Subtraction to One Million – Page No. 17

Problem Solving Comparison Problems with Addition and Substraction

Use the information in the table for 1–3.

Question 1.
How many square miles larger is the surface area of Lake Huron than the surface area of Lake Erie?
Think: How can a bar model help represent the problem? What equation can be written?
Go Math Grade 4 Answer Key Homework Practice FL Chapter 1 Place Value, Addition, and Subtraction to One Million Common Core - Place Value, Addition, and Subtraction to One Million img 14

Question 1.
Go Math Grade 4 Answer Key Homework Practice FL Chapter 1 Place Value, Addition, and Subtraction to One Million Common Core - Place Value, Addition, and Subtraction to One Million img 15

Question 2.
Which lake has a surface area that is 14,938 square miles greater than the surface area of Lake Ontario? Draw a model and write a number sentence to solve the problem.
_________

Answer:

Explanation:
The surface area of Lake Ontario is 7,340 square miles.
14,938
+7,340
22,278 square miles
Go Math grade 4 answer key chapter 1 Homework Practice FL img-1

Question 3.
Lake Victoria has the largest surface area of all lakes in Africa. Its surface area is 26,828 square miles. How much larger is the surface area of Lake Superior than that of Lake Victoria?
_____ square milles

Answer: 4,872 square miles

Explanation:
The surface area of Lake Victoria is 26,828 square miles.
The surface area of Lake Superior is 31,700 square miles.
31,700
-26,828
04,872
The surface area of Lake Superior is 4,872 square miles larger than Lake Victoria.

Question 4.
At 840,000 square miles, Greenland is the largest island in the world. The second-largest island is New Guinea, at 306,000 square miles. How much larger is Greenland than New Guinea?
_____ square milles

Answer: 534,000 square miles

Explanation:
Given,
At 840,000 square miles, Greenland is the largest island in the world.
The second-largest island is New Guinea, at 306,000 square miles.
840,000
-306,000
534,000
Greenland is 534,000 square miles larger than New Guinea.

Common Core – Place Value, Addition, and Subtraction to One Million – Page No. 18

Lesson Check

Question 1.
The Mariana Trench in the Pacific Ocean is about 36,201 feet deep. The Puerto Rico Trench in the Atlantic Ocean is about 27,493 feet deep. Based on these data, how many feet deeper is the Mariana Trench than the Puerto Rico Trench?
Options:
a. 8,708 feet
b. 9,718 feet
c. 9,808 feet
d. 63,694 feet

Answer: 8,708 feet

Explanation:
Given,
The Mariana Trench in the Pacific Ocean is about 36,201 feet deep.
The Puerto Rico Trench in the Atlantic Ocean is about 27,493 feet deep.
36,201
-27,493
08,708 feet
Mariana Trench is 8708 feet deeper than the Puerto Rico Trench.
Thus the correct answer is option a.

Question 2.
At 1,932 feet, Crater Lake, Oregon, is the deepest lake in the United States. The world’s deepest lake, Lake Baykal in Russia, is 3,383 feet deeper. How deep is Lake Baykal?
Options:
a. 3,383 feet
b. 4,215 feet
c. 4,315 feet
d. 5,315 feet

Answer: 5,315 feet

Explanation:
At 1,932 feet, Crater Lake, Oregon, is the deepest lake in the United States.
The world’s deepest lake, Lake Baykal in Russia, is 3,383 feet deeper.
3383
+1932
5315
Thus the correct answer is option d.

Spiral Review

Question 3.
Which of the following amounts is greater than $832,458?
Options:
a. $82,845
b. $832,458
c. $823,845
d. $832,485

Answer: $832,485

Explanation:
We have to compare all the options with $832,458
a. $82,845 < $832,458
b. $832,458 = $832,458
c. $823,845 < $832,458
d. $832,485 > $832,458
Thus the correct answer is option d.

Question 4.
A stadium in Pennsylvania seats 107,282 people. A stadium in Arizona seats 71,706 people. Based on these facts, how many more people does the stadium in Pennsylvania seat than the stadium in Arizona?
Options:
a. 35,576
b. 35,586
c. 36,576
d. 178,988

Answer: 35,576

Explanation:
Given,
A stadium in Pennsylvania seats 107,282 people. A stadium in Arizona seats 71,706 people.
107,282
-71,706 
35,576
Thus the correct answer is option a.

Question 5.
Which of the following numbers is 399,713 rounded to the place value of the underlined digit?
Options:
a. 390,000
b. 398,000
c. 399,800
d. 400,000

Answer: 400,000

Explanation:
The number 399,713 rounded to the place value of the underlined digit is 400,000.
Thus the correct answer is option d.

Question 6.
About 400,000 people visited an art museum in December. Which number could be the exact number of people who visited the art museum?
Options:
a. 478,051
b. 452,223
c. 352,483
d. 348,998

Answer: 352,483

Explanation:
About 400,000 people visited an art museum in December.
The number that could be the exact number of people who visited the art museum is 352,483.
Thus the correct answer is option c.

Common Core – Place Value, Addition, and Subtraction to One Million – Page No. 19

Lesson 1.1

Find the value of the underlined digit.
Go Math Grade 4 Answer Key Homework Practice FL Chapter 1 Place Value, Addition, and Subtraction to One Million Common Core - Place Value, Addition, and Subtraction to One Million img 16

Question 1.
6,493
____

Answer: 90

Explanation:
Every digit in a number has a place value and the place value can be defined as the value represented by a digit in a number on the basis of its position in the number. So the place value of the digit 9 in 6,493 is 90.

Question 2.
16,403
____

Answer: 10,000

Explanation:
Every digit in a number has a place value and the place value can be defined as the value represented by a digit in a number on the basis of its position in the number. So the place value of the digit 1 in 16,403 is 10,000.

Question 3.
725,360
____

Answer: 300

Explanation:
Every digit in a number has a place value and the place value can be defined as the value represented by a digit in a number on the basis of its position in the number. So the place value of the digit 3 in 725,360 is 300.

Question 4.
952,635
____

Answer: 900,000

Explanation:
Every digit in a number has a place value and the place value can be defined as the value represented by a digit in a number on the basis of its position in the number. So the place value of the digit 9 in 952,635 is 900,000.

Compare the values of the underlined digits in 46,395 and 14,906.

Question 5.
The value of 4 in ____ is ____ times the value of 4 in ____.

Answer: The value of 4 in 46,395 is 10 times the value of 4 in 14,906.

Explanation:
Every digit in a number has a place value and the place value can be defined as the value represented by a digit in a number on the basis of its position in the number. So the place value of the digit 4 in 46,395 is 40,000. And the place value of the digit 4 in 14,906 is 4,000. So, the value of 4 in 46,395 is 10 times the value of 4 in 14,906.

Lesson 1.2

Read and write the number in two other forms.

Question 6.
304,001
word form: _______
expanded form: _______

Answer:
word form: three hundred four thousand one
expanded form: 300,000 + 4000 + 1

Explanation:
Convert the number 304,001 into the word form three hundred four thousand one.
The expanded form of 304,001 is 300,000 + 4000 + 1

Question 7.
two hundred eight thousand, five hundred sixty-one
standard form: _______
_______

Answer:
The standard form of two hundred eight thousand, five hundred sixty-one is 208,561.
The expanded form of 208,561 is 200,000 + 8,000 + 500 + 60 + 1

Use the number 751,486.

Question 8.
Write the name of the period that has the digits 486.
_________

Answer: The name of the period that has the digits 486 is Ones.

Question 9.
Write the name of the period that has the digits 751.
_________

Answer: The name of the period that has the digits 751 is thousands.

Question 10.
Write the digit in the thousands place.
The digit in the thousands place: ____

Answer: The digit in the thousands place is 1.

Question 11.
Write the value of the digit 5.
____

Answer: The value of the digit 5 in 751,486 is 50,000.

Lesson 1.3

Compare. Write <, >, or =.

Question 12.
6,930 ____ 7,023

Answer: <

Explanation:
The number 6,930 is less than 7,023
6,930 < 7,023

Question 13.
98,903 ____ 98,930

Answer: <

Explanation:
The number 98,903 is less than 98,930
98,903 < 98,930

Question 14.
549,295 ____ 547,364

Answer: >

Explanation:
The number 549,295 is greater than 547,364
549,295 > 547,364

Order from least to greatest.

Question 15.
$26,940; $25,949; $26,490
Options:
a. $25,949; $26,490; $26,940
b. $26,490; $25,949; $26,940
c. $26,940; $25,949; $26,490

Answer: $25,949; $26,490; $26,940

Explanation:
We have to write the numbers from the least to the greatest.
$25,949 < $26,490 < $26,940
The order from the least to the greatest is $25,949; $26,490; $26,940
Thus the correct answer is option a.

Question 16.
634,943; 639,443; 589,932
Options:
a. 639,443; 589,932; 634,943
b. 634,943; 639,443; 589,932
c. 589,932; 634,943; 639,443

Answer: 589,932; 634,943; 639,443

Explanation:
We have to write the numbers from the least to the greatest.
589,932 < 634,943 < 639,443
The order from the least to the greatest is 589,932; 634,943; 639,443
Thus the correct answer is option c.

Common Core – Place Value, Addition, and Subtraction to One Million – Page No. 20

Lesson 1.4

Round to the place value of the underlined digit.

Question 1.
286,476
____

Answer: 286,000

Explanation:
Look at the digit to the right. If the digit to the right is less than 5, the digit in the rounding place stays the same.
Change all the digits to the right of the rounding place to zero.
The place value of 6 in 286,476 is 286,000.

Question 2.
289,342
____

Answer: 289,000

Explanation:
Look at the digit to the right. If the digit to the right is less than 5, the digit in the rounding place stays the same.
Change all the digits to the right of the rounding place to zero.
The place value of 9 in 289,342 is 289,000.

Question 3.
245,001
____

Answer: 250,000

Explanation:
Look at the digit to the right. If the digit to the right is less than 5, the digit in the rounding place stays the same.
Change all the digits to the right of the rounding place to zero.
The place value of 4 in 245,001 is 250,000.

Question 4.
183,002
____

Answer: 200,000

Explanation:
Look at the digit to the right. If the digit to the right is less than 5, the digit in the rounding place stays the same.
Change all the digits to the right of the rounding place to zero.
The place value of 1 in 183,002 is 200,000.

Lesson 1.5

Rename the number.

Question 5.
82 thousands = ____

Answer: 82,000

Explanation:
82 thousands = 82 × 1000 = 82,000.

Question 6.
600,000 = ____ ten thousands

Answer: 60

Explanation:
600,000 = 60 × 10000
60 × 10000 = 60 ten thousands

Question 7.
9,200 = ____ hundreds

Answer: 92

Explanation:
9,200 = 92 × 100 = 92 hundreds

Question 8.
8 ten thousands 4 hundreds = ____

Answer: 80,400

Explanation:
8 ten thousands 4 hundreds
8 × 10,000 + 4 × 100 = 80,000 + 400 = 80,400

Lesson 1.6

Estimate. Then find the sum.

Question 9.
94,903
+ 49,995
————
Estimate: ________
Sum: ________

Answer:
Estimate: 140000
Sum: 144898

Explanation:
Estimate:
The number rounded to 94,903 is 90,000
The number rounded to 49,995 is 50,000
90,000
+50,000
140,000
Sum:
94,903
+ 49,995
144,898

Question 10.
420,983
+ 39,932
————
Estimate: ________
Sum: ________

Answer:
Estimate: 460,000
Sum: 460915

Explanation:
Estimate:
The number rounded to 420,983 is 420,000
The number rounded to 39,932 is 40,000
420,000
+40,000
460,000
Sum:
420,983
+39,932
460,915

Question 11.
540,943
+ 382,093
————
Estimate: ________
Sum: ________

Answer:
Estimate: 940,000
Sum: 923036

Explanation:
Estimate:
The number rounded to 540,943 is 540,000
The number rounded to 382,093 is 400,000
540,000
+400,000
940,000
Sum:
540,943
+ 382,093
923,036

Lesson 1.7

Estimate. Then find the difference.

Question 12.
25,953
– 9,745
————
Estimate: ________
Difference: ________

Answer:
Estimate: 15,000
Difference: 16,208

Explanation:
Estimate:
The number rounded to 25,953 is 25,000
The number rounded to 9,745 is 10,000.
25,000
-10,000
15,000
Difference:
25,953
– 9,745
16,208

Question 13.
740,758
– 263,043
————
Estimate: ________
Difference: ________

Answer:
Estimate: 450,000
Difference: 477715

Explanation:
Estimate:
The number rounded to 740,758 is 750,000
The number rounded to 263,043 is 300,000
750,000
-300,000
450,000
Difference:
740,758
– 263,043
477,715

Question 14.
807,632
– 592,339
————
Estimate: ________
Difference: ________

Answer:
Estimate: 200,000
Difference: 215293

Explanation:
Estimate:
The number rounded to 807,632 is 800,000
The number rounded to 592,339 is 600,000
800,000
-600,000
200,000
Difference:
807,632
– 592,339
215293

Lesson 1.8

Question 15.
The attendance for the first game of the football season was 93,584. The attendance for the second game was 104,227. How many more people attended the second game than the first game?
______ people

Answer: 10643 people

Explanation:
Given,
The attendance for the first game of the football season was 93,584.
The attendance for the second game was 104,227.
104,227
-93,584
10,643
Thus, 10,643 more people attended the second game than the first game.

Question 16.
Abby and Lee sold raffle tickets to raise money for a new playground. Abby sold 1,052 tickets. Lee sold 379 more tickets than Abby. How many tickets did Lee sell?
______ tickets

Answer: 1431 tickets

Explanation:
Given,
Abby and Lee sold raffle tickets to raise money for a new playground.
Abby sold 1,052 tickets. Lee sold 379 more tickets than Abby.
1,052
+379 
1431
Therefore, Lee sell 1431 tickets.

Conclusion

All these solutions explained in the Go Math Grade 4 Answer Key Homework Practice FL Chapter 1 Place Value, Addition, and Subtraction to One Million are easy to understand. Check out HMH Go Math Grade 4 Chapter 1 Solution Key to score good grades in the exams. For any queries go through the exercise problems of Go Math Grade 4 Answer Key Chapter 1 Place Value, Addition, and Subtraction to One Million pdf.

Go Math Grade 4 Answer Key Chapter 7 Add and Subtract Fractions

go-math-grade-4-answer-key-chapter-7-add-and-subtract-fractions

One of the best study guides for grade 4 students is Go Math Grade 4 Answer Key Chapter 7 Add and Subtract Fractions. Make use of these pdf formatted chapter 7 Go Math HMH 4th Grade Answer Key for free and learn the topics efficiently. Download the Go Math Grade 4 Answer Key Chapter 7 Add and Subtract Fractions pdf from here and get the step-wise answers to all the questions. From this page, you’ll find the different possible models & techniques that students use to find the correct way to solve the fractions.

Go Math Grade 4 Answer Key Chapter 7 Add and Subtract Fractions

Approaching the best ways will make you understand the concepts of adding and subtracting fractions. Master in the Go Math Grade 4 Chapter 7 Add and Subtract Fractions by using the clear cut explanation for all the questions with images. Obtain the knowledge to write the fractions as sum and subtractions from Go Math Grade 4 Solution Key of Chapter 7 Add and Subtract Fractions.

Lesson: 1 – Add and Subtract Parts of a Whole

Lesson: 2 – Add and Subtract Parts of a Whole

Lesson: 3 – Add and Subtract Parts of a Whole

Lesson: 4 – Add and Subtract Parts of a Whole

Lesson: 5 – Add Fractions Using Models

Lesson: 6 – Subtract Fractions Using Models

Lesson: 7 – Subtract Fractions Using Models

Lesson: 8 – Add and Subtract Fractions

Lesson: 9 – Add and Subtract Fractions

Lesson: 10 – Add and Subtract Fractions

Lesson: 11 – Rename Fractions and Mixed Numbers

Lesson: 12 – Rename Fractions and Mixed Numbers

Lesson: 13 – Add and Subtract Mixed Numbers

Lesson: 14 – Add and Subtract Mixed Numbers

Lesson: 15 – Record Subtraction with Renaming

Lesson: 16 – Record Subtraction with Renaming

Lesson: 17 – Fractions and Properties of Addition

Lesson: 18 – Fractions and Properties of Addition

Lesson: 19 – Fractions and Properties of Addition

Lesson: 20 – Fractions and Properties of Addition

Lesson: 21 – Fractions and Properties of Addition

Lesson: 22 – Fractions and Properties of Addition

Add and Subtract Parts of a Whole Page No – 389

Use the model to write an equation.

Question 1:
Go Math Grade 4 Answer Key Chapter 7 Add and Subtract Fractions Page 389 Question 1

Answer: 3/8 + 2/8 = 5/8

Explanation:
By seeing the above 3 figures we can say that the fraction of the shaded part of the first circle is 3/8, the fraction of the second figure is 2/8
By adding the 2 fractions we get the fraction of the third circle.
3/8 + 2/8 = 5/8

Question 2:
Go Math Grade 4 Answer Key Chapter 7 Add and Subtract Fractions Page 389 Question 2

Answer: 4/5 – 3/5 = 1/5

Explanation:
The fraction of the shaded part for the above rectangle is 4/5
The fraction of the box is 3/5
The equation for the above figure is 4/5 – 3/5 = 1/5

Question 3:
Go Math Grade 4 Answer Key Chapter 7 Add and Subtract Fractions Page 389 Question 3

Answer: 1/4 + 2/4 = 3/4

Explanation:
The name of the fraction for the shaded part of first figure is 1/4
The name of the fraction for the shaded part of second figure is 1/4
The name of the fraction for the shaded part of third figure is 3/4
So, The equation for the above figure is 1/4 + 2/4 = 3/4

Question 4:
Go Math Grade 4 Answer Key Chapter 7 Add and Subtract Fractions Page 389 Question 4

\(\frac { 2 }{ 6 } +\frac { 3 }{ 6 } =\frac { }{ } \)

Answer: \(\frac { 2 }{ 6 } +\frac { 3 }{ 6 } =\frac { 5 }{ 6 } \)

Explanation:
The name of the fraction for the shaded part of first figure is 2/6
The name of the fraction for the shaded part of second figure is 3/6
The name of the fraction for the shaded part of third figure is 5/6
So, The equation for the above figure is \(\frac { 2 }{ 6 } +\frac { 3 }{ 6 } =\frac { 5 }{ 6 } \)

Question 5:
Go Math Grade 4 Answer Key Chapter 7 Add and Subtract Fractions Page 389 Question 5

\(\frac { 3 }{ 5 } -\frac { 2 }{ 5 } =\frac { }{ } \)

Answer: \(\frac { 3 }{ 5 } -\frac { 2 }{ 5 } =\frac { 1 }{ 5 } \)

Explanation:
The name of the fraction for the shaded part of figure is 3/5
The name of the fraction for the shaded part of closed box is 2/5
So, The equation for the above figure is \(\frac { 3 }{ 5 } -\frac { 2 }{ 5 } =\frac { 1 }{ 5 } \)

Question 6:
Jake ate \(\frac { 4 }{ 8 } \) of a pizza. Millie ate \(\frac { 3}{ 8 } \) of the same pizza. How much of the pizza was eaten by Jake and Millie?

Answer: 7/8 of pizza

Explanation:
Given that,
Jake ate \(\frac { 4 }{ 8 } \) of a pizza.
Millie ate \(\frac { 3}{ 8 } \) of the same pizza.
To find how much of the pizza was eaten by Jake and Millie
We have to add both the fractions
\(\frac { 4 }{ 8 } \) + \(\frac { 3 }{ 8 } \) = \(\frac { 7 }{ 8 } \)
Thus the fraction of the pizza eaten by Jake and Millie is \(\frac { 7 }{ 8 } \)

Question 7:
Kate ate \(\frac { 1 }{ 4 } \) of her orange. Ben ate \(\frac { 2 }{ 4 } \) of his banana. Did Kate and Ben eat \(\frac { 1 }{ 4 } +\frac { 2}{ 4 } =\frac { 3}{ 4 } \) of their fruit?

Answer: No, one whole refers to orange and the other whole to a banana.

Add and Subtract Parts of a Whole Page No – 390

Question 1:
A whole pie is cut into 8 equal slices. Three of the slices are served. How much of the pie is left?
(a) \(\frac { 1 }{ 8 } \)
(b) \(\frac { 3 }{ 8 } \)
(c) \(\frac { 5 }{ 8} \)
(d)\(\frac { 7 }{ 8 } \)

Answer: \(\frac { 5 }{ 8} \)

Explanation:
Given,
A whole pie is cut into 8 equal slices. Three of the slices are served.
The fraction of 8 slices is 8/8.
Out of which 3/8 are served.
8/8 – 3/8 = 5/8
Therefore \(\frac { 5 }{ 8} \) of the pie is left.
Thus the correct answer is option c.

Question 2:
An orange is divided into 6 equal wedges. Jody eats 1 wedge. Then she eats 3 more wedges. How much of the orange did Jody eat?
(a) \(\frac { 1 }{ 6} \)
(b) \(\frac { 4}{ 6 } \)
(c) \(\frac { 5}{ 6 } \)
(d) \(\frac { 6}{ 6} \)

Answer: \(\frac { 4}{ 6 } \)

Explanation:
Given,
An orange is divided into 6 equal wedges.
Jody eats 1 wedge.
Then she eats 3 more wedges.
The fraction of orange that Jody eat is \(\frac { 4}{ 6 } \).
Thus the correct answer is option b.

Question 3:
Which list of distances is in order from least to greatest?
(a) \(\frac { 1 }{ 8 } \) Mile, \(\frac { 3 }{ 16 } \) Mile, \(\frac { 3 }{ 4 } \) Mile
(b) \(\frac { 3 }{ 4 } \) Mile, \(\frac { 1 }{ 8 } \) Mile, \(\frac { 3 }{ 16 } \) Mile
(c) \(\frac { 1 }{ 8} \) Mile, \(\frac { 3 }{ 4 } \) Mile, \(\frac { 3 }{ 16 } \) Mile
(d)\(\frac { 3 }{ 16 } \) Mile, \(\frac { 1 }{ 8 } \) Mile, \(\frac { 3 }{ 4 } \) Mile

Answer: \(\frac { 1 }{ 8 } \) Mile, \(\frac { 3 }{ 16 } \) Mile, \(\frac { 3 }{ 4 } \) Mile

Explantion:
Compare the three fractions 1/8, 3/4 and 3/16
Make the common denominators.
1/8 × 2/2 = 2/16
3/4 × 4/4 = 12/16
The fractions are 2/16, 12/16 and 3/16
The numerator with the highest number will be the greatest.
The fractions from least to greatest is \(\frac { 1 }{ 8 } \) Mile, \(\frac { 3 }{ 16 } \) Mile, \(\frac { 3 }{ 4 } \) Mile.
Thus the correct answer is option d.

Question 4:
Jeremy walked 6/8 of the way to school and ran the rest of the way. What fraction, in simplest form, shows the part of the way that Jeremy walked?
(a) \(\frac { 1 }{ 4 } \)
(b) \(\frac { 3 }{ 8 } \)
(c) \(\frac { 1 }{ 2} \)
(d)\(\frac { 3 }{ 4 } \)

Answer: \(\frac { 3 }{ 4 } \)

Explanation:
Given,
Jeremy walked 6/8 of the way to school and ran the rest of the way.
The simplest form of 6/8 is 3/8.
The simplest form of part of the way that Jeremy walked is 3/8.
Thus the correct answer is option b.

Question 5:
An elevator starts on the 100th floor of a building. It descends 4 floors every 10 seconds. At what floor will the elevator be 60 seconds after it starts?
(a) 60th floor
(b) 66th floor
(c) 72nd floor
(d) 76th floor

Answer: 76th floor

Explanation:
Given,
An elevator starts on the 100th floor of a building.
It descends 4 floors every 10 seconds.
4 floors – 10 seconds
? – 60 seconds
60 × 4/10 = 240/10 = 24 floors
100 – 24 = 76th floor
Thus the correct answer is option d.

Question 6:
For a school play, the teacher asked the class to set up chairs in 20 rows with 25 chairs in each row. After setting up all the chairs, they were 5 chairs short. How many chairs did the class set up?
(a) 400
(b) 450
(c) 495
(d) 500

Answer: 495

Explanation:
Given,
For a school play, the teacher asked the class to set up chairs in 20 rows with 25 chairs in each row.
After setting up all the chairs, they were 5 chairs short.
20 × 25 = 500
500 – 5 = 495
Therefore the class set up 495 chairs.
Thus the correct answer is c.

Add and Subtract Parts of a Whole Page No – 393

Question 1:
Write \(\frac { 3 }{ 4 }\) as a sum of unit fractions.
Go Math Grade 4 Answer Key Chapter 7 Add and Subtract Fractions Page 393 Question 1
\(\frac { 3 }{ 4 } = \)

Answer:
The sum of the unit fraction for 3/4 is 1/4 + 1/4 + 1/4

Explanation:
A unit fraction is a rational number written as a fraction where the numerator is one and the denominator is a positive integer. The sum of the unit fraction for 3/4 is 1/4 + 1/4 + 1/4.

Write the fraction as a sum of unit fractions.
Question 2:
Go Math Grade 4 Answer Key Chapter 7 Add and Subtract Fractions Page 393 Question 2
\(\frac { 5 }{ 6 } = \)

Answer:
The sum of the unit fraction for 5/6 is 1/6 + 1/6 + 1/6 + 1/6 + 1/6

Explanation:
A unit fraction is a rational number written as a fraction where the numerator is one and the denominator is a positive integer. The sum of the unit fraction for 5/6 is 1/6 + 1/6 + 1/6 + 1/6 + 1/6

Question 3:
Go Math Grade 4 Answer Key Chapter 7 Add and Subtract Fractions Page 393 Question 3
\(\frac { 2 }{ 3 } = \)

Answer:
The sum of the unit fraction for 2/3 is 1/3 + 1/3.

Explanation:
A unit fraction is a rational number written as a fraction where the numerator is one and the denominator is a positive integer. The sum of the unit fraction for 2/3 is 1/3 + 1/3.

Question 4:
\(\frac { 4 }{ 12 } = \)

Answer:
The sum of the unit fraction for 4/12 is 1/12 + 1/12 + 1/12 + 1/12

Explanation:
A unit fraction is a rational number written as a fraction where the numerator is one and the denominator is a positive integer. The sum of the unit fraction for 4/12 is 1/12 + 1/12 + 1/12 + 1/12

Question 5:
\(\frac { 6 }{ 8 } = \)

Answer:
The sum of the unit fraction for 6/8 is 1/8 + 1/8 + 1/8 + 1/8 + 1/8 + 1/8

Explanation:
A unit fraction is a rational number written as a fraction where the numerator is one and the denominator is a positive integer. The sum of the unit fraction for 6/8 is 1/8 + 1/8 + 1/8 + 1/8 + 1/8 + 1/8

Question 6:
\(\frac { 8 }{ 10 } = \)

Answer:
The sum of the unit fraction for 8/10 is 1/10 + 1/10 + 1/10 + 1/10 + 1/10 + 1/10 + 1/10 + 1/10

Explanation:
A unit fraction is a rational number written as a fraction where the numerator is one and the denominator is a positive integer. The sum of the unit fraction for 8/10 is 1/10 + 1/10 + 1/10 + 1/10 + 1/10 + 1/10 + 1/10 + 1/10

Question 7:
\(\frac { 6 }{ 6 } = \)

Answer:
The sum of the unit fraction for 6/6 is 1/6 + 1/6 + 1/6 + 1/6 + 1/6 + 1/6

Explanation:
A unit fraction is a rational number written as a fraction where the numerator is one and the denominator is a positive integer. The sum of the unit fraction for 6/6 is 1/6 + 1/6 + 1/6 + 1/6 + 1/6 + 1/6

Question 8:
Compare Representations How many different ways can you write a fraction that has a numerator of 2 as a sum of fractions? Explain.

Answer:
Let’s say we have the fraction 2/9.
We can split this one fraction into two by modifying the numerator, like so: 2/9 = 1/9 + 1/9
This works because since both fractions have a numerator of 9, you can easily add the numerators to give 2, and that will give 2/9 in return. However, you can’t separate the denominators.
2/9 is not equal to 2/6 + 2/3
2/9 = 1/9 + 1/9
2/9 = 0.5/9 + 1.5/9 (which simplifies to 1/18 + 3/18, also giving 2/9)
2/9 = 0.5/9 + 0.5/9 + 0.5/9 + 0.5/9 = 1/18 + 1/18 + 1/18 + 1/18
I basically split it up into more and more fractions that add up to give 2/9. So, in short, there are infinitely many ways to do it.

Add and Subtract Parts of a Whole Page No – 394

Question 9:
Holly’s garden is divided into 5 equal sections. She will fence the garden into 3 areas by grouping some equal sections together. What part of the garden could each fenced area be?
Go Math Grade 4 Answer Key Chapter 7 Add and Subtract Fractions Page 394 Question 9
a. What information do you need to use?

Answer:
We need the information about the equal sections and fence the garden into 3 areas by grouping some equal sections together.

b. How can writing an equation help you solve the problem?

Answer: The equation helps to find what part of the garden could each fenced area be.

Explanation:
If you write an equation with 3 addends whose sum is 5/5, you could find the possible sizes of each fenced area. The size of each section is 1/5. Each addend represents the size of a fenced area.

c. How can drawing a model help you write an equation?

Answer: If you draw a model that shows 5 fifth-size parts representing the sections, you can see how to group the parts into 3 areas in different ways.

d. Show how you can solve the problem.

Answer:
Go Math Grade 4 Answer Key Chapter 7 img_1

Question 9:
Complete the sentence.
The garden can be fenced into ______, ______, and ______ parts or ______, ______, and ______ parts.

Answer: 3/5, 1/5 and 1/5 parts or 2/5, 2/5 and 1/5 parts

Add and Subtract Parts of a Whole Page No – 395

Question 1:
Go Math Grade 4 Answer Key Chapter 7 Add and Subtract Fractions Page 394 Question 1
Answer: 1/5 + 1/5 + 1/5 + 1/5

Explanation:
The sum of the unit fractions for 4/5 is 1/5 + 1/5 + 1/5 + 1/5.

Question 2:
\(\frac { 3 }{ 8 }= \)

Answer: 1/8 + 1/8 + 1/8

Explanation:
The sum of the unit fractions for 3/8 is 1/8 + 1/8 + 1/8

Question 3:
\(\frac { 6 }{ 12 }= \)

Answer: 1/12 + 1/12 + 1/12 + 1/12 + 1/12 + 1/12

Explanation:
The sum of the unit fractions for 6/12 is 1/12 + 1/12 + 1/12 + 1/12 + 1/12 + 1/12

Question 4:
\(\frac { 4 }{ 4 }= \)

Answer: 1/4 + 1/4 + 1/4 + 1/4

Explanation:
The sum of the unit fractions for 4/4 is 1/4 + 1/4 + 1/4 + 1/4

Question 5:
\(\frac { 7 }{ 10 }= \)

Answer: 1/10 + 1/10 + 1/10 + 1/10 + 1/10 + 1/10 + 1/10

Explanation:
The sum of the unit fractions for 7/10 is 1/10 + 1/10 + 1/10 + 1/10 + 1/10 + 1/10 + 1/10

Question 6:
\(\frac { 6 }{ 6 } =\)

Answer: 1/6 + 1/6 + 1/6 + 1/6 + 1/6 + 1/6

Explanation:
The sum of the unit fractions for 6/6 is 1/6 + 1/6 + 1/6 + 1/6 + 1/6 + 1/6

Question 7:
Miguel’s teacher asks him to color \(\frac { 4 }{ 8 }\) of his grid. He must use 3 colors: red, blue, and green. There must be more green sections than red sections. How can Miguel color the sections of his grid to follow all the rules?

Answer: 1/8 red, 1/8 blue, and 2/8 green

Explanation:
If there are 8 tiles, coloring \(\frac { 4 }{ 8 }\) means coloring 4 tiles. Using those three colors, we could use each 1 time with 1 leftover. Since we must have more green, we would use it twice; this would give us 2 green, 1 red and 1 blue.
Since the grid is not necessarily 8 squares, we must account for this by saying 2/8 green, 1/8 red, and 1/8 blue.

Question 8:
Petra is asked to color \(\frac { 6 }{ 6 }\) of her grid. She must use 3 colors: blue, red, and pink. There must be more blue sections than red sections or pink sections. What are the different ways Petra can color the sections of her grid and follow all the rules?

Answer: 3/6 blue, 2/6 red, 1/6 pink

Explanation:
1. 3 blues, 2 red, 1 pink.
2. 3 blues, 2 pink, 1 red.
3. 4 blues, 1 red, 1 pink
The different ways in which Petra can color the sections of her grid and follow the rules are;
1. 3 blues, 2 red, 1 pink.
2. 3 blues, 2 pink, 1 red.
3. 4 blues, 1 red, 1 pink
All these three ways follows the rules that; there must be three colors an also Blue sections are more than red sections or pink sections.

Add and Subtract Parts of a Whole Page No – 396

Question 1:
Jorge wants to write \(\frac { 4 }{ 5 } \) as a sum of unit fractions. Which of the following should he write?
(a) \(\frac { 3 }{ 5 } +\frac { 1 }{ 5 } \)
(b) \(\frac { 2 }{ 5 } +\frac { 2 }{ 5 } \)
(c) \(\frac { 1 }{ 5 } +\frac { 1 }{ 5 }+\frac { 2 }{ 5 } \)
(d) \(\frac { 1 }{ 5 } +\frac { 1 }{ 5 } +\frac { 1 }{ 5 } +\frac { 1 }{ 5 } \)

Answer: \(\frac { 1 }{ 5 } +\frac { 1 }{ 5 } +\frac { 1 }{ 5 } +\frac { 1 }{ 5 } \)

Explanation:
Given,
Jorge wants to write \(\frac { 4 }{ 5 } \) as a sum of unit fractions.
The sum of the unit fraction for \(\frac { 4 }{ 5 } \) is \(\frac { 1 }{ 5 } +\frac { 1 }{ 5 } +\frac { 1 }{ 5 } +\frac { 1 }{ 5 } \)
Thus the correct answer is option d.

Question 2:
Which expression is equivalent to \(\frac { 7 }{ 8 } \) ?
(a) \(\frac { 5 }{ 8 } +\frac { 2 }{ 8}+\frac { 1 }{ 8 } \)
(b) \(\frac { 3 }{ 8 } +\frac {3 }{ 8 } +\frac { 1 }{ 8 } +\frac { 1 }{ 8 } \)
(c) \(\frac { 4 }{ 8 } +\frac { 2 }{ 8 }+\frac { 1 }{ 8 } \)
(d) \(\frac { 4 }{ 8 } +\frac { 2 }{ 8 }+\frac { 2 }{ 8 } \)

Answer: \(\frac { 4 }{ 8 } +\frac { 2 }{ 8 }+\frac { 1 }{ 8 } \)

Explanation:
The fraction equivalent to \(\frac { 7 }{ 8 } \) is \(\frac { 4 }{ 8 } +\frac { 2 }{ 8 }+\frac { 1 }{ 8 } \).
Thus the correct answer is option c.

Question 3:
An apple is cut into 6 equal slices. Nancy eats 2 of the slices. What fraction of the apple is left?
(a) \(\frac { 1 }{ 6 } \)
(b) \(\frac { 2 }{ 6 } \)
(c) \(\frac { 3 }{ 6 } \)
(d) \(\frac { 4 }{ 6 } \)

Answer: \(\frac { 4 }{ 6 } \)

Explanation:
Given,
An apple is cut into 6 equal slices. Nancy eats 2 of the slices.
6 – 2 = 4
\(\frac { 6 }{ 6 } \) – \(\frac { 2 }{ 6 } \) = \(\frac { 4 }{ 6 } \)
Thus the correct answer is option d.

Question 4:
Which of the following numbers is a prime number?
(a) 1
(b) 11
(c) 21
(d) 51

Answer: 11

Explanation:
A prime number is a natural number greater than 1 that is not a product of two smaller natural numbers.
11 is a multiple of 1 and itself.
Thus the correct answer is option b.

Question 5:
A teacher has a bag of 100 unit cubes. She gives an equal number of cubes to each of the 7 groups in her class. She gives each group as many cubes as she can. How many unit cubes are left over?
(a) 1
(b) 2
(c) 3
(d) 6

Answer: 2

Explanation:
Given,
A teacher has a bag of 100 unit cubes. She gives an equal number of cubes to each of the 7 groups in her class.
She gives each group as many cubes as she can.
100 divided by 7 is 14 r 2, so there are 2 leftover.
Thus the correct answer is option b.

Question 6:
Jessie sorted the coins in her bank. She made 7 stacks of 6 dimes and 8 stacks of 5 nickels. She then found 1 dime and 1 nickel. How many dimes and nickels does Jessie have in all?
(a) 84
(b) 82
(c) 80
(d) 28

Answer: 84

Explanation:
Given,
Jessie sorted the coins in her bank. She made 7 stacks of 6 dimes and 8 stacks of 5 nickels.
She then found 1 dime and 1 nickel.
43 dimes and 41 nickles
43 + 41 = 84
Jessie has 84 dimes and nickels in all.
Thus the correct answer is option a.

Add and Subtract Parts of a Whole Page No – 399

Question 1:
Adrian’s cat ate \(\frac { 3 }{ 5 } \) of a bag of cat treats in September and \(\frac { 1 }{ 5 } \) of the same bag of cat treats in October. What part of the bag of cat treats did Adrian’s cat eat in both months? Use the model to find the sum \(\frac { 3 }{ 5 } \)+\(\frac { 1 }{ 5 } \). How many fifth-size pieces are shown?
Go Math Grade 4 Answer Key Chapter 7 Add and Subtract Fractions Page 399 Question 1
Use the model to find the sum \(\frac { 3 }{ 5 } \)+\(\frac { 1 }{ 5 } \). How many fifth-size pieces are shown? fifth-size pieces

Answer: 4/5

Explanation:
Given,
Adrian’s cat ate \(\frac { 3 }{ 5 } \) of a bag of cat treats in September and \(\frac { 1 }{ 5 } \) of the same bag of cat treats in October.
From the above figure, we can see that there are 4 fifth size pieces.
\(\frac { 3 }{ 5 } \)+\(\frac { 1 }{ 5 } \) = \(\frac { 4 }{ 5 } \).

Use the model to find the sum.
Question 2:
Go Math Grade 4 Answer Key Chapter 7 Add and Subtract Fractions Page 399 Question 2
\(\frac { 1 }{ 4 } +\frac { 2 }{ 4 } =\frac { }{ } \)

Answer: 3/4

Explanation:
From the above figure, we can see that there are 3 one-fourth shaded parts.
So, \(\frac { 1 }{ 4 } +\frac { 2 }{ 4 } =\frac { 3 }{ 4 } \)

Question 3:
Go Math Grade 4 Answer Key Chapter 7 Add and Subtract Fractions Page 399 Question 3
\(\frac { 6 }{ 10 } +\frac { 3 }{ 10 } =\frac { }{ } \)

Answer: 9/10

Explanation:
From the above figure, we can see that there are 9 one-tenth shaded parts.
So, \(\frac { 6 }{ 10 } +\frac { 3 }{ 10 } =\frac { 9 }{ 10 } \).

Find the sum. Use models to help.
Question 4:
\(\frac { 3 }{ 6 } +\frac { 3 }{ 6 } =\frac { }{ } \)

Answer: 6/6 = 1

Explanation:
3/6 and 3/6 has same numerators and same denominators so we have to add both the fractions.
\(\frac { 3 }{ 6 } +\frac { 3 }{ 6 } =\frac { 6 }{ 6 } \)
6/6 = 1

Question 5:
\(\frac { 1 }{ 3 } +\frac { 1 }{ 3 } =\frac { }{ } \)

Answer: 2/3

Explanation:
1/3 and 1/3 has same numerators and same denominators so we have to add both the fractions.
\(\frac { 1 }{ 3 } +\frac { 1 }{ 3 } =\frac { 2 }{ 3 } \)

Question 6:
\(\frac { 5 }{ 8 } +\frac { 2 }{ 8 } =\frac { }{ } \)

Answer: 7/8

Explanation:
Given the expressions 5/8 and 2/8.
The above fractions have the same denominators but the numerators are different.
So, \(\frac { 5 }{ 8 } +\frac { 2 }{ 8 } =\frac { 7 }{ 8 } \)

Find the sum. Use models or iTools to help.
Question 7:
\(\frac { 5 }{ 8 } +\frac { 2 }{ 8 } =\frac { }{ } \)
Answer: 7/8

Explanation:
Given the expressions 5/8 and 2/8.
The above fractions have the same denominators but the numerators are different.
So, \(\frac { 5 }{ 8 } +\frac { 2 }{ 8 } =\frac { 7 }{ 8 } \)

Question 8:
\(\frac { 2 }{ 5 } +\frac { 2 }{ 5 } =\frac { }{ } \)
Answer: 4/5

Explanation:
2/5 and 2/5 have the same numerators and same denominators so we have to add both the fractions.
\(\frac { 2 }{ 5 } +\frac { 2 }{ 5 } =\frac { 4 }{ 5 } \)

Question 9:
\(\frac { 4 }{ 6 } +\frac { 1 }{ 6 } =\frac { }{ } \)
Answer: 5/6

Explanation:
Given the fractions 4/6 and 1/6.
The above fractions have the same denominators but the numerators are different.
\(\frac { 4 }{ 6 } +\frac { 1 }{ 6 } =\frac { 5 }{ 6 } \)

Question 10:
Jason is making a fruit drink. He mixes \(\frac { 2 }{ 8 } \) quart of grape juice with \(\frac { 3 }{ 8 } \) quart of apple juice. Then he adds \(\frac { 1 }{ 8 } \) quart of lemonade. How much fruit drink does Jason make?
\(\frac { }{ } \) quart.
Answer: \(\frac { 6 }{ 8 } \) quart.

Explanation:
Given that,
Jason is making a fruit drink. He mixes \(\frac { 2 }{ 8 } \) quart of grape juice with \(\frac { 3 }{ 8 } \) quart of apple juice.
Then he adds \(\frac { 1 }{ 8 } \) quart of lemonade
Add all the three fractions to how much fruit drink Jason makes.
2/8 + 3/8 + 1/8 = \(\frac { 6 }{ 8 } \) quart.

Question 11:
A sum has five addends. Each addend is a unit fraction. The sum is 1. What are the addends?

Answer: 1/5

Explanation:
Given that,
A sum has five addends. Each addend is a unit fraction. The sum is 1.
1/5 + 1/5 + 1/5 + 1/5 + 1/5 = 5/5 = 1
Thus the addend is 1/5.

Question 12:
In a survey, \(\frac { 4 }{ 12 } \) of the students chose Friday and \(\frac { 5 }{ 12 } \) chose Saturday as their favorite day of the week. What fraction shows the students who chose Friday or Saturday as their favorite day? Shade the model to show your answer.
Go Math Grade 4 Answer Key Chapter 7 Add and Subtract Fractions Page 399 Question 12
\(\frac { }{ } \)
Answer: \(\frac { 9 }{ 12 } \)

Explanation:
Given that,
In a survey, \(\frac { 4 }{ 12 } \) of the students chose Friday and \(\frac { 5 }{ 12 } \) chose Saturday as their favorite day of the week.
Add both the fractions 4/12 and 5/12
\(\frac { 4 }{ 12 } \) + \(\frac { 5 }{ 12 } \) = \(\frac { 9 }{ 12 } \)

Add and Subtract Parts of a Whole Page No – 400

Question 13:
Model Mathematics Jin is putting colored sand in a jar. She filled \(\frac {2 }{ 10} \) of the jar with blue sand and \(\frac { 4}{ 10} \) of the jar with pink sand. Describe one way to model the part of the jar filled with sand.
Go Math Grade 4 Answer Key Chapter 7 Add and Subtract Fractions Page 400 Question 13

Answer: \(\frac { 4}{ 10} \)

Explanation:
the answer is 4/10 because 4/10 + 2/10= 6/10+ 4/10 = 10/10. a bit confusing
4 + 2 = 6 right the, 6 + 4 = 10 so 10/10.

Have you ever seen a stained glass window in a building or home? Artists have been designing stained glass windows for hundreds of years.

Go Math Grade 4 Answer Key Chapter 7 Add and Subtract Fractions Page 400 Question 13 - i

Help design the stained glass sail on the boat below.

Materials • color pencils

Look at the eight triangles in the sail. Use the guide below to color the triangles:

Go Math Grade 4 Answer Key Chapter 7 Add and Subtract Fractions Page 400 Question 13 - ii

  • \(\frac {2 }{8 } \) blue
  • \(\frac {3 }{8 } \) red
  • \(\frac { 2}{ 8} \) orange
  • \(\frac {1 }{8 } \) yellow

Question 14:
Write an Equation Write an equation that shows the fraction of triangles that are red or blue.
Answer: \(\frac {3 }{8 } \) red

Question 15:
What color is the greatest part of the sail? Write a fraction for that color. How do you know that fraction is greater than the other fractions? Explain.
Answer: Red

Explanation:
Among all the colors Red color has the greatest part of the sail.

Add Fractions Using Models – Page No 401

Find the sum. Use fraction strips to help.

Question 1:
Go Math Grade 4 Answer Key Chapter 7 Add and Subtract Fractions Page 401 Question 1

Answer: 3/6

Question 2:
\(\frac { 4 }{ 10 } +\frac { 5 }{ 10 } =\frac { }{ } \)

Answer: 9/10
HMH Go Math Grade 4 Answer Key Chapter 7 Add and Subtract Fractions Img_6

Question 3:
\(\frac { 1 }{ 3 } +\frac { 2 }{ 3 } =\frac { }{ } \)

Answer: 3/3
HMH Go Math Grade 4 Answer Key Chapter Add & Subtract Fractions Img_7

Question 4:
\(\frac { 2 }{ 4 } +\frac { 1 }{ 4 } =\frac { }{ } \)

Answer: 3/4
HMH Go Math Grade 4 Key Chapter 7 Add and Subtract Fractions Img_8

Question 5:
\(\frac { 2 }{ 12 } +\frac { 4 }{ 12 } =\frac { }{ } \)

Answer: 6/12
HMH Go Math Grade 4 Key Chapter 7 Add & Subtract Fractions Img_9

Question 6:
\(\frac { 1 }{ 6 } +\frac { 3 }{ 6 } =\frac { }{ } \)

Answer: 3/6
Go Math Grade 4 Key Chapter 7 Add & Subtract Fractions Img_10

Question 7:
\(\frac { 3 }{ 12 } +\frac { 9 }{ 12 } =\frac { }{ } \)

Answer: 12/12

Go Math Grade 4 Answer Key Chapter 7 Add & Subtract Fractions Img_11

Question 8:
\(\frac { 3 }{ 8 } +\frac { 4 }{ 8 } =\frac { }{ } \)

Answer: 7/8

Go Math 4th Grade Key Chapter 7 Add & Subtract Fractions Img_12

Question 9:
\(\frac { 3 }{ 4 } +\frac { 1 }{ 4 } =\frac { }{ } \)

Answer: 4/4
Go Math 4th Grade Answer Key Chapter 7 Add & Subtract Fractions Img_13

Question 9:
\(\frac { 1 }{ 5 } +\frac { 2 }{ 5 } =\frac { }{ } \)

Answer: 3/5

Explanation:
Go Math Grade 4 Answer Key Chapter Img_14

Question 10:
\(\frac { 6 }{ 10 } +\frac { 3 }{ 10 } =\frac { }{ } \)

Answer: 9/10
Go Math Grade 4 Answer Key Chapter 7 Img_15

Question 11:
Lola walks \(\frac { 4 }{ 10} \) mile to her friend’s house. Then she walks \(\frac { 5 }{ 10 } \) mile to the store. How far does she walk in all?

Answer: \(\frac { 9 }{ 10 } \) mile

Explanation:
Given,
Lola walks \(\frac { 4 }{ 10} \) mile to her friend’s house.
Then she walks \(\frac { 5 }{ 10 } \) mile to the store.
\(\frac { 4 }{ 10} \) + \(\frac { 5 }{ 10 } \) = \(\frac { 9 }{ 10 } \)
Therefore she walked \(\frac { 9 }{ 10 } \) mile in all.

Question 12:
Evan eats \(\frac { 1 }{ 8 } \) of a pan of lasagna and his brother eats \(\frac { 2 }{ 8 } \) of it. What fraction of the pan of lasagna do they eat in all?
Answer: \(\frac { 3 }{ 8 } \) of the pan

Explanation:
Given,
Evan eats \(\frac { 1 }{ 8 } \) of a pan of lasagna and his brother eats \(\frac { 2 }{ 8 } \) of it.
\(\frac { 1 }{ 8 } \) + \(\frac { 2 }{ 8 } \)
= \(\frac { 3 }{ 8 } \)

Question 13:
Jacqueline buys \(\frac { 2 }{ 4 } \) yard of green ribbon and \(\frac { 1 }{ 4 } \) yard of pink ribbon. How many yards of ribbon does she buy in all?

Answer: \(\frac { 3 }{ 4 } \) yard

Explanation:
Given,
Jacqueline buys \(\frac { 2 }{ 4 } \) yard of green ribbon and \(\frac { 1 }{ 4 } \) yard of pink ribbon.
\(\frac { 2 }{ 4 } \) + \(\frac { 1 }{ 4 } \)
= \(\frac { 3 }{ 4 } \)
Thus Jacqueline bought \(\frac { 3 }{ 4 } \) yards of ribbon in all.

Question 14:
Shu mixes \(\frac { 2 }{ 3 } \) pound of peanuts with \(\frac { 1 }{ 3 } \) pound of almonds. How many pounds of nuts does Shu mix in all?

Answer: 3/3 pound

Explanation:
Given,
Shu mixes \(\frac { 2 }{ 3 } \) pound of peanuts with \(\frac { 1 }{ 3 } \) pound of almonds.
\(\frac { 2 }{ 3 } \) + \(\frac { 1 }{ 3 } \)
= \(\frac { 3 }{ 3 } \)
Therefore Shu mix \(\frac { 3 }{ 3 } \) pounds of nuts in all.

Add Fractions Using Models – Lesson Check – Page No 402

Question 1:
Mary Jane has \(\frac { 3 }{ 8 } \) of a medium pizza left. Hector has \(\frac { 2 }{ 8 } \) of another medium pizza left. How much pizza do they have altogether?

(a) \(\frac { 1 }{ 8 } \)
(b) \(\frac { 4 }{ 8 } \)
(c) \(\frac { 5 }{ 8 } \)
(d) \(\frac { 6 }{ 8 } \)

Answer: \(\frac { 5 }{ 8 } \)

Explanation:
Given,
Mary Jane has \(\frac { 3 }{ 8 } \) of a medium pizza left.
Hector has \(\frac { 2 }{ 8 } \) of another medium pizza left.
To find how much pizza do they have altogether we have to add both the fractions.
\(\frac { 3 }{ 8 } \) + \(\frac { 2 }{ 8 } \) = \(\frac { 5 }{ 8 } \)
Therefore Mary Jane and Hector has \(\frac { 5 }{ 8 } \) pizza altogether.
Thus the correct answer is option c.

Question 2:
Jeannie ate \(\frac { 1 }{ 4 } \) of an apple. Kelly ate \(\frac { 2 }{ 4 } \) of the apple. How much did they eat in all?

(a) \(\frac { 1 }{ 8 } \)
(b) \(\frac { 2 }{ 8 } \)
(c) \(\frac { 3 }{ 8 } \)
(d) \(\frac { 3 }{ 4 } \)

Answer: \(\frac { 3 }{ 4 } \)

Explanation:
Given,
Jeannie ate \(\frac { 1 }{ 4 } \) of an apple.
Kelly ate \(\frac { 2 }{ 4 } \) of the apple.
\(\frac { 1 }{ 4 } \) + \(\frac { 2 }{ 4 } \) = \(\frac { 3 }{ 4 } \)
Thus the correct answer is option d.

Question 3:
Karen is making 14 different kinds of greeting cards. She is making 12 of each kind. How many greeting cards is she making?

(a) 120
(b) 132
(c) 156
(d) 168

Answer: 168

Explanation:
Given,
Karen is making 14 different kinds of greeting cards.
She is making 12 of each kind.
To find how many greeting cards she is making we have to multiply 14 and 12.
14 × 12 = 168.
Thus the correct answer is option d.

Question 4:
Jefferson works part-time and earns $1,520 in four weeks. How much does he earn each week?

(a) $305
(b) $350
(c) $380
(d) $385

Answer: $380

Explanation:
Jefferson works part-time and earns $1,520 in four weeks.
1520 – 4 weeks
? – 1 week
1520/4 = $380
Thus the correct answer is option c.

Question 5:
By installing efficient water fixtures, the average American can reduce water use to about 45 gallons of water per day. Using such water fixtures, about how many gallons of water would the average American use in December?

(a) about 1,200 gallons
(b) about 1,500 gallons
(c) about 1,600 gallons
(d) about 2,000 gallons

Answer: about 1,500 gallons

Explanation:
Given,
By installing efficient water fixtures, the average American can reduce water use to about 45 gallons of water per day.
1 day – 45 gallons
31 days – ?
45 × 31 = 1395 gallons
The number near to 1395 is 1500 gallons.
Thus the correct answer is option b.

Question 6:
Collin is making a bulletin board and note center. He is using square cork tiles and square dry-erase tiles. One of every 3 squares will be a cork square. If he uses 12 squares for the center, how many will be cork squares?

(a) 3
(b) 4
(c) 6
(d) 8

Answer: 4

Explanation:
Given that,
Collin is making a bulletin board and note center.
He is using square cork tiles and square dry-erase tiles.
One of every 3 squares will be a cork square.
12/3 = 4
Thus the correct answer is option b.

Add Fractions Using Models – Lesson Check – Page No 405

Question 1:
Lisa needs 4/5 pound of shrimp to make shrimp salad. She has 1/5 pound of shrimp. How much more shrimp does Lisa need to make the salad?
Add Fractions Using Models - Lesson Check - Page No 405 Q1
Subtract \(\frac { 4 }{ 5 } – \frac { 1 }{ 5 }\). Use the model to help.
Shade the model to show how much shrimp Lisa needs.
Then shade the model to show how much shrimp Lisa has.
Compare the difference between the two shaded rows.
\(\frac { 4 }{ 5 } – \frac { 1 }{ 5 } = \frac {■ }{ 5} \)
Lisa needs _____ pound more shrimp.

Answer: 3/5

Explanation:
Given that,
Lisa needs 4/5 pounds of shrimp to make shrimp salad. She has 1/5 pound of shrimp.
The denominators have the same numbers and numerators have different numbers.
4/5 – 3/5 = 1/5
Thus Lisa needs 1/5 pounds more shrimp.

Use the model to find the difference.

Question 2:
\(\frac { 3 }{ 6 } – \frac { 2 }{ 6 } = \frac {■ }{ 6} \)
Add Fractions Using Models - Lesson Check - Page No 405 Q2

Answer: 1/6

Explanation:
Given two fractions 3/6 and 2/6
Denominators are same but the numerators are different.
3/6 – 2/6 = 1/6

Question 3:
\(\frac { 8 }{ 10 } – \frac { 5 }{ 10 } = \frac {■ }{ 10} \)
Add Fractions Using Models - Lesson Check - Page No 405 Q 3

Answer: 3/10

Explanation:
Given two fractions 8/10 and 5/10
Denominators are the same but the numerators are different.
8/10 – 5/10 = 3/10

Subtract. Use models to help.

Question 4:
\(\frac { 5 }{ 8 } – \frac { 2 }{ 8 } = \frac { }{ } \)

Answer: 3/8

Explanation:
Given two fractions 5/8 and 2/8
Denominators are same but the numerators are different.
\(\frac { 5 }{ 8 } – \frac { 2 }{ 8 } = \frac { 3 }{ 8 } \)
Go Math Grade 4 Answer Key Chapter 7 Add and Subtract Fractions img_1

Question 5:
\(\frac { 7 }{ 12 } – \frac { 2 }{ 12 } = \frac { }{ } \)

Answer: 5/12

Explanation:
Given two fractions 7/12 and 2/12
Denominators are same but the numerators are different.
\(\frac { 7 }{ 12 } – \frac { 2 }{ 12 } = \frac { 5 }{ 12 } \)
Go Math Grade 4 Answer Key Chapter 7 Add and Subtract Fractions img_2

Question 6:
\(\frac { 3 }{4 } – \frac { 2 }{ 4 } = \frac { }{ } \)

Answer: 1/4

Explanation:
Given two fractions 3/4 and 2/4
Denominators are same but the numerators are different.
\(\frac { 3 }{4 } – \frac { 2 }{ 4 } = \frac { 1 }{ 4 } \)
Go Math Grade 4 Answer Key Chapter 7 Add & Subtract Fractions img_3

Question 7:
\(\frac { 2 }{ 3 } – \frac { 1 }{ 3 } = \frac { }{ } \)

Answer: 1/3

Go Math Grade 4 Answer Key Chapter 7 Add & Subtract Fractions img_4

Question 8:
\(\frac { 7 }{ 8 } – \frac { 5 }{ 8 } = \frac { }{ } \)

Answer: 2/8

Go Math Grade 4 Answer Key Chapter 7 Add and Subtract Fractions img_5

Question 9:
Explain how you could find the unknown addend in \(\frac { 2 }{ 6 } \) + _____ = 1 without using a model.
Answer: 4/6

Explanation:
1 can be written in the fraction form as 6/6
2/6 + x = 6/6
x = 6/6 – 2/6
x = 4/6

Add Fractions Using Models – Lesson Check – Page No 406

Question 10:
Mrs. Ruiz served a pie for dessert two nights in a row. The drawings below show the pie after her family ate dessert on each night. What fraction of the pie did they eat on the second night?
Go Math Grade 4 Answer Key Chapter 7 Add Fractions Using Models - Lesson Check - Page No 406 Q 10

\( \frac { }{ } \)

Answer: 10/12

a. What do you need to know?

Answer: We need to find the fraction of the pie did they eat on the second night.

b. How can you find the number of pieces eaten on the second night?

Answer: We can find the number of pieces eaten on the second night by dividing the number of eaten pieces by the total number of pieces.

c. Explain the steps you used to solve the problem.
Complete the sentences.
After the first night, _______ pieces were left.
After the second night, _______ pieces were left.
So, _______ of the pie was eaten on the second night.

Answer:
After the first night, 9 pieces were left.
After the second night, 2 pieces were left.
So, 10 of the pie was eaten on the second night.

Question 11:
Make Connection Between Models Judi ate \(\frac { 7}{8} \) of a small pizza and Jack ate \(\frac { 2}{ 8 } \) of a second small pizza. How much more of a pizza did Judi eat?
\(\frac { }{ } \)
Answer: \(\frac {5}{8} \)

Explanation:
Given,
Make Connection Between Models Judi ate \(\frac { 7}{8} \) of a small pizza and Jack ate \(\frac { 2}{ 8 } \) of a second small pizza.
\(\frac {7}{8} \) – \(\frac {2}{8} \) = \(\frac {5}{8} \)
Therefore Judi eat \(\frac {5}{8} \) of a pizza.

Question 12:
Keiko sewed \(\frac { 3}{4} \) yard of lace on her backpack. Pam sewed \(\frac { 1}{4} \) yard of lace on her backpack. Shade the model to show how much more lace Keiko sewed on her backpack than Pam
Go Math Grade 4 Answer Key Chapter 7 Add Fractions Using Models - Lesson Check - Page No 406 Q 12
\(\frac { ■ }{  ■ } \)
Answer: 2/4

Explanation:
Given,
Keiko sewed \(\frac { 3}{4} \) yard of lace on her backpack. Pam sewed \(\frac { 1}{4} \) yard of lace on her backpack.
\(\frac {3}{4} \) – \(\frac {1}{4} \) = \(\frac {2}{4} \)

Subtract Fractions Using Models – Page No 407

Subtract. Use fraction strips to help.
Question 1:
Go Math Grade 4 Answer Key Chapter 7 Subtract Fractions Using Models Q1
Answer: 3/5

Explanation:
Given the fraction, 4/5 and 1/5
The denominators of both the fractions are the same so subtract the numerators.
4/5 – 1/5 = 3/5

Question 2:
\(\frac { 3}{ 4 } – \frac { 1}{ 4 } = \frac { —}{ — } \)

Answer: 2/4

Explanation:
Given the fractions \(\frac { 3}{ 4 } \) and [/latex] \frac { 1}{ 4 } [/latex]
The denominators of both the fractions are the same so subtract the numerators.
\(\frac { 3}{ 4 } – \frac { 1}{ 4 } = \frac { 2 }{ 4 } \)

Question 3:
\(\frac { 5}{ 6 } – \frac { 1}{ 6 } = \frac { —}{ — } \)

Answer: 4/6

Explanation:
Given the fractions \(\frac { 5 }{ 6 } \) and [/latex] \frac { 1 }{ 6 } [/latex]
The denominators of both the fractions are the same so subtract the numerators.
\(\frac { 5}{ 6 } – \frac { 1}{ 6 } = \frac { 4 }{ 6 } \)

Question 4:
\(\frac { 7}{ 8 } – \frac { 1}{ 8 } = \frac { —}{ — } \)

Answer: 6/8

Explanation:
Given the fractions \(\frac { 7 }{ 8 } \) and [/latex] \frac { 1 }{ 8 } [/latex]
The denominators of both the fractions are the same so subtract the numerators.
\(\frac { 7}{ 8 } – \frac { 1}{ 8 } = \frac { 6 }{ 8 } \)

Question 5:
\(\frac { 1}{ 3 } – \frac { 2}{ 3 } = \frac { —}{ — } \)

Answer: 1/3

Explanation:
Given the fractions \(\frac { 1 }{ 3 } \) and [/latex] \frac { 2 }{ 3 } [/latex]
The denominators of both the fractions are the same so subtract the numerators.
\(\frac { 1}{ 3 } – \frac { 2}{ 3 } = \frac { 1}{ 3 } \)

Question 6:
\(\frac { 8}{ 10 } – \frac { 2}{ 10 } = \frac { —}{ — } \)

Answer: 6/10

Explanation:
Given the fractions \(\frac { 8 }{ 10 } \) and [/latex] \frac { 2 }{ 10 } [/latex]
The denominators of both the fractions are the same so subtract the numerators.
\(\frac { 8}{ 10 } – \frac { 2}{ 10 } = \frac { 6 }{ 10 } \)

Question 7:
\(\frac { 3}{ 4 } – \frac { 1}{ 4 } = \frac { —}{ — } \)

Answer: 2/4

Explanation:
Given the fractions \(\frac { 3 }{ 4 } \) and [/latex] \frac { 1 }{ 4 } [/latex]
The denominators of both the fractions are the same so subtract the numerators.
\(\frac { 3}{ 4 } – \frac { 1}{ 4 } = \frac { 2 }{ 4 } \)

Question 8:
\(\frac { 7}{ 6 } – \frac {5}{ 6 } = \frac { —}{ — } \)

Answer: 2/6

Explanation:
Given the fractions \(\frac { 7 }{ 6 } \) and [/latex] \frac { 5 }{ 6 } [/latex]
The denominators of both the fractions are the same so subtract the numerators.
\(\frac { 7}{ 6 } – \frac {5}{ 6 } = \frac { 2 }{ 6 } \)

Problem Solving
Use the table for 9 and 10.
Go Math Grade 4 Answer Key Chapter 7 Subtract Fractions Using Models Q9
Question 9:
Ena is making trail mix. She buys the items shown in the table. How many more pounds of pretzels than raisins does she buy?
\(\frac { —}{ — } \)

Answer: 5/8 pound

Explanation:
Given that,
Ena is making trail mix.
pretzels = 7/8
Raisins = 2/8
To find the number of more pounds of pretzels than raisins she buy
we have to subtract both the fractions.
7/8 – 2/8 = 5/8

Question 10:
How many more pounds of granola than banana chips does she buy?
\(\frac { —}{ — } \)

Answer: 2/8 pound

Explanation:
Granola = 5/8
Banana Chips = 3/8
To find How many more pounds of granola than banana chips does she buy we have to subtract both the fractions.
5/8 – 3/8 = 2/8 pounds

Subtract Fractions Using Models – Page No 408

Question 1:
Lee reads for \(\frac { 3}{ 4} \) hour in the morning and \(\frac {2}{ 4} \) hour in the afternoon. How much longer does Lee read in the morning than in the afternoon?
(a) 5 hours
(b) \(\frac { 5}{ 4} \)
(c) \(\frac { 4}{ 4} \)
(d) \(\frac { 1}{ 4} \)

Answer: \(\frac { 1}{ 4} \)

Explanation:
Given,
Lee reads for \(\frac { 3}{ 4} \) hour in the morning and \(\frac {2}{ 4} \) hour in the afternoon.
\(\frac { 3}{ 4} \) – \(\frac {2}{ 4} \) = \(\frac { 1}{ 4} \)
Lee read \(\frac { 1}{ 4} \) hour in the morning than in the afternoon.
Thus the correct answer is option d.

Question 2:
Which equation does the model below represent?
Go Math Grade 4 Answer Key Chapter 7 Add Fractions Using Models - Lesson Check - Page No 408 Q2
(a) \(\frac { 3}{ 6} – \frac { 2}{ 6} = \frac { 1}{ 6} \)
(b) \(\frac { 2}{ 6} – \frac { 1}{ 6} = \frac { 1}{ 6} \)
(c) \(\frac { 5}{ 6} – \frac { 3}{ 6} = \frac { 2}{ 6} \)
(d) 1 – \( \frac { 3}{ 6} = \frac {3}{ 6} \)

Answer: \(\frac { 5}{ 6} – \frac { 3}{ 6} = \frac { 2}{ 6} \)

Explanation:
From the above figure we can say that \(\frac { 5}{ 6} – \frac { 3}{ 6} = \frac { 2}{ 6} \)
Thus the correct answer is option c.

Question 3:
A city received 2 inches of rain each day for 3 days. The meteorologist said that if the rain had been snow, each inch of rain would have been 10 inches of snow. How much snow would that city have received in the 3 days?

(a) 20 inches
(b) 30 inches
(c) 50 inches
(d) 60 inches

Answer: 60 inches

Explanation:
Given,
A city received 2 inches of rain each day for 3 days.
2 × 3 inches = 6 inches
The meteorologist said that if the rain had been snow, each inch of rain would have been 10 inches of snow.
6 × 10 inches = 60 inches
Therefore the city has received 60 inches of snow in 3 days.
Thus the correct answer is option d.

Question 4:
At a party there were four large submarine sandwiches, all the same size. During the party, \(\frac { 2}{ 3} \) of the chicken sandwich, \(\frac { 3}{ 4} \) of the tuna sandwich, \(\frac { 7}{ 12} \) of the roast beef sandwich, and \(\frac { 5}{ 6} \) of the veggie sandwich were eaten. Which sandwich had the least amount left?

(a) chicken
(b) tuna
(c) roast beef
(d) veggie

Answer: veggie

Explanation:
Given,
At a party there were four large submarine sandwiches, all the same size. During the party, \(\frac { 2}{ 3} \) of the chicken sandwich, \(\frac { 3}{ 4} \) of the tuna sandwich, \(\frac { 7}{ 12} \) of the roast beef sandwich, and \(\frac { 5}{ 6} \) of the veggie sandwich were eaten.
Compare the fractions \(\frac { 2}{ 3} \), \(\frac { 3}{ 4} \) , \(\frac { 7}{ 12} \) and \(\frac { 5}{ 6} \).
Among all the fractions veggie has the least fraction.
Thus the correct answer is option d.

Question 5:
Deena uses \(\frac { 3}{ 8} \) cup milk and \(\frac { 2}{ 8} \) cup oil in a recipe. How much liquid does she use in all?

(a) \(\frac {1}{ 8} \) cup
(b) \(\frac {5}{ 8} \) cup
(c) \(\frac {6}{ 8} \) cup
(d) 5 cups

Answer: \(\frac {5}{ 8} \) cup

Explanation:
Given,
Deena uses \(\frac { 3}{ 8} \) cup milk and \(\frac { 2}{ 8} \) cup oil in a recipe.
\(\frac { 3}{ 8} \) + \(\frac { 2}{ 8} \) = \(\frac {5}{ 8} \) cup
Therefore she used \(\frac {5}{ 8} \) cup of milk in all.
Thus the correct answer is option b.

Question 6:
In the car lot, \(\frac { 4}{ 12} \) of the cars are white and \(\frac { 3}{ 12} \) of the cars are blue. What fraction of the cars in the lot are either white or blue?
(a) \(\frac { 1}{ 12} \)
(b) \(\frac { 7}{ 24} \)
(c) \(\frac { 7}{ 12} \)
(d) 7

Answer: \(\frac { 7}{ 12} \)

Explanation:
Given,
In the car lot, \(\frac { 4}{ 12} \) of the cars are white and \(\frac { 3}{ 12} \) of the cars are blue.
\(\frac { 4}{ 12} \) + \(\frac { 3}{ 12} \) = \(\frac { 7}{ 12} \)
Thus the correct answer is option c.

Subtract Fractions Using Models – Page No 411

Question 1:
9 twelfth-size parts − 5 twelfth-size parts =
\(\frac { —}{ — } \)

Answer: 4/12

Explanation:
9 twelfth-size parts − 5 twelfth-size parts
9 × \(\frac { 1 }{ 12 } \) = \(\frac { 9 }{ 12 } \)
5 × \(\frac { 1 }{ 12 } \) = \(\frac { 5 }{ 12 } \)
The denominators of both the fractions are the same so subtract the numerators.
\(\frac { 9 }{ 12 } \) – \(\frac { 5 }{ 12 } \) = \(\frac { 4 }{ 12 } \)

Question 2:
\(\frac { 3}{ 12} + \frac {8}{ 12 } = \frac { —}{ — } \)

Answer: 11/12

Explanation:
Given the fractions,
\(\frac { 3 }{ 12 } \) and \(\frac { 8 }{ 12 } \)
Add both the fractions
The denominators of both the fractions are the same so add the numerators.
\(\frac { 3}{ 12} + \frac {8}{ 12 } = \frac { 11 }{ 12 } \)

Question 3:
\(\frac { 1}{ 3 } + \frac {1}{ 3 } = \frac { —}{ — } \)

Answer: 2/3

Explanation:
Given the fractions,
\(\frac { 1 }{ 3 } \) and \(\frac { 1 }{ 3 } \)
Add both the fractions
The denominators of both the fractions are the same so add the numerators.
\(\frac { 1}{ 3 } + \frac {1}{ 3 } = \frac { 2 }{ 3 } \)

Question 4:
\(\frac { 3}{ 4 } – \frac {1}{ 4 } = \frac { —}{ — } \)

Answer: 2/4

Explanation:
Given the fractions,
\(\frac { 3 }{ 4 } \) and \(\frac { 1 }{ 4 } \)
Subtract both the fractions
The denominators of both the fractions are the same so Subtract the numerators.
\(\frac { 3}{ 4 } – \frac {1}{ 4 } = \frac { 2 }{ 4 } \)

Question 5:
\(\frac { 2}{ 6 } + \frac {2}{ 6 } = \frac { —}{ — } \)

Answer: 4/6

Explanation:
Given the fractions,
\(\frac { 2 }{ 6 } \) and \(\frac { 2 }{ 6 } \)
Add both the fractions
The denominators of both the fractions are the same so add the numerators.
\(\frac { 2}{ 6 } + \frac {2}{ 6 } = \frac { 4 }{ 6 } \)

Question 6:
\(\frac { 3}{ 8 } – \frac {1}{ 8 } = \frac { —}{ — } \)

Answer: 2/8

Explanation:
Given the fractions,
\(\frac { 3 }{ 8 } \) and \(\frac { 1 }{ 8 } \)
Subtract both the fractions
The denominators of both the fractions are the same so Subtract the numerators.
\(\frac { 3}{ 8 } – \frac {1}{ 8 } = \frac { 2 }{ 8 } \)

Question 7:
\(\frac { 6}{ 10 } – \frac {2}{ 10 } = \frac { —}{ — } \)

Answer: 4/10

Explanation:
Given the fractions,
\(\frac { 6 }{ 10 } \) and \(\frac { 2 }{ 10 } \)
Subtract both the fractions
The denominators of both the fractions are the same so Subtract the numerators.
\(\frac { 6}{ 10 } – \frac {2}{ 10 } = \frac { 4 }{ 10 } \)

Question 8:
\(\frac { 1}{ 2 } – \frac {1}{2 } = \frac { —}{ — } \)

Answer: 0

Explanation:
Given the fractions,
\(\frac { 1 }{ 2 } \) and \(\frac { 1 }{ 2 } \)
Subtract both the fractions
The denominators of both the fractions are the same so Subtract the numerators.
\(\frac { 1}{ 2 } – \frac {1}{2 } \) = 0

Question 9:
\(\frac {5}{ 6 } – \frac {4}{ 6 } = \frac { —}{ — } \)

Answer: 1/6

Explanation:
Given the fractions,
\(\frac { 5 }{ 6 } \) and \(\frac { 4 }{ 6 } \)
Subtract both the fractions
The denominators of both the fractions are the same so Subtract the numerators.
\(\frac {5}{ 6 } – \frac {4}{ 6 } = \frac { 1 }{ 6 } \)

Question 10:
\(\frac { 4}{ 5 } – \frac {2}{ 5 } = \frac { —}{ — } \)

Answer: 2/5

Explanation:
Given the fractions,
\(\frac { 4 }{ 5 } \) and \(\frac { 2 }{ 5 } \)
Subtract both the fractions
The denominators of both the fractions are the same so Subtract the numerators.
\(\frac { 4}{ 5 } – \frac {2}{ 5 } = \frac { 2 }{ 5 } \)

Question 11:
\(\frac { 1}{ 4 } + \frac {1}{ 4 } = \frac { —}{ — } \)

Answer: 2/4

Explanation:
Given the fractions,
\(\frac { 1 }{ 4 } \) and \(\frac { 1 }{ 4 } \)
Add both the fractions
The denominators of both the fractions are the same so add the numerators.
\(\frac { 1}{ 4 } + \frac {1}{ 4 } = \frac { 2 }{ 4 } \)

Question 12:
\(\frac { 9}{ 10 } – \frac {5}{ 10 } = \frac { —}{ — } \)

Answer: 4/10

Explanation:
Given the fractions,
\(\frac { 9 }{ 10 } \) and \(\frac { 5 }{ 10 } \)
Subtract both the fractions
The denominators of both the fractions are the same so Subtract the numerators.
\(\frac { 9}{ 10 } – \frac {5}{ 10 } = \frac { 4 }{ 10 } \)

Question 13:
\(\frac { 1}{ 12 } + \frac {7}{ 12 } = \frac { —}{ — } \)

Answer: 8/12

Explanation:
Given the fractions,
\(\frac { 1 }{ 12 } \) and \(\frac { 7 }{ 12 } \)
Add both the fractions
The denominators of both the fractions are the same so add the numerators.
\(\frac { 1}{ 12 } + \frac {7}{ 12 } = \frac { 8 }{ 12 } \)

Question 14:
Christopher mixes \(\frac { 3}{ 8} \) gallon of red paint with \(\frac { 5}{ 8} \) gallon of blue paint to make purple paint. He uses \(\frac { 2}{8} \) gallon of the purple paint. How much purple paint is left?
\(\frac { —}{ — } \) gallon

Answer: \(\frac { 6 }{ 8 } \) gallon

Explanation:
Given,
Christopher mixes \(\frac { 3}{ 8} \) gallon of red paint with \(\frac { 5}{ 8} \) gallon of blue paint to make purple paint.
He uses \(\frac { 2}{8} \) gallon of the purple paint.
\(\frac { 3}{ 8} \) + \(\frac { 5}{ 8} \) = \(\frac { 8 }{ 8 } \)
\(\frac { 8 }{ 8 } \) – \(\frac { 2 }{ 8 } \) = \(\frac { 6 }{ 8 } \) gallon

Question 15:
A city worker is painting a stripe down the center of Main Street. Main Street is \(\frac { 8}{ 10} \) mile long. The worker painted \(\frac { 4}{ 10} \) mile of the street. Explain how to find what part of a mile is left to paint.
\(\frac { —}{ — } \) mile

Answer: \(\frac { 4 }{ 10 } \) mile

Explanation:
Given,
A city worker is painting a stripe down the center of Main Street.
Main Street is \(\frac { 8}{ 10} \) mile long.
The worker painted \(\frac { 4}{ 10} \) mile of the street.
\(\frac { 8 }{ 10 } \) – \(\frac { 4 }{ 10 } \) = \(\frac { 4 }{ 10 } \) mile

Question 16:
Sense or Nonsense? Brian says that when you add or subtract fractions with the same denominator, you can add or subtract the numerators and keep the same denominator. Is Brian correct? Explain.

Answer: correct

Explanation:
The statement of Brian is correct because when you add or subtract fractions with the same denominator, you can add or subtract the numerators and keep the same denominator.

Question 17:
The length of a rope was \(\frac { 6}{8} \) yard. Jeff cut the rope into 3 pieces. Each piece is a different length measured in eighths of a yard. What is the length of each piece of rope?

Answer: \(\frac { 2}{8} \)

Explanation:
Given,
The length of a rope was \(\frac { 6}{8} \) yard.
Jeff cut the rope into 3 pieces. Each piece is a different length measured in eighths of a yard.
Divide \(\frac { 6}{8} \) into 3 pieces.
\(\frac { 6}{8} \) ÷ 3 = \(\frac { 2}{8} \)

Question 18:
For 18a–18d, choose Yes or No to show if the sum or difference is correct.

a. \(\frac { 3}{ 5 } – \frac {1}{ 5 } = \frac {4 }{5 } \)
(i) yes
(ii) no

Answer: no

Explanation:
The denominators of both the fractions are the same so Subtract the numerators.
\(\frac { 3}{ 5 } – \frac {1}{ 5 } = \frac {2 }{5 } \)
Thus the above statement is not correct.

b. \(\frac { 1}{ 4 } – \frac {2}{4 } = \frac {3 }{8 } \)
(i) yes
(ii) no

Answer: no

Explanation:
The denominators of both the fractions are the same so Subtract the numerators.
\(\frac { 1}{ 4 } – \frac {2}{4 } = \frac {1 }{4 } \)
Thus the above statement is not correct.

c. \(\frac { 5}{ 8} – \frac {4}{ 8 } = \frac {1 }{8 } \)
(i) yes
(ii) no

Answer: yes

Explanation:
The denominators of both the fractions are the same so Subtract the numerators.
\(\frac { 5}{ 8} – \frac {4}{ 8 } = \frac {1 }{8 } \)
Thus the above statement is correct.

d. \(\frac { 4}{ 9 } – \frac {2}{ 9 } = \frac {6 }{9 } \)
(i) yes
(ii) no
Answer: no

Explanation:
The denominators of both the fractions are the same so Subtract the numerators.
d. \(\frac { 4}{ 9 } – \frac {2}{ 9 } = \frac {2 }{9 } \)
Thus the above statement is not correct.

Sense or Nonsense? – Page No. 412

Question 19.
Harry says that \(\frac{1}{4}\) + \(\frac{1}{8}\) = \(\frac{2}{8}\). Jane says \(\frac{1}{4}\) + \(\frac{1}{8}\) = \(\frac{3}{8}\).
Whose answer makes sense? Whose answer is nonsense? Explain your reasoning. Draw a model to help.
Go Math Grade 4 Answer Key Chapter 7 Add and Subtract Fractions Page No. 412 Q 19
Type below:
___________

Answer: Jane’s Answer Makes Sense. Because the numerators are the same but the denominators are different. So, in order to add the fractions first, they have to make the denominators equal.
1/4 + 1/8 = 2/8 + 1/8 = 3/8

Add and Subtract Fractions – Page No. 413

Find the sum or difference.

Question 1.
Go Math Grade 4 Answer Key Chapter 7 Add and Subtract Fractions Common Core - New Page No. 413 Q 1

Answer: 12/12

Explanation:
The denominators of both the fractions are the same so add the numerators.
\(\frac{4}{12}\) + \(\frac{8}{12}\)
= \(\frac{12}{12}\)

Question 2.
\(\frac{3}{6}-\frac{1}{6}\) = \(\frac{□}{□}\)

Answer: 2/6

Explanation:
The denominators of both the fractions are the same so Subtract the numerators.
\(\frac{3}{6}\) – \(\frac{1}{6}\)
= \(\frac{2}{6}\)

Question 3.
\(\frac{4}{5}-\frac{3}{5}\) = \(\frac{□}{□}\)

Answer: 1/5

Explanation:
The denominators of both the fractions are the same so Subtract the numerators.
\(\frac{4}{5}\) – \(\frac{3}{5}\)
= \(\frac{1}{5}\)

Question 4.
\(\frac{6}{10}+\frac{3}{10}\) = \(\frac{□}{□}\)

Answer: 9/10

Explanation:
The denominators of both the fractions are the same so add the numerators.
\(\frac{6}{10}+\frac{3}{10}\) = \(\frac{9}{10}\)

Question 5.
1 – \(\frac{3}{8}\) = \(\frac{□}{□}\)

Answer: 5/8

Explanation:
The denominators of both the fractions are the same so Subtract the numerators.
1 – \(\frac{3}{8}\)
= \(\frac{8}{8}\) – \(\frac{3}{8}\)
= \(\frac{5}{8}\)

Question 6.
\(\frac{1}{4}+\frac{2}{4}\) = \(\frac{□}{□}\)

Answer: 3/4

Explanation:
The denominators of both the fractions are the same so add the numerators.
\(\frac{1}{4}+\frac{2}{4}\) = \(\frac{3}{4}\)

Question 7.
\(\frac{9}{12}-\frac{5}{12}\) = \(\frac{□}{□}\)

Answer: 4/12

Explanation:
The denominators of both the fractions are the same so Subtract the numerators.
\(\frac{9}{12}-\frac{5}{12}\) = \(\frac{4}{12}\)

Question 8.
\(\frac{5}{6}-\frac{2}{6}\) = \(\frac{□}{□}\)

Answer: 3/6

Explanation:
The denominators of both the fractions are the same so Subtract the numerators.
\(\frac{5}{6}-\frac{2}{6}\) = \(\frac{3}{6}\)

Question 9.
\(\frac{2}{3}+\frac{1}{3}\) = \(\frac{□}{□}\)

Answer: 3/3 = 1

Explanation:
The denominators of both the fractions are the same so add the numerators.
\(\frac{2}{3}+\frac{1}{3}\) = \(\frac{3}{3}\) = 1

Problem Solving

Use the table for 10 and 11.
Go Math Grade 4 Answer Key Chapter 7 Add and Subtract Fractions Common Core - New Page No. 413 Q 10

Question 10.
Guy finds how far his house is from several locations and makes the table shown. How much farther away from Guy’s house is the library than the cafe?
\(\frac{□}{□}\)

Answer: \(\frac{5}{10}\) mile

Explanation:
The distance from Guy’s house to the library is \(\frac{9}{10}\) mile
The distance from Guy’s house to the cafe is \(\frac{4}{10}\) mile
To find how much farther away from Guy’s house is the library than the cafe subtract both the fractions.
\(\frac{9}{10}\) – \(\frac{4}{10}\) = \(\frac{5}{10}\) mile

Question 11.
If Guy walks from his house to school and back, how far does he walk?
\(\frac{□}{□}\)

Answer: 10/10 mile

Explanation:
The distance from Guy’s house to school = \(\frac{5}{10}\) mile
From school to house \(\frac{5}{10}\) mile
\(\frac{5}{10}\) + \(\frac{5}{10}\) = \(\frac{10}{10}\) mile

Add and Subtract Fractions – Lesson Check – Page No. 414

Question 1.
Mr. Angulo buys \(\frac{5}{8}\) pound of red grapes and \(\frac{3}{8}\)pound of green grapes. How many pounds of grapes did Mr. Angulo buy in all?
Options:
a. \(\frac{1}{8}\) pound
b. \(\frac{2}{8}\) pound
c. 1 pound
d. 2 pounds

Answer: 1 pound

Explanation:
Given that,
Mr. Angulo buys \(\frac{5}{8}\) pound of red grapes and \(\frac{3}{8}\)pound of green grapes.
\(\frac{5}{8}\) + \(\frac{3}{8}\)
= \(\frac{8}{8}\)
= 1
Thus the correct answer is option c.

Question 2.
Which equation does the model below represent?
Go Math Grade 4 Answer Key Chapter 7 Add and Subtract Fractions Page No. 414 Q 2
Options:
a. \(\frac{7}{8}\) + \(\frac{2}{8}\) = \(\frac{9}{8}\)
b. \(\frac{5}{8}\) – \(\frac{2}{8}\) = \(\frac{3}{8}\)
c. \(\frac{8}{8}\) – \(\frac{5}{8}\) = \(\frac{3}{8}\)
d. \(\frac{7}{8}\) – \(\frac{2}{8}\) = \(\frac{5}{8}\)

Answer: \(\frac{7}{8}\) – \(\frac{2}{8}\) = \(\frac{5}{8}\)

Explanation:
By seeing the above figure we can say that, the equation of the model is
\(\frac{7}{8}\) – \(\frac{2}{8}\) = \(\frac{5}{8}\)
Thus the correct answer is option d.

Spiral Review

Question 3.
There are 6 muffins in a package. How many packages will be needed to feed 48 people if each person has 2 muffins?
Options:
a. 4
b. 8
c. 16
d. 24

Answer: 16

Explanation:
There are 6 muffins in a package.
Number of people = 48
48/6 = 8
Also given that each person gets 2 muffins.
8 × 2 = 16
Thus the correct answer is option c.

Question 4.
Camp Oaks gets 32 boxes of orange juice and 56 boxes of apple juice. Each shelf in the cupboard can hold 8 boxes of juice. What is the least number of shelves
needed for all the juice boxes?
Options:
a. 4
b. 7
c. 11
d. 88

Answer: 11

Explanation:
Given,
Camp Oaks gets 32 boxes of orange juice and 56 boxes of apple juice.
Each shelf in the cupboard can hold 8 boxes of juice.
First, add the boxes of orange juice and apple juice.
32 + 56 = 88 boxes of juice
Now divide 88 by 8
88/8 = 11
Thus the correct answer is option c.

Question 5.
A machine makes 18 parts each hour. If the machine operates 24 hours a day, how many parts can it make in one day
Options:
a. 302
b. 332
c. 362
d. 432

Answer: 432

Explanation:
Given,
A machine makes 18 parts each hour.
Multiply the number of parts with the number of hours.
18 × 24 = 432 parts in a day.
Thus the correct answer is option d.

Question 6.
Which equation does the model below represent?
Go Math Grade 4 Answer Key Chapter 7 Add and Subtract Fractions Common Core - New Page No. 414 Q 6
Options:
a. \(\frac{5}{6}\) – \(\frac{4}{6}\) = \(\frac{1}{6}\)
b. \(\frac{4}{5}\) – \(\frac{1}{5}\) = \(\frac{3}{5}\)
c. \(\frac{5}{5}\) – \(\frac{4}{5}\) = \(\frac{1}{5}\)
d. \(\frac{6}{6}\) – \(\frac{4}{6}\) = \(\frac{2}{6}\)

Answer: \(\frac{5}{6}\) – \(\frac{4}{6}\) = \(\frac{1}{6}\)

Explanation:
By observing the figure we can say that the equation is \(\frac{5}{6}\) – \(\frac{4}{6}\) = \(\frac{1}{6}\).
Thus the correct answer is option a.

Add and Subtract Fractions – Page No. 415

Choose the best term from the box.
Go Math Grade 4 Answer Key Chapter 7 Add and Subtract Fractions Page No. 415 Q 1

Question 1.
A ___________ always has a numerator of 1.
________________

Answer: unit fraction

Explanation:
A unit fraction is a rational number written as a fraction where the numerator is one and the denominator is a positive integer.

Write the fraction as a sum of unit fractions.

Question 2.
Type below:
____________

Answer: 1/3 + 1/3 + 1/3

Explanation:
A unit fraction is a rational number written as a fraction where the numerator is one and the denominator is a positive integer. The unit fraction of 3/3 is 1/3 + 1/3 + 1/3

Question 3.
Type below:
____________

Answer: 1/12 + 1/12 + 1/12 + 1/12

A unit fraction is a rational number written as a fraction where the numerator is one and the denominator is a positive integer. The unit fraction of 4/12 is 1/12 + 1/12 + 1/12 + 1/12.

Use the model to write an equation.

Question 4.
Go Math Grade 4 Answer Key Chapter 7 Add and Subtract Fractions Page No. 415 Q 4
Type below:
_________

Answer: 1/5

Explanation:
By using the above model we can write the equation
3/5 – 2/5 = 1/5

Question 5.
Go Math Grade 4 Answer Key Chapter 7 Add and Subtract Fractions Page No. 415 Q 5
Type below:
_________

Answer: 4/6

Explanation:
By using the above model we can write the equation
5/6 – 1/6 = 4/6

Use the model to solve the equation.

Question 6.
Go Math Grade 4 Answer Key Chapter 7 Add and Subtract Fractions Page No. 415 Q 6
\(\frac{3}{8}+\frac{2}{8}\) = \(\frac{□}{□}\)

Answer: 5/8

Explanation:
The denominators of both the fractions are the same so add the numerators.
\(\frac{3}{8}+\frac{2}{8}\) = \(\frac{5}8}\)

Question 7.
\(\frac{4}{10}+\frac{5}{10}\) = \(\frac{□}{□}\)

Answer: 9/10

Explanation:
The denominators of both the fractions are the same so add the numerators.
\(\frac{4}{10}+\frac{5}{10}\) = \(\frac{9}{10}\)

Find the sum or difference.

Question 8.
\(\frac{9}{12}-\frac{7}{12}\) = \(\frac{□}{□}\)

Answer: 2/12

Explanation:
The denominators of both the fractions are the same so subtract the numerators.
\(\frac{9}{12}-\frac{7}{12}\) = \(\frac{2}{12}\)

Question 9.
\(\frac{2}{3}+\frac{1}{3}\) = \(\frac{□}{□}\)

Answer: 3/3

Explanation:
The denominators of both the fractions are the same so add the numerators.
\(\frac{2}{3}+\frac{1}{3}\) = \(\frac{3}{3}\)

Question 10.
\(\frac{1}{5}+\frac{3}{5}\) = \(\frac{□}{□}\)

Answer: 4/5

Explanation:
The denominators of both the fractions are the same so add the numerators.
\(\frac{1}{5}+\frac{3}{5}\) = \(\frac{4}{5}\)

Question 11.
\(\frac{2}{6}+\frac{2}{6}\) = \(\frac{□}{□}\)

Answer: 4/6

Explanation:
The denominators of both the fractions are the same so add the numerators.
\(\frac{2}{6}+\frac{2}{6}\) = \(\frac{4}{6}\)

Question 12.
\(\frac{4}{4}-\frac{2}{4}\) = \(\frac{□}{□}\)

Answer: 2/4

Explanation:
The denominators of both the fractions are the same so subtract the numerators.
\(\frac{4}{4}-\frac{2}{4}\) = \(\frac{2}{4}\)

Question 13.
\(\frac{7}{8}-\frac{4}{8}\) = \(\frac{□}{□}\)

Answer: 3/8

Explanation:
The denominators of both the fractions are the same so subtract the numerators.
\(\frac{7}{8}-\frac{4}{8}\) = \(\frac{3}{8}\)

Add and Subtract Fractions – Page No. 416

Question 14.
Tyrone mixed \(\frac{7}{12}\) quart of red paint with \(\frac{1}{12}\) quart of yellow paint. How much paint does Tyrone have in the mixture?
\(\frac{□}{□}\) quart

Answer: 8/12 quart

Explanation:
Given that,
Tyrone mixed \(\frac{7}{12}\) quart of red paint with \(\frac{1}{12}\) quart of yellow paint.
Add both the fraction of paints.
\(\frac{7}{12}\) + \(\frac{1}{12}\) = \(\frac{8}{12}\) quart
Therefore Tyrone has \(\frac{8}{12}\) quart in the mixture.

Question 15.
Jorge lives \(\frac{6}{8}\) mile from school and \(\frac{2}{8}\) mile from a ballpark. How much farther does Jorge live from school than from the ballpark?
\(\frac{□}{□}\) mile

Answer: 4/8 mile

Explanation:
Given,
Jorge lives \(\frac{6}{8}\) mile from school and \(\frac{2}{8}\) mile from a ballpark.
Subtract both the fractions.
\(\frac{6}{8}\) – \(\frac{2}{8}\) = \(\frac{4}{8}\)
Therefore Jorge live \(\frac{4}{8}\) mile from school than from the ballpark.

Question 16.
Su Ling started an art project with 1 yard of felt. She used \(\frac{2}{6}\) yard on Tuesday and \(\frac{3}{6}\) yard on Wednesday. How much felt does Su Ling have left?
\(\frac{□}{□}\) yard

Answer: 1/6 yard

Explanation:
Given,
Su Ling started an art project with 1 yard of felt.
She used \(\frac{2}{6}\) yard on Tuesday and \(\frac{3}{6}\) yard on Wednesday.
\(\frac{3}{6}\) – \(\frac{2}{6}\) = \(\frac{1}{6}\) yard
Therefore, Su Ling \(\frac{1}{6}\) yard left.

Question 17.
Eloise hung artwork on \(\frac{2}{5}\) of a bulletin board. She hung math papers on \(\frac{1}{5}\) of the same bulletin board. What part of the bulletin board has artwork or math papers?
\(\frac{□}{□}\)

Answer: 3/5

Explanation:
Given,
Eloise hung artwork on \(\frac{2}{5}\) of a bulletin board.
She hung math papers on \(\frac{1}{5}\) of the same bulletin board.
\(\frac{2}{5}\) + \(\frac{1}{5}\) = \(\frac{3}{5}\)
\(\frac{3}{5}\) part of the bulletin board has artwork or math papers.

Add and Subtract Fractions – Page No. 419

Write the unknown numbers. Write mixed numbers above
the number line and fractions greater than one below the number line.

Question 1.
Go Math Grade 4 Answer Key Chapter 7 Add and Subtract Fractions Page No. 419 Q 1
Type below:
___________

Answer:
Go-Math-Grade-4-Answer-Key-Chapter-7-Add-and-Subtract-Fractions-Page-No.-419-Q-1

Write the mixed number as a fraction.

Question 2.
1 \(\frac{1}{8}\) = \(\frac{□}{□}\)

Answer: 9/8

Explanation:
Given the expression,
1 \(\frac{1}{8}\)
Convert from the mixed fraction to the improper fraction.
1 \(\frac{1}{8}\) = (1 × 8 + 1)/8 = 9/8

Question 3.
1 \(\frac{3}{5}\) = \(\frac{□}{□}\)

Answer: \(\frac{8}{5}\)

Explanation:
Given the expression,
1 \(\frac{3}{5}\)
Convert from the mixed fraction to the improper fraction.
1 \(\frac{3}{5}\) = (5 × 1 + 3)/5 = \(\frac{8}{5}\)

Question 4.
1 \(\frac{2}{3}\) = \(\frac{□}{□}\)

Answer: 5/3

Explanation:
Given the expression,
1 \(\frac{2}{3}\)
Convert from the mixed fraction to the improper fraction.
1 \(\frac{2}{3}\) = (3 × 1 + 2)/3 = \(\frac{5}{3}\)

Write the fraction as a mixed number.

Question 5.
\(\frac{11}{4}\) = _____ \(\frac{□}{□}\)

Answer: 2 \(\frac{3}{4}\)

Explanation:
Given the expression,
\(\frac{11}{4}\)
Convert from the improper fraction to the mixed fraction.
\(\frac{11}{4}\) = 2 \(\frac{3}{4}\)

Question 6.
\(\frac{6}{5}\) = _____ \(\frac{□}{□}\)

Answer: 1 \(\frac{1}{5}\)

Explanation:
Given the expression,
\(\frac{6}{5}\)
Convert from the improper fraction to the mixed fraction.
\(\frac{6}{5}\) = 1 \(\frac{1}{5}\)

Question 7.
\(\frac{13}{10}\) = _____ \(\frac{□}{□}\)

Answer: 1 \(\frac{3}{10}\)

Explanation:
Given the expression,
\(\frac{13}{10}\)
Convert from the improper fraction to the mixed fraction.
\(\frac{13}{10}\) = 1 \(\frac{3}{10}\)

Write the mixed number as a fraction.

Question 8.
2 \(\frac{7}{10}\) = \(\frac{□}{□}\)

Answer: \(\frac{27}{10}\)

Explanation:
Given the expression,
2 \(\frac{7}{10}\)
Convert from the mixed fraction to the improper fraction.
2 \(\frac{7}{10}\) = \(\frac{27}{10}\)

Question 9.
3 \(\frac{2}{3}\) = \(\frac{□}{□}\)

Answer: \(\frac{11}{3}\)

Explanation:
Given the expression,
3 \(\frac{2}{3}\)
Convert from the mixed fraction to the improper fraction.
3 \(\frac{2}{3}\) = \(\frac{11}{3}\)

Question 10.
4 \(\frac{2}{5}\) = \(\frac{□}{□}\)

Answer: \(\frac{22}{5}\)

Explanation:
Given the expression,
4 \(\frac{2}{5}\)
Convert from the mixed fraction to the improper fraction.
4 \(\frac{2}{5}\) = \(\frac{22}{5}\)

Use Repeated Reasoning Algebra Find the unknown numbers.

Question 11.
\(\frac{13}{7}\) = 1 \(\frac{■}{7}\)
■ = _____

Answer: 1 \(\frac{6}{7}\)

Explanation:
Given the expression,
\(\frac{13}{7}\)
Convert from the mixed fraction to the improper fraction.
\(\frac{13}{7}\) = 1 \(\frac{6}{7}\)

Question 12.
■ \(\frac{5}{6}\) = \(\frac{23}{6}\)
■ = _____

Answer: 3

Explanation:
Given the expression,
■ \(\frac{5}{6}\) = \(\frac{23}{6}\)
■ \(\frac{5}{6}\) × 6 = 23
■ ×  = 23 – 5
■ = 18/6
■ = 3

Question 13.
\(\frac{57}{11}\) = ■ \(\frac{■}{11}\)
_____ \(\frac{□}{□}\)

Answer: 5 \(\frac{2}{11}\)

Explanation:
Given the expression,
\(\frac{57}{11}\) = ■ \(\frac{■}{11}\)
Convert from the improper fraction to the mixed fraction.
\(\frac{57}{11}\) = 5 \(\frac{2}{11}\)

Question 14.
Pen has \(\frac{1}{2}\)-cup and \(\frac{1}{8}\)-cup measuring cups. What are two ways he could measure out 1 \(\frac{3}{4}\) cups of flour?
Type below:
_________________

Answer: 3 \(\frac{1}{2}\)-cups and 2 \(\frac{1}{8}\)-cup

Explanation:
Pen has \(\frac{1}{2}\)-cup and \(\frac{1}{8}\)-cup measuring cups.
1 \(\frac{3}{4}\) = \(\frac{1}{2}\) + \(\frac{1}{2}\) + \(\frac{1}{2}\) + \(\frac{1}{8}\) + \(\frac{1}{8}\)
= 1 \(\frac{3}{4}\) = 3 \(\frac{1}{2}\)-cups + 2 \(\frac{1}{8}\)-cup

Question 15.
Juanita is making bread. She needs 3 \(\frac{1}{2}\) cups of flour. Juanita only has a \(\frac{1}{4}\)-cup measuring cup. How many \(\frac{1}{4}\) cups of flour will Juanita use to prepare the bread?
_____ \(\frac{1}{4}\) cups of flour

Answer: 14 \(\frac{1}{4}\) cups of flour

Explanation:
Juanita is making bread. She needs 3 \(\frac{1}{2}\) cups of flour. Juanita only has a \(\frac{1}{4}\)-cup measuring cup.
3 \(\frac{1}{2}\) = \(\frac{1}{4}\) + \(\frac{1}{4}\) + \(\frac{1}{4}\) + \(\frac{1}{4}\) + \(\frac{1}{4}\) + \(\frac{1}{4}\) + \(\frac{1}{4}\) + \(\frac{1}{4}\) + \(\frac{1}{4}\) + \(\frac{1}{4}\) + \(\frac{1}{4}\) + \(\frac{1}{4}\) + \(\frac{1}{4}\)
Therefore she needs 14 \(\frac{1}{4}\) cups of flour.

Add and Subtract Fractions – Page No. 420

Use the recipe to solve 16–18.
Go Math Grade 4 Answer Key Chapter 7 Add and Subtract Fractions Page No. 420 Q 16

Question 16.
Reason Quantitatively Cal is making energy squares. How many \(\frac{1}{2}\) cups of peanut butter are used in the recipe?
_____ \(\frac{1}{2}\) cups of peanut butter

Answer: 3 \(\frac{1}{2}\) cups of peanut butter

Explanation:
Given that 1 \(\frac{1}{2}\) cups of peanut butter are used in the recipe.
We have to find how many \(\frac{1}{2}\) cups of peanut butter are used in the recipe.
\(\frac{1}{2}\) + \(\frac{1}{2}\)  + \(\frac{1}{2}\)
Therefore 3 \(\frac{1}{2}\) cups of peanut butter are used in the recipe.

Question 17.
Suppose Cal wants to make 2 times as many energy squares as the recipe makes. How many cups of bran cereal should he use? Write your answer as a mixed number and as a fraction greater than 1 in simplest form.
Type below:
____________

Answer:
Take the amount of bran Cal is using and multiply it by 2
Given that 3 \(\frac{1}{4}\) cups of bran cereal is used in the recipe.
3 \(\frac{1}{4}\) × 2
= \(\frac{13}{4}\) × 2
= \(\frac{13}{2}\)
= 6 \(\frac{1}{2}\)
Thus 6 \(\frac{1}{2}\) cups of bran cereal he should use.

Question 18.
Cal added 2 \(\frac{3}{8}\) cups of raisins. Write this mixed number as a fraction greater than 1 in the simplest form.
\(\frac{□}{□}\)

Answer: \(\frac{19}{8}\)

Explanation:
Given,
Cal added 2 \(\frac{3}{8}\) cups of raisins.
Convert from the mixed fraction to the improper fraction.
2 \(\frac{3}{8}\) = \(\frac{19}{8}\)

Question 19.
Jenn is preparing brown rice. She needs 1 \(\frac{1}{2}\) cups of brown rice and 2 cups of water. Jenn has only a \(\frac{1}{8}\)– cup measuring cup. How many \(\frac{1}{8}\) cups each of rice and water will Jenn use to prepare the rice?
brown rice: ________ \(\frac{1}{8}\) cups
water: _________ \(\frac{1}{8}\) cups

Answer:
Number of water cups = 16
Number of brown rice cups = 12

Explanation:
Brown rice needed = 1 1/2 cups = 3/2 cups
Water needed = 2 cups
Measuring cups = 1/8
No. of cups used of water = 2/1/8 = 16
No. of cups used of rice = 3/2/1/8 = 12 cups

Question 20.
Draw a line to show the mixed number and fraction that have the same value.
Go Math Grade 4 Answer Key Chapter 7 Add and Subtract Fractions Page No. 420 Q 20
Type below:
____________

Answer:
Go-Math-Grade-4-Answer-Key-Chapter-7-Add-and-Subtract-Fractions-Page-No.-420-Q-20

Rename Fractions and Mixed Numbers – Page No. 421

Write the mixed number as a fraction.

Question 1.
2 \(\frac{3}{5}\)
Go Math Grade 4 Answer Key Chapter 7 Add and Subtract Fractions Common Core - New Page No. 421 Q 1

Question 2.
4 \(\frac{1}{3}\)
\(\frac{□}{□}\)

Answer: \(\frac{13}{3}\)

Explanation:
\(\frac{3}{3}\) + \(\frac{3}{3}\) + \(\frac{3}{3}\) + \(\frac{3}{3}\) + \(\frac{1}{3}\) = \(\frac{13}{3}\)

Question 3.
1 \(\frac{2}{5}\)
\(\frac{□}{□}\)

Answer: \(\frac{7}{5}\)

Explanation:
\(\frac{5}{5}\) + \(\frac{2}{5}\) = \(\frac{7}{5}\)

Question 4.
3 \(\frac{3}{2}\)
\(\frac{□}{□}\)

Answer: \(\frac{9}{2}\)

Explanation:
\(\frac{2}{2}\) + \(\frac{2}{2}\) + \(\frac{2}{2}\) + \(\frac{2}{2}\) + \(\frac{1}{2}\) = \(\frac{9}{2}\)

Question 5.
4 \(\frac{1}{8}\)
\(\frac{□}{□}\)

Answer: \(\frac{33}{8}\)

Explanation:
\(\frac{8}{8}\) + \(\frac{8}{8}\) + \(\frac{8}{8}\) + \(\frac{8}{8}\) + \(\frac{1}{8}\) = \(\frac{33}{8}\)

Question 6.
1 \(\frac{7}{10}\)
\(\frac{□}{□}\)

Answer: \(\frac{17}{10}\)

Explanation:
\(\frac{10}{10}\) + \(\frac{7}{10}\) = \(\frac{17}{10}\)

Question 7.
5 \(\frac{1}{2}\)
\(\frac{□}{□}\)

Answer: \(\frac{11}{2}\)

Explanation:
\(\frac{2}{2}\) + \(\frac{2}{2}\) + \(\frac{2}{2}\) + \(\frac{2}{2}\) + \(\frac{2}{2}\) + \(\frac{1}{2}\) = \(\frac{11}{2}\)

Question 8.
2 \(\frac{3}{8}\)
\(\frac{□}{□}\)

Answer: \(\frac{19}{8}\)

Explanation:
\(\frac{8}{8}\) + \(\frac{8}{8}\) + \(\frac{3}{8}\)

Write the fraction as a mixed number.

Question 9.
\(\frac{31}{6}\)
______ \(\frac{□}{□}\)

Answer: 5 \(\frac{1}{6}\)

Explanation:
\(\frac{6}{6}\) + \(\frac{6}{6}\) + \(\frac{6}{6}\) + \(\frac{6}{6}\) + \(\frac{6}{6}\) + \(\frac{1}{6}\)
1 + 1 + 1 + 1 + 1 + \(\frac{1}{6}\) = 5 \(\frac{1}{6}\)

Question 10.
\(\frac{20}{10}\)
______ \(\frac{□}{□}\)

Answer: 2

Explanation:
\(\frac{10}{10}\) + \(\frac{10}{10}\) = 1 + 1 = 2

Question 11.
\(\frac{15}{8}\)
______ \(\frac{□}{□}\)

Answer: 1 \(\frac{7}{8}\)

Explanation:
\(\frac{8}{8}\) + \(\frac{7}{8}\)
1 + \(\frac{7}{8}\) = 1 \(\frac{7}{8}\)

Question 12.
\(\frac{13}{6}\)
______ \(\frac{□}{□}\)

Answer: 2 \(\frac{1}{6}\)

Explanation:
\(\frac{6}{6}\) + \(\frac{6}{6}\) + \(\frac{1}{6}\)
= 1 + 1 + \(\frac{1}{6}\) = 2 \(\frac{1}{6}\)

Question 13.
\(\frac{23}{10}\)
______ \(\frac{□}{□}\)

Answer: 2 \(\frac{3}{10}\)

Explanation:
\(\frac{10}{10}\) + \(\frac{10}{10}\) + \(\frac{3}{10}\)
1 + 1 + \(\frac{3}{10}\) = 2 \(\frac{3}{10}\)

Question 14.
\(\frac{19}{5}\)
______ \(\frac{□}{□}\)

Answer: 3 \(\frac{4}{5}\)

Explanation:
\(\frac{5}{5}\) + \(\frac{5}{5}\) + \(\frac{5}{5}\) + \(\frac{4}{5}\)
1 + 1 + 1 + \(\frac{4}{5}\) = 3 \(\frac{4}{5}\)

Question 15.
\(\frac{11}{3}\)
______ \(\frac{□}{□}\)

Answer: 3 \(\frac{2}{3}\)

Explanation:
\(\frac{3}{3}\) + \(\frac{3}{3}\) + \(\frac{3}{3}\) + \(\frac{2}{3}\)
= 1 + 1 + 1 \(\frac{2}{3}\)
= 3 \(\frac{2}{3}\)

Question 16.
\(\frac{9}{2}\)
______ \(\frac{□}{□}\)

Answer: 4 \(\frac{1}{2}\)

Explanation:
\(\frac{2}{2}\) + \(\frac{2}{2}\) + \(\frac{2}{2}\) + \(\frac{2}{2}\) + \(\frac{1}{2}\)
= 1 + 1 + 1 + 1 + \(\frac{1}{2}\)
= 4 \(\frac{1}{2}\)

Question 17.
A recipe calls for 2 \(\frac{2}{4}\) cups of raisins, but Julie only has a \(\frac{1}{4}\) -cup measuring cup. How many \(\frac{1}{4}\) cups does Julie need to measure out 2 \(\frac{2}{4}\) cups of raisins?
She needs ______ \(\frac{1}{4}\) cups

Answer: 10 \(\frac{1}{4}\) cups

Explanation:
Given,
A recipe calls for 2 \(\frac{2}{4}\) cups of raisins, but Julie only has a \(\frac{1}{4}\) -cup measuring cup.
\(\frac{4}{4}\) + \(\frac{4}{4}\) + \(\frac{1}{4}\) + \(\frac{1}{4}\)
= 10 \(\frac{1}{4}\) cups

Question 18.
If Julie needs 3 \(\frac{1}{4}\) cups of oatmeal, how many \(\frac{1}{4}\) cups of oatmeal will she use?
She will use ______ \(\frac{1}{4}\) cups of oatmeal

Answer: 13 \(\frac{1}{4}\) cups of oatmeal

Explanation:
\(\frac{4}{4}\) + \(\frac{4}{4}\) + \(\frac{1}{4}\) + \(\frac{1}{4}\) + \(\frac{1}{4}\)
= 13 \(\frac{1}{4}\)
Therefore Julie needs 13 \(\frac{1}{4}\) cups of oatmeal.

Rename Fractions and Mixed Numbers – Lesson Check – Page No. 422

Question 1.
Which of the following is equivalent to \(\frac{16}{3}\) ?
Options:
a. 3 \(\frac{1}{5}\)
b. 3 \(\frac{2}{5}\)
c. 5 \(\frac{1}{3}\)
d. 5 \(\frac{6}{3}\)

Answer: 5 \(\frac{1}{3}\)

Explanation:
Convert from improper fraction to the mixed fraction.
\(\frac{16}{3}\) = \(\frac{3}{3}\) + \(\frac{3}{3}\) + \(\frac{3}{3}\) + \(\frac{3}{3}\) + \(\frac{3}{3}\) + \(\frac{1}{3}\)
= 5 \(\frac{1}{3}\)
Thus the correct answer is option c.

Question 2.
Stacey filled her \(\frac{1}{2}\)cup measuring cup seven times to have enough flour for a cake recipe. How much flour does the cake recipe call for?
Options:
a. 3 cups
b. 3 \(\frac{1}{2}\) cups
c. 4 cups
d. 4 \(\frac{1}{2}\) cups

Answer: 3 \(\frac{1}{2}\) cups

Explanation:
Given,
Stacey filled her \(\frac{1}{2}\)cup measuring cup seven times to have enough flour for a cake recipe.
\(\frac{2}{2}\) + \(\frac{2}{2}\) + \(\frac{2}{2}\) + \(\frac{1}{2}\)
1 + 1 + 1 + \(\frac{1}{2}\)
= 3 \(\frac{1}{2}\) cups
Thus the correct answer is option b.

Spiral Review

Question 3.
Becki put some stamps into her stamp collection book. She put 14 stamps on each page. If she completely filled 16 pages, how many stamps did she put in the book?
Options:
a. 224
b. 240
c. 272
d. 275

Answer: 224

Explanation:
Becki put some stamps into her stamp collection book.
She put 14 stamps on each page.
If she completely filled 16 pages
Multiply 14 with 16 pages.
14 × 16 = 224 pages
Thus the correct answer is option a.

Question 4.
Brian is driving 324 miles to visit some friends. He wants to get there in 6 hours. How many miles does he need to drive each hour?
Options:
a. 48 miles
b. 50 miles
c. 52 miles
d. 54 miles

Answer: 54 miles

Explanation:
Brian is driving 324 miles to visit some friends. He wants to get there in 6 hours.
Divide the number of miles by hours.
324/6 = 54 miles
Thus the correct answer is option d.

Question 5.
During a bike challenge, riders have to collect various colored ribbons. Each \(\frac{1}{2}\) mile they collect a red ribbon, each \(\frac{1}{8}\) mile they collect a green ribbon, and each \(\frac{1}{4}\) mile they collect a blue ribbon. Which colors of ribbons will be collected at the \(\frac{3}{4}\) mile marker?
Options:
a. red and green
b. red and blue
c. green and blue
d. red, green, and blue

Answer: green and blue

Explanation:
Given,
During a bike challenge, riders have to collect various colored ribbons.
Each \(\frac{1}{2}\) mile they collect a red ribbon, each \(\frac{1}{8}\) mile they collect a green ribbon, and each \(\frac{1}{4}\) mile they collect a blue ribbon.
Green and Blue colors of ribbons will be collected at the \(\frac{3}{4}\) mile marker.
Thus the correct answer is option c.

Question 6.
Stephanie had \(\frac{7}{8}\) pound of bird seed. She used \(\frac{3}{8}\) pound to fill a bird feeder. How much bird seed does Stephanie have left?
Options:
a. \(\frac{3}{8}\) pound
b. \(\frac{4}{8}\) pound
c. 1 pound
d. \(\frac{10}{8}\) pound

Answer: \(\frac{4}{8}\) pound

Explanation:
Given,
Stephanie had \(\frac{7}{8}\) pound of bird seed.
She used \(\frac{3}{8}\) pound to fill a bird feeder.
\(\frac{7}{8}\) – \(\frac{3}{8}\) = \(\frac{4}{8}\) pound
Thus the correct answer is option b.

Rename Fractions and Mixed Numbers – Page No. 425

Write the sum as a mixed number with the fractional part less than 1.

Question 1.
1 \(\frac{1}{6}\)
+3 \(\frac{3}{6}\)
———————–
_______ \(\frac{□}{□}\)

Answer: 4 \(\frac{2}{3}\)

Explanation:
1 \(\frac{1}{6}\)
+3 \(\frac{3}{6}\)
4 \(\frac{4}{6}\) = 4 \(\frac{2}{3}\)

Question 2.
1 \(\frac{4}{5}\)
+7 \(\frac{2}{5}\)
———————–
_______ \(\frac{□}{□}\)

Answer: 9 \(\frac{1}{5}\)

Explanation:
1 \(\frac{4}{5}\)
+7 \(\frac{2}{5}\)
8 \(\frac{6}{5}\) = 9 \(\frac{1}{5}\)

Question 3.
2 \(\frac{1}{2}\)
+3 \(\frac{1}{2}\)
———————–
_______

Answer: 6

Explanation:
2 \(\frac{1}{2}\)
+3 \(\frac{1}{2}\)
5 \(\frac{2}{2}\) = 6

Find the difference.

Question 4.
3 \(\frac{7}{12}\)
-2 \(\frac{5}{12}\)
———————–
_______ \(\frac{□}{□}\)

Answer: 1 \(\frac{1}{6}\)

Explanation:
3 \(\frac{7}{12}\)
-2 \(\frac{5}{12}\)
1 \(\frac{2}{12}\) = 1 \(\frac{1}{6}\)

Question 5.
4 \(\frac{2}{3}\)
-3 \(\frac{1}{3}\)
———————–
_______ \(\frac{□}{□}\)

Answer: 1 \(\frac{1}{3}\)

Explanation:
4 \(\frac{2}{3}\)
-3 \(\frac{1}{3}\)
1 \(\frac{1}{3}\)

Question 6.
6 \(\frac{9}{10}\)
-3 \(\frac{7}{10}\)
———————–
_______ \(\frac{□}{□}\)

Answer: 3 \(\frac{1}{5}\)

Explanation:
6 \(\frac{9}{10}\)
-3 \(\frac{7}{10}\)
3 \(\frac{2}{10}\)

Write the sum as a mixed number with the fractional part less than 1.

Question 7.
7 \(\frac{4}{6}\)
+4 \(\frac{3}{6}\)
———————–
_______ \(\frac{□}{□}\)

Answer: 12 \(\frac{1}{6}\)

Explanation:
7 \(\frac{4}{6}\)
+4 \(\frac{3}{6}\)
12 \(\frac{1}{6}\)

Question 8.
8 \(\frac{1}{3}\)
+3 \(\frac{2}{3}\)
———————–
_______ \(\frac{□}{□}\)

Answer: 12

Explanation:
8 \(\frac{1}{3}\)
+3 \(\frac{2}{3}\)
11 \(\frac{3}{3}\) = 12

Question 9.
5 \(\frac{4}{8}\)
+3 \(\frac{5}{8}\)
———————–
_______ \(\frac{□}{□}\)

Answer: 9 \(\frac{1}{8}\)

Explanation:
5 \(\frac{4}{8}\)
+3 \(\frac{5}{8}\)
9 \(\frac{1}{8}\)

Question 10.
5 \(\frac{5}{12}\)
+4 \(\frac{2}{12}\)
———————–
_______ \(\frac{□}{□}\)

Answer: 9 \(\frac{7}{12}\)

Explanation:
5 \(\frac{5}{12}\)
+4 \(\frac{2}{12}\)
9 \(\frac{7}{12}\)

Find the difference.

Question 11.
5 \(\frac{7}{8}\)
-2 \(\frac{3}{8}\)
———————–
_______ \(\frac{□}{□}\)

Answer: 3 \(\frac{1}{2}\)

Explanation:
5 \(\frac{7}{8}\)
-2 \(\frac{3}{8}\)
3 \(\frac{1}{2}\)

Question 12.
5 \(\frac{7}{12}\)
-4 \(\frac{1}{12}\)
———————–
_______ \(\frac{□}{□}\)

Answer: 1 \(\frac{1}{2}\)

Explanation:
5 \(\frac{7}{12}\)
-4 \(\frac{1}{12}\)
1 \(\frac{1}{2}\)

Question 13.
3 \(\frac{5}{10}\)
-1 \(\frac{3}{10}\)
———————–
_______ \(\frac{□}{□}\)

Answer: 2 \(\frac{1}{5}\)

Explanation:
3 \(\frac{5}{10}\)
-1 \(\frac{3}{10}\)
2 \(\frac{1}{5}\)

Question 14.
7 \(\frac{3}{4}\)
-2 \(\frac{2}{4}\)
———————–
_______ \(\frac{□}{□}\)

Answer: 5 \(\frac{1}{4}\)

Explanation:
7 \(\frac{3}{4}\)
-2 \(\frac{2}{4}\)
5 \(\frac{1}{4}\)

Practice: Copy and Solve Find the sum or difference.

Question 15.
\(1 \frac{3}{8}+2 \frac{7}{8}\) = _______ \(\frac{□}{□}\)

Answer: 4 \(\frac{1}{4}\)

Explanation:
First add the whole numbers
1 + 2 = 3
3/8 + 7/8 = 10/8
Convert from improper fraction to the mixed fraction
10/8 = 5/4 = 1 1/4
3 + 1 1/4 = 4 1/4

Question 16.
\(6 \frac{5}{8}\) – 4 = _______ \(\frac{□}{□}\)

Answer: 2 \(\frac{5}{8}\)

Explanation:
\(6 \frac{5}{8}\) – 4
Subtract the whole numbers
6 – 4 = 2
= 2 \(\frac{5}{8}\)

Question 17.
\(9 \frac{1}{2}+8 \frac{1}{2}\) = _______

Answer: 18

Explanation:
9 \(\frac{1}{2}\)
+ 8 \(\frac{1}{2}\)
18

Question 18.
\(6 \frac{3}{5}+4 \frac{3}{5}\) = _______ \(\frac{□}{□}\)

Answer: 11 \(\frac{1}{5}\)

Explanation:
6 \(\frac{3}{5}\)
+ 4 \(\frac{3}{5}\)
11 \(\frac{1}{5}\)

Question 19.
\(8 \frac{7}{10}-\frac{4}{10}\) = _______ \(\frac{□}{□}\)

Answer: 8 \(\frac{3}{10}\)

Explanation:
8 \(\frac{7}{10}\)
 – \(\frac{4}{10}\)
8 \(\frac{3}{10}\)

Question 20.
\(7 \frac{3}{5}-6 \frac{3}{5}\) = _______

Answer: 1

Explanation:
7 \(\frac{3}{5}\)
+ 6 \(\frac{3}{5}\)
1

Rename Fractions and Mixed Numbers – Page No. 426

Solve. Write your answer as a mixed number.

Question 21.
Make Sense of Problems The driving distance from Alex’s house to the museum is 6 \(\frac{7}{10}\) miles. What is the round-trip distance?
_______ \(\frac{□}{□}\) miles

Answer: 13 \(\frac{2}{5}\) miles

Explanation:
Given that,
The driving distance from Alex’s house to the museum is 6 \(\frac{7}{10}\) miles.
To find the round-trip distance we have to multiply the driving distance with 2.
6 \(\frac{7}{10}\) × 2 = 13 \(\frac{4}{10}\)
= 13 \(\frac{2}{5}\) miles

Question 22.
The driving distance from the sports arena to Kristina’s house is 10 \(\frac{9}{10}\) miles. The distance from the sports arena to Luke’s house is 2 \(\frac{7}{10}\) miles. How much greater is the driving distance between the sports arena and Kristina’s house than between the sports arena and Luke’s house?
_______ \(\frac{□}{□}\) miles

Answer: 8 \(\frac{1}{5}\) miles

Explanation:
Given,
The driving distance from the sports arena to Kristina’s house is 10 \(\frac{9}{10}\) miles.
The distance from the sports arena to Luke’s house is 2 \(\frac{7}{10}\) miles.
10 \(\frac{9}{10}\) –  2 \(\frac{7}{10}\)
First, subtract the whole numbers and then subtract the fractions
10 – 2 = 8
\(\frac{9}{10}\) – \(\frac{7}{10}\) = \(\frac{1}{5}\)
= 8 \(\frac{1}{5}\) miles

Question 23.
Pedro biked from his house to the nature preserve, a distance of 23 \(\frac{4}{5}\) miles. Sandra biked from her house to the lake, a distance of 12 \(\frac{2}{5}\) miles. How many miles less did Sandra bike than Pedro?
_______ \(\frac{□}{□}\) miles

Answer: 11 \(\frac{2}{5}\) miles

Explanation:
Pedro biked from his house to the nature preserve, a distance of 23 4/5 miles. Converting 23 4/5 miles to an improper fraction, it becomes 119/5 miles.
Sandra biked from her house to the lake, a distance of 12 2/5 miles.
Converting 12 2/5 miles to an improper fraction, it becomes 62/5 miles.
Therefore, the difference in the number of miles biked by Sandra and Pedro is
119/5 – 62/5 = 57/5 = 11 2/5 miles

Question 24.
During the Martinez family trip, they drove from home to a ski lodge, a distance of 55 \(\frac{4}{5}\) miles, and then drove an additional 12 \(\frac{4}{5}\) miles to visit friends. If the family drove the same route back home, what was the distance traveled during their trip?
_______ \(\frac{□}{□}\) miles

Answer: 68 \(\frac{3}{5}\) miles

Explanation:
Given,
During the Martinez family trip, they drove from home to a ski lodge, a distance of 55 \(\frac{4}{5}\) miles, and then drove an additional 12 \(\frac{4}{5}\) miles to visit friends.
55 \(\frac{4}{5}\) + 12 \(\frac{4}{5}\) = 67 \(\frac{8}{5}\) = 68 \(\frac{3}{5}\) miles

Question 25.
For 25a–25d, select True or False for each statement.
a. 2 \(\frac{3}{8}\) + 1 \(\frac{6}{8}\) is equal to 4 \(\frac{1}{8}\).
i. True
ii. False

Answer: True

Explanation:
Given the statement 2 \(\frac{3}{8}\) + 1 \(\frac{6}{8}\) is equal to 4 \(\frac{1}{8}\).
First add the whole numbers
2 + 1 = 3
\(\frac{3}{8}\) + \(\frac{6}{8}\) = \(\frac{9}{8}\)
Convert the improper fraction to the mixed fraction.
\(\frac{9}{8}\) = 1 \(\frac{1}{8}\)
3 +1 \(\frac{1}{8}\) = 4 \(\frac{1}{8}\).
Thus the above statement is true.

Question 25.
b. 1 \(\frac{1}{6}\) + 1 \(\frac{4}{12}\) is equal to 2 \(\frac{2}{12}\).
i. True
ii. False

Answer: False

Explanation:
1 \(\frac{1}{6}\) + 1 \(\frac{4}{12}\) is equal to 2 \(\frac{2}{12}\).
First add the whole numbers
1 + 1 = 2
\(\frac{1}{6}\) = \(\frac{2}{12}\)

\(\frac{2}{12}\) + \(\frac{4}{12}\) = \(\frac{6}{12}\)
= 2 \(\frac{6}{12}\)
Thus the above statement is false.

Question 25.
c. 5 \(\frac{5}{6}\) – 2 \(\frac{4}{6}\) is equal to 1 \(\frac{3}{6}\).
i. True
ii. False

Answer: False

Explanation:
5 \(\frac{5}{6}\) – 2 \(\frac{4}{6}\) is equal to 1 \(\frac{3}{6}\).
5 – 2 = 3
\(\frac{5}{6}\) – \(\frac{4}{6}\) = \(\frac{1}{6}\)
= 3 \(\frac{1}{6}\)
Thus the above statement is false.

Question 25.
d. 5 \(\frac{5}{8}\) – 3 \(\frac{2}{8}\) is equal to 2 \(\frac{3}{8}\).
i. True
ii. False

Answer: True

Explanation:
5 \(\frac{5}{8}\) – 3 \(\frac{2}{8}\) is equal to 2 \(\frac{3}{8}\)
First, subtract the whole numbers
5 – 3 = 2
\(\frac{5}{8}\) – \(\frac{2}{8}\) = \(\frac{3}{8}\)
= 2 \(\frac{3}{8}\)
Thus the above statement is true.

Add and Subtract Mixed Numbers – Page No. 427

Find the sum. Write the sum as a mixed number, so the fractional part is less than 1.

Question 1.
Go Math Grade 4 Answer Key Chapter 7 Add and Subtract Fractions Common Core - New Page No. 427 Q 1

Question 2.
4 \(\frac{1}{2}\)
+2 \(\frac{1}{2}\)
_______ \(\frac{□}{□}\)

Answer: 7

4 \(\frac{1}{2}\)
+2 \(\frac{1}{2}\)
6 \(\frac{2}{2}\) = 6 + 1 = 7

Question 3.
2 \(\frac{2}{3}\)
+3 \(\frac{2}{3}\)
_______ \(\frac{□}{□}\)

Answer: 6 \(\frac{1}{3}\)

Explanation:
2 \(\frac{2}{3}\)
+3 \(\frac{2}{3}\)
5 \(\frac{4}{3}\)
= 5 + 1 \(\frac{1}{3}\)
= 6 \(\frac{1}{3}\)

Question 4.
6 \(\frac{4}{5}\)
+7 \(\frac{4}{5}\)
_______ \(\frac{□}{□}\)

Answer: 14 \(\frac{3}{5}\)

Explanation:
6 \(\frac{4}{5}\)
+7 \(\frac{4}{5}\)
13 \(\frac{8}{5}\)
13 + 1 \(\frac{3}{5}\)
= 14 \(\frac{3}{5}\)

Question 5.
9 \(\frac{3}{6}\)
+2 \(\frac{2}{6}\)
_______ \(\frac{□}{□}\)

Answer: 11 \(\frac{5}{6}\)

Explanation:
9 \(\frac{3}{6}\)
+2 \(\frac{2}{6}\)
11 \(\frac{5}{6}\)

Question 6.
8 \(\frac{4}{12}\)
+3 \(\frac{6}{12}\)
_______ \(\frac{□}{□}\)

Answer: 11 \(\frac{10}{12}\)

Explanation:
8 \(\frac{4}{12}\)
+3 \(\frac{6}{12}\)
11 \(\frac{10}{12}\)

Question 7.
4 \(\frac{3}{8}\)
+1 \(\frac{5}{8}\)
_______ \(\frac{□}{□}\)

Answer: 6

Explanation:
4 \(\frac{3}{8}\)
+1 \(\frac{5}{8}\)
5 \(\frac{8}{8}\)
= 5 + 1 = 6

Question 8.
9 \(\frac{5}{10}\)
+6 \(\frac{3}{10}\)
_______ \(\frac{□}{□}\)

Answer: 15 \(\frac{8}{10}\)

Explanation:
9 \(\frac{5}{10}\)
+6 \(\frac{3}{10}\)
15 \(\frac{8}{10}\)

Find the difference.

Question 9.
6 \(\frac{7}{8}\)
-4 \(\frac{3}{8}\)
_______ \(\frac{□}{□}\)

Answer: 2 \(\frac{4}{8}\)

Explanation:
6 \(\frac{7}{8}\)
-4 \(\frac{3}{8}\)
2 \(\frac{4}{8}\)

Question 10.
4 \(\frac{2}{3}\)
-3 \(\frac{1}{3}\)
_______ \(\frac{□}{□}\)

Answer: 1 \(\frac{1}{3}\)

Explanation:
4 \(\frac{2}{3}\)
-3 \(\frac{1}{3}\)
1 \(\frac{1}{3}\)

Question 11.
6 \(\frac{4}{5}\)
-3 \(\frac{3}{5}\)
_______ \(\frac{□}{□}\)

Answer: 3 \(\frac{1}{5}\)

Explanation:
6 \(\frac{4}{5}\)
-3 \(\frac{3}{5}\)
3 \(\frac{1}{5}\)

Question 12.
7 \(\frac{3}{4}\)
-2 \(\frac{1}{4}\)
_______ \(\frac{□}{□}\)

Answer: 5 \(\frac{1}{2}\)

Explanation:
7 \(\frac{3}{4}\)
-2 \(\frac{1}{4}\)
5 \(\frac{2}{4}\) = 5 \(\frac{1}{2}\)

Problem Solving

Question 13.
James wants to send two gifts by mail. One package weighs 2 \(\frac{3}{4}\) pounds. The other package weighs 1 \(\frac{3}{4}\) pounds. What is the total weight of the packages?
_______ \(\frac{□}{□}\)

Answer: 4 \(\frac{1}{2}\)

Explanation:
2 \(\frac{3}{4}\)
+ 1 \(\frac{3}{4}\)
4 \(\frac{1}{2}\)

Question 14.
Tierra bought 4 \(\frac{3}{8}\) yards blue ribbon and 2 \(\frac{1}{8}\) yards yellow ribbon for a craft project. How much more blue ribbon than yellow ribbon did Tierra buy?
_______ \(\frac{□}{□}\)

Answer: 2 \(\frac{1}{4}\)

Explanation:
Given,
4 \(\frac{3}{8}\)
-2 \(\frac{1}{8}\) 
2 \(\frac{1}{4}\)

Add and Subtract Mixed Numbers – Lesson Check – Page No. 428

Question 1.
Brad has two lengths of copper pipe to fit together. One has a length of 2 \(\frac{5}{12}\) feet and the other has a length of 3 \(\frac{7}{12}\) feet. How many feet of pipe does he have in all?
Options:
a. 5 feet
b. 5 \(\frac{6}{12}\) feet
c. 5 \(\frac{10}{12}\) feet
d. 6 feet

Answer: 5 feet

Explanation:
Given,
Brad has two lengths of copper pipe to fit together. One has a length of 2 \(\frac{5}{12}\) feet and the other has a length of 3 \(\frac{7}{12}\) feet.
Add both the lengths
2 \(\frac{5}{12}\) + 3 \(\frac{7}{12}\)
= 5 \(\frac{12}{12}\) = 5 feet
Thus the correct answer is option a.

Question 2.
A pattern calls for 2 \(\frac{1}{4}\) yards of material and 1 \(\frac{1}{4}\) yards of lining. How much total fabric is needed?
Options:
a. 2 \(\frac{2}{4}\) yards
b. 3 yards
c. 3 \(\frac{1}{4}\) yards
d. 3 \(\frac{2}{4}\) yards

Answer: 3 \(\frac{2}{4}\) yards

Explanation:
Given,
A pattern calls for 2 \(\frac{1}{4}\) yards of material and 1 \(\frac{1}{4}\) yards of lining.
2 \(\frac{1}{4}\) + 1 \(\frac{1}{4}\)
= 3 + \(\frac{1}{4}\) + \(\frac{1}{4}\)
= 3 \(\frac{2}{4}\) yards
Thus the correct answer is option d.

Spiral Review

Question 3.
Shanice has 23 baseball trading cards of star players. She agrees to sell them for $16 each. How much will she get for the cards?
Options:
a. $258
b. $358
c. $368
d. $468

Answer: $368

Explanation:
Given,
Shanice has 23 baseball trading cards of star players. She agrees to sell them for $16 each.
To find how much will she get for the cards
23 × 16 = 368
Therefore she will get $368 for the cards.
Thus the correct answer is option c.

Question 4.
Nanci is volunteering at the animal shelter. She wants to spend an equal amount of time playing with each dog. She has 145 minutes to play with all 7 dogs. About how much time can she spend with each dog?
Options:
a. about 10 minutes
b. about 20 minutes
c. about 25 minutes
d. about 26 minutes

Answer: about 20 minutes

Explanation:
Given,
Nanci is volunteering at the animal shelter. She wants to spend an equal amount of time playing with each dog. She has 145 minutes to play with all 7 dogs.
145/7 = 20.7
Therefore she can spend about 20 minutes with each dog.
Thus the correct answer is option b.

Question 5.
Frieda has 12 red apples and 15 green apples. She is going to share the apples equally among 8 people and keep any extra apples for herself. How many apples will Frieda keep for herself?
Options:
a. 3
b. 4
c. 6
d. 7

Answer: 3

Explanation:
Given,
Frieda has 12 red apples and 15 green apples.
She is going to share the apples equally among 8 people and keep any extra apples for herself.
12 + 15 = 27
27/8
27 – 24 = 3
Thus Frieda keep for herself 3 apples.
Thus the correct answer is option a.

Question 6.
The Lynch family bought a house for $75,300. A few years later, they sold the house for $80,250. How much greater was the selling price than the purchase price?
Options:
a. $4,950
b. $5,050
c. $5,150
d. $5,950

Answer: $4,950

Explanation:
Given,
The Lynch family bought a house for $75,300.
A few years later, they sold the house for $80,250.
$80,250 – $75,300 = $4,950
Thus the correct answer is option a.

Add and Subtract Mixed Numbers – Page No. 431

Question 1.
Rename both mixed numbers as fractions. Find the difference.
3 \(\frac{3}{6}\) = \(\frac{■}{6}\)
−1 \(\frac{4}{6}\) = – \(\frac{■}{6}\)
—————————————-
_______ \(\frac{□}{□}\)

Answer: 1 \(\frac{5}{6}\)

Explanation:
Convert from mixed fractions to the improper fractions.
3 \(\frac{3}{6}\) = \(\frac{21}{6}\)
1 \(\frac{4}{6}\) = \(\frac{10}{6}\)
\(\frac{21}{6}\)
– \(\frac{10}{6}\)
\(\frac{11}{6}\)
Convert from improper fractions to the mixed fractions.
\(\frac{11}{6}\) = 1 \(\frac{5}{6}\)

Find the difference.

Question 2.
1 \(\frac{1}{3}\)
− \(\frac{2}{3}\)
———————
\(\frac{□}{□}\)

Answer: \(\frac{2}{3}\)

Explanation:
Convert from mixed fractions to improper fractions.
1 \(\frac{1}{3}\) = \(\frac{4}{3}\)
\(\frac{4}{3}\)
– \(\frac{2}{3}\)
\(\frac{2}{3}\)

Question 3.
4 \(\frac{7}{10}\)
− 1 \(\frac{9}{10}\)
———————
______ \(\frac{□}{□}\)

Answer: 2 \(\frac{8}{10}\)

Explanation:
Convert from mixed fractions to improper fractions.
4 \(\frac{7}{10}\) = \(\frac{47}{10}\)
1 \(\frac{9}{10}\) = \(\frac{19}{10}\)
\(\frac{47}{10}\)
– \(\frac{19}{10}\)
\(\frac{28}{10}\) = 2 \(\frac{8}{10}\)

Question 4.
3 \(\frac{5}{12}\)
− \(\frac{8}{12}\)
———————
_____ \(\frac{□}{□}\)

Answer: 2 \(\frac{9}{12}\)

Explanation:
Convert from mixed fractions to improper fractions.
3 \(\frac{5}{12}\) = \(\frac{41}{12}\)
\(\frac{41}{12}\)
− \(\frac{8}{12}\)
2 \(\frac{9}{12}\)

Question 5.
8 \(\frac{1}{10}\)
− 2 \(\frac{9}{10}\)
———————
\(\frac{□}{□}\)

Answer: 5 \(\frac{1}{5}\)

Explanation:
Convert from mixed fractions to improper fractions.
8 \(\frac{1}{10}\) = \(\frac{81}{10}\)
2 \(\frac{9}{10}\) = \(\frac{29}{10}\)
\(\frac{81}{10}\)
–\(\frac{29}{10}\)
\(\frac{52}{10}\) = 5 \(\frac{1}{5}\)

Question 6.
2
− 1 \(\frac{1}{4}\)
———————
\(\frac{□}{□}\)

Answer: \(\frac{3}{4}\)

Explanation:
Convert from mixed fractions to improper fractions.
1 \(\frac{1}{4}\) = \(\frac{5}{4}\)
2
− 1 \(\frac{1}{4}\)
\(\frac{3}{4}\)

Question 7.
4 \(\frac{1}{5}\)
− 3 \(\frac{2}{5}\)
———————
\(\frac{□}{□}\)

Answer: \(\frac{4}{5}\)

Explanation:
Convert from mixed fractions to improper fractions.
4 \(\frac{1}{5}\) = \(\frac{21}{5}\)
3 \(\frac{2}{5}\) = \(\frac{17}{5}\)
\(\frac{21}{5}\)
–\(\frac{17}{5}\)
\(\frac{4}{5}\)

Practice: Copy and Solve Find the difference.

Question 8.
\(4 \frac{1}{6}-2 \frac{5}{6}\)
_____ \(\frac{□}{□}\)

Answer: 1 \(\frac{1}{3}\)

Explanation:
Convert from mixed fractions to improper fractions.
4 \(\frac{1}{6}\) = \(\frac{25}{6}\)
2 \(\frac{5}{6}\) = \(\frac{17}{6}\)
\(\frac{25}{6}\)
–\(\frac{17}{6}\)
\(\frac{8}{6}\) = 1 \(\frac{1}{3}\)

Question 9.
\(6 \frac{9}{12}-3 \frac{10}{12}\)
_____ \(\frac{□}{□}\)

Answer: 2 \(\frac{11}{12}\)

Explanation:
Convert from mixed fractions to improper fractions.
6 \(\frac{9}{12}\)
– 3 \(\frac{10}{12}\)
2 \(\frac{11}{12}\)

Question 10.
\(3 \frac{3}{10}-\frac{7}{10}\)
_____ \(\frac{□}{□}\)

Answer: 2 \(\frac{3}{5}\)

Explanation:
Convert from mixed fractions to improper fractions.
3 \(\frac{3}{10}\) = \(\frac{33}{10}\)
\(\frac{33}{10}\)
– \(\frac{7}{10}\)
2 \(\frac{3}{5}\)

Question 11.
4 – 2 \(\frac{3}{5}\)
_____ \(\frac{□}{□}\)

Answer: 1 \(\frac{2}{5}\)

Explanation:
Convert from mixed fractions to improper fractions.
2 \(\frac{3}{5}\) = \(\frac{13}{5}\)
4
–\(\frac{13}{5}\) 
1 \(\frac{2}{5}\)

Question 12.
Lisa mixed 4 \(\frac{2}{6}\) cups of orange juice with 3 \(\frac{1}{6}\) cups of pineapple juice to make fruit punch. She and her friends drank 3 \(\frac{4}{6}\) cups of the punch. How much of the fruit punch is left?
_____ \(\frac{□}{□}\) cups

Answer: 3 \(\frac{5}{6}\) cups

Explanation:
Given,
Lisa mixed 4 \(\frac{2}{6}\) cups of orange juice with 3 \(\frac{1}{6}\) cups of pineapple juice to make fruit punch.
She and her friends drank 3 \(\frac{4}{6}\) cups of the punch.
Convert from mixed fractions to improper fractions.
4 \(\frac{2}{6}\)
+ 3 \(\frac{1}{6}\)
7 \(\frac{3}{6}\)
Now subtract 3 \(\frac{4}{6}\) from 7 \(\frac{3}{6}\).
7 \(\frac{3}{6}\)
-3 \(\frac{4}{6}\)
3 \(\frac{5}{6}\)

Add and Subtract Mixed Numbers – Page No. 432

Rename the fractions to solve.

Many instruments are coiled or curved so that they are easier for the musician to play, but they would be quite long if straightened out completely.
Go Math Grade 4 Answer Key Chapter 7 Add and Subtract Fractions Page No. 432 Q 13

Question 13.
Analyze Relationships Trumpets and cornets are brass instruments. Fully stretched out, the length of a trumpet is 5 \(\frac{1}{4}\) feet and the length of a cornet is 4 \(\frac{2}{4}\) feet. The trumpet is how much longer than the cornet?
\(\frac{□}{□}\) feet

Answer: \(\frac{3}{4}\) feet

Explanation:
Given,
Trumpets and cornets are brass instruments. Fully stretched out, the length of a trumpet is 5 \(\frac{1}{4}\) feet and the length of a cornet is 4 \(\frac{2}{4}\) feet.
5 \(\frac{1}{4}\) – 4 \(\frac{2}{4}\)
First subtract the whole numbers
5 – 4 = 1
\(\frac{1}{4}\) – \(\frac{2}{4}\) = \(\frac{1}{4}\)
1 – \(\frac{1}{4}\) = \(\frac{3}{4}\) feet
Therefore trumpet is \(\frac{3}{4}\) feet longer than the cornet.

Question 14.
Tubas, trombones, and French horns are brass instruments. Fully stretched out, the length of a tuba is 18 feet, the length of a trombone is 9 \(\frac{11}{12}\) feet, and the length of a French horn is 17 \(\frac{1}{12}\) feet. The tuba is how much longer than the French horn? The French horn is how much longer than the trombone?
Type below:
_____________

Answer:
First, convert the fractions to decimals making the trombone 8.93 feet and the french horn 17.21 feet. The tuba would be 0.79 feet longer than the french horn, and the french horn would be 8.23 feet longer than the trombone. However, if you need the answer to remain a fraction, the tuba would be 11/14 feet longer than a french horn, and a french horn would be 8 3/14 feet longer than a trombone.

Question 15.
The pitch of a musical instrument is related to its length. In general, the greater the length of a musical instrument, the lower its pitch. Order the brass instruments identified on this page from lowest pitch to the highest pitch.
____________
____________
____________

Answer:
Tuba
French Horn
Trombone

Explanation:
By seeing the above answer we can write the order of the brass instruments from the lowest pitch to the highest pitch. The order is tuba, french horn, and trombone.

Question 16.
Alicia had 3 \(\frac{1}{6}\)yards of fabric. After making a tablecloth, she had 1 \(\frac{3}{6}\) yards of fabric. Alicia said she used 2 \(\frac{3}{6}\) yards of fabric for the tablecloth. Do you agree? Explain.
______

Answer: Yes

Explanation:
An easier way to do this is to make the fractions improper fractions.
3 1/6 can be rewritten as 19/6. 1 4/6 can be rewritten as 10/6.
Multiply the denominator by the number at its side, and add it to the numerator.
2 3/6 is 15/6.
Subtract 10/6 from 19/6.
19/6-10/6=9/6.
9/6 is not 15/6, therefore she did not use 2 3/6 yards of fabric.

Record Subtraction with Renaming – Page No. 433

Find the difference.

Question 1.
Go Math Grade 4 Answer Key Chapter 7 Add and Subtract Fractions Common Core - New Page No. 433 Q 1

Question 2.
6
– 3 \(\frac{2}{5}\)
_______ \(\frac{□}{□}\)

Answer: 2 \(\frac{3}{5}\)

Explanation:
First subtract the whole numbers
6 – 3 = 3
Next subtract the fractions,
3 – \(\frac{2}{5}\) = 2 \(\frac{3}{5}\)

Question 3.
5 \(\frac{1}{4}\)
– 2 \(\frac{3}{4}\)
_______ \(\frac{□}{□}\)

Answer: 2 \(\frac{1}{2}\)

Explanation:
First subtract the whole numbers
5 – 2 = 3
Next subtract the fractions,
\(\frac{1}{4}\) – \(\frac{3}{4}\) = – \(\frac{1}{2}\)
3 – \(\frac{1}{2}\)
= 2 \(\frac{1}{2}\)

Question 4.
9 \(\frac{3}{8}\)
– 8 \(\frac{7}{8}\)
_______ \(\frac{□}{□}\)

Answer: \(\frac{1}{2}\)

Explanation:
First subtract the whole numbers
9 – 8 = 1
Next subtract the fractions,
\(\frac{3}{8}\) – \(\frac{7}{8}\)
= – \(\frac{4}{8}\)
= – \(\frac{1}{2}\)
= 1 – \(\frac{1}{2}\)
= \(\frac{1}{2}\)

Question 5.
12 \(\frac{3}{10}\)
– 7 \(\frac{7}{10}\)
_______ \(\frac{□}{□}\)

Answer: 4 \(\frac{3}{5}\)

Explanation:
First subtract the whole numbers
12 – 7 = 5
Next subtract the fractions,
\(\frac{3}{10}\) – \(\frac{7}{10}\) = – \(\frac{4}{10}\)
5 – \(\frac{4}{10}\)
5 – \(\frac{2}{5}\) = 4 \(\frac{3}{5}\)

Question 6.
8 \(\frac{1}{6}\)
– 3 \(\frac{5}{6}\)
_______ \(\frac{□}{□}\)

Answer: 4 \(\frac{1}{3}\)

Explanation:
First subtract the whole numbers
8 – 3 = 5
Next subtract the fractions,
\(\frac{1}{6}\) – \(\frac{5}{6}\) = – \(\frac{2}{3}\)
5 – \(\frac{2}{3}\) = 4 \(\frac{1}{3}\)

Question 7.
7 \(\frac{3}{5}\)
– 4 \(\frac{4}{5}\)
_______ \(\frac{□}{□}\)

Answer: 2 \(\frac{4}{5}\)

Explanation:
First subtract the whole numbers
7 – 4 = 3
Next subtract the fractions,
\(\frac{3}{5}\) – \(\frac{4}{5}\) = – \(\frac{1}{5}\)
3 – \(\frac{1}{5}\) = 2 \(\frac{4}{5}\)

Question 8.
10 \(\frac{1}{2}\)
– 8 \(\frac{1}{2}\)
_______ \(\frac{□}{□}\)

Answer: 2

Explanation:
First subtract the whole numbers
10 – 8 = 2
\(\frac{1}{2}\) – \(\frac{1}{2}\) = 0

Question 9.
7 \(\frac{1}{6}\)
– 2 \(\frac{5}{6}\)
_______ \(\frac{□}{□}\)

Answer: 4 \(\frac{1}{3}\)

Explanation:
First subtract the whole numbers
7 – 2 = 5
Next subtract the fractions,
\(\frac{1}{6}\) – \(\frac{5}{6}\) = – \(\frac{4}{6}\)
5 – \(\frac{4}{6}\) = 4 \(\frac{1}{3}\)

Question 10.
9 \(\frac{3}{12}\)
– 4 \(\frac{7}{12}\)
_______ \(\frac{□}{□}\)

Answer: 2 \(\frac{2}{3}\)

Explanation:
First subtract the whole numbers
9 – 4 = 5
Next subtract the fractions,
\(\frac{3}{12}\) – \(\frac{7}{12}\) = – \(\frac{4}{12}\) = – \(\frac{1}{3}\)
5 – \(\frac{1}{3}\) = 2 \(\frac{2}{3}\)

Question 11.
9 \(\frac{1}{10}\)
– 8 \(\frac{7}{10}\)
_______ \(\frac{□}{□}\)

Answer: \(\frac{2}{5}\)

Explanation:
First subtract the whole numbers
9 – 8 = 1
Next subtract the fractions,
\(\frac{1}{10}\) – \(\frac{7}{10}\) = – \(\frac{6}{10}\)
1 – \(\frac{3}{5}\) = \(\frac{2}{5}\)

Question 12.
9 \(\frac{1}{3}\)
– \(\frac{2}{3}\)
_______ \(\frac{□}{□}\)

Answer: 8 \(\frac{2}{3}\)

Explanation:
9 \(\frac{1}{3}\)
– \(\frac{2}{3}\)
8 \(\frac{2}{3}\)

Question 13.
3 \(\frac{1}{4}\)
– 1 \(\frac{3}{4}\)
_______ \(\frac{□}{□}\)

Answer: 1 \(\frac{1}{2}\)

3 \(\frac{1}{4}\)
– 1 \(\frac{3}{4}\)
1 \(\frac{1}{2}\)

Question 14.
4 \(\frac{5}{8}\)
– 1 \(\frac{7}{8}\)
_______ \(\frac{□}{□}\)

Answer: 2 \(\frac{3}{4}\)

Explanation:
First subtract the whole numbers
4 – 1 = 3
Next subtract the fractions,
\(\frac{5}{8}\) – \(\frac{7}{8}\) = – \(\frac{1}{4}\)
3 – \(\frac{1}{4}\) = 2 \(\frac{3}{4}\)

Question 15.
5 \(\frac{1}{12}\)
– 3 \(\frac{8}{12}\)
_______ \(\frac{□}{□}\)

Answer: 1 \(\frac{5}{12}\)

Explanation:
First subtract the whole numbers
5 – 3 = 2
Next subtract the fractions,
\(\frac{1}{12}\) – \(\frac{8}{12}\) = – \(\frac{7}{12}\)
2 – \(\frac{7}{12}\) = 1 \(\frac{5}{12}\)

Question 16.
7
– 1 \(\frac{3}{5}\)
_______ \(\frac{□}{□}\)

Answer: 5 \(\frac{2}{5}\)

Explanation:
7
– 1 \(\frac{3}{5}\)
5 \(\frac{2}{5}\)

Problem Solving

Question 17.
Alicia buys a 5-pound bag of rocks for a fish tank. She uses 1 \(\frac{1}{8}\) pounds for a small fish bowl. How much is left?
_______ \(\frac{□}{□}\)

Answer: 3 \(\frac{7}{8}\)

Explanation:
Given,
Alicia buys a 5-pound bag of rocks for a fish tank. She uses 1 \(\frac{1}{8}\) pounds for a small fish bowl.
First subtract the whole numbers
5 – 1 = 4
4 – 1 \(\frac{1}{8}\)
= 3 \(\frac{7}{8}\)

Question 18.
Xavier made 25 pounds of roasted almonds for a fair. He has 3 \(\frac{1}{2}\) pounds left at the end of the fair. How many pounds of roasted almonds did he sell at the fair?
_______ \(\frac{□}{□}\)

Answer: 21 \(\frac{1}{2}\)

Explanation:
Given,
Xavier made 25 pounds of roasted almonds for a fair.
He has 3 \(\frac{1}{2}\) pounds left at the end of the fair.
First subtract the whole numbers
25 – 3 = 22
22 – \(\frac{1}{2}\) = 21 \(\frac{1}{2}\)

Record Subtraction with Renaming – Lesson Check – Page No. 434

Question 1.
Reggie is making a double-layer cake. The recipe for the first layer calls for 2 \(\frac{1}{4}\) cups sugar. The recipe for the second layer calls for 1 \(\frac{1}{4}\) cups sugar. Reggie has 5 cups of sugar. How much will he have left after making both recipes?
Options:
a. 1 \(\frac{1}{4}\) cups
b. 1 \(\frac{2}{4}\) cups
c. 2 \(\frac{1}{4}\) cups
d. 2 \(\frac{2}{4}\) cups

Answer: 1 \(\frac{2}{4}\) cups

Explanation:
Given,
Reggie is making a double-layer cake. The recipe for the first layer calls for 2 \(\frac{1}{4}\) cups sugar.
The recipe for the second layer calls for 1 \(\frac{1}{4}\) cups sugar.
Reggie has 5 cups of sugar.
2 \(\frac{1}{4}\) + 1 \(\frac{1}{4}\) = 3 \(\frac{1}{2}\)
5 – 3 \(\frac{1}{2}\) = 1 \(\frac{2}{4}\) cups
Thus the correct answer is option b.

Question 2.
Kate has 4 \(\frac{3}{8}\) yards of fabric and needs 2 \(\frac{7}{8}\) yards to make a skirt. How much extra fabric will she have left after making the skirt?
Options:
a. 2 \(\frac{4}{8}\) yards
b. 2 \(\frac{2}{8}\) yards
c. 1 \(\frac{4}{8}\) yards
d. 1 \(\frac{2}{8}\) yards

Answer: 1 \(\frac{4}{8}\) yards

Explanation:
Given,
Kate has 4 \(\frac{3}{8}\) yards of fabric and needs 2 \(\frac{7}{8}\) yards to make a skirt.
First, subtract the whole numbers
4 – 2 = 2
Next, subtract the fractions,
\(\frac{3}{8}\) – \(\frac{7}{8}\) = – \(\frac{4}{8}\)
2 – \(\frac{4}{8}\) = 1 \(\frac{4}{8}\) yards
Thus the correct answer is option c.

Spiral Review

Question 3.
Paulo has 128 glass beads to use to decorate picture frames. He wants to use the same number of beads on each frame. If he decorates 8 picture frames, how many beads will he put on each frame?
Options:
a. 6
b. 7
c. 14
d. 16

Answer: 16

Explanation:
Given,
Paulo has 128 glass beads to use to decorate picture frames. He wants to use the same number of beads on each frame
128/8 = 16
Thus the correct answer is option d.

Question 4.
Madison is making party favors. She wants to make enough favors so each guest gets the same number of favors. She knows there will be 6 or 8 guests at the party. What is the least number of party favors Madison should make?
Options:
a. 18
b. 24
c. 30
d. 32

Answer: 24

Explanation:
Given,
Madison is making party favors. She wants to make enough favors so each guest gets the same number of favors.
She knows there will be 6 or 8 guests at the party.
To find the least number of party favors, we have to consider the number of guests.
In this case, there are two possibilities—6 or 8.
For 6: 6, 12, 18, 24 (Add 6 to each number)
For 8: 8, 16, 24 (Add 8 to each number)
Now in both series, the least number (that is in common) is 24. Hence, Madison should make at least 24 party favors.
Thus the correct answer is option b.

Question 5.
A shuttle bus makes 4 round-trips between two shopping centers each day. The bus holds 24 people. If the bus is full on each one-way trip, how many passengers are carried by the bus each day?
Options:
a. 96
b. 162
c. 182
d. 192

Answer: 96

Explanation:
Given,
A shuttle bus makes 4 round-trips between two shopping centers each day. The bus holds 24 people.
4 × 24 = 96
Thus the correct answer is option a.

Question 6.
To make a fruit salad, Marvin mixes 1 \(\frac{3}{4}\) cups of diced peaches with 2 \(\frac{1}{4}\) cups of diced pears. How many cups of peaches and pears are in the fruit salad?
Options:
a. 4 cups
b. 3 \(\frac{2}{4}\) cups
c. 3 \(\frac{1}{4}\) cups
d. 3 cups

Answer: 4 cups

Explanation:
Given,
To make a fruit salad, Marvin mixes 1 \(\frac{3}{4}\) cups of diced peaches with 2 \(\frac{1}{4}\) cups of diced pears.
1 \(\frac{3}{4}\) + 2 \(\frac{1}{4}\)
= 4 cups
Thus the correct answer is option a.

Record Subtraction with Renaming – Page No. 437

Question 1.
Complete. Name the property used.
\(\left(3 \frac{4}{10}+5 \frac{2}{10}\right)+\frac{6}{10}\)
______ \(\frac{□}{□}\)

Answer:
The property used is associative property.
9 \(\frac{2}{10}\)

Explanation:
The associative property states that you can add or multiply regardless of how the numbers are grouped.
Given,
\(\left(3 \frac{4}{10}+5 \frac{2}{10}\right)+\frac{6}{10}\)
First add the whole numbers in the group.
(3 \(\frac{4}{10}\) + 5 \(\frac{2}{10}\)) + \(\frac{6}{10}\)
3 + 5 = 8
8 + \(\frac{4}{10}\) + \(\frac{2}{10}\) + \(\frac{6}{10}\)
Now add the fractions
8 + \(\frac{6}{10}\) + \(\frac{6}{10}\)
8 + \(\frac{12}{10}\)
Convert from improper fractions to the mixed fractions.
\(\frac{12}{10}\) = 1 \(\frac{2}{10}\)
8 + 1 \(\frac{2}{10}\) = 9 \(\frac{2}{10}\)
Thus \(\left(3 \frac{4}{10}+5 \frac{2}{10}\right)+\frac{6}{10}\) = 9 \(\frac{2}{10}\)

Use the properties and mental math to find the sum.

Question 2.
\(\left(2 \frac{7}{8}+3 \frac{2}{8}\right)+1 \frac{1}{8}\)
______ \(\frac{□}{□}\)

Answer: 7 \(\frac{1}{4}\)

Explanation:
The associative property states that you can add or multiply regardless of how the numbers are grouped.
Given
\(\left(2 \frac{7}{8}+3 \frac{2}{8}\right)+1 \frac{1}{8}\)
First add the whole numbers in the group.
(2 \(\frac{7}{8}\) + 3 \(\frac{2}{8}\)) + 1 \(\frac{1}{8}\)
2 + 3 = 5
5 + \(\frac{7}{8}\) + \(\frac{2}{8}\) + 1 \(\frac{1}{8}\)
5 + \(\frac{9}{8}\) + 1 \(\frac{1}{8}\)
6 + \(\frac{10}{8}\) = 7 \(\frac{1}{4}\)
Thus \(\left(2 \frac{7}{8}+3 \frac{2}{8}\right)+1 \frac{1}{8}\) = 7 \(\frac{1}{4}\)

Question 3.
\(1 \frac{2}{5}+\left(1+\frac{3}{5}\right)\)
______

Answer: 3

Explanation:
The associative property states that you can add or multiply regardless of how the numbers are grouped.
Given,
\(1 \frac{2}{5}+\left(1+\frac{3}{5}\right)\)
First add the whole numbers in the group.
1 + \(\frac{3}{5}\) = 1 \(\frac{3}{5}\)
1 \(\frac{2}{5}\) + 1 \(\frac{3}{5}\)
1 + 1 + \(\frac{5}{5}\)
1 + 1 + 1 = 3
Thus \(1 \frac{2}{5}+\left(1+\frac{3}{5}\right)\) = 3

Question 4.
\(5 \frac{3}{6}+\left(5 \frac{5}{6}+4 \frac{3}{6}\right)\)
______ \(\frac{□}{□}\)

Answer: 15 \(\frac{5}{6}\)

Explanation:
The associative property states that you can add or multiply regardless of how the numbers are grouped.
Given,
\(5 \frac{3}{6}+\left(5 \frac{5}{6}+4 \frac{3}{6}\right)\)
First add the whole numbers in the group.
5 + 4 = 9
\(\frac{5}{6}\) + \(\frac{3}{6}\) = \(\frac{8}{6}\)
5 \(\frac{3}{6}\) + 9 \(\frac{8}{6}\)
5 \(\frac{3}{6}\) + 10 \(\frac{2}{6}\) = 15 \(\frac{5}{6}\)
Thus \(5 \frac{3}{6}+\left(5 \frac{5}{6}+4 \frac{3}{6}\right)\) = 15 \(\frac{5}{6}\)

Question 5.
\(\left(1 \frac{1}{4}+1 \frac{1}{4}\right)+2 \frac{3}{4}\)
______ \(\frac{□}{□}\)

Answer: 5 \(\frac{1}{4}\)

Explanation:
The associative property states that you can add or multiply regardless of how the numbers are grouped.
Given,
\(\left(1 \frac{1}{4}+1 \frac{1}{4}\right)+2 \frac{3}{4}\)
First add the whole numbers in the group.
(1 \(\frac{1}{4}\) + 1 \(\frac{1}{4}\)) + 2 \(\frac{3}{4}\)
1 + 1 = 2
2 \(\frac{1}{4}\) + \(\frac{1}{4}\) + 2 \(\frac{3}{4}\)
2 \(\frac{1}{2}\) + 2 \(\frac{3}{4}\)
Add the whole numbers
2 + 2 = 4
4 \(\frac{1}{2}\) + \(\frac{3}{4}\) = 5 \(\frac{1}{4}\)
Thus \(\left(1 \frac{1}{4}+1 \frac{1}{4}\right)+2 \frac{3}{4}\) = 5 \(\frac{1}{4}\)

Question 6.
\(\left(12 \frac{4}{9}+1 \frac{2}{9}\right)+3 \frac{5}{9}\)
______ \(\frac{□}{□}\)

Answer: 17 \(\frac{2}{9}\)

Explanation:
The associative property states that you can add or multiply regardless of how the numbers are grouped.
Given,
\(\left(12 \frac{4}{9}+1 \frac{2}{9}\right)+3 \frac{5}{9}\)
First add the whole numbers in the group.
12 + 1 = 13
Add the fraction in the group.
\(\frac{4}{9}\) + \(\frac{2}{9}\) + 3 \(\frac{5}{9}\)
= 13 \(\frac{6}{9}\) + 3 \(\frac{5}{9}\)
= 16 \(\frac{11}{9}\)
= 17 \(\frac{2}{9}\)
Thus \(\left(12 \frac{4}{9}+1 \frac{2}{9}\right)+3 \frac{5}{9}\) = 17 \(\frac{2}{9}\)

Question 7.
\(\left(\frac{3}{12}+1 \frac{8}{12}\right)+\frac{9}{12}\)
______ \(\frac{□}{□}\)

Answer: 2 \(\frac{2}{3}\)

Explanation:
The associative property states that you can add or multiply regardless of how the numbers are grouped.
Given,
\(\left(\frac{3}{12}+1 \frac{8}{12}\right)+\frac{9}{12}\)
First add the fractions in the group.
\(\frac{3}{12}\) + \(\frac{8}{12}\) = \(\frac{11}{12}\)
1 \(\frac{11}{12}\) + \(\frac{9}{12}\) = 1 \(\frac{20}{12}\)
= 2 \(\frac{2}{3}\)
Thus \(\left(\frac{3}{12}+1 \frac{8}{12}\right)+\frac{9}{12}\) = 2 \(\frac{2}{3}\)

Use the properties and mental math to find the sum.

Question 8.
\(\left(45 \frac{1}{3}+6 \frac{1}{3}\right)+38 \frac{2}{3}\)
______ \(\frac{□}{□}\)

Answer: 90 \(\frac{1}{3}\)

Explanation:
Given,
\(\left(45 \frac{1}{3}+6 \frac{1}{3}\right)+38 \frac{2}{3}\)
First add the whole numbers in the group.
45 + 6 = 51
(51 \(\frac{1}{3}\) + \(\frac{1}{3}\)) + 38 \(\frac{2}{3}\)
51 \(\frac{2}{3}\) + 38 \(\frac{2}{3}\)
= 89 \(\frac{4}{3}\)
= 90 \(\frac{1}{3}\)
Thus \(\left(45 \frac{1}{3}+6 \frac{1}{3}\right)+38 \frac{2}{3}\) = 90 \(\frac{1}{3}\)

Question 9.
\(\frac{1}{2}+\left(103 \frac{1}{2}+12\right)\)
______ \(\frac{□}{□}\)

Answer: 116

Explanation:
Given,
\(\frac{1}{2}+\left(103 \frac{1}{2}+12\right)\)
First add the whole numbers in the group.
103 + \(\frac{1}{2}\) + 12 = 115 \(\frac{1}{2}\)
115 \(\frac{1}{2}\) + \(\frac{1}{2}\) = 116
Thus \(\frac{1}{2}+\left(103 \frac{1}{2}+12\right)\) = 116

Question 10.
\(\left(3 \frac{5}{10}+10\right)+11 \frac{5}{10}\)
______

Answer: 25

Explanation:
Given,
\(\left(3 \frac{5}{10}+10\right)+11 \frac{5}{10}\)
First add the whole numbers in the group.
3 + 10 = 13
13 + \(\frac{5}{10}\) + 11 \(\frac{5}{10}\)
Add the whole numbers
13 + 11 = 24
24 + \(\frac{5}{10}\) + \(\frac{5}{10}\) = 25
Thus \(\left(3 \frac{5}{10}+10\right)+11 \frac{5}{10}\) = 25

Question 11.
Pablo is training for a marathon. He ran 5 \(\frac{4}{8}\) miles on Friday, 6 \(\frac{5}{8}\) miles on Saturday, and 7 \(\frac{4}{8}\) miles on Sunday. How many miles did he run on all three days?
______ \(\frac{□}{□}\) miles

Answer: 19 \(\frac{5}{8}\) miles

Explanation:
Given,
Pablo is training for a marathon. He ran 5 \(\frac{4}{8}\) miles on Friday, 6 \(\frac{5}{8}\) miles on Saturday, and 7 \(\frac{4}{8}\) miles on Sunday.
Add all the fractions to find how many miles he runs on all three days.
5 \(\frac{4}{8}\) + 6 \(\frac{5}{8}\) + 7 \(\frac{4}{8}\)
First add the whole numbers
5 + 6 + 7 = 18
18 + \(\frac{4}{8}\) + \(\frac{5}{8}\) + \(\frac{4}{8}\)
= 18 + \(\frac{13}{8}\)
= 19 \(\frac{5}{8}\) miles
Therefore Pablo runs 19 \(\frac{5}{8}\) miles on all three days.

Question 12.
At lunchtime, Dale’s Diner served a total of 2 \(\frac{2}{6}\) pots of vegetable soup, 3 \(\frac{5}{6}\) pots of chicken soup, and 4 \(\frac{3}{6}\) pots of tomato soup. How many pots of soup were served in all?
______ \(\frac{□}{□}\) pots

Answer: 10 \(\frac{2}{3}\) pots

Explanation:
Given,
At lunchtime, Dale’s Diner served a total of 2 \(\frac{2}{6}\) pots of vegetable soup, 3 \(\frac{5}{6}\) pots of chicken soup, and 4 \(\frac{3}{6}\) pots of tomato soup.
2 \(\frac{2}{6}\) + 3 \(\frac{5}{6}\) + 4 \(\frac{3}{6}\)
First add the whole numbers
2 + 3 + 4 = 9
Next add the fractions.
\(\frac{2}{6}\) + \(\frac{5}{6}\) + \(\frac{3}{6}\)
= \(\frac{10}{6}\)
9 + \(\frac{10}{6}\) = 10 \(\frac{2}{3}\) pots
Therefore 10 \(\frac{2}{3}\) pots of soup were served in all.

Use the expressions in the box for 13–14.
Go Math Grade 4 Answer Key Chapter 7 Add and Subtract Fractions Page No. 437 Q 13

Question 13.
Which property of addition would you use to regroup the addends in Expression A?
______ property

Answer: Associative Property

Explanation:
The associative property states that you can add or multiply regardless of how the numbers are grouped.
Expression A is \(\frac{1}{8}\) + (\(\frac{7}{8}\) + \(\frac{4}{8}\))
The denominators of all three fractions are the same. So, the property for expression A is Associative Property.

Question 14.
Which two expressions have the same value?
________ and _________

Answer: A and C

Explanation:
Expression A is \(\frac{1}{8}\) + (\(\frac{7}{8}\) + \(\frac{4}{8}\))
\(\frac{1}{8}\) + (\(\frac{11}{8}\) = \(\frac{12}{8}\)
Expression B is 1/2 + 2
1/2 + 4/2 = 5/2
Expression C is \(\frac{3}{7}\) + (\(\frac{1}{2}\) + \(\frac{4}{7}\))
\(\frac{1}{2}\) + \(\frac{4}{7}\) = \(\frac{7}{14}\) + \(\frac{8}{14}\) = \(\frac{15}{14}\)
\(\frac{15}{14}\) + \(\frac{3}{7}\) = \(\frac{15}{14}\) + \(\frac{6}{14}\) = \(\frac{21}{14}\)
Thus the expressions A and C has the same value.

Question 15.
Match the equation with the property used.
Go Math Grade 4 Answer Key Chapter 7 Add and Subtract Fractions Page No. 437 Q 15
Type below:
_________

Answer:
Go-Math-Grade-4-Answer-Key-Chapter-7-Add-and-Subtract-Fractions-Page-No.-437-Q-15

Record Subtraction with Renaming – Page No. 438

Pose a Problem
Go Math Grade 4 Answer Key Chapter 7 Add and Subtract Fractions Page No. 438 Q 16

Question 16.
Costumes are being made for the high school musical. The table at the right shows the amount of fabric needed for the costumes of the male and female leads. Alice uses the expression \(7 \frac{3}{8}+1 \frac{5}{8}+2 \frac{4}{8}\) to find the total amount of fabric needed for the costume of the female lead. To find the value of the expression using mental math, Alice used the properties of addition.
\(7 \frac{3}{8}+1 \frac{5}{8}+2 \frac{4}{8}=\left(7 \frac{3}{8}+1 \frac{5}{8}\right)+2 \frac{4}{8}\)
Alice added 7 + 1 and was able to quickly add \(\frac{3}{8}\) and \(\frac{5}{8}\) to the sum of 8 to get 9. She added 2 \(\frac{4}{8}\) to 9, so her answer was 11 \(\frac{4}{8}\).
So, the amount of fabric needed for the costume of the female lead actor is 11 \(\frac{4}{8}\) yards.
Write a new problem using the information for the costume for the male lead actor.
Pose a Problem                     Solve your problem. Check your solution.
Type below:
_____________

Answer:
Alice used the expressions 1 2/8 + 2 3/8 + 5 6/8 to find the total amount of frabric needed for the costume of the male lead. What is the total amount of fabric needed for the costume?
Answer: Alice wrote the expressions as (1 2/8 + 5 6/8) + 2 3/8 and simplified it by adding the whole number parts and the fraction parts in the parentheses.
Then she added the mixed number: 1 + 5 + 1 + 2 3/8 = 9 3/8.
So, the male leads costume needed 9 3/8 yards of fabric.

Question 16.
Identify Relationships Explain how using the properties of addition makes both problems easier to solve.
Type below:
____________

Answer:
The properties make the properties the easier to solve because you can rearrange the mixed numbers so that their fraction parts add to 1.

Fractions and Properties of Addition – Page No. 439

Use the properties and mental math to find the sum.

Question 1.
Go Math Grade 4 Answer Key Chapter 7 Add and Subtract Fractions Common Core - New Page No. 439 Q 1

Question 2.
\(10 \frac{1}{8}+\left(3 \frac{5}{8}+2 \frac{7}{8}\right)\)
_______ \(\frac{□}{□}\)

Answer: 16 \(\frac{5}{8}\)

Explanation:
Given,
\(10 \frac{1}{8}+\left(3 \frac{5}{8}+2 \frac{7}{8}\right)\)
First add the whole numbers in the bracket.
3 + 2 = 5
10 \(\frac{1}{8}\) + 5 + \(\frac{5}{8}\) + \(\frac{7}{8}\)
10 \(\frac{1}{8}\) + 5 + \(\frac{12}{8}\)
10 + 5 = 15
15 + \(\frac{1}{8}\) + \(\frac{12}{8}\)
15 + \(\frac{13}{8}\)
16 \(\frac{5}{8}\)
\(10 \frac{1}{8}+\left(3 \frac{5}{8}+2 \frac{7}{8}\right)\) = 16 \(\frac{5}{8}\)

Question 3.
\(8 \frac{1}{5}+\left(3 \frac{2}{5}+5 \frac{4}{5}\right)\)
_______ \(\frac{□}{□}\)

Answer: 17 \(\frac{2}{5}\)

Explanation:
\(8 \frac{1}{5}+\left(3 \frac{2}{5}+5 \frac{4}{5}\right)\)
8 \(\frac{1}{5}\) + 3 \(\frac{2}{5}\) + 5 \(\frac{4}{5}\)
3 + 5 = 8
8 \(\frac{1}{5}\) + 8 + \(\frac{2}{5}\) + \(\frac{4}{5}\)
8 \(\frac{1}{5}\) + 8 + \(\frac{6}{5}\)
8 + 8 = 16
16 + \(\frac{1}{5}\) + \(\frac{6}{5}\)
16 + \(\frac{7}{5}\)
17 \(\frac{2}{5}\)
\(8 \frac{1}{5}+\left(3 \frac{2}{5}+5 \frac{4}{5}\right)\) = 17 \(\frac{2}{5}\)

Question 4.
\(6 \frac{3}{4}+\left(4 \frac{2}{4}+5 \frac{1}{4}\right)\)
_______ \(\frac{□}{□}\)

Answer: 16 \(\frac{1}{2}\)

Explanation:
\(6 \frac{3}{4}+\left(4 \frac{2}{4}+5 \frac{1}{4}\right)\)
First add the whole numbers in the bracket.
6 \(\frac{3}{4}\) + 4 \(\frac{2}{4}\) + 5 \(\frac{1}{4}\)
4 + 5 = 9
6 \(\frac{3}{4}\) + 9 \(\frac{3}{4}\)
6 + 9 = 15
15 + \(\frac{3}{4}\) + \(\frac{3}{4}\)
16 \(\frac{1}{2}\)
\(6 \frac{3}{4}+\left(4 \frac{2}{4}+5 \frac{1}{4}\right)\) = 16 \(\frac{1}{2}\)

Question 5.
\(\left(6 \frac{3}{6}+10 \frac{4}{6}\right)+9 \frac{2}{6}\)
_______ \(\frac{□}{□}\)

Answer: 26 \(\frac{3}{6}\)

Explanation:
\(\left(6 \frac{3}{6}+10 \frac{4}{6}\right)+9 \frac{2}{6}\)
6 \(\frac{3}{6}\) + 10 \(\frac{4}{6}\) + 9 \(\frac{2}{6}\)
First add the whole numbers in the bracket.
6 + 10 = 16
16 + \(\frac{3}{6}\) + \(\frac{4}{6}\) + 9 \(\frac{2}{6}\)
16 + \(\frac{7}{6}\) + 9 \(\frac{2}{6}\)
16 + 9 = 25
25 + \(\frac{7}{6}\) + \(\frac{2}{6}\)
25 + \(\frac{9}{6}\)
= 26 \(\frac{3}{6}\)
\(\left(6 \frac{3}{6}+10 \frac{4}{6}\right)+9 \frac{2}{6}\) = 26 \(\frac{3}{6}\)

Question 6.
\(\left(6 \frac{2}{5}+1 \frac{4}{5}\right)+3 \frac{1}{5}\)
_______ \(\frac{□}{□}\)

Answer: 11 \(\frac{2}{5}\)

Explanation:
\(\left(6 \frac{2}{5}+1 \frac{4}{5}\right)+3 \frac{1}{5}\)
6 \(\frac{2}{5}\) + 1 \(\frac{4}{5}\) + 3 \(\frac{1}{5}\)
First add the whole numbers in the bracket.
6 + 1 = 7
7 \(\frac{2}{5}\) + \(\frac{4}{5}\) + 3 \(\frac{1}{5}\)
7 + \(\frac{6}{5}\) + 3 \(\frac{1}{5}\)
7 + 3 = 10
10 + \(\frac{6}{5}\) + \(\frac{1}{5}\)
10 + \(\frac{7}{5}\) = 11 \(\frac{2}{5}\)
Therefore \(\left(6 \frac{2}{5}+1 \frac{4}{5}\right)+3 \frac{1}{5}\) = 11 \(\frac{2}{5}\)

Question 7.
\(7 \frac{7}{8}+\left(3 \frac{1}{8}+1 \frac{1}{8}\right)\)
_______ \(\frac{□}{□}\)

Answer: 12 \(\frac{1}{8}\)

Explanation:
\(7 \frac{7}{8}+\left(3 \frac{1}{8}+1 \frac{1}{8}\right)\)
7 \(\frac{7}{8}\) + 3 \(\frac{1}{8}\) + 1 \(\frac{1}{8}\)
First add the whole numbers in the bracket.
3 + 1 = 4
7 \(\frac{7}{8}\) + 4 + \(\frac{1}{8}\) + \(\frac{1}{8}\)
7 \(\frac{7}{8}\) + 4 +\(\frac{2}{8}\)
7 + 4 = 11
11 + \(\frac{7}{8}\) + \(\frac{2}{8}\)
11 + \(\frac{9}{8}\) = 12 \(\frac{1}{8}\)
Thus \(7 \frac{7}{8}+\left(3 \frac{1}{8}+1 \frac{1}{8}\right)\) = 12 \(\frac{1}{8}\)

Question 8.
\(14 \frac{1}{10}+\left(20 \frac{2}{10}+15 \frac{7}{10}\right)\)
_______ \(\frac{□}{□}\)

Answer: 50

Explanation:
\(14 \frac{1}{10}+\left(20 \frac{2}{10}+15 \frac{7}{10}\right)\)
First add the whole numbers in the bracket.
14 \(\frac{1}{10}\) + 20 \(\frac{2}{10}\) + 15 \(\frac{7}{10}\)
20 + 15 = 35
14 \(\frac{1}{10}\) + 35 + \(\frac{2}{10}\) + \(\frac{7}{10}\)
14 \(\frac{1}{10}\) + 35 \(\frac{9}{10}\)
49 \(\frac{1}{10}\) + \(\frac{9}{10}\)
49 + 1 = 50
Thus \(14 \frac{1}{10}+\left(20 \frac{2}{10}+15 \frac{7}{10}\right)\) = 50

Question 9.
\(\left(13 \frac{2}{12}+8 \frac{7}{12}\right)+9 \frac{5}{12}\)
_______ \(\frac{□}{□}\)

Answer: 31 \(\frac{2}{12}\)

Explanation:
\(\left(13 \frac{2}{12}+8 \frac{7}{12}\right)+9 \frac{5}{12}\)
13 \(\frac{2}{12}\) + 8 \(\frac{7}{12}\) + 9 \(\frac{5}{12}\)
First add the whole numbers in the bracket.
13 + 8 = 21
21 + \(\frac{2}{12}\) + \(\frac{7}{12}\) + 9 \(\frac{5}{12}\)
21 + \(\frac{9}{12}\) + 9 \(\frac{5}{12}\)
30 + \(\frac{9}{12}\) + \(\frac{5}{12}\) = 31 \(\frac{2}{12}\)
Thus \(\left(13 \frac{2}{12}+8 \frac{7}{12}\right)+9 \frac{5}{12}\) = 31 \(\frac{2}{12}\)

Problem Solving

Question 10.
Nate’s classroom has three tables of different lengths. One has a length of 4 \(\frac{1}{2}\) feet, another has a length of 4 feet, and a third has a length of 2 \(\frac{1}{2}\) feet. What is the length of all three tables when pushed end to end?
_______ \(\frac{□}{□}\)

Answer: 11

Explanation:
Given,
Nate’s classroom has three tables of different lengths. One has a length of 4 \(\frac{1}{2}\) feet, another has a length of 4 feet, and a third has a length of 2 \(\frac{1}{2}\) feet.
4 \(\frac{1}{2}\) + 4 + 2 \(\frac{1}{2}\)
4 + 4 + 2 = 10
\(\frac{1}{2}\) + \(\frac{1}{2}\) = 1
10 + 1 = 11
Therefore the length of all three tables when pushed end to end is 11 feet.

Question 11.
Mr. Warren uses 2 \(\frac{1}{4}\) bags of mulch for his garden and another 4 \(\frac{1}{4}\) bags for his front yard. He also uses \(\frac{3}{4}\) bag around a fountain. How many total bags of mulch does Mr. Warren use?
_______ \(\frac{□}{□}\)

Answer: 7 \(\frac{1}{4}\)

Explanation:
Given,
Mr. Warren uses 2 \(\frac{1}{4}\) bags of mulch for his garden and another 4 \(\frac{1}{4}\) bags for his front yard.
He also uses \(\frac{3}{4}\) bag around a fountain.
2 \(\frac{1}{4}\) + 4 \(\frac{1}{4}\) + \(\frac{3}{4}\)
2 + 4 = 6
6 + \(\frac{1}{4}\) + \(\frac{1}{4}\) + \(\frac{3}{4}\)
= 7 \(\frac{1}{4}\)

Fractions and Properties of Addition – Lesson Check – Page No. 440

Question 1.
A carpenter cut a board into three pieces. One piece was 2 \(\frac{5}{6}\) feet long. The second piece was 3 \(\frac{1}{6}\) feet long. The third piece was 1 \(\frac{5}{6}\) feet long. How long was the board?
Options:
a. 6 \(\frac{5}{6}\) feet
b. 7 \(\frac{1}{6}\) feet
c. 7 \(\frac{5}{6}\) feet
d. 8 \(\frac{1}{6}\) feet

Answer: c. 7 \(\frac{5}{6}\) feet

Explanation:
Given,
A carpenter cut a board into three pieces. One piece was 2 \(\frac{5}{6}\) feet long. The second piece was 3 \(\frac{1}{6}\) feet long.
The third piece was 1 \(\frac{5}{6}\) feet long.
Add three pieces.
2 \(\frac{5}{6}\) + 3 \(\frac{1}{6}\)
= 5 + \(\frac{6}{6}\)
= 5 + 1 = 6
6 + 1 \(\frac{5}{6}\)
= 7 \(\frac{5}{6}\) feet
Thus the correct answer is option c.

Question 2.
Harry works at an apple orchard. He picked 45 \(\frac{7}{8}\) pounds of apples on Monday. He picked 42 \(\frac{3}{8}\) pounds of apples on Wednesday. He picked 54 \(\frac{1}{8}\) pounds of apples on Friday. How many pounds of apples did Harry pick those three days?
Options:
a. 132 \(\frac{3}{8}\) pounds
b. 141 \(\frac{3}{8}\) pounds
c. 142 \(\frac{1}{8}\) pounds
d. 142 \(\frac{3}{8}\) pounds

Answer: 142 \(\frac{3}{8}\) pounds

Explanation:
Given,
Harry works at an apple orchard. He picked 45 \(\frac{7}{8}\) pounds of apples on Monday.
He picked 42 \(\frac{3}{8}\) pounds of apples on Wednesday.
He picked 54 \(\frac{1}{8}\) pounds of apples on Friday.
45 \(\frac{7}{8}\) + 42 \(\frac{3}{8}\) + 54 \(\frac{1}{8}\)
Add the whole numbers first
45 + 42 + 54 = 141
141 + \(\frac{7}{8}\) + \(\frac{3}{8}\) + \(\frac{1}{8}\)
141 + 1 \(\frac{3}{8}\)
= 142 \(\frac{3}{8}\) pounds
Thus the correct answer is option d.

Spiral Review

Question 3.
There were 6 oranges in the refrigerator. Joey and his friends ate 3 \(\frac{2}{3}\) oranges. How many oranges were left?
Options:
a. 2 \(\frac{1}{3}\) oranges
b. 2 \(\frac{2}{3}\) oranges
c. 3 \(\frac{1}{3}\) oranges
d. 9 \(\frac{2}{3}\) oranges

Answer: 9 \(\frac{2}{3}\) oranges

Explanation:
Given,
There were 6 oranges in the refrigerator.
Joey and his friends ate 3 \(\frac{2}{3}\) oranges.
6 + 3 \(\frac{2}{3}\)
= 9 \(\frac{2}{3}\) oranges
Thus the correct answer is option d.

Question 4.
Darlene was asked to identify which of the following numbers is prime. Which number should she choose?
Options:
a. 2
b. 12
c. 21
d. 39

Answer: 2

Explanation:
A prime number is an integer, or whole number, that has only two factors 1 and itself.
In the above options, all are composite numbers except 2.
Therefore 2 is a prime number.
Thus the correct answer is option a.

Question 5.
A teacher has 100 chairs to arrange for an assembly. Which of the following is NOT a way the teacher could arrange the chairs?
Options:
a. 10 rows of 10 chairs
b. 8 rows of 15 chairs
c. 5 rows of 20 chairs
d. 4 rows of 25 chairs

Answer: 8 rows of 15 chairs

Explanation:
A teacher has 100 chairs to arrange for an assembly.
15 × 8 = 120
So, 8 rows of 15 chairs are not the way to arrange the chairs.
Thus the correct answer is option b.

Question 6.
Nic bought 28 folding chairs for $16 each. How much money did Nic spend on chairs?
Options:
a. $196
b. $348
c. $448
d. $600

Answer: c. $448

Explanation:
Given,
Nic bought 28 folding chairs for $16 each.
28 × 16 = 448
Thus the correct answer is option c.

Fractions and Properties of Addition – Lesson Check – Page No. 443

Question 1.
Last week, Sia ran 1 \(\frac{1}{4}\) miles each day for 5 days and then took 2 days off. Did she run at least 6 miles last week? First, model the problem. Describe your model.
Type below:
_________

Answer:
I will model the problem using fraction strips. I need a 1 strip for the whole and a 1/4 part for each of the 5 days. My model has a total of five 1 strops and five 1/4 parts.

Question 1.
Then, regroup the parts in the model to find the number of whole miles Sia ran.
Sia ran ___________ whole miles and ___________ mile.
Finally, compare the total number of miles she ran to 6 miles.
So, Sia ___________ run at least 6 miles last week.
6 \(\frac{1}{4}\) miles _____ 6 miles

Answer:
Sia ran 6 whole miles and 1/4 mile.
So, Sia did run at least 6 miles last week.
6 \(\frac{1}{4}\) miles > 6 miles

Question 2.
What if Sia ran only \(\frac{3}{4}\) mile each day. Would she have run at least 6 miles last week? Explain.
_____

Answer: No

Explanation:
She would have run \(\frac{3}{4}\) + \(\frac{3}{4}\) + \(\frac{3}{4}\) + \(\frac{3}{4}\) + \(\frac{3}{4}\) + \(\frac{3}{4}\) = \(\frac{15}{4}\) or 3 \(\frac{3}{4}\) miles.

Question 3.
A quarter is \(\frac{1}{4}\) dollar. Noah has 20 quarters. How much money does he have? Explain.
$ _____

Answer: 5

Explanation:
Since each quarter is 1/4 dollar, each group of 4 quarters is 1 dollar. Since 4/4 + 4/4 + 4/4 + 4/4 + 4/4 = 20/4, Noah has 1 + 1 + 1 + 1 + 1 = 5 dollars

Question 4.
How many \(\frac{2}{5}\) parts are in 2 wholes?
_____

Answer: 5

Explanation:
\(\frac{2}{5}\)/2 = 5

Fractions and Properties of Addition – Lesson Check – Page No. 444

Question 5.
A company shipped 15,325 boxes of apples and 12,980 boxes of oranges. How many more boxes of apples than oranges did the company ship?
_____ boxes

Answer: 2345 boxes

Explanation:
Given,
A company shipped 15,325 boxes of apples and 12,980 boxes of oranges.
Subtract 12,980 from 15,325 boxes
15,325 – 12,980 = 2,345 boxes.

Question 6.
Analyze A fair sold a total of 3,300 tickets on Friday and Saturday. It sold 100 more on Friday than on Saturday. How many tickets did the fair sell on Friday?
_____ tickets

Answer: 1700 tickets

Explanation:
Given,
Analyze A fair sold a total of 3,300 tickets on Friday and Saturday. It sold 100 more on Friday than on Saturday.
3,300 – 100 = 3,200 tickets
3200/2 = 1,600 tickets
It sold 1600 tickets on saturday and 1700 tickets on Friday.

Question 7.
Emma walked \(\frac{1}{4}\) mile on Monday, \(\frac{2}{4}\) mile on Tuesday, and \(\frac{3}{4}\) mile on Wednesday. If the pattern continues, how many miles will she walk on Friday? Explain how you found the number of miles.
\(\frac{□}{□}\) miles

Answer: \(\frac{5}{4}\) miles

Explanation:
I made a table that shows each day and the distance she walked. Then I looked for a pattern. The pattern showed that she walked 1/4 mile more each day. I continued the pattern to show she walked 4/4 mile on Thursday and 5/4 miles on Friday.

Question 8.
Jared painted a mug \(\frac{5}{12}\) red and \(\frac{4}{12}\) blue. What part of the mug is not red or blue?
\(\frac{□}{□}\)

Answer: \(\frac{3}{12}\)

Explanation:
Given,
Jared painted a mug \(\frac{5}{12}\) red and \(\frac{4}{12}\) blue.
We have to find What part of the mug is not red or blue that means \(\frac{3}{12}\) part is neither red nor blue.

Question 9.
Choose the number that correctly completes the sentence.
Each day, Mrs. Hewes knits \(\frac{1}{3}\) of a scarf in the morning and \(\frac{1}{3}\) of a scarf in the afternoon.
It will take Mrs. Hewes Go Math Grade 4 Answer Key Chapter 7 Add and Subtract Fractions Page No. 444 Q 9 days to knit 2 scarves.
_____

Answer: 3

Explanation:
Given,
Each day, Mrs. Hewes knits \(\frac{1}{3}\) of a scarf in the morning and \(\frac{1}{3}\) of a scarf in the afternoon.
\(\frac{1}{3}\) + \(\frac{1}{3}\) = \(\frac{2}{3}\)
Thus it takes 3 days to knit 2 scarves.

Fractions and Properties of Addition – Page No. 445

Read each problem and solve.

Question 1.
Each child in the Smith family was given an orange cut into 8 equal sections. Each child ate \(\frac{5}{8}\) of the orange. After combining the leftover sections, Mrs. Smith noted that there were exactly 3 full oranges left. How many children are in the Smith family?
Go Math Grade 4 Answer Key Chapter 7 Add and Subtract Fractions Common Core - New Page No. 445 Q 1

Question 2.
Val walks 2 \(\frac{3}{5}\) miles each day. Bill runs 10 miles once every 4 days. In 4 days, who covers the greater distance?
_________

Answer: Val

Explanation:
Given,
Val walks 2 \(\frac{3}{5}\) miles each day. Bill runs 10 miles once every 4 days.
2 \(\frac{3}{5}\) × 4
Convert from mixed fraction to the improper fraction.
2 \(\frac{3}{5}\) = \(\frac{13}{5}\) × 4 = 10.4
10.4 > 10
Thus Val covers the greater distance.

Question 3.
Chad buys peanuts in 2-pound bags. He repackages them into bags that hold \(\frac{5}{6}\) pound of peanuts. How many 2-pound bags of peanuts should Chad buy so that he can fill the \(\frac{5}{6}\) -pound bags without having any peanuts left over?
_________ 2-pound bags

Answer: 5

Explanation:
Given,
Chad buys peanuts in 2-pound bags. He repackages them into bags that hold \(\frac{5}{6}\) pound of peanuts.
\(\frac{5}{6}\) + \(\frac{5}{6}\) + \(\frac{5}{6}\) + \(\frac{5}{6}\) + \(\frac{5}{6}\)
Thus 5 2-pound bags of peanuts are left.

Question 4.
A carpenter has several boards of equal length. He cuts \(\frac{3}{5}\) of each board. After cutting the boards, the carpenter notices that he has enough pieces left over to make up the same length as 4 of the original boards. How many boards did the carpenter start with?
_________

Answer: 10

Explanation:
Given,
A carpenter has several boards of equal length. He cuts \(\frac{3}{5}\) of each board. After cutting the boards, the carpenter notices that he has enough pieces left over to make up the same length as 4 of the original boards.
4 of the original boards have a summed length of 20 units. 5 x 4 = 20.
Since 2/5 is left from each board, you simply add them until the 2’s add to 20.
So, 2 x 10 = 20. Hence, there are 10 2/5 boards.
That’s just 4 of the boards that the 2/5 make up, but that should also mean that there are 10 3/5 boards as well.
30/5 + 20/5 = 50/5 = 10

Fractions and Properties of Addition – Lesson Check – Page No. 446

Question 1.
Karyn cuts a length of ribbon into 4 equal pieces, each 1 \(\frac{1}{4}\) feet long. How long was the ribbon?
Options:
a. 4 feet
b. 4 \(\frac{1}{4}\) feet
c. 5 feet
d. 5 \(\frac{1}{4}\) feet

Answer: 5 feet

Explanation:
Given,
Karyn cuts a length of ribbon into 4 equal pieces, each 1 \(\frac{1}{4}\) feet long.
1 \(\frac{1}{4}\) × 4
Convert from the mixed fraction to the improper fraction.
1 \(\frac{1}{4}\) = \(\frac{5}{4}\)
\(\frac{5}{4}\) × 4 = 5 feet
Thus the correct answer is option c.

Question 2.
Several friends each had \(\frac{2}{5}\) of a bag of peanuts left over from the baseball game. They realized that they could have bought 2 fewer bags of peanuts between them. How many friends went to the game?
Options:
a. 6
b. 5
c. 4
d. 2

Answer: 5

Explanation:
Given,
Several friends each had \(\frac{2}{5}\) of a bag of peanuts left over from the baseball game.
They realized that they could have bought 2 fewer bags of peanuts between them
2 ÷ \(\frac{2}{5}\) = 5
Thus the correct answer is option b.

Spiral Review

Question 3.
A frog made three jumps. The first was 12 \(\frac{5}{6}\) inches. The second jump was 8 \(\frac{3}{6}\) inches. The third jump was 15 \(\frac{1}{6}\) inches. What was the total distance the frog jumped?
Options:
a. 35 \(\frac{3}{6}\) inches
b. 36 \(\frac{1}{6}\) inches
c. 36 \(\frac{3}{6}\) inches
d. 38 \(\frac{1}{6}\) inches

Answer: 36 \(\frac{3}{6}\) inches

Explanation:
Given,
A frog made three jumps. The first was 12 \(\frac{5}{6}\) inches. The second jump was 8 \(\frac{3}{6}\) inches. The third jump was 15 \(\frac{1}{6}\) inches.
First add the whole numbers
12 + 8 + 15 = 35
Next add the fractions,
\(\frac{5}{6}\) + \(\frac{3}{6}\) + \(\frac{1}{6}\) = 1 \(\frac{3}{6}\)
35 + \(\frac{3}{6}\) = 36 \(\frac{3}{6}\) inches
Thus the correct answer is option c.

Question 4.
LaDanian wants to write the fraction \(\frac{4}{6}\) as a sum of unit fractions. Which expression should he write?
Options:
a. \(\frac{1}{6}+\frac{1}{6}+\frac{1}{6}+\frac{1}{6}\)
b. \(\frac{2}{6}+\frac{2}{6}\)
c. \(\frac{3}{6}+\frac{1}{6}\)
d. \(\frac{1}{6}+\frac{1}{6}+\frac{2}{6}\)

Answer: \(\frac{1}{6}+\frac{1}{6}+\frac{1}{6}+\frac{1}{6}\)

Explanation:
Given,
LaDanian wants to write the fraction \(\frac{4}{6}\) as a sum of unit fractions.
The unit fraction for \(\frac{4}{6}\) is \(\frac{1}{6}+\frac{1}{6}+\frac{1}{6}+\frac{1}{6}\)
Thus the correct answer is option a.

Question 5.
Greta made a design with squares. She colored 8 out of the 12 squares blue. What fraction of the squares did she color blue?
Options:
a. \(\frac{1}{4}\)
b. \(\frac{1}{3}\)
c. \(\frac{2}{3}\)
d. \(\frac{3}{4}\)

Answer: \(\frac{2}{3}\)

Explanation:
Given,
Greta made a design with squares. She colored 8 out of the 12 squares blue.
\(\frac{8}{12}\)
= \(\frac{2}{3}\)
Thus the correct answer is option c.

Question 6.
The teacher gave this pattern to the class: the first term is 5 and the rule is add 4, subtract 1. Each student says one number. The first student says 5. Victor is tenth in line. What number should Victor say?
Options:
a. 17
b. 19
c. 20
d. 21

Answer:
given
a=5
d=4-1=3
to find t10
tn=a + (n-1) d
t10=5 + (10-1) 3
t10=5 + 27
t10 = 32
victor is tenth in line,therefore he should say the number 32

Fractions and Properties of Addition – Page No. 447

Question 1.
A painter mixed \(\frac{1}{4}\) quart of red paint with \(\frac{3}{4}\) blue paint to make purple paint.
Go Math Grade 4 Answer Key Chapter 7 Add and Subtract Fractions Page No. 447 Q 1
How much purple paint did the painter make?
_____ quart of purple paint

Answer: 1

Explanation:
Given,
A painter mixed \(\frac{1}{4}\) quart of red paint with \(\frac{3}{4}\) blue paint to make purple paint.
\(\frac{1}{4}\) + \(\frac{3}{4}\) = \(\frac{4}{4}\) or 1.

Question 2.
Ivan biked 1 \(\frac{2}{3}\) hours on Monday, 2 \(\frac{1}{3}\) hours on Tuesday, and 2 \(\frac{2}{3}\) hours on Wednesday. What is the total number of hours Ivan spent biking?
Ivan spen _______ hours biking.
_____ \(\frac{□}{□}\)

Answer: 6 \(\frac{2}{3}\)

Explanation:
Given,
Ivan biked 1 \(\frac{2}{3}\) hours on Monday, 2 \(\frac{1}{3}\) hours on Tuesday, and 2 \(\frac{2}{3}\) hours on Wednesday.
1 \(\frac{2}{3}\) + 2 \(\frac{1}{3}\) + 2 \(\frac{2}{3}\)
First add the whole numbers,
1 + 2 + 2 = 5
2/3 + 1/3 + 2/3 = 5/3
Convert from improper fraction to the mixed fraction.
5/3 = 1 2/3
5 + 1 1/3  = 6 \(\frac{2}{3}\)

Question 3.
Tricia had 4 \(\frac{1}{8}\) yards of fabric to make curtains. When she finished she had 2 \(\frac{3}{8}\) yards of fabric left. She said she used 2 \(\frac{2}{8}\) yards of fabric for the curtains. Do you agree? Explain.
______

Answer: No

Explanation:
When I subtract 2 \(\frac{3}{8}\) and 4 \(\frac{1}{8}\), the answer is not 2 \(\frac{2}{8}\).
The mixed number 4 \(\frac{1}{8}\) needs to be regrouped as a mixed number with a fraction greater than 1.
4 \(\frac{1}{8}\) = 3 \(\frac{9}{8}\)
So, 3 \(\frac{9}{8}\) – 2 \(\frac{3}{8}\) = 1 \(\frac{6}{8}\) or 1 \(\frac{3}{4}\)

Fractions and Properties of Addition – Page No. 448

Question 4.
Miguel’s class went to the state fair. The fairground is divided into sections. Rides are in \(\frac{6}{10}\) of the fairground. Games are in \(\frac{2}{10}\) of the fairground. Farm exhibits are in \(\frac{1}{10}\) of the fairground.
Part A
Use the model. What fraction of the fairground is rides and games?
Go Math Grade 4 Answer Key Chapter 7 Add and Subtract Fractions Page No. 448 Q 4
The fraction of the fairground with games and rides is ______ .
\(\frac{□}{□}\)

Answer: \(\frac{8}{10}\)

Explanation:
Given,
Miguel’s class went to the state fair. The fairground is divided into sections. Rides are in \(\frac{6}{10}\) of the fairground.
Games are in \(\frac{2}{10}\) of the fairground.
\(\frac{6}{10}\) + \(\frac{2}{10}\) = \(\frac{8}{10}\)

Question 4.
Part B
How much greater is the part of the fairground with rides than with farm exhibits? Explain how the model could be used to find the answer.
\(\frac{□}{□}\)

Answer: \(\frac{5}{10}\)

Explanation:
I could shade 6 sections to represent the section with the rides, and then I could cross out 1 section to represent the farm exhibits. This leaves 5 sections, so the part of the fairground with rides is 5/10 or 1/2 greater than the part with farm exhibits.

Question 5.
Rita is making chili. The recipe calls for 2 \(\frac{3}{4}\) cups of tomatoes. How many cups of tomatoes, written as a fraction greater than one, are used in the recipe?
_____ cups

Answer: 11/4 cups

Explanation:
Given,
Rita is making chili. The recipe calls for 2 \(\frac{3}{4}\) cups of tomatoes.
Convert from the mixed fraction to the improper fraction.
2 \(\frac{3}{4}\) = 11/4 cups

Question 6.
Lamar’s mom sells sports equipment online. She sold \(\frac{9}{10}\) of the sports equipment. Select a way \(\frac{9}{10}\) can be written as a sum of fractions. Mark all that apply.
Options:
a. \(\frac{1}{10}+\frac{1}{10}+\frac{1}{10}+\frac{1}{10}+\frac{2}{10}\)
b. \(\frac{3}{10}+\frac{2}{10}+\frac{3}{10}+\frac{1}{10}\)
c. \(\frac{2}{10}+\frac{2}{10}+\frac{2}{10}+\frac{2}{10}\)
d. \(\frac{4}{10}+\frac{1}{10}+\frac{1}{10}+\frac{3}{10}\)
e. \(\frac{4}{10}+\frac{3}{10}+\frac{1}{10}+\frac{1}{10}+\frac{1}{10}\)
f. \(\frac{2}{10}+\frac{2}{10}+\frac{2}{10}+\frac{3}{10}\)

Answer: \(\frac{3}{10}+\frac{2}{10}+\frac{3}{10}+\frac{1}{10}\)

Explanation:
a. \(\frac{1}{10}+\frac{1}{10}+\frac{1}{10}+\frac{1}{10}+\frac{2}{10}\) = 6/10 ≠ 9/10
b. \(\frac{3}{10}+\frac{2}{10}+\frac{3}{10}+\frac{1}{10}\) = 9/10
c. \(\frac{2}{10}+\frac{2}{10}+\frac{2}{10}+\frac{2}{10}\) = 8/10
d. \(\frac{4}{10}+\frac{1}{10}+\frac{1}{10}+\frac{3}{10}\) = 9/10
e. \(\frac{4}{10}+\frac{3}{10}+\frac{1}{10}+\frac{1}{10}+\frac{1}{10}\) = 10/10 ≠ 9/10
f. \(\frac{2}{10}+\frac{2}{10}+\frac{2}{10}+\frac{3}{10}\) = 9/10
Thus the suitable answers are b, d, f.

Fractions and Properties of Addition – Page No. 449

Question 7.
Bella brought \(\frac{8}{10}\) gallon of water on a hiking trip. She drank \(\frac{6}{10}\) gallon of water. How much water is left?
\(\frac{□}{□}\) gallons

Answer: \(\frac{2}{10}\) gallons

Explanation:
Given,
Bella brought \(\frac{8}{10}\) gallon of water on a hiking trip.
She drank \(\frac{6}{10}\) gallon of water.
To find how much water is left we have to subtract the two fractions.
\(\frac{8}{10}\) – \(\frac{6}{10}\) = \(\frac{2}{10}\) gallons

Question 8.
In a survey, \(\frac{6}{10}\) of the students chose Saturday and \(\frac{1}{10}\) chose Monday as their favorite day of the week. What fraction shows the students who chose Saturday or Monday as their favorite day?
Part A
Shade the model to show your answer.
Go Math Grade 4 Answer Key Chapter 7 Add and Subtract Fractions Page No. 449 Q 8
\(\frac{□}{□}\)

Answer: \(\frac{7}{10}\)

Explanation:
Given,
In a survey, \(\frac{6}{10}\) of the students chose Saturday and \(\frac{1}{10}\) chose Monday as their favorite day of the week.
\(\frac{6}{10}\) + \(\frac{1}{10}\) = \(\frac{7}{10}\)

Question 8.
Part B
How are the numerator and denominator of your answer related to the model? Explain.
Type below:
___________

Answer:
The numerator shows the number of parts shaded. The denominator shows the size of the parts.

Question 9.
Match the equation with the property used.
Go Math Grade 4 Answer Key Chapter 7 Add and Subtract Fractions Page No. 449 Q 9
Type below:
__________________

Answer:
Go-Math-Grade-4-Answer-Key-Chapter-7-Add-and-Subtract-Fractions-Page-No.-449-Q-9

Fractions and Properties of Addition – Page No. 450

Question 10.
For numbers 10a–10e, select Yes or No to show if the sum or difference is correct.
(a) \(\frac{2}{8}+\frac{1}{8}=\frac{3}{8}\)
i. yes
ii. no

Answer: Yes

Explanation:
Denominators are the same but the numerators are different. So, add the numerators.
\(\frac{2}{8}+\frac{1}{8}=\frac{3}{8}\)
Thus the above statement is true.

Question 10.
(b) \(\frac{4}{5}+\frac{1}{5}=\frac{5}{5}\)
i. yes
ii. no

Answer: Yes

Explanation:
Denominators are the same but the numerators are different. So, add the numerators.
\(\frac{4}{5}+\frac{1}{5}=\frac{5}{5}\)
Thus the above statement is true.

Question 10.
(c) \(\frac{4}{6}+\frac{1}{6}=\frac{5}{12}\)
i. yes
ii. no

Answer: No

Explanation:
Denominators are the same but the numerators are different. So, add the numerators.
\(\frac{4}{6}+\frac{1}{6}=\frac{5}{6}\)
Thus the above statement is false.

Question 10.
(d) \(\frac{6}{12}-\frac{4}{12}=\frac{2}{12}\)
i. yes
ii. no

Answer: Yes

Explanation:
Denominators are the same but the numerators are different. So, subtract the numerators.
\(\frac{6}{12}-\frac{4}{12}=\frac{2}{12}\)
Thus the above statement is true.

Question 10.
(e) \(\frac{7}{9}-\frac{2}{9}=\frac{9}{9}\)
i. yes
ii. no

Answer: No

Explanation:
Denominators are the same but the numerators are different. So, subtract the numerators.
\(\frac{7}{9}-\frac{2}{9}=\frac{5}{9}\)
Thus the above statement is false.

Question 11.
Gina has 5 \(\frac{2}{6}\) feet of silver ribbon and 2 \(\frac{4}{6}\) of gold ribbon. How much more silver ribbon does Gina have than gold ribbon?
______ \(\frac{□}{□}\) feet more silver ribbon.

Answer: 2 \(\frac{4}{6}\) feet more silver ribbon.

Explanation:
Given,
Gina has 5 \(\frac{2}{6}\) feet of silver ribbon and 2 \(\frac{4}{6}\) of gold ribbon.
5 \(\frac{2}{6}\) – 2 \(\frac{4}{6}\)
= \(\frac{32}{6}\) – \(\frac{16}{6}\)
= \(\frac{16}{6}\)
Convert from improper fraction to the mixed fraction.
2 \(\frac{4}{6}\) feet more silver ribbon
Therefore Gina has 2 \(\frac{4}{6}\) feet more silver ribbon than gold ribbon.

Question 12.
Jill is making a long cape. She needs 4 \(\frac{1}{3}\) yards of blue fabric for the outside of the cape. She needs 3 \(\frac{2}{3}\) yards of purple fabric for the lining of the cape.
Part A
Jill incorrectly subtracted the two mixed numbers to find how much more blue fabric than purple fabric she should buy. Her work is shown below.
\(4 \frac{1}{3}-3 \frac{2}{3}=\frac{12}{3}-\frac{9}{3}=\frac{3}{3}\)
Why is Jill’s work incorrect?
Type below:
__________________

Answer:
Jill changed only the whole number parts of the mixed number to thirds. She forgot to add the fraction part of the mixed number.

Question 12.
Part B
How much more blue fabric than purple fabric should Jill buy? Show your work.
\(\frac{□}{□}\)

Answer:
4 \(\frac{1}{3}\) – 3 \(\frac{2}{3}\)
= \(\frac{13}{3}\) – \(\frac{11}{3}\) = \(\frac{2}{3}\)
Jill should buy \(\frac{2}{3}\) yard more blue fabric than purple fabric.

Fractions and Properties of Addition – Page No. 451

Question 13.
Russ has two jars of glue. One jar is \(\frac{1}{5}\) full. The other jar is \(\frac{2}{5}\) full.
Go Math Grade 4 Answer Key Chapter 7 Add and Subtract Fractions Page No. 451 Q 13
Use the fractions to write an equation to find the amount of glue Russ has.
Go Math Grade 4 Answer Key Chapter 7 Add and Subtract Fractions Page No. 451 Question 13
Type below:
_________________

Answer:
Go-Math-Grade-4-Answer-Key-Chapter-7-Add-and-Subtract-Fractions-Page-No.-451-Question-13

Explanation:
Given,
Russ has two jars of glue. One jar is \(\frac{1}{5}\) full.
The other jar is \(\frac{2}{5}\) full.
\(\frac{1}{5}\) + \(\frac{2}{5}\) = \(\frac{3}{5}\)

Question 14.
Gertie ran \(\frac{3}{4}\) mile during physical education class. Sarah ran \(\frac{2}{4}\) mile during the same class. How much farther did Gertie run than Sarah? Shade the model to show your answer.
Go Math Grade 4 Answer Key Chapter 7 Add and Subtract Fractions Page No. 451 Q 14
\(\frac{□}{□}\)

Answer: \(\frac{1}{4}\)

Explanation:
Given that,
Gertie ran \(\frac{3}{4}\) mile during physical education class.
Sarah ran \(\frac{2}{4}\) mile during the same class.
\(\frac{3}{4}\) – \(\frac{2}{4}\) = \(\frac{1}{4}\)

Question 15.
Teresa planted marigolds in \(\frac{2}{8}\) of her garden and petunias in \(\frac{3}{8}\) of her garden. What fraction of the garden has marigolds and petunias?
\(\frac{□}{□}\)

Answer: \(\frac{5}{8}\)

Explanation:
Given,
Teresa planted marigolds in \(\frac{2}{8}\) of her garden and petunias in \(\frac{3}{8}\) of her garden.
Add both the fractions 2/8 and 3/8 to find the fraction of the garden has marigolds and petunias.
\(\frac{2}{8}\) + \(\frac{3}{8}\) = \(\frac{5}{8}\)

Question 16.
Draw a line to show the mixed number and fraction that have the same value.
Go Math Grade 4 Answer Key Chapter 7 Add and Subtract Fractions Page No. 451 Q 16

Answer:
Go-Math-Grade-4-Answer-Key-Chapter-7-Add-and-Subtract-Fractions-Page-No.-451-Q-16

Question 17.
Each day, Tally’s baby sister eats \(\frac{1}{4}\) cup of rice cereal in the morning and \(\frac{1}{4}\) cup of rice cereal in the afternoon. It will take Tally’s sister Go Math Grade 4 Answer Key Chapter 7 Add and Subtract Fractions Page No. 451 Q 17 days to eat 2 cups of rice cereal.
Type below:
_________________

Answer: 4

Explanation:
Each day she eats 1/2 cups of rice. But we want to know how long it will take to each 2 cups worth. so lets make an equation.
1/2 × x = 2
x = 4
Thus It will take 4 days to eat 2 cups of rice cereal.

Fractions and Properties of Addition – Page No. 452

Question 18.
Three girls are selling cases of popcorn to earn money for a band trip. In week 1, Emily sold 2 \(\frac{3}{4}\) cases, Brenda sold 4 \(\frac{1}{4}\) cases, and Shannon sold 3 \(\frac{1}{2}\) cases.
Part A
How many cases of popcorn have the girls sold in all? Explain how you found your answer.
______ \(\frac{□}{□}\)

Answer: 10 \(\frac{1}{2}\) cases

Explanation:
Given,
Three girls are selling cases of popcorn to earn money for a band trip. In week 1, Emily sold 2 \(\frac{3}{4}\) cases, Brenda sold 4 \(\frac{1}{4}\) cases, and Shannon sold 3 \(\frac{1}{2}\) cases.
First I add the whole numbers 2 + 4 + 3 = 9 cases. Then I add the fractions by combining 3/4 + 1/4 into one whole.
So, 9 + 1 + 1/2 = 10 \(\frac{1}{2}\) cases

Question 18.
Part B
The girls must sell a total of 35 cases in order to have enough money for the trip. Suppose they sell the same amount in week 2 and week 3 of the sale as in week 1. Will the girls have sold enough cases of popcorn to go on the trip? Explain.
______

Answer: No

Explanation:
Given,
The girls must sell a total of 35 cases in order to have enough money for the trip.
Suppose they sell the same amount in week 2 and week 3 of the sale as in week 1.
If I add the sales from the 3 weeks, or 10 1/2 + 10 1/2 + 10 1/2, the sum is only 31 1/2 cases of popcorn. Thus is less than 35 cases.

Question 19.
Henry ate \(\frac{3}{8}\) of a sandwich. Keith ate \(\frac{4}{8}\) of the same sandwich. How much more of the sandwich did Keith eat than Henry?
\(\frac{□}{□}\) of the sandwich

Answer: \(\frac{1}{8}\) of the sandwich

Explanation:
Given,
Henry ate \(\frac{3}{8}\) of a sandwich.
Keith ate \(\frac{4}{8}\) of the same sandwich.
\(\frac{4}{8}\) – \(\frac{3}{8}\) = \(\frac{1}{8}\) of the sandwich

Question 20.
For numbers 20a–20d, choose True or False for each sentence.
a. \(1 \frac{4}{9}+2 \frac{6}{9}\) is equal to 4 \(\frac{1}{9}\)
i. True
ii. False

Answer: True

Explanation:
\(1 \frac{4}{9}+2 \frac{6}{9}\) = 4 \(\frac{1}{9}\)
First add the whole numbers
1 + 2 = 3
4/9 + 6/9 = 10/9
Convert it into the mixed fractions
10/9 = 1 \(\frac{1}{9}\)
3 + 1 \(\frac{1}{9}\) = 4 \(\frac{1}{9}\)
Thus the above statement is true.

Question 20.
b. \(3 \frac{5}{6}+2 \frac{3}{6}\) is equal to 5 \(\frac{2}{6}\)
i. True
ii. False

Answer: False

Explanation:
First add the whole numbers
3 + 2 = 5
5/6 + 3/6 = 8/6
Convert it into the mixed fractions
8/6 = 1 \(\frac{2}{6}\)
5 + 1 \(\frac{2}{6}\) = 6 \(\frac{2}{6}\)
Thus the above statement is false.

Question 20.
c. \(4 \frac{5}{8}-2 \frac{4}{8}\) is equal to 2 \(\frac{3}{8}\)
i. True
ii. False

Answer: False

Explanation:
\(4 \frac{5}{8}-2 \frac{4}{8}\)
First subtract the whole numbers
4 – 2 = 2
5/8 – 4/8 = 1/8
= 2 \(\frac{1}{8}\)
Thus the above statement is false.

Question 20.
d. \(5 \frac{5}{8}-3 \frac{2}{8}\) is equal to 2 \(\frac{3}{8}\)
i. True
ii. False

Answer: True

Explanation:
\(5 \frac{5}{8}-3 \frac{2}{8}\)
5 – 3 = 2
5/8 – 2/8 = 3/8
= 2 \(\frac{3}{8}\)
\(5 \frac{5}{8}-3 \frac{2}{8}\) = 2 \(\frac{3}{8}\)
Thus the above statement is true.

Question 21.
Justin lives 4 \(\frac{3}{5}\) miles from his grandfather’s house. Write the mixed number as a fraction greater than one.
4 \(\frac{3}{5}\) = \(\frac{□}{□}\)

Answer: \(\frac{23}{5}\)

Explanation:
Justin lives 4 \(\frac{3}{5}\) miles from his grandfather’s house.
Convert from mixed fractions to an improper fraction.
4 \(\frac{3}{5}\) = \(\frac{23}{5}\)

Fractions and Properties of Addition – Page No. 457

Question 1.
Use the picture to complete the equations.
Go Math Grade 4 Answer Key Chapter 7 Add and Subtract Fractions Page No. 457 Q 1
\(\frac{3}{4}\) = _ + _ + _
\(\frac{3}{4}\) = _ × \(\frac{1}{4}\)
Type below:
___________

Answer: 3

Explanation:
\(\frac{3}{4}\)
The unit fraction of \(\frac{3}{4}\) is \(\frac{1}{4}\) + \(\frac{1}{4}\) + \(\frac{1}{4}\)
\(\frac{3}{4}\) = 3 × \(\frac{1}{4}\)
Thus the whole number is 3.

Write the fraction as a product of a whole number and a unit fraction.

Question 2.
\(\frac{4}{5}\) = ______ × \(\frac{1}{5}\)

Answer: 4

Explanation:
The unit fraction for \(\frac{4}{5}\) is \(\frac{1}{5}\) + \(\frac{1}{5}\) + \(\frac{1}{5}\) + \(\frac{1}{5}\)
\(\frac{4}{5}\) = 4 × \(\frac{1}{5}\)
Thus the whole number is 4.

Question 3.
\(\frac{3}{10}\) = ______ × \(\frac{1}{10}\)

Answer: 3

Explanation:
The unit fraction for \(\frac{3}{10}\) is \(\frac{1}{10}\) + \(\frac{1}{10}\) + \(\frac{1}{10}\)
\(\frac{3}{10}\) = 3 × \(\frac{1}{10}\)
Thus the whole number is 3.

Question 4.
\(\frac{8}{3}\) = ______ × \(\frac{1}{3}\)

Answer: 8

Explanation:
The unit fraction for \(\frac{8}{3}\) is \(\frac{1}{3}\) + \(\frac{1}{3}\) + \(\frac{1}{3}\) + \(\frac{1}{3}\) + \(\frac{1}{3}\) + \(\frac{1}{3}\) + \(\frac{1}{3}\) + \(\frac{1}{3}\)
\(\frac{8}{3}\) = 8 × \(\frac{1}{3}\)
Thus the whole number is 8.

List the next four multiples of the unit fraction.

Question 5.
\(\frac{1}{6}\) ,
Type below:
___________

Answer: 2/6, 3/6, 4/6, 5/6

Explanation:
The next four multiples of \(\frac{1}{6}\) is \(\frac{2}{6}\) , \(\frac{3}{6}\) , \(\frac{4}{6}\) , \(\frac{5}{6}\)

Question 6.
\(\frac{1}{3}\) ,
Type below:
___________

Answer: 2/3, 3/3, 4/3, 5/3

Explanation:
The next four multiples of \(\frac{1}{3}\) is \(\frac{2}{3}\), \(\frac{3}{3}\), \(\frac{4}{3}\) and \(\frac{5}{3}\)

Write the fraction as a product of a whole number and a unit fraction.

Question 7.
\(\frac{5}{6}\) = ______ × \(\frac{1}{6}\)

Answer: 5

Explanation:
The unit fraction for \(\frac{5}{6}\) is \(\frac{1}{6}\) + \(\frac{1}{6}\) + \(\frac{1}{6}\) + \(\frac{1}{6}\) + \(\frac{1}{6}\)
\(\frac{5}{6}\) = 5 × \(\frac{1}{6}\)
Thus the whole number is 5.

Question 8.
\(\frac{9}{4}\) = ______ × \(\frac{1}{4}\)

Answer: 9

Explanation:
The unit fraction for \(\frac{9}{4}\) is \(\frac{1}{4}\) + \(\frac{1}{4}\) + \(\frac{1}{4}\) + \(\frac{1}{4}\) + \(\frac{1}{4}\) + \(\frac{1}{4}\) + \(\frac{1}{4}\) + \(\frac{1}{4}\) + \(\frac{1}{4}\)
\(\frac{9}{4}\) = 9 × \(\frac{1}{4}\)
Thus the whole number is 9.

Question 9.
\(\frac{3}{100}\) = ______ × \(\frac{1}{100}\)

Answer: 3

Explanation:
The unit fraction for \(\frac{3}{100}\) is \(\frac{1}{100}\) + \(\frac{1}{100}\) + \(\frac{1}{100}\)
\(\frac{3}{100}\) = 3 × \(\frac{1}{100}\)
Thus the whole number is 3.

List the next four multiples of the unit fraction.

Question 10.
\(\frac{1}{10}\) ,
Type below:
___________

Answer: 2/10, 3/10, 4/10, 5/10

Explanation:
The next four multiples of \(\frac{1}{10}\) is 2/10, 3/10, 4/10, 5/10

Question 11.
\(\frac{1}{8}\) ,
Type below:
___________

Answer: 2/8, 3/8, 4/8, 5/8

Explanation:
The next four multiples of \(\frac{1}{8}\) is 2/8, 3/8, 4/8, 5/8.

Question 12.
Robyn uses \(\frac{1}{2}\) cup of blueberries to make each loaf of blueberry bread. Explain how many loaves of blueberry bread she can make with 2 \(\frac{1}{2}\) cups of blueberries.
_____ loaves of blueberry bread

Answer: 5 loaves of blueberry bread

Explanation:
Given,
Robyn uses \(\frac{1}{2}\) cup of blueberries to make each loaf of blueberry bread.
The unit fraction for 2 \(\frac{1}{2}\) is \(\frac{1}{2}\) + \(\frac{1}{2}\) + \(\frac{1}{2}\) + \(\frac{1}{2}\) + \(\frac{1}{2}\)
=  5 loaves of blueberry bread

Question 13.
Nigel cut a loaf of bread into 12 equal slices. His family ate some of the bread and now \(\frac{5}{12}\) of the loaf is left. Nigel wants to put each of the leftover slices in its own bag. How many bags does Nigel need?
_____ bags

Answer: 5 bags

Explanation:
Given,
Nigel cut a loaf of bread into 12 equal slices. His family ate some of the bread and now \(\frac{5}{12}\) of the loaf is left.
Nigel wants to put each of the leftover slices in its own bag.
\(\frac{5}{12}\) = \(\frac{1}{12}\) + \(\frac{1}{12}\) + \(\frac{1}{12}\) + \(\frac{1}{12}\) + \(\frac{1}{12}\)
= 5 bags

Question 14.
Which fraction is a multiple of \(\frac{1}{5}\)? Mark all that apply.
Options:
a. \(\frac{4}{5}\)
b. \(\frac{5}{7}\)
c. \(\frac{5}{9}\)
d. \(\frac{3}{5}\)

Answer: \(\frac{4}{5}\), \(\frac{3}{5}\)

Explanation:
The multiples of the \(\frac{1}{5}\) is \(\frac{4}{5}\), \(\frac{3}{5}\).

Fractions and Properties of Addition – Page No. 458

Sense or Nonsense?

Question 15.
Whose statement makes sense? Whose statement is nonsense? Explain your reasoning.
Go Math Grade 4 Answer Key Chapter 7 Add and Subtract Fractions Page No. 458 Q 15
Type below:
_________________

Answer: The boy’s statement makes sense. Because 4/5 is not the multiple of 1/4.

Question 15.
For the statement that is nonsense, write a new statement that makes sense.
Type below:
_________________

Answer: 4/5 is the multiple of 1/5.

Conclusion:

Just click on the links available above and practice the concepts of add and subtract fractions for homework help & standard tests. Help students to practice all chapter 7 questions from Go Math Answer Key to write the answers perfectly. For more questions just go with our Go Math Grade 4 Answer Key Chapter 7 Add and Subtract Fractions Homework Practice FL pdf article.

Go Math Grade 4 Answer Key Homework Practice FL Chapter 6 Fraction Equivalence and Comparison

go-math-grade-4-chapter-7-add-and-subtract-fractions-pages-131-153-answer-key

Free PDF Download links of Go Math Grade 4 Answer Key Homework Practice FL Chapter 6 Fraction Equivalence and Comparison are prevailing here for free. Simply refer to them once and make use of the Go Math Grade 4 Chapter 6 Fraction Equivalence and Comparison Answer Key Homework Practice FL to practice all questions covered in the Go Math Grade 4 Chapter 6 Textbook. The biggest goal of preparing these Answer Key Homework Practice FL by our team is to make students understand the concepts and secure well in the exams.

 

Go Math Grade 4 Answer Key Homework Practice FL Chapter 6 Fraction Equivalence and Comparison

Find all step-by-step explanations for every question from these Answer key of grade 4 HMH Go Math Ch 6 Homework Practice FL. Hence, take a look at the list of the toppers recommended HMH Go Math Grade 4 Solution Key for Chapter 6 Fraction Equivalence and Comparison and learn all the basics easily. In Chapter 6 go math HMH grade 4 Solution Key, you will find the topics like Equivalent Fractions, Comparing fractions, pair of fractions as a pair of fractions with a common denominator, and so on.

Lesson: 1 – Equivalent Fractions

Lesson: 2

Lesson: 3

Lesson: 4

Lesson: 5

Lesson: 6

Lesson: 7

Lesson: 8

Lesson: 9

Common Core – Fraction Equivalence and Comparison – Page No. 113

Equivalent Fractions

Use the model to write an equivalent fraction.

Question 1.
Go Math Grade 4 Answer Key Homework Practice FL Chapter 6 Fraction Equivalence and Comparison Common Core - Fraction Equivalence and Comparison img 1
\(\frac{4}{6}=\frac{2}{3}\)

Explanation:
The first image has 4 parts shaded out of 6 parts. Divide 8/10 with 2. You will get 2/3. That means 2 parts are shaded out of 3 parts.

Question 2.
Go Math Grade 4 Answer Key Homework Practice FL Chapter 6 Fraction Equivalence and Comparison Common Core - Fraction Equivalence and Comparison img 2
\(\frac{3}{4}\) = \(\frac{□}{□}\)

Answer: \(\frac{3}{4}\) = \(\frac{6}{8}\)

Explanation:
The first image has 3 parts shaded out of 4 parts. Multiply 8/10 with 2. You will get 6/8. That means 6 parts are shaded out of 8 parts.

Tell whether the fractions are equivalent. Write = or ≠.

Question 3.
\(\frac{8}{10}\) _____ \(\frac{4}{5}\)

Answer: \(\frac{8}{10}\) = \(\frac{4}{5}\)

Explanation:
Multiply the numerator and denominator of 4/5 with 2.
8/10 = (2/2) × (4/5)
= 8/10
So, 8/10 = 4/5.

Question 4.
\(\frac{1}{2}\) _____ \(\frac{7}{12}\)

Answer: \(\frac{1}{2}\) ≠ \(\frac{7}{12}\)

Explanation:
Multiply the numerator and denominator of 1/2 with 6
1/2 = (6/6) x (1/2)
= (6/12)
So, 1/2 ≠ 7/12

Question 5.
\(\frac{3}{4}\) _____ \(\frac{8}{12}\)

Answer: \(\frac{3}{4}\) ≠ \(\frac{8}{12}\)

Explanation:
Multiply the numerator and denominator of 3/4 with 3.
3/4 = (3/3) × (3/4)
= (9/12)
So, 3/4 ≠ 8/12

Question 6.
\(\frac{2}{3}\) _____ \(\frac{4}{6}\)

Answer: \(\frac{2}{3}\) = \(\frac{4}{6}\)

Explanation:
Multiply the numerator and denominator of 2/3 with 2.
2/3 = (2/2) × (2/3)
= 4/6
So, 2/3 = 4/6.

Question 7.
\(\frac{5}{8}\) _____ \(\frac{4}{10}\)

Answer: \(\frac{5}{8}\) ≠ \(\frac{4}{10}\)

Explanation:
Multiply the numerator and denominator of 5/8 with 2
5 / 8 =(2/2) x (5/ 8)
= (10/16)
So, 5/8 ≠ 4/10

Question 8.
\(\frac{2}{6}\) _____ \(\frac{4}{12}\)

Answer:

Question 9.
\(\frac{20}{100}\) _____ \(\frac{1}{5}\)

Answer: \(\frac{20}{100}\) = \(\frac{1}{5}\)

Explanation:
Cross Multiply the 20/100 with 20/20
20/100 = (20/20) x (20/100)
= (1/5)
So, 20/100 = 1/5.

Question 10.
\(\frac{5}{8}\) _____ \(\frac{9}{10}\)

Answer: \(\frac{5}{8}\) ≠ \(\frac{9}{10}\)

Explanation:
Multiply the numerator and denominator of 5/8 with 2
5/8 = (2/2) x (5/8)
= 10/16
So, 5/8 ≠ 9/10

Question 11.
Jamal finished \(\frac{5}{6}\) of his homework. Margaret finished \(\frac{3}{4}\) of her homework, and Steve finished \(\frac{10}{12}\) of his homework. Which two students finished the same amount of homework?
_________

Answer: Jamal and Steve

Explanation:
As per the given data,
Jamal finished work = 5 /6 of his homework
Margaret finished work = 3 /4th of her homework
Steve finished work = 10/12 of his homework
Multiply the numerator and denominator of 5/6 with 2
Then, (2/2) x (5/6) = 10/12
Then, Jamal and Steve finished the same amount of homework.

Question 12.
Sophia’s vegetable garden is divided into 12 equal sections. She plants carrots in 8 of the sections. Write two fractions that are equivalent to the part of Sophia’s garden that is planted with carrots.
Type below:
_________

Answer: 2/3 and 4/6

Explanation:
As per the given data,
Sophia’s vegetable garden is divided into 12 equal sections
She plants carrots in 8 of the sections out of 12 sections = 8/12
By simplifying the 8/12, we will get 4/6
Again simplify the 4 /6 by dividing method, you will get 2 /3
2/3 = (2/2) x (2/3)
= 4/6
Then, the equivalent fractions are 2/3, 4 /6

Common Core – Fraction Equivalence and Comparison – Page No. 114

Question 1.
A rectangle is divided into 8 equal parts. Two parts are shaded. Which fraction is equivalent to the shaded area of the rectangle?
Options:
a. \(\frac{1}{4}\)
b. \(\frac{1}{3}\)
c. \(\frac{2}{6}\)
d. \(\frac{3}{4}\)

Answer: \(\frac{1}{4}\)

Explanation:
As per the given data,
A rectangle is divided into 8 equal parts
Two parts are shaded
Then, the shaded area of rectangle = 2/8
By simplifying the 2/8, you will get 1/4
So, the shaded area of rectangle = 1/4
Thus the correct answer is option a.

Question 2.
Jeff uses 3 fifth-size strips to model \(\frac{3}{5}\). He wants to use tenth-size strips to model an equivalent fraction. How many tenth-size strips will he need?
Options:
a. 10
b. 6
c. 5
d. 3

Answer: 6

Explanation:
From the given data,
Jeff uses 3 fifth –size strips to model = 3 / 5 size strips
If he wants to use tenth – size strips to an equivalent fraction = 1 / 10 size strips
The number of strips = x
(1/10) x = 3/5
x = 30/5
then, the required number of tenth size trips = 6
Thus the correct answer is option b.

Question 3.
Cassidy places 40 stamps on each of 8 album pages. How many stamps does she place in all?
Options:
a. 300
b. 320
c. 360
d. 380

Answer: 320

Explanation:
As per the given data,
Cassidy places 40 stamps on each of 8 album pages = 8 x 40 = 320
So, totally placed stamps on album pages by Cassidy = 320 stamps
Thus the correct answer is option b.

Question 4.
Maria and 3 friends have 1,200 soccer cards. If they share the soccer cards equally, how many will each person receive?
Options:
a. 30
b. 40
c. 300
d. 400

Answer: 300

Explanation:
As per the given data,
Maria and 3 friends have 1200 soccer cards
If soccer cards shared equally by four members = 1200/4 = 300
Then, each person received soccer cards = 300
Thus the correct answer is option c.

Question 5.
Six groups of students sell 162 balloons at the school carnival. There are 3 students in each group. If each student sells the same number of balloons, how many balloons does each student sell?
Options:
a. 9
b. 18
c. 27
d. 54

Answer: 9

Explanation:
As per the given, data,
Six groups of students sell 162 balloons at the school carnival
There are 3 students in each group
Then, total number of students in 6 groups = 6 x 3 = 18
If each student sells the same number of balloons = 162 / 18 = 9
Number of balloons sells by each student = 9
Thus the correct answer is option a.

Question 6.
Four students each made a list of prime numbers.
Eric: 5, 7, 17, 23
Maya: 3, 5, 13, 17
Bella: 2, 3, 17, 19
Jordan: 7, 11, 13, 21
Who made an error and included a composite number?
Options:
a. Eric
b. Maya
c. Bella
d. Jordan

Answer: Jordan

Explanation:
As per the given data,
Four students each made a list of prime numbers.
Eric: 5, 7, 17, 23
Maya: 3, 5, 13, 17
Bella: 2, 3, 17, 19
Jordan: 7, 11, 13, 21
21 is not a prime number.
So, An error made by Jordan.
Thus the correct answer is option d.

Common Core – Fraction Equivalence and Comparison – Page No. 115

Write two equivalent fractions for each.

Question 1.
Go Math Grade 4 Answer Key Homework Practice FL Chapter 6 Fraction Equivalence and Comparison Common Core - Fraction Equivalence and Comparison img 3

Question 2.
\(\frac{2}{3}\)
Type below:
_________

Answer: 4/6 and 8/12

Explanation:
2/3
(2/3) x (2/2) = 4/6
(2/3) x (4/4) = 8/12
Then, the equivalent fractions of 2/3 = 4/6 and 8/12

Question 3.
\(\frac{1}{2}\)
Type below:
_________

Answer: 2/4 and 4/8

Explanation:
1/2
(1/2) x (2/2) = 2/4
(1/2) x (4/4) = 4/8
Then, the equivalent fractions of 1/2 = 2/4, 4/8

Question 4.
\(\frac{4}{5}\)
Type below:
_________

Answer: 8/10 and 80/100

Explanation:
4/5
(4/5) x (2/2) = 8/10
(4/5) x (20/20) = 80/100
Then, the equivalent fractions of 4/5 = 8/10 and 80/100

Tell whether the fractions are equivalent. Write # or ≠.

Question 5.
\(\frac{1}{4}\) _____ \(\frac{3}{12}\)

Answer: \(\frac{1}{4}\) = \(\frac{3}{12}\)

Explanation:
1/4
Multiply the numerator and denominator of 1/4 with 3
Then, (1/4) x (3/3) = 3/12
So, 1/4 = 3/12

Question 6.
\(\frac{4}{5}\) _____ \(\frac{5}{10}\)

Answer: \(\frac{4}{5}\) ≠ \(\frac{5}{10}\)

Explanation:
4/5
Multiply numerator and denominator of 4/5 with 2
(4/5) x (2/2) = 8/10
Then 4/5 ≠ 5/10

Question 7.
\(\frac{3}{8}\) _____ \(\frac{2}{6}\)

Answer: \(\frac{3}{8}\) ≠ \(\frac{2}{6}\)

Explanation:
3/8 ≠ 2/6

Question 8.
\(\frac{3}{4}\) _____ \(\frac{6}{8}\)

Answer: \(\frac{3}{4}\) = \(\frac{6}{8}\)

Explanation:
3/4
Multiply the numerator and denominator of 3/4 with 2
Then, (3/4) x (2/2) = 6/8
So, 3/4 = 6/8

Question 9.
\(\frac{5}{6}\) _____ \(\frac{10}{12}\)

Answer: \(\frac{5}{6}\) = \(\frac{10}{12}\)

Explanation:
5/6
Multiply the numerator and denominator with 2
(5/6) x (2/2) = 10/12
So, 5/6 = 10/12

Question 10.
\(\frac{6}{12}\) _____ \(\frac{5}{8}\)

Answer: \(\frac{6}{12}\) ≠ \(\frac{5}{8}\)

Explanation:
6/12 ≠ 5/8

Question 11.
\(\frac{2}{5}\) _____ \(\frac{4}{10}\)

Answer: \(\frac{2}{5}\) = \(\frac{4}{10}\)

Explanation:
2/5
Multiply the numerator and denominator of 2/5 with 2
(2/5) x (2/2) = 4/10
So, 2/5 = 4/10

Question 12.
\(\frac{2}{4}\) _____ \(\frac{3}{12}\)

Answer: \(\frac{2}{4}\) ≠ \(\frac{3}{12}\)

Explanation:
2/4
Multiply the numerator and denominator of 2/4 with 3
(2/4) x (3/3) = 6/12
So, 2/4 ≠ 3/ 12

Question 13.
Jan has a 12-ounce milkshake. Four ounces in the milkshake are vanilla, and the rest is chocolate. What are two equivalent fractions that represent the fraction of the milkshake that is vanilla?
Type below:
_________

Answer: 1/3 and 2/6

Explanation:
As per the given data,
Jan has a 12-ounce milkshake
Four ounces in the milkshake are vanilla = 4/12 = 1/3
Then, 8-ounces in milkshake are chocolate = 8/12 = 2/3
4/12 = 1/3
By multiplying 1/3 with 2
(1/3) x (2/2) = 2/6
So, the equivalent fractions of vanilla milkshake are 1/3 and 2/6.

Question 14.
Kareem lives \(\frac{4}{10}\) of a mile from the mall. Write two equivalent fractions that show what fraction of a mile Kareem lives from the mall.
Type below:
_________

Answer: 2/5 and 8/20

Explanation:
As per the given data,
Kareem lives 4/10 of a mile from the mall
To find the equivalent fractions of 4/10
Simplify the 4/10 = 2/5
Multiply the numerator and denominator of 2/5 with 4
(2/5) x (4/4) = 8/20
Then, the equivalent fraction of a mile Kareem lives from the mall = 2/5 and 8/20

Common Core – Fraction Equivalence and Comparison – Page No. 116

Question 1.
Jessie colored a poster. She colored \(\frac{2}{5}\) of the poster red. Which fraction is equivalent to \(\frac{2}{5}\) ?
Options:
a. \(\frac{4}{10}\)
b. \(\frac{7}{10}\)
c. \(\frac{4}{5}\)
d. \(\frac{2}{2}\)

Answer: \(\frac{4}{10}\)

Explanation:
As per the given data,
Jessie colored a poster
She colored 2/5th of the poster red
Multiply the numerator and denominator of 2/5 with 2
Then, (2/5) x (2/2) = 4 /10
So, the equivalent fraction of 2/5 is 4/10.
Thus the correct answer is option a.

Question 2.
Jessie colored a poster. She colored \(\frac{1}{4}\) of the poster red. Which fraction is equivalent to \(\frac{1}{4}\) ?
Options:
a. \(\frac{2}{5}, \frac{3}{12}\)
b. \(\frac{2}{8}, \frac{4}{12}\)
c. \(\frac{3}{4}, \frac{6}{8}\)
d. \(\frac{2}{8}, \frac{3}{12}\)

Answer: \(\frac{2}{8}, \frac{3}{12}\)

Explanation:
As per the given data,
Marcus makes a punch that is 1/4th of cranberry juice
Multiply the numerator and denominator of 1/4 with 2
Then, (1/4) x (2/2) = 2/8
Multiply the numerator and denominator of 1/4 with 3
Then, (1/4) x (3/3) = 3/12
Equivalent fractions of 1/4 are 2/8 and 3/12.
Thus the correct answer is option d.

Question 3.
An electronics store sells a large flat screen television for $1,699. Last month, the store sold 8 of these television sets. About how much money did the store make on the television sets?
Options:
a. $160,000
b. $16,000
c. $8,000
d. $1,600

Answer: $16,000

Explanation:
As per the given data,
An electronics store sells a large flat-screen television for $1,699
Last month, the store sold 8 of these television sets = 8 x $1,699 = $13,952. The money is about to $16,000.
Thus the correct answer is option b.

Question 4.
Matthew has 18 sets of baseball cards. Each set has 12 cards. About how many baseball cards does Matthew have in all?
Options:
a. 300
b. 200
c. 150
d. 100

Answer: 200

Explanation:
From the given data,
Matthew has 18 sets of basketball cards
Each set has 12 cards = 12 x 18 = 216
Total number of basketball cards with Matthew = 216.
So, it is near to 200.
Thus the correct answer is option b.

Question 5.
Diana had 41 stickers. She put them in 7 equal groups. She put as many as possible in each group. She gave the leftover stickers to her sister. How many stickers did Diana give to her sister?
Options:
a. 3
b. 4
c. 5
d. 6

Answer: 6

Explanation:
As per the given data,
Diana has 41 stickers
She put them in 7 equal groups = 41/7
= 5 (remaining 6)
She gave the leftover stickers to her sister
The number of stickers Diana give to her sister = 6
Thus the correct answer is option d.

Question 6.
Christopher wrote the number pattern below. The first term is 8.
8, 6, 9, 7, 10, …
Which is a rule for the pattern?
Options:
a. Add 2, add 3.
b. Add 6, subtract 3.
c. Subtract 6, add 3.
d. Subtract 2, add 3

Answer: Subtract 2, add 3

Explanation:
From the given data,
Christopher wrote the number pattern = 8, 6, 9, 7, 10, …..
The first number in the pattern = 8
8 – 2 = 6 + 3 = 9 – 2 = 7 +3 = 10 ….
So, the rule for the above pattern is to subtract 2, add 3.
Thus the correct answer is option d.

Common Core – Fraction Equivalence and Comparison – Page No. 117

Write the fraction in simplest form.

Question 1.
Go Math Grade 4 Answer Key Homework Practice FL Chapter 6 Fraction Equivalence and Comparison Common Core - Fraction Equivalence and Comparison img 4

Explanation:
To write the 6/10 in a simplest form
Divide the numerator and denominator of 6/10 with 2
(6 ÷2)/(10 ÷2) = 3/5
So, the simplest form of 6/10 = 3/5

Question 2.
\(\frac{6}{8}\) = \(\frac{□}{□}\)

Answer: \(\frac{3}{4}\)

Explanation:
To write the 6/8in a simplest form
Divide the numerator and denominator of 6/8 with 2
(6 ÷2)/(8 ÷2) = 3/4
So, the simplest form of 6/8 = 3/4

Question 3.
\(\frac{5}{5}\) = \(\frac{□}{□}\)

Answer: \(\frac{1}{1}\)

Explanation:
To write the 5/5in a simplest form
Divide the numerator and denominator of 5/5 with 5
(5 ÷5)/(5 ÷5) = 1/1
So, the simplest form of 5/5 = 1

Question 4.
\(\frac{8}{12}\) = \(\frac{□}{□}\)

Answer: \(\frac{2}{3}\)

Explanation:
To write the 8/12in a simplest form
Divide the numerator and denominator of 8/12 with 4
(8 ÷4)/(12 ÷4) = 2/3
So, the simplest form of 8/12 = 2/3

Question 5.
\(\frac{100}{100}\) = \(\frac{□}{□}\)

Answer: 1

Explanation:
Any number divided by the same number will be always equal to 1.
The simplest form of 100/100 = 1

Question 6.
\(\frac{2}{6}\) = \(\frac{□}{□}\)

Answer: \(\frac{1}{3}\)

Explanation:
To write the 2/6in a simplest form
Divide the numerator and denominator of 2/6 with 2
(2 ÷2)/(6 ÷2) = 1/3
So, the simplest form of 2/6 = 1/3

Question 7.
\(\frac{2}{8}\) = \(\frac{□}{□}\)

Answer: \(\frac{1}{4}\)

Explanation:
To write the 2/8in a simplest form
Divide the numerator and denominator of 2/8 with 2
(2 ÷2)/(8 ÷2) = 1/4
So, the simplest form of 2/8 = 1/4

Question 8.
\(\frac{4}{10}\) = \(\frac{□}{□}\)

Answer: \(\frac{2}{5}\)

Explanation:
To write the 4/10 in a simplest form
Divide the numerator and denominator of 4 /10 with 2
(4 ÷2)/(10 ÷2) = 2/5
So, the simplest form of 4/10 = 2/5

Tell whether the fractions are equivalent. Write = or ≠. (if you dont have ≠on your keyboard, copy and paste this one: ≠ )

Question 9.
\(\frac{6}{12}\) ______ \(\frac{1}{12}\)

Answer: \(\frac{6}{12}\) ≠ \(\frac{1}{12}\)

Explanation:
6/12 ≠ 1/12

Question 10.
\(\frac{3}{4}\) ______ \(\frac{5}{6}\)

Answer: \(\frac{3}{4}\) ≠ \(\frac{5}{6}\)

Explanation:
3/4 ≠ 5/6

Question 11.
\(\frac{6}{10}\) ______ \(\frac{3}{5}\)

Answer: \(\frac{6}{10}\) = \(\frac{3}{5}\)

Explanation:
6/10
Divide the numerator and denominator of 6/10 with 2
(6 ÷ 2)/( 10 ÷ 2) = 3/5
So, 6/10 = 3/5

Question 12.
\(\frac{3}{12}\) ______ \(\frac{1}{3}\)

Answer: \(\frac{3}{12}\) ≠ \(\frac{1}{3}\)

Explanation:
3/12 ≠ 1/3

Question 13.
\(\frac{6}{10}\) ______ \(\frac{60}{100}\)

Answer: \(\frac{6}{10}\) = \(\frac{60}{100}\)

Explanation:
6/10
Multiply the numerator and denominator of 6/10 with 10
(6 x 10)/(10 x 10) = 60/100
So, 6/10 = 60/100

Question 14.
\(\frac{11}{12}\) ______ \(\frac{9}{10}\)

Answer: \(\frac{11}{12}\) ≠ \(\frac{9}{10}\)

Explanation:
11/12 ≠ 9/10

Question 15.
\(\frac{2}{5}\) ______ \(\frac{8}{20}\)

Answer: \(\frac{2}{5}\) = \(\frac{8}{20}\)

Explanation:
2/5
Multiply the numerator and denominator of 2/5 with 4
(2 x 4)/(5 x 4) = 8/20
So, 2/5 = 8/20

Question 16.
\(\frac{4}{8}\) ______ \(\frac{1}{2}\)

Answer: \(\frac{4}{8}\) = \(\frac{1}{2}\)

Explanation:
4/8
Divide the numerator and denominator of 4/8 with 4
(4 x 4)/(8 x 4) = 1/2
So, 4/8 = 1/2

Question 17.
At Memorial Hospital, 9 of the 12 babies born on Tuesday were boys. In simplest form, what fraction of the babies born on Tuesday were boys?
\(\frac{□}{□}\)

Answer: \(\frac{3}{4}\)

Explanation:
As per the given data,
At the memorial hospital, 9 of the 12 babies born on Tuesday were boys = 9/12
Divide the numerator and denominator of 9/12 with 3
(9 ÷ 3)/(12 ÷ 3) = 3/4
So, in the simplest form
3/4 of the babies born on Tuesday were boys.

Question 18.
Cristina uses a ruler to measure the length of her math textbook. She says that the book is \(\frac{4}{10}\) meter long. Is her measurement in simplest form? If not, what is the length of the book in simplest form?
\(\frac{□}{□}\)

Answer: \(\frac{2}{5}\)

Explanation:
As per the given data,
Cristiana uses a ruler to measure the length of her math textbook
She says that the book is 4/10meter long
It is not in the simplest form
Divide the numerator and denominator of 4/10 with 2
(4÷ 2)/( 10 ÷ 2) = 2/5
The length of the book in the simplest form = 2/5.

Common Core – Fraction Equivalence and Comparison – Page No. 118

Question 1.
Six out of the 12 members of the school choir are boys. In simplest form, what fraction of the choir is boys?
Options:
a. \(\frac{1}{6}\)
b. \(\frac{6}{12}\)
c. \(\frac{1}{2}\)
d. \(\frac{12}{6}\)

Answer: \(\frac{1}{2}\)

Explanation:
As per the given data,
Six out of the 12 members of the school choir are boys = 6/12
To write the simplest form of 6/12, divide the numerator and denominator with 6
Then, (6 ÷ 6)/(12 ÷ 6) = 1/2
In the simplest form, 1/2 of the choir is boys.
Thus the correct answer is option c.

Question 2.
Six out of the 12 members of the school choir are boys. In simplest form, what fraction of the choir is boys?
Options:
a. \(\frac{5}{6}\)
b. \(\frac{6}{8}\)
c. \(\frac{8}{10}\)
d. \(\frac{2}{12}\)

Answer: \(\frac{5}{6}\)

Explanation:
5/6 is in the simplest form
6/8 simplest form = 3/4
8/10 simplest form = 4/5
2/12 simplest form = 1/6
Thus the correct answer is option a.

Question 3.
Each of the 23 students in Ms. Evans’ class raised $45 for the school by selling coupon books. How much money did the class raise in all?
Options:
a. $207
b. $225
c. $1,025
d. $1,035

Answer: $1,035

Explanation:
As per the given data,
Each of the 23 students in Ms. Evan’s class raised $45 for the school by selling coupon books
= 23 x $45
= $1,035
Thus the correct answer is option d.

Question 4.
Which pair of numbers below have 4 and 6 as common factors?
Options:
a. 12, 18
b. 20, 24
c. 28, 30
d. 36, 48

Answer: 36, 48

Explanation:
36, 48
Here, 36 = 4 x 9
= 2 x 2 x 3 x 3
48 = 6 x 8
= 2 x 3 x 4 x 2
Thus the correct answer is option d.

Question 5.
Bart uses \(\frac{3}{12}\) cup milk to make muffins. Which fraction is equivalent to \(\frac{3}{12}\) ?
Options:
a. \(\frac{1}{4}\)
b. \(\frac{1}{3}\)
c. \(\frac{1}{2}\)
d. \(\frac{2}{3}\)

Answer: \(\frac{1}{4}\)

Explanation:
As per the given data,
Bart uses 3/12 cup milk to make muffins
Divide the fraction with 3
(3 ÷ 3)/(12 ÷ 3) = 1/4
So, the equivalent fraction for 3/12 = 1/4
Thus the correct answer is option a.

Question 6.
Ashley bought 4 packages of juice boxes. There are 6 juice boxes in each package. She gave 2 juice boxes to each of 3 friends. How many juice boxes does Ashley have left?
Options:
a. 24
b. 22
c. 18
d. 12

Answer: 18

Explanation:
As per the given data,
Ashley bought 4 packages of juice boxes
There are 6 juice boxes in each package = 6 x 4 = 24
She gave 2 juice boxes to each of 3 friends = 2 x 3 = 6 juice boxes
So, 24 – 6 = 18
Total number of juice boxes left with Ashley = 18
Thus the correct answer is option c.

Common Core – Fraction Equivalence and Comparison – Page No. 119

Write the pair of fractions as a pair of fractions with a common denominator.

Question 1.
\(\frac{2}{3} \text { and } \frac{3}{4}\)
Go Math Grade 4 Answer Key Homework Practice FL Chapter 6 Fraction Equivalence and Comparison Common Core - Fraction Equivalence and Comparison img 5

Explanation:
2/3 and 3/4
List the multiples of 3 = 3, 6, 9, 12, 15, 18, 21, 24, …
List the multiples of 4 = 4, 8, 12, 16, 20, …
Common multiple of 3 and 4 is 12
For the Common pair of fractions, multiply the common denominator with fractions
That is, (2 x 12) ÷( 3 x 12) and ( 3 x 12 ) ÷ ( 4 x 12)
So, common pair of fractions = 8/12 and 9/12

Question 2.
\(\frac{1}{4} \text { and } \frac{2}{3}\)
Type below:
_________

Answer: 3/4 and 8/12

Explanation:
1/4 and 2/3
List the multiples of 4 = 4, 8, 12, 16, 20, …
List the multiples of 3 = 3, 6, 9, 12, 15, 18, …
Common multiple of 4 and 3 is 12
For the Common pair of fractions, multiply the common denominator with fractions
That is, (1 x 12) ÷( 4 x 12) and ( 2 x 12 ) ÷ ( 3 x 12)
So, common pair of fractions = 3/12 and 8/12

Question 3.
\(\frac{3}{10} \text { and } \frac{1}{2}\)
Type below:
_________

Answer: 3/10 and 5/10

Explanation:
3/10 and 1/2
List the multiples of 10 = 10, 20, 30, 40, 50, ….
List the multiples of 2 = 2, 4, 6, 8, 10, 12, 14, ….
Common multiple of 10 and 2 is 10
For the Common pair of fractions, multiply the common denominator with fractions
That is, (3 x 10) ÷( 10 x 10) and ( 1 x 10 ) ÷ ( 2 x 10)
So, common pair of fractions = 3/10 and 5/10

Question 4.
\(\frac{3}{5} \text { and } \frac{3}{4}\)
Type below:
_________

Answer: 12/20 and 15/20

Explanation:
3/5 and 3/4
List the multiples of 5 = 5, 10, 15, 20, 25, 30, ….
List the multiples of 4 = 4, 8, 12, 16, 20, 24, …
Common multiple of 5 and 4 is 20
For the Common pair of fractions, multiply the common denominator with fractions
That is, (3 x 20) ÷( 5 x 20) and ( 3 x 20 ) ÷ ( 4 x 20)
So, common pair of fractions = 12/20 and 15/20

Question 5.
\(\frac{2}{4} \text { and } \frac{7}{8}\)
Type below:
_________

Answer: 4/8 and 7/8

Explanation:
2/4 and 7/8
List the multiples of 4 = 4, 8, 12, 16, 20, 24, …
List the multiples of 8 = 8, 16, 24, 32, 40, ….
Common multiple of 4 and 8 is 8
For the Common pair of fractions, multiply the common denominator with fractions
That is, (2 x 8) ÷( 4 x 8) and ( 7 x 8 ) ÷ ( 8 x 8)
So, common pair of fractions = 4/8 and 7/8

Question 6.
\(\frac{2}{3} \text { and } \frac{5}{12}\)
Type below:
_________

Answer: 8/12 and 5/12

Explanation:
2/3 and 5/12
List the multiples of 3 = 3, 6, 9, 12, 15, 18, …
List the multiples of 12 = 12, 24, 36, 48, 60, …
Common multiple of 3 and 12 is 12
For the Common pair of fractions, multiply the common denominator with fractions
That is, (2 x 12) ÷( 3 x 12) and ( 5 x 12 ) ÷ ( 12 x 12)
So, common pair of fractions = 8/12 and 5/12

Question 7.
\(\frac{1}{4} \text { and } \frac{1}{6}\)
Type below:
_________

Answer: 3/12 and 2/12

Explanation:
1/4 and 1/6
List the multiples of 4 = 4, 8, 12, 16, 20, 24, …
List the multiples of 6 = 6, 12, 18, 24, 30, …
Common multiple of 4 and 6 is 12
For the Common pair of fractions, multiply the common denominator with fractions
That is, (1 x 12) ÷( 4 x 12) and ( 1 x 12 ) ÷ ( 6 x 12)
So, common pair of fractions = 3/12 and 2/12.

Tell whether the fractions are equivalent. Write = or ≠.

Question 8.
\(\frac{1}{2}\) ______ \(\frac{2}{5}\)

Answer: \(\frac{1}{2}\) ≠ \(\frac{2}{5}\)

Explanation:
Multiply the numerator and denominator of 1/2 with 2
(1 x 2) ÷ (2 x 2) = 2/4
So, 1/2 ≠ 2/5

Question 9.
\(\frac{1}{2}\) ______ \(\frac{3}{6}\)

Answer: \(\frac{1}{2}\) = \(\frac{3}{6}\)

Explanation:
1/2
Multiply the numerator and denominator of 1/2 with 3
(1 x 3) ÷ (2 x 3) = 3/6
So, 1/2 = 3/6

Question 10.
\(\frac{3}{4}\) ______ \(\frac{5}{6}\)

Answer: \(\frac{3}{4}\) ≠ \(\frac{5}{6}\)

Explanation:
3/4 ≠ 5/6

Question 11.
\(\frac{6}{10}\) ______ \(\frac{3}{5}\)

Answer: \(\frac{6}{10}\) = \(\frac{3}{5}\)

Explanation:
6/10
Divide the numerator and denominator of 6/10 with 2
(6 ÷ 2)/(10 ÷2) = 3/5
So, 6/10 = 3/5

Question 12.
\(\frac{6}{8}\) ______ \(\frac{3}{4}\)

Answer: \(\frac{6}{8}\) = \(\frac{3}{4}\)

Explanation:
6/8
Divide the numerator and denominator of 6/8 with 2
(6 ÷2)/(8 ÷2) = 3/4
So, 6/8 = 3/4

Question 13.
\(\frac{3}{4}\) ______ \(\frac{2}{3}\)

Answer: \(\frac{3}{4}\) ≠ \(\frac{2}{3}\)

Explanation:
3/4 ≠ 2/3

Question 14.
\(\frac{2}{10}\) ______ \(\frac{4}{5}\)

Answer: \(\frac{2}{10}\) ≠ \(\frac{4}{5}\)

Explanation:
2/10
Divide the numerator and denominator of 2/10 with 8
(8 ÷ 2)/(10 ÷ 2) = 4/5
So, 2/10 ≠ 4/5

Question 15.
\(\frac{1}{4}\) ______ \(\frac{3}{12}\)

Answer: \(\frac{1}{4}\) = \(\frac{3}{12}\)

Explanation:
1/4
Multiply the numerator and denominator of 1/4 with 3
(1 x 3)/(4 x 3) = 3/12
So, 1/4 = 3/12

Question 16.
Adam drew two same size rectangles and divided them into the same number of equal parts. He shaded \(\frac{1}{3}\) of one rectangle and \(\frac{1}{4}\) of other rectangle. What is the least number of parts into which both rectangles could be divided?
_________

Answer: 12 parts

Explanation:
As per the given data,
Adam drew two same size rectangles and divided them into the same number of equal parts
He shaded 1/3 of one rectangle
1/4 of another rectangle
List the multiples of 3 = 3, 6, 9, 12, 15, 18, …
List the multiples of 4 = 4, 8, 12, 16, 20, …
A common multiple of 3 and 4 is 12
So, the least number of parts which rectangles could be divided = 12 parts

Question 17.
Mera painted equal sections of her bedroom wall to make a pattern. She painted \(\frac{2}{5}\) of the wall white and \(\frac{1}{2}\) of the wall lavender. Write an equivalent fraction for each using a common denominator.
Type below:
_________

Answer: 1/2 are 4/10 and 5/10

Explanation:
As per the given data,
Mera painted equal sections of her bedroom wall to make a pattern
She painted 2/5 of the wall white and 1/2 of the wall lavender
List the multiples of 5 = 5, 10, 15, 20, 25, 30, …
List the multiples of 2 = 2 ,4, 6, 8, 10, 12, 14, …
The common denominator of 2/5 and 1/2 = 10
Multiply the 2/5 and 1/2 with 10
(2 x 10)/(5 x 10) and (1 x 10)/(2 x 10)
4/10 and 5/10
So, common fractions of 2/5 and 1/2 are 4/10 and 5/10.

Common Core – Fraction Equivalence and Comparison – Page No. 120

Question 1.
Which of the following is a common denominator of \(\frac{1}{4}\) and \(\frac{5}{6}\) ?
Options:
a. 8
b. 9
c. 12
d. 15

Answer: 12

Explanation:
The common denominator of 1/4 and 5/6
List the multiples of 4 = 4, 8, 12, 16, 20, 24, …
List the multiples of 6 = 6, 12, 18, 24, 30, ….
So, the common denominator of 1/4 and 5/6 is 12
The correct answer is option c.

Question 2.
Two fractions have a common denominator of 8. Which of the following could be the two fractions?
Options:
a. \(\frac{1}{2} \text { and } \frac{2}{3}\)
b. \(\frac{1}{4} \text { and } \frac{1}{2}\)
c. \(\frac{3}{4} \text { and } \frac{1}{6}\)
d. \(\frac{1}{2} \text { and } \frac{4}{5}\)

Answer: \(\frac{1}{4} \text { and } \frac{1}{2}\)

Explanation:
As per the given data,
Two fractions have a common denominator of 8
a. 1/2 and 2/3
List the multiples of 2 = 2, 4, 6, 8,10, ….
List the multiples of 3 = 3, 6, 9, 12, …
There is no common denominator of 8 for 1/2 and 2/3
b. 1/4 and 1 /2
List the multiples of 2 = 2, 4, 6, 8,10, ….
List the multiples of 4 = 4, 8, 12, 16, …
Here, the common denominator of 1 /4 and 1 /2 is 8
So, the answer is 1/4 and 1/2
The correct answer is option b.

Question 3.
Which number is 100,000 more than seven hundred two thousand, eighty-three?
Options:
a. 703,083
b. 712,083
c. 730,083
d. 802,083

Answer: 802,083

Explanation:
100,000 + 702,083 = 802,083
The correct answer is option d.

Question 4.
Aiden baked 8 dozen muffins. How many total muffins did he bake?
Options:
a. 64
b. 80
c. 96
d. 104

Answer: 96

Explanation:
As per the given data,
Aiden baked 8 dozen muffins
1 dozen = 12
then, 8 dozens = 12 x 8 = 96
So, Aiden baked total 96 muffins.
The correct answer is option c.

Question 5.
On a bulletin board, the principal, Ms. Gomez, put 115 photos of the fourthgrade students in her school. She put the photos in 5 equal rows. How many photos did she put in each row?
Options:
a. 21
b. 23
c. 25
d. 32

Answer: 23

Explanation:
As per the given data,
On a bulletin board, the principal, Ms. Gomez, put 115 photos of the fourth-grade students in her school
She put the photos in 5 equal rows
Then, number of photos in each row = 115/5 = 23
So, Ms. Gomez put photos in each row = 23
The correct answer is option b.

Question 6.
Judy uses 12 tiles to make a mosaic. Eight of the tiles are blue. What fraction, in simplest form, represents the tiles that are blue?
Options:
a. \(\frac{2}{3}\)
b. \(\frac{2}{5}\)
c. \(\frac{3}{4}\)
d. \(\frac{12}{18}\)

Answer: \(\frac{2}{3}\)

Explanation:
As per the given data,
Judy uses 12 tiles to make a mosaic
Eight of the tiles are blue = 8/12
Divide the numerator and denominator of 8/12 with 4
(8 ÷ 4)/(12 ÷ 4) = 2/3
The simplest form of 8/12 is 2/3
The correct answer is option a.

Common Core – Fraction Equivalence and Comparison – Page No. 121

Question 1.
Miranda is braiding her hair. Then she will attach beads to the braid. She wants \(\frac{1}{3}\) of the beads to be red. If the greatest number of beads that will fit on the braid is 12, what other fractions could represent the part of the beads that are red?
Go Math Grade 4 Answer Key Homework Practice FL Chapter 6 Fraction Equivalence and Comparison Common Core - Fraction Equivalence and Comparison img 6

Explanation:
Miranda is braiding her hair. Then she will attach beads to the braid. She wants 1/3 of the beads to be red. If the greatest number of beads that will fit on the braid is 12.
1/3 × 2/2 = 2/6
1/3 × 3/3 = 3/9
1/3 × 4/4 = 4/12

Question 2.
Ms. Groves has trays of paints for students in her art class. Each tray has 5 colors. One of the colors is purple. What fraction of the colors in 20 trays is purple?
\(\frac{□}{□}\)

Answer: \(\frac{1}{5}\)

Explanation:
Given,
Ms. Groves has trays of paints for students in her art class.
Each tray has 5 colors.
If you have 20 trays that are 100 colors with 20 being purple. 20/ 100 is 1/5.

Question 3.
Miguel is making an obstacle course for field day. At the end of every sixth of the course, there is a tire. At the end of every third of the course, there is a cone. At the end of every half of the course, there is a hurdle. At which locations of the course will people need to go through more than one obstacle?
Type below:
_________

Answer: 1/3, 1/2, 2/3 and final locations

Explanation:
We have three fractions with different denominators: sixths, thirds, and halves.
The first step is to make all the denominators equal for 1/6, 1/3, 1/2.
In this case, we want sixths since LCM(2, 3, 6) = 6
since 1/3 = 2/6, and 1/2 = 3/6. Now we can start solving.
1. There are six tires at the following: 1/6, 2/6, 3/6, 4/6, 5/6, and 6/6.
2. There are three cones at the following (G.C.F.): 2/6 (or 1/3), 4/6 (or 2/3), and 6/6 (or 3/3).
3. There are two hurdles at the following (G.C.F.): 3/6 (or 1/2) and 6/6 (or 2/2).
We look for common numbers.
1. On 2/6, there are two obstacles: a tire and a cone.
2. On 3/6, there are two obstacles: a tire and a hurdle.
3. On 4/6, there are two obstacles: a tire and a cone.
4. At 6/6, there are three obstacles: a tire, cone, and a hurdle.
2/6 = 1/3
3/6 = 1/2
4/6 = 2/3
6/6 = 1
The answers are 1/3, 1/2, 2/3, and 1.

Question 4.
Preston works in a bakery where he puts muffins in boxes. He makes the following table to remind himself how many blueberry muffins should go in each box.
Go Math Grade 4 Answer Key Homework Practice FL Chapter 6 Fraction Equivalence and Comparison Common Core - Fraction Equivalence and Comparison img 7
How many blueberry muffins should Preston put in a box with 36 muffins?
_________

Answer: 12 blueberry muffins

Explanation:
Preston works in a bakery where he puts muffins in boxes. He makes the following table to remind himself how many blueberry muffins should go in each box.
So, he had 2 blueberry muffins out of 6 muffins.
2/6 × 2/2 = 4/12. 4 blueberry muffins out of 12 muffins.
2/6 × 4/4 = 8/24. 8 blueberry muffins out of 24 muffins.
2/6 × 6/6 = 12/36. 12 blueberry muffins out of 36 muffins.

Common Core – Fraction Equivalence and Comparison – Page No. 122

Question 1.
A used bookstore will trade 2 of its books for 3 of yours. If Val brings in 18 books to trade, how many books can she get from the store?
Options:
a. 9
b. 12
c. 18
d. 27

Answer: 12

Explanation:
A used bookstore will trade 2 of its books for 3 of yours.
If Val brings in 18 books to trade 2/3 × 6/6 = 12/18, she gets 12 books.
Thus the correct answer is option b.

Question 2.
Every \(\frac{1}{2}\) hour Naomi stretches her neck; every \(\frac{1}{3}\) hour she stretches her legs; and every \(\frac{1}{6}\) hour she stretches her arms. Which parts of her body will Naomi stretch when \(\frac{2}{3}\) of an hour has passed?
Options:
a. neck and legs
b. neck and arms
c. legs and arms
d. none

Answer: legs and arms

Explanation:
Summing \(\frac{1}{2}\)‘s only gives integer values giving 1, 2, 3, 4…or
integer values +\(\frac{1}{2}\) and 0 + \(\frac{1}{2}\) = \(\frac{1}{2}\), 1 \(\frac{1}{2}\), 2 \(\frac{1}{2}\)…
So neck is excluded
Every \(\frac{1}{3}\): \(\frac{1}{3}\) + \(\frac{1}{2}\) = \(\frac{2}{3}\)
Legs will be stretched at \(\frac{2}{3}\) hour
Every \(\frac{1}{6}\): \(\frac{1}{6}\) + \(\frac{1}{6}\) + \(\frac{1}{6}\) + \(\frac{1}{6}\) = \(\frac{4}{6}\)
Divide top and bottom by 2 giving:
(4 ÷ 2)/(6 ÷ 2) = \(\frac{2}{3}\)
Arms will be stretched at \(\frac{2}{3}\) hour.
Thus the correct answer is option c.

Question 3.
At the beginning of the year, the Wong family car had been driven 14,539 miles. At the end of the year, their car had been driven 21,844 miles. How many miles did the Wong family drive their car during that year?
Options:
a. 6,315 miles
b. 7,295 miles
c. 7,305 miles
d. 36,383 miles

Answer: 7,305 miles

Explanation:
If at the beginning of the year, the Wong family’s car had driven 14539 miles and at the end of the year, it had driven 21844 miles, then subtract 14539 from 21844 to determine the difference between the two values, which will tell you how many miles the Wong family drove their car for during the year.
21844 – 14539 = 7305 miles
Thus the correct answer is option c.

Question 4.
Widget Company made 3,600 widgets in 4 hours. They made the same number of widgets each hour. How many widgets did the company make in one hour?
Options:
a. 80
b. 90
c. 800
d. 900

Answer: 900

Explanation:
3,600 widgets in 4 hours therefore 3,600 / 4 for one hour = 900 widgets 900 widgets in one hour.
Thus the correct answer is option d.

Question 5.
Tyler is thinking of a number that is divisible by 2 and by 3. By which of the following numbers must Tyler’s number also be divisible?
Options:
a. 6
b. 8
c. 9
d. 12

Answer: 6

Explanation:
The number 6 is divisible by 2 and by 3.
Thus the correct answer is option a.

Question 6.
Jessica drew a circle divided into 8 equal parts. She shaded 6 of the parts. Which fraction is equivalent to the part of the circle that is shaded?
Options:
a. \(\frac{2}{3}\)
b. \(\frac{3}{4}\)
c. \(\frac{10}{16}\)
d. \(\frac{12}{18}\)

Answer: \(\frac{3}{4}\)

Explanation:
Jessica drew a circle divided into 8 equal parts. She shaded 6 of the parts.
6/8 = 3/4
Thus the correct answer is option b.

Common Core – Fraction Equivalence and Comparison – Page No. 123

Compare. Write < or > .

Question 1.
Go Math Grade 4 Answer Key Homework Practice FL Chapter 6 Fraction Equivalence and Comparison Common Core - Fraction Equivalence and Comparison img 8

Answer:
18 < 610
Explanation:
Go Math Grade 4 Answer Key Chapter 6 Fraction Equivalence and Comparison Common Core Compare Fractions Using Benchmarks img 20

Question 2.
\(\frac{4}{12}\) ______ \(\frac{4}{6}\)

Answer:
4/12 < 4/6

Explanation:
4/12 and 4/6
4/12 is less than 1/2
4/6 is greater than 1/2
So, 4/12 < 4/6

Question 3.
\(\frac{2}{8}\) ______ \(\frac{1}{2}\)

Answer:
2/8 < 1/2

Explanation:
2/8 and 1/2
2/8 is less than 1/2
1/2 is equal to 1/2
So, 2/8 < 1/2

Question 4.
\(\frac{3}{5}\) ______ \(\frac{3}{3}\)

Answer:
3/5 < 3/3

Explanation:
3/5 and 3/3
3/5 is greater than 1/2
3/3 is equal to 1
So, 3/5 < 3/3

Question 5.
\(\frac{7}{8}\) ______ \(\frac{5}{10}\)

Answer:
7/8 > 5/10

Explanation:
7/8 and 5/10
7/8 is greater than 1/2
5/10 is equal to 1/2
So, 5/10 < 7/8

Question 6.
\(\frac{9}{12}\) ______ \(\frac{1}{3}\)

Answer:
9/12 > 1/3

Explanation:
9/12 and 1/3
9/ 12 is greater than 1/2
1/3 is less than 1/2
1/3 < 9/12

Question 7.
\(\frac{4}{6}\) ______ \(\frac{7}{8}\)

Answer:
4/6 < 7/8

Explanation:
4/6 and 7/8
4/6 is greater than 1/2
7/8 is closer to 1
So, 4/6 < 7/8

Question 8.
\(\frac{2}{4}\) ______ \(\frac{2}{3}\)

Answer:
2/4 < 2/3

Explanation:
2/4 and 2/3
2/4 is equal to 1/2
2/3 is greater than 1/2
So, 2/4 < 2/3

Question 9.
\(\frac{3}{5}\) ______ \(\frac{1}{4}\)

Answer: 35 > 14

Explanation:
3/5 and 1/4
3/5 is greater than 1/2
1/4 is less than 1/2
So, 1/4 < 3/5

Question 10.
\(\frac{6}{10}\) ______ \(\frac{2}{5}\)

Answer:
6/10 > 2/5

Explanation:
6/10 and 2/5
6/10 is greater than 1/2
2/5 is less than 1/2
So, 2/5 < 6/10

Question 11.
\(\frac{1}{8}\) ______ \(\frac{2}{10}\)

Answer:
1/8 < 2/10

Explanation:
1/8 and 2/10
1/8 is less than 1/2
2/10 is less than 1/2 but greater than 1/8
So, 1/8 < 2/10

Question 12.
\(\frac{2}{3}\) ______ \(\frac{5}{12}\)

Answer:
2/3 > 5/12

Explanation:
2/3 and 5/12
2/3 is greater than 1/2
5/12 is less than 1/2
So, 5/12 < 2/3

Question 13.
\(\frac{4}{5}\) ______ \(\frac{5}{6}\)

Answer:
4/5< 5/6

Explanation:
4/5 and 5/6
4/5 is greater than 1/2
5/6 is greater than 1/2
Common denominator is 30
(4×6)/(5×6) and (5×5)/(6×5)
24/30 and 25/30
24/30 < 25/30
So, 4/5 < 5/6

Question 14.
\(\frac{3}{5}\) ______ \(\frac{5}{8}\)

Answer:
3/5 < 5/8

Explanation:
3/5 and 5/8
3/5 is greater than 1/2
5/8 is greater than 1/2
Common denominator is 40
(3×8)/(5×8) and (5×5)/(8×5)
24/40 and 25/ 40
24/40 < 25/40
3/5 < 5/8

Question 15.
\(\frac{8}{8}\) ______ \(\frac{3}{4}\)

Answer:
8/8 > 3/4

Explanation:
8/8 and 3/4
8/8 is equal to 1
3/4 is less than 1
3/4 < 8/8

Question 16.
Erika ran \(\frac{3}{8}\) mile. Maria ran \(\frac{3}{4}\) mile. Who ran farther?
__________

Answer: Maria

Explanation:
As per the data,
Erika ran 3/8 mile
Maria ran 3/4 mile
Multiply the numerator and denominator of 3/4 with 2
(3×2)/(4×2) = 6/8
3/8 < 6/8
So, 3/8 < 3/4
So, Maria ran faster than Erika

Question 17.
Carlos finished \(\frac{1}{3}\) of his art project on Monday. Tyler finished \(\frac{1}{2}\) of his art project on Monday. Who finished more of his art project on Monday?
__________

Answer:
Tyler

Explanation:
From the given data,
Carlos finished 1/3 of his art project on Monday
Tyler finished ½ of his art project on Monday
1/3 is less than 1/2
1/2 is equal to 1/2
So, 1/3 < 1/2
Then, Tyler finished more of his work on Monday

Common Core – Fraction Equivalence and Comparison – Page No. 124

Question 1.
Which symbol makes the statement true?
Go Math Grade 4 Answer Key Homework Practice FL Chapter 6 Fraction Equivalence and Comparison Common Core - Fraction Equivalence and Comparison img 9
Options:
a. >
b. <
c. =
d. none

Answer:
a. >

Explanation:
4/6 ? 3/8
By comparing 4/6 with 1/2, 4/6 > 1/2
By comparing 3/8 with 1/2, 3/8 < 1/2
So, 4/6 > 3/8
Thus the correct answer is option a.

Question 2.
Which of the following fractions is greater than \(\frac{3}{4}\)?
Options:
a. \(\frac{1}{4}\)
b. \(\frac{5}{6}\)
c. \(\frac{3}{8}\)
d. \(\frac{2}{3}\)

Answer: b. 5/6

Explanation:
From the given data,
By comparing the 3/4 with 1/2, 3/4 > 1/2
Same as above, compare the options with ½
a. 1/4 < 1/2
b. 5/6 > 1/2
c. 3/8 < 1/2
d. 2/3 > 1/2
5/6 and 2/3 are greater than the 1/2
So, compare the 5/6 with 2/3
Then, 5/6 > 2/3
So, 5/6 > 3/4
Thus the correct answer is option b.

Question 3.
Abigail is putting tiles on a table top. She needs 48 tiles for each of 8 rows. Each row will have 6 white tiles. The rest of the tiles will be purple. How many purple tiles will she need?
Options:
a. 432
b. 384
c. 336
d. 48

Answer:
c. 336

Explanation:
As per the given data
Abigail is putting tiles on a table top
Number of rows = 8
She needs 48 tiles for each of row = 48×8 = 384
Number of white tiles per row = 6×8 = 48
The rest of the tiles will be purple = 384 – 48 =336
So, the total number of purple color tiles = 336
Thus the correct answer is option c.

Question 4.
Each school bus going on the field trip holds 36 students and 4 adults. There are 6 filled buses on the field trip. How many people are going on the field trip?
Options:
a. 216
b. 240
c. 256
d. 360

Answer:
b. 240

Explanation:
From the given data,
Each school bus going on the field trip holds 36 students and 4 adults
There are 6 filled buses on the field trip
6 x (36 + 4) = 6 x 40 = 240
So, the total number of people on the field trip = 240
Thus the correct answer is option b.

Question 5.
Noah wants to display his 72 collector’s flags. He is going to put 6 flags in each row. How many rows of flags will he have in his display?
Options:
a. 12
b. 15
c. 18
d. 21

Answer:
a. 12

Explanation:
As mentioned in the data,
Noah wants to display his 72 collector’s flag
He is going to put 6 flags in each row = 6x = 72
X = 12
So, a total 12 number of rows of flags will have in his display.
Thus the correct answer is option a.

Question 6.
Julian wrote this number pattern on the board:
3, 10, 17, 24, 31, 38.
Which of the numbers in Julian’s pattern are composite numbers?
Options:
a. 3, 17, 31
b. 10, 24, 38
c. 10, 17, 38
d. 17, 24, 38

Answer:
b. 10, 24, 38

Explanation:
As per the given information
Julian wrote his number pattern on the board =3, 10, 17, 24, 31, 38
Factors of 3 = 1,3
Factors of 10 = 1,2,5,10
Factors of 17 = 1, 17
Factors of 24 = 1, 2, 3, 4, 6
Factors of 31 = 1, 31
Factors of 38 = 1, 2, 19, 38
So, the composite number is 10, 24, and 38, which numbers have more than 2 factors.
Thus the correct answer is option b.

Common Core – Fraction Equivalence and Comparison – Page No. 125

Compare. Write <, >, or =

Question 1.
Go Math Grade 4 Answer Key Homework Practice FL Chapter 6 Fraction Equivalence and Comparison Common Core - Fraction Equivalence and Comparison img 10
Answer:
1/5 < 2/10

Explanation:
Go Math Grade 4 Answer Key Chapter 6 Fraction Equivalence and Comparison Common Core Compare Fractions img 26

Question 2.
\(\frac{1}{5}\) ______ \(\frac{2}{10}\)

Answer:
1/5 = 2/10

Explanation:
1/5 and 2/10
Think: 10 is a common denominator
1/5 = (1×2) / (5×2) = 2/10
2/10 = 2/10
So, 1/5 = 2/10

Question 3.
\(\frac{2}{4}\) ______ \(\frac{2}{5}\)

Answer:
2/4 > 2/5

Explanation:
2/4 and 2/5
20 is a common denominator
2/4 = (2×5)/(4×5) = 10/20
2/5 = (2×4)/(5×4) = 8/20
10/20 > 8/20
So, 2/4 > 2/5

Question 4.
\(\frac{3}{5}\) ______ \(\frac{7}{10}\)

Answer:
3/5 < 7/10

Explanation:
3/5 and 7/10
10 is a common denominator
3/5 = (3×2)/(5×2) = 6/10
7/10
6/10 < 7/10
So, 3/5 < 7/10

Question 5.
\(\frac{4}{12}\) ______ \(\frac{1}{6}\)

Answer:
4/12 > 1/6

Explanation:
4/12 and 1/6
12 is a common denominator
4/12
1/6 = (1×2)/(6×2) = 2/12
4/12 > 2/12
So, 4/12 > 1/6

Question 6.
\(\frac{2}{6}\) ______ \(\frac{1}{3}\)

Answer:
2/6 = 1/3

Explanation:
2/6 and 1/3
6 is a common denominator
2/6
1/3 = (1×2)/(3×2) = 2/6
So, 2/6 =2/6
So, 2/6 = 1/3

Question 7.
\(\frac{1}{3}\) ______ \(\frac{2}{4}\)

Answer:
1/3 < 2/4

Explanation:
1/3 and 2/4
12 is a common denominator
1/3 = (1×4)/(3×4) = 4/12
2/4 = (2×3)/(4×3) = 6/12
4/12 < 6/12
So, 1/3 < 2/4

Question 8.
\(\frac{2}{5}\) ______ \(\frac{1}{2}\).

Answer:
2/5 < 1/2

Explanation:
2/5 and 1/2
10 is a common denominator
2/5 = (2×2)/(5×2) = 4/10
1/2 = (1×5)/(2×5) = 5/10
4/10 < 5/10
So, 2/5 < 1/2

Question 9.
\(\frac{4}{8}\) ______ \(\frac{2}{4}\)

Answer:
4/8 = 2/4

Explanation:
4/8 and 2/4
8 is a common denominator
4/8
2/4 = (2×2)/(4×2) = 4/8
2/4 = 4/8
So, 4/8 = 2/4

Question 10.
\(\frac{7}{12}\) ______ \(\frac{2}{4}\)

Answer:
7/12 < 2/4

Explanation:
7/12 and 2/4
12 is a common denominator
7/12
2/4 = (2×3)/(4×3) = 6/12
7/12 < 6/12
So, 7/12 < 2/4

Question 11.
\(\frac{1}{8}\) ______ \(\frac{3}{4}\)

Answer:
1/8 < 3/4

Explanation:
1/8 and 3/4
8 is a common denominator
1/8
3/4 = (3×2)/(4×2) = 6/8
1/8 < 6/8
So, 1/8 < 3/4

Question 12.
A recipe uses \(\frac{2}{3}\) of flour and \(\frac{5}{8}\) cup of blueberries. Is there more flour or more blueberries in the recipe?
more _________

Answer: flour

Explanation:
From the given data,
A recipe uses 2/3 of flour and 5/8 cup of blueberries
The common denominator is 24
2/3 = (2×8)/(3×8) = 16/24
5/8 = (5×3)/(8×3) = 15/24
16/24 > 15/24
So, 2/3 > 5/8
So, flour is more in the recipe

Question 13.
Peggy completed \(\frac{5}{6}\) of the math homework and Al completed \(\frac{4}{5}\) of the math homework. Did Peggy or Al complete more of the math homework?
_________

Answer:
Peggy completed more work than Al

Explanation:
As per the given data,
Peggy completed 5/6 of the math homework
A1 completed 4/5 of the math homework
30 is a common denominator
5/6 = (5×5)/(6×5) = 25/30
4/5 = (4×6)/(5×6) =24/30
25/30 > 24/30
So, 5/6 > 4/5
So, Peggy completed more work than Al

Common Core – Fraction Equivalence and Comparison – Page No. 126

Question 1.
Pedro fills a glass \(\frac{2}{4}\) full with orange juice. Which of the following fractions is greater than \(\frac{2}{4}\)?
Options:
a. \(\frac{3}{8}\)
b. \(\frac{4}{6}\)
c. \(\frac{5}{12}\)
d. \(\frac{1}{3}\)

Answer:
b. 46

Explanation:
46 > 24
Thus the correct answer is option b.

Question 2.
Today Ian wants to run less than \(\frac{7}{12}\) mile. Which of the following distances is less than \(\frac{7}{12}\) mile?
Options:
a. \(\frac{3}{4}\) mile
b. \(\frac{2}{3}\) mile
c. \(\frac{5}{6}\) mile
d. \(\frac{2}{4}\) mile

Answer:
d. 2/4 mile

Explanation:
2/4 is less than 7/12
Thus the correct answer is option d.

Question 3.
Ms. Davis traveled 372,645 miles last year on business. What is the value of 6 in 372,645?
Options:
a. 6
b. 60
c. 600
d. 6,000

Answer:
c. 600

Explanation:
Ms. Davis traveled 372, 645 miles last year on business
The value of 6 in 372,645 is 600.
Thus the correct answer is option c.

Question 4.
One section of an auditorium has 12 rows of seats. Each row has 13 seats. What is the total number of seats in that section?
Options:
a. 25
b. 144
c. 156
d. 169

Answer:
c. 156

Explanation:
From the given information
One section of an auditorium has 12 rows of seats
Each row has 13 seats = 13×12 = 156 seats
So, the total number of seats in the auditorium = 156 seats
Thus the correct answer is option c.

Question 5.
Sam has 12 black-and-white photos and 18 color photos. He wants to put the photos in equal rows so each row has either black-and-white photos only or color photos only. In how many rows can Sam arrange the photos?
Options:
a. 1, 2, 3, or 6 rows
b. 1, 3, 6, or 9 rows
c. 1, 2, or 4 rows
d. 1, 2, 3, 4, 6, or 9 rows

Answer:
a. 1, 2, 3, or 6 rows

Explanation:
As per the given information
Sam has 12 black and white photos 18 color photos
He wants to put the photos in equal rows
So each row has either black and white photos only or color photos only
H.C.F of 12 and 18 is 6
Rows of 6.
2 rows of black equal 12.
3 rows of white equals 18.
Thus the correct answer is option a.

Question 6.
The teacher writes \(\frac{10}{12}\) on the board. He asks students to write the fraction in the simplest form. Who writes the correct answer?
Options:
a. JoAnn writes \(\frac{10}{12}\)
b. Karen writes \(\frac{5}{12}\)
c. Lynn writes \(\frac{6}{5}\)
d. Mark writes \(\frac{5}{6}\)

Answer:
d. Mark writes 56

Explanation:
As per the given data,
The teacher writes 10/12 on the board
He asks students to write the fraction in simplest form
For the simplest form of 10/12, divide the 10/12 with 2
(10÷2)/(12÷2) = 5/6
5/6 is the simplest form of 10/12
So, Mark writes the correct answer.
Thus the correct answer is option d.

Common Core – Fraction Equivalence and Comparison – Page No. 127

Write the fractions in order from least to greatest.

Question 1.
\(\frac{5}{8}, \frac{2}{12}, \frac{8}{10}\)
Go Math Grade 4 Answer Key Homework Practice FL Chapter 6 Fraction Equivalence and Comparison Common Core - Fraction Equivalence and Comparison img 11

Answer:
2/12, 5/8, 8/10

Explanation:
Go Math Grade 4 Answer Key Chapter 6 Fraction Equivalence and Comparison Common Core Compare and Order Fractions img 29

Question 2.
\(\frac{1}{5}, \frac{2}{3}, \frac{5}{8}\)
Type below:
_________

Answer:
1/5, 5/8, 2/3

Explanation:
chapter 6 - compare fractions and order fractions- image4
1/5, 2/3, 5/8
1/5 is closer to 0
2/3 is greater than 1/2
5/8 greater than 1/2
1/5 < 5/8 < 2/3

Question 3.
\(\frac{1}{2}, \frac{2}{5}, \frac{6}{10}\)
Type below:
_________

Answer:
2/5, 1/2, 6/10

Explanation:
chapter 6 - compare fractions and order fractions- image5
1/2, 2/5, 6/10
1/2 is equal to 1/2
2/5 is less than 1/2
6/10 is greater than 1/2

Question 4.
\(\frac{4}{6}, \frac{7}{12}, \frac{5}{10}\)
Type below:
_________

Answer:
5/10 < 7/12 < 4/6

Explanation:
chapter 6 - compare fractions and order fractions- image6
4/6, 7/12, 5/10
4/6 is closer to 1
7/12 is greater than 1/2
5/10 is equal to 1/2

Question 5.
\(\frac{1}{4}, \frac{3}{6}, \frac{1}{8}\)
Type below:
_________

Answer:
18 < 14 < 36

Explanation:
chapter 6 - compare fractions and order fractions- image7
1/4, 3/6, 1/8
1/4 is less than 1/2
3/6 is equal to 1/2
1/8 is closer to 0

Question 6.
\(\frac{1}{8}, \frac{3}{6}, \frac{7}{12}\)
Type below:
_________

Answer:
1/8 < 7/12 < 3/6

Explanation:
chapter 6 - compare fractions and order fractions- image8
1/8, 3/6, 7/12
1/8 is closer to 0
3/6 is equal to 1/2
7/12 is greater than 1/2

Question 7.
\(\frac{8}{100}, \frac{3}{5}, \frac{7}{10}\)
Type below:
_________

Answer:
8/100 < 3/5 < 7/10

Explanation:

chapter 6 - compare fractions and order fractions- image9
8/100, 3/5, 7/10
8/100 is closer to 0
3/5 is greater than 1/2
7/10 is closer to 1

Question 8.
\(\frac{3}{4}, \frac{7}{8}, \frac{1}{5}\)
Type below:
_________

Answer:
15 < 34 < 78
Explanation:
chapter 6 - compare fractions and order fractions- image10
3/4, 7/8, 1/5
3/4 is greater than 1/2
7/8 is closer to 1
1/5 is closer to 0

Question 9.
Amy’s math notebook weighs \(\frac{1}{2}\) pound, her science notebook weighs \(\frac{7}{8}\) pound, and her history notebook weighs \(\frac{3}{4}\) pound. What are the weights in order from lightest to heaviest?
Type below:
__________

Answer:
12 pound, 34 pound, 78 pound

Explanation:
From the given data,
Amy’s math notebook weighs 1/2 pound
Science notebook weighs 7/8 pound
History notebook weighs 3/4 pound
7/8 is closer to 1
3/4 is greater than 1/2
1/2 < 3/4 < 7/8
So, Amy’s math notebook weight < history notebook weight < science notebook

Question 10.
Carl has three picture frames. The thicknesses of the frames are \(\frac{4}{5}\) inch, \(\frac{3}{12}\) inch, and \(\frac{5}{6}\) inch. What are the thicknesses in order from least to greatest?
Type below:
__________

Answer:
3/12 inch, 4/5 inch, 5/6 inch

Explanation:
As per the given data,
Carl has three picture frames
The thickness of the frames are 4/5 inch, 3/12 inch, 5/6 inch
4/5 is greater than 1/2
3/12 is less than 1/2
5/6 is closer to 1
3/12 < 4/5 < 5/6

Common Core – Fraction Equivalence and Comparison – Page No. 128

Question 1.
Juan’s three math quizzes this week took him \(\frac{1}{3}\) hour, \(\frac{4}{6}\) hour, and \(\frac{1}{5}\) hour to complete. Which list shows the lengths of time in order from least to greatest?
Options:
a. \(\frac{1}{3} hour, \frac{4}{6} hour, \frac{1}{5} hour\)
b. \(\frac{1}{5} hour, \frac{1}{3} hour, \frac{4}{6} hour\)
c. \(\frac{1}{3} hour, \frac{1}{5} hour, \frac{4}{6} hour\)
d. \(\frac{4}{6} hour, \frac{1}{3} hour, \frac{1}{5} hour\)

Answer:
b. 1/5 hour, 1/3 hour, 4/6 hour

Explanation:
From the given information
Juan’s three math quizzes this week took him 1/3 hour, 4/6 hour, and 1/5 hour
Compare 1/3 and 1/2
1/3 is less than 1/2
4/6 is greater than 1/2
1/5 is closer to 0
1/5 < 1/3 < 4/6
So, Juan’s math quizzes times from least to greatest is 1/5, 1/3, 4/6
Thus the correct answer is option b.

Question 2.
On three days last week, Maria ran \(\frac{3}{4}\) mile, \(\frac{7}{8}\) mile, and \(\frac{3}{5}\) mile. What are the distances in order from least to greatest?
Options:
a. \(\frac{3}{4} mile, \frac{7}{8} mile, \frac{3}{5} mile\)
b. \(\frac{3}{5} mile, \frac{3}{4} mile, \frac{7}{8} mile\)
c. \(\frac{7}{8} mile, \frac{3}{4} mile, \frac{3}{5} mile\)
d. \(\frac{7}{8} mile, \frac{3}{5} mile, \frac{3}{4} mile\)

Answer:
b. 3/5 mile, 3/4 mile, 7/8 mile

Explanation:
As per the information
On three days last week, Maria ran 3/4 mile, 7/8 mile, and 3/5 mile
3/4 is greater than 1/2
7/8 is closer to 1
3/5 is greater than 1/2
Compare 3/5 and 3/4
3/4 is greater than 3/5
So, 3/5 < 3/4 < 7/8
Distance from least to greatest is 3/5, 3/4 , 7/8
Thus the correct answer is option b.

Question 3.
Santiago collects 435 cents in nickels. How many nickels does he collect?
Options:
a. 58
b. 78
c. 85
d. 87

Answer:
d. 87

Explanation:
As per the given data,
Santiago collects 435 cents in nickels
1 nickel worth is 5 cents
Then, nickels per 435 cents = 435/5 = 87
So, Santiago collects 87 nickels.
Thus the correct answer is option d.

Question 4.
Lisa has three classes that each last 50 minutes. What is the total number of minutes the three classes last?
Options:
a. 15 minutes
b. 150 minutes
c. 153 minutes
d. 156 minutes

Answer:
b. 150 minutes

Explanation:
From the given data,
Lisa has three classes that each last 50 minutes
The total number of minutes the three classes last = 3×50 =150 minutes.
Thus the correct answer is option b.

Question 5.
Some students were asked to write a composite number. Which student did NOT write a composite number?
Options:
a. Alicia wrote 2.
b. Bob wrote 9.
c. Arianna wrote 15.
d. Daniel wrote 21.

Answer:
a. Alicia wrote 2.

Explanation:
As per the information
Some students were asked to write a composite number
a. Alicia wrote 2
Factors of 2 is 1 and 2
b. Bob wrote 9
Factors of 9 is 1, 3, 9
c. Arianna wrote 15
Factors of 15 is 1, 3, 5, 15
d. Daniel wrote 21
Factors of 21 is 1,3,7,21
So, Alicia did not write a composite number.
Thus the correct answer is option a.

Question 6.
Mrs. Carmel serves \(\frac{6}{8}\) of a loaf of bread with dinner. Which fraction is equivalent to \(\frac{6}{8}\) ?
Options:
a. \(\frac{2}{4}\)
b. \(\frac{9}{16}\)
c. \(\frac{2}{3}\)
d. \(\frac{3}{4}\)

Answer: d. 3/4

Explanation:
As per the given information
Mrs. Carmel serves 6/8 of a loaf of bread with dinner
To find the equivalent fraction of 6/8, simplify the 6/8 by dividing with the 2
(6÷2)/(8÷2) = 3/4
So, the equivalent fraction of 6/8 is 3/4
Thus the correct answer is option d.

Common Core – Fraction Equivalence and Comparison – Page No. 129

Lesson 6.1

Tell whether the fractions are equivalent. Write = or ≠.

Question 1.
\(\frac{5}{10}\) ______ \(\frac{1}{2}\)

Answer: \(\frac{5}{10}\) = \(\frac{1}{2}\)

Explanation:
Divide \(\frac{5}{10}\) by 2
\(\frac{5}{10}\) ÷ 5 = \(\frac{1}{2}\)
So, \(\frac{5}{10}\) = \(\frac{1}{2}\)

Question 2.
\(\frac{2}{3}\) ______ \(\frac{3}{6}\)

Answer: \(\frac{2}{3}\) ≠ \(\frac{3}{6}\)

Explanation:
The fraction \(\frac{2}{3}\) is not equal to \(\frac{3}{6}\)

Question 3.
\(\frac{6}{8}\) ______ \(\frac{3}{4}\)

Answer: \(\frac{6}{8}\) = \(\frac{3}{4}\)

Explanation:
Divide \(\frac{6}{8}\) by 2.
\(\frac{6}{8}\) ÷ 2 = \(\frac{3}{4}\)
So, \(\frac{6}{8}\) = \(\frac{3}{4}\)

Question 4.
\(\frac{7}{12}\) ______ \(\frac{4}{6}\)

Answer: \(\frac{7}{12}\) ≠ \(\frac{4}{6}\)

Explanation:
The fraction \(\frac{7}{12}\) is not equal to \(\frac{4}{6}\).

Lesson 6.2

Write two equivalent fractions for each.

Question 5.
\(\frac{2}{3}\)
Type below:
_________

Answer: 4/6 and 8/12

Explanation:
2/3
(2/3) x (2/2) = 4/6
(2/3) x (4/4) = 8/12
Then, the equivalent fractions of 2/3 = 4/6 and 8/12

Question 6.
\(\frac{5}{10}\)
Type below:
_________

Answer: 1/2

Explanation:
\(\frac{5}{10}\) × 2/2 = 10/20
\(\frac{5}{10}\) ÷ 5 = 1/2

Question 7.
\(\frac{4}{12}\)
Type below:
_________

Answer: 1/3

Explanation:
\(\frac{4}{12}\) ÷ 3 = 1/3
\(\frac{4}{12}\) × 3/3 = 4/12

Question 8.
\(\frac{4}{5}\)
Type below:
_________

Answer:
8/10 and 80/100

Explanation:
4/5
(4/5) x (2/2) = 8/10
(4/5) x (20/20) = 80/100
Then, the equivalent fractions of 4/5 = 8/10 and 80/100

Lesson 6.3

Write the fraction in simplest form.

Question 9.
\(\frac{6}{12}\)
\(\frac{□}{□}\)

Answer:
1/2

Explanation:
6/12 in simplest form
Divide the 6/12 with 6
(6/6)/(12/6) = 1/2
So, the simplest form of 6/12 is 1/2

Question 10.
\(\frac{2}{10}\)
\(\frac{□}{□}\)

Answer:
1/5

Explanation:
2/10 in simplest form
Divide the 2/10 with 2
(2/2)/(10/2) = 1/5
So, the simplest form of 2/10 is 1/5

Question 11.
\(\frac{4}{6}\)
\(\frac{□}{□}\)

Answer:
2/3

Explanation:
4/6 in simplest form
Divide the 4/6 with 2
(4/2)/(6/2) = 2/3
So, the simplest form of 4/6 is 2/3

Question 12.
\(\frac{3}{12}\)
\(\frac{□}{□}\)

Answer: 1/4

Explanation:
3/12 in the simplest form
Divide 3/12 with 3.
3/12 ÷ 3 = 1/4
So the simplest form of \(\frac{3}{12}\) is \(\frac{1}{4}\)

Question 13.
\(\frac{6}{10}\)
\(\frac{□}{□}\)

Answer: 3/5

Explanation:
\(\frac{6}{10}\) in the simplest form.
Divide the \(\frac{6}{10}\) with 2
\(\frac{6}{10}\) ÷ 2 = \(\frac{3}{5}\)

Lesson 6.4

Write the pair of fractions as a pair of fractions with a common denominator.

Question 14.
\(\frac{2}{3} \text { and } \frac{5}{6}\)
Type below:
_________

Answer: 8/12 and 10/12

Explanation:
The common denominator of \(\frac{2}{3} \text { and } \frac{5}{6}\)
List the multiples of 3 = 3, 6, 9, 12, 15, 18, 21,….
List the multiples of 6 = 6, 12, 18, 24, 30, 36, ….
Then, the common denominator of \(\frac{2}{3} \text { and } \frac{5}{6}\) is 12
For the Common pair of fractions, multiply the common denominator with fractions
So, common pair of fractions = 8/12 and 10/12

Question 15.
\(\frac{3}{5} \text { and } \frac{1}{2}\)
Type below:
_________

Answer: 6/10 and 5/10

Explanation:
Common denominator of \(\frac{3}{5} \text { and } \frac{1}{2}\)
List the multiples of 5 = 5, 10, 15, 20, 25, 30, …..
List the multiples of 2 = 2, 4, 6, 8, 10, 12, 14, 16, 18, 20….
Then, the common denominator of \(\frac{3}{5} \text { and } \frac{1}{2}\) is 10.
For the Common pair of fractions, multiply the common denominator with fractions
So, the common pair of fractions = 6/10 and 5/10.

Question 16.
\(\frac{1}{4} \text { and } \frac{5}{12}\)
Type below:
_________

Answer: 3/12 and 5/12

Explanation:
The common denominator of \(\frac{1}{4} \text { and } \frac{5}{12}\)
List the multiples of 4 = 4, 8, 12, 16, 20, 24, . . .
List the multiples of 12 = 12, 24, 36, 48…
Then, the common denominator of \(\frac{1}{4} \text { and } \frac{5}{12}\) is 12
For the Common pair of fractions, multiply the common denominator with fractions
So, common pair of fractions = 3/12 and 5/12.

Question 17.
\(\frac{7}{8} \text { and } \frac{3}{4}\)
Type below:
_________

Answer: 7/8 and 6/8

Explanation:
Common denominator of \(\frac{7}{8} \text { and } \frac{3}{4}\)
List the multiples of 8 = 8, 16, 24, 32, . . . .
List the multiples of 4 = 4, 8, 12, 16,….
Then, the common denominator of \(\frac{7}{8} \text { and } \frac{3}{4}\) is 8
For the Common pair of fractions, multiply the common denominator with fractions
So, the common pair of fractions = 7/8 and 6/8

Question 18.
\(\frac{3}{10} \text { and } \frac{1}{5}\)
Type below:
_________

Answer: \(\frac{3}{10} \text { and } \frac{2}{10}\)

Explanation:
Common denominator of \(\frac{3}{10} \text { and } \frac{1}{5}\)
List the multiples of 5 = 5, 10, 15, 20, 25, 30, …..
List the multiples of 10 = 10, 20, 30, 40, 50 ….
Then, the common denominator of \(\frac{3}{10} \text { and } \frac{1}{5}\) is 10
For the Common pair of fractions, multiply the common denominator with fractions
So, the common pair of fractions = \(\frac{3}{10} \text { and } \frac{2}{10}\)

Question 19.
\(\frac{3}{4} \text { and } \frac{1}{3}\)
Type below:
_________

Answer: 9/12 and 4/12

Explanation:
The common denominator of \(\frac{3}{4} \text { and } \frac{1}{3}\)
List the multiples of 3 = 3, 6, 9, 12, 15, 18, 21, 24, ….
List the multiples of 4 = 4, 8, 12, 16, 20, 24, . . .
Then, the common denominator of \(\frac{3}{4} \text { and } \frac{1}{3}\) is 12
For the Common pair of fractions, multiply the common denominator with fractions
So, the common pair of fractions = 9/12 and 4/12

Common Core – Fraction Equivalence and Comparison – Page No. 130

Lesson 6.5

Question 1.
Mr. Renner is decorating a bulletin board with groups of shapes. Each group has 3 shapes, and \(\frac{2}{3}\) of the shapes are snowflakes. If Mr. Renner is using 4 groups of shapes, how many snowflakes will he need? Complete the table to find the fraction of the shapes for each number of group that are
Go Math Grade 4 Answer Key Homework Practice FL Chapter 6 Fraction Equivalence and Comparison Common Core - Fraction Equivalence and Comparison img 12
How many snowflake shapes will Mr. Renner use?
_______ snowflakes shapes

Answer: 8 snowflakes shapes

Explanation:
Given,
Mr. Renner is decorating a bulletin board with groups of shapes.
Each group has 3 shapes, and \(\frac{2}{3}\) of the shapes are snowflakes.
There are 4 groups and in every group, there are 2 snowflakes so in total there are 8. because 4 × 2=8
Mr. Renner uses 8 snowflakes shapes.

Question 2.
Nell made a pizza. She cut the pizza into fourths. Then she cut each fourth into four pieces. Nell and her friends ate 6 of the smaller pieces of the pizza.
What fraction of the pizza did Nell and her friends eat?
What fraction of the pizza did Nell and her friends NOT eat?

Answer:
okay so four is slice, and then do it again. the answer is at the bottom,

pizza: 16 pieces
Nell and her friends ate 6 smaller pizza
3/8 is the answer.

Lesson 6.6 – 6.7

Compare. Write <,>, or =.

Question 3.
\(\frac{2}{6}\) ______ \(\frac{3}{4}\)

Answer: \(\frac{2}{6}\) < \(\frac{3}{4}\)

Explanation:
\(\frac{2}{6}\) = \(\frac{1}{3}\)
\(\frac{1}{3}\) is less than \(\frac{3}{4}\)
So, \(\frac{2}{6}\) < \(\frac{3}{4}\)

Question 4.
\(\frac{6}{8}\) ______ \(\frac{1}{4}\)

Answer: \(\frac{6}{8}\) > \(\frac{1}{4}\)

Explanation:
\(\frac{6}{8}\) = \(\frac{3}{4}\)
\(\frac{3}{4}\) is greater than \(\frac{1}{4}\)
So, \(\frac{6}{8}\) > \(\frac{1}{4}\)

Question 5.
\(\frac{5}{6}\) ______ \(\frac{2}{4}\)

Answer: \(\frac{5}{6}\) > \(\frac{2}{4}\)

Explanation:
\(\frac{2}{4}\) = \(\frac{1}{2}\)
\(\frac{5}{6}\) is greater than \(\frac{1}{2}\)
So, \(\frac{5}{6}\) > \(\frac{2}{4}\)

Question 6.
\(\frac{1}{3}\) ______ \(\frac{4}{12}\)

Answer: \(\frac{1}{3}\) = \(\frac{4}{12}\)

Explanation:
\(\frac{4}{12}\) = \(\frac{1}{3}\)
\(\frac{1}{3}\) = \(\frac{4}{12}\)

Question 7.
\(\frac{1}{6}\) ______ \(\frac{1}{8}\)

Answer: \(\frac{1}{6}\) > \(\frac{1}{8}\)

Explanation:
Given the fractions \(\frac{1}{6}\) and \(\frac{1}{8}\)
Here the numerators are same so we have to compare the denominators.
The denominator with the smallest number will be the greatest fraction.
\(\frac{1}{6}\) is greater than \(\frac{1}{8}\)
So, \(\frac{1}{6}\) > \(\frac{1}{8}\)

Question 8.
\(\frac{2}{3}\) ______ \(\frac{4}{6}\)

Answer: \(\frac{2}{3}\) = \(\frac{4}{6}\)

Explanation:
\(\frac{4}{6}\) ÷ 2 = \(\frac{2}{3}\)
\(\frac{2}{3}\) = \(\frac{2}{3}\)
So, \(\frac{2}{3}\) = \(\frac{4}{6}\)

Question 9.
\(\frac{3}{10}\) ______ \(\frac{3}{12}\)

Answer: \(\frac{3}{10}\) > \(\frac{3}{12}\)

Explanation:
Given the fractions \(\frac{3}{10}\) and \(\frac{3}{12}\)
Here the numerators are same so we have to compare the denominators.
The denominator with the smallest number will be the greatest fraction.
Thus \(\frac{3}{10}\) > \(\frac{3}{12}\)

Question 10.
\(\frac{7}{8}\) ______ \(\frac{4}{4}\)

Answer: \(\frac{7}{8}\) < \(\frac{4}{4}\)

Explanation:
\(\frac{4}{4}\) = 1
1 is greater than \(\frac{7}{8}\)
Thus \(\frac{7}{8}\) < \(\frac{4}{4}\)

Lesson 6.8

Write the fractions in order from least to greatest.

Question 12.
\(\frac{1}{2}, \frac{1}{4}, \frac{5}{8}\)
Type below:
__________

Answer: 1/4, 5/8 and 1/2

Explanation:
1/4 × 2/2 = 2/8
5/8 × 1/1 = 5/8
1/2 × 4/4 = 4/8
Compare the numerators of the above fractions.
The numerator with the largest number will be the greatest fraction.
The fraction from least to greatest is 1/4, 5/8 and 1/2

Question 13.
\(\frac{2}{3}, \frac{1}{6}, \frac{9}{10}\)
Type below:
__________

Answer: 1/6, 2/3 and 9/10

Explanation:
Given the fractions \(\frac{2}{3}, \frac{1}{6}, \frac{9}{10}\)
2/3 × 10/10 = 20/30
1/6 × 5/5 = 5/30
9/10 × 3/3 = 27/30
Compare the numerators of the above fractions.
The numerator with the largest number will be the greatest fraction.
The fraction from least to greatest is 1/6, 2/3 and 9/10.

Question 14.
\(\frac{3}{5}, \frac{3}{4}, \frac{3}{8}\)
Type below:
__________

Answer: \(\frac{3}{8}, \frac{3}{5}, \frac{3}{4}\)

Explanation:
Given the fractions \(\frac{3}{5}, \frac{3}{4}, \frac{3}{8}\)
Here the numerators are the same so we have to compare the denominators.
The denominator with the smallest number will be the greatest fraction.
Thus the fractions from least to greatest are \(\frac{3}{8}, \frac{3}{5}, \frac{3}{4}\).

Conclusion:

I think the explained answers provided in this chapter are important for all 4th-grade students. Make sure to share these pdf links with our friends and assist them while practicing mathematical concepts. In case, do you feel anything is missing in Homework Practice FL, you can refer to the Go Math Grade 4 Answer Key Chapter 6 Fraction Equivalence and Comparison. Try solving these chapter problems and enhance your subject knowledge.

Go Math Grade 3 Answer Key Chapter 9 Compare Fractions

go-math-grade-3-chapter-9-compare-fractions-answer-key

In Order to attempt the 3rd Grade Exam, you need to have strong fundamentals. Download Go Math Grade 3 Answer Key Chapter 9 Compare Fractions free of cost and get the homework help you need. Attempt the Mid Chapter Checkpoint Test to know your preparation level within the Chapter. Practice as much as you can using the 3rd Grade Go Math Solution Key and clear the exam with better grades.

3rd Grade Go Math Answer Key Chapter 9 Compare Fractions

You will have topics like Comparing Fractions with the Same Numerator and Denominators, Equivalent Fractions, Compare and order fractions, and so on. You will not feel the Concept of Comparing Fractions difficult anymore with our HMH Go Math Grade 3 Answer Key Chapter 9 Compare Fractions. Make the most out of the 3rd Grade Go Math Solution Key Chapter 9 and solve various questions in it and master the subject.

Lesson 1: Compare Fractions 

Lesson 2: Problem Solving • Compare Fractions 

Lesson 3: Compare Fractions with the Same Denominator 

Lesson 4: Compare Fractions with the Same Numerator 

Lesson 5: Compare Fractions 

Mid -Chapter Checkpoint

Lesson 6: Compare and Order Fractions

Lesson 7: Model Equivalent Fractions 

Lesson 9.7 – Page No. 548

Lesson 9: Equivalent Fractions 

Review/ Test

Compare Fractions – Page No. 509

Share and Show

Question 1.
At the park, people can climb a rope ladder to its top. Rosa climbed \(\frac{2}{8}\) of the way up the ladder. Justin climbed \(\frac{2}{6}\) of the way up the ladder. Who climbed higher on the rope ladder?
First, what are you asked to find?
Type below:
____________

Answer: Justin climbed higher on the rope ladder.

Explanation:

Given,
Rosa climbed \(\frac{2}{8}\) of the way up the ladder
Justin climbed \(\frac{2}{6}\) of the way up the ladder
We are asked to find who climbed higher on the rope ladder
By comparing the denominators we can say that Justin Climbed higher than Rosa on the rope ladder.

Question 2.
Then, model and compare the fractions.
Type below:
____________

Answer:

Go Math Grade 3 Chapter 9 Answer Key Compare fractions solution img_1

Question 3.
Last, find the greater fraction.
\(\frac{2}{6}\) _____ \(\frac{2}{8}\)

Answer: \(\frac{2}{6}\) > \(\frac{2}{8}\)

Explanation:

When comparing fractions such as \(\frac{2}{8}\) and \(\frac{2}{6}\), you could also convert the fractions (if necessary) so they have the same denominator and then compare which numerator is larger.

Question 4.
___________ climbed higher on the rope ladder

Answer: Justin climbed higher on the rope ladder.

Question 5.
What if Cara also tried the rope ladder and climbed \(\frac{2}{4}\) of the way up? Who climbed highest on the rope ladder: Rosa, Justin, or Cara? Explain how you know.
___________

Answer: If Cara also tried the rope ladder and climbed \(\frac{2}{4}\) of the way up then Cara would be climbed highest on the rope ladder.
Because comparing fractions \(\frac{2}{4}\), \(\frac{2}{6}\), \(\frac{2}{8}\) Cara climbed high among the three. The fraction \(\frac{2}{4}\) is the greater than other 2 fractions. So by seeing this we can say that Cara climbed hhighest on the rope ladder.

Compare Fractions – Page No. 510

Use the table for 4–5.
Go Math Grade 3 Answer Key Chapter 9 Compare Fractions Compare Fractions img 1

Question 1.
Suri is spreading jam on 8 biscuits for breakfast. The table shows the fraction of biscuits spread with each jam flavor. Which flavor did Suri use on the most biscuits?
___________

Answer: Raspberry

Explanation:

The above table shows the fraction of jam frosted on the biscuits.
First, check the denominators to compare the fractions.
The denominators are the same. So, Compare with the numerators. The numerator of Raspberry is larger than other two flavors. So, Suri used Raspberry flavor on the most biscuits.

Question 2.
What’s the Question? The answer is strawberry
Type below:
___________

Answer:

Suri is spreading jam on 8 biscuits for breakfast. The table shows the fraction of biscuits spread with each jam flavor. She frosted \(\frac{3}{8}\) of the biscuits with peach jam, \(\frac{4}{8}\) with raspberry jam, and \(\frac{1}{8}\) with strawberry jam. Which flavor of jam did Suri use least on the biscuits?

Question 3.
Suppose Suri had also used plum jam on the biscuits. She frosted \(\frac{1}{2}\) of the biscuits with peach jam, \(\frac{1}{4}\) with raspberry jam, \(\frac{1}{8}\) with strawberry jam, and \(\frac{1}{8}\) with plum jam. Which flavor of jam did Suri use on the most biscuits?
___________

Answer: Peach

Explanation:

The fraction of peach jam is greater than raspberry jam, strawberry jam, and plum jam.
So, the answer is the peach jam.

Question 4.
Ms. Gordon has many snack bar recipes. One recipe uses \(\frac{1}{3}\) cup oatmeal, \(\frac{1}{4}\) vcup of milk, and \(\frac{1}{2}\) cup flour. Which ingredient will Ms. Gordon use the most of?
___________

Answer: flour

\(\frac{1}{2}\) > \(\frac{1}{3}\) and \(\frac{1}{4}\)
So, by comparing fractions we can say that Ms. Gordon use the most of flour for snack bar recipes.

Question 5.
Rick lives \(\frac{4}{6}\) mile from school. Noah lives \(\frac{3}{6}\) mile from school. Use the fractions and symbols to show which distance is longer.
Go Math Grade 3 Answer Key Chapter 9 Compare Fractions Compare Fractions img 2
\(\frac{4}{6}\) _____ \(\frac{3}{6}\)

Answer: \(\frac{4}{6}\) > \(\frac{3}{6}\)

Explanation:

First of all, compare the denominators. If the denominators are the same then check the numerators.
Here 4 is greater than 3. So, \(\frac{4}{6}\) > \(\frac{3}{6}\)

Problem Solving Compare Fractions – Page No. 511

Solve.

Question 1.
Luis skates \(\frac{2}{3}\) mile from his home to school. Isabella skates \(\frac{2}{4}\) mile to get to school. Who skates farther?
Think: Use fraction strips to act it out.
Luis

Answer: Luis

Explanation:

Given,
Luis skates \(\frac{2}{3}\) mile from his home to school.
Isabella skates \(\frac{2}{4}\) mile to get to school.
To find Who stakes farther we have to compare the fractions.
\(\frac{2}{3}\), \(\frac{2}{4}\)
The numerator of both the fractions is the same and the denominators are different.
So, first, make the denominators equal.
\(\frac{2}{3}\) × \(\frac{4}{4}\) = \(\frac{8}{12}\)
\(\frac{2}{4}\) × \(\frac{3}{3}\) = \(\frac{6}{12}\)
Now denominators are same.
Compare fractions \(\frac{8}{12}\) and \(\frac{6}{12}\)
8 is greater than 6. So, \(\frac{8}{12}\) > \(\frac{6}{12}\)
Therefore Luis Skates farther to school.

Question 2.
Sandra makes a pizza. She puts mushrooms on \(\frac{2}{8}\) of the pizza. She adds green peppers to \(\frac{5}{8}\) of the pizza. Which topping covers more of the pizza?
___________

Answer: Green Peppers

Explanation:

Sandra makes a pizza. She puts mushrooms on \(\frac{2}{8}\) of the pizza.
She adds green peppers to \(\frac{5}{8}\) of the pizza.
Compare the fractions of mushrooms and green peppers.
\(\frac{2}{8}\), \(\frac{5}{8}\)
The denominators are the same. So compare the numerators.
2 is lesser than 5. Thus \(\frac{2}{8}\) < \(\frac{5}{8}\)
Thus Green Peppers covers more of the pizza.

Question 3.
The jars of paint in the art room have different amounts of paint. The green paint jar is \(\frac{4}{8}\) full. The purple paint jar is \(\frac{4}{6}\) full. Which paint jar is less full?
The _______ paint jar

Answer: The green paint jar

Explanation:

The jars of paint in the art room have different amounts of paint.
The green paint jar is \(\frac{4}{8}\) full.
The purple paint jar is \(\frac{4}{6}\) full.
The numerators of both the fractions are the same.
Compare the denominators of green paint and purple paint jars.
The denominator with the greatest number will be the smallest fraction.
Therefore \(\frac{4}{8}\) < \(\frac{4}{6}\)
Thus green paint jar is less full.

Question 4.
Jan has a recipe for bread. She uses \(\frac{2}{3}\) cup of flour and \(\frac{1}{3}\) cup of chopped onion. Which ingredient does she use more of, flour or onion?
_______

Answer: Flour

Explanation:

Jan has a recipe for bread. She uses \(\frac{2}{3}\) cup of flour and \(\frac{1}{3}\) cup of chopped onion.
Compare the fraction of flour and onion.
The denominators of both the fractions are the same. So, compare the numerators.
2 is greater than 1. Thus \(\frac{2}{3}\) > \(\frac{1}{3}\)
That means Jan used more flour for bread.

Question 5.
Edward walked \(\frac{3}{4}\) mile from his home to the park. Then he walked \(\frac{2}{4}\) mile from the park to the library. Which distance is shorter?
_______

Answer: \(\frac{2}{4}\) mile

Explanation:

Edward walked \(\frac{3}{4}\) mile from his home to the park.
Then he walked \(\frac{2}{4}\) mile from the park to the library.
To find the shorter distance we have to compare the fractions of Edward from home to park and from park to library.
\(\frac{3}{4}\), \(\frac{2}{4}\)
The denominators of both the fractions are the same.
So compare the numerators.
3 is greater than 2. Thus \(\frac{3}{4}\) > \(\frac{2}{4}\)
Thus the distance from the park to the library is shorter.

Problem Solving Compare Fractions – Page No. 512

Lesson Check

Question 1.
Ali and Jonah collect seashells in identical buckets. When they are finished, Ali’s bucket is \(\frac{2}{6}\) full and Jonah’s bucket is \(\frac{3}{6}\) full. Which of the following correctly compares the fractions?
Options:
a. \(\frac{2}{6}\) = \(\frac{3}{6}\)
b. \(\frac{2}{6}\) > \(\frac{3}{6}\)
c. \(\frac{3}{6}\) < \(\frac{2}{6}\)
d. \(\frac{3}{6}\) > \(\frac{2}{6}\)

Answer: \(\frac{3}{6}\) > \(\frac{2}{6}\)

Explanation:

Given that, Ali and Jonah collect seashells in identical buckets.
When they are finished, Ali’s bucket is \(\frac{2}{6}\) full and Jonah’s bucket is \(\frac{3}{6}\) full
Compare fractions \(\frac{2}{6}\) and \(\frac{3}{6}\)
We observe that the denominators are the same. So, compare the numerators of both the fractions.
3 is greater than 2.
Thus \(\frac{3}{6}\) > \(\frac{2}{6}\)
So, the correct answer is option D.

Question 2.
Rosa paints a wall in her bedroom. She puts green paint on \(\frac{5}{8}\) of the wall and blue paint on \(\frac{3}{8}\) of the wall. Which of the following correctly compares the fractions?
Options:
a. \(\frac{5}{8}\) > \(\frac{3}{8}\)
b. \(\frac{5}{8}\) < \(\frac{3}{8}\)
c. \(\frac{3}{8}\) > \(\frac{5}{8}\)
d. \(\frac{3}{8}\) = \(\frac{5}{8}\)

Answer: \(\frac{5}{8}\) > \(\frac{3}{8}\)

Explanation:

Given:
Rosa paints a wall in her bedroom.
She puts green paint on \(\frac{5}{8}\) of the wall and blue paint on \(\frac{3}{8}\) of the wall.
The denominators are same so compare the numerators of both the fractions.
5 is greater than 3. So, \(\frac{5}{8}\) > \(\frac{3}{8}\)
Thus the correct answer is option A.

Spiral Review

Question 3.
Dan divides a pie into eighths. How many equal parts are there?
Options:
a. 3
b. 6
c. 8
d. 10

Answer: 8

Dan divides a pie into eighths.
Eighths are nothing but the name of the parts. Eighths is equal to 8.
So, the correct answer is option C.

Question 4.
Which shows equal parts?
a. Go Math Grade 3 Answer Key Chapter 9 Compare Fractions Problem Solving Compare Fractions img 3
b. Go Math Grade 3 Answer Key Chapter 9 Compare Fractions Problem Solving Compare Fractions img 4
c. Go Math Grade 3 Answer Key Chapter 9 Compare Fractions Problem Solving Compare Fractions img 5
d. Go Math Grade 3 Answer Key Chapter 9 Compare Fractions Problem Solving Compare Fractions img 6

Answer:

Go Math Grade 3 Answer Key Chapter 9 Compare Fractions Problem Solving Compare Fractions img 4

Explanation:

Among all the figures circle is equally divided into 6 parts.
So, the answer is option B.

Question 5.
Charles places 30 pictures on his bulletin board in 6 equal rows. How many pictures are in each row?
Options:
a. 3
b. 4
c. 5
d. 6

Answer: 5

Explanation:

Given, Charles places 30 pictures on his bulletin board in 6 equal rows.
Number of pictures in each row = x
x × 6 = 30
x = 30/6 = 5
Therefore there are 5 pictures in each row.

Question 6.
Which of the following describes a pattern in the table?
Go Math Grade 3 Answer Key Chapter 9 Compare Fractions Problem Solving Compare Fractions img 7
Options:
a. Add 1.
b. Add 4.
c. Multiply by 2.
d. Multiply by 5.

Answer: Multiply by 5

Explanation:

The above table shows that number of tables is multiplied by 5.
So, the correct answer is option D.

Compare Fractions with the Same Denominator – Page No. 517

Compare. Write <, >, or =.

Question 1.
Go Math Grade 3 Answer Key Chapter 9 Compare Fractions Compare Fractions with the Same Denominator img 8

Answer: \(\frac{3}{4}\) > \(\frac{1}{4}\)

Explanation:

Check whether the denominators of the two fractions are the same.
Here the denominators of \(\frac{3}{4}\) and \(\frac{1}{4}\) are same. So compare the numerators.
3 is greater than 1.
Therefore, \(\frac{3}{4}\) > \(\frac{1}{4}\)

Question 2.
\(\frac{3}{6}\) ______ \(\frac{0}{6}\)

Answer: \(\frac{3}{6}\) > \(\frac{0}{6}\)

Explanation:

First, check whether the denominators of the two fractions are the same or not.
After that compare the numerators.
3 > 0
So, \(\frac{3}{6}\) > \(\frac{0}{6}\)

Question 3.
\(\frac{1}{2}\) ______ \(\frac{1}{2}\)

Answer: \(\frac{1}{2}\) = \(\frac{1}{2}\)

Explanation:

First, compare fractions with the same denominators.
If both are the same, then compare the numerators of both fractions.
The denominators and numerators are same for \(\frac{1}{2}\)
Thus \(\frac{1}{2}\) = \(\frac{1}{2}\)

Question 4.
\(\frac{5}{6}\) ______ \(\frac{6}{6}\)

Answer: \(\frac{5}{6}\) < \(\frac{6}{6}\)

Explanation:

Compare the denominators of the fractions
The denominators of \(\frac{5}{6}\) and \(\frac{6}{6}\)
Now compare the numerators of the fractions.
5 < 6
So, \(\frac{5}{6}\) < \(\frac{6}{6}\)

Question 5.
\(\frac{7}{8}\) ______ \(\frac{5}{8}\)

Answer: \(\frac{7}{8}\) > \(\frac{5}{8}\)

Explanation:

Check whether the denominators are the same or not.
The denominator of \(\frac{7}{8}\) and \(\frac{5}{8}\) are same.
Now check the numerators 7 > 5.
Thus \(\frac{7}{8}\) > \(\frac{5}{8}\)

Question 6.
\(\frac{2}{3}\) ______ \(\frac{2}{3}\)

Answer: \(\frac{2}{3}\) = \(\frac{2}{3}\)

Explanation:

Compare the denominators of 2 fractions.
Here numerators and denominators are the same.
So, \(\frac{2}{3}\) = \(\frac{2}{3}\)

Question 7.
\(\frac{8}{8}\) ______ \(\frac{0}{8}\)

Answer: \(\frac{8}{8}\) > \(\frac{0}{8}\)

Explanation:

Check whether the denominators are the same. Now compare the numerators of two fractions.
8 > 0.
So, \(\frac{8}{8}\) > \(\frac{0}{8}\)

Question 8.
\(\frac{1}{6}\) ______ \(\frac{1}{6}\)

Answer: \(\frac{1}{6}\) = \(\frac{1}{6}\)

Explanation:

When the denominators are the same, the whole is divided into the same size pieces. Now compare the numerators of both the fractions.
\(\frac{1}{6}\) = \(\frac{1}{6}\)

Question 9.
\(\frac{3}{4}\) ______ \(\frac{2}{4}\)

Answer: \(\frac{3}{4}\) > \(\frac{2}{4}\)

Explanation:

Check whether the denominators are the same. If same that means the whole is divided into the same size pieces.
The denominators of \(\frac{3}{4}\) and \(\frac{2}{4}\)
Now compare the numerators of both fractions.
3 is greater than 2.
Thus \(\frac{3}{4}\) > \(\frac{2}{4}\)

Question 10.
\(\frac{1}{6}\) ______ \(\frac{2}{6}\)

Answer: \(\frac{1}{6}\) < \(\frac{2}{6}\)

Explanation:

Check whether the denominators are the same. If same that means the whole is divided into the same size pieces.
The denominators of \(\frac{1}{6}\) and \(\frac{2}{6}\)
Now check the numerators 1 is less than 2.
Therefore, \(\frac{1}{6}\) < \(\frac{2}{6}\)

Question 11.
\(\frac{1}{2}\) ______ \(\frac{0}{2}\)

Answer: \(\frac{1}{2}\) > \(\frac{0}{2}\)

Explanation:

When the denominators are same, the whole is divided into the same pieces. Now check the numerators.
1 is greater than 0
Thus \(\frac{1}{2}\) > \(\frac{0}{2}\)

Question 12.
\(\frac{3}{8}\) ______ \(\frac{3}{8}\)

Answer: \(\frac{3}{8}\) = \(\frac{3}{8}\)

Explanation:

Check the denominators of two fractions. If both are equal then compare the numerators.
The numerators of both fractions are equal.
So, \(\frac{3}{8}\) = \(\frac{3}{8}\)

Question 13.
\(\frac{1}{4}\) ______ \(\frac{4}{4}\)

Answer: \(\frac{1}{4}\) < \(\frac{4}{4}\)

Explanation:

Compare fractions with the same denominators.
The denominators of \(\frac{1}{4}\) and \(\frac{4}{4}\) are same.
Compare numerators 1 and 4.
1 is less than 4.
Thus \(\frac{1}{4}\) < \(\frac{4}{4}\)

Question 14.
\(\frac{5}{8}\) ______ \(\frac{4}{8}\)

Answer: \(\frac{5}{8}\) > \(\frac{4}{8}\)

Explanation:

Check whether the denominators are the same or not.
Now compare the numerators.
5 is greater than 4.
\(\frac{5}{8}\) > \(\frac{4}{8}\)

Question 15.
\(\frac{4}{6}\) ______ \(\frac{6}{6}\)

Answer: \(\frac{4}{6}\) < \(\frac{6}{6}\)

Explanation:

Check whether the denominators of both the fractions are the same or not.
The denominators of \(\frac{4}{6}\) and \(\frac{6}{6}\) are the same.
Now compare the numerators 4 and 6.
4 is lesser than 6. So, \(\frac{4}{6}\) < \(\frac{6}{6}\)

Problem Solving

Question 16.
Ben mowed \(\frac{5}{6}\) of his lawn in one hour. John mowed \(\frac{4}{6}\) of his lawn in one hour. Who mowed less of his lawn in one hour?
___________

Answer: John

Explanation:

Given,
Ben mowed \(\frac{5}{6}\) of his lawn in one hour.
John mowed \(\frac{4}{6}\) of his lawn in one hour.
Compare the denominators of both the fractions.
As the denominators are the same compare the numerators of the fractions.
4 is less than 5.
\(\frac{4}{6}\) < \(\frac{5}{6}\)
So, John mowed less of his lawn in one hour.

Question 17.
Darcy baked 8 muffins. She put blueberries in \(\frac{5}{8}\) of the muffins. She put raspberries in \(\frac{3}{8}\) of the muffins. Did more muffins have blueberries or raspberries?
___________

Answer: Blueberries

Explanation:

Given,
Darcy baked 8 muffins.
She put blueberries in \(\frac{5}{8}\) of the muffins.
She put raspberries in \(\frac{3}{8}\) of the muffins.
\(\frac{5}{8}\) is greater than \(\frac{3}{8}\)
Thus there are more blueberries in muffins.

Compare Fractions with the Same Denominator – Page No. 518

Lesson Check

Question 1.
Julia paints \(\frac{2}{6}\) of a wall in her room white. She paints more of the wall green. Which fraction could show the part of the wall that is green?
Options:
a. \(\frac{1}{6}\)
b. \(\frac{2}{6}\)
c. \(\frac{3}{6}\)
d. \(\frac{0}{6}\)

Answer: \(\frac{3}{6}\)

Explanation:

First compare the denominators of given options and then compare the numerators
A. \(\frac{3}{6}\) < \(\frac{2}{6}\)
B. \(\frac{2}{6}\) = \(\frac{2}{6}\)
C. \(\frac{3}{6}\) > \(\frac{2}{6}\)
D. \(\frac{0}{6}\) < \(\frac{2}{6}\)
So, the answer is option C.

Question 2.
Liam is comparing fraction circles. Which of the following statements is true?
Options:
a. \(\frac{1}{2}\) = \(\frac{1}{2}\)
b. \(\frac{3}{4}\) > \(\frac{4}{4}\)
c. \(\frac{4}{6}\) < \(\frac{3}{6}\)
d. \(\frac{2}{8}\) = \(\frac{3}{8}\)

Answer: \(\frac{1}{2}\) = \(\frac{1}{2}\)

Explanation:

Check whether the denominators are same. If both are same then compare the numerators.
\(\frac{1}{2}\) = \(\frac{1}{2}\)
Option A is the correct answer.

Spiral Review

Question 3.
Mr. Edwards buys 2 new knobs for each of his kitchen cabinets. The kitchen has 9 cabinets. How many knobs does he buy?
Options:
a. 20
b. 18
c. 16
d. 12

Answer: 18

Explanation:

Given, Mr. Edwards buys 2 new knobs for each of his kitchen cabinets.
The kitchen has 9 cabinets.
Number of knobs he buys = x
x = 9 × 2 = 18
Thus the correct answer is option B.

Question 4.
Allie builds a new bookcase with 8 shelves. She can put 30 books on each shelf. How many books can the bookcase hold?
Options:
a. 30
b. 38
c. 240
d. 300

Answer: 240

Explanation:

Given that Allie builds a new bookcase with 8 shelves.
She can put 30 books on each shelf.
Let the number of books can the bookcase hold = y
y = 30 × 8
y = 240
Thus the correct answer is option C.

Question 5.
The Good Morning Café has 28 customers for breakfast. There are 4 people sitting at each table. How many tables are filled?
Options:
a. 8
b. 7
c. 6
d. 4

Answer: 7

Explanation:

The Good Morning Café has 28 customers for breakfast.
There are 4 people sitting at each table.
Number of tables be t
t ×4 = 28
t = 28/4 = 4
Thus the number of tables filled = 7

Question 6.
Ella wants to use the Commutative Property of Multiplication to help find the product 5 × 4. Which number sentence can she use?
Options:
a. 5 + 4 = 10
b. 5 × 5 = 25
c. 5 − 4 = 1
d. 4 × 5 = 20

Answer: 4 × 5 = 20

Explanation:

According to the commutative property of multiplication, changing the order of the numbers we are multiplying, does not change the product.
a × b = b × a
5 × 4 = 4 × 5 = 20
So, the correct answer is option D.

Compare Fractions with the Same Numerator – Page No. 523

Compare. Write <, >, or =.

Question 1.
Go Math Grade 3 Answer Key Chapter 9 Compare Fractions Compare Fractions with the Same Numerator img 9

Question 2.
\(\frac{3}{8}\) ______ \(\frac{3}{6}\)

Answer: \(\frac{3}{8}\) < \(\frac{3}{6}\)

Explanation:

When comparing fractions with the same numerator, the fraction with the smaller denominator is greater.
So, \(\frac{3}{8}\) < \(\frac{3}{6}\)

Question 3.
\(\frac{2}{3}\) ______ \(\frac{2}{4}\)

Answer: \(\frac{2}{3}\) > \(\frac{2}{4}\)

Explanation:

Compare the fractions \(\frac{2}{3}\) and \(\frac{2}{4}\)
The numerators of both the fractions are the same. So compare the denominators.
The fraction with the smaller denominator is greater
So, \(\frac{2}{3}\) > \(\frac{2}{4}\)

Question 4.
\(\frac{2}{8}\) ______ \(\frac{2}{3}\)

Answer: \(\frac{2}{8}\) < \(\frac{2}{3}\)

Explanation:

Compare \(\frac{2}{8}\) and \(\frac{2}{3}\)
When comparing fractions with the same numerator, the fraction with the smaller denominator is greater.
\(\frac{2}{8}\) < \(\frac{2}{3}\)

Question 5.
\(\frac{3}{6}\) ______ \(\frac{3}{4}\)

Answer: \(\frac{3}{6}\) < \(\frac{3}{4}\)

Explanation:

Compare the fractions \(\frac{3}{6}\) and \(\frac{3}{4}\)
The numerators are the same and the denominators are different.
The number with the smallest number will be the greatest.
So, \(\frac{3}{6}\) < \(\frac{3}{4}\)

Question 6.
\(\frac{1}{2}\) ______ \(\frac{1}{6}\)

Answer: \(\frac{1}{2}\) > \(\frac{1}{6}\)

Explanation:

When comparing fractions with the same numerator, the fraction with the smaller denominator is greater.
2 is greater than 6.
\(\frac{1}{2}\) > \(\frac{1}{6}\)

Question 7.
\(\frac{5}{6}\) ______ \(\frac{5}{8}\)

Answer: \(\frac{5}{6}\) > \(\frac{5}{8}\)

Explanation:

We observe that numerators are the same and the denominators are different.
The fraction with the smallest number will be the greatest.
So, \(\frac{5}{6}\) > \(\frac{5}{8}\)

Question 8.
\(\frac{4}{8}\) ______ \(\frac{4}{8}\)

Answer: \(\frac{4}{8}\) = \(\frac{4}{8}\)

Explanation:

The numerators and denominators of both the fractions are the same.
So, \(\frac{4}{8}\) = \(\frac{4}{8}\)

Question 9.
\(\frac{6}{8}\) ______ \(\frac{6}{6}\)

Answer: \(\frac{6}{8}\) < \(\frac{6}{6}\)

Explanation:

Compare the fractions \(\frac{6}{8}\) and \(\frac{6}{6}\)
We observe that numerators are the same and the denominators are different.
So, \(\frac{6}{8}\) < \(\frac{6}{6}\)

Problem Solving

Question 10.
Javier is buying food in the lunch line. The tray of salad plates is \(\frac{3}{8}\) full. The tray of fruit plates is \(\frac{3}{4}\) full. Which tray is more full?
The tray of ______ plates

Answer: The fruit plate tray

Explanation:

Javier is buying food in the lunch line. The tray of salad plates is \(\frac{3}{8}\) full.
The tray of fruit plates is \(\frac{3}{4}\) full.
Compare the fraction of salad plates and fruit plates.
\(\frac{3}{8}\) and \(\frac{3}{4}\)
The numerators are same. So compare the denominators.
So, \(\frac{3}{8}\) <\(\frac{3}{4}\)
Thus the fruit plate tray is more full than salad plate tray.

Question 11.
Rachel bought some buttons. Of the buttons, \(\frac{2}{4}\) are yellow and \(\frac{2}{8}\) are red. Rachel bought more of which color buttons?
More _______ buttons

Answer: Yellow

Explanation:

Rachel bought some buttons. Of the buttons, \(\frac{2}{4}\) are yellow and \(\frac{2}{8}\) are red.
Compare \(\frac{2}{4}\) and \(\frac{2}{8}\)
The fraction with the smaller denominator is greater.
\(\frac{2}{4}\) >\(\frac{2}{8}\)
Therefore there are more yellow buttons.

Compare Fractions with the Same Numerator – Page No. 524

Lesson Check

Question 1.
Which symbol makes the statement true?
\(\frac{3}{4}\) O \(\frac{3}{8}\)
a. >
b. <
c. =
d. none

Answer: >

Explanation:

In the above statement, the fractions are of the same numerators. So, we need to check the denominators.
The number with the highest number will be the least fraction.
So, \(\frac{3}{4}\) > \(\frac{3}{8}\)
Thus the correct answer is option A.

Question 2.
Which symbol makes the statement true?
\(\frac{2}{4}\) O \(\frac{2}{3}\)
a. >
b. <
c. =
d. none

Answer: <

Explanation:

The fractions are of the same numerators. So, we need to see the denominators.
The number with the highest number will be the least fraction.
So, \(\frac{2}{4}\) < \(\frac{2}{3}\)
So, the correct answer is option B.

Spiral Review

Question 3.
Anita divided a circle into 6 equal parts and shaded 1 of the parts. Which fraction names the part she shaded?
Options:
a. \(\frac{1}{6}\)
b. \(\frac{1}{5}\)
c. \(\frac{5}{6}\)
d. \(\frac{1}{1}\)

Answer: \(\frac{1}{6}\)

Explanation:

Given that, Anita divided a circle into 6 equal parts and shaded 1 of the parts.
So, the fraction of the shaded part is \(\frac{1}{6}\)
Thus the correct answer is option A.

Question 4.
Which fraction names the shaded part of the rectangle?
Go Math Grade 3 Answer Key Chapter 9 Compare Fractions Compare Fractions with the Same Numerator img 10
Options:
a. \(\frac{1}{8}\)
b. \(\frac{2}{8}\)
c. \(\frac{6}{8}\)
d. \(\frac{8}{8}\)

Answer: \(\frac{2}{8}\)

Explanation:

The rectangle is divided into 8 equal parts. Out of 8 two parts are shaded.
So, the fraction name of the shaded part is \(\frac{2}{8}\)
The correct answer is option B.

Question 5.
Chip worked at the animal shelter for 6 hours each week for several weeks. He worked for a total of 42 hours. Which of the following can be used to find the number of weeks Chip worked at the animal shelter?
Options:
a. 6 + 42
b. 42 − 6
c. 42 ÷ 6
d. 42 × 6

Answer: 42 ÷ 6

Explanation:

Chip worked at the animal shelter for 6 hours each week for several weeks.
Number of hours he worked = 42 hours
Number of weeks he worked at the animal shelter = x
x × 6 = 42
x = 42 ÷ 6
Thus the correct answer is option C.

Question 6.
Mr. Jackson has 20 quarters. If he gives 4 quarters to each of his children, how many children does Mr. Jackson have?
Options:
a. 3
b. 4
c. 5
d. 6

Answer: 5

Explanation:

Given, Mr. Jackson has 20 quarters.
If he gives 4 quarters to each of his children
Number of children Mr. Jackson have = y
y × 4 = 20
y = 20/4 = 5
Therefore, Mr. Jackson has 5 children.

Compare Fractions – Page No. 529

Compare. Write <, >, or =. Write the strategy you used.

Question 1.
Go Math Grade 3 Answer Key Chapter 9 Compare Fractions Compare Fractions img 11

Answer: Same Numerator

Question 2.
\(\frac{2}{3}\) ______ \(\frac{7}{8}\)

Answer: \(\frac{2}{3}\) < \(\frac{7}{8}\)

Missing pieces

Explanation:

Compare the fractions \(\frac{2}{3}\), \(\frac{7}{8}\)
The numerators and denominators are different here.
\(\frac{2}{3}\) × \(\frac{8}{8}\) = \(\frac{16}{24}\)
\(\frac{7}{8}\) × \(\frac{3}{3}\) = \(\frac{21}{24}\)
16 is less than 24.
So, \(\frac{16}{24}\) < \(\frac{21}{24}\)
That means \(\frac{2}{3}\) < \(\frac{7}{8}\)

Question 3.
\(\frac{3}{4}\) ______ \(\frac{1}{4}\)

Answer: \(\frac{3}{4}\) > \(\frac{1}{4}\)

Explanation:

The Denominator is the same here. So compare the numerators.
\(\frac{3}{4}\), \(\frac{1}{4}\)
3 is greater than 1.
Thus \(\frac{3}{4}\) > \(\frac{1}{4}\)

Name a fraction that is less than or greater than the given fraction. Draw to justify your answer.

Question 4.
greater than \(\frac{1}{3}\)
Type below:
___________

Answer: \(\frac{2}{3}\)

Explanation:

Go Math Chapter 9 Answer Key Grade 3 Compare fractions image_5

\(\frac{2}{3}\) is greater than \(\frac{1}{3}\)

Question 5.
less than \(\frac{3}{4}\)
Type below:
___________

Answer: \(\frac{2}{4}\)

Explanation:

Go Math Solution Key Grade 3 Chapter 9 solution image_6

\(\frac{2}{4}\) is less than \(\frac{3}{4}\)

Problem Solving

Question 6.
At the third-grade party, two groups each had their own pizza. The blue group ate \(\frac{7}{8}\) pizza. The green group ate \(\frac{2}{8}\) pizza. Which group ate more of their pizza?
The _______ group

Answer: The blue group

Explanation:

Given,
At the third-grade party, two groups each had their own pizza.
The blue group ate \(\frac{7}{8}\) pizza.
The green group ate \(\frac{2}{8}\) pizza.
Compare the fractions of the blue group and green group.
\(\frac{7}{8}\) and \(\frac{2}{8}\)
The denominators are the same here. So compare the numerators.
The numerator with the greatest number will be the greatest fraction.
Therefore \(\frac{7}{8}\) > \(\frac{2}{8}\)
So, the blue group ate more pizza.

Question 7.
Ben and Antonio both take the same bus to school. Ben’s ride is \(\frac{7}{8}\) mile. Antonio’s ride is \(\frac{3}{4}\) mile. Who has a longer bus ride?
___________

Answer: Ben

Explanation:

Ben and Antonio both take the same bus to school.
Ben’s ride is \(\frac{7}{8}\) mile.
Antonio’s ride is \(\frac{3}{4}\) mile.
Compare the fractions \(\frac{7}{8}\), \(\frac{3}{4}\)
Make the denominators equal to compare the fractions.
\(\frac{3}{4}\) × \(\frac{8}{8}\) = \(\frac{24}{32}\)
\(\frac{7}{8}\) × \(\frac{4}{4}\) = \(\frac{28}{32}\)
\(\frac{28}{32}\) > \(\frac{24}{32}\)
\(\frac{7}{8}\) > \(\frac{3}{4}\)
Thus Ben has longer bus ride.

Compare Fractions – Page No. 530

Lesson Check

Question 1.
Which statement is correct?
Options:
a. \(\frac{2}{3}\) > \(\frac{7}{8}\)
b. \(\frac{2}{3}\) < \(\frac{7}{8}\)
c. \(\frac{2}{3}\) = \(\frac{7}{8}\)
d. \(\frac{7}{8}\) = \(\frac{2}{3}\)

Answer: \(\frac{2}{3}\) < \(\frac{7}{8}\)

Explanation:

A. \(\frac{2}{3}\) > \(\frac{7}{8}\)
Here the numerator of one fraction is greater than the numerator of the other fraction.
So, \(\frac{2}{3}\) is not greater than \(\frac{7}{8}\)
B. \(\frac{2}{3}\) < \(\frac{7}{8}\)
Here the numerator of one fraction is greater than the numerator of the other fraction.
2 is less than 7.
Thus the statement \(\frac{2}{3}\) < \(\frac{7}{8}\) is true.
Option B is the answer.

Question 2.
Which symbol makes the statement true?
\(\frac{2}{4}\) O \(\frac{2}{6}\)
a. >
b. <
c. =
d. none

Answer: >

Explanation:

Compare the fractions \(\frac{2}{4}\) & \(\frac{2}{6}\)
The numerator of both the fractions are the same.
So we need to compare the denominators.
The denominator with the greater number will the smallest fraction.
Therefore, \(\frac{2}{4}\) > \(\frac{2}{6}\)
The correct answer is option A.

Spiral Review

Question 3.
Cam, Stella, and Rose each picked 40 apples. They put all their apples in one crate. How many apples are in the crate?
Options:
a. 40
b. 43
c. 120
d. 123

Answer: 120

Explanation:

Given that, Cam, Stella, and Rose each picked 40 apples.
They put all their apples in one crate.
That means each person picked 40 apples = 40 + 40 + 40 = 120 apples
Therefore there are 120 apples in the crate.
The correct answer is option C.

Question 4.
Each shape is 1 whole. Which fraction is represented by the shaded part of the model?
Go Math Grade 3 Answer Key Chapter 9 Compare Fractions Compare Fractions img 12
Options:
a. \(\frac{2}{4}\)
b. \(\frac{4}{4}\)
c. \(\frac{8}{4}\)
d. \(\frac{8}{1}\)

Answer: \(\frac{8}{4}\)

Explanation:

From the figure we observe that there are 2 squares.
Each square is divided into 4 equal parts.
Total number of shaded parts = 4
So, the fraction of 1 whole shaded part is \(\frac{8}{4}\)
Thus the correct answer is option C.

Question 5.
Which related multiplication fact can you use to find
16 ÷ ■ = 2?
Options:
a. 4 × 4 = 16
b. 8 × 2 = 16
c. 8 × 1 = 8
d. 4 × 2 = 8

Answer: 8 × 2 = 16

Explanation:

16 ÷ ■ = 2
■ = 16/2 = 8
So, the related multiplication fact of 16 ÷ ■ = 2 is 8 × 2 = 16.
The correct answer is option B.

Question 6,
What is the unknown factor?
9 × ■ = 36
Options:
a. 7
b. 6
c. 4
d. 3

Answer: 4

Explanation:

■ is the unknown factor
9 × ■ = 36
■ = 36/9 = 4
Thus the correct answer is option C.

Mid -Chapter Checkpoint – Page No. 531

Concepts and Skills

Question 1.
When two fractions refer to the same whole, explain why the fraction with a lesser denominator has larger pieces than the fraction with a greater denominator.
Type below:
___________

Answer: If two fractions have the same numerator but different denominators the fraction with greater denominator is smaller.
Example: Let us consider an apple that is divided into equal parts.
If the apple is divided among 2 people.
Then each get \(\frac{1}{2}\) part of the apple.
If the apple is divided among 4 people.
Then each get \(\frac{1}{4}\) part of an apple.
Hence, in the first case, people get more amount of apple than the second.
Hence, if as a whole the denominator is less the fraction is greater.

Question 2.
When two fractions refer to the same whole and have the same denominators, explain why you can compare only the numerators
Type below:
___________

Answer: If the denominators are the same, then the fraction with the greater numerator is the greater fraction. The fraction with the lesser numerator is the lesser fraction.
Example:
Let us consider a pizza which is cut into 4 parts.
One person ate 3 pieces then the fraction is \(\frac{3}{4}\)
And the other person ate 1 piece then the fraction is \(\frac{1}{4}\)
Now compare the fractions \(\frac{1}{4}\) and \(\frac{3}{4}\)
First person ate 2 pieces more than the second person.
Hence the fraction with the greater numerator is the greater fraction.

Compare. Write <, >, or =.

Question 3.
\(\frac{1}{6}\) ______ \(\frac{1}{4}\)

Answer: \(\frac{1}{6}\) < \(\frac{1}{4}\)

Explanation:

\(\frac{1}{6}\), \(\frac{1}{4}\)
In this case the numerators are same and the denominators are different.
We know that the two fractions have the same numerator but different denominators the fraction with greater denominator is smaller.
So, \(\frac{1}{6}\) < \(\frac{1}{4}\)

Question 4.
\(\frac{1}{8}\) ______ \(\frac{1}{8}\)

Answer: \(\frac{1}{8}\) = \(\frac{1}{8}\)

Explanation:

Compare the fractions \(\frac{1}{8}\) & \(\frac{1}{8}\)
The numerators and the denominators are same here. Thus these are the equivalent fractions.
Hence, \(\frac{1}{8}\) = \(\frac{1}{8}\)

Question 5.
\(\frac{2}{8}\) ______ \(\frac{2}{3}\)

Answer: \(\frac{2}{8}\) < \(\frac{2}{3}\)

Explanation:

Compare the fractions \(\frac{2}{8}\) & \(\frac{2}{3}\)
The numerators are same and denominators are different.
As we know the denominators with the greatest number will be the smallest fraction.
\(\frac{2}{8}\) < \(\frac{2}{3}\)

Question 6.
\(\frac{4}{2}\) ______ \(\frac{1}{2}\)

Answer: \(\frac{4}{2}\) > \(\frac{1}{2}\)

Explanation:

Compare both the fractions \(\frac{4}{2}\) and \(\frac{1}{2}\)
In this case, the denominators are the same but the numerators are different.
So compare the numerators.
4 is greater than 1.
Thus \(\frac{4}{2}\) > \(\frac{1}{2}\)

Question 7.
\(\frac{7}{8}\) ______ \(\frac{3}{8}\)

Answer: \(\frac{7}{8}\) > \(\frac{3}{8}\)

Explanation:

Comparing the fractions \(\frac{7}{8}\) and \(\frac{3}{8}\)
The denominators are same but the numerators are different.
We know that the denominators are the same, then the fraction with the greater numerator is the greater fraction. The fraction with the lesser numerator is the lesser fraction.
7 is greater than 3.
So, \(\frac{7}{8}\) > \(\frac{3}{8}\)

Question 8.
\(\frac{5}{6}\) ______ \(\frac{2}{3}\)

Answer: \(\frac{5}{6}\) > \(\frac{2}{3}\)

Explanation:

Compare the fractions \(\frac{5}{6}\) and \(\frac{2}{3}\)
In this case the numerators and the denominators are different.
So, we have to make the denominators equal.
\(\frac{5}{6}\) × \(\frac{3}{3}\) = \(\frac{15}{18}\)
\(\frac{2}{3}\) × \(\frac{6}{6}\) = \(\frac{12}{18}\)
Now the denominators are same. So compare the numerators 15 is greater than 12.
So, \(\frac{15}{18}\) > \(\frac{12}{18}\)
That means \(\frac{5}{6}\) > \(\frac{2}{3}\)

Question 9.
\(\frac{2}{4}\) ______ \(\frac{3}{4}\)

Answer: \(\frac{2}{4}\) < \(\frac{3}{4}\)

Explanation:

The denominators are the same. So compare the fractions with the numerators.
2 is less than 3.
So, \(\frac{2}{4}\) < \(\frac{3}{4}\)

Question 10.
\(\frac{6}{6}\) ______ \(\frac{6}{8}\)

Answer: \(\frac{6}{6}\) > \(\frac{6}{8}\)

Explanation:

Here the numerators are same but the denominators are different.
We know that denominators with the greatest number will be the smallest fraction.
Therefore, \(\frac{6}{6}\) > \(\frac{6}{8}\)

Question 11.
\(\frac{3}{4}\) ______ \(\frac{7}{8}\)

Answer: \(\frac{3}{4}\) < \(\frac{7}{8}\)

Explanation:

Compare the fractions \(\frac{3}{4}\) and \(\frac{7}{8}\)
The numerators and denominators are different.
So, we have to make the denominators equal.
\(\frac{3}{4}\) × \(\frac{8}{8}\) = \(\frac{24}{32}\)
\(\frac{7}{8}\) × \(\frac{4}{4}\) = \(\frac{28}{32}\)
Now the denominators are equal. So compare the numerators of both the fractions.
\(\frac{24}{32}\) < \(\frac{28}{32}\)
Therefore \(\frac{3}{4}\) < \(\frac{7}{8}\)

Name a fraction that is less than or greater than the given fraction. Draw to justify your answer.

Question 12.
greater than \(\frac{2}{6}\)
Type below:
___________

Answer: \(\frac{4}{6}\)
\(\frac{4}{6}\) > \(\frac{2}{6}\)

Question 13.
less than \(\frac{2}{3}\)
Type below:
___________

Answer: \(\frac{1}{3}\)
\(\frac{1}{3}\) < \(\frac{2}{3}\)

Mid -Chapter Checkpoint – Page No. 532

Question 14.
Two walls in Tiffany’s room are the same size. Tiffany paints \(\frac{1}{4}\) of one wall. Roberto paints \(\frac{1}{8}\) of the other wall. Who painted a greater amount in Tiffany’s room?
___________

Answer: Tiffany

Explanation:

Given that, Two walls in Tiffany’s room are the same size. Tiffany paints \(\frac{1}{4}\) of one wall.
Roberto paints \(\frac{1}{8}\) of the other wall.
The numerators are the same and the denominators are different here.
So, we have to make the denominators same.
\(\frac{1}{4}\) × \(\frac{8}{8}\) = \(\frac{8}{32}\)
\(\frac{1}{8}\) × \(\frac{4}{4}\) = \(\frac{4}{32}\)
Now compare the fractions \(\frac{8}{32}\) and \(\frac{4}{32}\)
8 is greater than 4.
\(\frac{8}{32}\) > \(\frac{4}{32}\)
Therefore, Tiffany painted greater amount than Roberto.

Question 15.
Matthew ran \(\frac{5}{8}\) mile during track practice. Pablo ran \(\frac{5}{6}\) mile. Who ran farther?
___________

Answer: Pablo

Explanation:

Given,
Matthew ran \(\frac{5}{8}\) mile during track practice.
Pablo ran \(\frac{5}{6}\) mile.
Compare the fractions \(\frac{5}{8}\) and \(\frac{5}{6}\)
Numerators are the same and denominators are different.
The denominator with the greatest number will be the smallest fraction.
Thus \(\frac{5}{8}\) < \(\frac{5}{6}\)
Pablo ran farther than Matthew.

Question 16.
Mallory bought 6 roses for her mother. Two-sixths of the roses are red and \(\frac{4}{6}\) is yellow. Did Mallory buy fewer red roses or yellow roses?
___________

Answer: Red Roses

Explanation:

Mallory bought 6 roses for her mother.
Two-sixths of the roses are red and \(\frac{4}{6}\) is yellow.
Compare fractions \(\frac{4}{6}\) and \(\frac{2}{6}\)
The denominators of both the fractions are the same and numerators are different.
4 is greater than 2. So, \(\frac{4}{6}\) > \(\frac{2}{6}\)
Therefore Mallory buys fewer red roses.

Question 17.
Lani used \(\frac{2}{3}\) cup of raisins, \(\frac{3}{8}\) cup of cranberries, and \(\frac{3}{4}\) cup of oatmeal to bake cookies. Which ingredient did Lani use the least amount of?
___________

Answer: Cranberries

Explanation:

Lani used \(\frac{2}{3}\) cup of raisins, \(\frac{3}{8}\) cup of cranberries, and \(\frac{3}{4}\) cup of oatmeal to bake cookies.
Compare the fractions \(\frac{2}{3}\), \(\frac{3}{8}\) and \(\frac{3}{4}\)
The numerators and denominators are different in this case.
The denominator with the highest number will be the smallest fraction.
Therefore Lani used the least amount of cranberries.

Compare and Order Fractions – Page No. 537

Write the fractions in order from greatest to least.

Question 1.
Go Math Grade 3 Answer Key Chapter 9 Compare Fractions Compare and Order Fractions img 13

Question 2.
\(\frac{2}{8}\), \(\frac{5}{8}\), \(\frac{1}{8}\)
Type below:
___________

Answer: \(\frac{5}{8}\), \(\frac{2}{8}\), \(\frac{1}{8}\)

Explanation:

The denominators of three fractions are same. So compare the numerators of the fractions.
5 is greater than 2 is greater than 1.
Thus the order of the fraction is \(\frac{5}{8}\), \(\frac{2}{8}\), \(\frac{1}{8}\)

Question 3.
\(\frac{1}{3}\), \(\frac{1}{6}\), \(\frac{1}{2}\)
Type below:
___________

Answer: \(\frac{1}{2}\), \(\frac{1}{3}\), \(\frac{1}{6}\)

Explanation:

Compare the three fractions \(\frac{1}{3}\), \(\frac{1}{6}\), \(\frac{1}{2}\)
We observe that the numerators of the fractions are same and the denominators are different.
We know that the denominators with the smallest number will be the greater fraction.
Therefore, \(\frac{1}{2}\), \(\frac{1}{3}\), \(\frac{1}{6}\)

Question 4.
\(\frac{2}{3}\), \(\frac{2}{6}\), \(\frac{2}{8}\)
Type below:
___________

Answer: \(\frac{2}{3}\), \(\frac{2}{6}\), \(\frac{2}{8}\)

Explanation:

Comparing the fractions \(\frac{2}{3}\), \(\frac{2}{6}\), \(\frac{2}{8}\)
The numerators are the same but the denominators are different.
We know that the denominators with the greatest number will be the smallest fraction.
3 is greater than 6 is greater than 8.
So, the order of the fraction is \(\frac{2}{3}\), \(\frac{2}{6}\), \(\frac{2}{8}\)

Write the fractions in order from least to greatest.

Question 5.
\(\frac{2}{4}\), \(\frac{4}{4}\), \(\frac{3}{4}\)
Type below:
___________

Answer: \(\frac{2}{4}\), \(\frac{3}{4}\), \(\frac{4}{4}\)

Explanation:

The denominators are same but the numerators are different.
So compare the numerators of three fractions.
2 < 3 < 4
\(\frac{2}{4}\), \(\frac{3}{4}\), \(\frac{4}{4}\)

Question 6.
\(\frac{4}{6}\), \(\frac{5}{6}\), \(\frac{2}{6}\)
Type below:
___________

Answer: \(\frac{2}{6}\), \(\frac{4}{6}\), \(\frac{5}{6}\)

Explanation:

Compare the fractions \(\frac{4}{6}\), \(\frac{5}{6}\), \(\frac{2}{6}\)
The denominators are same but the numerators are different.
So compare the numerators of the three fractions.
The order of fractions from least to greatest is \(\frac{2}{6}\), \(\frac{4}{6}\), \(\frac{5}{6}\)

Question 7.
\(\frac{7}{8}\), \(\frac{0}{8}\), \(\frac{3}{8}\)
Type below:
___________

Answer: \(\frac{0}{8}\), \(\frac{3}{8}\), \(\frac{7}{8}\)

Explanation:

Compare the fractions \(\frac{7}{8}\), \(\frac{0}{8}\), \(\frac{3}{8}\)
The numerators of the fractions are different. But the denominators are same.
0 < 3 < 7
The order from least to greatest is \(\frac{0}{8}\), \(\frac{3}{8}\), \(\frac{7}{8}\)

Question 8.
\(\frac{3}{4}\), \(\frac{3}{6}\), \(\frac{3}{8}\)
Type below:
___________

Answer: \(\frac{3}{8}\), \(\frac{3}{6}\), \(\frac{3}{4}\)

Explanation:

Compare the fractions \(\frac{3}{4}\), \(\frac{3}{6}\), \(\frac{3}{8}\)
The numerators are same but the denominators of the three fractions are different.
So, compare the denominators.
The denominators with the greatest number will be the smallest fraction.
So, the order of fractions from least to greatest is \(\frac{3}{8}\), \(\frac{3}{6}\), \(\frac{3}{4}\)

Problem Solving

Question 9.
Mr. Jackson ran \(\frac{7}{8}\) mile on Monday. He ran \(\frac{3}{8}\) mile on Wednesday and \(\frac{5}{8}\) mile on Friday. On which day did Mr. Jackson run the shortest distance?
On ___________

Answer: Wednesday

Explanation:

Mr. Jackson ran \(\frac{7}{8}\) mile on Monday.
He ran \(\frac{3}{8}\) mile on Wednesday and \(\frac{5}{8}\) mile on Friday
The denominators of the fractions are the same. So, compare the numerators.
Compare to all Mr. Jackson run the shortest distance on Wednesday.

Question 10.
Delia has three pieces of ribbon. Her red ribbon is \(\frac{2}{4}\) foot long. Her green ribbon is \(\frac{2}{3}\) foot long. Her yellow ribbon is \(\frac{2}{6}\) foot long. She wants to use the longest piece for a project. Which color ribbon should Delia use?
The _______ ribbon

Answer: Green

Explanation:

Delia has three pieces of ribbon. Her red ribbon is \(\frac{2}{4}\) foot long.
Her green ribbon is \(\frac{2}{3}\) foot long.
Her yellow ribbon is \(\frac{2}{6}\) foot long.
Compare the fractions to know which color should Delia use.
\(\frac{2}{4}\), \(\frac{2}{3}\) and \(\frac{2}{6}\)
The numerators of the three fractions are the same but the denominators are different.
\(\frac{2}{3}\) is longest among all.
so, the answer is Green ribbon.

Compare and Order Fractions – Page No. 538

Lesson Check

Question 1.
Which list orders the fractions from least to greatest?
Options:
a. \(\frac{1}{8}\), \(\frac{1}{3}\), \(\frac{1}{6}\)
b. \(\frac{1}{3}\), \(\frac{1}{6}\), \(\frac{1}{8}\)
c. \(\frac{1}{8}\), \(\frac{1}{6}\), \(\frac{1}{3}\)
d. \(\frac{1}{6}\), \(\frac{1}{8}\), \(\frac{1}{3}\)

Answer: \(\frac{1}{8}\), \(\frac{1}{6}\), \(\frac{1}{3}\)

Explanation:

When the numerators are the same, think about the denominators to compare and order fractions.
The denominator with the greatest number is the smallest fraction.
\(\frac{1}{8}\) < \(\frac{1}{6}\) < \(\frac{1}{3}\)
So, the order is \(\frac{1}{8}\), \(\frac{1}{6}\), \(\frac{1}{3}\)

Question 2.
Which list orders the fractions from greatest to least?
Options:
a. \(\frac{3}{8}\), \(\frac{3}{6}\), \(\frac{3}{4}\)
b. \(\frac{3}{4}\), \(\frac{3}{6}\), \(\frac{3}{8}\)
c. \(\frac{3}{4}\), \(\frac{3}{8}\), \(\frac{3}{4}\)
d. \(\frac{3}{6}\), \(\frac{3}{4}\), \(\frac{3}{8}\)

Answer: \(\frac{3}{4}\), \(\frac{3}{6}\), \(\frac{3}{8}\)

Explanation:

If the numerators are the same, think about the denominators to compare and order fractions.
The denominators with the smallest number will be the greatest fraction.
\(\frac{3}{4}\) > \(\frac{3}{6}\) > \(\frac{3}{8}\)
Thus the fractions from greatest to least are \(\frac{3}{4}\), \(\frac{3}{6}\), \(\frac{3}{8}\)

Spiral Review

Question 3.
What fraction of the group of cars is shaded?
Go Math Grade 3 Answer Key Chapter 9 Compare Fractions Compare and Order Fractions img 14
Options:
a. \(\frac{3}{8}\)
b. \(\frac{1}{2}\)
c. \(\frac{5}{8}\)
d. \(\frac{3}{5}\)

Answer: \(\frac{3}{8}\)

Explanation:

Total number of cars = 8
Number of shaded cars among those 8 cars = 3
So, the fraction of the shaded cars = 3/8
Thus the answer is option A.

Question 4.
Wendy has 6 pieces of fruit. Of these, 2 pieces are bananas. What fraction of Wendy’s fruit is bananas?
Options:
a. \(\frac{2}{6}\)
b. \(\frac{2}{4}\)
c. \(\frac{4}{6}\)
d. \(\frac{2}{2}\)

Answer: \(\frac{2}{6}\)

Explanation:

Given that, Wendy has 6 pieces of fruit.
Of these, 2 pieces are bananas.
The fraction of Wendy’s fruit is 2/6
Thus the correct answer is \(\frac{2}{6}\) i.e., option A.

Question 5.
Toby collects data and makes a bar graph about his classmates’ pets. He finds that 9 classmates have dogs, 2 classmates have fish, 6 classmates have cats, and 3 classmates have gerbils. Which pet will have the longest bar on the bar graph?
Options:
a. dog
b. fish
c. cat
d. gerbil

Answer: dog

Explanation:

Number of classmates who have dogs = 9
Number of classmates who have fish = 2
Number of classmates who have cats = 6
Number of classmates who have gerbils = 3
So, dogs will have the longest bar on the bar graph.

Question 6.
The number sentence is an example of which multiplication property?
6 × 7 = (6 × 5) + (6 × 2)
Options:
a. Associative
b. Commutative
c. Distributive
d. Identity

Answer: Distributive

Explanation:

6 × 7 = (6 × 5) + (6 × 2)
Here 7 is distributed into 5 + 2
According to the distributive property, multiplying the sum of two or more addends by a number will give the same result as multiplying each addend individually by the number and then adding the products together.
So, the answer is option C.

Model Equivalent Fractions – Page No. 543

Shade the model. Then divide the pieces to find the equivalent fraction.

Question 1.
Go Math Grade 3 Answer Key Chapter 9 Compare Fractions Model Equivalent Fractions img 15

Answer: \(\frac{4}{8}\)
Explanation:

The figure shows that there are 8 equal parts and 4 of them are shaded.
The Fraction of the shaded part is \(\frac{4}{8}\)
Thus, \(\frac{4}{8}\) = \(\frac{2}{4}\)

Question 2.
Go Math Grade 3 Answer Key Chapter 9 Compare Fractions Model Equivalent Fractions img 16
\(\frac{1}{3}\) = \(\frac{■}{6}\)
\(\frac{□}{□}\)

Answer: \(\frac{2}{6}\)

Explanation:

Go Math Grade 3 Key Chapter 9 Review solution image 7

There are 6 equal parts in which 2 parts are shaded. Now the fraction for the shaded part is \(\frac{2}{6}\)

\(\frac{1}{3}\) = \(\frac{2}{6}\)

Use the number line to find the equivalent fraction.

Question 3.
Go Math Grade 3 Answer Key Chapter 9 Compare Fractions Model Equivalent Fractions img 17
\(\frac{1}{2}\) = \(\frac{■}{4}\)
\(\frac{□}{□}\)

Answer: \(\frac{2}{4}\)

HMH Go Math Answer Grade 3 Chapter 9 Review Solution image_8

The fraction \(\frac{1}{2}\) and \(\frac{2}{4}\) lies on the same point.

Therefore, \(\frac{1}{2}\) = \(\frac{2}{4}\)

Question 4.
Go Math Grade 3 Answer Key Chapter 9 Compare Fractions Model Equivalent Fractions img 18
\(\frac{3}{4}\) = \(\frac{■}{8}\)
\(\frac{□}{□}\)

Answer: \(\frac{6}{8}\)

Go Math Grade 3 Compare fractions key review solution image_9

The above figure shows that the point \(\frac{6}{8}\) and \(\frac{3}{4}\) lies on the same point on the number line.

Thus \(\frac{3}{4}\) = \(\frac{6}{8}\)

Problem Solving

Question 5.
Mike says that \(\frac{3}{3}\) of his fraction model is shaded blue. Ryan says that \(\frac{6}{6}\) of the same model is shaded blue. Are the two fractions equivalent? If so, what is another equivalent fraction?
___________

Answer: \(\frac{2}{2}\)

Explanation:

Mike says that \(\frac{3}{3}\) of his fraction model is shaded blue.
Ryan says that \(\frac{6}{6}\) of the same model is shaded blue.
The two fractions are equivalent.
\(\frac{3}{3}\) = \(\frac{6}{6}\) = \(\frac{2}{2}\)

Question 6.
Brett shaded \(\frac{4}{8}\) of a sheet of notebook paper. Aisha says he shaded \(\frac{1}{2}\) of the paper. Are the two fractions equivalent? If so, what is another equivalent fraction?
___________

Answer: \(\frac{2}{4}\)

Explanation:

\(\frac{1}{2}\) = \(\frac{4}{8}\)
So, the two fractions are equivalent.
The another equivalent fraction is \(\frac{2}{4}\).

Model Equivalent Fractions – Page No. 544

Lesson Check

Question 1.
Find the fraction equivalent to \(\frac{2}{3}\).
Go Math Grade 3 Answer Key Chapter 9 Compare Fractions Model Equivalent Fractions img 19
Options:
a. \(\frac{3}{2}\)
b. \(\frac{4}{6}\)
c. \(\frac{3}{6}\)
d. \(\frac{1}{3}\)

Answer: \(\frac{4}{6}\)

Go Math Grade 3 Answer Key Compare fractions review solution image_10

\(\frac{2}{3}\) = \(\frac{4}{6}\)

Question 2.
Find the fraction equivalent to \(\frac{1}{4}\).
Go Math Grade 3 Answer Key Chapter 9 Compare Fractions Model Equivalent Fractions img 20
Options:
a. \(\frac{1}{2}\)
b. \(\frac{2}{4}\)
c. \(\frac{2}{8}\)
d. \(\frac{6}{8}\)

Answer: \(\frac{2}{8}\)

Explanation:

The fugure shows that the \(\frac{1}{4}\) and \(\frac{2}{8}\) lies on the same point.
So, the equivalent fraction of \(\frac{1}{4}\) is \(\frac{2}{8}\)
Thus the correct answer is option C.

Spiral Review

Question 3.
Eric practiced piano and guitar for a total of 8 hours this week. He practiced the piano for \(\frac{1}{4}\) of that time. How many hours did Eric practice the piano this week?
Options:
a. 6 hours
b. 4 hours
c. 3 hours
d. 2 hours

Answer: 2 hours

Explanation:

Eric practiced piano and guitar for a total of 8 hours this week.
He practiced the piano for \(\frac{1}{4}\) of that time.
To find how many hours did Eric practice the piano this week
You need to multiply the total number of hours with a fraction of the time he practiced.
8 ×  \(\frac{1}{4}\) = 2 hours
Thus the correct answer is option D.

Question 4.
Kylee bought a pack of 12 cookies. One-third of the cookies are peanut butter. How many of the cookies in the pack are peanut butter?
Options:
a. 9
b. 6
c. 4
d. 3

Answer: 4

Explanation:

Given,
Kylee bought a pack of 12 cookies.
One-third of the cookies are peanut butter.
To find the number of cookies in the pack is peanut butter.
Multiply number of cookies with a fraction of cookies are peanut butter
12 × \(\frac{1}{3}\) = 4
So, the correct answer is option C.

Question 5.
There are 56 students going to the game. The coach puts 7 students in each van. Which number sentence can be used to find how many vans are needed to take the students to the game?
Options:
a. 56 + 7 = ■
b. ■ + 7 = 56
c. ■ × 7 = 56
d. 56 − 7 = ■

Answer: ■ × 7 = 56

Explanation:

There are 56 students going to the game.
The coach puts 7 students in each van.
Let ■ be the number of vans
56 ÷ 7 = ■
■ × 7 = 56
■ = 8
Thus the correct answer is option C.

Question 6.
Which number sentence can be used to describe the picture?
Go Math Grade 3 Answer Key Chapter 9 Compare Fractions Model Equivalent Fractions img 21
Options:
a. 2 + 4 = 6
b. 4 − 2 = 2
c. 4 × 1 = 4
d. 8 ÷ 2 = 4

Answer: 8 ÷ 2 = 4

Explanation:

Number of counters = 8
Number of equal groups = 4
Number in each group = 2
The division equation is 8 ÷ 2 = 4
So, the answer is option D.

Lesson 9.7 – Page No. 548

Question 13.
Christy bought 8 muffins. She chose 2 apple, 2 banana, and 4 blueberry. She and her family ate the apple and banana muffins for breakfast. What fraction of the muffins did they eat? Write an equivalent fraction. Draw a picture.
\(\frac{□}{□}\)

Answer: \(\frac{4}{8}\)

Explanation:

Given:
Christy bought 8 muffins.
She chose 2 apple, 2 banana, and 4 blueberries. She and her family ate the apple and banana muffins for breakfast.
They had 2 apples and 2 banana muffins for their breakfast.
Only 4 blueberries are left out of 8 muffins.
The fraction of the muffins they ate = \(\frac{4}{8}\) or \(\frac{1}{2}\)

Question 14.
After dinner, \(\frac{2}{3}\) of the cornbread is left. Suppose 4 friends want to share it equally. What fraction names how much of the whole pan of corn bread each friend will get? Use the model on the right. Explain your answer.
Go Math Grade 3 Answer Key Chapter 9 Compare Fractions Lesson 9.7 img 22
\(\frac{□}{□}\)

Answer: \(\frac{1}{6}\)

Explanation:

To know the fraction of the whole pan of cornbread that each friend get. I divide each third into 2 equal pieces to get 4 pieces in all.
\(\frac{2}{3}\) = \(\frac{1}{6}\) + \(\frac{1}{6}\) + \(\frac{1}{6}\) + \(\frac{1}{6}\)
That means each friend gets \(\frac{1}{6}\) of cornbread of whole pan.

Question 15.
There are 16 people having lunch. Each person wants \(\frac{1}{4}\) of a pizza. How many whole pizzas are needed? Draw a picture to show your answer.
______ pizzas

Answer: 4 pizzas

Explanation:

Given that, There are 16 people having lunch.
Each person wants \(\frac{1}{4}\) of a pizza.
Multiply the total number of people with fraction of each pizza for one person.
= 16 × \(\frac{1}{4}\)
= 16/4 = 4
Therefore 4 pizzas are needed for 16 people.

Question 16.
Lucy has 5 oatmeal bars, each cut in half. What fraction names all of the oatmeal bar halves?
Go Math Grade 3 Answer Key Chapter 9 Compare Fractions Lesson 9.7 img 23
\(\frac{□}{□}\)

Answer: \(\frac{10}{2}\)

Explanation:

Lucy has 5 oatmeal bars, each cut in half.
Total number of oatmeal bars = 5
5 oatmeal bars are divided into halves = 5 + 5 = 10
So. the fraction name for all of the oatmeal bar halves = \(\frac{10}{2}\)

Question 16.
What if Lucy cuts each part of the oatmeal bar into 2 equal pieces to share with friends? What fraction names all of the oatmeal bar pieces now?
\(\frac{□}{□}\)

Answer: \(\frac{10}{2}\)

Explanation:

The above figure shows that there are 5 oatmeal bars.
And each oatmeal bar is divided into halves = 5 × 2 = 10
The fraction for the all of the oatmeal bar halves = \(\frac{10}{2}\)
The equivalent fraction of \(\frac{10}{2}\) is \(\frac{20}{4}\)

Question 17.
Mr. Peters made a pizza. There is \(\frac{4}{8}\) of the pizza left over. Select the fractions that are equivalent to the part of the pizza that is left over. Mark all that apply.
Go Math Grade 3 Answer Key Chapter 9 Compare Fractions Lesson 9.7 img 24
Options:
a. \(\frac{5}{8}\)
b. \(\frac{3}{4}\)
c. \(\frac{2}{4}\)
d. \(\frac{1}{2}\)

Answer: \(\frac{2}{4}\)

Explanation:

Mr. Peters made a pizza. There is \(\frac{4}{8}\) of the pizza left over.
The equivalent fraction of \(\frac{4}{8}\) is \(\frac{2}{4}\)
So, the correct answer is option C.

Equivalent Fractions – Page No. 549

Each shape is 1 whole. Shade the model to find the equivalent fraction.

Question 1.
Go Math Grade 3 Answer Key Chapter 9 Compare Fractions Equivalent Fractions img 25

Answer: \(\frac{3}{6}\)

Explanation:

The circle is divided into 6 equal groups. Each group is \(\frac{1}{6}\) of the whole circle.
There are 3 shaded parts in the circle.
So, the fraction of the shaded part is \(\frac{3}{6}\).
\(\frac{1}{2}\) = \(\frac{3}{6}\)

Question 2.
Go Math Grade 3 Answer Key Chapter 9 Compare Fractions Equivalent Fractions img 26
\(\frac{3}{4}\) = \(\frac{6}{■}\)
\(\frac{□}{□}\)

Answer: \(\frac{6}{8}\)

Explanation:

The square is divided into 8 equal parts. Each group is \(\frac{1}{8}\) of the whole square.
There are 6 shaded parts in the square.
Thus the fraction of the shaded part is 6/8
So, \(\frac{3}{4}\) = \(\frac{6}{8}\)

Circle equal groups to find the equivalent fraction.

Question 3.
Go Math Grade 3 Answer Key Chapter 9 Compare Fractions Equivalent Fractions img 27
\(\frac{2}{4}\) = \(\frac{■}{2}\)
\(\frac{□}{□}\)

Answer: \(\frac{1}{2}\)

Explanation:

Rectangle is divided into 4 equal parts. The fraction of each group is 1/4.
There are 2 shaded parts out of 4 = \(\frac{2}{4}\)
latex]\frac{2}{4}[/latex] = \(\frac{1}{2}\)

Question 4
Go Math Grade 3 Answer Key Chapter 9 Compare Fractions Equivalent Fractions img 28
\(\frac{4}{6}\) = \(\frac{■}{3}\)
\(\frac{□}{□}\)

Answer: \(\frac{2}{3}\)

Explanation:

The figure shows that the rectangle is divided into 6 equal parts and 4 are shaded among them.
The fraction of the shaded part is \(\frac{4}{6}\)
We can also write it as \(\frac{2}{3}\)
Thus, \(\frac{4}{6}\) = \(\frac{2}{3}\)

Problem Solving

Question 5.
May painted 4 out of 8 equal parts of a poster board blue. Jared painted 2 out of 4 equal parts of a same-size poster board red. Write fractions to show which part of the poster board each person painted.
Type below:
____________

Answer: May \(\frac{4}{8}\); Jared \(\frac{2}{4}\)

Explanation:

Given that, May painted 4 out of 8 equal parts of a poster board blue.
4 parts are shaded out of 8 parts. So, the fraction of the shaded part is \(\frac{4}{8}\)
Jared painted 2 out of 4 equal parts of a same-size poster board red.
The fraction of the shaded part is \(\frac{2}{4}\)

Question 6.
Are the fractions equivalent? Draw a model to explain.
____________

Answer: Yes

Equivalent Fractions – Page No. 550

Lesson Check

Question 1.
Which fraction is equivalent to \(\frac{6}{8}\)?
Go Math Grade 3 Answer Key Chapter 9 Compare Fractions Equivalent Fractions img 29
Options:
a. \(\frac{1}{4}\)
b. \(\frac{1}{3}\)
c. \(\frac{3}{4}\)
d. \(\frac{4}{6}\)

Answer: \(\frac{3}{4}\)

Explanation:

Go Math Grade 3 Chapter 9 Answer Key Review solution_1

\(\frac{6}{8}\) = \(\frac{3}{4}\)
So, the correct answer is option C.

Question 2.
Which fraction is equivalent to \(\frac{1}{3}\) ?
Go Math Grade 3 Answer Key Chapter 9 Compare Fractions Equivalent Fractions img 30
Options:
a. \(\frac{1}{6}\)
b. \(\frac{2}{8}\)
c. \(\frac{2}{6}\)
d. \(\frac{2}{3}\)

Answer: \(\frac{2}{6}\)

Explanation:

Go Math Grade 3 Key Chapter 9 Review solution image_4

\(\frac{1}{3}\) = \(\frac{2}{6}\)
The correct answer is option C.

Spiral Review

Question 3.
Which number sentence is shown by the array?
Go Math Grade 3 Answer Key Chapter 9 Compare Fractions Equivalent Fractions img 31
Options:
a. 8 − 2 = 6
b. 8 × 1 = 8
c. 2 + 8 = 10
d. 16 ÷ 2 = 8

Answer: 16 ÷ 2 = 8

Explanation:

Number of counters = 16
Number of rows = 2
Divide Number of counters by number of rows
= 16 ÷ 2 = 8
Thus the correct answer is option D.

Question 4.
Cody put 4 plates on the table. He put 1 apple on each plate. Which number sentence can be used to find the total number of apples on the table?
Options:
a. 4 + 1 = 5
b. 4 − 1 = 3
c. 4 × 1 = 4
d. 4 ÷ 2 = 2

Answer: 4 × 1 = 4

Explanation:

Cody put 4 plates on the table.
He put 1 apple on each plate.
The total number of apples on each table = 4 × 1 = 4
So, the correct answer is option C.

Question 5.
Which number sentence is a related fact to 7 × 3 = 21?
Options:
a. 7 + 3 = 10
b. 7 − 3 = 4
c. 7 × 2 = 14
d. 21 ÷ 3 = 7

Answer: 21 ÷ 3 = 7

Explanation:

The related division fact of 7 × 3 = 21 is 21 ÷ 3 = 7
The correct answer is option D.

Question 6.
Find the quotient.
4)\(\bar{3 6}\)
Options:
a. 9
b. 8
c. 7
d. 6

Answer: 9

Explanation:

36 ÷ 4 = 9
4 divides 36 nine times. So the quotient is 9.
So, the correct answer is option A.

Review/Test – Page No. 551

Question 1.
Alexa and Rose read books that have the same number of pages. Alexa’s book is divided into 8 equal chapters. Rose’s book is divided into 6 equal chapters. Each girl has read 3 chapters of her book.
Write a fraction to describe what part of the book each girl read. Then tell who read more pages. Explain.
Type below:
_____________

Answer: Rose read more pages than Alexa

Explanation:

Given that,
Alexa and Rose read books that have the same number of pages.
Alexa’s book is divided into 8 equal chapters.
Rose’s book is divided into 6 equal chapters.
Each girl has read 3 chapters of her book.
The fraction of Alexa’s book = \(\frac{3}{8}\)
The fraction of Rose’s book = \(\frac{3}{6}\)
Now, compare the fractions to find who read more pages.
\(\frac{3}{8}\) & \(\frac{3}{6}\)
The numerators of the two fractions are the same. So compare the denominators.
The denominator of the greater number will be the smallest fraction.
\(\frac{3}{8}\) < \(\frac{3}{6}\)
By this, we can say that Rose read more pages than Alexa.

Question 2.
David, Maria, and Simone are shading same-sized index cards for a science project. David shaded \(\frac{2}{4}\) of his index card. Maria shaded \(\frac{2}{8}\) of her index card and Simone shaded \(\frac{2}{6}\) of her index card.
For 2a–2d, choose Yes or No to indicate whether the comparisons are correct.
a. \(\frac{2}{4}\) > \(\frac{2}{8}\)
i. yes
ii. no

Answer: Yes

Explanation:

\(\frac{2}{4}\) > \(\frac{2}{8}\)
The denominators with the smallest number will be the greatest fraction.
Thus the statement \(\frac{2}{4}\) > \(\frac{2}{8}\) is true.

Question 2.
b. \(\frac{2}{8}\) > \(\frac{2}{6}\)
i. yes
ii. no

Answer: No

Explanation:

\(\frac{2}{8}\) > \(\frac{2}{6}\)
The numerators are same so compare the denominators.
The denominator with the greatest number will be the smallest fraction.
\(\frac{2}{8}\) < \(\frac{2}{6}\)
Thus the statement is false.

Question 2.
c. \(\frac{2}{6}\) < \(\frac{2}{4}\)
i. yes
ii. no

Answer: Yes

Explanation:

\(\frac{2}{6}\), \(\frac{2}{4}\)
The numerators are same so compare the denominators.
The denominator with the greatest number will be the smallest fraction.
\(\frac{2}{6}\) < \(\frac{2}{4}\)
Thus the statement is correct.

Question 2.
d. \(\frac{2}{8}\) = \(\frac{2}{4}\)
i. yes
ii. no

Answer: No

Explanation:

\(\frac{2}{8}\) = \(\frac{2}{4}\)
The numerators and denominators are different.
So, \(\frac{2}{8}\) is not equal to \(\frac{2}{4}\)
The statement is false.

Question 3.
Dan and Miguel are working on the same homework assignment. Dan has finished \(\frac{1}{4}\) of the assignment. Miguel has finished \(\frac{3}{4}\) of the assignment. Which statement is correct? Mark all that apply.
Options:
a. Miguel has completed the entire assignment.
b. Dan has not completed the entire assignment.
c. Miguel has finished more of the assignment than Dan.
d. Dan and Miguel have completed equal parts of the assignment.

Answer: B & C are the correct statements.

Explanation:

Given, Dan and Miguel are working on the same homework assignment.
Dan has finished \(\frac{1}{4}\) of the assignment.
Miguel has finished \(\frac{3}{4}\) of the assignment.
A. Miguel has completed the entire assignment.
Miguel has finished \(\frac{3}{4}\) of the assignment. So the statement is false.
B. Dan has not completed the entire assignment.
Dan has finished \(\frac{1}{4}\) of the assignment. So the statement is true.
C. Miguel has finished more of the assignment than Dan.
\(\frac{3}{4}\) > \(\frac{1}{4}\)
So, the statement is true.
D. Dan and Miguel have completed equal parts of the assignment.
\(\frac{3}{4}\) is not equal to \(\frac{1}{4}\)
Thus the statement is false.
So the correct answer is B & C.

Review/Test – Page No. 552

Question 4.
Bryan cut two peaches that were the same size for lunch. He cut one peach into fourths and the other into sixths. Bryan ate \(\frac{3}{4}\) of the first peach. His brother ate \(\frac{5}{6}\) of the second peach. Who ate more peach? Explain the strategy you used to solve the problem.
___________

Answer: Bryan’s brother

Explanation:

Given that,
Bryan cut two peaches that were the same size for lunch.
He cut one peach into fourths and the other into sixths.
Bryan ate \(\frac{3}{4}\) of the first peach.
His brother ate \(\frac{5}{6}\) of the second peach.
Compare the fractions \(\frac{3}{4}\) and \(\frac{5}{6}\)
The numerators and denominators are different.
\(\frac{3}{4}\) × \(\frac{6}{6}\) = \(\frac{18}{24}\)
\(\frac{5}{6}\) × \(\frac{3}{4}\) = \(\frac{15}{24}\)
\(\frac{15}{24}\) < \(\frac{18}{24}\)
By this we can say that Bryan’s brother ate more peach.

Question 5.
A nature center offers 2 guided walks. The morning walk is \(\frac{2}{3}\) mile. The evening walk is \(\frac{3}{6}\) mile. Which walk is shorter? Explain how you can use the model to find the answer.
Go Math Grade 3 Answer Key Chapter 9 Compare Fractions Review/Test img 32
___________

Answer: Evening walk

Explanation:

Given, The morning walk is \(\frac{2}{3}\) mile
The evening walk is \(\frac{3}{6}\) mile.
The shorter among both is \(\frac{3}{6}\) i.e, evening walk

HMH Go math Answer Key Grade 3 Chapter 9 Review solution image_3

\(\frac{2}{3}\) > \(\frac{3}{6}\)

Question 6.
Chun lives \(\frac{3}{8}\) mile from school. Gail lives \(\frac{5}{8}\) mile from school. Use the fractions and symbols to show which distance is longer.
Go Math Grade 3 Answer Key Chapter 9 Compare Fractions Review/Test img 33
\(\frac{□}{□}\) mile

Answer: \(\frac{3}{8}\) < \(\frac{5}{8}\)

Explanation:

Given,
Chun lives \(\frac{3}{8}\) mile from school.
Gail lives \(\frac{5}{8}\) mile from school.
Denominators are the same so we have to compare the numerators.
3 is less than 5. \(\frac{3}{8}\) < \(\frac{5}{8}\)

Review/Test – Page No. 553

Question 7.
Mrs. Reed baked four pans of lasagna for a family party. Use the rectangles to represent the pans.
Go Math Grade 3 Answer Key Chapter 9 Compare Fractions Review/Test img 34
Part A
Draw lines to show how Mrs. Reed could cut one pan of lasagna into thirds, one into fourths, one into sixths, and one into eighths.
Type below:
___________

Answer:

Mrs. Reed could cut one pan of lasagna into thirds:

Go Math Solution Key Grade 3 Chapter 9 Review solution image_ 2a

Mrs. Reed could cut one pan of lasagna into fourths:

Compare fractions Go Math Grade 3 Answer Key Review solution Image_ 2b

Mrs. Reed could cut one pan of lasagna into sixths:

HMH Go Math Key Grade 3 Chapter 9 Review Solution Image_2c

Mrs. Reed could cut one pan of lasagna into eighths:

Go Math Grade 3 Chapter 9 Key Review Solution Image_2d

Question 7.
Part B
At the end of the dinner, equivalent amounts of lasagna in two pans were left. Use the models to show the lasagna that might have been left over. Write two pairs of equivalent fractions to represent the models.
Type below:
___________

Answer:

Answer Key for Go Math Grade chapter 9 Review solution image_2e

\(\frac{1}{3}\) = \(\frac{2}{6}\)

Go Math Grade 3 Chapter 9 key review solution image_2f

\(\frac{1}{4}\) = \(\frac{2}{8}\)

Question 8.
Tom rode his horse for \(\frac{4}{6}\) mile. Liz rode her horse for an equal distance. What is an equivalent fraction that describes how far Liz rode? Use the models to show your work.
Go Math Grade 3 Answer Key Chapter 9 Compare Fractions Review/Test img 35
\(\frac{4}{6}\) = \(\frac{□}{□}\)

Answer: \(\frac{4}{6}\) = \(\frac{2}{3}\)

Explanation:

The above figure shows that the fraction of the first figure \(\frac{4}{6}\) is equal to the fraction of the second figure i.e., \(\frac{2}{3}\).

Question 9.
Avery prepares 2 equal-size oranges for the bats at the zoo. One dish has \(\frac{3}{8}\) of an orange. Another dish has \(\frac{1}{4}\) of an orange. Which dish has more orange? Show your work. \(\frac{□}{□}\)

Answer: First, we need to find an equivalent fraction to \(\frac{1}{4}\) so it would have the same denominator as \(\frac{3}{8}\)
\(\frac{1}{4}\) = \(\frac{2}{8}\) – equivalent fractions
Now we can compare the fractions:
\(\frac{2}{8}\) < \(\frac{3}{8}\)
Therefore \(\frac{1}{4}\) < \(\frac{3}{8}\)
So, the answer is \(\frac{3}{8}\)

Review/Test – Page No. 554

Question 10.
Jenna painted \(\frac{1}{8}\)of one side of a fence. Mark painted \(\frac{1}{6}\) of the other side of the same fence. Use >, =, or < to compare the parts that they painted.
\(\frac{1}{8}\) ______ \(\frac{1}{6}\)

Answer: \(\frac{1}{8}\) < \(\frac{1}{6}\)

Explanation:

Jenna painted \(\frac{1}{8}\)of one side of a fence.
Mark painted \(\frac{1}{6}\) of the other side of the same fence.
The numerators of both fractions are the same. So compare the denominators.
\(\frac{1}{8}\) & \(\frac{1}{6}\)
The denominator with the greatest number will be the smallest fraction.
So, \(\frac{1}{8}\) < \(\frac{1}{6}\)

Question 11.
Bill used \(\frac{1}{3}\) cup of raisins and \(\frac{2}{3}\) cup of banana chips to make a snack. For 11a–11d, select True or False for each comparison.
a. \(\frac{1}{3}\) > \(\frac{2}{3}\)
i. True
ii. False

Answer: False

Explanation:

The denominators are the same here. So check the numerators.
1 is less than 2.
\(\frac{1}{3}\) > \(\frac{2}{3}\)
The statement is false.

Question 11.
b. \(\frac{2}{3}\) = \(\frac{1}{3}\)
i. True
ii. False

Answer: False

Explanation:

The denominators and numerators are not equal in this equation.
So, the statement is false.

Question 11.
c. \(\frac{1}{3}\) < \(\frac{2}{3}\)
i. True
ii. False

Answer: True

Explanation:

The denominators of both the fractions are the same.
Compare the numerators. 1 is less than 2.
So, \(\frac{1}{3}\) < \(\frac{2}{3}\).
The statement is true.

Question 11.
d. \(\frac{2}{3}\) > \(\frac{1}{3}\)
i. True
ii. False

Answer: True

Explanation:

The denominators of both the fractions are the same.
Compare the numerators. 2 is greater than 1.
\(\frac{2}{3}\) > \(\frac{1}{3}\)
The statement is true.

Question 12.
Jorge, Lynne, and Crosby meet at the playground. Jorge lives \(\frac{5}{6}\) mile from the playground. Lynne lives \(\frac{4}{6}\) mile from the playground. Crosby lives \(\frac{7}{8}\) mile from the playground.
Part A
Who lives closer to the playground, Jorge or Lynne?
Explain how you know.
_____

Answer: Lynne

Explanation:

Jorge lives \(\frac{5}{6}\) mile from the playground.
Lynne lives \(\frac{4}{6}\) mile from the playground.
The denominators are the same. So, compare the numerators.
5 is greater than 4. So, \(\frac{5}{6}\) > \(\frac{4}{6}\)
Therefore, Lynne lives closer to the playground.

Question 12.
Part B
Who lives closer to the playground, Jorge or Crosby?
Explain how you know.
_____

Answer: Jorge

Explanation:

Jorge lives \(\frac{5}{6}\) mile from the playground.
Crosby lives \(\frac{7}{8}\) mile from the playground.
Compare the fraction of the both Jorge and Crosby.
\(\frac{5}{6}\) × \(\frac{8}{8}\) = \(\frac{40}{48}\)
\(\frac{7}{8}\) × \(\frac{6}{6}\) = \(\frac{42}{48}\)
\(\frac{40}{48}\) < \(\frac{42}{48}\)
Therefore, Jorge lives closer to the playground.

Review/Test – Page No. 555

Question 13.
Ming needs \(\frac{1}{2}\) pint of red paint for an art project. He has 6 jars that have the following amounts of red paint in them. He wants to use only 1 jar of paint. Mark all of the jars of paints that Ming could use.
Options:
a. \(\frac{2}{3}\) pint
b. \(\frac{1}{4}\) pint
c. \(\frac{4}{6}\) pint
d. \(\frac{3}{4}\) pint
e. \(\frac{3}{8}\) pint
f. \(\frac{2}{6}\) pint

Answer: \(\frac{2}{3}\) pint, \(\frac{3}{4}\) pint, \(\frac{4}{6}\) pint

Explanation:

We have to find all the jars that have an amount of paint greater than \(\frac{1}{2}\)
A. \(\frac{2}{3}\) pint > \(\frac{1}{2}\)
B. \(\frac{1}{4}\) pint < \(\frac{1}{2}\)
C. \(\frac{4}{6}\) pint > \(\frac{1}{2}\)
D. \(\frac{3}{4}\) pint < \(\frac{1}{2}\)
E. \(\frac{3}{8}\) pint < \(\frac{1}{2}\)
F. \(\frac{2}{6}\) pint < \(\frac{1}{2}\)

Question 14.
There are 12 people having lunch. Each person wants \(\frac{1}{3}\) of a sub sandwich. How many whole sub sandwiches are needed? Use the models to show your answer.
Go Math Grade 3 Answer Key Chapter 9 Compare Fractions Review/Test img 36
______ sub sandwiches

Answer: 4 sub sandwiches

Explanation:

Given that, There are 12 people having lunch.
Each person wants \(\frac{1}{3}\) of a sub sandwich.
Multiply the number of people with Each person wants of a sub sandwich.
12 × \(\frac{1}{3}\)
3 divides 12 four times. So, the answer is 4 sub sandwiches.

Question 15.
Mavis mixed \(\frac{2}{4}\) quart of apple juice with \(\frac{1}{2}\) quart of cranberry juice. Compare the fractions. Choose the symbol that makes the statement true.
\(\frac{2}{4}\) Go Math Grade 3 Answer Key Chapter 9 Compare Fractions Review/Test img 37 \(\frac{1}{2}\)
______

Answer:  \(\frac{2}{4}\) = \(\frac{1}{2}\)

Question 16.
Pat has three pieces of fabric that measure \(\frac{3}{6}\), \(\frac{5}{6}\), and \(\frac{2}{6}\) yards long. Write the lengths in order from least to greatest.
Type below:
___________

Answer: \(\frac{2}{6}\), \(\frac{3}{6}\), \(\frac{5}{6}\)

Explanation:

The denominators of \(\frac{3}{6}\), \(\frac{5}{6}\), and \(\frac{2}{6}\) are same. So, Compare the numerators 2 < 3, 5

Review/Test – Page No. 556

Question 17.
Cora measures the heights of three plants. Draw a line to match each height on the left to the word on the right that describes its place in the order of heights.
Go Math Grade 3 Answer Key Chapter 9 Compare Fractions Review/Test img 38
Type below:
___________

Answer:

Chapter 9 Go Math Grade 3 Answer Key Review solution image_1

Explanation:

\(\frac{4}{6}\) < \(\frac{4}{4}\)
\(\frac{4}{8}\) < \(\frac{4}{4}\)
\(\frac{4}{6}\) > \(\frac{4}{8}\)
The numerators are same. So, compare the denominators. The greatest fraction will have the lesser denominator.
\(\frac{4}{8}\) < \(\frac{4}{6}\) < \(\frac{4}{4}\).

Question 18.
Danielle drew a model to show equivalent fractions.
Go Math Grade 3 Answer Key Chapter 9 Compare Fractions Review/Test img 39
Use the model to complete the number sentence.
\(\frac{1}{2}\) = \(\frac{□}{□}\) = \(\frac{□}{□}\)
Type below:
___________

Answer: \(\frac{1}{2}\) = \(\frac{2}{4}\) = \(\frac{4}{8}\)

Total number of boxes are 4 and each are grouped into 2 = 8
Out of 4 boxes 2 boxes are shaded = \(\frac{2}{4}\)
\(\frac{2}{4}\) = \(\frac{1}{2}\)
Next out of 8 grouped squares 4 are shaded = \(\frac{4}{8}\)
\(\frac{4}{8}\) = \(\frac{1}{2}\)
Therefore the equivalent fractions are \(\frac{1}{2}\) = \(\frac{2}{4}\) = \(\frac{4}{8}\)

Question 19.
Floyd caught a fish that weighed \(\frac{2}{3}\) pound. Kira caught a fish that weighed \(\frac{7}{8}\) pound. Whose fish weighed more? Explain the strategy you used to solve the problem.
_____

Answer: Kira

Explanation:

We need to find equivalent fractions with the same denominator:
So Make the denominators of \(\frac{2}{3}\) and \(\frac{7}{8}\) equal.
\(\frac{2}{3}\) × \(\frac{8}{8}\) = \(\frac{16}{24}\)
\(\frac{7}{8}\) × \(\frac{3}{3}\) = \(\frac{21}{24}\)
\(\frac{16}{24}\) < \(\frac{21}{24}\)
Therefore \(\frac{2}{3}\) < \(\frac{7}{8}\)

Question 20.
Sam went for a ride on a sailboat. The ride lasted \(\frac{3}{4}\) hour.
Go Math Grade 3 Answer Key Chapter 9 Compare Fractions Review/Test img 40
What fraction is equivalent to \(\frac{3}{4}\)?
\(\frac{□}{□}\)

Answer: \(\frac{6}{8}\)

Explanation:

Total number of boxes are 4 and each is grouped into 2 = 8
Out of 4 boxes 3 boxes are shaded = \(\frac{3}{4}\)
Next out of 8 grouped squares 6 are shaded = \(\frac{6}{8}\)
Thus the fraction is equivalent to \(\frac{3}{4}\) is \(\frac{6}{8}\)

Conclusion

Learn the Go Math Answer Key for Grade 3 Chapter 9 Compare Fractions provided and improve your math as well as problem-solvingskills. You can acheieve greater heights and fall in love with Math with our Go Math Grade 3 Ch 9 Compare Fractions. To solve more such similar kinds of questions access Go Math Grade 3 Answer Key Chapter 9 Compare Fractions Extra Practice

Go Math Grade 5 Answer Key Chapter 10 Convert Units of Measure

go-math-grade-5-chapter-10-convert-units-of-measure-answer-key

Students can get Simple and Easy Tricks to Solve the Problems on Convert Units of Measure from Go Math Grade 5 Chapter 10 Convert Units of Measure. Utilize the Go Math Grade 5 Answer Key to score better grades and stand out from the crowd. Download the Go Math Grade 5 Answer Key Chapter 10 Convert Units of Measure free of cost and become a pro in the concepts underlying. Unlike other sources available out there we have mentioned the Solutions for all the Problems related to Converting Units of Measurement.

Go Math Grade 5 Answer Key Chapter 10 Convert Units of Measure

You can use the HMH Go Math Solution Key Grade Chapter 10 to get Answers for all the Problems in Chapter Test, Review Test, Cumulative Practice, etc. Convert Units of Measure has topics related to Customary Length, Weight, Customary Capacity, Metric Measure, etc. Thus Students are Advised to click on the below available links to find solutions along with the step by step explanation for the problems.

Chapter 10 – Lesson 1: Customary Length

Chapter 10 – Lesson 2: Customary Capacity 

Chapter 10 – Lesson 3: Weight

Chapter 10 – Lesson 4: Multistep Measurement

Chapter 10 – Mid-Chapter Checkpoint

Chapter 10 – Lesson 5: Metric Measures

Chapter 10 – Lesson 6: Problem Solving • Customary and Metric Conversions

Chapter 10 – Lesson 7: Elapsed Time

Chapter 10 – Review/Test

Share and Show – Lesson 1: Customary Length – Page No. 587

Convert.

Question 1.
2 mi = ______ yd

Answer: 3520 yards

Explanation:
Am changing the smaller unit into the larger unit.
We know that,
1 mile = 1760 yards
2 miles = 2 × 1760 yards = 3520 yards
Thus 2 miles = 3520 yards

Question 2.
6 yd = ______ ft

Answer: 18 feet

Explanation:
Am changing the smaller unit into the larger unit.
We know that,
1 yard = 3 feet
6 yards = 6 × 3 feet = 18 feet
6 yd = 18 ft.

Question 3.
90 in. = ______ ft ______ in.

Answer: 7 feet 6 in.
Am changing the larger unit into the smaller unit.
Convert inches to feet
1 feet = 12 inches
90 inches = 84 inches + 6 inches
84/12 = 7 feet
Thus 90 in. = 7 ft 6 in.

On Your Own

Convert.

Question 4.
57 ft = ______ yd

Answer: 19 yard

Explanation:
Convert the smaller unit to the larger unit.
We know that
1 yard = 3 feet
1 foot = 1/3 yard
57 × 1/3 = 19
Thus 57 feet = 19 yards
57 ft = 19 yd

Question 5.
13 ft = ______ in.

Answer: 156 inches

Explanation:
1 feet = 12 inches
13 feet = 13 × 12 inches = 156 inches
13 ft = 156 in.

Question 6.
240 in. = ______ ft

Answer: 20 feet

Explanation:
Convert the smaller unit to the larger unit.
1 feet = 12 inches
1 inch = 1/12 feet
240 inches = 240 × 1/12 feet = 20 feet
240 in. = 20 ft

Question 7.
6 mi = ______ ft

Answer: 31680 feet

Explanation:
1 mile = 5280 feet
6 miles = 6 × 5280 feet = 31680 feet
Thus 6 mi. = 31680 ft.

Question 8.
96 ft = ______ yd

Answer: 32 yard

Explanation:
1 yard = 3 feet
1 feet = 1/3 yard
96 feet = 96 × 1/3 yard = 32 yard
96 feet = 32 yard

Question 9.
75 in. = ______ ft ______ in

Answer: 6 feet 3 inches

Explanation:
1 feet = 12 inches
1 inch = 1/12 feet
75 inches = 75 × 1/12 feet = 6 feet + 3 inches
75 in. = 6 ft. 3 in.

Practice: Copy and Solve Convert.

Question 10.
60 in. = ■ ft
■ = ______ ft

Answer: 5 feet

Explanation:
Convert inches into feet
1 feet = 12 inches
■ be the unknown number.
60 in = ■ ft
60 inches × 1/12 = 5 feet
60 in = 5 feet

Question 11.
■ ft = 7 yd 1 ft
■ = ______ ft

Answer: 22 feet

Explanation:
■ ft = 7 yd 1 ft
1 yard = 3 feet
7 yard = 7 × 3 feet = 21 feet
21 ft + 1 ft = 22ft
■ = 22 feet

Question 12.
4 mi = ■ yd
■ = ______ yd

Answer: 7040 yard

Explanation:
We know that,
1 mile = 1760 yard
4 miles = 4 × 1760 yard = 7040 yard
4 mi. = 7040 yard

Question 13.
125 in. = ■ ft ■ in.
125 in. = ______ ft ______ in.

Answer: 10 ft 5 in.

Explanation:
Convert inches to feet.
12 inches = 1 feet
1 inch = 1/12 feet
120 × 1/12 = 10 feet
125 inches = 10 feet + 5 inches
Thus, 125 in. = 10 ft 5 in.

Question 14.
46 ft = ■ yd ■ ft
46 ft = ______ yd ______ ft

Answer: 15 yd 1 ft

Explanation:
We know that,
Converting the Larger unit into the smaller units
1 yard = 3 feet
1 foot = 1/3 yard
46 feet = 1/3 × 46 = 15 yard + 1 feet
Thus 46 ft = 15 yd 1 ft

Question 15.
42 yd 2 ft = ■ ft
■ = ______ ft

Answer: 128 feet

Explanation:
Converting larger unit into the smaller units
We know that,
1 yard = 3 feet
42 yard = 42 × 3 feet = 126 feet
42 yd 2 ft = 126 + 2 = 128 feet
Thus ■ = 128 ft
42 yd 2 ft = 128 ft

Compare. Write <, >, or =.

Question 16.
8 ft ______ 3 yd

Answer: 8 ft < 3 yd

Explanation:
1 yard = 3 feet
3 yards = 3 × 3 feet = 9 feet
8 is less than 9
Thus, 8 ft < 3 yd

Question 17.
2 mi ______ 10,500 ft

Answer: 2 mi > 10,500 ft

Explanation:
1 mile = 5280 feet
2 miles = 2 × 5280 = 10,560 feet
10,560 ft is greater than 10,500 feet
Thus, 2 mi > 10,500 ft

Question 18.
108 in. ______ 166 in.

Answer: 108 in. < 166 in.

Explanation:
108 is less than 166
Therefore, 108 in. < 166 in.

Problem Solving – Lesson 1: Customary Length – Page No. 588

Question 19.
Javon is helping his dad build a tree house. He has a piece of trim that is 13 feet long. How many pieces can Javon cut that are 1 yard long? How much of a yard will he have left over?
Go Math Grade 5 Answer Key Chapter 10 Convert Units of Measure Lesson 1: Customary Length img 1
Type below:
__________

Answer: 4 pieces, 4 yard 1 foot long

Explanation:

Javon is helping his dad build a tree house. He has a piece of trim that is 13 feet long.
Converting from feet into yards
We know that,
1 foot = 1/3 yard
13 feet = 12 feet + 1 foot
13 feet = 4 yard 1 foot
Javon can cut into 4 pieces.

Question 20.
Test Prep Katy’s driveway is 120 feet long. How many yards long is Katy’s driveway?
Options:
a. 60 yards
b. 40 yards
c. 20 yards
d. 10 yards

Answer: 40 yards

Explanation:
Katy’s driveway is 120 feet long.
Converting from feet into yards
We know that,
1 yard = 3 feet
1 foot = 1/3 yard
120 feet = 120 × 1/3 yard = 40 yard
Thus the correct answer is option B.

Compare and Contrast

When you compare and contrast, you tell how two or more things are alike and different. You can compare and contrast information in a table. Complete the table below. Use the table to answer the questions.
Go Math Grade 5 Answer Key Chapter 10 Convert Units of Measure Lesson 1: Customary Length img 2

Question 21.
How are the items in the table alike? How are they different?
Type below:
__________

Answer:
The table is the conversion from yards to feet and inches.
Go-Math-Grade-5-Answer-Key-Chapter-10-Convert-Units-of-Measure-img-2

Question 22.
What do you notice about the relationship between the number of larger units and the number of smaller units as the length increases?
Type below:
__________

Answer:
Converting the larger unit to the smaller unit.
1 yard = 3 feet
1 feet = 12 inches
3 feet = 12 × 3 = 36 inches
1 yard = 36 inches
The above table shows the conversion from yards to inches.

Share and Show – Lesson 2: Customary Capacity – Page No. 593

Go Math Grade 5 Answer Key Chapter 10 Convert Units of Measure Lesson 2: Customary Capacity img 3

Question 1.
Use the picture to complete the statements and convert 3 quarts to pints.
a. 1 quart = ______ pints

Answer: 2 pints

Explanation:
Convert the unit quarts to pints
We know that,
1 quart = 2 pints

Question 1.
b. 1 quart is ______ than 1 pint.

Answer: bigger

Explanation:
Convert the unit quarts to pints
1 quart = 2 pints
The unit quarts is greater than pints
1 quart is bigger than 1 pint.

Question 1.
c. 3 qt __________ pt in 1 qt = ____ pt
Type below:
__________

Answer: 6 pint

Explanation:
Convert the unit quarts to pints
1 quart = 2 pints
3 quarts = 3 × 2 pints = 6 pints
So, 3 qt 6 pt in 1 qt = 2 pt

Convert.

Question 2.
3 gal = ______ pt

Answer: 24 pt

Explanation:

Convert gallons to pints
We know that,
1 gallon = 8 pints
3 gallons = 3 × 8 pints = 24 pints
Thus 3 gal = 24 pt

Question 3.
5 qt = ______ pt

Answer: 10 pt

Explanation:

Convert the unit quarts to pints
1 quart = 2 pints
5 quarts = 5 × 2 pints = 10 pints
So, 5 qt = 10 pt

Question 4.
6 qt = ______ c

Answer: 24 c

Explanation:
Convert quarts to cups
1 quart = 4 cups
6 quarts = 6 × 4 cups = 24 cups
6 qt = 24 c

On Your Own – Lesson 2: Customary Capacity – Page No. 594

Convert.

Question 5.
38 c = ______ pt

Answer: 19 pt

Explanation:
Convert pints to cups
1 pint = 2 cups
1 cup = 1/2 pint
38 c = 1/2 × 38 = 19 pints
Thus, 38 c = 19 pints

Question 6.
36 qt = ______ gal

Answer: 9 gal

Explanation:
Convert Quarts to Gal
1 gal = 4 quarts
1 quart = 1/4 gallon
36 quarts = 1/4 × 36 = 9 gallons
So, 36 qt = 9 gal

Question 7.
104 fl oz = ______ c

Answer: 13 c

Explanation:
Convert fluid ounces to cups
1 cup = 8 fluid ounces
1 fluid ounces = 1/8 cups
104 fluid ounces = 1/8 × 104 = 13 cups
104 fl oz = 13 c

Question 8.
4 qt = ______ c

Answer: 16 c

Explanation:

Convert quarts to cups
1 quart = 4 cups
4 quarts = 4 × 4 cups
4 quarts = 16 cups
4 qt = 4 c

Question 9.
7 gal = ______ pt

Answer: 56 pt

Explanation:
Convert gallon to pints
1 gal = 8 pints
7 gallons = 7 × 8 pints = 56 pints
7 gal = 56 pt

Question 10.
96 fl oz = ______ pt

Answer: 6 pt

Explanation:
Convert fluid ounces into pints.
1 pint = 16 fluid ounces
1 fluid ounces = 1/16 pint
96 fluid ounces = 1/16 × 96 = 6
96 fl oz = 6 pt

Practice: Copy and Solve Convert.

Question 11.
200 c = ______ qt

Answer: 50 qt

Explanation:
Convert cups to quarts
1 quart = 4 cups
1 cup = 1/4 quart
200 cups = 1/4 × 200 = 50 qt
200 c = 50 qt

Question 12.
22 pt = ______ fl oz

Answer: 352 fl oz

Explanation:
Convert pints to fluid ounces
1 pint = 16 fluid ounces
22 pints = 16 × 22 = 352 fluid ounces
So, 22 pt = 352 fl oz

Question 13.
8 gal = ______ qt

Answer: 32 qt

Explanation:
Convert gallon to quarts.
1 gallon = 4 quarts
8 gallons = 8 × 4 quarts = 32 quarts
8 gal = 32 qt

Question 14.
72 fl oz = ______ c

Answer: 9 c

Explanation:
Convert fluid ounces to cups
1 cup = 8 fluid ounces
1 fluid ounce = 1/8 cup
72 fluid ounces = 1/8 cup × 72 = 9 cups
72 fl oz = 9 c

Question 15.
2 gal = ______ pt

Answer: 16 pt

Explanation:
Convert gallon to pints
1 gal = 8 pints
2 gal = 2 × 8 pints = 16 pints
2 gal = 16 pt

Question 16.
48 pt = ______ gal

Answer: 6 gal

Explanation:
Convert pints to gallons
1 gal = 8 pints
1 pint = 1/8 gal
48 pints = 1/8 × 48 pint = 6 gal
48 pint = 6 gal

Compare. Write <, >, or =.

Question 17.
28 c ______ 14 pt

Answer: 28 c = 14 pt

Explanation:
Convert cups to a pint
1 pint = 2 cups
14 pints = 14 × 2 cups = 28 cups
28 cups = 14 pints
Thus, 28 c = 14 pt

Question 18.
25 pt ______ 13 qt

Answer: 25 pt < 13 qt

Explanation:
Convert pints to quarts
1 quart = 2 pints
13 quarts = 13 × 2 pints = 26 pints
25 pints is less than 26 pints
So, 25 pt < 13 qt

Question 19.
20 qt ______ 80 c

Answer: 20 qt = 80 c

Explanation:
1 quart = 4 cups
20 quarts = 20 × 4 cups = 80 cups
20 qt = 80 c

Question 20.
12 gal ______ 50 qt

Answer: 12 gal < 50 qt

Explanation:
1 gal = 4 quarts
12 gal = 12 × 4 quarts = 48 quarts
12 gal = 48 quarts
48 quarts is less than 50 quarts
So, 12 gal < 50 qt

Question 21.
320 fl oz ______ 18 pt

Answer: 320 fl oz > 18 pt

Explanation:
1 pint = 16 fluid ounces
320 fluid ounces = 320 × 1/16 fluid ounces = 20 pints
20 is greater than 18 pints
320 fl oz > 18 pt

Question 22.
15 qt ______ 63 c

Answer: 15 qt < 63 c

Explanation:
Convert quarts to cups
1 quart = 4 pints
15 quarts = 4 × 15 = 60 cups
60 cup is less than 63 cups
So, 15 qt < 63 c

Question 23.
Which of exercises 17–22 could you solve mentally? Explain your answer for one exercise.

Problem Solving – Lesson 2: Customary Capacity – Page No. 4120

Show your work. For 24–26, use the table.
Go Math Grade 5 Answer Key Chapter 10 Convert Units of Measure Lesson 2: Customary Capacity img 4

Question 24.
Complete the table, and make a graph showing the relationship between pints and quarts. Draw a line through the points to make the graph.
Go Math Grade 5 Answer Key Chapter 10 Convert Units of Measure Lesson 2: Customary Capacity img 5

Answer:
Go-Math-Grade-5-Answer-Key-Chapter-10-Convert-Units-of-Measure-img-5

Question 25.
Describe any pattern you notice in the pairs of numbers you graphed. Write a rule to describe the pattern.
Type below:
__________

Answer: I observed a straight line in the pair of numbers.

Question 26.
Explain how you can use your graph to find the number of quarts equal to 5 pints.
Type below:
__________

Answer: The number of quarts for 5 pints is 3.5
The point lies between 3 and 4.
The X-axis is 5 and Y-axis is 3.5

Question 27.
Test Prep Shelby made 5 quarts of juice for a picnic. How many cups of juice did Shelby make?
Options:
a. 1 cup
b. 5 cups
c. 10 cups
d. 20 cups

Answer: 20 cups

Explanation:
Shelby made 5 quarts of juice for a picnic.
1 quarts = 4 cups
5 quarts = 5 × 4 cups = 20 cups
5 quart = 20 cups
Thus the correct answer is option D.

Share and Show – Lesson 3: Weight – Page No. 599

Question 1.
Use the picture to complete each equation.
Go Math Grade 5 Answer Key Chapter 10 Convert Units of Measure Lesson 3: Weight img 6
a. 1 pound = ______ ounces

Answer: 16 ounces

Explanation:
Convert pounds to ounces
1 pound = 16 ounces

Question 1.
b. 2 pounds = ______ ounces

Answer: 32 ounces

Explanation:
Convert pounds to ounces
1 pound = 16 ounces
2 pounds = 2 × 16 ounces = 32 ounces
2 pounds = 32 ounces
2 pounds = 32 ounces

Question 1.
c. 3 pounds = ______ ounces

Answer: 48 ounces

Explanation:
Convert pounds to ounces
1 pound = 16 ounces
3 pounds = 3 × 16 ounces = 48 ounces
So, 3 pounds = 48 ounces

Question 1.
d. 4 pounds = ______ ounces

Answer: 64 ounces

Explanation:
Convert pounds to ounces
1 pound = 16 ounces
4 pounds = 4 × 16 ounces = 64 ounces
4 pounds = 64 ounces

Question 1.
e. 5 pounds = ______ ounces

Answer: 80 ounces

Explanation:
Convert pounds to ounces
1 pound = 16 ounces
5 pounds = 5 × 16 ounces = 80 ounces
5 pounds = 80 ounces

Convert.

Question 2.
15 pounds = ______ ounces

Answer: 240 ounces

Explanation:
Convert pounds to ounces
1 pound = 16 ounces
15 pounds = 15 × 16 ounces = 240 ounces
15 pounds = 240 ounces

Question 3.
3 T = ______ lb

Answer: 6,000 lb

Explanation:
1 ton = 2,000 lb
3 ton = 3 × 2,000 lb = 6,000 lb
3 T = 6000 lb

Question 4.
320 oz = ______ lb

Answer: 20 lb

Explanation:
Convert ounces to lb.
1 lb = 16 ounces
1 ounce = 1/16 lb
320 oz = 1/16 × 320 = 20 lb
320 oz = 20 lb

On Your Own – Lesson 3: Weight – Page No. 600

Convert.

Question 5.
5 T = ______ lb

Answer: 10,000 lb

Explanation:
Convert Ton to lb
1 T = 2,000 lb
5 T = 5 × 2,000 lb = 10,000 lb
5 T = 10,000 lb

Question 6.
19 T = ______ lb

Answer: 38,000 lb

Explanation:
Convert Ton to lb
1 T = 2,000 lb
19 T = 19 × 2000 lb = 38,000 lb
19 T = 38,000 lb

Question 7.
16,000 lb = ______ T

Answer: 8 T

Explanation:
Convert Ton to lb
1 T = 2,000 lb
1 lb = 1/2000 T
16000 lb = 16000 × 1/2000 = 8T
16,000 lb = 8T

Question 8.
192 oz = ______ lb

Answer: 12 lb

Explanation:
Convert ouncers to pound
1 pound = 16 ounces
192 ounces = 192 × /16 = 12 lb
192 oz = 12 lb

Question 9.
416 oz = ______ lb

Answer: 26 lb

Explanation:
Convert ouncers to pound
1 pound = 16 ounces
1 ounce = 1/16 lb
416 oz = 416 × 1/16 = 26 lb
416 oz = 26 lb

Question 10.
24 lb = ______ oz

Answer: 384 oz

Explanation:
Convert ouncers to pound
1 pound = 16 ounces
24 lb = 24 × 16 ounces = 384 oz
24 lb = 384 oz

Practice: Copy and Solve Convert.

Question 11.
23 lb = ______ oz

Answer: 368 oz

Explanation:
Convert lb to ounces
1 lb = 16 oz
23 lb = 23 × 16 oz = 368 ounces
23 lb = 368 oz

Question 12.
6 T = ______ lb

Answer: 12,000 lb

Explanation:
Convert tons to pounds
1 T = 2,000 lb
6 T = 6 × 2,000 lb = 12,000 lb
6 T = 12,000 lb

Question 13.
144 oz = ______ lb

Answer: 9 lb

Explanation:
Convert ounces to pounds
1 pound = 16 ounces
1 oz= 1/16 lb
144 oz = 144 × 1/16 = 9 lb
Thus, 144 oz = 9 lb

Question 14.
15 T = ______ lb

Answer: 30,000 lb

Explanation:
Convert tons to pounds
1 T = 2,000 lb
15 T = 15 × 2,000 lb = 30,000 lb
15 T = 30,000 lb

Question 15.
352 oz = ______ lb

Answer: 22 lb

Explanation:
Convert ounces to the pound
1 lb = 16 oz
1 oz = 1/16 lb
352 oz = 352 × 1/16 = 22 lb
352 oz = 22 lb

Question 16.
18 lb = ______ oz

Answer: 288 oz

Explanation:
1 lb = 16 oz
18 lb = 18 × 16 oz = 288 oz
18 lb = 288 oz

Compare. Write >, >, or =.

Question 17.
130 oz ______ 8 lb

Answer: 130 oz > 8 lb

Explanation:
First convert lb to ounces
1 lb = 16 oz
8 lb = 8 × 16 oz = 128 oz
8 lb = 128 oz
128 oz is less than 130 oz
So, 130 oz > 8 lb

Question 18.
34 lb ______ 544 oz

Answer: 34 lb = 544 oz

Explanation:
First convert lb to ounces
1 lb = 16 oz
34 lb = 34 × 16 oz = 544 oz
So, 34 lb = 544 oz

Question 19.
14 lb ______ 229 oz

Answer: 14 lb < 229 oz

Explanation:
First convert lb to ounces
1 lb = 16 oz
14 lb = 14 × 16 oz = 224 oz
14 lb = 224 oz
224 oz is less than 229
So, 14 lb < 229 oz

Question 20.
16 T ______ 32,000 lb

Answer: 16 T = 32,000 lb

Explanation:
Convert ton to pounds
1 Ton = 2,000 lb
16 T = 16 × 2,000 lb = 32,000 lb
16 T = 32,000 lb

Question 21.
5 lb ______ 79 oz

Answer: 5 lb > 79 oz

Explanation:
Convert lb to oz
1 lb = 16 oz
5 lb = 5 × 16 oz = 80 oz
80 is greater than 76
Thus, 5 lb > 79 oz

Question 22.
85,000 lb ______ 40 T

Answer: 85,000 lb > 40 T

Explanation:
Convert ton to pounds
1 Ton = 2,000 lb
40 T = 40 × 2000 = 80,000 lb
80,000 lb is less than 85,000 lb
Thus, 85,000 lb > 40 T

Problem Solving

Question 23.
Explain how you can use mental math to compare 7 pounds to 120 ounces.
7 pounds ______ 120 ounces.

Answer: 7 pounds < 120 ounces.

Explanation:
Convert pounds to ounces
1 pound = 16 ounces
7 Pounds = 7 × 16 ounces = 112 ounces
112 ounces is less than 120 ounces
7 pounds < 120 ounces.

Question 24.
Test Prep Carlos used 32 ounces of walnuts in a muffin recipe. How many pounds of walnuts did Carlos use?
Options:
a. 8 pounds
b. 4 pounds
c. 2 pounds
d. 1 pound

Answer: 2 pounds

Explanation:
Given that, Carlos used 32 ounces of walnuts in a muffin recipe.
1 pound = 16 ounces
2 pounds = 2 × 16 oz = 32 ounces
32 oz = 2 pounds
Thus the answer is option C.

Problem Solving – Lesson 3: Weight – Page No. 4160

Pose a Problem
Go Math Grade 5 Answer Key Chapter 10 Convert Units of Measure Lesson 3: Weight img 7

Question 25.
Kia wants to have 4 pounds of munchies for her party. She has 36 ounces of popcorn and wants the rest to be pretzel sticks. How many ounces of pretzel sticks
does she need to buy?
Go Math Grade 5 Answer Key Chapter 10 Convert Units of Measure Lesson 3: Weight img 8
64 – 36 =
So, Kia needs to buy ____ ounces of pretzel sticks.
Write a new problem using different amounts of snacks. Some weights should be in pounds and others in ounces. Make sure the amount of snacks given is less than the total amount of snacks needed.
Pose a Problem                 Draw a bar model for your problem.
Then solve.
• Write an expression you could use to solve your problem.
Explain how the expression represents the problem.
Type below:
___________

Answer:
Kia wants to have 3 pounds of munchies for her party. She has 20 ounces of popcorn and wants the rest to be pretzel sticks. How many ounces of pretzel sticks does she need to buy?
Go Math Grade 5 Answer Key Chapter 10 img-1
48 oz – 20 oz = 28 oz
She needs to buy 28 ounces of pretzel sticks.

Share and Show – Lesson 4: Multistep Measurement Problems – Page No. 605

Solve.

Question 1.
After each soccer practice, Scott runs 4 sprints of 20 yards each. If he continues his routine, how many practices will it take for Scott to have sprinted a total of 2 miles combined?
Scott sprints _____ yards each practice. Since there are _____ yards in 2 miles, he will need to continue his routine for _____ practices.
Type below:
__________

Answer:
Multiply 20 yard × 4 = 80 yards
Now convert from yards to miles
1 mile = 1760 yard
x = 1760 × 2 = 3520 yards
p = 3520 yards/80 yard = 44
Thus he will need to do 44 practices.

Question 2.
A worker at a mill is loading 5-lb bags of flour into boxes to deliver to a local warehouse. Each box holds 12 bags of flour. If the warehouse orders 3 Tons of flour, how many boxes are needed to fulfill the order?
_____ bags

Answer: 100 bags

Explanation:
A worker at a mill is loading 5-lb bags of flour into boxes to deliver to a local warehouse.
Each box holds 12 bags of flour.
Pounds of flour per box
x = 12 × 5 lb = 60 lb
We need to multiply by the conversion rule
1 T = 2000 lb
Find out how many pounds are in 3 tons. Pounds of flour in the warehouse.
y = 3 T × 2000 lb = 6,000 lb
Divide by the number of pounds per box. 100 boxes are needed.
b = 6000/60 = 100 boxes.

Question 3.
Cory brings five 1-gallon jugs of juice to serve during parent night at his school. If the paper cups he is using for drinks can hold 8 fluid ounces, how many drinks can Cory serve for parent night?
_____ drinks

Answer: 80 drinks

Explanation:
First, convert from gallons to quarts
We are converting larger unit to the smaller unit.
1 gal = 4 qt
We need to multiply by the conversion rule.
x = 5 gal × 4 qt
x = 20 qt
Next, convert from quarts to pints.
2 pt = 1 qt
We need to multiply by the conversion rule.
y = 20 qt × 2 pt = 40 pt
Next, convert from pints to cups.
We are converting from a larger unit to a smaller unit.
1 pt = 2 cups
y = 40 pt × 2 c = 80 c
Now convert from cups to ounces
1 c = 8 oz
y = 80 c × 8 oz = 640 oz
d = 640 oz/ 8 oz = 80 drinks
Cory can serve 80 drinks for parent night.

On Your Own – Lesson 4: Multistep Measurement Problems – Page No. 606

Solve.

Question 4.
A science teacher needs to collect lake water for a lab she is teaching. The lab requires each student to use 4 fluid ounces of lake water. If 68 students are participating, how many pints of lake water will the teacher need to collect?
_____ pints

Answer: 1 pint

Explanation:
Find the total number of ounces the students use.
s = 68 × 4 oz = 272 oz
First, convert from ounces to cups.
1 c = 8 oz
Find how many cups are in 272 ounces
y = 272 oz ÷ 8 oz = 34 c
Now from cups to pints
1 pt = 2 c
Find how mant pints are in 34 cups
x = 34 c ÷ 2 c = 17 pt
18 pt – 17 pt = 1 pt
Thus 1 pint is leftover.

Question 5.
A string of decorative lights is 28 feet long. The first light on the string is 16 inches from the plug. If the lights on the string are spaced 4 inches apart, how many lights are there on the string?
_____ lights

Answer: 81 lights

Explanation:
There are no lights for the first 12 inches
After that, each of the remaining 27 feet is composed of
3 sets of:
4 inches with no light
27 feet × 3 = 81 lights

Question 6.
When Jamie’s car moves forward such that each tire makes one full rotation, the car has traveled 72 inches. How many full rotations will the tires need to make for Jamie’s car to travel 10 yards?
_____ rotations

Answer: 5 rotations

Explanation:
When Jamie’s car moves forward such that each tire makes one full rotation, the car has traveled 72 inches.
Convert from a smaller unit to the larger unit.
Convert from inches to yards.
36 in = 1 yard
x = 72 in/ 36 in = 2 yards
Find out how many rotations are needed.
y = 10 yard/ 2 yard = 5 yard
The tired need to make 5 rotations for Jame’s car to travel 10 yards.

Question 7.
A male African elephant weighs 7 Tons. If a male African lion at the local zoo weighs \(\frac{1}{40}\) of the weight of the male African elephant, how many pounds does the lion weigh?
_____ lb

Answer: 350 lb

Explanation:
Convert from Tons to pounds
1 T = 2,000 lb
Find out how many pounds are in 7 Tons.
y = 7 T × 2000 lb = 14,000 lb
The weight of the elephant is 14,000 lb
Find the weight of the lion
l = 14,000 × 1/40 = 350 lb
Therefore the weight of the lion is 350 pounds.

Question 8.
An office supply company is shipping a case of wooden pencils to a store. There are 64 boxes of pencils in the case. If each box of pencils weighs 2.5 ounces, what is the weight, in pounds, of the case of wooden pencils?
_____ pounds

Answer: 10 pounds

Explanation:
First, we need to find the total weight of the case of pencils
w = 64 boxes × 2.5 oz = 160 oz
Now convert from ounces to pounds
We are converting from a smaller unit to a larger unit.
1 lb = 16 oz
y = 160 oz/16 oz = 10 lb
Thus total is 10 pounds.

Question 9.
A gallon of unleaded gasoline weighs about 6 pounds. About how many ounces does 1 quart of unleaded gasoline weigh?
HINT: 1 quart = \(\frac{1}{4}\) of a gallon
_____ ounces

Answer: 24 ounces

Explanation:
A gallon of unleaded gasoline weighs about 6 pounds.
Convert from pounds to ounces.
1 lb = 16 ounces
y = 6 × 16 ounces = 96 ounces
The weight of a quart of unleaded gasoline
qw = 96 × 1/4 = 24 oz

UNLOCK the Problem – Lesson 4: Multistep Measurement Problems – Page No. 4200

Go Math Grade 5 Answer Key Chapter 10 Convert Units of Measure Lesson 4: Multistep Measurement Problems img 9

Question 10.
At a local animal shelter there are 12 small-size dogs and 5 medium-size dogs. Every day, the small-size dogs are each given 12.5 ounces of dry food and the medium-size dogs are each given 18 ounces of the same dry food. How many pounds of dry food does the shelter serve in one day?
a. What are you asked to find?
Type below:
___________

Answer: We are asked to find how many pounds of dry food does the shelter serves in one day

Question 10.
b. What information will you use?
Type below:
___________

Answer:
I will use the information about the dry food given to the small size dogs and medium size dogs.

Question 10.
c. What conversion will you need to do to solve the problem?
Type below:
___________

Answer: We need to convert from ounces to pounds.

Question 10.
d. Show the steps you use to solve the problem.
Type below:
___________

Answer:
First, convert from ounces to pounds.
The total amount of food given to the small size and medium size dogs = 12.5 ounces + 18 ounces = 30.5 ounces
1 ounce = 0.625 pounds
32.5 ounces = 30.5 × 0.625 = 1.906 pounds

Question 10.
e. Complete the sentences.
The small-size dogs eat a total of ___ ounces of dry food each day.
The medium-size dogs eat a total of ___ ounces of dry food each day.
The shelter serves ___ ounces, or ___ pounds, of dry food each day.
Type below:
___________

Answer:
The small-size dogs eat a total of 12.5 ounces of dry food each day.
The medium-size dogs eat a total of 18 ounces of dry food each day.
The shelter serves 30.5 ounces, or 1.906 pounds, of dry food each day.

Question 11.
Test Prep For a class assignment, students are asked to record the total amount of water they drink in one day. Melinda records that she drank four 8-fluid ounce glasses of water and two 1-pint bottles. How many quarts of water did Melinda drink during the day?
Options:
a. 2 quarts
b. 4 quarts
c. 6 quarts
d. 8 quarts

Answer: 2 quarts

Explanation:
Given that,
Melinda records that she drank four 8-fluid ounce glasses of water and two 1-pint bottles
Convert from fluid ounces to quarts.
1 quart = 32 fluid ounces
2 1-pint bottles = 2 pints
1 pint = 16 fluid ounces
2 pints = 32 fluid ounces
We know that,
32 fluid ounces = 1 quart
1 quart + 1 quart = 2 quarts.
Thus the correct answer is option A.

Mid-Chapter Checkpoint – Vocabulary – Page No. 609

Choose the best term from the box.
Go Math Grade 5 Answer Key Chapter 10 Convert Units of Measure Mid-Chapter Checkpoint img 10

Question 1.
The _______ of an object is how heavy the object is.
___________

Answer: Weight

Question 2.
The _______ of a container is the amount the container can hold.
___________

Answer: Capacity

Concepts and Skills

Convert.

Question 3.
5 mi = _____ yd

Answer: 8800 yd

Explanation:

Convert from miles to yards.
1 mile = 1760 yard
5 miles = 5 × 1760 yard = 8800 yard
5 mi = 8800 yd

Question 4.
48 qt = _____ gal

Answer: 12 gal

Explanation:
Convert from quart to gal
1 gal = 4 quart
1 quart = 1/4
48 qt = 48 × 1/4 = 12 gal
48 qt = 12 gal

Question 5.
9 T = _____ lb

Answer: 18,000 lb

Explanation:
Convert from tons to lb
1 T = 2,000 lb
9 T = 9 × 2,000 lb = 18,000 lb
9  = 18,000 lb

Question 6.
336 oz = _____ lb

Answer: 21 lb

Explanation:
Convert from ounces to pound
1 pound = 16 ounces
1 oz = 1/16 lb
336 oz = 336 × 1/16 lb = 21 lb
336 oz = 21 lb

Question 7.
14 ft = _____ yd _____ ft

Answer: 4 yard 2 ft

Explanation:
Convert from feet to yards.
1 yard = 3 feet
1 feet = 1/3 yard
12 feet = 1/3 × 12 ft = 4 yard
14 ft = 4 yard 2 ft

Compare. Write <, >, or =.

Question 9.
96 fl oz _____ 13 c

Answer: 96 fl oz < 13 c

Explanation:
Convert from Cups to fluid ounces
1 cup = 8 oz
13 c = 13 × 8 oz = 104 oz
96 oz is less than 104 oz
Thus, 96 fl oz < 13 c

Question 10.
25 lb _____ 384 oz

Answer: 25 lb > 384 oz

Explanation:
Convert from lb to ounces
1 lb = 16 oz
25 lb = 25 × 16 oz = 400 oz
400 oz is greater than 384 oz
So, 25 lb > 384 oz

Question 11.
8 yd _____ 288 in.

Answer: 8 yd = 288 in.

Explanation:
Convert from yards to inches
1 yard = 36 inches
8 yard = 8 × 36 inches = 288 inches
8 yard = 288 inches

Solve.

Question 12.
A standard coffee mug has a capacity of 16 fluid ounces. If Annie needs to fill 26 mugs with coffee, how many total quarts of coffee does she need?
_____ qt

Answer: 13 qt

Explanation:
Find the number of ounces.
s = 16 oz × 26 = 416 oz
Next, convert from ounces to cups.
1 c = 8 oz
Find how many cups are in 104 ounces.
y = 416 oz ÷ 8 oz = 52 c
Next, convert from cups to pints.
1 pt = 2 c
y = 52 c ÷ 2 c = 26 pt
Convert from pints to quarts
1 qt = 2 pints
y = 26 pint ÷ 2 pint = 13 qt
y = 13 qt
Thus she need 13 quarts of coffee.

Mid-Chapter Checkpoint – Vocabulary – Page No. 610

Question 13.
The length of a classroom is 34 feet. What is this measurement in yards and feet?
_____ yd _____ ft

Answer: 11 yard 1 foot

Explanation:
Given that, The length of a classroom is 34 feet.
Convert from feet from the yard
1 yard = 3 feet
34 feet = 33 feet + 1 foot
1 feet = 1/3 yard
33 feet = 33 × 1/3 = 11 yard
34 feet = 11 yard 1 foot

Question 14.
Charlie’s puppy, Max, weighs 8 pounds. How many ounces does Max weigh?
_____ oz

Answer: 128 ounces

Explanation:
Convert from pounds from ounces
1 pound = 16 ounces
8 pounds = 8 × 16 oz = 128 ounces
8 pounds = 128 oz

Question 15.
Milton purchases a 5-gallon aquarium for his bedroom. To fill the aquarium with water, he uses a container with a capacity of 1 quart. How many times will Milton fill and empty the container before the aquarium is full?
_____ times

Answer: 20 times

Explanation:
Convert from gallon to quart
1 gallon = 4 quart
5 gallon = 5 × 4 quart = 20 quart
5 gallon = 20 quart

Question 16.
Sarah uses a recipe to make 2 gallons of her favorite mixed berry juice. Two of the containers she plans to use to store the juice have a capacity of 1 quart. The rest of the containers have a capacity of 1 pint. How many pint-sized containers will Sarah need?
_____ pint-sized container

Answer: 12 pint-sized container

Explanation:
Sarah uses a recipe to make 2 gallons of her favorite mixed berry juice.
Two of the containers she plans to use to store the juice have a capacity of 1 quart.
1 gallon = 4 quart
Find how many quarts are in 2 gals.
y = 8 qt – 2 qt = 6 qt
Next, convert from quarts to pints.
2 pt = 1 qt
y = 6 qt × 2 pt
y = 12 pint
She will need 12 pint sized container.

Question 17.
The average length of a female white-beaked dolphin is about 111 inches. What is this length in feet and inches?
_____ ft _____ in.

Answer: 9 ft 3 in.

Explanation:
The average length of a female white-beaked dolphin is about 111 inches.
Convert from inches to feet.
1 feet = 12 inch
111 inch = 108 in. + 3 in
9 feet = 108 inches
111 inches = 9 feet 3 inches

Share and Show – Lesson 5: Metric Measures – Page No. 613

Complete the equation to show the conversion.

Question 1.
8.47 L _____ 10 = _____ dL

Answer: 8.47 L × 10 = 84.7 dL

Explanation:
Find the relationships between the units.
Determine the operation to be used.
Now convert from liter to deciliter.
84.7 L × 10 = 8.47 dL

Question 1.
8.47 L _____ 100 = _____ cL

Answer: 8.47 L × 100 = 847 cL

Explanation:
Find the relationships between the units.
Determine the operation to be used.
Convert from liter to centiliter.
8.47 L × 100 = 847 centiliter

Question 1.
8.47 L _____ 1,000 = _____ mL

Answer: 8.47 L × 1,000 = 8,470 mL

Explanation:
Find the relationships between the units.
Determine the operation to be used.
Convert the liter to the milliliter.
8.47 L × 1000 = 8470 mL

Question 2.
9,824 dg _____ 10 = _____ g

Answer: 9,824 dg ÷ 10 = 982.4 g

Explanation:
Find the relationships between the units.
Determine the operation to be used.
Now convert from decigram to gram
1 gram = 10 decigram
1 decigram = 1/10 gram
To convert 9824 dg to g we have to divide by 10.
9,824 dg ÷ 10 = 9824 × 1/10 = 982.4 grams
Thus, 9,824 dg ÷ 10 = 982.4 g

Question 2.
9,824 dg _____ 100 = _____ dag

Answer: 9,824 dg ÷ 100 = 98.24 dag

Explanation:
Find the relationships between the units.
Determine the operation to be used.
Now convert from decigram to dekagrams.
We know that,
1 dg = 0.01 dag
1 dg = 1/100 dag
9824 ÷ 100 = 9824 × 1/00 = 98.24 dag
Thus, 9,824 dg ÷ 100 = 98.24 dag

Question 2.
9,824 dg _____ 1,000 = _____ hg

Answer: 9,824 dg ÷ 1,000 = 9.824 hg

Explanation:
Find the relationships between the units.
Determine the operation to be used.
Now convert from decigram to dekagrams.
1 dg = 0.001 hg
1 dg = 1/000 hg
9,824 dg = 9824 × 1/1000 = 9824 ÷ 1000 = 9.824 hg
Thus, 9,824 dg ÷ 1,000 = 9.824 hg

Convert.

Question 3.
4,250 cm = _____ m

Answer: 42.50 m

Explanation:
Find the relationships between the units.
Converting from centimeters to meters.
1 cm = 0.01 m
1 cm = 1/100 m
4250 cm = 4250 × 1/100 = 4250/100 = 42.50 meters
So, 4,250 cm = 42.50 m

Question 4.
6,000 mL = _____ L

Answer: 6 L

Explanation:
Find the relationships between the units.
Converting from milliliters to liters.
1 liter = 1000 milliliters
1 milliliter = 1/1000 L
6000 mL = 6000 × 1/1000 L = 6 L
6,000 mL = 6 L

Question 5.
4 dg = _____ cg

Answer: 40 cg

Explanation:
Find the relationships between the units.
Converting from decigram to the centigram
We know that,
1 dg = 10 cg
4 dg = 4 × 10 cg = 40 cg
4 dg = 40 cg

On Your Own

Convert.

Question 6.
8 kg = _____ g

Answer: 8000 g

Explanation:
Find the relationships between the units.
Converting from kilograms to grams.
1 kg = 1000 grams
8 kg = 8 × 1000 grams = 8000 grams
8 kg = 8000 g

Question 7.
5 km = _____ m

Answer: 5000 m

Explanation:

Find the relationships between the units.
Converting from kilometers to meters
1 km = 1000 meters
5 km = 5 × 1000 meters = 5000 meters
5 km = 5000 m

Question 8.
40 mm = _____ cm

Answer: 4 cm

Explanation:
Converting from millimeters to centimeters
1 cm = 10 mm
1 mm = 1/10 cm
40 mm = 40 × 1/10 cm = 4 cm
40 mm = 4 cm

Question 9.
7 g = _____ mg

Answer: 7000 mg

Explanation:
Converting from grams to milligrams
1 gram = 1000 mg
7 g = 7 × 1000 mg = 7000 mg
7 g = 7000 mg

Question 10.
6,000 g = _____ kg

Answer: 6

Explanation:
Converting from grams to kilograms.
1 kg = 1000 grams
1 gram = 1/1000 kg
6000 grams = 6000 × 1/1000 = 6 kg
6000 grams = 6 kg

Question 11.
1,521 mL = _____ L

Answer: 1.521 L

Explanation:
Convert from liter to milliliters.
1 Liter = 1000 milliliters
1 milliliter = 1/1000 liter
1521 = 1521 × 1/1000 = 1.521 L
1521 mL = 1.521 L

Compare. Write <, >, or =.

Question 12.
32 hg _____ 3.2 kg

Answer: 32 hg = 3.2 kg

Explanation:
Converting from hectogram to kilogram
1 hg = 0.1 kg
32 hg = 32 × 0.1 kg = 3.2 kg
32 hg = 3.2 kg

Question 13.
6 km _____ 660 m

Answer: 6 km > 660 m

Explanation:
1 kilometer = 1000 meters
6 kilometer = 6 × 1000 meter = 6000 meters
6000 meters is greater than 600 m
6 km > 600 m

Question 14.
525 mL _____ 525 cL

Answer: 525 mL < 525 cL

Explanation:
Converting from milliliters to centiliters.
1 mL = 0.1 cL
525 mL = 525 × 0.1 = 52.5 mL
525 mL is less than 52.5 mL
Thus, 525 mL < 525 cL

Problem Solving – Lesson 5: Metric Measures – Page No. 614

For 15–16, use the table.
Go Math Grade 5 Answer Key Chapter 10 Convert Units of Measure Lesson 5: Metric Measures img 11

Question 15.
Kelly made one batch of peanut and pretzel snack mix. How many grams does she need to add to the snack mix to make 2 kilograms?
_____ g

Answer: 575 grams

Explanation:
Kelly made one batch of peanut and pretzel snack mix.
From the above figure, we can see that batch of peanut and pretzel snack mix is 1425 grams
To find how many grams she needs to add to the snack mix to make 2 kilograms
We have to subtract 1425 grams from 2 kgs
1 kg = 1000 grams
2 kg = 2000 grams
2000 g – 1425 g = 575 grams
Thus she needs to add 575 grams to make 2 kilograms.

Question 16.
Kelly plans to take juice on her camping trip. Which will hold more juice, 8 cans or 2 bottles? How much more?
__________

Answer: 2 bottles

Explanation:
Kelly plans to take the juice on her camping trip.
The capacity of the bottle is more than a bottle. Thus 2 bottles can hold more juice.

Question 17.
Erin’s water bottle holds 600 milliliters of water. Dylan’s water bottle holds 1 liter of water. Whose water bottle has the greater capacity? How much greater?
__________

Answer: Dylan

Explanation:
Erin’s water bottle holds 600 milliliters of water.
Dylan’s water bottle holds 1 liter of water.
First, convert from liter to milliliters
1 liter = 1000 milliliters
By this, we can say that Dylan’s water bottle has a greater capacity.

Question 18.
Liz and Alana each participated in the high jump at the track meet. Liz’s high jump was 1 meter. Alana’s high jump was 132 centimeters. Who jumped higher? How much higher?
Type below:
__________

Answer: Alana

Explanation:
Liz and Alana each participated in the high jump at the track meet.
Liz’s high jump was 1 meter.
Alana’s high jump was 132 centimeters.
Convert from centimeters to meter.
1 meter = 100 cm
132 cm = 132 ÷ 100 meter
132 cm = 1.32 m
1 m is less than 1.32 m
Alana jumped higher than Liz.

Question 19.
Are there less than 1 million, exactly 1 million, or greater than 1 million milligrams in 1 kilogram? Explain how you know.
__________ milligrams

Answer: Exactly 1 million

Explanation:
Convert 1 kilogram to milligrams.
multiply by 6 powers of 10, which equals one million.
Thus 1 kg = 1,000,000 mg
There are exactly 1 million milligrams in 1 kilogram.

Question 20.
Test Prep Monica has 426 millimeters of fabric. How many centimeters of fabric does Monica have?
Options:
a. 4,260 centimeters
b. 42.6 centimeters
c. 4.26 centimeters
d. 0.426 centimeters

Answer: 42.6 centimeters

Explanation:
Converting from millimeters to centimeters.
1 millimeter = 0.1 cm
426 mm = 426 × 0.1 = 42.6 cm
Monica has 42.6 cm of fabric.
Thus the correct answer is option B

Share and Show – Lesson 6: Problem Solving Customary and Metric Conversions – Page No. 619

Question 1.
Edgardo has a drink cooler that holds 10 gallons of water. He is filling the cooler with a 1-quart container. How many times will he have to fill the quart container to fill the cooler?
Go Math Grade 5 Answer Key Chapter 10 Convert Units of Measure Lesson 6: Problem Solving Customary and Metric Conversions img 12
First, make a table to show the relationship between gallons and quarts. You can use a conversion table to find how many quarts are in a gallon.
Go Math Grade 5 Answer Key Chapter 10 Convert Units of Measure Lesson 6: Problem Solving Customary and Metric Conversions img 13
Then, look for a rule to help you complete your table. number of gallons × ____ = number of quarts
Finally, use the table to solve the problem.
Edgardo will need to fill the quart container ____ times.
____ times

Answer:

gal 1 2 3 4 10
qt 4 8 12 16 40

Edgardo will need to fill the quart container 40 times.

Question 2.
What if Edgardo only uses 32 quarts of water to fill the cooler. How can you use your table to find how many gallons that is?
____ gallons

Answer: 8

Explanation:
Convert from quarts to gallons.
1 gallon = 4 quarts
1 quart = 1/4 gal
32 quart = 32 × 1/4 = 8 gallons

Question 3.
If Edgardo uses a 1-cup container to fill the cooler, how will that affect the number of times he has to fill a container to fill the cooler? Explain.
Type below:
__________

Answer: Multiply by 16

gal 1 2 3 4 10
cups 16 32 48 64 160

On Your Own – Lesson 6: Problem Solving Customary and Metric Conversions – Page No. 620

Question 4.
Jeremy made a belt that was 6.4 decimeters long. How many centimeters long is the belt Jeremy made?
____ cm

Answer: 64 cm

Explanation:
Jeremy made a belt that was 6.4 decimeters long.
Converting from decimeters to centimeters.
1 decimeter = 10 centimeter
6.4 decimeter = 6.4 × 10 cm = 64 cm
Jeremy made 64 cm long belt.

Question 5.
Dan owns 9 DVDs. His brother Mark has 3 more DVDs than Dan has. Their sister, Marsha, has more DVDs than either of her brothers. Together, the three have 35 DVDs. How many DVDs does Marsha have?
____ DVDs

Answer: 14 DVDs

Explanation:
Given that,
Dan owns 9 DVDs.
His brother Mark has 3 more DVDs than Dan has.
Their sister, Marsha, has more DVDs than either of her brothers.
Together, the three have 35 DVDs.
His brother Mark has 3 more DVDs than Dan has.
That means Mark has 9 + 3 = 12 DVDs
Total number of DVDs = 35 – 9 – 12 = 14 DVDs
Thus Marsha has 14 DVDs.

Question 6.
Kevin is making a picture frame. He has a piece of trim that is 4 feet long. How many 14-inch-long pieces can Kevin cut from the trim? How much of a foot will he have left over?
Type below:
__________

Answer: 1/2 ft

Explanation:
Kevin is making a picture frame. He has a piece of trim that is 4 feet long.
Converting from feet to inches
1 feet = 12 inches
x = 4 × 12 = 48 inches
Calculate how many 14 inch pieces. He can cut 3 (14 inch) pieces
y = 48 ÷ 14 = 3
48 – 42 = 6 inches
Feet leftover
6 in/12 in = 1/2 feet
Thus 1/2 foot will be left over.

Question 7.
Explain how you could find the number of cups in five gallons of water.
Type below:
__________

Answer:
There are 16 cups in a gallon.
To convert gallons to cups, multiply the gallon value by 16.
1 gal = 16 cups
5 gallons = 5 × 16 cups = 80 cups

Question 8.
Carla uses 2 \(\frac{3}{4}\) cups of flour and 1 \(\frac{3}{8}\) cups of sugar in her cookie recipe. How many cups does she use in all?
_____ \(\frac{□}{□}\) cups

Answer: 4 \(\frac{1}{8}\) cups

Explanation:
Given:
Carla uses 2 \(\frac{3}{4}\) cups of flour and 1 \(\frac{3}{8}\) cups of sugar in her cookie recipe.
Add 2 \(\frac{3}{4}\) and 1 \(\frac{3}{8}\)
2 \(\frac{3}{4}\) + 1 \(\frac{3}{8}\)
2 + 1 + \(\frac{3}{4}\) + \(\frac{3}{8}\)
3 + \(\frac{3}{4}\) + \(\frac{3}{8}\)
3 + \(\frac{9}{8}\)
3 + 1\(\frac{1}{8}\)
3 + 1 + \(\frac{1}{8}\)
4 \(\frac{1}{8}\) cups

Question 9.
Tony needs 16-inch-long pieces of gold chain to make each of 3 necklaces. He has a piece of chain that is 4 \(\frac{1}{2}\) feet long. How much chain will he have left after making the necklaces?
Options:
a. 6 inches
b. 12 inches
c. 18 inches
d. 24 inches

Answer: 6 inches

Explanation:
Given that,
Tony needs 16-inch-long pieces of gold chain to make each of 3 necklaces.
He has a piece of chain that is 4 \(\frac{1}{2}\) feet long.
Converting from feet to inches.
1 foot = 12 inches
4 feet = 12 × 4 = 48 inches
1/2 feet = 6 inches
48 + 6 = 54 inches
Tony needs 16-inch-long pieces of gold chain to make each of 3 necklaces.
16 × 3 = 48
54 inches – 48 inches = 6 inches
Thus the correct answer is option A.

Share and Show – Lesson 7: Elapsed Time – Page No. 625

Convert.

Question 1.
540 min = _____ hr

Answer: 9 hr

Explanation:
Convert from minutes to hours.
1 hour = 60 min
1 min = 1/60 hour
540 min = 540 × 1/60 hour = 9 hour
540 min = 9 hr

Question 2.
8 d = _____ hr

Answer: 192 hr

Explanation:
Convert from days to hours.
1 day = 24 hours
8 days = 8 × 24 hr = 192 hr
8 d = 192 hr

Question 3.
110 hr = _____ d _____ hr

Answer: 4 d 14 hr

Explanation:
Convert from hours to days.
110 hr = 96 hr + 14 hr
1 day = 24 hour
96 hours = 96/24 = 4 days
110 hour = 4 d 14 hr

Find the end time.

Question 4.
Start time: 9:17 A.M.
Elapsed time: 5 hr 18 min
Go Math Grade 5 Answer Key Chapter 10 Convert Units of Measure Lesson 7: Elapsed Time img 14
End time: _____ : _____ P.M.

Answer: 2:35 P.M.

Explanation:
You can use a number line or a clock to find the end time.
Add the hours to the start time.
10:17 A.M
11:17 A.M.
12:17 A.M.
1:17 A.M.
2:17 A.M.
Next, add the minutes.
x = 2:17 P.M. + 0:18 min = 2:35 P.M
Thus the end time is 2:35 P.M

On Your Own

Convert.

Question 5.
3 min = _____ sec

Answer: 180 sec

Explanation:
Convert from minutes to seconds.
1 min = 60 sec
3 min = 3 × 60 sec = 180 sec
3 min = 180 sec

Question 6.
240 min = _____ hr

Answer: 4 hr

Explanation:
Convert from minutes to hours.
1 hour = 60 min
1 min = 1/60 hr
240 min = 240 × 1/60 = 4 hour
240 min = 4 hour

Question 7.
1 hr = _____ sec

Answer: 3600 sec

Explanation:
1 hour = 60 min
1 min = 60 sec
60 min = 60 × 60 sec = 3600 sec
1 hour = 3600 sec

Question 8.
3 yr = _____ d

Answer: 1095 d

Explanation:
Convert from years to days.
1 year = 365 days
3 years = 3 × 365 = 1095 days
So, 3 yr = 1095 d

Question 9.
208 wk = _____ yr

Answer: 4 yr

Explanation:
Convert weeks to years.
1 year = 52 weeks
1 week = 1/52 yr
208 wk = 208 × 1/52 yr = 4 yr
208 wk = 4 yr

Question 10.
350 min = _____ hr _____ min

Answer: 5 hr 50 min

Explanation:
Convert from minutes to hours.
60 min = 1 hour
1 min = 1/60 hour
Add hours
60 min =1 hr
120 min = 2 hr
180 min = 3 hr
240 min = 4 hr
300 min = 5 hr
360 min = 6 hr
350 min = 300 min + 50 min
350 min = 5 hr 50 min

Find the start, elapsed, or end time.

Question 11.
Start time: 11:38 A.M.
Elapsed time: 3 hr 10 min
End time: _____ : _____ P.M.

Answer: 2:48 P.M.

Explanation:
You can use a number line or a clock to find the end time.
Add the hours to the start time.
12:38 pm
1:38 pm
2:38 pm
Add the minutes next.
x = 2:38 pm + 10 min = 2:48 pm
Thus the end time is 2:48 P.M.

Question 12.
Elapsed time: 2 hr 37 min
End time: 1:15 P.M.
Start time: _____ : _____ A.M.

Answer: 10:38 A.M.

Explanation:
You can use a number line or a clock to find the end time.
x = 0:15 min – 0:37 min
x = -0:22 min
y = 60 min – 22 min = 38 min
time = 12:38 pm
Next subtract the hours from the time.
11:38 am
10:38 am
Thus the start time is 10:38 A.M.

Question 13.
Elapsed time: 2 \(\frac{1}{4}\) hr
End time: 5:30 P.M.
Start time: _____ : _____ P.M.

Answer: 3:15 P.M.

Explanation:
You can use a number line or a clock to find the end time.
x = 30 min – 15 min = 15 min
time 5:30
subtract the hours
5:30 pm
4:30 pm
3:30 pm
y = 3:30 pm – 15 min = 3:15 P.M
Thus the start time is 3:15 P.M.

Question 14.
Start time: 7:41 P.M.
End time: 8:50 P.M.
Elapsed time: _____ hr _____ min

Answer: 1 hr 9 min

Explanation:
You can use a number line or a clock to find the end time.
x = 50 min – 41 min = 9 min
subtract the hours
8 – 7 = 1 hour
1 hour 9 min
Elapsed time: 1 hour 9 min

Problem Solving – Lesson 7: Elapsed Time – Page No. 626

For 15–17, use the graph.
Go Math Grade 5 Answer Key Chapter 10 Convert Units of Measure Lesson 7: Elapsed Time img 15

Question 15.
Which Internet services downloaded the podcast in less than 4 minutes?
_________
_________

Answer:
Groove Box
Internet -C

Explanation:
From the above figure, we can observe that the Internet services downloaded the podcast in less than 4 minutes is Groove Box and Internet -C
Groove box took 173 sec and Internet-C took 196 seconds.
Convert from minutes to seconds
1 min = 60 sec
1 sec= 1/60 min
173 sec = 2 min 53 sec
196 sec = 180 sec + 16 sec = 3 min 16 sec

Question 16.
Which service took the longest to download the podcast? How much longer did it take than Red Fox in minutes and seconds?
Type below:
_________

Answer: Top Hat

Explanation:
The figure shows that Top Hat took the longest time to download the podcast.
It took 1050 sec to download the podcast.
Convert from minutes to seconds
1 min = 60 sec
1 sec= 1/60 min
1050 sec = 1020 sec 30 sec
1020 sec = 1020 × 1/60 min = 17 min
1050 sec = 17 min 30 sec.
To find how much time it took than Red Fox, we need to subtract the time from red fox and top hat.
1050 sec – 310 sec = 740 sec
Convert from seconds to minutes.
1 min = 60 sec
1 sec = 1/60 min
740 sec = 720 + 20 sec
720 sec = 720 × 1/60 = 12 min
740 sec = 12 min 20 sec

Question 17.
Which service was faster, Red Fox or Internet-C? How much faster in minutes and seconds?
Type below:
_________

Answer: Internet-C

Explanation:
From the above figure, we can see that Internet-C is faster than Red Fox.
Internet-C took 196 sec
Red Fox took 310 sec
310 sec – 196 sec = 114 sec
Convert from seconds to minutes.
1 min = 60 sec
1 sec = 1/60 min
114 sec = 60 sec + 54 sec
114 sec = 1 min 54 sec

Question 18.
Explain how you could find the number of seconds in a full 24-hour day. Then solve.
Type below:
_________

Answer:
Convert from hours to minutes
1 hour = 60 min
24 hour = 24 × 60 min = 1440 min
Now convert from minutes to seconds.
1 min = 60 sec
1440 min = 1440 × 60 = 86400 sec
Thus a day has 86400 seconds.

Question 19.
Test Prep Samit and his friends went to a movie at 7:30 P.M. The movie ended at 9:55 P.M. How long was the movie?
Options:
a. 2 hours 25 minutes
b. 2 hours 5 minutes
c. 1 hour 25 minutes
d. 1 hour 5 minutes

Answer: 2 hours 25 minutes

Explanation:
Samit and his friends went to a movie at 7:30 P.M. The movie ended at 9:55 P.M.
Subtract the starting time and ending time of the movie.
9 hour 55 min
-7 hour 30 min
2 hour 25 min
Therefore the movie is 2hr 25 min long.
Thus the correct answer is option A.

Chapter Review/Test – Vocabulary – Page No. 4350

Choose the best term from the box.

Question 1.
A metric unit of mass that is equal to \(\frac{1}{1,000}\) of a gram is called a ________.
___________

Answer: Milligram
A metric unit of mass that is equal to \(\frac{1}{1,000}\) of a gram is called a Milligram.

Question 2.
A metric unit for measuring length that is equal to 10 meters is called a _________.
___________

Answer: Dekameter
A metric unit for measuring length that is equal to 10 meters is called a Dekameter.

Concepts and Skills

Convert.

Question 3.
96 oz = ______ lb

Answer: 6 lb

Explanation:
Convert from ounces to pounds.
1 pound = 16 ounces
1 ounce = 1/16 pound
96 oz = 96 × 1/16 lb = 6 lb
96 oz = 16 lb

Question 4.
5 kg = ______ g

Answer: 5000 g

Explanation:
Convert from kg to grams.
1 kg = 1000 g
5 kg = 5 × 1000 g = 5000 g
5 kg = 5000 g
Thus 5 kg = 5000 grams

Question 5.
500 min = ______ hr ______ min

Answer: 8 hr 20 min

Explanation:
Convert from minutes to hours.
1 hour = 60 min
1 min = 1/60 hour
500 min = 500 × 1/60
500 min = 480 min + 20 min
That means 480 × 1/60 + 20 min
= 8 hour 20 min
500 min = 8 hour 20 min

Question 6.
65 yd 2 feet = ______ ft

Answer: 197 ft

Explanation:
65 yd 2 feet
Convert from yard to feet.
1 yard = 3 feet
65 yard = 65 × 3 feet = 195 feet + 2 feet = 197 feet
65 yd 2 feet = 197 feet

Compare. Write <, >, or =.

Question 7.
7 wk ______ 52 d

Answer: 7 wk < 52 d

Explanation:
First, convert from weeks to days.
1 week = 7 days
7 weeks = 7 × 7 = 49 days
49 is less than 52 days
Thus 7 wk < 52 d

Question 8.
4 L ______ 3,000 mL

Answer: 4 L > 3,000 mL

Explanation:
Convert from liters to milliliters.
1 L = 1000 mL
4 L = 4 × 1000 mL = 4000 mL
4000 mL is greater than 3000 mL
Thus, 4 L > 3,000 mL

Question 9.
72 in. ______ 2 yd

Answer: 72 in. = 2 yd

Explanation:
Convert from inches to yards.
1 yard = 3 feet
1 feet = 12 inches
3 feet = 3 × 12 in. = 36 in.
2 yards = 2 × 36 in. = 72 in.
Thus, 72 in. = 2 yd

Solve.

Question 10.
A girl walks 5,000 meters in one hour. If the girl walks at the same speed for 4 hours, how many kilometers will she have walked?
______ km

Answer: 20 km

Explanation:
A girl walks 5,000 meters in one hour.
Convert from meters to kilometers.
1000 m = 1 km
5000 m = 5 km
If the girl walks at the same speed for 4 hours,
Then multiply 5 km × 4 = 20 km
Therefore, she will walk 20 km for 4 hours.

Chapter Review/Test – Page No. 4360

Fill in the bubble completely to show your answer.

Question 11.
Howard cuts 54 centimeters off a 1-meter board. How much of the board does Howard have left?
Options:
a. 53 centimeters
b. 53 meters
c. 46 meters
d. 46 centimeters

Answer: 46 centimeters

Explanation:
Given that,
Howard cuts 54 centimeters off a 1-meter board.
We know that,
1 meter = 100 cm
100 cm – 54 cm = 46 cm
Therefore, 46 centimeters of the board is left.
Thus the correct answer is option D.

Question 12.
Joe’s dog has a mass of 28,000 grams. What is the mass of Joe’s dog in kilograms?
Options:
a. 2,800 kilograms
b. 280 kilograms
c. 28 kilograms
d. 2.8 kilograms

Answer: 28 kilograms

Explanation:
Joe’s dog has a mass of 28,000 grams.
Convert from grams into the kilograms
1 kg = 1000 g
1 g = 1/1000 kg
28000 g = 28000 × 1/1000 = 28 kg
The mass of Joe’s dog is 28 kg.
Thus the correct answer is option C.

Question 13.
Cathy drank 600 milliliters of water at school and another 400 milliliters at home. How many liters of water did Cathy drink?
Options:
a. 1,000 liters
b. 100 liters
c. 10 liters
d. 1 liter

Answer: 1 liter

Explanation:
Cathy drank 600 milliliters of water at school and another 400 milliliters at home.
600 milliliters + 400 milliliters = 1000 milliliters
We know that,
1 litre = 1000 milliliters
Therefore Cathy drink 1 liter of water.
Thus the correct answer is option D.

Question 14.
Mr. Banks left work at 5:15 P.M. It took him 1 \(\frac{1}{4}\) hours to drive home. At what time did Mr. Banks arrive home?
Options:
a. 6:15 P.M.
b. 6:30 P.M.
c. 6:45 P.M.
d. 7:30 P.M.

Answer: 6:30 P.M.

Explanation:
Mr. Banks left work at 5:15 P.M. It took him 1 \(\frac{1}{4}\) hours to drive home.
1 \(\frac{1}{4}\) hours = 1:15 hour
Add 5:15 P.M. with 1:15 hour
5 hour 15 mins
1 hour 15 mins
6 hour 30 mins
Therefore, Mr. Banks arrives home at 6:30 P.M.
Thus the correct answer is option B.

Chapter Review/Test – Page No. 4370

Fill in the bubble completely to show your answer.

Question 15.
A turtle walks 12 feet in one hour. How many inches does the turtle walk in one hour?
Options:
a. 12 inches
b. 24 inches
c. 124 inches
d. 144 inches

Answer: 144 inches

Explanation:
Given that, A turtle walks 12 feet in one hour.
Convert from 1 foot to inches.
1 foot = 12 inches
12 feet = 12 × 12 inches = 144 inches
The turtle walks 144 inches in an hour.
Thus the correct answer is option D.

Question 16.
Jason and Doug competed in the long jump at a track meet. Jason’s long jump was 98 inches. Doug’s long jump was 3 yards. How much longer was Doug’s jump than Jason’s jump?
Options:
a. 1 inch
b. 10 inches
c. 12 inches
d. 20 inches

Answer: 10 inches

Explanation:
Jason and Doug competed in the long jump at a track meet.
Jason’s long jump was 98 inches.
Doug’s long jump was 3 yards.
1 yard = 3 feet
3 yards = 9 feet
9 feet = 9 × 12 = 108 inches
108 inches – 98 inches = 10 inches
Doug’s jump 10 inches longer than Jason’s jump.
The correct answer is option B.

Question 17.
Sarita used 54 ounces of apples to make an apple pie. How many pounds and ounces of apples did Sarita use?
Options:
a. 2 pounds 6 ounces
b. 3 pounds 6 ounces
c. 4 pounds 6 ounces
d. 8 pounds 6 ounces

Answer: 3 pounds 6 ounces

Explanation:
Sarita used 54 ounces of apples to make an apple pie.
Converting from ounces to pounds
We know that,
1 pound = 16 ounces
1 ounce = 1/16 pound
48 ounce + 6 ounce = 56 ounces
48 ounces = 48 × 1/16 pound = 3 pound
3 pound 6 ounces
Sarita uses 3 pounds 6 ounces of apples.
Therefore the correct answer is option B.

Question 18.
Morgan measures the capacity of a juice glass to be 12 fluid ounces. If she uses the glass to drink 4 glasses of water throughout the day, how many pints of water does Morgan drink?
Options:
a. 3 pints
b. 6 pints
c. 24 pints
d. 48 pints

Answer: 3 pints

Explanation:
3 pints because 12× 4 is 48 and 48 divided by 8 is 6 so then there are 6 cups.
1 pint = 2 cups
So, 6 cups = 3 × 2 pints
6 cups make 3 pints.
Thus Morgan drinks 3 pints of water.
Thus the correct answer is option A.

Chapter Review/Test – Page No. 4380

Constructed Response

Question 19.
Louisa needs 3 liters of lemonade and punch for a picnic. She has 1,800 milliliters of lemonade. How much punch does she need? Explain how you found your answer.
______ mL

Answer: 1200 mL

Explanation:
Given:
Louisa needs 3 liters of lemonade and punch for a picnic.
She has 1,800 milliliters of lemonade.
Convert from liters to milliliters
1 L = 1000 milliliters
3 L = 3 × 1000 milliliters = 3000 mL
3000 mL – 1800 mL = 1200 mL
Therefore, she need 1200 mL punch.

Question 20.
Maddie bought 10 quarts of ice cream. How many gallons and quarts of ice cream did Maddie buy? Explain how you found your answer.
______ gallons ______ quarts of ice cream

Answer: 2 gal 2 quarts of ice cream

Explanation:
Maddie bought 10 quarts of ice cream.
Convert quarts to gallons.
1 gal = 4 quarts
2 gal = 2 × 4 quarts = 8 quarts
2 gal 2 quart
Thus, Maddie buys 2 gal 2 quart of ice cream.

Performance Task

Question 21.
The Drama Club is showing a video of their recent play. The first showing began at 2:30 P.M. The second showing was scheduled to start at 5:25 P.M. with a \(\frac{1}{2}\)-hour break between the showings.
A). How long is the video in hours and minutes?
______ hours and ______ minutes

Answer: 2 hour 25 minutes

Explanation:
The Drama Club is showing a video of their recent play.
The first showing began at 2:30 P.M.
The second showing was scheduled to start at 5:25 P.M. with a \(\frac{1}{2}\)-hour break between the showings.
5 hour 25 minutes
2 hour 30 minutes

4 hour 85 minutes
2 hour 30 minutes
2 hour 55 minutes
\(\frac{1}{2}\)-hour break
2 hour 55 minutes
– 0 hour 30 minutes
2 hour 25 minutes

Question 21.
B). Explain how you can use a number line to find the answer.
Type below:
_________

Answer:

Question 21.
C). The second showing started 20 minutes late. Will the second showing be over by 7:45 P.M.? Explain why your answer is reasonable.
______

Answer: No

Explanation:
If the show starts 20 minutes late that means at 5:45 P.M then it will not end at 7:45 P.M.
5:45 P.M + 2:25 = 8:15 P.M.
So, the answer is no.

Conclusion

I wish the knowledge shared in this article on Go Math Grade 5 Chapter 10 Answer Key Convert Units of Measure has helped you a lot. Assess your preparation level by solving problems from Mid Chapter Checkpoint and Review Test. Refer to the Detailed Solutions Provided and understand where you are lagging.

Go Math Grade 7 Answer Key Chapter 7 Writing and Solving One-Step Inequalities

go-math-grade-7-chapter-7-writing-and-solving-one-step-inequalities-answer-key

Quick and easy learning is possible with Go Math Grade 7 Answer Key Chapter 7 Writing and Solving One-Step Inequalities. The answers in Go Math Grade 7 are prepared by the concerned subject experts. Go Math Answer Key helps the students of 7th grade to prove their best in the exams. So, Download Go Math Grade 7 Answer Key Chapter 7 Writing and Solving One-Step Inequalities and kickstart your preparation.

Go Math Grade 7 Answer Key Chapter 7 Writing and Solving One-Step Inequalities

It is important to gain knowledge along with the marks. Go Math Answer Key team’s main aim is to provide quality education for students of all grades. You can learn the basics of Writing and Solving One-Step Inequalities in Go Math Grade 7 Chapter 7 Answer key. Check out the topics before you start practicing. Only practice will help you to score the best marks in the exams. Refer to our Go Math Grade 7 Answer Key Chapter 7 Writing and Solving One-Step Inequalities while doing your homework and also during exam preparation. You can know how to draw the number line with the help of Go Math 7th Grade Chapter 7 Writing and Solving One-Step Inequalities.

Chapter 7 – Lesson 1:

Chapter 7 – Lesson: 2

Chapter 7 – Lesson: 3

Chapter 7 – Writing and Solving One-Step Inequalities Lesson: 4

Chapter 7 – Lesson: 5

Chapter 7 – Performance Tasks

Guided Practice – Page No. 208

Write the resulting inequality.

Question 1.
−5 ≤ −2; Add 7 to both sides
Type below:
___________

Answer: 2 ≤ 5

Explanation:
Add 7 to both sides of the inequality.
-5 + 7 ≤ -2 + 7
2 ≤ 5

Question 2.
−6 < −3; Divide both sides by -3
Type below:
___________

Answer: 2 > 1

Explanation:
Divide both sides by -3. switch the inequality sign since you are dividing by a negative number.
-6/-3 > -3/-3
2 > 1

Question 3.
7 > −4; Subtract 7 from both sides
Type below:
___________

Answer: 0 > -11

Explanation:
7 – 7 > -4 – 7
Subtract 7 from both sides
0 > -11

Question 4.
−1 ≥ −8; Multiply both sides by -2
Type below:
___________

Answer: 2 ≤ 16

Explanation:
Multiply both sides by -2 switch the inequality sign since you are multiplying by a negative number.
-1(-2) ≤ -8(-2)
2 ≤ 16

Solve each inequality. Graph and check the solution.

Question 5.
n−5 ≥ −2
Type below:
___________

Answer:
To graph inequalities, locate the number opposite the variable of the inequality on a number line. If the inequality is either a ≤ or a ≥, we use a closed dot, meaning the number is a solution as well. If the inequality is either a > or a <, use an open dot, indicating that the number is not a solution.
From here, shade the line going to the left if the inequality is either ≤ or < and shade the line going to the right if the inequality is either ≥ or >.
Solve the inequality first:
n – 5 ≥ -2
n – 5 + 5 ≥ -2 + 5
n ≥ 3
The number opposite the variable is 3, we look for this in the number line. Since the inequality is ≥, we use a closed dot and shade the line going to the right. Its graph would look like the one below:

Question 6.
3 + x < 7
Type below:
___________

Answer:
To graph inequalities, locate the number opposite the variable of the inequality on a number line. If the inequality is either a ≤ or a ≥, we use a closed dot, meaning the number is a solution as well. If the inequality is either a > or a <, use an open dot, indicating that the number is not a solution.
From here, shade the line going to the left if the inequality is either ≤ or < and shade the line going to the right if the inequality is either ≥ or >.
Solve the inequality first:
3 + x < 7
3 – 3 + x < 7 – 3
x < 4
The number opposite the variable is 4, we look for this in the number line. Since the inequality is <, we use a hollow dot and shade the line going to the left. Its graph would look like the one below:

Question 7.
−7y ≤ 14
Type below:
___________

Answer:
To graph inequalities, locate the number opposite the variable of the inequality on a number line. If the inequality is either a ≤ or a ≥, we use a closed dot, meaning the number is a solution as well. If the inequality is either a > or a <, use an open dot, indicating that the number is not a solution.
From here, shade the line going to the left if the inequality is either ≤ or < and shade the line going to the right if the inequality is either ≥ or >.
Solve the inequality first:
−7y ≤ 14
-7y/-7 ≤ 14/-7
y ≥ -2
The number opposite the variable is -2, we look for this in the number line. Since the inequality is ≥, we use a closed dot and shade the line going to the right. Its graph would look like the one below:

Question 8.
\(\frac{b}{5}\) > −1
Type below:
___________

Answer:
To graph inequalities, locate the number opposite the variable of the inequality on a number line. If the inequality is either a ≤ or a ≥, we use a closed dot, meaning the number is a solution as well. If the inequality is either a > or a <, use an open dot, indicating that the number is not a solution.
From here, shade the line going to the left if the inequality is either ≤ or < and shade the line going to the right if the inequality is either ≥ or >.
Solve the inequality first:
\(\frac{b}{5}\) > −1
Multiply 5 on both sides.
(5)\(\frac{b}{5}\) > −1(5)
b > -5
The number opposite the variable is -5, we look for this in the number line. Since the inequality is >, we use a hollow dot and shade the line going to the right. Its graph would look like the one below:

Question 9.
For a scientific experiment, a physicist must make sure that the temperature of a metal at 0 °C gets no colder than -80 °C. The physicist changes the metal’s temperature at a steady rate of -4 °C per hour. For how long can the physicist change the temperature?
a. Let t represent the temperature in degrees Celsius. Write an inequality. Use the fact that the rate of change in temperature times the number of hours equals the final temperature.
Type below:
___________

Answer:
We need to use the fact that the final temperature is equal to the rate of change in temperature times the number of hours.
We are given that the rate of change is -4°C per hour so the final temperature is -4 times the number of hours.
Let t represent the number of hours. The final temperature is then -4t degrees Celsius after t hours.
If the temperature must be no colder than -80°C, then the final temperature must be greater than or equal to -80.
The inequality is then -4t ≥ -80.

Question 9.
b. Solve the inequality in part a. How long can the physicist change the temperature of the metal?
Type below:
___________

Answer:
To solve the inequality for t, we need to divide both sides by -4. Remember to switch the inequality symbol since you are dividing by a negative number.
Dividing both sides by -4 then gives:
-4t/-4 ≤ -80/-4
t ≤ 20
The number of hours that the physicist can change the temperature of the metal is then at most 20 hours.

Question 9.
c. The physicist has to repeat the experiment if the metal gets cooler than -80 °C. How many hours would the physicist have to cool the metal for this to happen?
Type below:
___________

Answer:
From part (b), we know that the physicist can change the temperature for at most 20 hours to keep the temperature no colder than -80°C. This means the temperature will reach a temperature cooler than -80°C if he cools the metal for more than 20 hours.

Essential Question Check-In

Question 10.
Suppose you are solving an inequality. Under what circumstances do you reverse the inequality symbol?
Type below:
___________

Answer: You must reverse the inequality sign any time you multiply or divide both sides of the inequality by a negative number.

Page No. 209

In 11–16, solve each inequality. Graph and check the solution.

Question 11.
x − 35 > 15
Type below:
___________

Answer:
To graph inequalities, locate the number opposite the variable of the inequality on a number line. If the inequality is either a ≤ or a ≥, we use a closed dot, meaning the number is a solution as well. If the inequality is either a > or a <, use an open dot, indicating that the number is not a solution.
From here, shade the line going to the left if the inequality is either ≤ or < and shade the line going to the right if the inequality is either ≥ or >.
First, solve the inequality:
x − 35 > 15
Add 35 on both sides
x – 35 + 35 > 15 + 35
x > 50
The number opposite the variable is 50, we look for this in the number line. Since the inequality is >, we use a hollow dot and shade the line going to the right. Its graph would like the one below:

Question 12.
193 + y ≥ 201
Type below:
___________

Answer:
To graph inequalities, locate the number opposite the variable of the inequality on a number line. If the inequality is either a ≤ or a ≥, we use a closed dot, meaning the number is a solution as well. If the inequality is either a > or a <, use an open dot, indicating that the number is not a solution.
From here, shade the line going to the left if the inequality is either ≤ or < and shade the line going to the right if the inequality is either ≥ or >.
First, solve the inequality:
193 + y ≥ 201
193 + y – 193 ≥ 201 – 193
y ≥ 8
The number opposite the variable is 8, we look for this in the number line. Since the inequality is ≥, we use a closed dot and shade the line going to the right. Its graph would like the one below:

Question 13.
−\(\frac{q}{7}\) ≥ −1
Type below:
___________

Answer:
To graph inequalities, locate the number opposite the variable of the inequality on a number line. If the inequality is either a ≤ or a ≥, we use a closed dot, meaning the number is a solution as well. If the inequality is either a > or a <, use an open dot, indicating that the number is not a solution.
From here, shade the line going to the left if the inequality is either ≤ or < and shade the line going to the right if the inequality is either ≥ or >.
First, solve the inequality:
−\(\frac{q}{7}\) ≥ −1
Multiply both sides by -7
(-7)−\(\frac{q}{7}\) ≥ −1(-7)
q ≤ 7
The number opposite the variable is 7, we look for this in the number line. Since the inequality is ≤, we use a closed dot and shade the line going to the left. Its graph would like the one below:

Question 14.
−12x < 60
Type below:
___________

Answer:
To graph inequalities, locate the number opposite the variable of the inequality on a number line. If the inequality is either a ≤ or a ≥, we use a closed dot, meaning the number is a solution as well. If the inequality is either a > or a <, use an open dot, indicating that the number is not a solution.
From here, shade the line going to the left if the inequality is either ≤ or < and shade the line going to the right if the inequality is either ≥ or >.
First, solve the inequality:
−12x < 60
Divide both sides by -12
-12x/12 < 60/-12
x > -5
The number opposite the variable is -5, we look for this in the number line. Since the inequality is >, we use a hollow dot and shade the line going to the right. Its graph would like the one below:

Question 15.
5 > z − 3
Type below:
___________

Answer:
To graph inequalities, locate the number opposite the variable of the inequality on a number line. If the inequality is either a ≤ or a ≥, we use a closed dot, meaning the number is a solution as well. If the inequality is either a > or a <, use an open dot, indicating that the number is not a solution.
From here, shade the line going to the left if the inequality is either ≤ or < and shade the line going to the right if the inequality is either ≥ or >.
First, solve the inequality:
5 > z − 3
Add both sides by 3
5 + 3 > z – 3 + 3
8 > z
z < 8
The number opposite the variable is 8, we look for this in the number line. Since the inequality is <, we use a hollow dot and shade the line going to the left. Its graph would like the one below:

Question 16.
0.5 ≤ \(\frac{y}{8}\)
Type below:
___________

Answer:
To graph inequalities, locate the number opposite the variable of the inequality on a number line. If the inequality is either a ≤ or a ≥, we use a closed dot, meaning the number is a solution as well. If the inequality is either a > or a <, use an open dot, indicating that the number is not a solution.
From here, shade the line going to the left if the inequality is either ≤ or < and shade the line going to the right if the inequality is either ≥ or >.
First, solve the inequality:
0.5 ≤ \(\frac{y}{8}\)
Multiply both sides by 8
(8)0.5 ≤ \(\frac{y}{8}\)(8)
4 ≤ y
y ≥ 4
The number opposite the variable is 4, we look for this in the number line. Since the inequality is ≥, we use a closed dot and shade the line going to the right. Its graph would like the one below:

Question 17.
The vet says that Lena’s puppy will grow to be at most 28 inches tall. Lena’s puppy is currently 1 foot tall. How many more inches will the puppy grow?
Type below:
___________

Answer: not more than 16 inches

Explanation:
Let x be the additional inches the puppy can grow remember 1 foot is 12 inches so the height of the puppy is 12 + x
12 + x ≤ 28
x ≤ 16

Question 18.
In a litter of 7 kittens, each kitten weighs less than 3.5 ounces. Find all the possible values of the combined weights of the kittens.
Type below:
___________

Answer:
All of the kittens must weigh more than 0 ounces so the smallest combined weight is more than 0 ounces. Since there are 7 kittens, each kitten weights less than 3.5 ounces, and 7 × 3.5 = 24.5, then the combined weights of the kittens must be less than 24.5 ounces.
This gives the inequality 0 < w < 24.5
where w is the combined weight of the kittens in ounces.

Question 19.
Geometry
The sides of the hexagon shown are equal in length. The perimeter of the hexagon is at most 42 inches. Find the possible side lengths of the hexagon.
Go Math Grade 7 Answer Key Chapter 7 Writing and Solving One-Step Inequalities img 1
Type below:
___________

Answer: 0 < s ≤ 7

Explanation:
Let s be the side lengths of the hexagon since its sides are all equal in length.
The side lengths of the hexagon must be greater than 0 since lengths can’t be negative or 0 so s > 0.
The perimeter of the figure is the sum of its side lengths so the perimeter of the hexagon must be 6s since it has 6 sides that are all s inches long.
The perimeter is at most 42 inches so 6s ≤ 42.
Dividing both sides by 6 then gives s ≤ 7.
Combining the inequalities s > 0 and s ≤ 7 then gives possible side lengths of 0 < s ≤ 7.

Question 20.
To get a free meal at his favorite restaurant, Tom needs to spend $50 or more at the restaurant. He has already spent $30.25. How much more does Tom need to spend to get his free meal?
Type below:
___________

Answer: at least $ 19.75

Explanation:
Let x be the additional amount he needs to spend. subtract 30.25 on both sides to solve for x.
x + 30.25 ≥ 50
x ≥ 19.75

Question 21.
To cover a rectangular region of her yard, Penny needs at least 170.5 square feet of sod. The length of the region is 15.5 feet. What are the possible widths of the region?
Type below:
___________

Answer: at least 11 feet

Explanation:
Area is the length times width so let w be the width.
Divide both sides by 15.5 to solve for w.
15.5w ≥ 170.5
w ≥ 11

Question 22.
Draw Conclusions
A submarine descends from sea level to the entrance of an underwater cave. The elevation of the entrance is -120 feet. The rate of change in the submarine’s elevation is no greater than -12 feet per second. Can the submarine reach the entrance to the cave in less than 10 seconds? Explain.
Type below:
___________

Answer:
No. Since the rate of descent is less than -12 feet per second and the submarine is descending for less than 10 seconds, the submarine elevation will still be greater than -120. The submarine would have to descend at a rate greater than -12 feet per second to reach the entrance in less than 10 seconds or descend for more than 10 seconds at a rate less than -12 feet per second to reach the entrance.

Page No. 210

The sign shows some prices at a produce stand.
Go Math Grade 7 Answer Key Chapter 7 Writing and Solving One-Step Inequalities img 2

Question 23.
Selena has $10. What is the greatest amount of spinach she can buy?
Type below:
___________

Answer: 3 \(\frac{1}{3}\) pounds

Explanation:
Let x be the number of pounds of spinach. divide both sides by 3 to solve for x.
3x ≤ 10
x ≤ \(\frac{10}{3}\)
x ≤ 3 \(\frac{1}{3}\) pounds

Question 24.
Gary has enough money to buy at most 5.5 pounds of potatoes. How much money does Gary have?
Type below:
___________

Answer: $2.75

Explanation:
Let x be the amount of money he has
Multiply the price per pound of potatoes by the number of pounds.
5.5(0.50) ≤ x
2.75 ≤ x

Question 25.
Florence wants to spend no more than $3 on onions. Will she be able to buy 2.5 pounds of onions? Explain.
Type below:
___________

Answer:
Since each pound of onions costs $1.25, then 2.5 pounds of onions cost $1.25 × 2.5 ≈ 3.13.
Since $3.13 is greater than $3, she will not have enough money if she wants to spend no more than $3.

H.O.T.

Focus on Higher Order Thinking

Question 26.
Counterexamples
John says that if one side of an inequality is 0, you don’t have to reverse the inequality symbol when you multiply or divide both sides by a negative number. Find an inequality that you can use to disprove John’s statement. Explain your thinking.
Type below:
___________

Answer:
A possible counterexample is -2x ≤ 0. Solving this correctly gives x ≥ 0 which means the inequality is true for all non-negative values. If you don’t switch the inequality sign you would get x ≤ 0 which means the inequality would be true for all non-positive numbers
x = -3 is a possible value for x ≤ 0 but -2x = -2(-3) = 6 which is not less than or equal to 0.

Question 27.
Look for a Pattern
Solve x + 1 > 10, x + 11 > 20, and x + 21 > 30. Describe a pattern. Then use the pattern to predict the solution of x + 9,991 > 10,000.
Type below:
___________

Answer:
x + 1 > 10
x > 9
Subtract both sides by 11
x + 11 > 20
x > 9
Subtract both sides by 21.
x + 21 > 30
x > 9
The pattern is that when the number on the left side of the inequality is 9 less than the number on the right side of the inequality, the answer is x > 9.
Since 9991 is 9 less than 10000, x + 9991 > 10,000 ha the solution x > 9

Question 28.
Persevere in Problem Solving
The base of a rectangular prism has a length of 13 inches and a width of \(\frac{1}{2}\) inch. The volume of the prism is less than 65 cubic inches. Find all possible heights of the prism. Show your work.
Type below:
___________

Answer: 0 < h < 10

Explanation:
Let h be the height of the prism. It is given that the prism has a length of 13 inches and a width of 1/2 inches.
Using the formula v = lbh
13(1/2) h< 65
Multiply 2/13 on both sides
2/13 (13/2)h< 2/13 × 65
h < 2 × 5
h < 10
Since the height must be a positive number, then h > 0. Combining h > 0 and h < 10 then gives the final answer of 0 < h < 10.

Guided Practice – Page No. 214

Draw algebra tiles to model each two-step inequality.

Question 1.
4x − 5 < 7
Type below:
___________

Answer:
On the left side, draw 4 positive rectangles to model 4x and 5 negative squares to represent -5. On the right side, draw 7 positive squares to represent 7. then draw < in the middle.
Go Math Grade 7 Chapter 6 answer key solution img-3

Question 2.
−3x + 6 > 9
Type below:
___________

Answer:
On the left side, draw 3 negative rectangles to model -3x and 6 positive squares to represent 6. On the right side, draw 9 positive squares to represent 9. then draw > in the middle.
Go Math Grade 7 Chapter 6 answer key solution img-4

Question 3.
The booster club needs to raise at least $7,000 for new football uniforms. So far, they have raised $1,250. Write an inequality to find the average amounts each of the 92 members can raise to meet the club’s objective.
Type below:
___________

Answer: 1250 + 92a ≥ 7000

Explanation:
The amount to be raised is $7000. The amount already raised is $1250. The number of members is 92.
The inequality is then of the form: amount already raised + number of members × amount each member raises ≥ target amount.
The inequality is then:
1250 + 92a ≥ 7000

Question 4.
Analyze what each part of 7x − 18 ≤ 32 means mathematically.
Type below:
___________

Answer:
x is the variable so it is the solution. 7x is the solution multiplied by 7. -18 means 7x is subtracted by 18. ≤ 32 means the result is no more than 32.

Question 5.
Write a real-world problem to represent 7x − 18 ≤ 32.
Type below:
___________

Answer:
A real-world problem could be: The temperature of a metal is currently at -18°C. A scientist will warm the metal at a rate of 7°C per hour until the temperature is 32°C. How many hours will it take to warm up the metal?

Essential Question Check-In

Question 6.
Describe the steps you would follow to write a two-step inequality you can use to solve a real-world problem.
Type below:
___________

Answer:
The first step is to translate the words into an algebraic expression.
The next step is to determine the target amount.
The third step is to determine what inequality sign to use by determining if you need to be greater than, greater than or equal to, less than, less than, or equal to the target amount to write the inequality. Then solve the inequality sign, and target amount to write the inequality.
Then solve the inequality for the unknown value. Finally, interpret the solution in the context of the problem.

Independent Practice – Page No. 215

Question 7.
Three friends earned more than $200 washing cars. They paid their parents $28 for supplies and divided the rest of the money equally. Write an inequality to find possible amounts each friend earned. Identify what your variable represents.
Type below:
___________

Answer: 3x + 28 > 200

Explanation:
Let x be the amount each friend received. Since there are 3 friends, then 3x is the amount of money they split evenly.
The amount of money they split evenly was the amount left over after paying their parent’s $28.
Therefore 3x + 28 is the total amount they earned.
3x + 28 > 200

Question 8.
Nick has $7.00. Bagels cost $0.75 each, and a small container of cream cheese costs $1.29. Write an inequality to find the numbers of bagels Nick can buy. Identify what your variable represents.
Type below:
___________

Answer: 0.75x + 1.29 ≤ 7

Explanation:
Let x represent the number of bagels he can buy.
Then 0.75x is the total cost of the bagels and 0.75x + 1.29 is the total cost of his purchase.

Question 9.
Chet needs to buy 4 work shirts, all costing the same amount. After he uses a $25 gift certificate, he can spend no more than $75. Write an inequality to find the possible costs for a shirt. Identify what your variable represents.
Type below:
___________

Answer: 4x – 25 ≤ 75

Explanation:
Let x represent the cost of each skirt the 4x is the total costs of the shirts. Since he has a $25 gift card, the total amount he is spending is 4x – 25.
4x – 25 ≤ 75

Question 10.
Due to fire laws, no more than 720 people may attend a performance at Metro Auditorium. The balcony holds 120 people. There are 32 rows on the ground floor, each with the same number of seats. Write an inequality to find the numbers of people that can sit in a ground-floor row if the balcony is full. Identify what your variable represents.
Type below:
___________

Answer: 32x + 120 ≤ 720

Explanation:
Let x represent the number of people that can sit in each ground floor row. then 32x is the total number of people sitting in the ground floor.
Since 120 people are sitting in the balcony, the total number of people is 32x + 120.

Question 11.
Liz earns a salary of $2,100 per month, plus a commission of 5% of her sales. She wants to earn at least $2,400 this month. Write an inequality to find amounts of sales that will meet her goal. Identify what your variable represents.
Type below:
___________

Answer: 2100 + 0.05x ≥ 2400

Explanation:
Let x represent the number of sales then 0.05x is the amount she earns in commission and 2100 + 0.05x is her total earnings.
2100 + 0.05x ≥ 2400

Question 12.
Lincoln Middle School plans to collect more than 2,000 cans of food in a food drive. So far, 668 cans have been collected. Write an inequality to find numbers of cans the school can collect on each of the final 7 days of the drive to meet this goal. Identify what your variable represents.
Type below:
___________

Answer: 7x + 668 > 2000

Explanation:
Let x represent the number of cans collected each day. Then 7x is the total number of cans collected on the final 7 days of the drive.
Since they have collected 668 cans already, the total number of cans collected is 7x + 668.
They want to collect more than 2000 cans, so the inequality is:
7x + 668 > 2000

Question 13.
Joanna joins a CD club. She pays $7 per month plus $10 for each CD that she orders. Write an inequality to find how many CDs she can purchase in a month if she spends no more than $100. Identify what your variable represents.
Type below:
___________

Answer: 7 + 10x ≤ 100

Explanation:
Let x represent the number of CDs then 10x is the total amount spent on CDs and 7 + 10x is the total purchase amount for the month.
7 + 10x ≤ 100

Question 14.
Lionel wants to buy a belt that costs $22. He also wants to buy some shirts that are on sale for $17 each. He has $80. What inequality can you write to find the number of shirts he can buy? Identify what your variable represents.
Type below:
___________

Answer: 22 + 17x ≤ 80

Explanation:
Let x represent the number of shirts he can buy then 17x is the total cost of the shirts and 22 + 17x is the total cost
22 + 17x ≤ 80

Page No. 216

Question 15.
Write a situation for 15x − 20 ≤ 130 and solve.
Type below:
___________

Answer:
You are given in the inequality 15x − 20 ≤ 130 and need to write a situation that is represented by this inequality. A possible situation could be:
You are going shopping to buy some shirts. The shirts cost $15 each. You have a $20 gift card and $130 in cash. How many shirts can you buy?
This solution is modeled by the inequality 15x − 20 ≤ 130 because if we let x be the number of shirts you buy, then 15x is the cost of the shirts before you use the gift card since each shirt is $15. The total cost after using the $20 gift card would then be 15x – 20 dollars. Since you have $130 in cash, you can spend at most $130 so the total cost of 15x – 20 must then be less than or equal to 130.
Therefore, 15x − 20 ≤ 130.
Solving this gives:
15x − 20 ≤ 130
Add 20 on both sides
15x ≤ 130
Divide both sides by 15.
x ≤ 10

Analyze Relationships

Write >, <, ≥, or ≤ in the blank to express the given relationship.

Question 16.
m is at least 25
______

Answer: m ≥ 25

Explanation:
m is at least 25 means m ≥ 25 since at least means it can equal or needs to be bigger.

Question 17.
k is no greater than 9
______

Answer: k ≤ 9

Explanation:
k is no greater than 9 means k ≤ 9 since no greater means it can equal or must be smaller.

Question 18.
p is less than 48
______

Answer: p < 48

Explanation:
p is less than 48 means p < 48 since the < symbol in words is “less than”.

Question 19.
b is no more than -5
______

Answer: b ≤ -5

Explanation:
b is no more than -5 means b ≤ -5 since no more means it can equal or must be smaller.

Question 20.
h is at most 56
______

Answer: h ≤ 56

Explanation:
h is at most 56 means h ≤ 56 since at most means it can equal or must be smaller.

Question 21.
w is no less than 0
______

Answer: w ≥ 0

Explanation:
w is no less than 0 means w ≥ 0 since no less than means it can equal or is bigger.

Question 22.
Critical Thinking
Marie scored 95, 86, and 89 on three science tests. She wants her average score for 6 tests to be at least 90. What inequality can you write to find the average scores that she can get on her next three tests to meet this goal? Use s to represent the lowest average score.
Type below:
______

Answer: (95 + 86 + 89 + 3s)/6 ≥ 90

Explanation:
Let s be the average score on the remaining 3 tests. Then 3s is the sum of these 3 remaining tests. Since averages are found by adding up all the members, which gives 95 + 86 + 89 + 3s, and dividing by the total amount of numbers, which is 6, the inequality to the left can be used to find the lowest average she can get to have minimum total average of 90.
(95 + 86 + 89 + 3s)/6 ≥ 90

H.O.T.

Focus on Higher Order Thinking

Question 23.
Communicate Mathematical Ideas
Write an inequality that expresses the reason the lengths 5 feet, 10 feet, and 20 feet could not be used to make a triangle. Explain how inequality demonstrates that fact.
Type below:
__________

Answer:
A side of a triangle must be greater than the difference of the other two sides and smaller than the sum of the other two sides.
Since 20 > 5 +10, the side with length 20 is not less than the sum of the other two sides.

Question 24.
Analyze Relationships
The number m satisfies the relationship m < 0. Write an inequality expressing the relationship between -m and 0. Explain your reasoning.
Type below:
__________

Answer: Multiplying both sides of m < 0 by -1 gives -m > 0 since you must switch the inequality sign when you multiply by a negative number.

Question 25.
Analyze Relationships The number n satisfies the relationship n > 0. Write three inequalities to express the relationship between n and \(\frac{1}{n}\).
Type below:
__________

Answer: 0 < n < 1: n < \(\frac{1}{n}\), n = 1: n = \(\frac{1}{n}\), n > 1: n > \(\frac{1}{n}\)

Explanation:
Since n is positive, there are three cases for the value of n to consider when comparing n and \(\frac{1}{n}\).
0 < n < 1: In case, n is a fraction smaller than 1 such as \(\frac{1}{4}\). Its reciprocal is the n bigger than 1, such as 1/\(\frac{1}{4}\) = 4.
Therefore, n < \(\frac{1}{n}\)
n = 1: If n = 1, then the reciprocal is also 1 so n = \(\frac{1}{n}\)
n > 1: If n is a value greater than 1, such as \(\frac{3}{2}\), then the reciprocal is smaller than 1, such as 1/\(\frac{3}{2}\) = \(\frac{2}{3}\).
Therefore n > \(\frac{1}{n}\)

Page No. 220

Question 1.
Describe how to solve the inequality 3x + 4 < 13 using algebra tiles.
Go Math Grade 7 Answer Key Chapter 7 Writing and Solving One-Step Inequalities img 3
Type below:
__________

Answer:
First, remove 4 positive squares from each side. This leaves 9 positive squares on the right side.
Then divide each side into 3 equal groups.
Each group would then have 3 positive squares on the left side. since the rectangles on the left are positive, the answer would be x < 3.

Solve each inequality. Graph and check the solution.

Question 2.
5d − 13 < 32
Type below:
__________

Answer:
To graph inequalities, locate the number opposite the variable of the inequality on a number line. If the inequality is either a ≤ or a ≥, we use a closed dot, meaning the number is a solution as well. If the inequality is either a > or a <, use an open dot, indicating that the number is not a solution.
From here, shade the line going to the left if the inequality is either ≤ or < and shade the line going to the right if the inequality is either ≥ or >.
First, solve the inequality:
5d − 13 < 32
Add 13 on both sides
5d – 13 + 13 < 32 + 13
5d < 45
Divide 5 on both sides
5d/5 < 45/5
d < 9
The number opposite to the variable is 9, we look for this in the number line. Since the inequality is <, we use a hollow dot and shade the line going to the left. Its graph would like the one below:

Question 3.
−4b + 9 ≤ −7
Type below:
__________

Answer:
To graph inequalities, locate the number opposite the variable of the inequality on a number line. If the inequality is either a ≤ or a ≥, we use a closed dot, meaning the number is a solution as well. If the inequality is either a > or a <, use an open dot, indicating that the number is not a solution.
From here, shade the line going to the left if the inequality is either ≤ or < and shade the line going to the right if the inequality is either ≥ or >.
First, solve the inequality:
−4b + 9 ≤ −7
Subtract 9 on both sides
-4b + 9 – 9 ≤ −7 – 9
-4b ≤ −16
Divide -4 by both sides
-4b/-4 ≤ −16/-4
b ≥ 4
The number opposite to the variable is 4, we look for this in the number line. Since the inequality is ≥, we use a closed dot and shade the line going to the right. Its graph would like the one below:

Circle any given values that make the inequality true.

Question 4.
2m + 18 > −4
m = −12; m = −11; m = −10
Type below:
__________

Answer: m = -10

Explanation:
m = -12
2m + 18 > −4
2(-12)  + 18 = -24 + 18 = -6 > -4 not true
m = -11
2m + 18 > −4
2(-11) + 18 > -4
-22 + 18 > -4
-4 > -4 not true
m = -10
2m + 18 > −4
2(-10) + 18 > -4
-20 + 18 > -4
-2 > -4 true

Question 5.
−6y + 3 ≥ 0
y = 1; y = \(\frac{1}{2}\); y = 0
Type below:
__________

Answer: y = \(\frac{1}{2}\); y = 0

Explanation:
y = 1
−6y + 3 ≥ 0
-6(1) + 3 ≥ 0
-6 + 3 ≥ 0 not true
y = \(\frac{1}{2}\)
−6y + 3 ≥ 0
−6(\(\frac{1}{2}\)) + 3 ≥ 0
-3 + 3 ≥ 0
0 ≥ 0 true
y = 0
−6y + 3 ≥ 0
-6(0) + 3 ≥ 0
3 ≥ 0 true

Question 6.
Lizzy has 6.5 hours to tutor 4 students and spend 1.5 hours in a lab. She plans to tutor each student the same amount of time. The inequality 6.5 − 4t ≥ 1.5 can be used to find t, the amount of time in hours Lizzy could spend with each student. Solve the inequality. Graph and interpret the solution. Can Lizzy tutor each student for 1.5 hours? Explain.
Type below:
__________

Answer:
6.5 − 4t ≥ 1.5
Subtract both sides by 6.5
6.5 − 4t – 6.5 ≥ 1.5 – 6.5
− 4t ≥ 1.5
Divide both sides by -4 remember to switch the inequality sign since you are dividing by a negative number.
t ≤ 1.25
Since the inequality sign has an equal sign, draw a closed circle at 1.25. Since the inequality is less than, draw an arrow to the left the interpretation of this problem is that she can tutor each student no more than 1.25 hours.
Since 1.5 hours is more than 1.25 hours, she cannot tutor each student for 1.5 hours.

Essential Question Check-In

Question 7.
How do you solve a two-step inequality?
Type below:
__________

Answer:
To solve a two-step inequality you must use inverse operations.
Use subtraction to get rid of addition, and addition to get rid of subtraction for any terms that are being added or subtracted to the term that contains the variable. Use multiplication to get rid of any coefficient on the variable if it is a fraction and use division to get rid of any coefficient on the variable if its an integer. If the coefficient is negative, make sure to switch the inequality sign if you multiply or divide by a negative number.

Independent Practice – Page No. 221

Solve each inequality. Graph and check the solution.

Question 8.
2s + 5 ≥ 49
Type below:
__________

Answer:
To graph inequalities, locate the number opposite the variable of the inequality on a number line. If the inequality is either a ≤ or a ≥, we use a closed dot, meaning the number is a solution as well. If the inequality is either a > or a <, use an open dot, indicating that the number is not a solution.
From here, shade the line going to the left if the inequality is either ≤ or < and shade the line going to the right if the inequality is either ≥ or >.
First, solve the inequality:
2s + 5 ≥ 49
Subtract 5 on both sides
2s + 5 – 5 ≥ 49 – 5
2s ≥ 44
Divide both sides by 2.
s ≥ 22
The number opposite to the variable is 22, we look for this in the number line. Since the inequality is ≥, we use a closed dot and shade the line going to the right. Its graph would like the one below:

Question 9.
−3t + 9 ≥ −21
Type below:
__________

Answer:
To graph inequalities, locate the number opposite the variable of the inequality on a number line. If the inequality is either a ≤ or a ≥, we use a closed dot, meaning the number is a solution as well. If the inequality is either a > or a <, use an open dot, indicating that the number is not a solution.
From here, shade the line going to the left if the inequality is either ≤ or < and shade the line going to the right if the inequality is either ≥ or >.
First, solve the inequality:
−3t + 9 ≥ −21
Subtract 9 on both sides
-3t + 9 – 9 ≥ −21 – 9
-3t ≥ −30
Divide by -3 on both sides
t ≤ 10
The number opposite to the variable is 10, we look for this in the number line. Since the inequality is ≤, we use a closed dot and shade the line going to the left. Its graph would like the one below:

Question 10.
55 > −7v + 6
Type below:
__________

Answer:
To graph inequalities, locate the number opposite the variable of the inequality on a number line. If the inequality is either a ≤ or a ≥, we use a closed dot, meaning the number is a solution as well. If the inequality is either a > or a <, use an open dot, indicating that the number is not a solution.
From here, shade the line going to the left if the inequality is either ≤ or < and shade the line going to the right if the inequality is either ≥ or >.
First, solve the inequality:
55 > −7v + 6
Subtract 6 on both sides
55 – 6 > −7v + 6 – 6
49 > -7v
Divide -7 on both sides
-7 < v
v > -7
The number opposite to the variable is -7, we look for this in the number line. Since the inequality is >, we use a hollow dot and shade the line going to the right. Its graph would like the one below:

Question 11.
41 > 6m − 7
Type below:
__________

Answer:
To graph inequalities, locate the number opposite the variable of the inequality on a number line. If the inequality is either a ≤ or a ≥, we use a closed dot, meaning the number is a solution as well. If the inequality is either a > or a <, use an open dot, indicating that the number is not a solution.
From here, shade the line going to the left if the inequality is either ≤ or < and shade the line going to the right if the inequality is either ≥ or >.
First, solve the inequality:
41 > 6m − 7
Add 7 on both sides
41 + 7 > 6m – 7 + 7
48 > 6m
Divide by 6 on both sides
8 > m
m < 8
The number opposite to the variable is 8, we look for this in the number line. Since the inequality is <, we use a hollow dot and shade the line going to the left. Its graph would like the one below:

Question 12.
\(\frac{a}{-8}\) + 15 > 23
Type below:
__________

Answer:
To graph inequalities, locate the number opposite the variable of the inequality on a number line. If the inequality is either a ≤ or a ≥, we use a closed dot, meaning the number is a solution as well. If the inequality is either a > or a <, use an open dot, indicating that the number is not a solution.
From here, shade the line going to the left if the inequality is either ≤ or < and shade the line going to the right if the inequality is either ≥ or >.
First, solve the inequality:
\(\frac{a}{-8}\) + 15 > 23
Subtract 15 on both sides
\(\frac{a}{-8}\) + 15 – 15 > 23 – 15
(-8)\(\frac{a}{-8}\) > (8)(-8)
a < -64
The number opposite to the variable is -64, we look for this in the number line. Since the inequality is <, we use a hollow dot and shade the line going to the left. Its graph would like the one below:

Question 13.
\(\frac{f}{2}\) − 22 < 48
Type below:
__________

Answer:
To graph inequalities, locate the number opposite the variable of the inequality on a number line. If the inequality is either a ≤ or a ≥, we use a closed dot, meaning the number is a solution as well. If the inequality is either a > or a <, use an open dot, indicating that the number is not a solution.
From here, shade the line going to the left if the inequality is either ≤ or < and shade the line going to the right if the inequality is either ≥ or >.
First, solve the inequality:
\(\frac{f}{2}\) − 22 < 48
Add 22 on both sides
\(\frac{f}{2}\) − 22 + 22 < 48 + 22
\(\frac{f}{2}\) < 70
Multiply 2 on both sides
(2)\(\frac{f}{2}\) < 70(2)
f < 140
The number opposite to the variable is 140, we look for this in the number line. Since the inequality is <, we use a hollow dot and shade the line going to the left. Its graph would like the one below:

Question 14.
−25 + \(\frac{t}{2}\) ≥ 50
Type below:
__________

Answer:
To graph inequalities, locate the number opposite the variable of the inequality on a number line. If the inequality is either a ≤ or a ≥, we use a closed dot, meaning the number is a solution as well. If the inequality is either a > or a <, use an open dot, indicating that the number is not a solution.
From here, shade the line going to the left if the inequality is either ≤ or < and shade the line going to the right if the inequality is either ≥ or >.
First, solve the inequality:
−25 + \(\frac{t}{2}\) ≥ 50
Add 25 on both sides
−25 + \(\frac{t}{2}\) + 25 ≥ 50 + 25
\(\frac{t}{2}\) ≥ 75
Multiply 2 on both sides
(2)\(\frac{t}{2}\) ≥ 75 (2)
t ≥ 150
The number opposite to the variable is 150, we look for this in the number line. Since the inequality is ≥, we use a closed dot and shade the line going to the right. Its graph would like the one below:

Question 15.
10 + \(\frac{g}{-9}\) > 12
Type below:
__________

Answer:
To graph inequalities, locate the number opposite the variable of the inequality on a number line. If the inequality is either a ≤ or a ≥, we use a closed dot, meaning the number is a solution as well. If the inequality is either a > or a <, use an open dot, indicating that the number is not a solution.
From here, shade the line going to the left if the inequality is either ≤ or < and shade the line going to the right if the inequality is either ≥ or >.
First, solve the inequality:
10 + \(\frac{g}{-9}\) > 12
Subtract 10 on both sides
10 + \(\frac{g}{-9}\) -10 > 12 – 10
\(\frac{g}{-9}\) > 2
Divide both sides by -9
g < -18
The number opposite to the variable is -18, we look for this in the number line. Since the inequality is <, we use a hollow dot and shade the line going to the left. Its graph would like the one below:

Question 16.
25.2 ≤ − 1.5y + 1.2
Type below:
__________

Answer:
To graph inequalities, locate the number opposite the variable of the inequality on a number line. If the inequality is either a ≤ or a ≥, we use a closed dot, meaning the number is a solution as well. If the inequality is either a > or a <, use an open dot, indicating that the number is not a solution.
From here, shade the line going to the left if the inequality is either ≤ or < and shade the line going to the right if the inequality is either ≥ or >.
First, solve the inequality:
25.2 ≤ − 1.5y + 1.2
Subtract 1.2 on both sides
25.2 – 1.2 ≤ − 1.5y + 1.2 – 1.2
24 ≤ -1.5y
Divide both sides by 1.5
-16 ≥ y
y ≤ -16
The number opposite to the variable is -16, we look for this in the number line. Since the inequality is ≤, we use a closed dot and shade the line going to the left. Its graph would like the one below:

Question 17.
−3.6 ≥ −0.3a + 1.2
Type below:
__________

Answer:
To graph inequalities, locate the number opposite the variable of the inequality on a number line. If the inequality is either a ≤ or a ≥, we use a closed dot, meaning the number is a solution as well. If the inequality is either a > or a <, use an open dot, indicating that the number is not a solution.
From here, shade the line going to the left if the inequality is either ≤ or < and shade the line going to the right if the inequality is either ≥ or >.
First, solve the inequality:
−3.6 ≥ −0.3a + 1.2
Subtract 1.2 on both sides
−3.6 – 1.2 ≥ −0.3a + 1.2 – 1.2
-4.8 ≥ −0.3a
Divide both sides by -0.3
16 ≤ a
a ≥ 16
The number opposite to the variable is 16, we look for this in the number line. Since the inequality is ≥, we use a closed dot and shade the line going to the right. Its graph would like the one below:

Question 18.
What If?
The perimeter of a rectangle is at most 80 inches. The length of the rectangle is 25 inches. The inequality 80 − 2w ≥ 50 can be used to find w, the width of the rectangle in inches. Solve the inequality and interpret the solution. How will the solution change if the width must be at least 10 inches and a whole number?
Type below:
__________

Answer:
The perimeter of a rectangle is at most 80 inches. The length of the rectangle is 25 inches. The inequality 80 − 2w ≥ 50 can be used to find w, the width of the rectangle in inches.
Subtract 80 on both sides
-2w ≥ -30
Divide -2 on both sides
w ≤ 15
The interpretation of this solution is that width must be a positive real number that is at most 15 inches.
If the width must be at least 10 inches and a whole number, then the solution would be the whole number lengths such that 10 ≤ w ≤ 15 which is 10, 11, 12, 13, 14 or 15 inches.

Page No. 222

Question 19.
Interpret the Answer
Grace earns $7 for each car she washes. She always saves $25 of her weekly earnings. This week, she wants to have at least $65 in spending money. How many cars must she wash? Write and solve an inequality to represent this situation. Interpret the solution in context.
Type below:
__________

Answer:
Let x represent the number of cars she must wash then 7x is the total amount she makes from washing cars. Since she is saving $25 of her earnings, 7x – 25 is the amount of earnings she will have to spend.
7x – 25 ≥ 65
Add 25 on both sides
7x – 25 + 25 ≥ 65 + 25
7x ≥ 90
Divide both sides by 7.
x ≥ 90/7 ≈ 13

H.O.T.

Focus on Higher Order Thinking

Question 20.
Critical Thinking
Is there any value of x with the property that x < x – 1? Explain your reasoning.
Type below:
__________

Answer:
Subtracting x on both sides of x < x – 1 gives x – x < x – 1 -x
0 < -1.
Since this not a true statement, inequality is not true for any value of x.

Question 21.
Analyze Relationships A compound inequality consists of two simple equalities joined by the word “and” or “or.” Graph the solution sets of each of these compound inequalities. a. x > 2 and x < 7
Type below:
__________

Answer: Since the two inequalities are joined by “and”, we need to satisfy both inequalities. Therefore, we need all values greater than 2 but also we need values less than 7. Hence we can place a hollow dot on 2 and shade the line going to the right until another hollow dot on 7.

Question 21.
b. x < 2 or x > 7
Type below:
__________

Answer: Since the two inequalities are joined by “or”, we need to satisfy either of the inequalities. Therefore, we need all values less than 2 but also we need to graph the values greater than 7. Hence we can place a hollow dot on 2 and shade the line going to the left and another hollow dot on 7 and shade the line going to the right.

Question 21.
c. Describe the solution set of the compound inequality x < 2 and x > 7.
Type below:
__________

Answer: The solution set for x > 2 and x < 7 sould be 2 < x < 7.

Question 21.
d. Describe the solution set of the compound inequality x > 2 or x < 7.
Type below:
__________

Answer: The solution set for x < 2 and x > 7 sould be (-∞, 2) ∪ (7, ∞).

Question 22.
Communicate Mathematical Ideas
Joseph used the problem-solving strategy Work Backward to solve the inequality 2n + 5 < 13. Shawnee solved the inequality using the algebraic method you used in this lesson. Compare the two methods.
Type below:
__________

Answer: Both involve using the same operations. The only difference is that working backward is done mostly mentally while algebraically is done on paper. It is easier to determine which direction the inequality is pointing when using the algebraic method.

7.1 Writing and Solving One-Step Inequalities – Page No. 223

Solve each inequality.

Question 1.
n + 7 < −3
Type below:
__________

Answer: n < -10

Explanation:
Subtract 7 on both sides
n + 7 – 7 < −3 – 7
n < -10

Question 2.
5p ≥ −30
Type below:
__________

Answer: p ≥ -6

Explanation:
5p ≥ −30
Divide by 5 on both sides
p ≥ -6

Question 3.
14 < k + 11
Type below:
__________

Answer: 3 < k

Explanation:
14 < k + 11
Subtract 11 on both sides
14 – 11 < k + 11 – 11
3 < k

Question 4.
\(\frac{d}{-3}\) ≤ −6
Type below:
__________

Answer: d ≥ 18

Explanation:
\(\frac{d}{-3}\) ≤ −6
Multiply both sides by -3 remember to switch the inequality sign since you are multiplying both sides by a negative number.
d ≥ 18

Question 5.
c − 2.5 ≤ 2.5
Type below:
__________

Answer: c ≤ 5

Explanation:
c − 2.5 ≤ 2.5
Add 2.5 on both sides
c − 2.5 + 2.5 ≤ 2.5 + 2.5
c ≤ 5

Question 6.
12 ≥ −3b
Type below:
__________

Answer: -4 ≤ b

Explanation:
12 ≥ −3b
Divide by -3 on both sides
-4 ≤ b

Question 7.
Jose has scored 562 points on his math tests so far this semester. To get an A for the semester, he must score at least 650 points. Write and solve an inequality to find the minimum number of points he must score on the remaining tests in order to get an A.
Type below:
__________

Answer: x + 562 ≥ 650

Explanation:
Let x represent the score he must receive on the remaining tests. Then x + 562 is his total score for the semester.
x + 562 ≥ 650
Subtract 562 on both sides
x ≥ 88

7.2 Writing Two-Step Inequalities

Question 8.
During a scuba dive, Lainey descended to a point 20 feet below the ocean surface. She continued her descent at a rate of 20 feet per minute. Write an inequality you could solve to find the number of minutes she can continue to descend if she does not want to reach a point more than 100 feet below the ocean surface.
Type below:
__________

Answer:
Let x represent the number of minutes. Since she is descending 20 feet per minute, then -20x represents her altitude. It is negative since descending means her altitude is decreasing.
Since she started at 20 feet below the ocean surface, she started at -20 feet. Its negative since an altitude below the ocean surface must be represented by a negative number.
Her ending position is the sum of how far she has descended and her initial position so her ending position is -20 + (-20x) = -20 – 20x
She doesn’t want to travel more than 100 feet below the ocean surface so she needs to be higher than -100 feet.
The inequality is then -20 -20x ≥ -100.

7.3 Solving Two-Step Inequalities

Solve.

Question 9.
2s + 3 > 15
Type below:
__________

Answer: s > 6

Explanation:
2s + 3 > 15
Subtract 3 on both sides
2s + 3 – 3 > 15 – 3
2s > 12
Divide by 2 on both sides
s > 6

Question 10.
−\(\frac{d}{12}\) − 6 < 1
Type below:
__________

Answer: d > -84

Explanation:
−\(\frac{d}{12}\) − 6 < 1
Add 6 on both sides
−\(\frac{d}{12}\) − 6 + 6 < 1 + 6
d > -84

Question 11.
−6w − 18 ≥ 36
Type below:
__________

Answer: w ≤ -9

Explanation:
−6w − 18 ≥ 36
Add 18 on both sides
−6w − 18 + 18 ≥ 36 + 18
-6w ≥ 54
Divide by -6
w ≤ -9

Question 12.
\(\frac{z}{4}\) + 22 ≤ 38
Type below:
__________

Answer: z ≤ 64

Explanation:
Subtract 22 on both sides
\(\frac{z}{4}\) + 22 – 22 ≤ 38 – 22
\(\frac{z}{4}\) ≤ 16
z ≤ 64

Question 13.
\(\frac{b}{9}\) − 34 < −36
Type below:
__________

Answer: b < -18

Explanation:
\(\frac{b}{9}\) − 34 < −36
Add 34 on both sides
\(\frac{b}{9}\) − 34 + 34 < −36 + 34
\(\frac{b}{9}\) < -2
b < -18

Question 14.
−2p + 12 > 8
Type below:
__________

Answer: p < 2

Explanation:
−2p + 12 > 8
Subtract 12 on both sides
-2p + 12 – 12 > 8 – 12
-2p > -4
p < 2

Essential Question

Question 15.
How can you recognize whether a real-world situation should be represented by an equation or an inequality?
Type below:
__________

Answer:
You use an equation when the situation involves finding an exact answer. You use an inequality when the solution can have more than one value. Problems that require the use of inequalities have phrases in them such as “at least”, “no more”, “at most” and “no less than”.

Selected Response – Page No. 224

Question 1.
Which graph models the solution of the inequality −6 ≤ −3x?
Options:
a. Go Math Grade 7 Answer Key Chapter 7 Writing and Solving One-Step Inequalities img 4
b. Go Math Grade 7 Answer Key Chapter 7 Writing and Solving One-Step Inequalities img 5
c. Go Math Grade 7 Answer Key Chapter 7 Writing and Solving One-Step Inequalities img 6
d. Go Math Grade 7 Answer Key Chapter 7 Writing and Solving One-Step Inequalities img 7

Answer: Go Math Grade 7 Answer Key Chapter 7 Writing and Solving One-Step Inequalities img 6
Dividing both sides of −6 ≤ −3x by -3 gives 2 ≥ x.
Rewriting this so x is on the left side gives x ≤ 2. The graph must have a closed circle at 2 since the inequality has an equal sign and must be shaded to the left since its <.
Thus the correct answer is option C.

Question 2.
A taxi cab costs $1.75 for the first mile and $0.75 for each additional mile. You have $20 to spend on your ride. Which inequality could be solved to find how many miles you can travel, if n is the number of additional miles?
Options:
a. 1.75n + 0.75 ≥ 20
b. 1.75n + 0.75 ≤ 20
c. 0.75n + 1.75 ≥ 20
d. 0.75n + 1.75 ≤ 20

Answer: 1.75n + 0.75 ≤ 20

Explanation:
Let n represent the number of additional miles. Then 0.75n is the cost of the additional miles which gives a total cost of 1.75 + 0.75n. You can spend a maximum of $20 so the inequality is ≤.
Thus the correct answer is option B.

Question 3.
The inequality \(\frac{9}{5}\)C + 32 < −40 can be used to find Celsius temperatures that are less than -40° Fahrenheit. What is the solution of the inequality?
Options:
a. C < 40
b. C < −\(\frac{40}{9}\)
c. C < −40
d. C < −\(\frac{72}{5}\)

Answer: C < −40

Explanation:
\(\frac{9}{5}\)C + 32 < −40
Subtract 32 on both sides.
\(\frac{9}{5}\)C + 32 – 32 < −40 – 32
\(\frac{9}{5}\)C < -72
c < -40
Thus the correct answer is option C.

Question 4.
The 30 members of a choir are trying to raise at least $1,500 to cover travel costs to a singing camp. They have already raised $600. Which inequality could you solve to find the average amounts each member can raise that will at least meet the goal?
Options:
a. 30x + 600 > 1,500
b. 30x + 600 ≥ 1,500
c. 30x + 600 < 1,500
d. 30x + 600 ≤ 1,500

Answer: 30x + 600 ≥ 1,500

Explanation:
Given,
The 30 members of a choir are trying to raise at least $1,500 to cover travel costs to a singing camp. They have already raised $600.
Let x represent the average amount each member raises.
There are 30 members so the members raise a combined amount of 30x.
Since they have already raised $600, the total amount raised 30x + 600 they need to raise at least $1500 so the inequality is ≥.
Thus the correct answer is option B.

Question 5.
Which represents the solution for the inequality 3x − 7 > 5?
Options:
a. x < 4
b. x ≤ 4
c. x > 4
d. x ≥ 4

Answer: x > 4

Explanation:
Add 7 on both sides
3x − 7 > 5
3x − 7 + 7> 5 + 7
3x > 12
x > 4
Thus the correct answer is option C.

Question 6.
Which inequality has the following graphed solution?
Go Math Grade 7 Answer Key Chapter 7 Writing and Solving One-Step Inequalities img 8
Options:
a. 3x + 8 ≤ 2
b. 4x + 12 < 4
c. 2x + 5 ≤ 1
d. 3x + 6 < 3

Answer: 4x + 12 < 4

Explanation:
4x + 12 < 4
Subtract 12 on both sides
4x + 12 – 12 < 4 – 12
4x < -8
x < -2
Thus the correct answer is option B.

Question 7.
Divide: −36 ÷ 6.
Options:
a. 30
b. 6
c. -6
d. -30

Answer: -6

Explanation:
6 divides 36 six times
−36 ÷ 6 = -6
Thus the correct answer is option C.

Question 8.
Eleni bought 2 pounds of grapes at a cost of $3.49 per pound. She paid with a $10 bill. How much change did she get back?
Options:
a. $3.02
b. $4.51
c. $6.51
d. $6.98

Answer: $3.02

Explanation:
Given,
Eleni bought 2 pounds of grapes at a cost of $3.49 per pound. She paid with a $10 bill.
We have to find the total amount paid for the grapes
2 × 3.49 = 6.98
10 – 6.98 = 3.02
Thus the correct answer is option A.

Question 9.
In golf, the lower your score, the better. Negative scores are best of all. Teri scored +1 on each of the first three holes at a nine-hole miniature golf course. Her goal is a total score of -9 or better after she has completed the final six holes.
a. Let h represent the score Teri must average on each of the last six holes in order to meet her goal. Write a two-step inequality you can solve to find h.
Type below:
_____________

Answer: 3 +6h ≤ -9

Explanation:
If h is her average score for the last 6 holes, then 6h is her total score for the last 6 holes.
She currently has a score of 3 so ger total score for all 9 holes is 3 + 6h.
She wants a score of -9 or better and since smaller scores are better, the inequality is ≤.

Question 9.
b. Solve the inequality.
Type below:
_____________

Answer: h ≤ -2

Explanation:
6h ≤ -12
Divide by 6 on both sides
h ≤ -2

EXERCISES – Page No. 226

Simplify each expression.

Question 1.
\(\left(2 x+3 \frac{2}{5}\right)+\left(5 x-\frac{4}{5}\right)\)
Type below:
_____________

Answer: 7x + 2 \(\frac{3}{5}\)

Explanation:
We are given  the expression,
\(\left(2 x+3 \frac{2}{5}\right)+\left(5 x-\frac{4}{5}\right)\)
Group the like terms
(2x + 5x) + (3\(\frac{2}{5}\) – \(\frac{4}{5}\))
7x + 2 \(\frac{3}{5}\)

Question 2.
(−0.5x − 4) − (1.5x + 2.3)
Type below:
_____________

Answer: -2x – 6.3

Explanation:
(−0.5x − 4) − (1.5x + 2.3)
(−0.5x − 4) − 1.5x – 2.3
Combine the lilke terms
-0.5x – 1.5x – 4 – 2.3
-2x – 6.3

Question 3.
9(3t + 4b)
Type below:
_____________

Answer: 27t + 36b

Explanation:
9(3t + 4b)
9 × 3t + 9 × 4b
27t + 36b

Question 4.
0.7(5a − 13p)
Type below:
_____________

Answer: 3.5a – 9.1p

Explanation:
0.7(5a − 13p)
0.7 × 5a – 0.7 × 13p
3.5a – 9.1p

Factor each expression.

Question 5.
8x + 56
Type below:
_____________

Answer: 8(x + 7)

Explanation:
Since 56 ÷ 8 = 7 and 8 ÷ 8 = 1, factor out 8 from both terms
8x + 56 = 8(x + 7)

Question 6.
3x + 57
Type below:
_____________

Answer: 3(x + 19)

Explanation:
Since 3 ÷ 3 = 1 and 57 ÷ 3 = 19 factor out 3 from both terms.
3x + 57 = 3(x + 19)

Question 7.
1.6 + y = −7.3
_______

Answer: y = -8.9

Explanation:
subtract 1.6 on both sides
1.6 + y – 1.6 = -7.3 – 1.6
y = -8.9

Question 8.
−\(\frac{2}{3}\) n = 12
_______

Answer: n = -18

Explanation:
−\(\frac{2}{3}\) n = 12
n = −\(\frac{3}{2}\)(12)
n = -3 × 6
n = -18

Question 9.
The cost of a ticket to an amusement park is $42 per person. For groups of up to 8 people, the cost per ticket decreases by $3 for each person in the group. Marcos’s ticket cost $30. Write and solve an equation to find the number of people in Marcos’s group.
_______ people

Answer: 4

Explanation:
Given that,
The cost of a ticket to an amusement park is $42 per person.
For groups of up to 8 people, the cost per ticket decreases by $3 for each person in the group. Marcos’s ticket cost $30.
Let x represent the number of people in his group then the ticket price has changed by -3x dollars.
The original price is $42 so the discount price is 42 – 3x
42 – 3x = 30
-3x = 30 – 42
-3x = -12
Divide both sides by -3
x = 4
Thus there are 4 people in Marcos’s group.

Question 10.
8x − 28 = 44
_______

Answer:
To plot a point, starting from 0, count the number of units going to the left or right.
Given equation is
8x − 28 = 44
Add 28 on both sides
8x – 28 + 28 = 44 + 28
8x = 72
Divide by 8 on both sides
x = 9
To plot 9 on a number line, from 0, we move 9 units to the right.

Question 11.
−5z + 4 = 34
_______

Answer:
To plot a point, starting from 0, count the number of units going to the left or right.
Given equation is
−5z + 4 = 34
Subtract 4 on both sides
-5z + 4 – 4 = 34 – 4
-5z = 30
Divide both sides by -5
z = -6
To plot -6 on the number line, from 0, we move 6 units to the left.

EXERCISES – Page No. 227

Question 1.
Prudie needs $90 or more to be able to take her family out to dinner. She has already saved $30 and wants to take her family out to eat in 4 days.
a. Suppose that Prudie earns the same each day. Write an inequality to find how much she needs to earn each day.
Type below:
___________

Answer:
Let x be the amount she makes each day then 4x is the amount she will make in the 4 days before she takes her family out to eat and 4x + 30 is the total amount she will have saved.
4x + 30 ≥ 90

Question 1.
b. Suppose that Prudie earns $18 each day. Will she have enough money to take her family to dinner in 4 days? Explain.
_______

Answer:
4(18) + 30 = 72 + 30 = 102
She will have saved $102 in total if she earns $18 each day so she will have enough money.

Solve each inequality. Graph and check the solution.

Question 2.
11 − 5y < −19
Type below:
___________

Answer:
To graph inequalities, locate the number opposite the variable of the inequality on a number line. If the inequality is either a ≤ or a ≥, we use a closed dot, meaning the number is a solution as well. If the inequality is either a > or a <, use an open dot, indicating that the number is not a solution.
From here, shade the line going to the left if the inequality is either ≤ or < and shade the line going to the right if the inequality is either ≥ or >.
First, solve the inequality:
11 − 5y < −19
Subtract 11 on both sides
11 – 5y – 11 < −19 – 11
-5y < -30
Divide by -5 on both sides
y > 6
The number opposite to the variable is 6, we look for this in the number line. Since the inequality is >, we use a hollow dot and shade the line going to the right. Its graph would like the one below:

Question 3.
7x − 2 ≤ 61
Type below:
___________

Answer:
To graph inequalities, locate the number opposite the variable of the inequality on a number line. If the inequality is either a ≤ or a ≥, we use a closed dot, meaning the number is a solution as well. If the inequality is either a > or a <, use an open dot, indicating that the number is not a solution.
From here, shade the line going to the left if the inequality is either ≤ or < and shade the line going to the right if the inequality is either ≥ or >.
First, solve the inequality:
7x − 2 ≤ 61
Add 2 on both sides
7x – 2 + 2 ≤ 61 + 2
7x ≤ 63
Divide by 7 into both sides
x ≤ 9
The number opposite to the variable is 9, we look for this in the number line. Since the inequality is ≤, we use a closed dot and shade the line going to the left. Its graph would like the one below:

Unit 3 Performance Tasks – Page No. 228

Question 1.
Mechanical Engineer
A mechanical engineer is testing the amount of force needed to make a spring stretch by a given amount. The force y is measured in units called Newtons, abbreviated N. The stretch x is measured in centimeters. Her results are shown in the graph.
a. Write an equation for the line. Explain, using the graph and then using the equation, why the relationship is proportional.
Go Math Grade 7 Answer Key Chapter 7 Writing and Solving One-Step Inequalities img 9
Type below:
___________

Answer: The graph is linear and passes through the origin so the relationship is proportional. find k by using the formula k = y/x where (x, y) is a point on the line then plug k into the equation of a line y = kx.

Question 1.
b. Identify the rate of change and the constant of proportionality.
Type below:
___________

Answer: k = 8

Explanation:
Observe part a the units N/cm since the units for y are N and the units for x are cm and the units for k must be the units for y divided by the units for x.
8 N/ cm
k = 8

Question 1.
c. What is the meaning of the constant of proportionality in the context of the problem?
Type below:
___________

Answer:
Since the rate of change is 8 N/ cm this means that for every 1 cm stretch in the spring, the force required in Newton increases by 8 N.

Question 1.
d. The engineer applies a force of 41.6 Newtons to the spring. Write and solve an equation to find the corresponding stretch in the spring.
______ cm

Answer:
y = 8x
41.6 = 8x
x = 41.6/8
x = 5.2 cm

Question 2.
A math tutor charges $30 for a consultation, and then $25 per hour. An online tutoring service charges $30 per hour.
a. Does either service represent a proportional relationship? Explain.
Type below:
___________

Answer:
The math tutor charges $30 initially but has a constant rate of $25 per hour after. This means that it still is a proportional relationship. The online tutoring charges a constant rate of $30 per hour and thus is also a proportional relationship.

Question 2.
b. Write an equation for the cost c of h hours of tutoring for either service. Which service charges less for 4 hours of tutoring? Show your work.
Type below:
___________

Answer:
Using y as the total cost and x as the number of hours, we can represent each tutoring service. For Math tutor, we can write this as y = 30 + 25x while for the online tutoring, we can write this as y = 30x.
Substituting x = 4, we can see that:
For Math tutor:
y = 30 + 25x
y = 30 + 25(4)
y = 30 + 100
y = 130
For Online tutoring:
y = 30x
y = 30(4)
y = 120
Therefore, the online tutoring service charge less at $120.

Selected Response – Page No. 229

Question 1.
Which expression is equivalent to (9x − 3 \(\frac{1}{8}\)) − (7x + 1 \(\frac{3}{8}\))?
Options:
a. 2x − 4 \(\frac{1}{2}\)
b. 16x−4 \(\frac{1}{2}\)
c. 2x − 1 \(\frac{3}{4}\)
d. 16x − 1 \(\frac{3}{4}\)

Answer: 2x − 4 \(\frac{1}{2}\)

Explanation:
(9x − 3 \(\frac{1}{8}\)) − (7x + 1 \(\frac{3}{8}\))
9x − 3 \(\frac{1}{8}\)− 7x – 1 \(\frac{3}{8}\)
Combine the lilke terms
2x – 4 \(\frac{1}{2}\)
Thus the correct answer is option A.

Question 2.
Timothy began the week with $35. He bought lunch at school, paying $2.25 for each meal. Let x be the number of meals he bought at school and y be the amount of money he had left at the end of the week. Which equation represents the relationship in the situation?
Options:
a. y = 2.25x + 35
b. y = 35 − 2.25x
c. x = 35 − 2.25y
d. y = 2.25x − 35

Answer: y = 35 − 2.25x

Explanation:
Let x be the number of meals he buys means 2.25x is the amount of money he has spent on meals. The money he has left is then 35 – 2.25x
The expression is y = 35 – 2.25x
Thus the correct answer is option B.

Question 3.
Which expression factors to 8(x + 2)?
Options:
a. 8x + 2
b. 8x + 10
c. 16x
d. 8x + 16

Answer: 8x + 16

Explanation:
Given the expression
8(x + 2)
8x + 8(2)
8x + 16
Thus the correcr answer is option D.

Question 4.
Ramón’s toll pass account has a value of $32. Each time he uses the toll road, $1.25 is deducted from the account. When the value drops below $10, he must add value to the toll pass. Which inequality represents how many times Ramón can use the toll road without having to add value to the toll pass?
Options:
a. 10 − 1.25t ≥ 0
b. −1.25t + 32 < 10
c. 32 − 1.25t ≥ 10
d. 32 − 10t ≥ 1.25

Answer: 32 − 1.25t ≥ 10

Explanation:
Let t represent the number of times he uses the toll road then 1.25t is the amount deducted from his account. the remaining balance is then 32 – 1.25t.
Since his balance must be at least $10 for him to not have to add value, the inequality sign is ≥
Thus the correct answer is option C.

Question 5.
A taxi costs $1.65 for the first mile and $0.85 for each additional mile. Which equation could be solved to find the number x of additional miles traveled in a taxi given that the total cost of the trip is $20?
Options:
a. 1.65x + 0.85 = 20
b. 0.85x + 1.65 = 20
c. 1.65x − 0.85 = 20
d. 0.85x − 1.65 = 20

Answer: 0.85x + 1.65 = 20

Explanation:
Let x be the number of additional miles means 0.85x is the cost of the additional miles the total cost is then 1.65 + 0.85x
1.65 + 0.85x = 20
Thus the correct answer is option B.

Question 6.
A sales tax of 6% is added to the price of an item. If Marisa buys an item, which expression indicates how much she will pay in all?
Options:
a. n + 0.06
b. 0.06n
c. n + 0.06n
d. 0.06 + 0.06n

Answer: n + 0.06n

Explanation:
The total cost she will pay is the cost of the item n plus the cost of tax 0.06n.
The expression is n + 0.06n
Thus the correct answer is option C.

Question 7.
Which equation has the solution x = 12?
Options:
a. 4x + 3 = 45
b. 3x + 6 = 42
c. 2x − 5 = 29
d. 5x −8 = 68

Answer: 3x + 6 = 42

Explanation:
a. 4x + 3 = 45
Substitute x = 12 in the above equation.
4(12) + 3 = 45
48 + 3 = 45
51 ≠ 45
b. 3x + 6 = 42
Substitute x = 12 in the above equation.
3(12) + 6 = 42
36 + 6 = 42
42 = 42
c. 2x − 5 = 29
Substitute x = 12 in the above equation.
2(12) – 5 = 29
24 – 5 = 29
19 ≠ 29
d. 5x −8 = 68
Substitute x = 12 in the above equation.
5(12) – 8 = 68
60 – 8 = 68
52 ≠ 68
Thus the correct answer is option B.

Question 8.
The 23 members of the school jazz band are trying to raise at least $1,800 to cover the cost of traveling to a competition. The members have already raised $750. Which inequality could you solve to find the amount that each member should raise to meet the goal?
Options:
a. 23x + 750 > 1,800
b. 23x + 750 ≥ 1,800
c. 23x + 750 < 1,800
d. 23x + 750 ≤ 1,800

Answer: 23x + 750 ≥ 1,800

Explanation:
Let x represent the amount each member raises means 23x is the amount the members raise individually.
The total amount raised is then 23x + 750 since they have already raised $750.
Since they are trying to raise at least $1800, the inequality is ≥
Thus the correct answer is option B.

Page No. 230

Question 9.
What is the solution of the inequality 2x − 9 < 7?
Options:
a. x < 8
b. x ≤ 8
c. x > 8
d. x ≥ 8

Answer: x < 8

Explanation:
Given the inequality 2x − 9 < 7
Add 9 on both sides 2x – 9 + 9 < 7 + 9
2x < 16
Divide by 2 on both sides
2x/2 < 16/2
x < 8
Thus the correct answer is option A.

Question 10.
Which inequality has the solution n < 5?
Options:
a. 4n + 11 > −9
b. 4n + 11 < −9
c. −4n + 11 < −9
d. −4n + 11 > −9

Answer: −4n + 11 > −9

Explanation:
Given the inequality n < 5
To graph inequalities, locate the number opposite the variable of the inequality on a number line. If the inequality is either a ≤ or a ≥, we use a closed dot, meaning the number is a solution as well. If the inequality is either a > or a <, use an open dot, indicating that the number is not a solution.
a. 4n + 11 > −9
4n + 11 – 11 > -9 – 11
4n/4 > -20/4
n  -5
b. 4n + 11 < −9
4n + 11 – 11 < -9 – 11
4n < -20
4n/4 < -20/4
n < -5
c. −4n + 11 < −9
-4n + 11 – 11 < -9 – 11
-4n < -20
-4n/-4 < -20/-4
n > 5
d. −4n + 11 > −9
-4n + 11 – 11 > -9 – 11
-4n > -20
-4n/-4 > -20/-4
n < 5
Thus the correct answer is option D.

Question 11.
Which inequality has the solution shown?
Go Math Grade 7 Answer Key Chapter 7 Writing and Solving One-Step Inequalities img 10
Options:
a. 3x + 5 < 2
b. 4x + 12 < 4
c. 2x + 5 ≤ 1
d. 3x + 6 ≤ 3

Answer: 3x + 6 ≤ 3

Explanation:
The graph shows the inequality x ≤ -1 so the possible answers are C and D since A and B have < as the inequality signs. Solve C and D for x to see which one has x ≤ -1 as the solution.
c. 2x + 5 ≤ 1
2x + 5 – 5 ≤ 1 – 5
2x ≤ -4
x ≤ -2
d. 3x + 6 ≤ 3
3x + 6 – 6 ≤ 3 – 6
3x ≤ -3
x ≤ -1
Thus the correct answer is option D.

Question 12.
On a 4 \(\frac{1}{2}\) hour trip, Leslie drove \(\frac{2}{3}\) of the time. For how many hours did Leslie drive?
Options:
a. 3 hours
b. 3 \(\frac{1}{2}\) hours
c. 3 \(\frac{2}{3}\) hours
d. 3 \(\frac{5}{6}\) hours

Answer: 3 hours

Explanation:
Given that,
On a 4 \(\frac{1}{2}\) hour trip, Leslie drove \(\frac{2}{3}\) of the time.
Multiply the two fractions by first writing 4 \(\frac{1}{2}\) as an improper fraction then cancel the 2s and then simplifying the division.
4 \(\frac{1}{2}\)(\(\frac{2}{3}\)) = \(\frac{9}{2}\) × \(\frac{2}{3}\) = 3
Thus the correct answer is option A.

Question 13.
During a sale, the price of a sweater was changed from $20 to $16. What was the percent of decrease in the price of the sweater?
Options:
a. 4%
b. 20%
c. 25%
d. 40%

Answer:

Mini-Task

Question 14.
Max wants to buy some shorts that are priced at $8 each. He decided to buy a pair of sneakers for $39, but the total cost of the shorts and the sneakers must be less than $75.
a. Write an inequality to find out how many pairs of shorts Max can buy.
Type below:
____________

Answer: 39 + 8x < 75

Explanation:
Let x be the number of shorts he buys then 8x is the total cost of the shorts and 8x + 39 is the total cost of the shorts and sneakers his total must be less than $75 so the inequality is <.
39 + 8x < 75

Question 14.
b. Suppose that Max wants to buy 6 pairs of shorts. Will he have enough money? Explain.
______

Answer: No

Explanation:
Find the total amount he will spend buying 6 pairs of shorts this is more than the $75 he has so he will not have enough.
39 + 8(6) = 39 + 48 = 87

Question 14.
c. Solve the inequality to find the greatest number of pairs of shorts that Max can buy. Show your work.
______ pairs of shoes

Answer: 4 pairs of shoes

Explanation:
Use the above inequality,
39 + 8x < 75
Subtract 39 on both sides and then divide both sides by 8. Since you can’t buy a fraction of a pair of shorts the most pairs he can buy is 4.
39 + 8x – 39 < 75 – 39
8x < 36
x < 4.5

Summary:

We wish the solutions given in this article are satisfactory for all the students of the 7th class. This Go Math Answer Key of Grade 7 helps teachers to know the easy tricks to explain the problems to the students. Feel free to clarify your doubts by posting your questions in the below comment box. We will answer them as early as possible.

Go Math Grade 7 Answer Key Chapter 4 Rates and Proportionality

go-math-grade-7-chapter-4-rates-and-proportionality-answer-key

Students who are in search of the HMH Go Math Chapter 4 Rates and Proportionality Answer Key can get them on this page. With the help of the Go Math Grade 7 Answer Key 4th Chapter Rates and Proportionality parents can teach simple methods to solve the problems to their children. So, Download Go Math Grade 7 Rates and Proportionality Chapter 4 pdf for free of cost.

Go Math Grade 7 Answer Key Chapter 4 Rates and Proportionality

It is important for the students to learn the concepts given in Go Math Grade 7 Chapter 4 Answer Key to score the highest marks in the exams. The quick way of solving the problems will help you to save time in the exam. The topics covered in this chapter are Unit rates, Constant Rates of Change, Proportional Relationships, and Graphs. Click on the below-provided links and go through all the questions and answers.

Chapter 4 – Unit Rates

Chapter 4 – Constant Rates of Change

Chapter 4 – Proportional Relationships and Graphs

Chapter 4 – Module 4

Unit Rates – Guided Practice – Page No. 120

Question 1.
Brandon enters bike races. He bikes 8 \(\frac{1}{2}\) miles every \(\frac{1}{2}\) hour. Complete the table to find how far Brandon bikes for each time interval
Go Math Grade 7 Answer Key Chapter 4 Rates and Proportionality Lesson 1: Unit Rates img 1
Type below:
____________

Answer:
1 hour: 8 \(\frac{1}{2}\) + 8 \(\frac{1}{2}\) = 17
1 \(\frac{1}{2}\) hour: 17 + 8 \(\frac{1}{2}\) = 25 \(\frac{1}{2}\)
2 hour: 25 \(\frac{1}{2}\) + 8 \(\frac{1}{2}\) = 34
2 \(\frac{1}{2}\) hour: 34 + 8 \(\frac{1}{2}\) = 42 \(\frac{1}{2}\)

Go-Math-Grade-7-Answer-Key-Chapter-4-Rates-and-Proportionality-img-1

Find each unit rate.

Question 2.
Julio walks 3 \(\frac{1}{2}\) miles in 1 \(\frac{1}{4}\) hours.
________ \(\frac{□}{□}\)

Answer: 2 \(\frac{4}{5}\)

Explanation:
Divide the number of miles by the number of hours to find the unit rate in miles per hour.
3 \(\frac{1}{2}\) ÷ 1 \(\frac{1}{4}\) = \(\frac{7}{2}\)/\(\frac{5}{4}\)
\(\frac{7}{2}\) × \(\frac{4}{5}\) = \(\frac{14}{5}\)
Convert from improper fraction to the mixed fraction.
\(\frac{14}{5}\) = 2 \(\frac{4}{5}\) miles per hour

Question 3.
Kenny reads \(\frac{5}{8}\) page in \(\frac{2}{3}\) minute.
________ \(\frac{□}{□}\)

Answer: \(\frac{15}{16}\)

Explanation:
Divide the number of pages by the number of minutes to find the unit rate in pages per minute.
\(\frac{5}{8}\) ÷ \(\frac{2}{3}\)
= \(\frac{5}{8}\) × \(\frac{3}{2}\) = \(\frac{15}{16}\)
Thus Kenny read \(\frac{15}{16}\) pages per minute.

Question 4.
A garden snail moves \(\frac{1}{6}\) foot in \(\frac{1}{3}\) hour.
\(\frac{□}{□}\)

Answer: \(\frac{1}{2}\)

Explanation:
Given that,
A garden snail moves \(\frac{1}{6}\) foot in \(\frac{1}{3}\) hour.
Divide the number of feet by the number of hours to find the unit rate in feet per hour.
\(\frac{1}{6}\) ÷ \(\frac{1}{3}\)
\(\frac{1}{6}\) × \(\frac{3}{1}\) = \(\frac{1}{2}\)
Therefore A garden snail moves \(\frac{1}{2}\) feet per hour.

Question 5.
A fertilizer covers \(\frac{5}{8}\) square foot in \(\frac{1}{4}\) hour.
________ \(\frac{□}{□}\)

Answer: 2 \(\frac{1}{2}\) square feet per hour

Explanation:
Given,
A fertilizer covers \(\frac{5}{8}\) square foot in \(\frac{1}{4}\) hour.
Divide the number of square feet, which is \(\frac{5}{8}\) by the number of hours, which is \(\frac{1}{4}\), to find the unit rate in square feet per hour.
\(\frac{5}{8}\) ÷ \(\frac{1}{4}\)
\(\frac{5}{8}\) × \(\frac{4}{1}\) =\(\frac{5}{2}\)
Convert from improper fraction to the mixed fraction.
\(\frac{5}{2}\) = 2 \(\frac{1}{2}\) square feet per hour
Thus A fertilizer covers 2 \(\frac{1}{2}\) square feet per hour.

Find each unit rate. Determine which is lower.

Question 6.
Brand A: 240 mg sodium for \(\frac{1}{3}\) pickle or Brand B: 325 mg sodium for \(\frac{1}{2}\) pickle.
____________

Answer:
Find the unit rates in mg per pickle for each brand by dividing the number of mg by the number of pickles.
Brand A: 240 mg ÷ \(\frac{1}{3}\) = 240 × 3 = 720
Brand B: 325 mg ÷ \(\frac{1}{2}\) = 325 × 2 = 650
650 is less than 720 so Brand B has a lower unit rate.

Question 7.
Ingredient C: \(\frac{1}{4}\) cup for \(\frac{2}{3}\) serving or Ingredient D: \(\frac{1}{3}\) cup for \(\frac{3}{4}\) serving.
____________

Answer: Find the unit rates in cups per serving by dividing the number of cups by the number of servings.
Ingredient C: \(\frac{1}{4}\) ÷ \(\frac{2}{3}\)
\(\frac{1}{4}\) × \(\frac{3}{2}\) = \(\frac{3}{8}\)
Ingredient D: \(\frac{1}{3}\) ÷ \(\frac{3}{4}\)
\(\frac{1}{3}\) × \(\frac{4}{3}\) = \(\frac{4}{9}\)
\(\frac{3}{8}\) is less than \(\frac{4}{9}\), ingredient C has a lower unit rate.

Essential Question Check-In

Question 8.
How can you find a unit rate when given a rate?
Type below:
____________

Answer: To find a unit rate when given a rate such as 25 miles per 5 minutes, divide the first quantity by the second quantity.
In the example I gave, this would mean the unit rate is 25 ÷ 5 = 5 miles per minute.

Unit Rates – Independent Practice – Page No. 121

Go Math Grade 7 Answer Key Chapter 4 Rates and Proportionality Lesson 1: Unit Rates img 2

Question 9.
The information for two pay-as-you-go cell phone companies is given.
a. What is the unit rate in dollars per hour for each company?
On Call: ____________ dollars per hour
Talk Time: ____________ dollars per hour

Answer:
Divide the cost by the number of hours for each company to find the unit rates.
On Call: 10 ÷ 3.5 = 2 \(\frac{6}{7}\) ≈ 2.86
Talk Time: 1.25 ÷ \(\frac{1}{2}\) = 2.50

Question 9.
b. Analyze Relationships
Which company offers the best deal? Explain your answer.
____________

Answer: Talk time has the lowest unit rate so it offers the best deal.

Question 9.
c. What If?
Another company offers a rate of $0.05 per minute. How would you find the unit rate per hour?
____________ dollars per hour

Answer:
Since there are 60 minutes in 1 hour, $.0.05 per minute is
60 × 0.05 = $3 per hour.
Thus the unit rate per hour is $3.

Question 9.
d. Draw Conclusions
Is the rate in part c a better deal than On Call or Talk Time? Explain.
____________

Answer:
The unit rate in part c is greater than the unit rates from part a so it is not a better deal than the other two companies.

Question 10.
Represent Real-World Problems
Your teacher asks you to find a recipe that includes two ingredients with a rate of \(\frac{2 \text { units }}{3 \text { units }}\).
a. Give an example of two ingredients in a recipe that would meet this requirement.
Type below:
____________

Answer: A rate of 2/3 units means that there need to be 2 units of 1 ingredient for every 3 units of a second ingredient.
One example could then be 2 eggs per 3 cups of flour.
Another example could 2 teaspoons of vanilla per 3 teaspoons of sugar.

Question 10.
b. If you needed to triple the recipe, would the rate change? Explain.
____________

Answer: No, the rate would not change. Using the example I gave in
part a) of 2 eggs per 3 cups of flour, tripling the recipe would require using 3(2 eggs) = 6 eggs
3(3 cups of sugar) = 9 cups of flour.
Since 6 eggs/9 cups of flour = 2 eggs/3 cups of flour, the rate is still the same.

Question 11.
A radio station requires DJs to play 2 commercials for every 10 songs they play. What is the unit rate of songs to commercials?
____________ songs per commercials

Answer: 10 ÷ 2 = 5
Divide the number of songs by the number of commercials.
Thus the radio requires 5 songs per commercial.

Question 12.
Multistep
Terrance and Jesse are training for a long-distance race. Terrance trains at a rate of 6 miles every half hour and Jesse trains at a rate of 2 miles every 15 minutes.
a. What is the unit rate in miles per hour for each runner?
Terrance: ____________ mi per hour
Jesse: ____________ mi per hour

Answer:
Find the unit rates for each runner by dividing the number of miles by the number of hours. Remember that 15 minutes is 1/4 of an hour since there are 60 minutes in an hour
15 ÷ 60 = 1/4
a) Terrance: 6 ÷ 1/2 = 6 × 2 = 12 miles per hour.
Jesse: 2 ÷ 1/4 = 2 × 4 = 8 miles per hour.

Question 12.
b. How long will each person take to run a total of 50 miles at the given rates?
Terrance: ______ \(\frac{□}{□}\)
Jesse: ______ \(\frac{□}{□}\)

Answer:
Divide the number of miles by the unit rates found in part a to find the time.
Terrance: 50 ÷ 12 = 50/12 = 4 \(\frac{1}{6}\) hours
Jesse: 50 ÷ 8 = 50/8 = 6 \(\frac{1}{4}\) hours

Question 12.
c. Sandra runs at a rate of 8 miles in 45 minutes. How does her unit rate compare to Terrance’s and to Jesse’s?
______ \(\frac{□}{□}\) mi per hour

Answer:
We need to find the unit rate for Sandra by dividing the number of miles by the number of hours.
Remember that 45 minutes is 3/4 of an hour.
Since 45/60 = 3/4.
Sandra’s unit rate is smaller than Terrance’s but larger than Jesse’s.

Unit Rates – Page No. 122

Question 13.
Analyze Relationships
Eli takes a typing test and types all 300 words in \(\frac{1}{10}\) hour. He takes the test a second time and types the words in \(\frac{1}{12}\) hour. Was he faster or slower on the second attempt? Explain.
____________

Answer:
We need to find the unit rates for each time he took the test.
1st time: 300 ÷ 1/10 = 300 × 10 = 3000
2nd time: 300 ÷ 1/12 = 300 × 12 = 3600
The second time has a faster unit rate since 3600 > 3000.

Question 14.
Justify Reasoning
An online retailer sells two packages of protein bars.
Go Math Grade 7 Answer Key Chapter 4 Rates and Proportionality Lesson 1: Unit Rates img 3
a. Which package has the better price per bar?
____________

Answer:
Find the unit rates per bar by dividing the costs by the number of bars. the 12 pack has the better price per bar.
10-pack: 15.37 ÷ 10 = 1.537 ≈ 1.54
12-pack: 15.35 ÷ 12 ≈ 1.30

Question 14.
b. Which package has the better price per ounce?
____________

Answer:
First, find the total number of ounces by multiplying the number of bars times the number of ounces per bar. then find the unit rates per ounce by dividing the costs by the total number of ounces the 10-pack has the better price per ounce.
10-pack: 10 × 2.1 = 21 ounces
12-pack: 12 × 1.4 = 16.8 ounces
10-pack: 15.37 ÷ 21 ≈ 0.73
12-pack: 15.35 ÷ 16.8 ≈ 0.91

Question 14.
c. Which package do you think is a better buy? Justify your reasoning.
____________

Answer:
The 10-pack is a better deal since the price per ounce is a better measure to use than price per bar. The number of bars doesn’t tell you how you are actually buying since the bars can be very small meaning the number of ounces you are actually buying is small.

Question 15.
Check for Reasonableness
A painter painted about half a room in half a day. Coley estimated the painter would paint 7 rooms in 7 days. Is Coley’s estimate reasonable? Explain.
____________

Answer:
If a painter can paint half a room in a half day, then he can paint 1 room in 1 day.
This would be equivalent to painting 7 rooms in 7 days so his estimate is reasonable.
7 rooms ÷ 7 days = 1/2 room ÷ 1/2 days

Question 16.
Communicate Mathematical
Ideas If you know the rate of a water leak in gallons per hour, how can you find the number of hours it takes for 1 gallon to leak out? Justify your answer.
Type below:
____________

Answer: If you know the rate in gallons per hour, then the rate in hours per gallon is the reciprocal of the rate in gallons per hour.
Example:
If water is leaking at a rate of 5 gallons per hour, then it is leaking at 1/5 hour per gallon.

Constant Rates of Change – Guided Practice – Page No. 126

Question 1.
Based on the information in the table, is the relationship between time and the number of words typed a proportional relationship?
Go Math Grade 7 Answer Key Chapter 4 Rates and Proportionality Lesson 2: Constant Rates of Change img 4
The relationship ____________ proportional

Answer: is proportional

Explanation:
Since 45 ÷ 1 = 45, 90 ÷ 2 = 45, 135 ÷ 3 = 45 and 180 ÷ 4 = 45, the relationship is proportional.
Thus the relationship for the above table is proportional.

Find the constant of proportionality k. Then write an equation for the relationship between x and y.

Question 2.
Go Math Grade 7 Answer Key Chapter 4 Rates and Proportionality Lesson 2: Constant Rates of Change img 5
k = _______

Answer: 5

Explanation:
The equation is of the form y = kx so k = y/x.
Substituting values of x and y from the table gives k = 10/2 = 5.
Plugging this value into y = kx gives the equation y = 5x.
The relationship between x and y is y = 5x.

Question 3.
Go Math Grade 7 Answer Key Chapter 4 Rates and Proportionality Lesson 2: Constant Rates of Change img 6
k = \(\frac{□}{□}\)

Answer: k = \(\frac{1}{4}\)

Explanation:
The equation is of the form y = kx so k = y/x.
Substituting values of x and y from the table gives k = 2/8 = \(\frac{1}{4}\).
Plugging this value into y = kx gives the equation y = \(\frac{1}{4}\)x.

Essential Question Check-In

Question 4.
How can you represent a proportional relationship using an equation?
Type below:
____________

Answer: y = kx

Explanation:
A proportional relationship can always be represented by an equation of the form y = kx
where ks is the constant of proportionality and represents the rate of the change in the y quantity in relation to the x quantity.

Constant Rates of Change – Page No. 127

Information on three car-rental companies is given.
Go Math Grade 7 Answer Key Chapter 4 Rates and Proportionality Lesson 2: Constant Rates of Change img 7

Question 5.
Write an equation that gives the cost y of renting a car for x days from Rent-All.
y = _______ x

Answer: 18.50

Explanation:
Find the constant of proportionality by dividing the total costs by the number of days.
k = 55.50/3 = 18.50
The equation is y = 18.50x

Question 6.
What is the cost per day of renting a car from A-1?
$ _______ per day

Answer: $21.98

Explanation:
Since the cost of each half day is $10.99, the cost for each day is 2 × 10.99 = 21.98
The cost per day of renting a car from A-1 is $21.98

Question 7.
Analyze Relationships
Which company offers the best deal? Why?
The company that offers the best deal is ____________

Answer: Rent all

Explanation:
The costs per day were $18.50 for Rent-All, $21.98 for A-1 Rentals, and $19.25 for Car Town so Reant All offers the best deal since it offers the lowest cost per day.
Thus the company that offers the best deal is Rent-All.

Question 8.
Critique Reasoning
A skydiver jumps out of an airplane. After 0.8 second, she has fallen 100 feet. After 3.1 seconds, she has fallen 500 feet. Emtiaz says that the skydiver should fall about 187.5 feet in 1.5 seconds. Is his answer reasonable? Explain.
_______

Answer: No. He assumed the rate of descent was proportional but the rate is increasing as time increases.

Explanation:
Since 100 ÷ 0.8 = 125 the skydiver fell at a speed of 125 ft per second for the first 0.8 second.
Since 500 ÷ 3.1 = 161, the skydiver fell at a speed of about 161 ft per second for the first 3.1 seconds.
The rate of descent is then increased as time increases and is not proportional since 125 ≠ 161.
Since 187.5 ÷ 1.5 = 125, he assumed the rate of descent was proportional. His estimate is then not reasonable. The actual rate of descent should be between 125 and 161.3 since 1.5 seconds is between 0.8 and 3.1 seconds.

Steven earns extra money babysitting. He charges $31.25 for 5 hours and $50 for 8 hours.

Question 9.
Explain why the relationship between how much Steven charges and time is a proportional relationship.
Type below:
____________

Answer: The relationship is proportional since the ratios are equal.

Explanation:
Since 31.25 ÷ 5 = 6.25 and 50 ≈ 8 = 6.25 the relationship is proportional since the ratios are equal.

Question 10.
Interpret the Answer
Explain what the constant rate of change means in the context.
Type below:
____________

Answer: The rate of change means he charges $6.25.

Explanation:
The constant rate of change of 6.25 means he charges $6.25 per hour since the rate was found by dividing the charge by the number of hours.
The rate of change means he charges $6.25.

Question 11.
Write an equation to represent the relationship. Tell what the variables represent.
Type below:
____________

Answer: The equation is y = 6.25x
where x is the number of hours and ys is the total charge.

Explanation:
The rate of change is 6.25 so k = 6.25.
This gives an equation of y = 6.25x where x is the number of hours and y is the total charge.

Question 12.
How much would Steven charge for 3 hours?
$ _______

Answer: $18.75

Explanation:
y = 6.25 × 3 = 18.75
Thus Steve charges $18.75 for 3 hours.

Constant Rates of Change – Page No. 128

A submarine dives 300 feet every 2 minutes, and 6,750 feet every 45 minutes.

Question 13.
Find the constant rate at which the submarine dives. Give your answer in feet per minute and in feet per hour.
____________ feet per minute
____________ feet per hour

Answer: 150 feet per minute, 9000 feet per hour

Explanation:
Since 300 ÷ 2 = 150, the submarine is diving at 150 feet per minute.
Since 45 minutes = 3/4 of an hour and 6750 ÷ 3/4 = 9000, the submarine is diving at a rate of 9000 feet per hour.

Question 14.
Let x represent the time of the dive. Let y represent the depth of the submarine. Write an equation for the proportional relationship using the rate in feet per minute.
Type below:
____________

Answer: y = 150x

Explanation:
The submarine was diving at a rate of 150 feet per minute so k = 150.
The equation is then y = 150x.

Question 15.
Draw Conclusions
If you wanted to find the depth of a submarine during a dive, would it be more reasonable to use an equation with the rate in feet per minute or feet per hour? Explain your reasoning.
____________

Answer: Feet per minute

Explanation:
Since a submarine would only dive for a few minutes at a time and not dive for hours at a time, it is more reasonable to use the rate in feet per minute.

H.O.T.

Focus on Higher Order Thinking

Question 16.
Make a Conjecture
There is a proportional relationship between your distance from a thunderstorm and the time from when you see lightning and hear thunder. If there are 9 seconds between lightning and thunder, the storm is about 3 kilometers away. If you double the amount of time between lightning and thunder, do you think the distance in kilometers also double? justify your reasoning.
_______

Answer: Yes the distance will also double. If the relationship is proportional then distance/time = k
where k is the constant of proportionality.
Since the time was 9 seconds for 3km, then for 18 seconds the distance would be 6 km since 3/9 = 6/18.
6 is double 3 so the distance doubles when the time doubles.

Question 17.
Communicate Mathematical Ideas
A store sells 3 ears of corn for $1. They round prices to the nearest cent as shown in the table. Tell whether you would describe the relationship between cost and number of ears of corn as a proportional relationship. Justify your answer.
Go Math Grade 7 Answer Key Chapter 4 Rates and Proportionality Lesson 2: Constant Rates of Change img 8
_______

Answer:
Since 0.33 ÷ 1 = 0.33, 0.67 ÷ 2 = 0.335, 1.00 ÷ 3 = 0.33.., 1.34 ÷ 4 = 0.335, the relationship is approximately proportional since all the ratios are approximately equal. The difference in the ratios com from rounding the amount charged to the nearest cent.

Question 18.
Jack is 12 and his sister Sophia is 16. Jack says that the relationship between his age and Sophia’s age is proportional and the constant of proportionality is \(\frac{12}{16}\) Do you agree? Explain.
____________

Answer:
Given that current age of Jack = 12 years
Given that current age of Sophia = 16 years
Jack says that the relationship between his age and Sophia‘s is proportional
If Jack’s age is represented by y and Sophia’s age by x then we can write y=kx as they are in proportion
where k is called constant of proportion
Now let’s plug given ages of each that is y=12 and x=16 into y=kx to find the constant of proportionality
12=k×16
12/16=k
Which is the same as the given value of the constant of proportionality?
Hence Jack is right about his statement.
But if you think about practical life situation then the age of both will not be in proportion
For example, after 1 year Jack’s age will be 13 and Sophie’s age will be 17
then constant of proportionality using new values will be 13/17
Clearly 12/16 and 13/17 are not same.
So in practical life, the age of both will not in proportion.

Question 19.
Luke’s turkey chili recipe calls for 1.5 pounds of ground turkey for every 6 servings. How many servings can he make if he has 5 pounds of ground turkey? Show your work.
____________ servings

Answer: 20 servings

Explanation:
Given,
Luke’s turkey chili recipe calls for 1.5 pounds of ground turkey for every 6 servings.
So if 1.5 pounds of turkey can get you 6 servings
1.5 = 6
3 = 12
4.5 = 18
5 =?
to find what 5 pounds are equal to we must do 1.5/6 to find the unit rate of 0.25. We then add 0.25 to 18.25 servings or if you round you can get about 18 servings.
18 + 0.25 × 8 = 18 + 2 = 20

Proportional Relationships and Graphs – Guided Practice – Page No. 132

Complete each table. Tell whether the relationship is a proportional relationship. Explain why or why not.

Question 1.
A student reads 65 pages per hour.
Go Math Grade 7 Answer Key Chapter 4 Rates and Proportionality Lesson 3: Proportional Relationships and Graphs img 9
____________

Answer:
Given that,
A student reads 65 pages per hour.
3 hours: 3 × 65 = 195 pages
5 hours: 5 × 65 = 325 pages
10 hours: 10 × 65 = 650 pages
We need to find the number of hours for 585 pages by dividing the number of pages by 65 since the students read 65 pages per hour:
585 pages: 585 ÷ 65 = 9 hours
Go-Math-Grade-7-Answer-Key-Chapter-4-Rates-and-Proportionality-img-9
A relationship is proportional if the quotient of each ordered pair is constant. Since the student is reading at a constant rate of 65 pages per hour, and the quotient of each ordered pair in the table is 65, the relationship is proportional.

Question 2.
A babysitter makes $7.50 per hour.
Go Math Grade 7 Answer Key Chapter 4 Rates and Proportionality Lesson 3: Proportional Relationships and Graphs img 10
____________

Answer:
2 hours = 2 × 7.50 = 15
22.50 = 22.50 ÷ 7.50 = 3 hours
5 hours = 5 × 7.50 = 37.50
6. = 60 ÷ 7.50 = 8 hours
Go-Math-Grade-7-Answer-Key-Chapter-4-Rates-and-Proportionality-img-10

Tell whether the relationship is a proportional relationship. Explain why or why not.

Question 3.
Go Math Grade 7 Answer Key Chapter 4 Rates and Proportionality Lesson 3: Proportional Relationships and Graphs img 11
____________

Answer:
The relationship has the points (2, 4) and (8, 10).
Since 4 ÷ 2 = 2 and 10 ÷ 8 = 1.25, the relationship is not proportional since the ratios are not equal.

Question 4.
Go Math Grade 7 Answer Key Chapter 4 Rates and Proportionality Lesson 3: Proportional Relationships and Graphs img 12
____________

Answer:
The relationship appears to be proportional since the points appear to form a line that goes through the origin. That line would go through the points (1,2), (2, 4), (5, 10), and (8, 16).
Since 2 ÷ 1 = 2, 4 ÷ 2 = 2, 10 ÷ 5 = 2, and 16 ÷ 8 = 2, the relationship is proportional since all the ratios are equal.

Write an equation of the form y = kx for the relationship shown in each graph.

Question 5.
Go Math Grade 7 Answer Key Chapter 4 Rates and Proportionality Lesson 3: Proportional Relationships and Graphs img 13
y = ____________ x

Answer: y = 3.5x

Explanation:
One of the points is (8, 28) so k = 28/8 = 7/2. The equation is the y = 7/2 = 3.5x

Question 6.
Go Math Grade 7 Answer Key Chapter 4 Rates and Proportionality Lesson 3: Proportional Relationships and Graphs img 14
y = ____________ x

Answer: y = 0.25x

Explanation:
One of the points is (8,2) so k = 2/8 = 1/4.
The equation is the ya = 1/4x = 0.25 x.

Essential Question Check-In

Question 7.
How does a graph show a proportional relationship?
Type below:
____________

Answer:
A proportional relationship between two variables, x and y, exists if y = kx.
This equation is a line that passes through the origin and has a slope of k. The slope can be positive or negative. Therefore is the points lie on a line that goes through the origin, the graph shows a proportional relationship. If the points lie on a line that does not go through the origin, that is, has a non zero y-intercept, then the relationship is not proportional.

Proportional Relationships and Graphs – Guided Practice – Page No. 133

For Exercises 8–12, the graph shows the relationship between time and distance run by two horses.
Go Math Grade 7 Answer Key Chapter 4 Rates and Proportionality Lesson 3: Proportional Relationships and Graphs img 15

Question 8.
Explain the meaning of the point (0,0).
Type below:
____________

Answer: The point (0, 0) represents a distance of 0 miles in 0 min.

Question 9.
How long does it take each horse to run a mile?
Horse A runs 1 mile in ____________ minutes
Horse B runs 1 mile in ____________ minutes

Answer: Horse A: 4 min per mile.
Horse B: 2.5 min per mile

Explanation:
Line A goes through the point (8, 2) which gives a rate of 8/2 = 4 min per mile. Line B goes through the point (10, 4) which gives a rate of 10/4 = 2.5 min per mile.

Question 10.
Multiple Representations
Write an equation for the relationship between time and distance for each horse.
For Horse A : y = ____________ x
For Horse B : y = ____________ x

Answer: A: y = 1/4x, B: y = 2/5x

Explanation:
The graph has x representing the time in minutes and y representing the distance in miles so the slope of the line has units of miles per minute.
Since horse A runs 4 min per mile, it runs at a rate of 1/4 mi per min.
This gives the equation y = 1/4x.
Since horse B runs at a rate of 2.5 min per mi, it runs at a rate of 1/2.5 = 2/5 miles per min.
The equation is then y = 2/5x.

Question 11.
Draw Conclusions
At the given rates, how far would each horse run in 12 minutes?
Horse A will run ____________ miles
Horse B will run ____________ miles

Answer: Horse A will run 3 miles.
Horse B will run 4.8 miles.

Explanation:
Multiply the rates in mi per min from the above problem by the number of minutes.
A: 1/4 mi per min × 12 min = 3 miles
B: 2/5 mi per min × 12 min = 4.8 miles

Question 12.
Analyze Relationships
Draw a line on the graph representing a horse than runs faster than horses A and B.
Type below:
____________

Answer:
To have a line representing at the rate of the horse faster than horses A and B, the line should be a little bit steeper than the other two lines. This can be represented in the graph as:
Go-Math-Grade-7-Answer-Key-Chapter-4-img-1

Question 13.
A bullet train can travel at 170 miles per hour. Will a graph representing the distance in miles compared to the time in hours show a proportional relationship? Explain.
____________

Answer: Yes
Since the train is travelling at a constant rate, a graph representing distance in miles compared to time in hours will show a proportional relationship.

Question 14.
Critical Thinking
When would it be more useful to represent a proportional relationship with a graph rather than an equation?
Type below:
____________

Answer: It is more useful to represent a proportional relationship with a graph when comparing different and various situations.

Question 15.
Multiple Representations
Bargain DVDs cost $5 each at Mega Movie.
a. Graph the proportional relationship that gives the cost y in dollars of buying x bargain DVDs.
Go Math Grade 7 Answer Key Chapter 4 Rates and Proportionality Lesson 3: Proportional Relationships and Graphs img 16
Type below:
____________

Answer:
Since each DVD is $5 each, make sure to graph a line that corresponds to this rate.
Go-Math-Grade-7-Answer-Key-Chapter-4-Rates-and-Proportionality-img-16

Question 15.
b. Give an ordered pair on the graph and explain its meaning in the real world context.
Type below:
____________

Answer: An ordered pair in the graph is (3, 15) and this means that three DVDs cost $15.

Proportional Relationships and Graphs – Page No. 134

The graph shows the relationship between distance and time as Glenda swims.
Go Math Grade 7 Answer Key Chapter 4 Rates and Proportionality Lesson 3: Proportional Relationships and Graphs img 17

Question 16.
How far did Glenda swim in 4 seconds?
______ feet

Answer: 8 ft

Explanation:
The graph goes through the point (4, 8) so she swam 8 ft in 4 sec.

Question 17.
Communicate Mathematical Ideas
Is this a proportional relationship? Explain your reasoning.
____________

Answer: The relationship is proportional since the points lie on a line and the line connecting the points goes through the origin.

Question 18.
Multiple Representations
Write an equation that shows the relationship between time and distance.
Type below:
____________

Answer: y = 2x

Explanation:
The line goes through the point (4, 8) which means k = 8/4 = 2.
This give an equation of y = 2x.

H.O.T.

Focus on Higher Order Thinking

Question 19.
Make a Conjecture
If you know that a relationship is proportional and are given one ordered pair that is not (0,0), how can you find another pair?
Type below:
____________

Answer:
If you are given a point (a, b) that is not (0, 0) and that the relationship is proportional, then you can find k since k = y/x = b/a.
Then you can write the equation as y = b/ax. From there, you can plug in any value for x to find the corresponding y-coordinate.

The tables show the distance traveled by three cars.
Go Math Grade 7 Answer Key Chapter 4 Rates and Proportionality Lesson 3: Proportional Relationships and Graphs img 18

Question 20.
Communicate Mathematical Ideas
Which car is not traveling at a constant speed? Explain your reasoning.
____________

Answer:
Since 120 ÷ 2 = 180 ÷ 3 = 300 ÷ 5 = 360 ÷ 6 = 60, Car 1 is travelling at a constant speed.
Since 200 ÷ 5 = 400 ÷ 10 = 600 ÷ 15 = 800 ÷ 20 = 40, Car 2 is travelling at a constant speed.
Since 65 ÷ 1 ≠ 85 ÷ 2, Car 3 is not traveling at a constant speed.

Question 21.
Make a Conjecture
Car 4 is traveling at twice the rate of speed of car 2. How will the table values for car 4 compare to the table values for car 2?
Type below:
____________

Answer:
From problem 20, car 2 is traveling at 40 miles per hour. If car 4 is traveling twice that rate, then it is traveling at 80 miles per hour. This means all the values for the distances for car 4 will be double the values for the distances for car 2.

MODULE QUIZ – 4.1 Unit Rates – Page No. 135

Find each unit rate. Round to the nearest hundredth, if necessary.

Question 1.
$140 for 18 ft2
$ ______

Answer: $7.78 per ft2

Explanation:
Divide the cost of $140 by the number of square feet, 18 sq. ft, using a calculator:
140 ÷ 18 ≈ $7.78 per sq. ft.
If you are required by your teacher to do the division by hand, divide to three decimal points as shown below:
Since the dollar amounts must be rounded to two decimal places, then $7.77.. ≈ $7.78 so the cost per square foot is $7.78 sq. ft.

Question 1.
14 lb for $2.99
$ ______

Answer: $0.21 per lb

Explanation:
Divide the cost by the number of pounds.
2.99 ÷ 14 = $0.21

Circle the better deal in each pair. Then give the unit rate for the better deal.

Question 3.
\(\frac{\$ 56}{25 \text { gal }} or \frac{\$ 32.05}{15 \text { gal }}\)
____________

Answer:
Better deal: $32.05/15 gal
Unit rate: $2.14 per gal

Explanation:
Find the unit rates by dividing the cost by the number of gallons:
$56/25 gal = $2.24 per gal
$32.05/15 gal ≈ $2.14 per gal
Since 2.14 < 2.24, then the better deal is $32.25/15 gal.

Question 4.
\(\frac{\$ 160}{5 \mathrm{g}} \text { or } \frac{\$ 315}{9 \mathrm{g}}\)
____________

Answer:
Better deal: $160/5g
Unit rate: $32 per gram

Explanation:
Find the unit rates by dividing the cost by the number of grams:
$160/5 g = $32 per gram
$315/9 g = $35 per gram
Since 32 < 35, then the better deal is $160/5 g.

4.2 Constant Rates of Change

Question 5.
The table shows the amount of money Tyler earns for mowing lawns. Is the relationship a proportional relationship? Why or why not?
Go Math Grade 7 Answer Key Chapter 4 Rates and Proportionality MODULE QUIZ img 19
____________

Answer: not proportional

Explanation:
Since 15 ÷ 1 = 15 but 48 ÷ 3 = 16, the relationship is not proportional.

Question 6.
On a recent day, 8 euros were worth $9 and 24 euros were worth $27. Write an equation of the form y = kx to show the relationship between the number of euros and the value in dollars.
Type below:
____________

Answer: y = \(\frac{9}{8}\)x

Explanation:
k = \(\frac{value in dollars}{number of euros}\) = \(\frac{9}{8}\)
so, the equation is y = \(\frac{9}{8}\)x

4.3 Proportional Relationships and Graphs

Question 7.
The graph shows the number of servings in different amounts of frozen yogurt listed on a carton. Write an equation that gives the number of servings y in x pints.
Go Math Grade 7 Answer Key Chapter 4 Rates and Proportionality MODULE QUIZ img 20
Type below:
____________

Answer: y = \(\frac{5}{2}\)x

Explanation:
The graph goes through the point (2, 5) so k = \(\frac{5}{2}\).
This gives an equation of y = \(\frac{5}{2}\)x

Question 8.
A refreshment stand makes 2 large servings of frozen yogurt from 3 pints. Add the line to the graph and write its equation.
Type below:
____________

Answer: y = \(\frac{2}{3}\)x

Explanation:
If the situation states that 2 servings of frozen yogurt can be made from 3 pints, then we can say that k = \(\frac{2}{3}\), and therefore the equation of the line is y = \(\frac{2}{3}\)x. The graph of the line is shown below.
Go-Math-Grade-7-Answer-Key-Chapter-4-img-2

Essential Question

Question 9.
How can you use rates to determine whether a situation is a proportional relationship?
Type below:
____________

Answer: If the rate is constant, then the situation is a proportional relationship. If the rate is not constant, the situation cannot be a proportional relationship.

MIXED REVIEW – Selected Response – Page No. 136

Question 1.
Kori spent $46.20 on 12 gallons of gasoline. What was the price per gallon?
Options:
a. $8.35
b. $3.85
c. $2.59
d. $0.26

Answer: $3.85

Explanation:
Given that,
Kori spent $46.20 on 12 gallons of gasoline.
Divide the cost by the number of gallons to find the price per gallon.
46.20/12 = 3.85
Thus the correct answer is option B.

Question 2.
A rabbit can run short distances at a rate of 35 miles per hour. A fox can run short distances at a rate of 21 miles per half hour. Which animal is faster, and by how much?
Options:
a. The rabbit; 7 miles per hour
b. The fox; 7 miles per hour
c. The rabbit; 14 miles per hour
d. The fox; 14 miles per hour

Answer: The fox; 7 miles per hour

Explanation:
Given that,
A rabbit can run short distances at a rate of 35 miles per hour. A fox can run short distances at a rate of 21 miles per half hour.
If a fox runs 21 miles for half an hour then it can 42 miles per hour.
42 – 35 = 7 miles per hour
The fox is faster by 7 miles per hour.
Therefore the correct answer is option B.

Question 3.
A pet survey found that the ratio of dogs to cats is 25. Which proportion shows the number of dogs if the number of cats is 140?
Options:
a. \(\frac{2 \mathrm{dogs}}{5 \mathrm{cats}}=\frac{140 \mathrm{dogs}}{350 \mathrm{cats}}\)
b. \(\frac{2 \mathrm{dogs}}{5 \mathrm{cats}}=\frac{140 \mathrm{cats}}{350 \mathrm{dogs}}\)
c. \(\frac{2 \mathrm{dogs}}{5 \mathrm{cats}}=\frac{28 \mathrm{dogs}}{140 \mathrm{cats}}\)
d. \(\frac{2 \mathrm{dogs}}{5 \mathrm{cats}}=\frac{56 \mathrm{dogs}}{140 \mathrm{cats}}\)

Answer: \(\frac{2 \mathrm{dogs}}{5 \mathrm{cats}}=\frac{56 \mathrm{dogs}}{140 \mathrm{cats}}\)

Explanation:
Given,
A pet survey found that the ratio of dogs to cats is 25.
Since 5 × 25 = 140 and
2 × 28 = 56
= 56/140
Thus the correct answer is option D.

Question 4.
What is the cost of 2 kilograms of flour if 3 kilograms cost $4.86 and the unit price for each package of flour is the same?
Options:
a. $0.81
b. $2.86
c. $3.24
d. $9.72

Answer: $3.24

Explanation:
We need to find the unit price.
4.86/3 = 1.62
multiply the unit price by 2 to find the cost of 2 kg
1.62 × 2 = 3.24
Therefore the correct answer is option C.

Question 5.
One gallon of paint covers about 450 square feet. How many square feet will 1.5 gallons of paint cover?
Options:
a. 300ft2
b. 451.5ft2
c. 675ft2
d. 900ft2

Answer: 675ft2

Explanation:
Given,
One gallon of paint covers about 450 square feet.
We need to find how many square feet will 1.5 gallons of paint cover.
For that, we have to multiply the number of gallons by the number of square feet covered by each gallon.
1.5 × 450 = 675 sq. ft.
Thus the correct answer is option C.

Question 6.
The graph shows the relationship between the late fines the library charges and the number of days late.
Go Math Grade 7 Answer Key Chapter 4 Rates and Proportionality MIXED REVIEW img 21
Options:
a. y = 0.25x
b. y = 0.40x
c. y = 0.50x
d. y = 0.75x

Answer: y = 0.25x

Explanation:
The graph shows the relationship between the late fines the library charges and the number of days late
One of the points is (2, 0.5) so k = 0.5/2 = 0.25.
This gives an equation of y = 0.25x
Thus the correct answer is option A.

Mini-Task

Question 7.
School is 2 miles from home along a straight road. The table shows your distance from home as you walk home at a constant rate.
a. Is the relationship in the table proportional?
Go Math Grade 7 Answer Key Chapter 4 Rates and Proportionality MIXED REVIEW img 22
___________

Answer: no
Since 1.5/10 = 0.15 and 1/20 = 0.05, the relationship is not proportional since the ratios are not equal.

Question 7.
b. Find your distance from school for each time in the table.
Type below:
___________

Answer: Since the distance between school and home is 2 mi, the distance from school when the distance from home is 1.5 mi is 2 – 1.5 = 0.5 mi, for 1 mi its 2 – 1 = 1 mi, and for 0.5 mi its 2 – 0.5 = 1.5 mi.

Question 7.
c. Write an equation representing the relationship between the distance from school and time walking.
Type below:
___________

Answer: y = -0.05 x + 2

Explanation:
At time t = 0, you are 2 mi from home since the distance from home to school is 2 mi. This means the y-intercept, b is 2.
To find the slope of the line, find the rate of change:
m = (y2 – y1)/(x2 – x1) = (1 – 1.5)/(20 – 10) = -0.5/10 = -0.05
The line is then y = mx + b
y = 0.5x  + 2.

Module 4 – Page No. 161

EXERCISES

Question 1.
Steve uses \(\frac{8}{9}\) gallon of paint to paint 4 identical birdhouses. How many gallons of paint does he use for each birdhouse?
\(\frac{□}{□}\)

Answer: \(\frac{2}{9}\) gallons

Explanation:
Given that,
Steve uses \(\frac{8}{9}\) gallon of paint to paint 4 identical birdhouses.
Divide the number of gallons by the number of birdhouses.
\(\frac{8}{9}\) ÷ 4 = \(\frac{8}{9}\) × \(\frac{1}{4}\)
= \(\frac{2}{9}\) gallons
Thus Steve uses \(\frac{2}{9}\) gallons of paint for each birdhouse.

Question 2.
Ron walks 0.5 mile on the track in 10 minutes. Stevie walks 0.25 mile on the track in 6 minutes. Find the unit rate for each walker in miles per hour. Who is the faster walker?
Ron: ___________ miles per hour
Stevie: ___________ miles per hour
___________ is the faster walker.

Answer: Ron

Explanation:
Given,
Ron walks 0.5 miles on the track in 10 minutes.
Stevie walks 0.25 miles on the track in 6 minutes.
We can find each unit rate by dividing the number of miles by the number of minutes then multiply by 60 to get the number of miles per hour.
Ron: 0.5 ÷ 10 = 0.05 miles = 3 miles per hour
Stevie: 0.25/6 = 1/24 mi per min = 2.5 mi per hour

Question 3.
The table below shows the proportional relationship between Juan’s pay and the hours he works. Complete the table. Plot the data and connect the points with a line.
Go Math Grade 7 Answer Key Chapter 4 Rates and Proportionality img 23
Go Math Grade 7 Answer Key Chapter 4 Rates and Proportionality img 24
Type below:
____________

Answer:
First, find the constant of proportionality.
Let y represents pay and x represents the number of hours worked.
The constant of proportionality = y/x = 40/2 = 20
Hence, for 1 hour of work, he earns $20.
To find how many hours he needs to work $80, we divide 80 by the constant of proportionality.
80/20 = 4
For 1 hour he earns $20, so for 5 hours he earns 5 × 20 = $100.
For 1 hour he earns $20, so for 6 hours he earns 6 × 20 = $120.
Go-Math-Grade-7-Answer-Key-Chapter-4-Rates-and-Proportionality-img-23
Go-Math-Grade-7-Answer-Key-Chapter-4-Rates-and-Proportionality-img-24

Conclusion:

After the preparation of  Go Math Grade 7 Answer Key Chapter 4 Rates and Proportionality, we suggest the students solve the questions given in the Module Quiz. Test yourself by solving the questions given at the end of this chapter. By this, you can enhance your math skills and secure good marks in the exams. If you have any doubts regarding the solutions you can post your comment in the below comment section.